{ "21645374": { "QUESTION": "Do mitochondria play a role in remodelling lace plant leaves during programmed cell death?", "CONTEXTS": [ "Programmed cell death (PCD) is the regulated death of cells within an organism. The lace plant (Aponogeton madagascariensis) produces perforations in its leaves through PCD. The leaves of the plant consist of a latticework of longitudinal and transverse veins enclosing areoles. PCD occurs in the cells at the center of these areoles and progresses outwards, stopping approximately five cells from the vasculature. The role of mitochondria during PCD has been recognized in animals; however, it has been less studied during PCD in plants.", "The following paper elucidates the role of mitochondrial dynamics during developmentally regulated PCD in vivo in A. madagascariensis. A single areole within a window stage leaf (PCD is occurring) was divided into three areas based on the progression of PCD; cells that will not undergo PCD (NPCD), cells in early stages of PCD (EPCD), and cells in late stages of PCD (LPCD). Window stage leaves were stained with the mitochondrial dye MitoTracker Red CMXRos and examined. Mitochondrial dynamics were delineated into four categories (M1-M4) based on characteristics including distribution, motility, and membrane potential (\u0394\u03a8m). A TUNEL assay showed fragmented nDNA in a gradient over these mitochondrial stages. Chloroplasts and transvacuolar strands were also examined using live cell imaging. The possible importance of mitochondrial permeability transition pore (PTP) formation during PCD was indirectly examined via in vivo cyclosporine A (CsA) treatment. This treatment resulted in lace plant leaves with a significantly lower number of perforations compared to controls, and that displayed mitochondrial dynamics similar to that of non-PCD cells." ], "LABELS": [ "BACKGROUND", "RESULTS" ], "MESHES": [ "Alismataceae", "Apoptosis", "Cell Differentiation", "Mitochondria", "Plant Leaves" ], "YEAR": "2011", "reasoning_required_pred": "yes", "reasoning_free_pred": "yes", "final_decision": "yes", "LONG_ANSWER": "Results depicted mitochondrial dynamics in vivo as PCD progresses within the lace plant, and highlight the correlation of this organelle with other organelles during developmental PCD. To the best of our knowledge, this is the first report of mitochondria and chloroplasts moving on transvacuolar strands to form a ring structure surrounding the nucleus during developmental PCD. Also, for the first time, we have shown the feasibility for the use of CsA in a whole plant system. Overall, our findings implicate the mitochondria as playing a critical and early role in developmentally regulated PCD in the lace plant." }, "16418930": { "QUESTION": "Landolt C and snellen e acuity: differences in strabismus amblyopia?", "CONTEXTS": [ "Assessment of visual acuity depends on the optotypes used for measurement. The ability to recognize different optotypes differs even if their critical details appear under the same visual angle. Since optotypes are evaluated on individuals with good visual acuity and without eye disorders, differences in the lower visual acuity range cannot be excluded. In this study, visual acuity measured with the Snellen E was compared to the Landolt C acuity.", "100 patients (age 8 - 90 years, median 60.5 years) with various eye disorders, among them 39 with amblyopia due to strabismus, and 13 healthy volunteers were tested. Charts with the Snellen E and the Landolt C (Precision Vision) which mimic the ETDRS charts were used to assess visual acuity. Three out of 5 optotypes per line had to be correctly identified, while wrong answers were monitored. In the group of patients, the eyes with the lower visual acuity, and the right eyes of the healthy subjects, were evaluated.", "Differences between Landolt C acuity (LR) and Snellen E acuity (SE) were small. The mean decimal values for LR and SE were 0.25 and 0.29 in the entire group and 0.14 and 0.16 for the eyes with strabismus amblyopia. The mean difference between LR and SE was 0.55 lines in the entire group and 0.55 lines for the eyes with strabismus amblyopia, with higher values of SE in both groups. The results of the other groups were similar with only small differences between LR and SE." ], "LABELS": [ "BACKGROUND", "PATIENTS AND METHODS", "RESULTS" ], "MESHES": [ "Adolescent", "Adult", "Aged", "Aged, 80 and over", "Amblyopia", "Cataract", "Child", "Eye Diseases", "Female", "Humans", "Male", "Middle Aged", "Reference Values", "Refractive Errors", "Reproducibility of Results", "Retinal Diseases", "Strabismus", "Vision Tests", "Visual Acuity" ], "YEAR": "2006", "reasoning_required_pred": "no", "reasoning_free_pred": "no", "final_decision": "no", "LONG_ANSWER": "Using the charts described, there was only a slight overestimation of visual acuity by the Snellen E compared to the Landolt C, even in strabismus amblyopia. Small differences in the lower visual acuity range have to be considered." }, "9488747": { "QUESTION": "Syncope during bathing in infants, a pediatric form of water-induced urticaria?", "CONTEXTS": [ "Apparent life-threatening events in infants are a difficult and frequent problem in pediatric practice. The prognosis is uncertain because of risk of sudden infant death syndrome.", "Eight infants aged 2 to 15 months were admitted during a period of 6 years; they suffered from similar maladies in the bath: on immersion, they became pale, hypotonic, still and unreactive; recovery took a few seconds after withdrawal from the bath and stimulation. Two diagnoses were initially considered: seizure or gastroesophageal reflux but this was doubtful. The hypothesis of an equivalent of aquagenic urticaria was then considered; as for patients with this disease, each infant's family contained members suffering from dermographism, maladies or eruption after exposure to water or sun. All six infants had dermographism. We found an increase in blood histamine levels after a trial bath in the two infants tested. The evolution of these \"aquagenic maladies\" was favourable after a few weeks without baths. After a 2-7 year follow-up, three out of seven infants continue to suffer from troubles associated with sun or water." ], "LABELS": [ "BACKGROUND", "CASE REPORTS" ], "MESHES": [ "Baths", "Histamine", "Humans", "Infant", "Syncope", "Urticaria", "Water" ], "YEAR": "1997", "reasoning_required_pred": "yes", "reasoning_free_pred": "yes", "final_decision": "yes", "LONG_ANSWER": "\"Aquagenic maladies\" could be a pediatric form of the aquagenic urticaria." }, "17208539": { "QUESTION": "Are the long-term results of the transanal pull-through equal to those of the transabdominal pull-through?", "CONTEXTS": [ "The transanal endorectal pull-through (TERPT) is becoming the most popular procedure in the treatment of Hirschsprung disease (HD), but overstretching of the anal sphincters remains a critical issue that may impact the continence. This study examined the long-term outcome of TERPT versus conventional transabdominal (ABD) pull-through for HD.", "Records of 41 patients more than 3 years old who underwent a pull-through for HD (TERPT, n = 20; ABD, n = 21) were reviewed, and their families were thoroughly interviewed and scored via a 15-item post-pull-through long-term outcome questionnaire. Patients were operated on between the years 1995 and 2003. During this time, our group transitioned from the ABD to the TERPT technique. Total scoring ranged from 0 to 40: 0 to 10, excellent; 11 to 20 good; 21 to 30 fair; 31 to 40 poor. A 2-tailed Student t test, analysis of covariance, as well as logistic and linear regression were used to analyze the collected data with confidence interval higher than 95%.", "Overall scores were similar. However, continence score was significantly better in the ABD group, and the stool pattern score was better in the TERPT group. A significant difference in age at interview between the 2 groups was noted; we therefore reanalyzed the data controlling for age, and this showed that age did not significantly affect the long-term scoring outcome between groups." ], "LABELS": [ "PURPOSE", "METHODS", "RESULTS" ], "MESHES": [ "Child", "Child, Preschool", "Colectomy", "Female", "Hirschsprung Disease", "Humans", "Male", "Treatment Outcome" ], "YEAR": "2007", "reasoning_required_pred": "yes", "reasoning_free_pred": "no", "final_decision": "no", "LONG_ANSWER": "Our long-term study showed significantly better (2-fold) results regarding the continence score for the abdominal approach compared with the transanal pull-through. The stool pattern and enterocolitis scores were somewhat better for the TERPT group. These findings raise an important issue about the current surgical management of HD; however, more cases will need to be studied before a definitive conclusion can be drawn." }, "10808977": { "QUESTION": "Can tailored interventions increase mammography use among HMO women?", "CONTEXTS": [ "Telephone counseling and tailored print communications have emerged as promising methods for promoting mammography screening. However, there has been little research testing, within the same randomized field trial, of the efficacy of these two methods compared to a high-quality usual care system for enhancing screening. This study addressed the question: Compared to usual care, is tailored telephone counseling more effective than tailored print materials for promoting mammography screening?", "Three-year randomized field trial.", "One thousand ninety-nine women aged 50 and older recruited from a health maintenance organization in North Carolina.", "Women were randomized to 1 of 3 groups: (1) usual care, (2) tailored print communications, and (3) tailored telephone counseling.", "Adherence to mammography screening based on self-reports obtained during 1995, 1996, and 1997.", "Compared to usual care alone, telephone counseling promoted a significantly higher proportion of women having mammograms on schedule (71% vs 61%) than did tailored print (67% vs 61%) but only after the first year of intervention (during 1996). Furthermore, compared to usual care, telephone counseling was more effective than tailored print materials at promoting being on schedule with screening during 1996 and 1997 among women who were off-schedule during the previous year." ], "LABELS": [ "BACKGROUND", "DESIGN", "PARTICIPANTS", "INTERVENTION", "MAIN OUTCOME", "RESULTS" ], "MESHES": [ "Cost-Benefit Analysis", "Female", "Health Maintenance Organizations", "Humans", "Logistic Models", "Mammography", "Marketing of Health Services", "Middle Aged", "North Carolina", "Odds Ratio", "Pamphlets", "Patient Acceptance of Health Care", "Patient Satisfaction", "Reminder Systems", "Telephone" ], "YEAR": "2000", "reasoning_required_pred": "yes", "reasoning_free_pred": "no", "final_decision": "yes", "LONG_ANSWER": "The effects of the intervention were most pronounced after the first intervention. Compared to usual care, telephone counseling seemed particularly effective at promoting change among nonadherent women, the group for whom the intervention was developed. These results suggest that telephone counseling, rather than tailored print, might be the preferred first-line intervention for getting nonadherent women on schedule for mammography screening. Many questions would have to be answered about why the tailored print intervention was not more powerful. Nevertheless, it is clear that additional interventions will be needed to maintain women's adherence to mammography. Medical Subject Headings (MeSH): mammography screening, telephone counseling, tailored print communications, barriers." }, "23831910": { "QUESTION": "Double balloon enteroscopy: is it efficacious and safe in a community setting?", "CONTEXTS": [ "From March 2007 to January 2011, 88 DBE procedures were performed on 66 patients. Indications included evaluation anemia/gastrointestinal bleed, small bowel IBD and dilation of strictures. Video-capsule endoscopy (VCE) was used prior to DBE in 43 of the 66 patients prior to DBE evaluation.", "The mean age was 62 years. Thirty-two patients were female, 15 were African-American; 44 antegrade and 44 retrograde DBEs were performed. The mean time per antegrade DBE was 107.4\u00b130.0 minutes with a distance of 318.4\u00b1152.9 cm reached past the pylorus. The mean time per lower DBE was 100.7\u00b127.3 minutes with 168.9\u00b1109.1 cm meters past the ileocecal valve reached. Endoscopic therapy in the form of electrocautery to ablate bleeding sources was performed in 20 patients (30.3%), biopsy in 17 patients (25.8%) and dilation of Crohn's-related small bowel strictures in 4 (6.1%). 43 VCEs with pathology noted were performed prior to DBE, with findings endoscopically confirmed in 32 cases (74.4%). In 3 cases the DBE showed findings not noted on VCE." ], "LABELS": [ "METHODS", "RESULTS" ], "MESHES": [ "Community Health Centers", "Double-Balloon Enteroscopy", "Female", "Humans", "Intestinal Diseases", "Male", "Middle Aged" ], "YEAR": "2013", "reasoning_required_pred": "yes", "reasoning_free_pred": "yes", "final_decision": "yes", "LONG_ANSWER": "DBE appears to be equally safe and effective when performed in the community setting as compared to a tertiary referral center with a comparable yield, efficacy, and complication rate." }, "26037986": { "QUESTION": "30-Day and 1-year mortality in emergency general surgery laparotomies: an area of concern and need for improvement?", "CONTEXTS": [ "Emergency surgery is associated with poorer outcomes and higher mortality with recent studies suggesting the 30-day mortality to be 14-15%. The aim of this study was to analyse the 30-day mortality, age-related 30-day mortality and 1-year mortality following emergency laparotomy. We hope this will encourage prospective data collection, improvement of care and initiate strategies to establish best practice in this area.", "This was a retrospective study of patients who underwent emergency laparotomy from June 2010 to May 2012. The primary end point of the study was 30-day mortality, age-related 30-day mortality and 1-year all-cause mortality.", "477 laparotomies were performed in 446 patients. 57% were aged<70 and 43% aged>70 years. 30-day mortality was 12, 4% in those aged<70 years and 22% in those>70 years (p<0.001). 1-year mortality was 25, 15% in those aged under 70 years and 38% in those aged>70 years (p<0.001)." ], "LABELS": [ "AIMS", "METHODS", "RESULTS" ], "MESHES": [ "Adult", "Age Factors", "Aged", "Aged, 80 and over", "Cause of Death", "Cohort Studies", "Emergency Treatment", "Female", "General Surgery", "Humans", "Incidence", "Laparotomy", "Male", "Middle Aged", "Needs Assessment", "Retrospective Studies", "Risk Assessment", "Time Factors", "United Kingdom" ], "YEAR": "2015", "reasoning_required_pred": "maybe", "reasoning_free_pred": "yes", "final_decision": "maybe", "LONG_ANSWER": "Emergency laparotomy carries a high rate of mortality, especially in those over the age of 70 years, and more needs to be done to improve outcomes, particularly in this group. This could involve increasing acute surgical care manpower, early recognition of patients requiring emergency surgery, development of clear management protocols for such patients or perhaps even considering centralisation of emergency surgical services to specialist centres with multidisciplinary teams involving emergency surgeons and care of the elderly physicians in hospital and related community outreach services for post-discharge care." }, "26852225": { "QUESTION": "Is adjustment for reporting heterogeneity necessary in sleep disorders?", "CONTEXTS": [ "Anchoring vignettes are brief texts describing a hypothetical character who illustrates a certain fixed level of a trait under evaluation. This research uses vignettes to elucidate factors associated with sleep disorders in adult Japanese before and after adjustment for reporting heterogeneity in self-reports. This study also evaluates the need for adjusting for reporting heterogeneity in the management of sleep and energy related problems in Japan.", "We investigated a dataset of 1002 respondents aged 18 years and over from the Japanese World Health Survey, which collected information through face-to-face interview from 2002 to 2003. The ordered probit model and the Compound Hierarchical Ordered Probit (CHOPIT) model, which incorporated anchoring vignettes, were employed to estimate and compare associations of sleep and energy with socio-demographic and life-style factors before and after adjustment for differences in response category cut-points for each individual.", "The prevalence of self-reported problems with sleep and energy was 53 %. Without correction of cut-point shifts, age, sex, and the number of comorbidities were significantly associated with a greater severity of sleep-related problems. After correction, age, the number of comorbidities, and regular exercise were significantly associated with a greater severity of sleep-related problems; sex was no longer a significant factor. Compared to the ordered probit model, the CHOPIT model provided two changes with a subtle difference in the magnitude of regression coefficients after correction for reporting heterogeneity." ], "LABELS": [ "BACKGROUND", "METHODS", "RESULTS" ], "MESHES": [ "Adult", "Aged", "Female", "Health Status Disparities", "Health Surveys", "Humans", "Japan", "Male", "Middle Aged", "Physical Fitness", "Prevalence", "Self Report", "Self-Assessment", "Sleep Wake Disorders", "Socioeconomic Factors" ], "YEAR": "2016", "reasoning_required_pred": "yes", "reasoning_free_pred": "no", "final_decision": "no", "LONG_ANSWER": "Sleep disorders are common in the general adult population of Japan. Correction for reporting heterogeneity using anchoring vignettes is not a necessary tool for proper management of sleep and energy related problems among Japanese adults. Older age, gender differences in communicating sleep-related problems, the presence of multiple morbidities, and regular exercise should be the focus of policies and clinical practice to improve sleep and energy management in Japan." }, "17113061": { "QUESTION": "Do mutations causing low HDL-C promote increased carotid intima-media thickness?", "CONTEXTS": [ "Although observational data support an inverse relationship between high-density lipoprotein (HDL) cholesterol and coronary heart disease (CHD), genetic HDL deficiency states often do not correlate with premature CHD.", "Carotid intima-media thickness (cIMT) measurements were obtained in cases comprising 10 different mutations in LCAT, ABCA1 and APOA1 to further evaluate the relationship between low HDL resulting from genetic variation and early atherosclerosis.", "In a 1:2 case-control study of sex and age-related (+/-5 y) subjects (n=114), cIMT was nearly identical between cases (0.66+/-0.17 cm) and controls (0.65+/-0.18 cm) despite significantly lower HDL cholesterol (0.67 vs. 1.58 mmol/l) and apolipoprotein A-I levels (96.7 vs. 151.4 mg/dl) (P<0.05)" ], "LABELS": [ "BACKGROUND", "METHODS", "RESULTS" ], "MESHES": [ "Cholesterol, HDL", "Contrast Media", "Coronary Disease", "Female", "Humans", "Male", "Mutation", "Risk Factors" ], "YEAR": "2007", "reasoning_required_pred": "no", "reasoning_free_pred": "no", "final_decision": "no", "LONG_ANSWER": "Genetic variants identified in the present study may be insufficient to promote early carotid atherosclerosis." }, "10966337": { "QUESTION": "A short stay or 23-hour ward in a general and academic children's hospital: are they effective?", "CONTEXTS": [ "We evaluated the usefulness of a short stay or 23-hour ward in a pediatric unit of a large teaching hospital, Westmead Hospital, and an academic Children's hospital, The New Children's Hospital, to determine if they are a useful addition to the emergency service.", "This is a descriptive comparison of prospectively collected data on all children admitted to the short stay ward at Westmead Hospital (WH) during 1994 and the short stay ward at the New Children's Hospital (NCH) during 1997-98. These hospitals service an identical demographic area with the latter (NCH) a tertiary referral center. The following outcome measures were used: length of stay, appropriateness of stay, rate of admission to an in-hospital bed, and rate of unscheduled visits within 72 hours of discharge. Adverse events were reported and patient follow-up was attempted at 48 hours after discharge in all cases.", "The short stay ward accounted for 10.3% (Westmead Hospital) and 14.7% (New Children's Hospital) of admissions, with 56% medical in nature, 30% surgical, and the remainder procedural or psychological. Admission patterns were similar, with asthma, gastroenteritis, convulsion, pneumonia, and simple surgical conditions accounting for most short stay ward admissions. The short stay ward increased hospital efficiency with an average length of stay of 17.5 hours (Westmead Hospital) compared to 20.5 hours (New Children's Hospital). The users of the short stay ward were children of young age less than 2 years, with stay greater than 23 hours reported in only 1% of all admissions to the short stay ward. The rate of patient admission to an in-hospital bed was low, (4% [Westmead Hospital] compared to 6% [New Children's Hospital]), with the number of unscheduled visits within 72 hours of short stay ward discharge less than 1%. There were no adverse events reported at either short stay ward, with parental satisfaction high. The short stay ward was developed through reallocation of resources from within the hospital to the short stay ward. This resulted in estimated savings of $1/2 million (Westmead Hospital) to $2.3 million (New Children's Hospital) to the hospital, due to more efficient bed usage." ], "LABELS": [ "OBJECTIVE", "METHODS", "RESULTS" ], "MESHES": [ "Academic Medical Centers", "Acute Disease", "Adolescent", "Child", "Child, Preschool", "Critical Pathways", "Emergency Service, Hospital", "Follow-Up Studies", "Hospital Units", "Hospitals, General", "Hospitals, Pediatric", "Humans", "Infant", "Length of Stay", "New South Wales", "Outcome Assessment (Health Care)", "Pediatrics", "Prospective Studies", "Time Factors" ], "YEAR": "2000", "reasoning_required_pred": "yes", "reasoning_free_pred": "yes", "final_decision": "yes", "LONG_ANSWER": "This data demonstrates the robust nature of the short stay ward. At these two very different institutions we have shown improved bed efficient and patient care in a cost-effective way. We have also reported on greater parental satisfaction and early return of the child with their family to the community." }, "25432938": { "QUESTION": "Did Chile's traffic law reform push police enforcement?", "CONTEXTS": [ "The objective of the current study is to determine to what extent the reduction of Chile's traffic fatalities and injuries during 2000-2012 was related to the police traffic enforcement increment registered after the introduction of its 2005 traffic law reform.", "A unique dataset with assembled information from public institutions and analyses based on ordinary least square and robust random effects models was carried out. Dependent variables were traffic fatality and severe injury rates per population and vehicle fleet. Independent variables were: (1) presence of new national traffic law; (2) police officers per population; (3) number of traffic tickets per police officer; and (4) interaction effect of number of traffic tickets per police officer with traffic law reform. Oil prices, alcohol consumption, proportion of male population 15-24 years old, unemployment, road infrastructure investment, years' effects and regions' effects represented control variables.", "Empirical estimates from instrumental variables suggest that the enactment of the traffic law reform in interaction with number of traffic tickets per police officer is significantly associated with a decrease of 8% in traffic fatalities and 7% in severe injuries. Piecewise regression model results for the 2007-2012 period suggest that police traffic enforcement reduced traffic fatalities by 59% and severe injuries by 37%." ], "LABELS": [ "BACKGROUND", "METHODS", "RESULTS" ], "MESHES": [ "Accidents, Traffic", "Automobile Driving", "Chile", "Humans", "Law Enforcement", "Models, Statistical", "Police", "Risk Factors" ], "YEAR": "2015", "reasoning_required_pred": "yes", "reasoning_free_pred": "yes", "final_decision": "yes", "LONG_ANSWER": "Findings suggest that traffic law reforms in order to have an effect on both traffic fatality and injury rates reduction require changes in police enforcement practices. Last, this case also illustrates how the diffusion of successful road safety practices globally promoted by WHO and World Bank can be an important influence for enhancing national road safety practices." }, "18847643": { "QUESTION": "Therapeutic anticoagulation in the trauma patient: is it safe?", "CONTEXTS": [ "Trauma patients who require therapeutic anticoagulation pose a difficult treatment problem. The purpose of this study was to determine: (1) the incidence of complications using therapeutic anticoagulation in trauma patients, and (2) if any patient factors are associated with these complications.", "An 18-month retrospective review was performed on trauma patients>or= 15 years old who received therapeutic anticoagulation using unfractionated heparin (UH) and/or fractionated heparin (FH). Forty different pre-treatment and treatment patient characteristics were recorded. Complications of anticoagulation were documented and defined as any unanticipated discontinuation of the anticoagulant for bleeding or other adverse events.", "One-hundred-fourteen trauma patients were initiated on therapeutic anticoagulation. The most common indication for anticoagulation was deep venous thrombosis (46%). Twenty-four patients (21%) had at least 1 anticoagulation complication. The most common complication was a sudden drop in hemoglobin concentration requiring blood transfusion (11 patients). Five patients died (4%), 3 of whom had significant hemorrhage attributed to anticoagulation. Bivariate followed by logistic regression analysis identified chronic obstructive pulmonary disease (OR = 9.2, 95%CI = 1.5-54.7), UH use (OR = 3.8, 95%CI = 1.1-13.0), and lower initial platelet count (OR = 1.004, 95%CI = 1.000-1.008) as being associated with complications. Patients receiving UH vs. FH differed in several characteristics including laboratory values and anticoagulation indications." ], "LABELS": [ "PURPOSE", "METHODS", "RESULTS" ], "MESHES": [ "Adult", "Anticoagulants", "Cohort Studies", "Confidence Intervals", "Emergency Treatment", "Female", "Follow-Up Studies", "Heparin, Low-Molecular-Weight", "Humans", "Injury Severity Score", "Male", "Middle Aged", "Odds Ratio", "Postoperative Care", "Preoperative Care", "Probability", "Retrospective Studies", "Risk Assessment", "Safety Management", "Survival Analysis", "Thromboembolism", "Thrombolytic Therapy", "Trauma Centers", "Treatment Outcome", "Warfarin", "Wounds and Injuries" ], "YEAR": "2008", "reasoning_required_pred": "no", "reasoning_free_pred": "no", "final_decision": "no", "LONG_ANSWER": "Trauma patients have a significant complication rate related to anticoagulation therapy, and predicting which patients will develop a complication remains unclear. Prospective studies are needed to determine which treatment regimen, if any, is appropriate to safely anticoagulate this high risk population." }, "18239988": { "QUESTION": "Differentiation of nonalcoholic from alcoholic steatohepatitis: are routine laboratory markers useful?", "CONTEXTS": [ "Specific markers for differentiation of nonalcoholic (NASH) from alcoholic steatohepatitis (ASH) are lacking. We investigated the role of routine laboratory parameters in distinguishing NASH from ASH.", "Liver biopsies performed at our hospital over a 10-year period were reviewed, 95 patients with steatohepatitis identified and their data prior to biopsy reevaluated. The diagnosis NASH or ASH was assigned (other liver diseases excluded) on the basis of the biopsy and history of alcohol consumption (<140 g/week). Logistic regression models were used for analysis.", "NASH was diagnosed in 58 patients (61%; 30 f) and ASH in 37 (39%; 9 f). High-grade fibrosis (59% vs. 19%, P<0.0001) and an AST/ALT ratio>1 (54.1% vs 20.7%, P = 0.0008) were more common in ASH. The MCV was elevated in 53% of ASH patients and normal in all NASH patients (P<0.0001). Multivariate analysis identified the MCV (P = 0.0013), the AST/ALT ratio (P = 0.011) and sex (P = 0.0029) as relevant regressors (aROC = 0.92). The AST/ALT ratio (P<0.0001) and age (P = 0.00049) were independent predictors of high-grade fibrosis. Differences in MCV were more marked in high-grade fibrosis." ], "LABELS": [ "AIMS", "METHODS", "RESULTS" ], "MESHES": [ "Adolescent", "Adult", "Aged", "Alanine Transaminase", "Aspartate Aminotransferases", "Biomarkers", "Biopsy", "Diagnosis, Differential", "Erythrocyte Indices", "Fatty Liver", "Fatty Liver, Alcoholic", "Female", "Humans", "Liver", "Liver Cirrhosis", "Liver Cirrhosis, Alcoholic", "Liver Function Tests", "Male", "Middle Aged", "Predictive Value of Tests", "Retrospective Studies" ], "YEAR": "2008", "reasoning_required_pred": "yes", "reasoning_free_pred": "no", "final_decision": "yes", "LONG_ANSWER": "Higher MCVs and AST/ALT ratios in ASH reflect the severity of underlying liver disease and do not differentiate NASH from ASH. Instead, these biomarkers might prove useful in guiding selection of patients for liver biopsy and in targeting therapy." }, "25957366": { "QUESTION": "Prompting Primary Care Providers about Increased Patient Risk As a Result of Family History: Does It Work?", "CONTEXTS": [ "Electronic health records have the potential to facilitate family history use by primary care physicians (PCPs) to provide personalized care. The objective of this study was to determine whether automated, at-the-visit tailored prompts about family history risk change PCP behavior.", "Automated, tailored prompts highlighting familial risk for heart disease, stroke, diabetes, and breast, colorectal, or ovarian cancer were implemented during 2011 to 2012. Medical records of a cohort of community-based primary care patients, aged 35 to 65 years, who previously participated in our Family Healthware study and had a moderate or strong familial risk for any of the 6 diseases were subsequently reviewed. The main outcome measures were PCP response to the prompts, adding family history risk to problem summary lists, and patient screening status for each disease.", "The 492 eligible patients had 847 visits during the study period; 152 visits had no documentation of response to a family history prompt. Of the remaining 695 visits, physician responses were reviewed family history (n = 372, 53.5%), discussed family history (n = 159, 22.9%), not addressed (n = 155, 22.3%), and reviewed family history and ordered tests/referrals (n = 5, 0.7%). There was no significant change in problem summary list documentation of risk status or screening interventions for any of the 6 diseases." ], "LABELS": [ "BACKGROUND", "METHODS", "RESULTS" ], "MESHES": [ "Adult", "Aged", "Electronic Health Records", "Female", "Genetic Predisposition to Disease", "Heredity", "Humans", "Male", "Medical History Taking", "Middle Aged", "Practice Patterns, Physicians'", "Primary Health Care", "Prospective Studies", "Reminder Systems", "Risk Assessment", "Risk Factors" ], "YEAR": null, "reasoning_required_pred": "no", "reasoning_free_pred": "no", "final_decision": "no", "LONG_ANSWER": "No change occurred upon instituting simple, at-the-visit family history prompts geared to improve PCPs' ability to identify patients at high risk for 6 common conditions. The results are both surprising and disappointing. Further studies should examine physicians' perception of the utility of prompts for family history risk." }, "24866606": { "QUESTION": "Do emergency ultrasound fellowship programs impact emergency medicine residents' ultrasound education?", "CONTEXTS": [ "Recent years have seen a rapid proliferation of emergency ultrasound (EUS) programs in the United States. To date, there is no evidence supporting that EUS fellowships enhance residents' ultrasound (US) educational experiences. The purpose of this study was to determine the impact of EUS fellowships on emergency medicine (EM) residents' US education.", "We conducted a cross-sectional study at 9 academic medical centers. A questionnaire on US education and bedside US use was pilot tested and given to EM residents. The primary outcomes included the number of US examinations performed, scope of bedside US applications, barriers to residents' US education, and US use in the emergency department. The secondary outcomes were factors that would impact residents' US education. The outcomes were compared between residency programs with and without EUS fellowships.", "A total of 244 EM residents participated in this study. Thirty percent (95% confidence interval, 24%-35%) reported they had performed more than 150 scans. Residents in programs with EUS fellowships reported performing more scans than those in programs without fellowships (P = .04). Significant differences were noted in most applications of bedside US between residency programs with and without fellowships (P<.05). There were also significant differences in the barriers to US education between residency programs with and without fellowships (P<.05)." ], "LABELS": [ "OBJECTIVES", "METHODS", "RESULTS" ], "MESHES": [ "Attitude of Health Personnel", "Clinical Competence", "Educational Measurement", "Emergency Medicine", "Fellowships and Scholarships", "Internship and Residency", "Radiology", "Ultrasonography", "United States" ], "YEAR": "2014", "reasoning_required_pred": "yes", "reasoning_free_pred": "yes", "final_decision": "yes", "LONG_ANSWER": "Emergency US fellowship programs had a positive impact on residents' US educational experiences. Emergency medicine residents performed more scans overall and also used bedside US for more advanced applications in programs with EUS fellowships." }, "26578404": { "QUESTION": "Patient-Controlled Therapy of Breathlessness in Palliative Care: A New Therapeutic Concept for Opioid Administration?", "CONTEXTS": [ "Breathlessness is one of the most distressing symptoms experienced by patients with advanced cancer and noncancer diagnoses alike. Often, severity of breathlessness increases quickly, calling for rapid symptom control. Oral, buccal, and parenteral routes of provider-controlled drug administration have been described. It is unclear whether patient-controlled therapy (PCT) systems would be an additional treatment option.", "To investigate whether intravenous opioid PCT can be an effective therapeutic method to reduce breathlessness in patients with advanced disease. Secondary aims were to study the feasibility and acceptance of opioid PCT in patients with refractory breathlessness.", "This was a pilot observational study with 18 inpatients with advanced disease and refractory breathlessness receiving opioid PCT. Breathlessness was measured on a self-reported numeric rating scale. Richmond Agitation Sedation Scale scores, Palliative Performance Scale scores, vital signs, and a self-developed patient satisfaction questionnaire were used for measuring secondary outcomes. Descriptive and interference analyses (Friedman test) and post hoc analyses (Wilcoxon tests and Bonferroni corrections) were performed.", "Eighteen of 815 patients (advanced cancer; median age\u00a0=\u00a057.5\u00a0years [range 36-81]; 77.8% female) received breathlessness symptom control with opioid PCT; daily morphine equivalent dose at Day 1 was median\u00a0=\u00a020.3\u00a0mg (5.0-49.6\u00a0mg); Day 2: 13.0\u00a0mg (1.0-78.5\u00a0mg); Day 3: 16.0\u00a0mg (8.3-47.0\u00a0mg). Numeric rating scale of current breathlessness decreased (baseline: median\u00a0=\u00a05 [range 1-10]; Day 1: median\u00a0=\u00a04 [range 0-8], P\u00a0<\u00a00.01; Day 2: median\u00a0=\u00a04 [range 0-5], P\u00a0<\u00a00.01). Physiological parameters were stable over time. On Day 3, 12/12 patients confirmed that this mode of application provided relief of breathlessness." ], "LABELS": [ "CONTEXT", "OBJECTIVES", "METHODS", "RESULTS" ], "MESHES": [ "Administration, Intravenous", "Adult", "Aged", "Aged, 80 and over", "Analgesia, Patient-Controlled", "Analgesics, Opioid", "Dyspnea", "Feasibility Studies", "Female", "Humans", "Longitudinal Studies", "Male", "Middle Aged", "Neoplasms", "Palliative Care", "Patient Satisfaction", "Prospective Studies", "Self Report", "Severity of Illness Index" ], "YEAR": "2016", "reasoning_required_pred": "yes", "reasoning_free_pred": "yes", "final_decision": "yes", "LONG_ANSWER": "Opioid PCT is a feasible and acceptable therapeutic method to reduce refractory breathlessness in palliative care patients." }, "11729377": { "QUESTION": "Is there still a need for living-related liver transplantation in children?", "CONTEXTS": [ "To assess and compare the value of split-liver transplantation (SLT) and living-related liver transplantation (LRT).", "The concept of SLT results from the development of reduced-size transplantation. A further development of SLT, the in situ split technique, is derived from LRT, which itself marks the optimized outcome in terms of postoperative graft function and survival. The combination of SLT and LRT has abolished deaths on the waiting list, thus raising the question whether living donor liver transplantation is still necessary.", "Outcomes and postoperative liver function of 43 primary LRT patients were compared with those of 49 primary SLT patients (14 ex situ, 35 in situ) with known graft weight performed between April 1996 and December 2000. Survival rates were analyzed using the Kaplan-Meier method.", "After a median follow-up of 35 months, actual patient survival rates were 82% in the SLT group and 88% in the LRT group. Actual graft survival rates were 76% and 81%, respectively. The incidence of primary nonfunction was 12% in the SLT group and 2.3% in the LRT group. Liver function parameters (prothrombin time, factor V, bilirubin clearance) and surgical complication rates did not differ significantly. In the SLT group, mean cold ischemic time was longer than in the LRT group. Serum values of alanine aminotransferase during the first postoperative week were significantly higher in the SLT group. In the LRT group, there were more grafts with signs of fatty degeneration than in the SLT group." ], "LABELS": [ "OBJECTIVE", "SUMMARY BACKGROUND DATA", "METHODS", "RESULTS" ], "MESHES": [ "Adolescent", "Child", "Child, Preschool", "Fatty Liver", "Female", "Graft Survival", "Humans", "Immunosuppressive Agents", "Infant", "Liver", "Liver Transplantation", "Living Donors", "Male", "Postoperative Complications", "Reperfusion Injury", "Survival Rate" ], "YEAR": "2001", "reasoning_required_pred": "yes", "reasoning_free_pred": "yes", "final_decision": "yes", "LONG_ANSWER": "The short- and long-term outcomes after LRT and SLT did not differ significantly. To avoid the risk for the donor in LRT, SLT represents the first-line therapy in pediatric liver transplantation in countries where cadaveric organs are available. LRT provides a solution for urgent cases in which a cadaveric graft cannot be found in time or if the choice of the optimal time point for transplantation is vital." }, "17096624": { "QUESTION": "Do patterns of knowledge and attitudes exist among unvaccinated seniors?", "CONTEXTS": [ "To examine patterns of knowledge and attitudes among adults aged>65 years unvaccinated for influenza.", "Surveyed Medicare beneficiaries in 5 areas; clustered unvaccinated seniors by their immunization related knowledge and attitudes.", "Identified 4 clusters: Potentials (45%) would receive influenza vaccine to prevent disease; Fearful Uninformeds (9%) were unsure if influenza vaccine causes illness; Doubters (27%) were unsure if vaccine is efficacious; Misinformeds (19%) believed influenza vaccine causes illness. More Potentials (75%) and Misinformeds (70%) ever received influenza vaccine than did Fearful Uninformeds (18%) and Doubters (29%)." ], "LABELS": [ "OBJECTIVE", "METHODS", "RESULTS" ], "MESHES": [ "Aged", "Aged, 80 and over", "Female", "Health Knowledge, Attitudes, Practice", "Humans", "Immunization Programs", "Influenza A virus", "Influenza, Human", "Interviews as Topic", "Male", "United States" ], "YEAR": null, "reasoning_required_pred": "yes", "reasoning_free_pred": "yes", "final_decision": "yes", "LONG_ANSWER": "Findings suggest that cluster analyses may be useful in identifying groups for targeted health messages." }, "22694248": { "QUESTION": "Is there a model to teach and practice retroperitoneoscopic nephrectomy?", "CONTEXTS": [ "Although the retroperitoneal approach has been the preferred choice for open urological procedures, retroperitoneoscopy is not the preferred approach for laparoscopy. This study aims to develop a training model for retroperitoneoscopy and to establish an experimental learning curve.", "Fifteen piglets were operated on to develop a standard retroperitoneoscopic nephrectomy (RPN) training model. All procedures were performed with three ports. Intraoperative data (side, operative time, blood loss, peritoneal opening) were recorded. Animals were divided into groups A, the first eight, and B, the last seven cases. Data were statistically analyzed.", "We performed fifteen RPNs. The operative time varied from 15 to 50 minutes (median 30 minutes). Blood loss varied from 5 to 100 mL (median 20 mL). We experienced five peritoneal openings; we had two surgical vascular complications managed laparoscopically. There was statistical difference between groups A and B for peritoneal opening (p = 0.025), operative time (p = 0.0037), and blood loss (p = 0.026).", "RPN in a porcine model could simulate the whole procedure, from creating the space to nephrectomy completion. Experimental learning curve was eight cases, after statistical data analysis." ], "LABELS": [ "INTRODUCTION", "MATERIAL AND METHODS", "RESULTS", "DISCUSSION" ], "MESHES": [ "Animals", "Blood Loss, Surgical", "Feasibility Studies", "Laparoscopy", "Models, Animal", "Nephrectomy", "Operative Time", "Retroperitoneal Space", "Swine" ], "YEAR": "2013", "reasoning_required_pred": "yes", "reasoning_free_pred": "yes", "final_decision": "yes", "LONG_ANSWER": "RPN in a porcine model is feasible and could be very useful for teaching and practicing retroperitoneoscopy." }, "22990761": { "QUESTION": "Cardiovascular risk in a rural adult West African population: is resting heart rate also relevant?", "CONTEXTS": [ "Elevated resting heart rate (RHR) is a neglected marker in cardiovascular risk factor studies of sub-Saharan African populations. This study aimed to determine the prevalence of elevated RHR and other risk factors for cardiovascular disease (CVD) and to investigate any associations between RHR and these risk factors in a rural population in Ghana.", "Cross-sectional analysis.", "A total of 574 adults aged between 18-65 years were randomly sampled from a population register. Data collected included those on sociodemographic variables and anthropometric, blood pressure (BP), and RHR measurements. Within-person variability in RHR was calculated using data from repeat measurements taken 2 weeks apart.", "Of study participants, 36% were male. Prevalence of casual high BP was 19%. In the population, 10% were current cigarette smokers and habitual alcohol use was high at 56%. As measured by body mass index, 2% were obese and 14% had abdominal obesity. RHR was elevated (>90 bpm) in 19%. Overall, 79% of study participants were found to have at least one CVD risk factor. RHR was significantly associated with age, waist circumference, and BP. Individuals with an elevated RHR had a higher risk (OR 1.94, 95% CI 1.15-3.26%, p = 0.013) of casual high BP compared with participants with normal RHR independently of several established CVD risk factors. The regression dilution ratio of RHR was 0.75 (95% CI 0.62-0.89)." ], "LABELS": [ "INTRODUCTION", "DESIGN", "METHODS", "RESULTS" ], "MESHES": [ "Adolescent", "Adult", "Aged", "Asian Continental Ancestry Group", "Cardiovascular Diseases", "Comorbidity", "Cross-Sectional Studies", "Female", "Ghana", "Health Surveys", "Heart Rate", "Humans", "Male", "Middle Aged", "Prevalence", "Risk Assessment", "Risk Factors", "Rural Health", "Young Adult" ], "YEAR": "2014", "reasoning_required_pred": "yes", "reasoning_free_pred": "yes", "final_decision": "yes", "LONG_ANSWER": "Significant associations were observed between RHR and several established cardiovascular risk factors. Prospective studies are needed in sub-Saharan African populations to establish the potential value of RHR in cardiovascular risk assessment." }, "19394934": { "QUESTION": "Israeli hospital preparedness for terrorism-related multiple casualty incidents: can the surge capacity and injury severity distribution be better predicted?", "CONTEXTS": [ "The incidence of large-scale urban attacks on civilian populations has significantly increased across the globe over the past decade. These incidents often result in Hospital Multiple Casualty Incidents (HMCI), which are very challenging to hospital teams. 15 years ago the Emergency and Disaster Medicine Division in the Israeli Ministry of Health defined a key of 20 percent of each hospital's bed capacity as its readiness for multiple casualties. Half of those casualties are expected to require immediate medical treatment. This study was performed to evaluate the efficacy of the current readiness guidelines based on the epidemiology of encountered HMCIs.", "A retrospective study of HMCIs was recorded in the Israeli Defense Force (IDF) home front command and the Israeli National Trauma Registry (ITR) between November 2000 and June 2003. An HMCI is defined by the Emergency and Disaster Medicine Division in the Israeli Ministry of Health as>or=10 casualties or>or=4 suffering from injuries with an ISS>or=16 arriving to a single hospital.", "The study includes a total of 32 attacks, resulting in 62 HMCIs and 1292 casualties. The mean number of arriving casualties to a single hospital was 20.8+/-13.3 (range 4-56, median 16.5). In 95% of the HMCIs the casualty load was12 hr) in a Houston, Texas, NICU. Socioeconomic status, race, and mental health status were hypothesized to be related to SHSe and household smoking bans.", "Data were collected as part of The Baby's Breath Project, a hospital-based SHSe intervention trial targeting parents with a high-risk infant in the NICU who reported a smoker in the household (N = 99). Measures of sociodemographics, smoking, home and car smoking bans, and depression were collected.", "Overall, 26% of all families with a high-risk infant in the NICU reported a household smoker. Almost half of the families with a smoker reported an annual income of less than $25,000. 46.2% of families reported having a total smoking ban in place in both their homes and cars. Only 27.8% families earning less than $25,000 reported having a total smoking ban in place relative to almost 60% of families earning more (p<.01). African American and Caucasian families were less likely to have a smoking ban compared with Hispanics (p<.05). Mothers who reported no smoking ban were more depressed than those who had a household smoking ban (p<.02)." ], "LABELS": [ "INTRODUCTION", "METHODS", "RESULTS" ], "MESHES": [ "African Americans", "Air Pollution, Indoor", "Asian Continental Ancestry Group", "Caregivers", "Depression", "Environmental Exposure", "European Continental Ancestry Group", "Family Characteristics", "Female", "Follow-Up Studies", "Healthcare Disparities", "Hispanic Americans", "Humans", "Infant", "Infant, Newborn", "Intensive Care Units, Neonatal", "Male", "Prevalence", "Risk", "Risk Reduction Behavior", "Smoking", "Socioeconomic Factors", "Texas", "Tobacco Smoke Pollution" ], "YEAR": "2011", "reasoning_required_pred": "yes", "reasoning_free_pred": "yes", "final_decision": "yes", "LONG_ANSWER": "The most disadvantaged families were least likely to have protective health behaviors in place to reduce SHSe and, consequently, are most at-risk for tobacco exposure and subsequent tobacco-related health disparities. Innovative SHSe interventions for this vulnerable population are sorely needed." }, "23806388": { "QUESTION": "Do nomograms designed to predict biochemical recurrence (BCR) do a better job of predicting more clinically relevant prostate cancer outcomes than BCR?", "CONTEXTS": [ "To examine the ability of various postoperative nomograms to predict prostate cancer-specific mortality (PCSM) and to validate that they could predict aggressive biochemical recurrence (BCR). Prostate-specific antigen (PSA), grade, and stage are the classic triad used to predict BCR after radical prostatectomy (RP). Multiple nomograms use these to predict risk of BCR. A previous study showed that several nomograms could predict aggressive BCR (prostate-specific antigen doubling time [PSADT]\u00a0<9 months) more accurately than BCR. However, it remains unknown if they can predict more definitive endpoints, such as PCSM.", "We performed Cox analyses to examine the ability of 4 postoperative nomograms, the Duke Prostate Center (DPC) nomogram, the Kattan postoperative nomogram, the Johns Hopkins Hospital (JHH) nomogram, and the joint Center for Prostate Disease Research(CPDR)/Cancer of the Prostate Strategic Urologic Research Endeavor (CaPSURE) nomogram to predict BCR and PCSM among 1778 men in the Shared Equal Access Regional Cancer Hospital (SEARCH) database who underwent RP between 1990 and 2009. We also compared their ability to predict BCR and aggressive BCR in a subset of men. We calculated the c-index for each nomogram to determine its predictive accuracy for estimating actual outcomes.", "We found that each nomogram could predict aggressive BCR and PCSM in a statistically significant manner and that they all predicted PCSM more accurately than they predicted BCR (ie, with higher c-index values)." ], "LABELS": [ "OBJECTIVE", "METHODS", "RESULTS" ], "MESHES": [ "Aged", "Humans", "Male", "Middle Aged", "Neoplasm Grading", "Neoplasm Recurrence, Local", "Neoplasm Staging", "Nomograms", "Predictive Value of Tests", "Proportional Hazards Models", "Prostate-Specific Antigen", "Prostatectomy", "Prostatic Neoplasms", "Time Factors" ], "YEAR": "2013", "reasoning_required_pred": "yes", "reasoning_free_pred": "yes", "final_decision": "yes", "LONG_ANSWER": "Currently available nomograms used to predict BCR accurately predict PCSM and other more clinically relevant endpoints. Moreover, not only do they significantly predict PCSM, but do so with generally greater accuracy than BCR." }, "17919952": { "QUESTION": "Are reports of mechanical dysfunction in chronic oro-facial pain related to somatisation?", "CONTEXTS": [ "(i) To examine the association between self-reported mechanical factors and chronic oro-facial pain. (ii) To test the hypothesis that this relationship could be explained by: (a) reporting of psychological factors, (b) common association of self-reported mechanical factors with other unexplained syndromes.", "A population based cross-sectional study of 4200 randomly selected adults registered with a General Medical Practice in North West, England. The study examined the association of chronic oro-facial pain with a variety of self-reported mechanical factors: teeth grinding, facial trauma, missing teeth and the feeling that the teeth did not fit together properly. Information was also collected on demographic factors, psychological factors and the reporting of other frequently unexplained syndromes.", "An adjusted response rate of 72% was achieved. Only two mechanical factors: teeth grinding (odds ratio (OR) 2.0, 95% CI 1.3-3.0) and facial trauma (OR 2.0; 95% CI 1.3-2.9) were independently associated with chronic oro-facial pain after adjusting for psychological factors. However, these factors were also commonly associated with the reporting of other frequently unexplained syndromes: teeth grinding (odds ratio (OR) 1.8, 95% CI 1.5-2.2), facial trauma (OR 2.1; 95% CI 1.7-2.6)." ], "LABELS": [ "OBJECTIVES", "METHODS", "RESULTS" ], "MESHES": [ "Adolescent", "Adult", "Aged", "Bruxism", "Chronic Disease", "Cross-Sectional Studies", "Facial Injuries", "Facial Pain", "Female", "Humans", "Male", "Middle Aged", "Mouth, Edentulous", "Multivariate Analysis", "Prevalence", "Stress, Mechanical", "Surveys and Questionnaires" ], "YEAR": "2008", "reasoning_required_pred": "yes", "reasoning_free_pred": "no", "final_decision": "yes", "LONG_ANSWER": "Self-reported mechanical factors associated with chronic oro-facial pain are confounded, in part, by psychological factors and are equally common across other frequently unexplained syndromes. They may represent another feature of somatisation. Therefore the use of extensive invasive therapy such as occlusal adjustments and surgery to change mechanical factors may not be justified in many cases." }, "10966943": { "QUESTION": "Amblyopia: is visual loss permanent?", "CONTEXTS": [ "The records of 465 patients with an established diagnosis of age related macular degeneration who had attended a specialist macular clinic between 1990 and 1998 were scrutinised. A full clinical examination and standardised refraction had been carried out in 189 of these cases on a minimum of two occasions. Cases were looked for where an improvement of one or more lines of either distance or near acuity was recorded in the eye unaffected by macular disease. In each one of these cases the improvement in visual acuity could not be attributed to treatment of other existing pathology.", "12 such cases were detected. In nine of these the eye showing improvement of acuity had a history of amblyopia. The mean improvement in distance and near acuity in amblyopic eyes by 12 months was 3.3 and 1.9 lines logMAR respectively. The improvement in acuity generally occurred between 1 and 12 months from baseline and remained stable over the period of follow up." ], "LABELS": [ "METHODS", "RESULTS" ], "MESHES": [ "Amblyopia", "Distance Perception", "Female", "Follow-Up Studies", "Humans", "Macular Degeneration", "Male", "Neuronal Plasticity", "Retrospective Studies", "Visual Acuity" ], "YEAR": "2000", "reasoning_required_pred": "no", "reasoning_free_pred": "no", "final_decision": "no", "LONG_ANSWER": "Older people with a history of amblyopia who develop visual loss in the previously normal eye can experience recovery of visual function in the amblyopic eye over a period of time. This recovery in visual function occurs in the wake of visual loss in the fellow eye and the improvement appears to be sustained." }, "23690198": { "QUESTION": "Implementation of epidural analgesia for labor: is the standard of effective analgesia reachable in all women?", "CONTEXTS": [ "Social and cultural factors combined with little information may prevent the diffusion of epidural analgesia for pain relief during childbirth. The present study was launched contemporarily to the implementation of analgesia for labor in our Department in order to perform a 2 years audit on its use. The goal is to evaluate the epidural acceptance and penetration into hospital practice by women and care givers and safety and efficacy during childbirth.", "This audit cycle measured epidural analgesia performance against 4 standards: (1) Implementation of epidural analgesia for labor to all patients; (2) Acceptance and good satisfaction level reported by patients and caregivers. (3) Effectiveness of labor analgesia; (4) No maternal or fetal side effects.", "During the audit period epidural analgesia increased from 15.5% of all labors in the first trimester of the study to 51% in the last trimester (p<0.005). Satisfaction levels reported by patients and care givers were good. A hierarchical clustering analysis identified two clusters based on VAS (Visual Analogue Scale) time course: in 226 patients (cluster 1) VAS decreased from 8.5\u00b11.4 before to 4.1\u00b11.3 after epidural analgesia; in 1002 patients (cluster 2) VAS decreased from 8.12\u00b11.7 before (NS vs cluster 1), to 0.76\u00b10.79 after (p<0.001 vs before and vs cluster 2 after). No other differences between clusters were observed." ], "LABELS": [ "BACKGROUND", "PATIENTS AND METHODS", "RESULTS" ], "MESHES": [ "Adult", "Analgesia, Epidural", "Analgesia, Obstetrical", "Apgar Score", "Cesarean Section", "Cluster Analysis", "Female", "Hemodynamics", "Humans", "Infant, Newborn", "Pain Measurement", "Parity", "Patient Safety", "Patient Satisfaction", "Pregnancy" ], "YEAR": "2013", "reasoning_required_pred": "yes", "reasoning_free_pred": "yes", "final_decision": "yes", "LONG_ANSWER": "Present audit shows that the process of implementation of labor analgesia was quick, successful and safe, notwithstanding the identification of one cluster of women with suboptimal response to epidural analgesia that need to be further studies, overall pregnant womens'adhesion to labor analgesia was satisfactory." }, "17940352": { "QUESTION": "Does HER2 immunoreactivity provide prognostic information in locally advanced urothelial carcinoma patients receiving adjuvant M-VEC chemotherapy?", "CONTEXTS": [ "To evaluate the impact of HER2 immunoreactivity on clinical outcome in locally advanced urothelial carcinoma patients who received surgery alone, or methotrexate, vinblastine, epirubicin, and cisplatin (M-VEC) as adjuvant chemotherapy.", "We studied 114 formalin-fixed paraffin-embedded specimens obtained from locally advanced urothelial carcinoma patients receiving surgery alone or adjuvant M-VEC. The authors evaluated HER2 immunoreactivity using immunohistochemical staining and explored the influence of pathological parameters and HER2 immunoreactivity on progression-free survival (PFS) and disease-specific overall survival (OS) using univariate and multivariate Cox's analyses.", "Urothelial carcinoma of the bladder had a significantly higher frequency of HER2 immunoreactivity than that of the upper urinary tract (60.7 vs. 20.7%, p<0.0001). Overall, nodal status was a strong and independent prognostic indicator for clinical outcome. The HER2 immunoreactivity was significantly associated with PFS (p = 0.02) and disease-specific OS (p = 0.005) in advanced urothelial carcinoma patients. As for patients with adjuvant M-VEC, HER2 immunoreactivity was a significant prognostic factor for PFS (p = 0.03) and disease-specific OS (p = 0.02) using univariate analysis, but not multivariate analysis, and not for patients receiving watchful waiting." ], "LABELS": [ "INTRODUCTION", "MATERIALS AND METHODS", "RESULTS" ], "MESHES": [ "Adult", "Aged", "Aged, 80 and over", "Antineoplastic Combined Chemotherapy Protocols", "Carcinoma", "Chemotherapy, Adjuvant", "Cisplatin", "Disease-Free Survival", "Epirubicin", "Female", "Follow-Up Studies", "Humans", "Immunohistochemistry", "Lymphatic Metastasis", "Male", "Methotrexate", "Middle Aged", "Proportional Hazards Models", "Receptor, ErbB-2", "Time Factors", "Treatment Outcome", "Urinary Bladder Neoplasms", "Urothelium", "Vinblastine" ], "YEAR": "2007", "reasoning_required_pred": "yes", "reasoning_free_pred": "maybe", "final_decision": "maybe", "LONG_ANSWER": "HER2 immunoreactivity might have a limited prognostic value for advanced urothelial carcinoma patients with adjuvant M-VEC." }, "20537205": { "QUESTION": "Is halofantrine ototoxic?", "CONTEXTS": [ "Halofantrine is a newly developed antimalarial drug used for the treatment of Plasmodium falciparum malaria. The introduction of this drug has been delayed because of its possible side effects, and due to insufficient studies on adverse reactions in humans. There have been no studies investigating its effect on hearing.", "Thirty guinea pigs were divided into three groups: a control group, a halofantrine therapeutic dose group and a halofantrine double therapeutic dose group. One cochlea specimen from each animal was stained with haematoxylin and eosin and the other with toluidine blue.", "No changes were detected in the control group. The halofantrine therapeutic dose group showed loss and distortion of inner hair cells and inner phalangeal cells, and loss of spiral ganglia cells. In the halofantrine double therapeutic dose group, the inner and outer hair cells were distorted and there was loss of spiral ganglia cells." ], "LABELS": [ "INTRODUCTION", "METHODS", "RESULTS" ], "MESHES": [ "Animals", "Antimalarials", "Cochlea", "Dose-Response Relationship, Drug", "Guinea Pigs", "Hair Cells, Auditory, Outer", "Phenanthrenes", "Staining and Labeling" ], "YEAR": "2010", "reasoning_required_pred": "yes", "reasoning_free_pred": "yes", "final_decision": "yes", "LONG_ANSWER": "Halofantrine has mild to moderate pathological effects on cochlea histology, and can be considered an ototoxic drug." }, "28707539": { "QUESTION": "Visceral adipose tissue area measurement at a single level: can it represent visceral adipose tissue volume?", "CONTEXTS": [ "Measurement of visceral adipose tissue (VAT) needs to be accurate and sensitive to change for risk monitoring. The purpose of this study is to determine the CT slice location where VAT area can best reflect changes in VAT volume and body weight.", "60 plain abdominal CT images from 30 males\u00a0[mean age (range) 51 (41-68) years, mean body weight (range) 71.1 (101.9-50.9) kg] who underwent workplace screenings twice within a 1-year interval were evaluated. Automatically calculated and manually corrected areas of the VAT of various scan levels using \"freeform curve\" region of interest on CT were recorded and compared with body weight changes.", "The strongest correlations of VAT area with VAT volume and body weight changes were shown in a slice 3 cm above the lower margin of L3 with r values of 0.853 and 0.902, respectively." ], "LABELS": [ "OBJECTIVE", "METHODS", "RESULTS" ], "MESHES": [ "Abdomen", "Adult", "Aged", "Humans", "Intra-Abdominal Fat", "Male", "Middle Aged", "Multidetector Computed Tomography", "Reproducibility of Results", "Retrospective Studies" ], "YEAR": "2017", "reasoning_required_pred": "yes", "reasoning_free_pred": "yes", "final_decision": "yes", "LONG_ANSWER": "VAT area measurement at a single level 3 cm above the lower margin of the L3 vertebra is feasible and can reflect changes in VAT volume and body weight. Advances in knowledge: As VAT area at a CT slice 3cm above the lower margin of L3 can best reflect interval changes in VAT volume and body weight, VAT area measurement should be selected at this location." }, "7482275": { "QUESTION": "Necrotizing fasciitis: an indication for hyperbaric oxygenation therapy?", "CONTEXTS": [ "The accepted treatment protocol for necrotizing fasciitis (NF) consists of extensive surgery and wide spectrum antibiotics. Hyperbaric oxygenation (HBO) has been recommended as adjuvant therapy for NF, improving patient mortality and outcome. However, the beneficial effect of HBO for NF remains controversial.", "A retrospective evaluation of treatment outcome in 37 patients treated for NF between 1984 and 1993 was carried out. The mortality rate, morbidity criteria, and risk factors for grave prognosis were compared between a group of 25 patients who received HBO as part of their treatment protocol and a group of the remaining 12 patients treated by surgical excision and antibiotics alone.", "The two groups were found to be similar with regard to age, gender, the incidence of individual risk factors for ominous prognosis, and the Acute Physiology and Chronic Health Evaluation (APACHE) II score for disease's severity on presentation. The mortality rate among the HBO-treated patients was 36%, as opposed to 25% in the non-HBO group. The mean number of surgical d\u00e9bridements required per patient was significantly higher in the HBO group: 3.3 compared with 1.5 in the non-HBO-treated patients. Although the average length of hospitalization for survivors was shorter for the HBO group, the difference between the groups did not reach statistical significance." ], "LABELS": [ "BACKGROUND", "METHODS", "RESULTS" ], "MESHES": [ "Adolescent", "Adult", "Aged", "Aged, 80 and over", "Fasciitis, Necrotizing", "Female", "Humans", "Hyperbaric Oxygenation", "Male", "Middle Aged", "Retrospective Studies" ], "YEAR": "1995", "reasoning_required_pred": "no", "reasoning_free_pred": "no", "final_decision": "no", "LONG_ANSWER": "The results of this study cast doubt on the suggested advantage of HBO in reducing patient mortality and morbidity when used as adjuvant therapy for NF." }, "24183388": { "QUESTION": "Is the Hawkins sign able to predict necrosis in fractures of the neck of the astragalus?", "CONTEXTS": [ "To assess if the Hawkins sign can predict whether or not astragalus fractures of the neck will develop avascular necrosis. It is also assessed whether the occurrence of this complication is related to the displacement of the fracture, soft tissue injury, or delay in the reduction or surgery. The results were compared with those found in the literature.", "A retrospective study was conducted on 23 talar neck fractures recorded over a a period of thirteen years. The following variables were analysed: displacement of the fracture, soft tissue injury, delay and type of treatment, complications, observation of the Hawkins sign, and functional outcome.", "There were 7 type I Hawkins fractures, 11 type II, and 4 type III and 1 type IV. Four cases developed avascular necrosis (2 Hawkins type II and 2 type III). Hawkins sign was observed in 12 cases, of which none developed necrosis. Four cases with negative Hawkins sign developed necrosis. No statistically significant differences were found when comparing the development of avascular necrosis with the displacement of the fracture, soft tissue injury, or delay in treatment. Differences were found when comparing the development of avascular necrosis with the Hawkins sign (P=.03)." ], "LABELS": [ "OBJECTIVE", "MATERIAL AND METHODS", "RESULTS" ], "MESHES": [ "Adolescent", "Adult", "Child", "Child, Preschool", "Female", "Fractures, Bone", "Humans", "Male", "Middle Aged", "Osteonecrosis", "Prognosis", "Retrospective Studies", "Talus", "Young Adult" ], "YEAR": null, "reasoning_required_pred": "no", "reasoning_free_pred": "maybe", "final_decision": "maybe", "LONG_ANSWER": "A positive Hawkins sign rules out that the fractured talus has developed avascular necrosis, but its absence does not confirm it." }, "9645785": { "QUESTION": "Is a mandatory general surgery rotation necessary in the surgical clerkship?", "CONTEXTS": [ "Changes in the spectrum of general surgery and the delivery of surgical care have placed the requirement for a mandatory general surgery rotation in the surgical clerkship in question.", "We tested the hypothesis that equal mastery of surgical clerkship objectives can be obtained in a clerkship with and without general surgery. Students chose any two surgical rotations and were assessed by written examination, objective structured clinical examination (OSCE), ward evaluations, self-assessment objectives questionnaire, and satisfaction survey.", "Data for 54 students showed no differences in scores between groups on any parameter. No specific concerns related to the absence of general surgery were identified." ], "LABELS": [ "BACKGROUND", "METHODS", "RESULTS" ], "MESHES": [ "Attitude", "Clinical Clerkship", "Educational Measurement", "General Surgery", "Humans", "Medicine", "Specialization", "Students, Medical" ], "YEAR": "1998", "reasoning_required_pred": "no", "reasoning_free_pred": "no", "final_decision": "no", "LONG_ANSWER": "Effective undergraduate surgical education can be offered in many specialty settings. Removal of the requirement for general surgery in clerkship may lead to a more effective use of all educational opportunities. A careful analysis of local programs and facilities is necessary before suggesting this change to other institutions." }, "26298839": { "QUESTION": "Is Acupuncture Efficacious for Treating Phonotraumatic Vocal Pathologies?", "CONTEXTS": [ "To investigate the effectiveness of acupuncture in treating phonotraumatic vocal fold lesions.STUDY DESIGN/", "A total of 123 dysphonic individuals with benign vocal pathologies were recruited. They were given either genuine acupuncture (n\u00a0=\u00a040), sham acupuncture (n\u00a0=\u00a044), or no treatment (n\u00a0=\u00a039) for 6\u00a0weeks (two 30-minute sessions/wk). The genuine acupuncture group received needles puncturing nine voice-related acupoints for 30\u00a0minutes, two times a week for 6\u00a0weeks, whereas the sham acupuncture group received blunted needles stimulating the skin surface of the nine acupoints for the same frequency and duration. The no-treatment group did not receive any intervention but attended just the assessment sessions. One-hundred seventeen subjects completed the study (genuine acupuncture\u00a0=\u00a040; sham acupuncture\u00a0=\u00a043; and no treatment\u00a0=\u00a034), but only 84 of them had a complete set of vocal functions and quality of life measures (genuine acupuncture\u00a0=\u00a029; sham acupuncture\u00a0=\u00a033; and no-treatment\u00a0=\u00a022) and 42 of them with a complete set of endoscopic data (genuine acupuncture\u00a0=\u00a016; sham acupuncture\u00a0=\u00a015; and no treatment\u00a0=\u00a011).", "Significant improvement in vocal function, as indicated by the maximum fundamental frequency produced, and also perceived quality of life, were found in both the genuine and sham acupuncture groups, but not in the no-treatment group. Structural (morphological) improvements were, however, only noticed in the genuine acupuncture group, which demonstrated a significant reduction in the size of the vocal fold lesions." ], "LABELS": [ "OBJECTIVES", "METHODS", "RESULTS" ], "MESHES": [ "Acoustics", "Acupuncture Therapy", "Adult", "Dysphonia", "Female", "Hong Kong", "Humans", "Laryngoscopy", "Male", "Middle Aged", "Quality of Life", "Recovery of Function", "Speech Production Measurement", "Stroboscopy", "Surveys and Questionnaires", "Time Factors", "Treatment Outcome", "Video Recording", "Vocal Cords", "Voice Quality", "Wound Healing", "Young Adult" ], "YEAR": "2016", "reasoning_required_pred": "yes", "reasoning_free_pred": "yes", "final_decision": "yes", "LONG_ANSWER": "The findings showed that acupuncture of voice-related acupoints could bring about improvement in vocal function and healing of vocal fold lesions." }, "24153338": { "QUESTION": "Is aneurysm repair justified for the patients aged 80 or older after aneurysmal subarachnoid hemorrhage?", "CONTEXTS": [ "With the advancement of an aging society in the world, an increasing number of elderly patients have been hospitalized due to aneurysmal subarachnoid hemorrhage (aSAH). There is no study that compares the elderly cases of aSAH who receive the definitive treatment with those who treated conservatively. The aim of this study was to investigate the feasibility of the definitive surgery for the acute subarachnoid cases aged 80 or older.", "We reviewed 500 consecutive cases with acute aSAH with surgical indication for aneurysm repair. Inoperable cases such as dead-on-arrival and the cases with both pupils dilated were excluded. We compared the cases aged 80 or older that received clipping or coil embolization with the controls that the family selected conservative treatment.", "69 cases were included in this study (ranged 80-98, male:female=9:60). 56 cases (81.2%) had an aneurysm in the anterior circulation. 23 cases received clipping, 20 cases coil embolization and 26 cases treated conservatively. The cases with aneurysm repair showed significantly better clinical outcome than the controls, while World Federation of Neurological Surgeons (WFNS) grade on admission and premorbid modified Rankin Scale showed no difference between them." ], "LABELS": [ "BACKGROUND", "METHODS", "RESULTS" ], "MESHES": [ "Aged, 80 and over", "Aneurysm, Ruptured", "Cerebrovascular Circulation", "Cohort Studies", "Embolization, Therapeutic", "Female", "Humans", "Male", "Neurosurgical Procedures", "Prognosis", "Retrospective Studies", "Subarachnoid Hemorrhage", "Treatment Outcome" ], "YEAR": "2014", "reasoning_required_pred": "yes", "reasoning_free_pred": "yes", "final_decision": "yes", "LONG_ANSWER": "Better prognosis was obtained when ruptured aneurysm was repaired in the elderly than it was treated conservatively. From the results of this study, we should not hesitate to offer the definitive surgery for the elderly with aSAH." }, "18534072": { "QUESTION": "Do general practice characteristics influence uptake of an information technology (IT) innovation in primary care?", "CONTEXTS": [ "Recent evaluations of IT innovations in primary care have highlighted variations between centres and practices in uptake and use. We evaluated whether structural characteristics of a general practice were associated with variations in use of a web-based clinical information system underpinning a Managed Clinical Network in diabetes, between the years 2001 and 2003.", "Using a computerised audit trail, we calculated the numbers of web-based operations that occurred in each practice, stratified by staff type and year, and adjusted for the numbers of registered diabetic patients. In regression analyses, we determined whether total use was associated with structural characteristics of the practice (total list size, training status, numbers of GPs (general practitioners), mean age of the GPs, numbers of female GPs, level of deprivation of the population and whether staff had received advanced training in diabetes care).", "Initially there were a few practices which made very frequent use of the information system, with relatively high numbers of practices using the facility infrequently. However, overall use gradually became more evenly spread. This effect was particularly evident among nurse users. Frequent use by GPs was evident in only a small number of practices, with mean GP use decreasing over the three years. In linear regression analyses, none of the general practice variables were associated with online use, either overall or stratified by staff type, except for the numbers of diabetes-educated staff. This was consistently associated with increased use by nurses and GPs." ], "LABELS": [ "INTRODUCTION", "METHODS", "RESULTS" ], "MESHES": [ "Age Factors", "Diabetes Mellitus", "Diffusion of Innovation", "Disease Management", "Family Practice", "Humans", "Information Systems", "Internet", "Sex Factors", "Socioeconomic Factors", "Time Factors" ], "YEAR": "2008", "reasoning_required_pred": "no", "reasoning_free_pred": "no", "final_decision": "no", "LONG_ANSWER": "The analyses show that structural characteristics of a practice are not associated with uptake of a new IT facility, but that its use may be influenced by post-graduate education in the relevant clinical condition. For this diabetes system at least, practice nurse use was critical in spreading uptake beyond initial GP enthusiasts and for sustained and rising use in subsequent years." }, "8847047": { "QUESTION": "Prognosis of well differentiated small hepatocellular carcinoma--is well differentiated hepatocellular carcinoma clinically early cancer?", "CONTEXTS": [ "The purpose of this study is to examine whether or not well differentiated (w-d) hepatocellular carcinoma (HCC) is indeed clinically early cancer.", "Seventy six patients with solitary small HCCs up to 3 cm in diameter, who underwent hepatectomy, were observed for at least 2 years for possible recurrence. These patients were divided into two groups: 10 patients with w-d HCCs (Edmondson and Steiner's grade I) and 66 patients with less differentiated (l-d) HCCs (Edmondson and Steiner's grade I-II, II-III, and III).", "The histological analysis revealed that w-d HCCs had lower incidences of fibrous capsule formation (P<0.01), when compared to l-d HCCs. There were no significant differences in the incidence of intrahepatic metastasis, or portal vein invasion. In a resected specimen of w-d HCC, barium sulfate and gelatin were injected into portal vein and a transparent specimen was made. The transparent specimen showed that the portal vein in the tumor seemed to be intact. Microscopically, cancer cell infiltration into the fibrous frame of the portal tract was present. There were no significant differences in the disease free survival between the two groups. An analysis of tumor volume doubling time in recurrent foci suggested that minute cancerous foci had been present at the time of operation." ], "LABELS": [ "AIMS", "MATERIALS AND METHODS", "RESULTS" ], "MESHES": [ "Carcinoma, Hepatocellular", "Case-Control Studies", "Disease-Free Survival", "Female", "Follow-Up Studies", "Hepatectomy", "Humans", "Incidence", "Liver", "Liver Neoplasms", "Male", "Middle Aged", "Neoplasm Invasiveness", "Neoplasm Recurrence, Local", "Portal Vein", "Prognosis", "Time Factors" ], "YEAR": null, "reasoning_required_pred": "yes", "reasoning_free_pred": "no", "final_decision": "no", "LONG_ANSWER": "W-d HCCs were clinically demonstrated not to be early cancer, because there was no significant difference in disease free survival between the patients with w-d and l-d HCCs." }, "10575390": { "QUESTION": "Do follow-up recommendations for abnormal Papanicolaou smears influence patient adherence?", "CONTEXTS": [ "To compare adherence to follow-up recommendations for colposcopy or repeated Papanicolaou (Pap) smears for women with previously abnormal Pap smear results.", "Retrospective cohort study.", "Three northern California family planning clinics.", "All women with abnormal Pap smear results referred for initial colposcopy and a random sample of those referred for repeated Pap smear. Medical records were located and reviewed for 90 of 107 women referred for colposcopy and 153 of 225 women referred for repeated Pap smears.", "Routine clinic protocols for follow-up--telephone call, letter, or certified letter--were applied without regard to the type of abnormality seen on a Pap smear or recommended examination.", "Documented adherence to follow-up within 8 months of an abnormal result. Attempts to contact the patients for follow-up, adherence to follow-up recommendations, and patient characteristics were abstracted from medical records. The probability of adherence to follow-up vs the number of follow-up attempts was modeled with survival analysis. Cox proportional hazards models were used to examine multivariate relationships related to adherence.", "The rate of overall adherence to follow-up recommendations was 56.0% (136/243). Adherence to a second colposcopy was not significantly different from that to a repeated Pap smear (odds ratio, 1.40; 95% confidence interval, 0.80-2.46). The use of as many as 3 patient reminders substantially improved adherence to follow-up. Women without insurance and women attending 1 of the 3 clinics were less likely to adhere to any follow-up recommendation (hazard ratio for no insurance, 0.43 [95% confidence interval, 0.20-0.93], and for clinic, 0.35 [95% confidence interval, 0.15-0.73])." ], "LABELS": [ "OBJECTIVE", "DESIGN", "SETTING", "PATIENTS", "INTERVENTION", "MAIN OUTCOME MEASURES", "RESULTS" ], "MESHES": [ "Adolescent", "Adult", "Aged", "Cervix Uteri", "Colposcopy", "Female", "Humans", "Middle Aged", "Papanicolaou Test", "Patient Compliance", "Retrospective Studies", "Uterine Cervical Neoplasms", "Vaginal Smears" ], "YEAR": null, "reasoning_required_pred": "yes", "reasoning_free_pred": "no", "final_decision": "no", "LONG_ANSWER": "Adherence to follow-up was low in this family planning clinic population, no matter what type of follow-up was advised. Adherence was improved by the use of up to 3 reminders. Allocating resources to effective methods for improving adherence to follow-up of abnormal results may be more important than which follow-up procedure is recommended." }, "20084845": { "QUESTION": "Biomolecular identification of allergenic pollen: a new perspective for aerobiological monitoring?", "CONTEXTS": [ "Accurate and updated information on airborne pollen in specific areas can help allergic patients. Current monitoring systems are based on a morphologic identification approach, a time-consuming method that may represent a limiting factor for sampling network enhancement.", "To verify the feasibility of developing a real-time polymerase chain reaction (PCR) approach, an alternative to optical analysis, as a rapid, accurate, and automated tool for the detection and quantification of airborne allergenic pollen taxa.", "The traditional cetyl trimethyl ammonium bromide-based method was modified for DNA isolation from pollen. Taxon-specific DNA sequences were identified via bioinformatics or literature searches and were PCR amplified from the matching allergenic taxa; based on the sequences of PCR products, complementary or degenerate TaqMan probes were developed. The accuracy of the quantitative real-time PCR assay was tested on 3 plant species.", "The setup of a modified DNA extraction protocol allowed us to achieve good-quality pollen DNA. Taxon-specific nuclear gene fragments were identified and sequenced. Designed primer pairs and probes identified selected pollen taxa, mostly at the required classification level. Pollen was properly identified even when collected on routine aerobiological tape. Preliminary quantification assays on pollen grains were successfully performed on test species and in mixes." ], "LABELS": [ "BACKGROUND", "OBJECTIVE", "METHODS", "RESULTS" ], "MESHES": [ "Antigens, Plant", "Computational Biology", "DNA Primers", "DNA Probes", "DNA, Plant", "Environmental Monitoring", "Italy", "Molecular Probe Techniques", "Plant Leaves", "Plant Proteins", "Pollen", "Polymerase Chain Reaction" ], "YEAR": "2009", "reasoning_required_pred": "yes", "reasoning_free_pred": "yes", "final_decision": "yes", "LONG_ANSWER": "The real-time PCR approach revealed promising results in pollen identification and quantification, even when analyzing pollen mixes. Future perspectives could concern the development of multiplex real-time PCR for the simultaneous detection of different taxa in the same reaction tube and the application of high-throughput molecular methods." }, "15703931": { "QUESTION": "Does diabetes mellitus influence the efficacy of FDG-PET in the diagnosis of cervical cancer?", "CONTEXTS": [ "Compared with computed tomography (CT) and magnetic resonance imaging (MRI), positron emission tomography (PET) may have additional value in the assessment of primary and recurrent cervical cancer. However, the degree of tumour uptake of (18)F-2-fluoro-2-deoxy-D: -glucose (FDG) uptake is sometimes influenced by diabetes mellitus (DM). Therefore, we conducted this prospective study to compare the diagnostic ability of FDG-PET in patients with cervical cancer complicated by DM and those without DM.", "Patients with untreated locally advanced primary or clinically curable recurrent cervical carcinoma were enrolled. Both FDG-PET and MRI/CT scans were performed within 2 weeks. Patients were categorised into the following groups: hyperglycaemic DM (fasting blood sugar>126 mg/dl), euglycaemic DM and non-DM. The lesions were confirmed histologically or by clinical follow-up. The receiver operating characteristic curve method, with calculation of the area under the curve (AUC), was used to evaluate the discriminative power.", "From February 2001 to January 2003, 219 patients (75 with primary and 144 with recurrent cervical cancer) were eligible for analysis. Sixteen had hyperglycaemic DM, 12 had euglycaemic DM and 191 were in the non-DM group. The diagnostic power of PET in the hyperglycaemic DM, euglycaemic DM and non-DM groups did not differ significantly with regard to the identification of either metastatic lesions (AUC, 0.967/0.947/0.925, P>0.05) or primary tumours/local recurrence (AUC, 0.950/0.938/0.979, P>0.05). Considering all DM patients, PET showed a significantly higher detection power than MRI/CT scans in respect of metastatic lesions (AUC=0.956 vs 0.824, P=0.012)." ], "LABELS": [ "PURPOSE", "METHODS", "RESULTS" ], "MESHES": [ "Adult", "Aged", "Aged, 80 and over", "Artifacts", "Diabetes Complications", "Female", "Fluorodeoxyglucose F18", "Humans", "Middle Aged", "Positron-Emission Tomography", "Radiopharmaceuticals", "Reproducibility of Results", "Sensitivity and Specificity", "Uterine Cervical Neoplasms" ], "YEAR": "2005", "reasoning_required_pred": "no", "reasoning_free_pred": "no", "final_decision": "no", "LONG_ANSWER": "In comparison with its accuracy in non-DM patients, the accuracy of PET in cervical cancer patients with mild to moderate DM was not significantly reduced." }, "18269157": { "QUESTION": "Biomechanical and wound healing characteristics of corneas after excimer laser keratorefractive surgery: is there a difference between advanced surface ablation and sub-Bowman's keratomileusis?", "CONTEXTS": [ "To describe the biomechanical and wound healing characteristics of corneas after excimer laser keratorefractive surgery.", "Histologic, ultrastructural, and cohesive tensile strength evaluations were performed on 25 normal human corneal specimens, 206 uncomplicated LASIK specimens, 17 uncomplicated sub-Bowman's keratomileusis (SBK) specimens, 4 uncomplicated photorefractive keratectomy (PRK) specimens, 2 uncomplicated advanced surface ablation (ASA) specimens, 5 keratoconus specimens, 12 postoperative LASIK ectasia specimens, and 1 postoperative PRK ectasia specimen and compared to previously published studies.", "Histologic and ultrastructural studies of normal corneas showed significant differences in the direction of collagen fibrils and/or the degree of lamellar interweaving in Bowman's layer, the anterior third of the corneal stroma, the posterior two-thirds of the corneal stroma, and Descemet's membrane. Cohesive tensile strength testing directly supported these morphologic findings as the stronger, more rigid regions of the cornea were located anteriorly and peripherally. This suggests that PRK and ASA, and secondarily SBK, should be biomechanically safer than conventional LASIK with regard to risk for causing keratectasia after surgery. Because adult human corneal stromal wounds heal slowly and incompletely, all excimer laser keratorefractive surgical techniques still have some distinct disadvantages due to inadequate reparative wound healing. Despite reducing some of the risk for corneal haze compared to conventional PRK, ASA cases still can develop corneal haze or breakthrough haze from the hypercellular fibrotic stromal scarring. In contrast, similar to conventional LASIK, SBK still has the short- and long-term potential for interface wound complications from the hypocellular primitive stromal scar." ], "LABELS": [ "PURPOSE", "METHODS", "RESULTS" ], "MESHES": [ "Biomechanical Phenomena", "Bowman Membrane", "Compressive Strength", "Cornea", "Corneal Surgery, Laser", "Humans", "Keratoconus", "Lasers, Excimer", "Tensile Strength", "Wound Healing" ], "YEAR": "2008", "reasoning_required_pred": "yes", "reasoning_free_pred": "maybe", "final_decision": "yes", "LONG_ANSWER": "Ophthalmic pathology and basic science research show that SBK and ASA are improvements in excimer laser keratorefractive surgery compared to conventional LASIK or PRK, particularly with regard to maintaining corneal biomechanics and perhaps moderately reducing the risk of corneal haze. However, most of the disadvantages caused by wound healing issues remain." }, "25489696": { "QUESTION": "Does radiotherapy of the primary rectal cancer affect prognosis after pelvic exenteration for recurrent rectal cancer?", "CONTEXTS": [ "Radiotherapy reduces local recurrence rates but is also capable of short- and long-term toxicity. It may also render treatment of local recurrence more challenging if it develops despite previous radiotherapy.", "This study examined the impact of radiotherapy for the primary rectal cancer on outcomes after pelvic exenteration for local recurrence.", "We conducted a retrospective review of exenteration databases.", "The study took place at a quaternary referral center that specializes in pelvic exenteration.", "Patients referred for pelvic exenteration from October 1994 to November 2012 were reviewed. Patients who did and did not receive radiotherapy as part of their primary rectal cancer treatment were compared.", "The main outcomes of interest were resection margins, overall survival, disease-free survival, and surgical morbidities.", "There were 108 patients, of which 87 were eligible for analysis. Patients who received radiotherapy for their primary rectal cancer (n = 41) required more radical exenterations (68% vs 44%; p = 0.020), had lower rates of clear resection margins (63% vs 87%; p = 0.010), had increased rates of surgical complications per patient (p = 0.014), and had a lower disease-free survival (p = 0.022). Overall survival and disease-free survival in patients with clear margins were also lower in the primary irradiated patients (p = 0.049 and p<0.0001). This difference in survival persisted in multivariate analysis that corrected for T and N stages of the primary tumor.", "This study is limited by its retrospective nature and heterogeneous radiotherapy regimes among radiotherapy patients." ], "LABELS": [ "BACKGROUND", "OBJECTIVE", "DESIGN", "SETTING", "PATIENTS", "MAIN OUTCOME MEASURES", "RESULTS", "LIMITATIONS" ], "MESHES": [ "Adult", "Aged", "Aged, 80 and over", "Combined Modality Therapy", "Female", "Humans", "Male", "Middle Aged", "Neoplasm Recurrence, Local", "Neoplasm Staging", "Pelvic Exenteration", "Prognosis", "Rectal Neoplasms", "Retrospective Studies", "Survival Rate", "Treatment Outcome" ], "YEAR": "2015", "reasoning_required_pred": "yes", "reasoning_free_pred": "yes", "final_decision": "yes", "LONG_ANSWER": "Patients who previously received radiotherapy for primary rectal cancer treatment have worse oncologic outcomes than those who had not received radiotherapy after pelvic exenteration for locally recurrent rectal cancer." }, "14599616": { "QUESTION": "Can a practicing surgeon detect early lymphedema reliably?", "CONTEXTS": [ "Lymphedema may be identified by simpler circumference changes as compared with changes in limb volume.", "Ninety breast cancer patients were prospectively enrolled in an academic trial, and seven upper extremity circumferences were measured quarterly for 3 years. A 10% volume increase or greater than 1 cm increase in arm circumference identified lymphedema with verification by a lymphedema specialist. Sensitivity and specificity of several different criteria for detecting lymphedema were compared using the academic trial as the standard.", "Thirty-nine cases of lymphedema were identified by the academic trial. Using a 10% increase in circumference at two sites as the criterion, half the lymphedema cases were detected (sensitivity 37%). When using a 10% increase in circumference at any site, 74.4% of cases were detected (sensitivity 49%). Detection by a 5% increase in circumference at any site was 91% sensitive." ], "LABELS": [ "BACKGROUND", "METHODS", "RESULTS" ], "MESHES": [ "Arm", "Breast Neoplasms", "Early Diagnosis", "Female", "Follow-Up Studies", "Humans", "Lymphedema", "Middle Aged", "Postoperative Complications", "Prospective Studies", "Radiotherapy, Adjuvant", "Sensitivity and Specificity", "Time Factors" ], "YEAR": "2003", "reasoning_required_pred": "yes", "reasoning_free_pred": "maybe", "final_decision": "maybe", "LONG_ANSWER": "An increase of 5% in circumference measurements identified the most potential lymphedema cases compared with an academic trial." }, "22537902": { "QUESTION": "Colorectal cancer with synchronous liver metastases: does global management at the same centre improve results?", "CONTEXTS": [ "Synchronous liver metastases (SLM) occur in 20% of colorectal cancers (CRC). Resection of SLM and CLC can be undertaken at different centres (separate management, SM) or at the same centre (global management, GM).", "Retrospective study of SLM and CRC resections carried out during 01/2000 - 12/2006 by SM or GM, using a combined or delayed strategy.", "Morphologic characteristics and type of CRC and SLM resection were similar for the GM (n = 45) or SM (n = 66) groups. In patients with delayed liver resection (62 SM, 17 GM), chemotherapy prior to liver surgery was used in 92% and 38% of SM and GM patients (P<0.0001) and the median delay between procedures was 212 and 182 days, respectively (P = 0.04). First step of liver resection was more often performed during colorectal surgery in the GM group (62 vs. 6% for SM, P<0.0001) and the mean number of procedures (CRC+SLM) was lower (1.6 vs. 2.3, P = 0.003). Three-month mortality was 3% for GM and 0% for SM (n.s.). Overall survival rates were 67% and 51% for SM and GM at 3 years (n.s.), and 35 and 31% at 5 years (n.s.). Disease-free survival to 5 years was higher in SM patients (14% vs. 11%, P = 0.009)." ], "LABELS": [ "BACKGROUND", "METHODS", "RESULTS" ], "MESHES": [ "Colorectal Neoplasms", "Combined Modality Therapy", "Female", "Humans", "Liver Neoplasms", "Male", "Middle Aged", "Retrospective Studies", "Treatment Outcome" ], "YEAR": "2013", "reasoning_required_pred": "no", "reasoning_free_pred": "no", "final_decision": "no", "LONG_ANSWER": "GM of CRC and SLM was associated with fewer procedures but did not influence overall survival. SM was associated with a longer delay and increased use of chemotherapy between procedures, suggesting that more rigorous selection of SM patients for surgery may explain the higher disease-free survival after SLM resection." }, "19054501": { "QUESTION": "Is motion perception deficit in schizophrenia a consequence of eye-tracking abnormality?", "CONTEXTS": [ "Studies have shown that schizophrenia patients have motion perception deficit, which was thought to cause eye-tracking abnormality in schizophrenia. However, eye movement closely interacts with motion perception. The known eye-tracking difficulties in schizophrenia patients may interact with their motion perception.", "Two speed discrimination experiments were conducted in a within-subject design. In experiment 1, the stimulus duration was 150 msec to minimize the chance of eye-tracking occurrence. In experiment 2, the duration was increased to 300 msec, increasing the possibility of eye movement intrusion. Regular eye-tracking performance was evaluated in a third experiment.", "At 150 msec, speed discrimination thresholds did not differ between schizophrenia patients (n = 38) and control subjects (n = 33). At 300 msec, patients had significantly higher thresholds than control subjects (p = .03). Furthermore, frequencies of eye tracking during the 300 msec stimulus were significantly correlated with speed discrimination in control subjects (p = .01) but not in patients, suggesting that eye-tracking initiation may benefit control subjects but not patients. The frequency of eye tracking during speed discrimination was not significantly related to regular eye-tracking performance." ], "LABELS": [ "BACKGROUND", "METHODS", "RESULTS" ], "MESHES": [ "Adolescent", "Adult", "Discrimination (Psychology)", "Female", "Fixation, Ocular", "Humans", "Male", "Middle Aged", "Motion Perception", "Ocular Motility Disorders", "Psychomotor Performance", "Pursuit, Smooth", "Schizophrenic Psychology", "Temporal Lobe", "Visual Pathways", "Young Adult" ], "YEAR": "2009", "reasoning_required_pred": "yes", "reasoning_free_pred": "yes", "final_decision": "yes", "LONG_ANSWER": "Speed discrimination, per se, is not impaired in schizophrenia patients. The observed abnormality appears to be a consequence of impairment in generating or integrating the feedback information from eye movements. This study introduces a novel approach to motion perception studies and highlights the importance of concurrently measuring eye movements to understand interactions between these two systems; the results argue for a conceptual revision regarding motion perception abnormality in schizophrenia." }, "16432652": { "QUESTION": "Transgastric endoscopic splenectomy: is it possible?", "CONTEXTS": [ "We have previously reported the feasibility of diagnostic and therapeutic peritoneoscopy including liver biopsy, gastrojejunostomy, and tubal ligation by an oral transgastric approach. We present results of per-oral transgastric splenectomy in a porcine model. The goal of this study was to determine the technical feasibility of per-oral transgastric splenectomy using a flexible endoscope.", "We performed acute experiments on 50-kg pigs. All animals were fed liquids for 3 days prior to procedure. The procedures were performed under general anesthesia with endotracheal intubation. The flexible endoscope was passed per orally into the stomach and puncture of the gastric wall was performed with a needle knife. The puncture was extended to create a 1.5-cm incision using a pull-type sphincterotome, and a double-channel endoscope was advanced into the peritoneal cavity. The peritoneal cavity was insufflated with air through the endoscope. The spleen was visualized. The splenic vessels were ligated with endoscopic loops and clips, and then mesentery was dissected using electrocautery.", "Endoscopic splenectomy was performed on six pigs. There were no complications during gastric incision and entrance into the peritoneal cavity. Visualization of the spleen and other intraperitoneal organs was very good. Ligation of the splenic vessels and mobilization of the spleen were achieved using commercially available devices and endoscopic accessories." ], "LABELS": [ "BACKGROUND", "METHODS", "RESULTS" ], "MESHES": [ "Animals", "Endoscopy", "Models, Animal", "Spleen", "Splenectomy", "Stomach", "Swine" ], "YEAR": "2006", "reasoning_required_pred": "yes", "reasoning_free_pred": "yes", "final_decision": "yes", "LONG_ANSWER": "Transgastric endoscopic splenectomy in a porcine model appears technically feasible. Additional long-term survival experiments are planned." }, "19504993": { "QUESTION": "It's Fournier's gangrene still dangerous?", "CONTEXTS": [ "Fournier's gangrene is known to have an impact in the morbidity and despite antibiotics and aggressive debridement, the mortality rate remains high.", "To assess the morbidity and mortality in the treatment of Fournier's gangrene in our experience.", "The medical records of 14 patients with Fournier's gangrene who presented at the University Hospital Center \"Mother Teresa\" from January 1997 to December 2006 were reviewed retrospectively to analyze the outcome and identify the risk factor and prognostic indicators of mortality.", "Of the 14 patients, 5 died and 9 survived. Mean age was 54 years (range from 41-61): it was 53 years in the group of survivors and 62 years in deceased group. There was a significant difference in leukocyte count between patients who survived (range 4900-17000/mm) and those died (range 20.300-31000/mm3). Mean hospital stay was about 19 days (range 2-57 days)." ], "LABELS": [ "BACKGROUND", "OBJECTIVES", "METHODS", "RESULTS" ], "MESHES": [ "Adult", "Female", "Fournier Gangrene", "Humans", "Male", "Middle Aged", "Survival Rate" ], "YEAR": "2009", "reasoning_required_pred": "yes", "reasoning_free_pred": "yes", "final_decision": "yes", "LONG_ANSWER": "The interval from the onset of clinical symptoms to the initial surgical intervention seems to be the most important prognostic factor with a significant impact on outcome. Despite extensive therapeutic efforts, Fournier's gangrene remains a surgical emergency and early recognition with prompt radical debridement is the mainstays of management." }, "20571467": { "QUESTION": "Is it appropriate to implant kidneys from elderly donors in young recipients?", "CONTEXTS": [ "Kidneys from elderly donors tend to be implanted in recipients who are also elderly. We present the results obtained after 10 years of evolution on transplanting elderly kidneys into young recipients.", "Ninety-one consecutive transplants are studied, carried out in our center with kidneys from cadaver donors older than 60 years implanted in recipients younger than 60 years. The control group is made up of 91 transplants, matched with those from the study group, whose donor and recipient were younger than 60 years.", "There were no differences between groups with regard to recipient age, sex, cause of death and renal function of the donor, hepatitis C and cytomegalovirus serologies, cold ischemia time, tubular necrosis, immediate diuresis, need for dialysis, human leukocyte antigen incompatibilities, hypersensitized patients, acute rejection, waiting time on dialysis, and days of admission. Survival in both groups at 1, 5, and 10 years was 97.6%, 87.2%, and 76.6% vs. 98.8%, 87.5%, and 69.5% for the patient (P=0.642), 92.9%, 81.3%, and 64.2% vs. 93.9%, 76.4%, and 69.5% for the graft (P=0.980), and 94.4%, 92.6%, and 77.4% vs. 94.3%, 86.7%, and 84.4% for the graft with death censured (P=0.747), respectively. Creatininaemias at 1, 5, and 10 years were 172, 175, and 210 vs. 139, 134, and 155 (P<0.05)." ], "LABELS": [ "BACKGROUND", "METHODS", "RESULTS" ], "MESHES": [ "Adult", "Age Factors", "Aged", "Biomarkers", "Case-Control Studies", "Chi-Square Distribution", "Creatinine", "Donor Selection", "Female", "Glomerular Filtration Rate", "Graft Survival", "Humans", "Kaplan-Meier Estimate", "Kidney Transplantation", "Male", "Middle Aged", "Prospective Studies", "Resource Allocation", "Risk Assessment", "Risk Factors", "Time Factors", "Tissue Donors", "Treatment Outcome", "Young Adult" ], "YEAR": "2010", "reasoning_required_pred": "yes", "reasoning_free_pred": "yes", "final_decision": "yes", "LONG_ANSWER": "We conclude that patient and graft survival on transplanting kidneys from elderly donors to young recipients is superimposable on that obtained with young donors. However, renal function is better in the group of young donors." }, "24237112": { "QUESTION": "Do provider service networks result in lower expenditures compared with HMOs or primary care case management in Florida's Medicaid program?", "CONTEXTS": [ "To determine the impact of Florida's Medicaid Demonstration 4 years post-implementation on per member per month (PMPM) Medicaid expenditures and whether receiving care through HMOs versus provider service networks (PSNs) in the Demonstration was associated with PMPM expenditures.DATA: Florida Medicaid claims from two fiscal years prior to implementation of the Demonstration (FY0405, FY0506) and the first four fiscal years after implementation (FY0607-FY0910) from two urban Demonstration counties and two urban non-Demonstration counties.", "A difference-in-difference approach was used to compare changes in enrollee expenditures before and after implementation of the Demonstration overall and specifically for HMOs and PSNs.", "Claims data were extracted for enrollees in the Demonstration and non-Demonstration counties and collapsed into monthly amounts (N = 26,819,987 person-months).", "Among SSI enrollees, the Demonstration resulted in lower increases in PMPM expenditures over time ($40) compared with the non-Demonstration counties ($186), with Demonstration PSNs lowering PMPM expenditures by $7 more than HMOs. Savings were also seen among TANF enrollees but to a lesser extent." ], "LABELS": [ "OBJECTIVE", "STUDY DESIGN", "DATA EXTRACTION", "PRINCIPAL FINDINGS" ], "MESHES": [ "Adolescent", "Adult", "Aged", "Case Management", "Child", "Child, Preschool", "Female", "Florida", "Health Expenditures", "Health Maintenance Organizations", "Humans", "Infant", "Male", "Medicaid", "Middle Aged", "Primary Health Care", "United States", "Young Adult" ], "YEAR": "2014", "reasoning_required_pred": "yes", "reasoning_free_pred": "yes", "final_decision": "yes", "LONG_ANSWER": "The Medicaid Demonstration in Florida appears to result in lower PMPM expenditures. Demonstration PSNs generated slightly greater reductions in expenditures compared to Demonstration HMOs. PSNs appear to be a promising model for delivering care to Medicaid enrollees." }, "21402341": { "QUESTION": "Assessment of carotid artery stenosis before coronary artery bypass surgery. Is it always necessary?", "CONTEXTS": [ "Extracranial internal carotid artery stenosis is a risk factor for perioperative stroke in patients undergoing coronary artery bypass surgery (CAB). Although selective and non-selective methods of preoperative carotid screening have been advocated, it remains unclear if this screening is clinically relevant.AIM: To test whether selective carotid screening is as effective as non-selective screening in detecting significant carotid disease.", "The case records of patients consecutively undergoing CAB were reviewed. Patients were stratified retrospectively into high- or low-risk groups according to risk factors for significant carotid stenosis and perioperative stroke: peripheral vascular disease (PVD), carotid bruit, diabetes mellitus, age>70 years and/or history of cerebrovascular disease. Prevalence of carotid stenosis detected by ultrasonography, surgical management and perioperative stroke rates were determined in each group.", "Overall, 205 consecutive patients underwent preoperative carotid screening. The prevalence of significant carotid stenosis was 5.8%. Univariate analysis confirmed that PVD (P=0.005), carotid bruit (P=0.003) and diabetes mellitus (P=0.05) were significant risk factors for stenosis. Carotid stenosis was a risk factor for stroke (P=0.03). Prevalence of carotid stenosis was higher in the high-risk group (9.1%) than the low-risk group (1.2%) (P<0.05). All concomitant or staged carotid endarterectomies/CAB (5/205) and all patients who had perioperative strokes (5/205) were in the high-risk group (P=0.01)." ], "LABELS": [ "BACKGROUND", "METHODS", "RESULTS" ], "MESHES": [ "Aged", "Algorithms", "Carotid Stenosis", "Chi-Square Distribution", "Coronary Artery Bypass", "Coronary Artery Disease", "Endarterectomy, Carotid", "Female", "France", "Humans", "Male", "Patient Selection", "Predictive Value of Tests", "Preoperative Care", "Prevalence", "Retrospective Studies", "Risk Assessment", "Risk Factors", "Severity of Illness Index", "Stroke", "Ultrasonography, Doppler, Duplex" ], "YEAR": "2011", "reasoning_required_pred": "yes", "reasoning_free_pred": "maybe", "final_decision": "no", "LONG_ANSWER": "In our cohort, selective screening of patients aged>70 years, with carotid bruit, a history of cerebrovascular disease, diabetes mellitus or PVD would have reduced the screening load by 40%, with trivial impact on surgical management or neurological outcomes." }, "20082356": { "QUESTION": "Should direct mesocolon invasion be included in T4 for the staging of gastric cancer?", "CONTEXTS": [ "One of the sites most frequently invaded by gastric cancer is the mesocolon; however, the UICC does not mention this anatomical site as an adjacent structure involved in gastric cancer. The purpose of this study was to characterize and classify mesocolon invasion from gastric cancer.", "We examined 806 patients who underwent surgery for advanced gastric carcinoma from 1992 to 2007 at the Department of Surgery, Gangnam Severance Hospital, Korea. Among these, patients who showed macroscopically direct invasion into the mesocolon were compared to other patients with advanced gastric cancer.", "The curability, number and extent of nodal metastasis, and the survival of the mesocolon invasion group were significantly worse than these factors in the T3 group. However, the survival of the mesocolon invasion group after curative resection was much better than that of patients who had incurable factors." ], "LABELS": [ "BACKGROUND AND OBJECTIVES", "METHODS", "RESULTS" ], "MESHES": [ "Adult", "Aged", "Female", "Humans", "Lymphatic Metastasis", "Male", "Mesocolon", "Middle Aged", "Neoplasm Invasiveness", "Neoplasm Staging", "Stomach Neoplasms", "Survival Rate" ], "YEAR": "2010", "reasoning_required_pred": "maybe", "reasoning_free_pred": "yes", "final_decision": "maybe", "LONG_ANSWER": "Mesocolon invasion should be included in T4 for the staging of gastric cancer." }, "26606599": { "QUESTION": "Do Surrogates of Injury Severity Influence the Occurrence of Heterotopic Ossification in Fractures of the Acetabulum?", "CONTEXTS": [ "To determine the relationship between injury severity surrogates and other patient factors with the development and severity of heterotopic ossification (HO) following open reduction internal fixation of acetabular fractures treated with a posterior approach.", "Retrospective review.", "Academic level 1 trauma center.", "Two hundred forty-one patients who were treated through a posterior approach with a minimum of 6-month radiographic follow-up were identified from an acetabular fracture database.", "None.", "The occurrence and severity (Brooker Grade III/IV) of HO 6 months postsurgery.", "Length of stay (LOS) in the intensive care unit (ICU), non-ICU LOS>10 days, and HO prophylaxis with external radiation beam therapy (XRT) were significantly associated with the development of HO in a multivariate model [", "1-2 days, odds ratio (OR) = 4.33, 95% confidence interval (CI): 1.03-18.25; 3-6 days, OR = 4.1, 95% CI, 1.27-13.27;>6 days, OR = 11.7, 95% CI, 3.24-42.22; non-ICU LOS>10 days (vs. 0-6 days): OR = 7.6, 95% CI, 2.6-22.25; XRT HO prophylaxis: OR = 0.29, 95% CI, 0.10-0.85]. Other variables evaluated in multivariate modeling not significantly associated with development and severity of HO included age, gender, mechanism of injury, injury severity score, presence of neurologic injury, Letournel fracture type, occurrence of hip dislocation, interval from injury to surgery, operative time, and estimated blood loss." ], "LABELS": [ "OBJECTIVES", "DESIGN", "SETTING", "PARTICIPANTS", "INTERVENTION", "MAIN OUTCOME MEASURES", "RESULTS", "ICU LOS" ], "MESHES": [ "Acetabulum", "Adult", "Aged", "Aged, 80 and over", "Causality", "Comorbidity", "Female", "Fracture Fixation, Internal", "Fractures, Bone", "Humans", "Incidence", "Length of Stay", "Middle Aged", "Ohio", "Open Fracture Reduction", "Ossification, Heterotopic", "Prognosis", "Reproducibility of Results", "Retrospective Studies", "Risk Factors", "Sensitivity and Specificity", "Trauma Severity Indices" ], "YEAR": "2016", "reasoning_required_pred": "maybe", "reasoning_free_pred": "yes", "final_decision": "maybe", "LONG_ANSWER": "Surrogates of injury severity, including days in the ICU and non-ICU hospital LOS>10 days, were associated with the development of HO in our cohort of acetabular fracture patients. Prophylaxis with XRT was significantly protective against the development of HO, and the ability to provide prophylaxis is very likely related to the severity of injury." }, "11340218": { "QUESTION": "Does pretreatment with statins improve clinical outcome after stroke?", "CONTEXTS": [ "In primary and secondary prevention trials, statins have been shown to reduce the risk of stroke. In addition to lipid lowering, statins have a number of antiatherothrombotic and neuroprotective properties. In a preliminary observational study, we explored whether clinical outcome is improved in patients who are on treatment with statins when stroke occurs.", "We conducted a population-based case-referent study of 25- to 74-year-old stroke patients with, for each case of a patient who was on statin treatment at the onset of stroke (n=125), 2 referent patients who were not treated with statins but were matched for age, gender, year of onset, and stroke subtype (n=250).", "The unadjusted odds ratio for early discharge to home (versus late discharge or death) was 1.41 (95% CI 0.91 to 2.17) when patients on statin treatment were compared with referent stroke patients not on statins. Prognostic factors were, in general, more unfavorable among patients on statins. When this was adjusted for in a logistic regression model, the use of statins was a moderately strong but statistically nonsignificant predictor of discharge to home (multiple-adjusted odds ratio 1.42, 95% CI 0.90 to 2.22)." ], "LABELS": [ "BACKGROUND AND PURPOSE", "METHODS", "RESULTS" ], "MESHES": [ "Acute Disease", "Adult", "Age Distribution", "Aged", "Case-Control Studies", "Cerebral Hemorrhage", "Comorbidity", "Diabetes Mellitus", "Female", "Humans", "Hydroxymethylglutaryl-CoA Reductase Inhibitors", "Logistic Models", "Male", "Middle Aged", "Myocardial Infarction", "Odds Ratio", "Pilot Projects", "Prognosis", "Retrospective Studies", "Secondary Prevention", "Sex Distribution", "Simvastatin", "Stroke", "Sweden", "Treatment Outcome" ], "YEAR": "2001", "reasoning_required_pred": "no", "reasoning_free_pred": "maybe", "final_decision": "no", "LONG_ANSWER": "The statistical power of this case-referent study was such that only large beneficial effects of statins in acute stroke could be confirmed. However, the observed trend, together with experimental observations, is interesting enough to warrant a more detailed analysis of the relationship between statins and stroke outcome." }, "25481573": { "QUESTION": "Processing fluency effects: can the content and presentation of participant information sheets influence recruitment and participation for an antenatal intervention?", "CONTEXTS": [ "To assess the extent to which the title and font of participant information sheets (PISs) can influence pregnant women's and trainee midwives' perceptions of an antenatal intervention.", "Pregnant women (n=35) and trainee midwives (n=36) were randomly presented with one of four PISs where the title and font of the PIS had been manipulated to create four experimental conditions (i.e., Double Fluent; Double Awkward; Fluent Title-Awkward Font; Awkward Title-Fluent Font). After reading the PIS, participants rated their perceptions of the intervention (i.e., Attractiveness, Complexity, Expected Risk, Required Effort) using five-point Likert scales.", "A 4\u00d72 factorial multivariate analysis of variance revealed that pregnant women rated the Double Awkward condition as significantly more complex than the Double Fluent (p=.024) and Awkward Title-Fluent Font (p=.021) conditions." ], "LABELS": [ "OBJECTIVE", "METHODS", "RESULTS" ], "MESHES": [ "Adult", "Female", "Humans", "Midwifery", "Multivariate Analysis", "Pamphlets", "Patient Selection", "Pregnancy", "Pregnant Women", "Prenatal Care", "Reading" ], "YEAR": "2015", "reasoning_required_pred": "yes", "reasoning_free_pred": "yes", "final_decision": "yes", "LONG_ANSWER": "Font influenced pregnant women's ratings of intervention complexity." }, "25277731": { "QUESTION": "Sternal fracture in growing children : A rare and often overlooked fracture?", "CONTEXTS": [ "Sternal fractures in childhood are rare. The aim of the study was to investigate the accident mechanism, the detection of radiological and sonographical criteria and consideration of associated injuries.", "In the period from January 2010 to December 2012 all inpatients and outpatients with sternal fractures were recorded according to the documentation.", "A total of 4 children aged 5-14\u00a0years with a sternal fracture were treated in 2\u00a0years, 2\u00a0children were hospitalized for pain management and 2 remained in outpatient care." ], "LABELS": [ "BACKGROUND", "METHOD", "RESULTS" ], "MESHES": [ "Adolescent", "Chest Pain", "Child", "Child, Preschool", "Diagnosis, Differential", "Fractures, Bone", "Humans", "Male", "Rare Diseases", "Sternum" ], "YEAR": "2016", "reasoning_required_pred": "maybe", "reasoning_free_pred": "maybe", "final_decision": "maybe", "LONG_ANSWER": "Isolated sternal fractures in childhood are often due to typical age-related traumatic incidents. Ultrasonography is a useful diagnostic tool for fracture detection and radiography is the method of choice for visualization of the extent of the dislocation." }, "25475395": { "QUESTION": "Is there a correlation between androgens and sexual desire in women?", "CONTEXTS": [ "For women, the correlation between circulating androgens and sexual desire is inconclusive. Substitution with androgens at physiological levels improves sexual function in women who experience decreased sexual desire and androgen deficiency from surgical menopause, pituitary disease, and age-related decline in androgen production in the ovaries. Measuring bioactive testosterone is difficult and new methods have been proposed, including measuring the primary androgen metabolite androsterone glucuronide (ADT-G).AIM: The aim of this study was to investigate a possible correlation between serum levels of androgens and sexual desire in women and whether the level of ADT-G is better correlated than the level of circulating androgens with sexual desire.", "This was a cross-sectional study including 560 healthy women aged 19-65 years divided into three age groups. Correlations were considered to be statistically significant at P<0.05.", "Sexual desire was determined as the total score of the sexual desire domain of the Female Sexual Function Index. Total testosterone (TT), calculated free testosterone (FT), androstenedione, dehydroepiandrosterone sulfate (DHEAS), and ADT-G were analyzed using mass spectrometry.", "Sexual desire correlated overall with FT and androstenedione in the total cohort of women. In a subgroup of women aged 25-44 years with no use of systemic hormonal contraception, sexual desire correlated with TT, FT, androstenedione, and DHEAS. In women aged 45-65 years, androstenedione correlated with sexual desire. No correlations between ADT-G and sexual desire were identified." ], "LABELS": [ "INTRODUCTION", "METHODS", "MAIN OUTCOME MEASURE", "RESULTS" ], "MESHES": [ "Adult", "Age Factors", "Aged", "Androgens", "Androstenedione", "Androstenols", "Androsterone", "Cross-Sectional Studies", "Dehydroepiandrosterone Sulfate", "Female", "Humans", "Libido", "Middle Aged", "Molecular Sequence Data", "Socioeconomic Factors", "Testosterone", "Women's Health" ], "YEAR": "2015", "reasoning_required_pred": "yes", "reasoning_free_pred": "yes", "final_decision": "yes", "LONG_ANSWER": "In the present study, FT and androstenedione were statistically significantly correlated with sexual desire in the total cohort of women. ADT-G did not correlate more strongly than circulating androgens with sexual desire and is therefore not superior to measuring circulating androgens by mass spectrometry." }, "23177368": { "QUESTION": "Does immediate breast reconstruction compromise the delivery of adjuvant chemotherapy?", "CONTEXTS": [ "Immediate breast reconstruction (IBR) provides psychological benefit to many early breast cancer patients however concerns persist regarding its potential impact on chemotherapy delivery. We investigated the association between IBR, complications and adjuvant chemotherapy delivery.", "Retrospective analysis of patients in an academic breast service, who underwent mastectomy, with or without reconstruction, and received adjuvant chemotherapy.", "Comparisons were made between 107 patients who received IBR and 113 who received mastectomy alone. Those receiving IBR were on average younger, with lower body mass index (BMI) and better prognoses. Overall complication rates were comparable (mastectomy alone: 45.1% versus IBR: 35.5%, p = 0.2). There was more return to surgery in the IBR group with 11.5% of tissue expanders requiring removal, whilst more seromas occurred in the mastectomy group. There was no significant difference in the median time to chemotherapy." ], "LABELS": [ "BACKGROUND", "METHOD", "RESULTS" ], "MESHES": [ "Adult", "Aged", "Antineoplastic Agents", "Antineoplastic Combined Chemotherapy Protocols", "Breast Neoplasms", "Chemotherapy, Adjuvant", "Cisplatin", "Cyclophosphamide", "Doxorubicin", "Female", "Fluorouracil", "Humans", "Incidence", "Kaplan-Meier Estimate", "Logistic Models", "Mammaplasty", "Mastectomy", "Methotrexate", "Middle Aged", "Multivariate Analysis", "Postoperative Complications", "Retrospective Studies", "Taxoids", "Time Factors", "Treatment Outcome" ], "YEAR": "2013", "reasoning_required_pred": "no", "reasoning_free_pred": "no", "final_decision": "no", "LONG_ANSWER": "We found no evidence that IBR compromised the delivery of adjuvant chemotherapy, although there was a significant incidence of implant infection." }, "17179167": { "QUESTION": "Human papillomavirus and pterygium. Is the virus a risk factor?", "CONTEXTS": [ "Pterygium is a disease of unknown origin and pathogenesis that might be vision threatening. It is characterised by a wing-like conjunctival overgrowth of the cornea. Several studies have investigated human papillomavirus (HPV) as a risk factor for the development of pterygia, but the results are inconclusive.AIM: To investigate a large sample of pterygia for the presence of HPV in order to clarify the putative association between pterygia and HPV.", "100 specimens of pterygium from Danish patients and 20 normal conjunctival biopsy specimens were investigated for the presence of HPV with PCR technique using beta-globin primers to access the quality of the extracted DNA and the HPV primers MY09/11 and GP5+/6+. HPV-positive specimens underwent subsequent HPV typing with type-specific HPV primers and further investigation with DNA in situ hybridisation (ISH).", "90 of 100 investigated pterygia proved suitable for HPV analysis by PCR. As beta-globin could not be amplified, 10 specimens were excluded from the study. 4 of 90 pterygia harboured HPV. HPV type 6 was identified in all four HPV-positive pterygia. The 20 normal conjunctival biopsy specimens were beta-globin positive and HPV negative. All four pterygia that were HPV type 6 positive were DNA ISH negative." ], "LABELS": [ "BACKGROUND", "METHODS", "RESULTS" ], "MESHES": [ "Adult", "Aged", "Aged, 80 and over", "Cornea", "DNA, Viral", "Female", "Humans", "Male", "Middle Aged", "Papillomaviridae", "Papillomavirus Infections", "Polymerase Chain Reaction", "Pterygium", "Risk Factors", "Tumor Virus Infections" ], "YEAR": "2007", "reasoning_required_pred": "maybe", "reasoning_free_pred": "no", "final_decision": "maybe", "LONG_ANSWER": "The low presence of HPV DNA in pterygia does not support the hypothesis that HPV is involved in the development of pterygia in Denmark." }, "14612308": { "QUESTION": "Can PRISM predict length of PICU stay?", "CONTEXTS": [ "PRISM is claimed to score disease severity which has attributed an impact on length of PICU stay (LOS).", "To determine the impact of PRISM on LOS, and evaluate an Artificial Neural Network's (ANN) performance to estimate LOS from PRISM item patterns.", "Retrospectively we performed correlation and regression analyses on routinely scored PRISM data of all consecutive admissions to our level-III PICU from 1994 to 1999 (n>2000) with individual LOS. In addition, an ANN was trained on the chronologically first 75% of those data (inputs, PRISM items + age + sex; output, LOS). The ANN's performance was tested on the remaining most recent 25% of the data sets.", "The Spearman and Pearson coefficients of correlation between PRISM and LOS were 0.2 (p<0.001) and 0.08 (p = 0.0003), the latter being slightly higher when LOS was logarithmically transformed. Pearson's coefficient of correlation between ANN derived LOS estimate and actual LOS was 0.21 (p<0.001) (LOS logarithmically transformed: 0.34; p<0.001) in the independent validation sample." ], "LABELS": [ "UNLABELLED", "PRIMARY OBJECTIVE", "RESEARCH DESIGN AND METHODS", "MAIN RESULTS" ], "MESHES": [ "Cohort Studies", "Health Services Research", "Humans", "Intensive Care Units, Pediatric", "Length of Stay", "Netherlands", "Neural Networks (Computer)", "Patient Readmission", "Probability", "Retrospective Studies", "Severity of Illness Index" ], "YEAR": "2003", "reasoning_required_pred": "no", "reasoning_free_pred": "no", "final_decision": "no", "LONG_ANSWER": "The ANN with its intrinsic ability to detect non-linear correlation, and to relate specific item patterns to LOS, outperformed linear statistics but was still disappointing in estimating individual LOS. It might be speculated that therapeutic intervention modulates the natural course of the disease thus counteracting both disease severity as initially scored by PRISM, and LOS. This being true, the inverse of the correlation between PRISM (or PRISM based LOS estimate) and LOS might be a candidate indicator of quality of care." }, "27491658": { "QUESTION": "Can predilatation in transcatheter aortic valve implantation be omitted?", "CONTEXTS": [ "The use of a balloon expandable stent valve includes balloon predilatation of the aortic stenosis before valve deployment. The aim of the study was to see whether or not balloon predilatation is necessary in transcatheter aortic valve replacement (TAVI).", "Sixty consecutive TAVI patients were randomized to the standard procedure or to a protocol where balloon predilatation was omitted.", "There were no significant differences between the groups regarding early hemodynamic results or complication rates." ], "LABELS": [ "BACKGROUND", "METHODS", "RESULTS" ], "MESHES": [ "Adult", "Aged", "Aged, 80 and over", "Aortic Valve", "Aortic Valve Stenosis", "Balloon Valvuloplasty", "Cardiac Catheterization", "Dilatation", "Female", "Hemodynamics", "Humans", "Male", "Middle Aged", "Preoperative Care", "Prospective Studies", "Transcatheter Aortic Valve Replacement", "Treatment Outcome" ], "YEAR": "2016", "reasoning_required_pred": "yes", "reasoning_free_pred": "yes", "final_decision": "yes", "LONG_ANSWER": "TAVI can be performed safely without balloon predilatation and with the same early results as achieved with the standard procedure including balloon predilatation. The reduction in the number of pacing periods required may be beneficial for the patient." }, "19822586": { "QUESTION": "Autoerotic asphyxiation: secret pleasure--lethal outcome?", "CONTEXTS": [ "Voluntary asphyxiation among children, preteens, and adolescents by hanging or other means of inducing hypoxia/anoxia to enhance sexual excitement is not uncommon and can lead to unintended death. This study addresses autoerotic asphyxiation (AEA) with the intent of increasing pediatricians' knowledge of the syndrome and awareness of its typical onset among young patients. AEA is characteristically a clandestine and elusive practice. Provided with relevant information, pediatricians can identify the syndrome, demonstrate a willingness to discuss concerns about it, ameliorate distress, and possibly prevent a tragedy.", "A retrospective study was undertaken of published cases both fatal and nonfatal and included personal communications, referenced citations, clinical experience, and theoretical formulations as to causation. Characteristic AEA manifestations, prevalence, age range, methods of inducing hypoxia/anoxia, and gender weighting are presented. All sources were used as a basis for additional considerations of etiology and possibilities for intervention.", "AEA can be conceptualized as a personalized, ritualized, and symbolic biopsychosocial drama. It seems to be a reenactment of intense emotional feeling-states involving an identification and sadomasochistic relationship with a female figure. Inept AEA practitioners can miscalculate the peril of the situation that they have contrived and for numerous reasons lose their gamble with death." ], "LABELS": [ "OBJECTIVE", "METHODS", "RESULTS" ], "MESHES": [ "Adolescent", "Adult", "Asphyxia", "Child", "Female", "Humans", "Male", "Paraphilic Disorders" ], "YEAR": "2009", "reasoning_required_pred": "yes", "reasoning_free_pred": "maybe", "final_decision": "yes", "LONG_ANSWER": "Pediatricians should be alert to the earliest manifestations of AEA. Awareness of choking games among the young and, of those, a subset who eventually progress to potentially fatal AEA is strongly encouraged among all primary care professionals who may be able to interrupt the behavior." }, "27643961": { "QUESTION": "Major depression and alcohol use disorder in adolescence: Does comorbidity lead to poorer outcomes of depression?", "CONTEXTS": [ "Comorbid major depression (MD) and alcohol use disorder (AUD), particularly in adolescents, have been shown to be associated with poorer subsequent MD outcomes.", "Longitudinal data were used to model associations between a four-level classification of MD/AUD during the period 15-18 years (neither; MD-only; AUD-only; comorbid MD/AUD) and MD over the period 18-35 years. These associations were then adjusted for confounding by a series of factors measured in childhood.", "The three disorder groups had rates of adult MD during the period 18-35 years that were significantly (p<.05) higher than that of the group with no disorder. Furthermore, those in the comorbid MD/AUD group had significantly (p<.05) higher rates of adult MD than those in the AUD-only group, and marginally (p<.10) higher rates of adult MD than those in the MD-only group. After adjustment for confounding, the difference in rates of adult MD between the MD-only group and the MD/AUD group were no longer statistically significant. The factors that explained the associations were gender, childhood behavior problems, and exposure to physical and sexual abuse.", "The data were obtained by self-report, and may have been subject to biases." ], "LABELS": [ "BACKGROUND", "METHODS", "RESULTS", "LIMITATIONS" ], "MESHES": [ "Adolescent", "Adult", "Alcohol-Related Disorders", "Comorbidity", "Depressive Disorder, Major", "Female", "Humans", "Longitudinal Studies", "Male", "Prevalence", "Risk Factors", "Sex Offenses", "Young Adult" ], "YEAR": "2016", "reasoning_required_pred": "no", "reasoning_free_pred": "yes", "final_decision": "no", "LONG_ANSWER": "The results of these analyses suggest that marginally higher rates of depression to age 35 amongst the comorbid MD/AUD group were explained by increased exposure to adverse childhood circumstances amongst members of the comorbid group. Adolescent MD/AUD comorbidity is likely to be a risk marker, rather than a causal factor in subsequent MD." }, "23539689": { "QUESTION": "Cold preparation use in young children after FDA warnings: do concerns still exist?", "CONTEXTS": [ "To characterize the use and delivery of cough and cold medicines in children younger than 6 presenting to an inner-city pediatric emergency department (PED) following 2007 FDA warnings.", "A cross-sectional observational study was performed using a convenience sampling of PED patients during the fall of 2010. Caregivers were presented with 6 commonly used cough medicine preparations and were asked to demonstrate if and how they would administer these to their children.", "In all, 65 patients and their caregivers consented and participated in the study. During the demonstration, 82% (53/65) stated that they would treat with cough or cold medicines, and 72% (38/53) incorrectly dosed the medication they desired to give." ], "LABELS": [ "OBJECTIVE", "METHODS", "RESULTS" ], "MESHES": [ "Antitussive Agents", "Caregivers", "Child", "Child, Preschool", "Common Cold", "Cough", "Cross-Sectional Studies", "Drug Packaging", "Female", "Humans", "Infant", "Infant, Newborn", "Male", "Nasal Decongestants", "Nonprescription Drugs", "United States", "United States Food and Drug Administration" ], "YEAR": "2013", "reasoning_required_pred": "yes", "reasoning_free_pred": "maybe", "final_decision": "yes", "LONG_ANSWER": "Despite current recommendations, cough and cold medicines are still used in children younger than 6 years of age. A significant portion of caregivers report that they are still unaware of public warnings, potential side effects, and interactions with other medications." }, "22453060": { "QUESTION": "Does a 4 diagram manual enable laypersons to operate the Laryngeal Mask Supreme\u00ae?", "CONTEXTS": [ "Bystander resuscitation plays an important role in lifesaving cardiopulmonary resuscitation (CPR). A significant reduction in the \"no-flow-time\", quantitatively better chest compressions and an improved quality of ventilation can be demonstrated during CPR using supraglottic airway devices (SADs). Previous studies have demonstrated the ability of inexperienced persons to operate SADs after brief instruction. The aim of this pilot study was to determine whether an instruction manual consisting of four diagrams enables laypersons to operate a Laryngeal Mask Supreme\u00ae (LMAS) in the manikin.", "An instruction manual of four illustrations with speech bubbles displaying the correct use of the LMAS was designed. Laypersons were handed a bag containing a LMAS, a bag mask valve device (BMV), a syringe prefilled with air and the instruction sheet, and were asked to perform and ventilate the manikin as displayed. Time to ventilation was recorded and degree of success evaluated.", "A total of 150 laypersons took part. Overall 145 participants (96.7%) inserted the LMAS in the manikin in the right direction. The device was inserted inverted or twisted in 13 (8.7%) attempts. Eight (5.3%) individuals recognized this and corrected the position. Within the first 2 minutes 119 (79.3%) applicants were able to insert the LMAS and provide tidal volumes greater than 150 ml (estimated dead space). Time to insertion and first ventilation was 83.2 \u00b1 29 s. No significant difference related to previous BLS training (P = 0.85), technical education (P = 0.07) or gender could be demonstrated (P = 0.25)." ], "LABELS": [ "BACKGROUND", "METHODS", "RESULTS" ], "MESHES": [ "Cardiopulmonary Resuscitation", "Female", "First Aid", "Germany", "Humans", "Laryngeal Masks", "Male", "Manikins", "Manuals as Topic", "Pilot Projects", "Program Evaluation", "Resuscitation", "Volunteers", "Young Adult" ], "YEAR": "2012", "reasoning_required_pred": "yes", "reasoning_free_pred": "yes", "final_decision": "yes", "LONG_ANSWER": "In manikin laypersons could insert LMAS in the correct direction after onsite instruction by a simple manual with a high success rate. This indicates some basic procedural understanding and intellectual transfer in principle. Operating errors (n = 91) were frequently not recognized and corrected (n = 77). Improvements in labeling and the quality of instructional photographs may reduce individual error and may optimize understanding." }, "22227642": { "QUESTION": "Can we measure mesopic pupil size with the cobalt blue light slit-lamp biomicroscopy method?", "CONTEXTS": [ "The aim of this work is to assess a previously described slit-lamp biomicroscopy-based method (SLBM) for measuring pupil diameter and compare it to Colvard infrared pupillometry (CIP).", "Two examiners performed three repeated measurements with each instrument in 40 healthy eyes. We determined the agreement of SLBM and CIP, intraobserver and interobserver repeatabilities, and interobserver concordance (kappa) and SLBM ability for detecting pupil sizes over 6.0\u00a0mm.", "The mean (\u00b1standard deviation [SD]) pupil diameter was 5.81 \u00b1 0.70\u00a0mm with SLBM and 6.26 \u00b1 0.68\u00a0mm with CIP (p\u2009=\u20090.01) averaging both examiner's results. Mean differences between the SLBM and CIP were -0.60\u00a0mm and -0.30\u00a0mm for each examiner using the average of the three readings (p\u2009=\u20090.02), and they were very similar using the first reading. Intraobserver reproducibility: the width of the 95% LoA ranged from 1.79 to 2.30\u00a0mm. The ICCs were 0.97 and 0.92 for SLBM, and 0.96 and 0.90 for CIP. Interobserver reproducibility: the width of the LoA ranged from 1.82 to 2.09\u00a0mm. Kappa statistics were 0.39 and 0.49 for the first and mean SLBM readings, respectively, and 0.45 for both the first and mean CIP readings. Sensitivity and specificity of SLBM for detection of pupils larger than 6\u00a0mm ranged from 55.56% to 73.68% and from 76.19% to 95.45%, respectively. The best trade-off between sensitivity and specificity ranged from 5.4\u00a0mm to 6.2\u00a0mm." ], "LABELS": [ "BACKGROUND", "METHODS", "RESULTS" ], "MESHES": [ "Adult", "Dark Adaptation", "Diagnostic Techniques, Ophthalmological", "Female", "Humans", "Iris", "Light", "Male", "Mesopic Vision", "Microscopy", "Middle Aged", "Observer Variation", "Organ Size", "Prospective Studies", "Pupil", "ROC Curve", "Sensitivity and Specificity", "Young Adult" ], "YEAR": "2012", "reasoning_required_pred": "yes", "reasoning_free_pred": "no", "final_decision": "no", "LONG_ANSWER": "Although the SLBM is quite repeatable, it underestimates mesopic pupil size and shows a too wide range of agreement with CIP. SLBM shows low sensitivity in detecting pupils larger than 6\u00a0mm, which may be misleading when planning anterior segment surgery. Previous grading-consensus training strategies may increase interrater reproducibility, and compensation for the systematic underestimation could improve accuracy of the SLBM." }, "12380309": { "QUESTION": "Should circumcision be performed in childhood?", "CONTEXTS": [ "To evaluate prepuce development and retractibility in a group of boys. To point out the value of circumcision and prepucial forced dilation during childhood.", "Prepuce development and retractibility were evaluated in 400 boys ages between 0-16 year old.", "In boys under 1 year prepuce retractibility (assessed only in children who did not undergo forced dilation previously) was type I (non retractile) in 71.5% whereas type V (completely retractile) was only 5.5%. In adolescent boys type I prepuce was observed in 1 boy only, 1.6%, whereas type V was observed in 82.3%. Furthermore, it was observed that at the time of examination for the study 106 boys who had undergone forced dilation at an earlier age had balano-prepucial adhesions again, which demonstrates that prepuce adheres again to glans penis in many boys after a forced dilation is performed. Only 11 boys were considered in need for circumcision, three of them for prepucial orifice stenosis, which prevented normal micturition, causing a prepucial sac, one case due to a constrictive ring below the prepucial edge that would have prevented ulterior retractability, two cases with repetitive balanopostitis, and five cases secondary to xerosol balanitis, accounting for 2.7% of all examined boys." ], "LABELS": [ "OBJECTIVE", "METHODS", "RESULTS" ], "MESHES": [ "Adolescent", "Age Factors", "Child, Preschool", "Circumcision, Male", "Dilatation", "Humans", "Infant", "Male", "Penile Diseases", "Penis", "Phimosis", "Tissue Adhesions", "Unnecessary Procedures" ], "YEAR": "2002", "reasoning_required_pred": "no", "reasoning_free_pred": "no", "final_decision": "no", "LONG_ANSWER": "Incomplete separation between prepuce and glans penis is normal and common among new-borns, progressing until adolescence to spontaneous separation, at which time it is complete in the majority of boys. Accordingly to the criteria we have sustained for years and present study's findings, circumcision has few indications during childhood, as well as forced prepucial dilation." }, "22186742": { "QUESTION": "Does a colonoscopy after acute diverticulitis affect its management?", "CONTEXTS": [ "Medical records of 220 patients hospitalized for acute diverticulitis between June 1, 2002 and September 1, 2009 were reviewed. Acute diverticulitis was diagnosed by clinical criteria and characteristic CT findings. Fifteen patients were excluded either because of questionable CT or hematochezia. Mean age was 61.8\u00b114.3 years (61% females). Clinical parameters, laboratory results, imaging, endoscopic and histopathological reports, and long-term patients' outcome were analyzed.", "One hundred patients (aged 61.8\u00b113.3 y, 54.1% females), underwent an early (4 to 6 wk) colonoscopy after hospital discharge. There were no significant differences in patients' characteristics or survival between those with or without colonoscopy (4\u00b11.9 vs. 4.2\u00b12.1 y, P=0.62). No colonic malignancy was detected. However, in 32 patients (32%) at least 1 polyp was found. Only 1 was determined as an advanced adenoma. No new or different diagnosis was made after colonoscopy." ], "LABELS": [ "METHODS", "RESULTS" ], "MESHES": [ "Acute Disease", "Aged", "Colonoscopy", "Diverticulitis, Colonic", "Female", "Humans", "Male", "Middle Aged", "Retrospective Studies", "Survival Rate", "Tomography, X-Ray Computed" ], "YEAR": "2012", "reasoning_required_pred": "no", "reasoning_free_pred": "no", "final_decision": "no", "LONG_ANSWER": "Our results suggest that colonoscopy does not affect the management of patients with acute diverticulitis nor alter the outcome. The current practice of a routine colonoscopy after acute diverticulitis, diagnosed by typical clinical symptoms and CT needs to be reevaluated." }, "22188074": { "QUESTION": "Do instrumental activities of daily living predict dementia at 1- and 2-year follow-up?", "CONTEXTS": [ "To investigate whether problems in instrumental activities of daily living (IADL) can add to conventionally used clinical measurements in helping to predict a diagnosis of dementia at 1- and 2-year follow-up.", "Multicenter prospective cohort study.", "Memory clinics in Europe.", "Individuals aged 55 and older without dementia.", "IADLs were measured using pooled activities from five informant-based questionnaires. Structural equation modeling (SEM) was used to investigate the relation between IADLs and dementia. Age, sex, education, depression, and cognitive measures (Mini-Mental State Examination and verbal memory) were included in the model.", "Five hundred thirty-one participants had baseline and 1-year follow-up assessments; 69 (13.0%) of these had developed dementia at 1-year follow-up. At 2-year follow-up, 481 participants were seen, of whom 100 (20.8%) had developed dementia. Participants with IADL disabilities at baseline had a higher conversion rate (24.4%) than participants without IADL disabilities (16.7%) (chi-square\u00a0=\u00a04.28, degrees of freedom\u00a0=\u00a01, P\u00a0=\u00a0.04). SEM showed that IADL disability could help predict dementia in addition to the measured variables at 1-year follow-up (odds ratio (OR)\u00a0=\u00a02.20, 95% confidence interval (CI)\u00a0= 1.51-3.13) and 2-year follow-up (OR\u00a0=\u00a02.11, 95% CI\u00a0=\u00a01.33-3.33)." ], "LABELS": [ "OBJECTIVES", "DESIGN", "SETTING", "PARTICIPANTS", "MEASUREMENTS", "RESULTS" ], "MESHES": [ "Activities of Daily Living", "Aged", "Dementia", "Female", "Follow-Up Studies", "Humans", "Male", "Prognosis", "Prospective Studies", "Time Factors" ], "YEAR": "2011", "reasoning_required_pred": "yes", "reasoning_free_pred": "yes", "final_decision": "yes", "LONG_ANSWER": "IADL disability is a useful addition to the diagnostic process in a memory clinic setting, indicating who is at higher risk of developing dementia at 1- and 2-year follow-up." }, "27989969": { "QUESTION": "Does the Simultaneous Use of a Neuroendoscope Influence the Incidence of Ventriculoperitoneal Shunt Infection?", "CONTEXTS": [ "The relationship between the use of an endoscope during ventriculoperitoneal shunt (VPS) procedures and infection remains poorly defined. In this study, we sought to analyze whether the simultaneous use of an endoscope could in fact increase the infection rate associated with VPS procedures.", "This study included 438 VPS procedures, 49 in which an endoscope was used (11.2%) and 389 in which an endoscope was not used (88.8%). The infection rates in these 2 main groups were calculated and compared. Subsequently, 4 new groups were created, composed of patients with a shunt inserted for the first time (groups 1A and 1B) and patients with a shunt reviewed or inserted for a second time (groups 2A and 2B). Groups 1A and 2A comprised patients in whom an endoscope was used simultaneously with VPS surgery, and groups 1B and 2B comprised patients in whom an endoscope was not used. These groups were compared to determine the infection rate.", "The overall infection rate was 18.5%, including 22.4% in the groups in which an endoscope was used and 18% in those in which an endoscope was not used (P\u00a0=\u00a00.449). Groups 1A and 1B and groups 2A and 2B were matched for possible intervening risk factors. The infection rate was 28.6% in group 1A and 16.2% in group 1B (P\u00a0= 0.27), and 20% in group 2A and 19.8% in group 2B (P\u00a0= 0.977)." ], "LABELS": [ "BACKGROUND", "METHODS", "RESULTS" ], "MESHES": [ "Aged", "Aged, 80 and over", "Female", "Follow-Up Studies", "Humans", "Incidence", "Male", "Middle Aged", "Neuroendoscopes", "Retrospective Studies", "Surgical Wound Infection", "Ventriculoperitoneal Shunt" ], "YEAR": "2017", "reasoning_required_pred": "yes", "reasoning_free_pred": "no", "final_decision": "no", "LONG_ANSWER": "In the present study, the use of an endoscope during VPS procedures did not increase the risk of surgical infection." }, "18607272": { "QUESTION": "Body perception: do parents, their children, and their children's physicians perceive body image differently?", "CONTEXTS": [ "To compare children's, parents' and physicians' perceptions of children's body size.", "We administered a structured questionnaire of body size perception using a descriptive Likert scale keyed to body image figures to children ages 12 to 18 years. The same scale was given to parents of children ages 5 to 18 years. The sample consisted of 91 children and their parents being seen in the Pediatric Gastroenterology Clinic for concerns unrelated to overweight. Weight and height of the children were measured, and body mass index (BMI) was calculated. The children's BMI percentiles were categorized as underweight (<15th), normal (15th-85th), overweight (85th-95th), and obese (95th and above). The attending physician independently completed the body image and description scale and indicated the figure that most accurately represented the patient without reference to BMI standards. Accuracy of the patients', parents', and doctors' estimates were statistically compared.", "The sample population consisted of 6.4% underweight, 70.5% normal weight, 7.7% overweight, and 15.4% obese. Forty-four percent of parents underestimated children's body size using word descriptions and 47% underestimated using figures. Forty percent of the children underestimated their own body size using descriptions and 43% underestimated using figures. The physicians in this study had a higher percentage of correct estimates; however, they underestimated 33% of the patients using both word descriptions and figures. Some obese children were not recognized, and several average children were perceived as underweight." ], "LABELS": [ "OBJECTIVES", "PATIENTS AND METHODS", "RESULTS" ], "MESHES": [ "Adolescent", "Adult", "Awareness", "Body Image", "Body Mass Index", "Child", "Child, Preschool", "Female", "Health Knowledge, Attitudes, Practice", "Health Surveys", "Humans", "Male", "Obesity", "Overweight", "Parents", "Perception", "Physicians", "Psychology, Child", "Social Perception", "Surveys and Questionnaires", "Thinness" ], "YEAR": "2008", "reasoning_required_pred": "yes", "reasoning_free_pred": "yes", "final_decision": "yes", "LONG_ANSWER": "Many children underestimated their degree of overweight. Their parents and even their attending physicians shared this misperception. This study demonstrates the need to further educate physicians to recognize obesity and overweight so that they can counsel children and their families." }, "18235194": { "QUESTION": "Is a specialised training of phonological awareness indicated in every preschool child?", "CONTEXTS": [ "In a prospective study 218 preschool children were enrolled (stratified in 2 training programs, one specialized for phonologic awareness in order to prevent dyslexia, the other consisting in training of general perception) during the last year of kindergarten. After finishing the first grade 131 children were compared in their reading and writing abilities.", "In the whole group only a slight difference was found between both training modalities concerning their writing abilities. However, children with a history of hearing loss, actual hearing loss or pathologic middle ear findings profited most from the specialized training program compared to the control in their reading abilities." ], "LABELS": [ "OBJECTIVE AND METHODS", "RESULTS" ], "MESHES": [ "Child Language", "Child, Preschool", "Correction of Hearing Impairment", "Curriculum", "Dyslexia", "Education, Special", "Female", "Germany", "Hearing Disorders", "Humans", "Language Development Disorders", "Linguistics", "Male", "Multilingualism", "Pattern Recognition, Physiological", "Perception", "Phonetics", "Prospective Studies", "Reading", "Speech Disorders", "Vision Disorders", "Writing" ], "YEAR": "2008", "reasoning_required_pred": "maybe", "reasoning_free_pred": "maybe", "final_decision": "maybe", "LONG_ANSWER": "A specialized training program to improve phonologic awareness as a basis for reading and writing in every kindergarten and preschool child seems to be unnecessary. However, children with temporary hearing deficits benefit from such a program. For all other children general perception training may be sufficient." }, "18049437": { "QUESTION": "Is there any relationship between streptococcal infection and multiple sclerosis?", "CONTEXTS": [ "Multiple sclerosis (MS) is an immune-mediated inflammatory demyelinating disease of uncertain etiology. Although the mechanisms of inducting autoimmunity by some of the infectious agents have been investigated, there is not yet enough research on streptococcal infections.MATERIAL/", "To understand the effect of past group A streptococcal infection on MS, antistreptolysin O (ASO) and antideoxyribonuclease B (ADNase B) were measured in 21 patients with relapsing-remitting MS and 21 healthy blood donors by nephelometric assay.", "ADNase B levels in the patients with MS were found to be significantly higher than in the controls (p<0.001); however, ASO levels were similar in both groups." ], "LABELS": [ "BACKGROUND", "METHODS", "RESULTS" ], "MESHES": [ "Adult", "Antibodies, Bacterial", "Antistreptolysin", "Deoxyribonucleases", "Female", "Humans", "Male", "Middle Aged", "Multiple Sclerosis", "Streptococcal Infections", "Streptococcus pyogenes" ], "YEAR": "2007", "reasoning_required_pred": "yes", "reasoning_free_pred": "maybe", "final_decision": "yes", "LONG_ANSWER": "These findings indicate that a relationship between multiple sclerosis and streptococcal infections may exist, but to acquire a better understanding of the role of group A streptococci in the pathogenesis of multiple sclerosis, more studies with animal models are necessary." }, "15597845": { "QUESTION": "Is the combination with 2-methoxyestradiol able to reduce the dosages of chemotherapeutices in the treatment of human ovarian cancer?", "CONTEXTS": [ "The endogenous estradiol metabolite, 2-methoxyestradiol (2ME), has been shown to be a potent inhibitor of cell growth and a strong anti-angiogenic substance. We investigated for the first time whether in vitro combinations of 2ME with various chemotherapeutic compounds may result in an additive inhibitory effect on the proliferation of human ovary cancer cells.", "As a model two different human ovary cancer cell lines were used. All cell lines were incubated with equimolar concentrations of 2ME (0.8-25 microM) and the chemotherapeutics epirubicine, doxorubicine, paclitaxel, docetaxel, carboplatin, vinorelbine, 5-fluorouracil and mafosfamide. Proliferation was measured after four days using the ATP-chemosensitivity test.", "For both ovary cancer cell lines a significant additive effect of 2ME with epirubicine and carboplatin was observed at the lower concentration range of these chemotherapeutic substances." ], "LABELS": [ "PURPOSE OF INVESTIGATION", "METHOD", "RESULTS" ], "MESHES": [ "Antineoplastic Combined Chemotherapy Protocols", "Carboplatin", "Cell Line, Tumor", "Cell Proliferation", "Cyclophosphamide", "Dose-Response Relationship, Drug", "Doxorubicin", "Drug Administration Schedule", "Epirubicin", "Estradiol", "Female", "Fluorouracil", "Humans", "Ovarian Neoplasms", "Paclitaxel", "Taxoids", "Vinblastine" ], "YEAR": "2004", "reasoning_required_pred": "yes", "reasoning_free_pred": "yes", "final_decision": "yes", "LONG_ANSWER": "2ME is able to enhance the antiproliferative activity of certain chemotherapeutics at pharmacological relevant concentrations. This estradiol metabolite is currently in a phase II trial in patients with refractary metastatic breast cancer and the tolerability has been shown to be very good. The combination of 2ME with chemotherapeutics may therefore offer a new clinically relevant treatment regimen for hormone-dependent cancer." }, "24996865": { "QUESTION": "Assessing joint line positions by means of the contralateral knee: a new approach for planning knee revision surgery?", "CONTEXTS": [ "Reconstructing the natural joint line in knee revision surgery improves clinical and functional outcome but may be challenging when both cartilage and bone were removed during previous operations. Assessing joint lines (JLs) by means of bony landmarks is inadvisable because of large variations in human anatomy. Because of the inherent symmetry of the human body, we hypothesised that JLs may be directly assessed by measuring the distances from the bony landmarks to the JL of the contralateral knee by means of radiographic images.", "Using scaled weight-bearing radiographs in anteroposterior view of both knees, two independent observers measured the distances from the fibular head, the medial and lateral epicondyle, and the adductor tubercle to the JL. A two-sided p value of \u22640.05 was considered statistically significant.", "Two hundred knees of 100 patients (50 men and 50 women) were examined. For the fibular head, the mean difference between the treated and the control knee was 0.0 mm with narrow confidence limits ranging from -1.1 to 1.1." ], "LABELS": [ "PURPOSE", "METHODS", "RESULTS" ], "MESHES": [ "Adult", "Aged", "Aged, 80 and over", "Anatomic Landmarks", "Arthroplasty, Replacement, Knee", "Female", "Humans", "Knee Joint", "Male", "Middle Aged", "Preoperative Care", "Radiography", "Reoperation", "Weight-Bearing" ], "YEAR": "2015", "reasoning_required_pred": "yes", "reasoning_free_pred": "yes", "final_decision": "yes", "LONG_ANSWER": "As a new assessment method, we have suggested to assess the JL by means of radiographs of the contralateral knee. The most precise parameter was found to be the distance between the fibular head and the JL. The level of arthritis, age, gender, visibility of the landmarks, and misalignment did not influence measurement accuracy. This parameter is the first tibia-related landmark for assessing the JL, which advantageously corresponds to the tibia-first technique in revision surgery." }, "23361217": { "QUESTION": "Does the type of tibial component affect mechanical alignment in unicompartmental knee replacement?", "CONTEXTS": [ "There are a number of factors responsible for the longevity of unicompartmental knee replacements (UKR). These include the magnitude of postoperative alignment and the type of material used. The effect of component design and material on postoperative alignment, however, has not been explored.", "We retrospectively reviewed 89 patients who underwent UKR with robotic guidance. Patients were divided into two groups, according to whether they had received an all-polyethylene inlay component (Inlay group) or a metal-backed onlay component (Onlay group). We explored the magnitude of mechanical alignment correction obtained in both groups.", "Mean postoperative mechanical alignment was significantly closer to neutral in the Onlay group (mean=2.8\u00b0; 95% CI=2.4\u00b0, 3.2\u00b0) compared to the Inlay group (mean=3.9\u00b0; 95% CI=3.4\u00b0, 4.4\u00b0) (R2=0.65; P=0.003), adjusting for gender, BMI, age, side and preoperative mechanical alignment (Fig. 2). Further exploration revealed that the thickness of the tibial polyethyelene insert had a significant effect on postoperative alignment when added to the model (R2=0.68; P=0.01)." ], "LABELS": [ "PURPOSE", "MATERIALS AND METHODS", "RESULTS" ], "MESHES": [ "Arthroplasty, Replacement, Knee", "Biocompatible Materials", "Biomechanical Phenomena", "Body Mass Index", "Confidence Intervals", "Female", "Humans", "Knee Prosthesis", "Male", "Medical Audit", "Middle Aged", "Polyethylene", "Prosthesis Failure", "Retrospective Studies", "Tibia", "United States" ], "YEAR": "2013", "reasoning_required_pred": "yes", "reasoning_free_pred": "yes", "final_decision": "yes", "LONG_ANSWER": "Patients who received a metal-backed Onlay tibial component obtained better postoperative mechanical alignment compared to those who received all-polyethylene Inlay prostheses. The thicker overall construct of Onlay prostheses appears to be an important determinant of postoperative alignment. Considering their higher survivorship rates and improved postoperative mechanical alignment, Onlay prostheses should be the first option when performing medial UKR." }, "17489316": { "QUESTION": "Is tumour expression of VEGF associated with venous invasion and survival in pT3 renal cell carcinoma?", "CONTEXTS": [ "To determine whether there is a relationship between VEGF expression and renal vein and vena cava invasion in stage pT3 renal cell carcinoma and to evaluate the impact of VEGF expression on survival in pT3 renal cell carcinoma.", "78 patients with a pT3a or pT3b tumour without vena cava invasion or pT3b tumour with vena cava invasion were compared for age, gender, Fuhrman grade and immunohistochemical expression of VEGF. All these variables were submitted to univariate and multivariate analysis to establish their impact on survival.", "Only tumour size appeared to be significantly different between the 3 groups. On univariate analysis, invasion of the perirenal fat, lymph node involvement, distant metastases and VEGF expression were significantly associated with survival (p<0.01). On multivariate analysis, lymph node involvement, distant metastases and VEGF expression (OR 6.07) were identified as independent predictive factors of survival." ], "LABELS": [ "STUDY OBJECTIVE", "MATERIAL AND METHODS", "RESULTS" ], "MESHES": [ "Adult", "Aged", "Aged, 80 and over", "Carcinoma, Renal Cell", "Cause of Death", "Disease-Free Survival", "Female", "Follow-Up Studies", "Humans", "Kidney Neoplasms", "Lymphatic Metastasis", "Male", "Middle Aged", "Neoplasm Invasiveness", "Neoplasm Staging", "Prognosis", "Renal Veins", "Survival Rate", "Vascular Endothelial Growth Factor A", "Vascular Neoplasms", "Vena Cava, Inferior" ], "YEAR": "2007", "reasoning_required_pred": "maybe", "reasoning_free_pred": "maybe", "final_decision": "maybe", "LONG_ANSWER": "Progression of a pT3 tumour into the renal vein and vena cava is not associated with increased tumour expression of VEGF. However, VEGF is an independent prognostic factor in this group of poor prognosis renal tumours." }, "14518645": { "QUESTION": "Injury and poisoning mortality among young men--are there any common factors amenable to prevention?", "CONTEXTS": [ "Deaths from injury and poisoning (suicide, accidents, undetermined deaths, and homicide) are the major cause of death among young men aged 15-39 years in England and Wales and have been increasing in recent years.AIM: To describe common characteristics among young men who die from injury and poisoning.", "We employed a retrospective survey methodology to investigate factors associated with deaths by injury and poisoning among young men aged 15-39 years (n = 268) in Merseyside and Cheshire during 1995. Data were collected from Coroner's inquest notes and General Practitioner records.", "The most common cause of death was poisoning by alcohol and drugs (29.1%, n = 78). A high proportion of cases were unemployed (39.4%, n = 106). Cases were also more likely to be single compared to the general population (74.2% vs 55.5%). Self-destructive behaviour was evident in 77% of deaths (n = 206)." ], "LABELS": [ "BACKGROUND", "DESIGN", "MAIN RESULTS" ], "MESHES": [ "Adolescent", "Adult", "Cause of Death", "Data Collection", "England", "Humans", "Male", "Poisoning", "Retrospective Studies", "Risk Factors", "Self-Injurious Behavior", "Substance-Related Disorders", "Suicide", "Wales", "Wounds and Injuries" ], "YEAR": "2003", "reasoning_required_pred": "yes", "reasoning_free_pred": "yes", "final_decision": "yes", "LONG_ANSWER": "Alcohol and drug use are important contributory factors to injury and poisoning deaths. More research is needed into the effects of unemployment and being single on the health of young men, and to investigate the motivations behind risk taking and self-destructive behaviour." }, "20337202": { "QUESTION": "Continuation of pregnancy after antenatal corticosteroid administration: opportunity for rescue?", "CONTEXTS": [ "To determine the duration of continuing pregnancy after antenatal corticosteroid (AC) administration and to evaluate the potential opportunity for rescue AC.", "Retrospective analysis of women at 24-32 weeks' gestation who received AC at one institution.", "Six hundred ninety-two women received AC. Two hundred forty-seven (35.7%) delivered at>or = 34 weeks' gestation. Three hundred twenty-one (46.4%) delivered within 1 week of AC; 92 of those women (13.3%) delivered within 24 hours. Only 124 (17.9%) remained pregnant 1 week after AC and delivered at<34 weeks. The latter were compared to women delivering>2 week after AC but>or = 34 weeks. More likely to deliver at<34 weeks were those women who received AC for premature preterm rupture of membranes (OR 3.83, 95% CI 2.06-7.17), twins (OR 2.90, 95% CI 1.42-5.95) or before 28 weeks (OR 2.21, 95% CI 1.38-3.52)." ], "LABELS": [ "OBJECTIVE", "STUDY DESIGN", "RESULTS" ], "MESHES": [ "Adrenal Cortex Hormones", "Adult", "Drug Administration Schedule", "Feasibility Studies", "Female", "Fetal Membranes, Premature Rupture", "Gestational Age", "Humans", "Obstetric Labor, Premature", "Pregnancy", "Prenatal Care", "Retrospective Studies", "Treatment Outcome", "Young Adult" ], "YEAR": null, "reasoning_required_pred": "yes", "reasoning_free_pred": "maybe", "final_decision": "maybe", "LONG_ANSWER": "Rescue AC may apply to only 18% of cases, and we identified subsets of more likely candidates." }, "26449554": { "QUESTION": "Does either obesity or OSA severity influence the response of autotitrating CPAP machines in very obese subjects?", "CONTEXTS": [ "The pressures delivered by autotitrating continuous positive airways pressure (CPAP) devices not only treat obstructive sleep apnoea (OSA) effectively but also give potentially interesting physiological information about the forces impinging on the pharynx. In earlier work from this unit, we used correlations between autoCPAP pressure and both OSA severity and obesity, to construct an algorithm to estimate the fixed CPAP pressure a patient required for subsequent clinical use. We wished to discover if these relationships could be reliably extended to a much more obese group.", "We performed a prospective cohort study in an obese population. Measurements of obesity were made, OSA severity was recorded, and the 95th centile autoCPAP pressure was recorded during 1\u00a0week of autoCPAP. Spearman's rank correlation was performed between measurements of obesity and autoCPAP pressure, and between OSA severity and autoCPAP pressure.", "Fifty-four obese individuals (median body mass index (BMI) 43.0\u00a0kg/m(2)), 52\u00a0% of whom had OSA (apnoea-hypopnoea index (AHI)\u2009\u2265\u200915), had a median 95th centile autoCPAP pressure of 11.8\u2009cmH2O. We found no significant correlation between autoCPAP pressure and neck circumference, waist circumference or BMI. There was a moderate correlation between autoCPAP pressure and OSA severity (AHI r\u2009=\u20090.34, p\u2009=\u20090.02; oxygen desaturation index (ODI) r\u2009=\u20090.48, p\u2009<\u20090.001)." ], "LABELS": [ "PURPOSE", "METHODS", "RESULTS" ], "MESHES": [ "Adult", "Body Mass Index", "Cohort Studies", "Continuous Positive Airway Pressure", "Female", "Humans", "Male", "Middle Aged", "Obesity, Morbid", "Pharynx", "Polysomnography", "Prospective Studies", "Sleep Apnea, Obstructive", "Statistics as Topic", "Therapy, Computer-Assisted", "Treatment Outcome" ], "YEAR": "2016", "reasoning_required_pred": "yes", "reasoning_free_pred": "no", "final_decision": "yes", "LONG_ANSWER": "In this population, neither BMI nor neck circumference nor waist circumference is predictive of autoCPAP pressure. Therefore, the previously derived algorithm does not adequately predict the fixed CPAP pressure for subsequent clinical use in these obese individuals. In addition, some subjects without OSA generated high autoCPAP pressures, and thus, the correlation between OSA severity and autoCPAP pressure was only moderate." }, "26215326": { "QUESTION": "Does the clinical presentation of a prior preterm birth predict risk in a subsequent pregnancy?", "CONTEXTS": [ "The objective of the study was to determine whether risk of recurrent preterm birth differs based on the clinical presentation of a prior spontaneous preterm birth (SPTB): advanced cervical dilatation (ACD), preterm premature rupture of membranes (PPROM), or preterm labor (PTL).", "This retrospective cohort study included singleton pregnancies from 2009 to 2014 complicated by a history of prior SPTB. Women were categorized based on the clinical presentation of their prior preterm delivery as having ACD, PPROM, or PTL. Risks for sonographic short cervical length and recurrent SPTB were compared between women based on the clinical presentation of their prior preterm birth. Log-linear regression was used to control for confounders.", "Of 522 patients included in this study, 96 (18.4%) had prior ACD, 246 (47.1%) had prior PPROM, and 180 (34.5%) had prior PTL. Recurrent PTB occurred in 55.2% of patients with a history of ACD compared with 27.2% of those with PPROM and 32.2% with PTL (P = .001). The mean gestational age at delivery was significantly lower for those with a history of ACD (34.0 weeks) compared with women with prior PPROM (37.2 weeks) or PTL (37.0 weeks) (P = .001). The lowest mean cervical length prior to 24 weeks was significantly shorter in patients with a history of advanced cervical dilation when compared with the other clinical presentations." ], "LABELS": [ "OBJECTIVE", "STUDY DESIGN", "RESULTS" ], "MESHES": [ "Adult", "Female", "Fetal Membranes, Premature Rupture", "Gestational Age", "Humans", "Labor Stage, First", "Pregnancy", "Pregnancy Outcome", "Premature Birth", "Recurrence", "Retrospective Studies", "Risk Assessment" ], "YEAR": "2015", "reasoning_required_pred": "yes", "reasoning_free_pred": "yes", "final_decision": "yes", "LONG_ANSWER": "Patients with a history of ACD are at an increased risk of having recurrent preterm birth and cervical shortening in a subsequent pregnancy compared with women with prior preterm birth associated PPROM or PTL." }, "29112560": { "QUESTION": "Is the Distance Worth It?", "CONTEXTS": [ "It is unclear whether traveling long distances to high-volume centers would compensate for travel burden among patients undergoing rectal cancer resection.", "The purpose of this study was to determine whether operative volume outweighs the advantages of being treated locally by comparing the outcomes of patients with rectal cancer treated at local, low-volume centers versus far, high-volume centers.", "This was a population-based study.", "The National Cancer Database was queried for patients with rectal cancer.", "Patients with stage II or III rectal cancer who underwent surgical resection between 2006 and 2012 were included.", "The outcomes of interest were margins, lymph node yield, receipt of neoadjuvant chemoradiation, adjuvant chemotherapy, readmission within 30 days, 30-day and 90-day mortality, and 5-year overall survival.", "A total of 18,605 patients met inclusion criteria; 2067 patients were in the long-distance/high-volume group and 1362 in the short-distance/low-volume group. The median travel distance was 62.6 miles for the long-distance/high-volume group and 2.3 miles for the short-distance/low-volume group. Patients who were younger, white, privately insured, and stage III were more likely to have traveled to a high-volume center. When controlled for patient factors, stage, and hospital factors, patients in the short-distance/low-volume group had lower odds of a lymph node yield \u226512 (OR = 0.51) and neoadjuvant chemoradiation (OR = 0.67) and higher 30-day (OR = 3.38) and 90-day mortality (OR = 2.07) compared with those in the long-distance/high-volume group. The short-distance/low-volume group had a 34% high risk of overall mortality at 5 years compared with the long-distance/high-volume group.", "We lacked data regarding patient and physician decision making and surgeon-specific factors." ], "LABELS": [ "BACKGROUND", "OBJECTIVE", "DESIGN", "SETTINGS", "PATIENTS", "MAIN OUTCOME MEASURES", "RESULTS", "LIMITATIONS" ], "MESHES": [ "Adenocarcinoma", "Adenocarcinoma, Mucinous", "Aged", "Chemoradiotherapy", "Chemotherapy, Adjuvant", "Female", "Health Services Accessibility", "Hospitals, High-Volume", "Humans", "Lymph Node Excision", "Male", "Margins of Excision", "Middle Aged", "Neoplasm Staging", "Patient Readmission", "Rectal Neoplasms", "Risk Factors", "Survival Rate", "Travel", "Treatment Outcome", "United States" ], "YEAR": "2017", "reasoning_required_pred": "yes", "reasoning_free_pred": "yes", "final_decision": "yes", "LONG_ANSWER": "Our results indicate that when controlled for patient, tumor, and hospital factors, patients who traveled a long distance to a high-volume center had improved lymph node yield, neoadjuvant chemoradiation receipt, and 30- and 90-day mortality compared with those who traveled a short distance to a low-volume center. They also had improved 5-year survival. See Video Abstract at http://links.lww.com/DCR/A446." }, "24315783": { "QUESTION": "Aripiprazole: a new risk factor for pathological gambling?", "CONTEXTS": [ "It is commonly accepted that pathological gambling results from the interaction of multiple risk factors. Among these, dopamine replacement therapy (DRT) prescribed for Parkinson disease can be cited. Another dopamine agonist, aripiprazole, could be a new risk factor. We decided to explore this potential adverse drug reaction (ADR).", "Based on a cohort of 166 pathological gamblers starting treatment in our department, data of each of the 8 patients treated by aripiprazole at inclusion were analyzed.", "The patients involved were schizophrenic or bipolar, mostly young men with a history of addictive disorders and regular gambling prior to the prescription of aripiprazole. For each one of them, the causality of aripiprazole was considered, using an algorithm. The probability that pathological gambling is actually due to aripiprazole is \"possible\" in 7 cases out of 8, and \"doubtful\" in one." ], "LABELS": [ "OBJECTIVE", "METHOD", "RESULTS" ], "MESHES": [ "Adult", "Antipsychotic Agents", "Aripiprazole", "Bipolar Disorder", "Dopamine Agonists", "Female", "Gambling", "Humans", "Male", "Middle Aged", "Piperazines", "Quinolones", "Risk Factors", "Schizophrenia", "Young Adult" ], "YEAR": "2014", "reasoning_required_pred": "yes", "reasoning_free_pred": "yes", "final_decision": "yes", "LONG_ANSWER": "Adverse drug reactions were confronted with other already published case reports. Dopamine partial agonist mechanism of aripiprazole could explain the occurrence of pathological gambling." }, "9381529": { "QUESTION": "Immune suppression by lysosomotropic amines and cyclosporine on T-cell responses to minor and major histocompatibility antigens: does synergy exist?", "CONTEXTS": [ "Using murine models, we have shown that the lysosomotropic amine, chloroquine, is effective in the prevention of graft-versus-host disease (GVHD) mediated by donor T cells reactive with recipient minor histocompatibility antigens (MiHCs). Because lysosomotropic amines can suppress major histocompatibility complex (MHC) class II antigen presentation, their mechanism of action is potentially different from current immune suppressant drugs used to control GVHD such as cyclosporine.", "We investigated the use of cyclosporine and the lysosomotropic amines chloroquine and hydroxychloroquine in combination for additive or synergistic immunosuppression on T-cell responses in vitro to MiHC and MHC in mice.", "We found that similar concentrations of chloroquine and hydroxychloroquine suppress the T-cell response to MiHC in mice (C57BL/6 anti-BALB.B) and that lysosomotropic amines in combination with cyclosporine result in synergistic suppression of a proliferative response to MiHC. Similar suppression and synergy appear to be present in an alloreactive response (C57BL/6 anti-BALB/c). Direct inhibition by chloroquine of T-cell proliferative responses induced by anti-CD3epsilon in the absence of antigen-presenting cells is present at higher concentrations than that required to suppress responses to MiHC or MHC. Chloroquine appears to induce decreased T-cell viability at high concentrations. This effect does not appear to be due to decreased T-cell production of interleukin-2 or interferon-gamma. At lower concentrations (<25 microg/ml), chloroquine can also decrease the ability of antigen-presenting cells to stimulate an a C57BL/6 anti-BALB/c T-cell response and can inhibit MHC class II expression after activation with lipopolysaccharide." ], "LABELS": [ "BACKGROUND", "METHODS", "RESULTS" ], "MESHES": [ "Animals", "Cell Survival", "Cells, Cultured", "Chloroquine", "Concanavalin A", "Cyclosporine", "Drug Synergism", "Female", "Graft vs Host Disease", "Histocompatibility Antigens Class II", "Hydroxychloroquine", "Interferon-gamma", "Interleukin-2", "Kinetics", "Lymphocyte Activation", "Lysosomes", "Mice", "Mice, Inbred C57BL", "Mice, Inbred Strains", "Minor Histocompatibility Antigens", "Spleen", "T-Lymphocytes" ], "YEAR": "1997", "reasoning_required_pred": "yes", "reasoning_free_pred": "yes", "final_decision": "yes", "LONG_ANSWER": "Lysosomotropic amines in combination with cyclosporine appear to be synergistic in the suppression of T-cell proliferation to MiHC and MHC. Use of chloroquine in combination with cyclosporine may result in improved control of GVHD." }, "8985020": { "QUESTION": "Does induction chemotherapy have a role in the management of nasopharyngeal carcinoma?", "CONTEXTS": [ "To assess the outcomes of patients with nasopharyngeal carcinoma (NPC) whose treatment was determined by computerized tomography (CT) and/or magnetic resonance imaging staging and to analyze the impact of induction chemotherapy and accelerated fractionated radiotherapy.", "The analysis is based on 122 of 143 previously untreated patients with NPC treated with radiation therapy at The University of Texas M. D. Anderson Cancer Center between 1983 and 1992. Excluded were 4 patients treated with palliative intent, 4 children, 12 patients not staged with CT, and 1 patient who died of a cerebrovascular accident prior to completion of treatment. The stage distribution was as follows: AJCC Stage I-2, Stage II-7, Stage III-12, Stage IV-101; Tl-15, T2-33, T3-22, T4-52; N0-32, N1-10, N2-47, N3-32, Nx-1. Fifty-nine (48%) patients had squamous cell carcinoma; 63 (52%) had lymphoepitheliomas, undifferentiated NPC or poorly differentiated carcinoma, NOS (UNPC). Sixty-seven patients (65 with Stage IV disease) received induction chemotherapy. Fifty-eight patients (24 of whom had induction chemotherapy) were treated with the concomitant boost fractionation schedule. The median follow-up for surviving patients was 57 months.", "The overall actuarial 2- and 5-year survival rates were 78 and 68%, respectively. Forty-nine patients (40%) had disease recurrence. Thirty-three (27%) had local regional failures; 19 at the primary site only, 8 in the neck and 6 in both. Local failure occurred in 31% of patients staged T4 compared to 13% of T1-T3 (p = 0.007). Sixteen patients failed at distant sites alone. Among Stage IV patients the 5-year actuarial rates for patients who did and did not receive induction chemotherapy were as follows: overall survival: 68 vs. 56% (p = 0.02), freedom from relapse: 64 vs. 37% (p = 0.01), and local control: 86 vs. 56% (p = 0.009). The actuarial 5-year distant failure rate in patients with UNPC who were treated with induction chemotherapy and controlled in the primary and neck was 13%. In patients who did not receive chemotherapy, the actuarial 5-year local control rates for patients treated with concomitant boost or conventional fractionation were 66 and 67%, respectively." ], "LABELS": [ "PURPOSE", "METHODS AND MATERIALS", "RESULTS" ], "MESHES": [ "Adolescent", "Adult", "Aged", "Antineoplastic Combined Chemotherapy Protocols", "Combined Modality Therapy", "Humans", "Middle Aged", "Multivariate Analysis", "Nasopharyngeal Neoplasms", "Neoplasm Staging", "Radiotherapy Dosage", "Survival Rate", "Tomography, X-Ray Computed" ], "YEAR": "1996", "reasoning_required_pred": "yes", "reasoning_free_pred": "yes", "final_decision": "yes", "LONG_ANSWER": "While not providing conclusive evidence, this single institution experience suggests that neoadjuvant chemotherapy for Stage IV NPC patients improves both survival and disease control. Recurrence within the irradiated volume was the most prevalent mode of failure and future studies will evaluate regimens to enhance local regional control." }, "26708803": { "QUESTION": "Treatment of contralateral hydrocele in neonatal testicular torsion: Is less more?", "CONTEXTS": [ "Treatment of neonatal testicular torsion has two objectives: salvage of the involved testicle (which is rarely achieved) and preservation of the contralateral gonad. The second goal universally involves contralateral testicular scrotal fixation to prevent the future occurrence of contralateral torsion. However, there is controversy with regards to management of a synchronous contralateral hydrocele. It has been our policy not to address the contralateral hydrocele through an inguinal incision to minimize potential injury to the spermatic cord. Our objective in this study was to determine whether the decision to manage a contralateral hydrocele in cases of neonatal testicular torsion solely through a scrotal approach is safe and effective.", "We reviewed all cases of neonatal testicular torsion occurring at our institution between the years 1999 and 2006. Age at presentation, physical examination, ultrasonographic and intraoperative findings were recorded. Patients were followed after initial surgical intervention to determine the likelihood of developing a subsequent hydrocele or hernia.", "Thirty-seven patients were identified as presenting with neonatal torsion. Age of presentation averaged 3.5 days (range 1-14 days). Left-sided pathology was seen more commonly than the right, with a 25:12 distribution. All torsed testicles were nonviable. Twenty-two patients were noted to have a contralateral hydrocele at presentation. All hydroceles were opened through a scrotal approach at the time of contralateral scrotal fixation. No patient underwent an inguinal exploration to examine for a patent process vaginalis. None of the patients who presented with a hydrocele have developed a clinical hydrocele or hernia after an average 7.5 years (range 4.3-11.2) follow-up." ], "LABELS": [ "OBJECTIVE", "PATIENTS AND METHOD", "RESULTS" ], "MESHES": [ "Humans", "Infant, Newborn", "Male", "Retrospective Studies", "Spermatic Cord Torsion", "Testicular Hydrocele", "Urologic Surgical Procedures, Male" ], "YEAR": "2016", "reasoning_required_pred": "maybe", "reasoning_free_pred": "yes", "final_decision": "maybe", "LONG_ANSWER": "We have demonstrated that approaching a contralateral hydrocele in cases of neonatal testicular torsion solely through a scrotal incision is safe and effective. Inguinal exploration was not performed in our study and our long-term results demonstrate that such an approach would have brought no additional benefit. In avoiding an inguinal approach we did not subject our patients to unnecessary risk of testicular or vasal injury. Contralateral hydrocele is commonly seen in cases of neonatal testicular torsion. In our experience this is a condition of minimal clinical significance and does not warrant formal inguinal exploration for treatment. This conservative management strategy minimizes the potential of contralateral spermatic cord injury in the neonate. The aims of the study were met." }, "22427593": { "QUESTION": "Are normally sighted, visually impaired, and blind pedestrians accurate and reliable at making street crossing decisions?", "CONTEXTS": [ "The purpose of this study is to measure the accuracy and reliability of normally sighted, visually impaired, and blind pedestrians at making street crossing decisions using visual and/or auditory information.", "Using a 5-point rating scale, safety ratings for vehicular gaps of different durations were measured along a two-lane street of one-way traffic without a traffic signal. Safety ratings were collected from 12 normally sighted, 10 visually impaired, and 10 blind subjects for eight different gap times under three sensory conditions: (1) visual plus auditory information, (2) visual information only, and (3) auditory information only. Accuracy and reliability in street crossing decision-making were calculated for each subject under each sensory condition.", "We found that normally sighted and visually impaired pedestrians were accurate and reliable in their street crossing decision-making ability when using either vision plus hearing or vision only (P>0.05). Under the hearing only condition, all subjects were reliable (P>0.05) but inaccurate with their street crossing decisions (P<0.05). Compared to either the normally sighted (P = 0.018) or visually impaired subjects (P = 0.019), blind subjects were the least accurate with their street crossing decisions under the hearing only condition." ], "LABELS": [ "PURPOSE", "METHODS", "RESULTS" ], "MESHES": [ "Adult", "Automobiles", "Blindness", "Contrast Sensitivity", "Humans", "Middle Aged", "Psychomotor Performance", "Reproducibility of Results", "Safety", "Sensory Aids", "Signal Detection, Psychological", "Vision, Low", "Visual Acuity", "Visually Impaired Persons", "Walking" ], "YEAR": "2012", "reasoning_required_pred": "maybe", "reasoning_free_pred": "maybe", "final_decision": "maybe", "LONG_ANSWER": "Our data suggested that visually impaired pedestrians can make accurate and reliable street crossing decisions like those of normally sighted pedestrians. When using auditory information only, all subjects significantly overestimated the vehicular gap time. Our finding that blind pedestrians performed significantly worse than either the normally sighted or visually impaired subjects under the hearing only condition suggested that they may benefit from training to improve their detection ability and/or interpretation of vehicular gap times." }, "11570976": { "QUESTION": "Is it Crohn's disease?", "CONTEXTS": [ "Sulfasalazine is a widely used anti-inflammatory agent in the treatment of inflammatory bowel disease and several rheumatological disorders. Although as many as 20% of treated patients may experience reversible, dose-dependent side effects, less frequent but potentially severe, systemic reactions have also been reported.", "A severe systemic reaction to sulfasalazine developed in a 21-year old female with rheumatoid arthritis characterized by eosinophilia, granulomatous enteritis and myelotoxicity, cholestatic hepatitis, and seizures. The clinical course and management of this patient are presented as well as a review of the incidence and outcome of severe systemic reactions to sulfasalazine." ], "LABELS": [ "BACKGROUND", "CASE PRESENTATION" ], "MESHES": [ "Adult", "Antirheumatic Agents", "Arthritis, Rheumatoid", "Chemical and Drug Induced Liver Injury", "Crohn Disease", "Diagnosis, Differential", "Enteritis", "Eosinophilia", "Female", "Granuloma", "Humans", "Seizures", "Sulfasalazine" ], "YEAR": "2001", "reasoning_required_pred": "maybe", "reasoning_free_pred": "maybe", "final_decision": "maybe", "LONG_ANSWER": "Granulomatous myelotoxicity and enteritis developed in a 21 year old female within 3 weeks of initiating sulfasalazine for rheumatoid arthritis. Following a short course of corticosteroids, the patient had resolution of her cholestatic hepatitis, rash, eosinophilia, and gastrointestinal symptoms with no residual manifestations at 7 months follow-up. Although severe reactions to sulfasalazine are rare and unpredictable, practicing physicians should be aware of unusual clinical presentations of toxicity when prescribing sulfasalazine." }, "19757704": { "QUESTION": "Is Chaalia/Pan Masala harmful for health?", "CONTEXTS": [ "To determine the practices and knowledge of harmful effects regarding use of Chaalia and Pan Masala in three schools of Mahmoodabad and Chanesar Goth, Jamshed Town, Karachi, Pakistan.", "To achieve the objective a cross-sectional design was used in three government schools of Mahmoodabad and Chanesar Goth, Jamshed Town, Karachi. Students of either gender drawn from these schools fulfilling the inclusion and exclusion criteria were interviewed using a pre-coded structured questionnaire. Along with demographic data, questions regarding frequency of Chaalia and Pan Masala use, practices of this habit in friends and family and place of procurement of these substances, were inquired. Knowledge was assessed about harmful effects and its source of information. In addition, practices in relation to that knowledge were assessed.", "A total of 370 students were interviewed over a period of six weeks, of which 205 (55.4%) were boys. The ages of the students were between 10 and 15 years. Thirty one percent of the fathers and 62% of the mothers were uneducated. The frequency of use of any brand of Chaalia was found to be 94% and that of Pan Masala was 73.8%. Eighty five percent of them were regular users. A large majority (88%) procured the substances themselves from near their homes. Ninety five percent of the children had friends with the same habits. Eighty four percent were using the substances in full knowledge of their families. Chaalia was considered harmful for health by 96% and Pan Masala by 60%. Good taste was cited as a reason for continuing the habit by 88.5% of the children and use by friends by 57%. Knowledge about established harmful effects was variable. Knowledge about harmful effects was high in both \"daily\" and \"less than daily users\"." ], "LABELS": [ "OBJECTIVE", "METHODS", "RESULTS" ], "MESHES": [ "Adolescent", "Areca", "Child", "Confidence Intervals", "Cross-Sectional Studies", "Female", "Health Behavior", "Health Education", "Health Knowledge, Attitudes, Practice", "Humans", "Male", "Mastication", "Pakistan", "Psychotropic Drugs", "Risk-Taking", "Schools", "Socioeconomic Factors", "Students", "Substance-Related Disorders", "Surveys and Questionnaires" ], "YEAR": "2009", "reasoning_required_pred": "yes", "reasoning_free_pred": "maybe", "final_decision": "yes", "LONG_ANSWER": "The frequency of habits of Chaalia and Pan Masala chewing, by school children in lower socio-economic areas is extremely high. The probable reasons for this high frequency are taste, the widespread use of these substances by family members and friends, low cost and easy availability." }, "27592038": { "QUESTION": "Does multi-modal cervical physical therapy improve tinnitus in patients with cervicogenic somatic tinnitus?", "CONTEXTS": [ "Tinnitus can be related to many different aetiologies such as hearing loss or a noise trauma, but it can also be related to the somatosensory system of the cervical spine, called cervicogenic somatic tinnitus (CST). Case studies suggest a positive effect of cervical spine treatment on tinnitus complaints in patients with CST, but no experimental studies are available.", "To investigate the effect of a multimodal cervical physical therapy treatment on tinnitus complaints in patients with CST.", "Randomized controlled trial.", "Patients with a combination of severe subjective tinnitus (Tinnitus Functional Index (TFI): 25-90 points) and neck complaints (Neck Bournemouth Questionnaire (NBQ)\u00a0>\u00a014 points).", "All patients received cervical physical therapy for 6 weeks (12 sessions). Patients were randomized in an immediate-start therapy group (n\u00a0=\u00a019) and a 6-week delayed-start therapy group (n\u00a0=\u00a019).", "TFI and NBQ-scores were documented at baseline, after the wait-and-see period in the delayed-start group, after treatment and after 6 weeks follow-up. The Global Perceived Effect (GPE) was documented at all measuring moments, except at baseline.", "In all patients (n\u00a0=\u00a038) TFI and NBQ-scores decreased significantly after treatment (p\u00a0=\u00a00.04 and p\u00a0<\u00a00.001). NBQ-scores remained significantly lower after follow-up (p\u00a0=\u00a00.001). Immediately after treatment, 53% (n\u00a0=\u00a038) experienced substantial improvement of tinnitus. This effect was maintained in 24% of patients after follow-up at six weeks." ], "LABELS": [ "BACKGROUND", "OBJECTIVE", "DESIGN", "PATIENTS", "INTERVENTION", "MEASUREMENTS", "RESULTS" ], "MESHES": [ "Adult", "Aged", "Aged, 80 and over", "Cervical Vertebrae", "Female", "Humans", "Male", "Middle Aged", "Neck Pain", "Physical Therapy Modalities", "Somatosensory Disorders", "Tinnitus", "Treatment Outcome" ], "YEAR": "2016", "reasoning_required_pred": "yes", "reasoning_free_pred": "yes", "final_decision": "yes", "LONG_ANSWER": "Cervical physical therapy can have a positive effect on subjective tinnitus complaints in patients with a combination of tinnitus and neck complaints. Larger studies, using more responsive outcome measures, are however necessary to prove this effect." }, "27044366": { "QUESTION": "Detailed analysis of sputum and systemic inflammation in asthma phenotypes: are paucigranulocytic asthmatics really non-inflammatory?", "CONTEXTS": [ "The technique of induced sputum has allowed to subdivide asthma patients into inflammatory phenotypes according to their level of granulocyte airway infiltration. There are very few studies which looked at detailed sputum and blood cell counts in a large cohort of asthmatics divided into inflammatory phenotypes. The purpose of this study was to analyze sputum cell counts, blood leukocytes and systemic inflammatory markers in these phenotypes, and investigate how those groups compared with healthy subjects.", "We conducted a retrospective cross-sectional study on 833 asthmatics recruited from the University Asthma Clinic of Liege and compared them with 194 healthy subjects. Asthmatics were classified into inflammatory phenotypes.", "The total non-squamous cell count per gram of sputum was greater in mixed granulocytic and neutrophilic phenotypes as compared to eosinophilic, paucigranulocytic asthma and healthy subjects (p\u2009<\u20090.005). Sputum eosinophils (in absolute values and percentages) were increased in all asthma phenotypes including paucigranulocytic asthma, compared to healthy subjects (p\u2009<\u20090.005). Eosinophilic asthma showed higher absolute sputum neutrophil and lymphocyte counts than healthy subjects (p\u2009<\u20090.005), while neutrophilic asthmatics had a particularly low number of sputum macrophages and epithelial cells. All asthma phenotypes showed an increased blood leukocyte count compared to healthy subjects (p\u2009<\u20090.005), with paucigranulocytic asthmatics having also increased absolute blood eosinophils compared to healthy subjects (p\u2009<\u20090.005). Neutrophilic asthma had raised CRP and fibrinogen while eosinophilic asthma only showed raised fibrinogen compared to healthy subjects (p\u2009<\u20090.005)." ], "LABELS": [ "BACKGROUND", "METHODS", "RESULTS" ], "MESHES": [ "Adult", "Aged", "Asthma", "C-Reactive Protein", "Case-Control Studies", "Cross-Sectional Studies", "Eosinophils", "Female", "Fibrinogen", "Granulocytes", "Humans", "Inflammation", "Leukocyte Count", "Lymphocyte Count", "Macrophages", "Male", "Middle Aged", "Neutrophils", "Phenotype", "Retrospective Studies", "Sputum" ], "YEAR": "2016", "reasoning_required_pred": "maybe", "reasoning_free_pred": "no", "final_decision": "maybe", "LONG_ANSWER": "This study demonstrates that a significant eosinophilic inflammation is present across all categories of asthma, and that paucigranulocytic asthma may be seen as a low grade inflammatory disease." }, "9792366": { "QUESTION": "Is HIV/STD control in Jamaica making a difference?", "CONTEXTS": [ "To assess the impact of the comprehensive HIV/STD Control Program established in Jamaica since the late 1980s on the HIV/AIDS epidemic.", "AIDS case reports, HIV testing of blood donors, antenatal clinic attenders (ANC), food service workers, sexually transmitted disease (STD) clinic attenders, female prostitutes, homosexuals and other groups were used to monitor the HIV/AIDS epidemic. Primary and secondary syphilis and cases of congenital syphilis were also monitored. National knowledge, attitude and practice (KAP) surveys were conducted in 1988, 1989, 1992, 1994 and 1996.", "The annual AIDS incidence rate in Jamaica increased only marginally in the past three years from 18.5 per 100000 population to 21.4 in 1997. HIV prevalence in the general population groups tested has been about 1% or less. Among those at high risk, HIV prevalence rates have risen to 6.3% (95% confidence interval 5.0-8.0) in STD clinic attenders, around 10% and 21% in female prostitutes in Kingston and Montego Bay respectively and approximately 30% among homosexuals. Syphilis rates and congenital syphilis cases have declined. The proportion of men aged 15-49 years reporting sex with a non-regular partner declined from 35% in 1994 to 26% in 1996 (P<0.001). Women ever having used condoms increased from 51% in 1988 to 62.5% in 1992 and 73% in 1994 and 1996 (P<0.001), while condom use with a non-regular partner increased from 37% in 1992 to 73% in 1996 (P= 0.006). Condom use among men was high over the period: around 81% had ever used condoms and 77% used them with non-regular partners. Gay men, inner-city adults and adolescents aged 12-14 years all reported increases in condom use while condom sales and distribution increased from around 2 million in 1985 to 10 million in 1995." ], "LABELS": [ "OBJECTIVE", "METHODS", "RESULTS" ], "MESHES": [ "Adolescent", "Adult", "Child", "Condoms", "Confidence Intervals", "Female", "HIV Infections", "HIV Seroprevalence", "Health Knowledge, Attitudes, Practice", "Humans", "Incidence", "Jamaica", "Male", "Middle Aged", "Sexually Transmitted Diseases" ], "YEAR": "1998", "reasoning_required_pred": "yes", "reasoning_free_pred": "yes", "final_decision": "yes", "LONG_ANSWER": "HIV/STD control measures appear to have slowed the HIV/AIDS epidemic in Jamaica, however a significant minority of persons continue to have unprotected sex in high risk situations." }, "17562682": { "QUESTION": "Is Panton-Valentine leucocidin associated with the pathogenesis of Staphylococcus aureus bacteraemia in the UK?", "CONTEXTS": [ "The morbidity and mortality associated with Panton-Valentine leucocidin (PVL)-positive Staphylococcus aureus suggest that this toxin is a key marker of disease severity. Nevertheless, the importance of PVL in the pathogenesis of primary bacteraemia caused by S. aureus is uncertain. We have determined the prevalence of PVL-encoding genes among isolates of S. aureus from bacteraemic patients.", "Consecutive bacteraemia isolates of S. aureus (n=244) from patients hospitalized in 25 centres in the UK and Ireland during 2005 were screened for PVL and mecA genes. PVL-positive isolates were characterized by toxin gene profiling, PFGE, spa-typing and MIC determinations for a range of antimicrobials.", "Four out of 244 isolates (1.6%) were PVL-positive and susceptible to oxacillin [methicillin-susceptible S. aureus (MSSA)]. Eighty-eight out of 244 (36%) were oxacillin-resistant (methicillin-resistant S. aureus), but none was PVL-positive. The four patients (two males: 30 and 33 years; two females: 62 and 80 years) had infection foci of: skin and soft tissue, unknown, indwelling line, and surgical site, and were located at one centre in Wales, one in England and two in Ireland. One of four PVL-positive isolates was resistant to penicillin and fusidic acid, the remainder were susceptible to all antibiotics tested. Genotypic analyses showed that the four isolates represented three distinct strains; the two isolates from Ireland were related." ], "LABELS": [ "BACKGROUND", "METHODS", "RESULTS" ], "MESHES": [ "Adolescent", "Adult", "Aged", "Aged, 80 and over", "Anti-Bacterial Agents", "Bacteremia", "Bacterial Proteins", "Bacterial Toxins", "Child", "Child, Preschool", "Exotoxins", "Female", "Fusidic Acid", "Humans", "Infant", "Ireland", "Leukocidins", "Male", "Methicillin Resistance", "Microbial Sensitivity Tests", "Middle Aged", "Oxacillin", "Penicillin Resistance", "Penicillin-Binding Proteins", "Staphylococcal Infections", "Staphylococcus aureus", "United Kingdom" ], "YEAR": "2007", "reasoning_required_pred": "maybe", "reasoning_free_pred": "no", "final_decision": "maybe", "LONG_ANSWER": "We found that 1.6% of S. aureus (all MSSA) from bacteraemic patients were PVL-positive. This low incidence suggests that PVL-positive S. aureus are of no particular significance as causative agents of S. aureus bacteraemia." }, "15800018": { "QUESTION": "Are even impaired fasting blood glucose levels preoperatively associated with increased mortality after CABG surgery?", "CONTEXTS": [ "Impaired fasting glucose (IFG) below the diagnostic threshold for diabetes mellitus (DM) is associated with macrovascular pathology and increased mortality after percutaneous coronary interventions. The study goal was to determine whether pre-operative fasting blood glucose (fB-glu) is associated with an increased mortality after coronary artery bypass grafting (CABG).", "During 2001-03, 1895 patients underwent primary CABG [clinical DM (CDM) in 440/1895; complete data on fB-glu for n=1375/1455]. Using pre-operative fB-glu, non-diabetics were categorized as having normal fB-glu (<5.6 mmol/L), IFG (5.6or =6.1 mmol/L). fB-glu was normal in 59%. The relative risks of 30 day and 1 year mortality compared with patients with normal fB-glu was 1.7 [95% confidence interval (CI): 0.5-5.5] and 2.9 (CI: 0.8-11.2) with IFG, 2.8 (CI: 1.1-7.2) and 1.9 (CI: 0.5-6.3) with SDM vs. 1.8 (CI: 0.8-4.0) and 1.6 (CI: 0.6-4.3) if CDM, respectively. The receiver operator characteristic area for the continuous variable fB-glu and 1 year mortality was 0.65 (P=0.002)." ], "LABELS": [ "AIMS", "METHODS AND RESULTS" ], "MESHES": [ "Aged", "Blood Glucose", "Coronary Artery Bypass", "Diabetes Mellitus", "Diabetic Angiopathies", "Fasting", "Female", "Humans", "Male", "Multivariate Analysis", "Myocardial Infarction", "Preoperative Care", "ROC Curve", "Retrospective Studies", "Risk Factors", "Survival Analysis", "Sweden" ], "YEAR": "2005", "reasoning_required_pred": "yes", "reasoning_free_pred": "yes", "final_decision": "yes", "LONG_ANSWER": "The elevated risk of death after CABG surgery known previously to be associated with CDM seems also to be shared by a group of similar size that includes patients with IFG and undiagnosed DM." }, "14978612": { "QUESTION": "Does positron emission tomography change management in primary rectal cancer?", "CONTEXTS": [ "The influence of positron emission tomography in the management of recurrent rectal cancer is well established but its role in primary rectal cancer remains uncertain. This study therefore prospectively assesses the impact of position emission tomography scanning on the management of primary rectal cancer.", "Forty-six patients with advanced primary rectal cancer referred for consideration of adjuvant preoperative therapy underwent position emission tomography scanning. The referring physicians prospectively recorded each patient's stage following conventional imaging and the proposed treatment plan prior to position emission tomography scanning. This was then compared with subsequent stage and actual management implemented, and the appropriateness of position emission tomography-induced changes was noted by subsequent clinical follow-up.", "The surgical management of 36 of 46 patients (78 percent) was unchanged as a result of position emission tomography, even though position emission tomography upstaged disease in 3 of 36 cases (8 percent) and downstaged disease in 5 of 36 cases (14 percent). In 8 of 46 cases (17 percent), management was altered because of the position emission tomography scan findings, including 6 cases (13 percent) in which surgery was cancelled and 2 other cases (4 percent) in which the radiotherapy field was changed. Where available, follow-up confirmed the appropriateness of position emission tomography-induced management change in each case. Two patients had a change in therapy independent of the position emission tomography scan due to clinical circumstances. Overall tumor stage was changed following position emission tomography in 18 of 46 patients (39 percent)." ], "LABELS": [ "PURPOSE", "METHODS", "RESULTS" ], "MESHES": [ "Adult", "Aged", "Aged, 80 and over", "Carcinoma", "Female", "Humans", "Male", "Middle Aged", "Neoplasm Recurrence, Local", "Neoplasm Staging", "Patient Care Planning", "Predictive Value of Tests", "Prognosis", "Prospective Studies", "Radiotherapy, Adjuvant", "Rectal Neoplasms", "Sensitivity and Specificity", "Tomography, Emission-Computed" ], "YEAR": "2004", "reasoning_required_pred": "yes", "reasoning_free_pred": "yes", "final_decision": "yes", "LONG_ANSWER": "Position emission tomography scanning appears to accurately change the stage or appropriately alter the therapy of almost a third of patients with advanced primary rectal cancer. In view of this, we suggest that position emission tomography scanning be considered part of standard workup for such patients, particularly if neoadjuvant chemoradiation is being considered as part of primary management." }, "18843057": { "QUESTION": "Can you deliver accurate tidal volume by manual resuscitator?", "CONTEXTS": [ "One of the problems with manual resuscitators is the difficulty in achieving accurate volume delivery. The volume delivered to the patient varies by the physical characteristics of the person and method. This study was designed to compare tidal volumes delivered by the squeezing method, physical characteristics and education and practice levels.", "114 individuals trained in basic life support and bag-valve-mask ventilation participated in this study. Individual characteristics were obtained by the observer and the education and practice level were described by the subjects. Ventilation was delivered with a manual resuscitator connected to a microspirometer and volumes were measured. Subjects completed three procedures: one-handed, two-handed and two-handed half-compression.", "The mean (standard deviation) volumes for the one-handed method were 592.84 ml (SD 117.39), two-handed 644.24 ml (SD 144.7) and two-handed half-compression 458.31 ml (SD 120.91) (p<0.01). Tidal volume delivered by two hands was significantly greater than that delivered by one hand (r = 0.398, p<0.01). The physical aspects including hand size, volume and grip power had no correlation with the volume delivered. There were slight increases in tidal volume with education and practice, but correlation was weak (r = 0.213, r = 0.281, r = 0.131, p<0.01)." ], "LABELS": [ "OBJECTIVES", "METHODS", "RESULTS" ], "MESHES": [ "Adult", "Clinical Competence", "Emergency Medical Services", "Female", "Hand", "Hand Strength", "Health Personnel", "Humans", "Male", "Respiration, Artificial", "Tidal Volume", "Ventilators, Mechanical" ], "YEAR": "2008", "reasoning_required_pred": "no", "reasoning_free_pred": "no", "final_decision": "no", "LONG_ANSWER": "The tidal volume delivered by a manual resuscitator shows large variations. There were significant differences in the volume delivered by compression methods, but physical characteristics are not a predictor of tidal volume delivery. The manual resuscitator is not a suitable device for accurate ventilation." }, "23076787": { "QUESTION": "Can increases in the cigarette tax rate be linked to cigarette retail prices?", "CONTEXTS": [ "To explain China's cigarette pricing mechanism and the role of the Chinese State Tobacco Monopoly Administration (STMA) on cigarette pricing and taxation.", "Published government tobacco tax documentation and statistics published by the Chinese STMA are used to analyse the interrelations among industry profits, taxes and retail price of cigarettes in China.", "The 2009 excise tax increase on cigarettes in China has not translated into higher retail prices because the Chinese STMA used its policy authority to ensure that retail cigarette prices did not change. The government tax increase is being collected at both the producer and wholesale levels. As a result, the 2009 excise tax increase in China has resulted in higher tax revenue for the government and lower profits for the tobacco industry, with no increase in the retail price of cigarettes for consumers." ], "LABELS": [ "OBJECTIVE", "METHODS", "RESULTS" ], "MESHES": [ "China", "Commerce", "Government Regulation", "Humans", "Taxes", "Tobacco Industry", "Tobacco Products" ], "YEAR": "2012", "reasoning_required_pred": "no", "reasoning_free_pred": "no", "final_decision": "no", "LONG_ANSWER": "Numerous studies have found that taxation is one of the most effective policy instruments for tobacco control. However, these findings come from countries that have market economies where market forces determine prices and influence how cigarette taxes are passed to the consumers in retail prices. China's tobacco industry is not a market economy; therefore, non-market forces and the current Chinese tobacco monopoly system determine cigarette prices. The result is that tax increases do not necessarily get passed on to the retail price." }, "9199905": { "QUESTION": "Vertical lines in distal esophageal mucosa (VLEM): a true endoscopic manifestation of esophagitis in children?", "CONTEXTS": [ "We observed an endoscopic abnormally in a group of children with histological esophagitis. We termed this finding \"vertical lines in esophageal mucosa\" (VLEM). We examined the relationship between the presence of VLEM and significant histologic changes in esophageal mucosal biopsies.", "Between January 1, 1992, and August 31, 1994, the senior author (JFF) performed 255 esophageal biopsies. The procedure reports, available endoscopic photographs, and histology reports were reviewed to establish the endoscopic and histologic appearance of the esophageal mucosa. Intraepithelial cells were counted in a blind review of 42 randomly selected biopsies.", "The esophageal mucosa had a normal appearance on 160 endoscopic studies (Group 1) and VLEM were the only mucosal abnormalities in 41 endoscopies (Group 2). Histology was normal in 92 of 160 biopsies (57.5%) from Group 1, and 1 of 41 biopsies (2.4%) from Group 2. Most patients in Group 2 had eosinophilic esophagitis (34 of 41, 83%, specificity 0.85, sensitivity 0.5, p>0.001) which was of moderate to severe intensity (31 of 34, 91.2%, specificity 0.88, sensitivity 0.73, p<0.001)." ], "LABELS": [ "BACKGROUND", "METHODS", "RESULTS" ], "MESHES": [ "Adolescent", "Biopsy", "Cell Count", "Child", "Child, Preschool", "Endosonography", "Esophagitis", "Esophagoscopy", "Female", "Humans", "Infant", "Infant, Newborn", "Intestinal Mucosa", "Male", "Observer Variation", "Random Allocation", "Retrospective Studies", "Sensitivity and Specificity" ], "YEAR": "1997", "reasoning_required_pred": "yes", "reasoning_free_pred": "yes", "final_decision": "yes", "LONG_ANSWER": "Histology usually demonstrated moderate to severe inflammation when VLEM were present. VLEM may be a highly specific endoscopic feature of esophagitis in children." }, "23999452": { "QUESTION": "Does hypoglycaemia increase the risk of cardiovascular events?", "CONTEXTS": [ "Hypoglycaemia caused by glucose-lowering therapy has been linked to cardiovascular (CV) events. The ORIGIN trial provides an opportunity to further assess this relationship.", "A total of 12 537 participants with dysglycaemia and high CV-risk were randomized to basal insulin glargine titrated to a fasting glucose of \u2264 5.3 mmol/L (95 mg/dL) or standard glycaemic care. Non-severe hypoglycaemia was defined as symptoms confirmed by glucose \u2264 54 mg/dL and severe hypoglycaemia as a requirement for assistance or glucose \u2264 36 mg/dL. Outcomes were: (i) the composite of CV death, non-fatal myocardial infarction or stroke; (ii) mortality; (iii) CV mortality; and (iv) arrhythmic death. Hazards were estimated before and after adjustment for a hypoglycaemia propensity score. During a median of 6.2 years (IQR: 5.8-6.7), non-severe hypoglycaemic episodes occurred in 41.7 and 14.4% glargine and standard group participants, respectively, while severe episodes occurred in 5.7 and 1.8%, respectively. Non-severe hypoglycaemia was not associated with any outcome following adjustment. Conversely, severe hypoglycaemia was associated with a greater risk for the primary outcome (HR: 1.58; 95% CI: 1.24-2.02, P<0.001), mortality (HR: 1.74; 95% CI: 1.39-2.19, P<0.001), CV death (HR: 1.71; 95% CI: 1.27-2.30, P<0.001) and arrhythmic death (HR: 1.77; 95% CI: 1.17-2.67, P = 0.007). Similar findings were noted for severe nocturnal hypoglycaemia for the primary outcome and mortality. The severe hypoglycaemia hazard for all four outcomes was higher with standard care than with insulin glargine." ], "LABELS": [ "AIMS", "METHODS AND RESULTS" ], "MESHES": [ "Arrhythmias, Cardiac", "Diabetes Mellitus, Type 2", "Diabetic Angiopathies", "Female", "Glycated Hemoglobin A", "Humans", "Hypoglycemia", "Hypoglycemic Agents", "Insulin Glargine", "Insulin, Long-Acting", "Male", "Middle Aged", "Myocardial Infarction", "Prognosis", "Risk Factors", "Stroke" ], "YEAR": "2013", "reasoning_required_pred": "yes", "reasoning_free_pred": "maybe", "final_decision": "yes", "LONG_ANSWER": "Severe hypoglycaemia is associated with an increased risk for CV outcomes in people at high CV risk and dysglycaemia. Although allocation to insulin glargine vs. standard care was associated with an increased risk of severe and non-severe hypoglycaemia, the relative risk of CV outcomes with hypoglycaemia was lower with insulin glargine-based glucose-lowering therapy than with the standard glycaemic control. Trial Registration (ORIGIN ClinicalTrials.gov number NCT00069784)." }, "22534881": { "QUESTION": "Does the radiographic transition zone correlate with the level of aganglionosis on the specimen in Hirschsprung's disease?", "CONTEXTS": [ "The correlation between radiographic transition zone on contrast enema in Hirschsprung's disease and the total length of aganglionosis is known to be inaccurate. The aim of our study was to analyse this correlation more precisely to improve preoperative planning of the corrective surgery.", "From 1998 to 2009, 79 patients were operated on for Hirschsprung's disease. All available preoperative contrast enemas (n = 61) had been single blind reviewed by the same radiologist who defined the radiographic transition zone when present in vertebral level. Four groups were determined (rectal, rectosigmoid, long segment, and absence of transition zone) and by Kappa coefficient of agreement correlated to the length of aganglionosis in the pathological report.", "Radiological findings were concordant with the specimen in pathology in 8 cases of 19 in rectal form (42 %), in 20 cases of 35 in rectosigmoid form (57 %), in all 6 cases of long-segment form (100 %), in the 2 cases of total colonic form (100 %) with a global agreement of 58.1 %, \u03ba = 0.39 CI [0.24; 0.57]." ], "LABELS": [ "PURPOSE", "METHODS", "RESULTS" ], "MESHES": [ "Female", "Hirschsprung Disease", "Humans", "Infant", "Infant, Newborn", "Male", "Radiography", "Retrospective Studies" ], "YEAR": "2012", "reasoning_required_pred": "yes", "reasoning_free_pred": "no", "final_decision": "no", "LONG_ANSWER": "Correlation between level of radiographic transition zone on contrast enema and length of aganglionosis remains low. Systematic preoperative biopsy by coelioscopy or ombilical incision is mandatory." }, "24785562": { "QUESTION": "Is dexamethasone an effective alternative to oral prednisone in the treatment of pediatric asthma exacerbations?", "CONTEXTS": [ "A short course of systemic corticosteroids is an important therapy in the treatment of pediatric asthma exacerbations. Although a 5-day course of oral prednisone or prednisolone has become the most commonly used regimen, dexamethasone has also been used for a shorter duration (1-2 days) with potential for improvement in compliance and palatability. We reviewed the literature to determine if there is sufficient evidence that dexamethasone can be used as an effective alternative in the treatment of pediatric asthma exacerbations in the inpatient setting.", "A Medline search was conducted on the use of dexamethasone in the treatment of asthma exacerbations in children. The studies selected were clinical trials comparing the efficacy of dexamethasone with prednisone. Meta-analysis was performed examining physician revisitation rates and symptomatic return to baseline.", "Six completed pediatric clinical trials met the inclusion criteria. All of the pediatric trials found that prednisone is not superior to dexamethasone in treating mild to moderate asthma exacerbations. Meta-analysis demonstrated homogeneity between the dexamethasone and prednisone groups when examining symptomatic return to baseline and unplanned physician revisits after the initial emergency department encounter. Some studies found potential additional benefits of dexamethasone, including improved compliance and less vomiting." ], "LABELS": [ "BACKGROUND", "METHODS", "RESULTS" ], "MESHES": [ "Administration, Oral", "Anti-Inflammatory Agents", "Asthma", "Child", "Dexamethasone", "Disease Progression", "Glucocorticoids", "Humans", "Prednisone", "Treatment Outcome" ], "YEAR": "2014", "reasoning_required_pred": "yes", "reasoning_free_pred": "yes", "final_decision": "yes", "LONG_ANSWER": "The current literature suggests that dexamethasone can be used as an effective alternative to prednisone in the treatment of mild to moderate acute asthma exacerbations in children, with the added benefits of improved compliance, palatability, and cost. However, more research is needed to examine the role of dexamethasone in hospitalized children." }, "22564465": { "QUESTION": "Mammographic screening in Sami speaking municipalities and a control group. Are early outcome measures influenced by ethnicity?", "CONTEXTS": [ "Female citizens of Sami (the indigenous people of Norway) municipalities in northern Norway have a low risk of breast cancer. The objective of this study was to describe the attendance rate and outcome of the Norwegian Breast Cancer Screening Program (NBCSP) in the Sami-speaking municipalities and a control group.", "A retrospective registry-based study.", "The 8 municipalities included in the administration area of the Sami language law (Sami) were matched with a control group of 11 municipalities (non-Sami). Population data were accessed from Statistics Norway. Data regarding invitations and outcome in the NBCSP during the period 2001-2010 was derived from the Cancer Registry of Norway (CRN). The NBCSP targets women aged 50-69 years. Rates and percentages were compared using chi-square test with a p-value<0.05 as statistical significant.", "The attendance rate in the NBCSP was 78% in the Sami and 75% in the non-Sami population (p<0.01). The recall rates were 2.4 and 3.3% in the Sami and non-Sami population, respectively (p<0.01). The rate of invasive screen detected cancer was not significantly lower in the Sami group (p=0.14). The percentage of all breast cancers detected in the NBCSP among the Sami (67%) was lower compared with the non-Sami population (86%, p=0.06)." ], "LABELS": [ "OBJECTIVES", "STUDY DESIGN", "METHODS", "RESULTS" ], "MESHES": [ "Aged", "Breast Neoplasms", "Cohort Studies", "Ethnic Groups", "Female", "Humans", "Mammography", "Middle Aged", "Norway", "Population Groups", "Registries", "Retrospective Studies" ], "YEAR": "2012", "reasoning_required_pred": "yes", "reasoning_free_pred": "yes", "final_decision": "yes", "LONG_ANSWER": "Despite a lower risk of breast cancer, the Sami attended the NBCSP more frequently than the control group. The recall and cancer detection rate was lower among the Sami compared with the non-Sami group." }, "27456836": { "QUESTION": "Do Electrochemiluminescence Assays Improve Prediction of Time to Type 1 Diabetes in Autoantibody-Positive TrialNet Subjects?", "CONTEXTS": [ "To explore whether electrochemiluminescence (ECL) assays can help improve prediction of time to type 1 diabetes in the TrialNet autoantibody-positive population.", "TrialNet subjects who were positive for one or more autoantibodies (microinsulin autoantibody, GAD65 autoantibody [GADA], IA-2A, and ZnT8A) with available ECL-insulin autoantibody (IAA) and ECL-GADA data at their initial visit were analyzed; after a median follow-up of 24 months, 177 of these 1,287 subjects developed diabetes.", "Univariate analyses showed that autoantibodies by radioimmunoassays (RIAs), ECL-IAA, ECL-GADA, age, sex, number of positive autoantibodies, presence of HLA DR3/4-DQ8 genotype, HbA1c, and oral glucose tolerance test (OGTT) measurements were all significantly associated with progression to diabetes. Subjects who were ECL positive had a risk of progression to diabetes within 6 years of 58% compared with 5% for the ECL-negative subjects (P<0.0001). Multivariate Cox proportional hazards models were compared, with the base model including age, sex, OGTT measurements, and number of positive autoantibodies by RIAs. The model with positivity for ECL-GADA and/or ECL-IAA was the best, and factors that remained significantly associated with time to diabetes were area under the curve (AUC) C-peptide, fasting C-peptide, AUC glucose, number of positive autoantibodies by RIAs, and ECL positivity. Adding ECL to the Diabetes Prevention Trial risk score (DPTRS) improved the receiver operating characteristic curves with AUC of 0.83 (P<0.0001)." ], "LABELS": [ "OBJECTIVE", "RESEARCH DESIGN AND METHODS", "RESULTS" ], "MESHES": [ "Adolescent", "Adult", "Autoantibodies", "Blood Glucose", "C-Peptide", "Child", "Diabetes Mellitus, Type 1", "Disease Progression", "Female", "Glycated Hemoglobin A", "Humans", "Insulin Antibodies", "Longitudinal Studies", "Luminescence", "Male", "Proportional Hazards Models", "Prospective Studies", "Risk Factors", "Time Factors", "Young Adult" ], "YEAR": "2016", "reasoning_required_pred": "yes", "reasoning_free_pred": "yes", "final_decision": "yes", "LONG_ANSWER": "ECL assays improved the ability to predict time to diabetes in these autoantibody-positive relatives at risk for developing diabetes. These findings might be helpful in the design and eligibility criteria for prevention trials in the future." }, "27287237": { "QUESTION": "Assessment of appropriate antimicrobial prescribing: do experts agree?", "CONTEXTS": [ "Little is known about the validity and reliability of expert assessments of the quality of antimicrobial prescribing, despite their importance in antimicrobial stewardship. We investigated how infectious disease doctors' assessments compared with a reference standard (modal expert opinion) and with the assessments of their colleagues.", "Twenty-four doctors specialized in infectious diseases or clinical microbiology (16 specialists and 8 residents) from five hospitals were asked to assess the appropriateness of antimicrobial agents prescribed for a broad spectrum of indications in 56 paper cases. They were instructed how to handle guideline applicability and deviations. We created a reference standard of antimicrobial appropriateness using the modal assessment of 16 specialists. We calculated criterion validity and interrater and intrarater overall and specific agreement with an index expert (senior infectious disease physician) and analysed the influence of doctor characteristics on validity.", "Specialists agreed with the reference standard in 80% of cases (range 75%-86%), with a sensitivity and specificity of 75% and 84%, respectively. This did not differ by clinical specialty, hospital or years of experience, and residents had similar results. Specialists agreed with the index expert in 76% of cases and the index expert agreed with his previous assessments in 71% of cases." ], "LABELS": [ "OBJECTIVES", "METHODS", "RESULTS" ], "MESHES": [ "Anti-Bacterial Agents", "Communicable Diseases", "Drug Prescriptions", "Hospitals", "Humans", "Physicians", "Practice Patterns, Physicians'" ], "YEAR": "2016", "reasoning_required_pred": "yes", "reasoning_free_pred": "yes", "final_decision": "yes", "LONG_ANSWER": "Doctors specialized in infectious diseases and clinical microbiology assess the appropriateness of antimicrobials prescribed for a broad spectrum of indications with acceptable agreement and validity, regardless of their experience or hospital of employment. However, there is room for improvement, which merits attention in multidisciplinary discussions and education." }, "14697414": { "QUESTION": "Is there a favorable subset of patients with prostate cancer who develop oligometastases?", "CONTEXTS": [ "To analyze, retrospectively, the patterns and behavior of metastatic lesions in prostate cancer patients treated with external beam radiotherapy and to investigate whether patients with5 lesions.", "The treatment and outcome of 369 eligible patients with Stage T1-T3aN0-NXM0 prostate cancer were analyzed during a minimal 10-year follow-up period. All patients were treated with curative intent to a mean dose of 65 Gy. The full history of any metastatic disease was documented for each subject, including the initial site of involvement, any progression over time, and patient survival.", "The overall survival rate for the 369 patients was 75% at 5 years and 45% at 10 years. The overall survival rate of patients who never developed metastases was 90% and 81% at 5 and 10 years, respectively. However, among the 74 patients (20%) who developed metastases, the survival rate at both 5 and 10 years was significantly reduced (p<0.0001). The overall survival rate for patients who developed bone metastases was 58% and 27% at 5 and 10 years, respectively, and patients with bone metastases to the pelvis fared worse compared with those with vertebral metastases. With regard to the metastatic number, patients with5 lesions (73% and 36% at 5 and 10 years vs. 45% and 18% at 5 and 10 years, respectively; p = 0.02). In addition, both the metastasis-free survival rate and the interval measured from the date of the initial diagnosis of prostate cancer to the development of bone metastasis were statistically superior for patients with5 lesions (p = 0.01 and 0.02, respectively). However, the survival rate and the interval from the date of diagnosis of bone metastasis to the time of death for patients in both groups were not significantly different, statistically (p = 0.17 and 0.27, respectively)." ], "LABELS": [ "OBJECTIVE", "METHODS AND MATERIALS", "RESULTS" ], "MESHES": [ "Aged", "Aged, 80 and over", "Androgen Antagonists", "Antineoplastic Agents, Hormonal", "Bone Neoplasms", "Brain Neoplasms", "Humans", "Liver Neoplasms", "Lung Neoplasms", "Male", "Middle Aged", "Neoplasm Staging", "Orchiectomy", "Prostatic Neoplasms", "Retrospective Studies", "Survival Rate", "Treatment Outcome" ], "YEAR": "2004", "reasoning_required_pred": "yes", "reasoning_free_pred": "yes", "final_decision": "yes", "LONG_ANSWER": "Patients with5 lesions. Because existing sites of metastatic disease may be the primary sites of origin for additional metastases, our findings suggest that early detection and aggressive treatment of patients with a small number of metastatic lesions is worth testing as an approach to improving long-term survival." }, "26686513": { "QUESTION": "Cycloplegic autorefraction in young adults: is it mandatory?", "CONTEXTS": [ "The precise correction of refractive error is especially important in young adults. It is unclear whether cycloplegic refraction is necessary in this age group. The purpose of this study was to compare the non-cycloplegic and cycloplegic spherical equivalent (SE) refractive error measured in young adults.", "This was a prospective study of 1400 eyes (n\u2009=\u2009700) of enlisted soldiers aged 18 to 21\u00a0years who were consecutively evaluated in an outpatient army ophthalmology clinic. One drop of cyclopentolate 1\u00a0% was installed twice 10\u00a0min apart, and cycloplegic refraction was performed in both eyes 40\u00a0min later using an auto-refractor. The difference between non-cycloplegic and cycloplegic refractive measurements was analyzed.", "The mean difference in SE between non-cycloplegic and cycloplegic measurements was 0.68\u2009\u00b1\u20090.83\u00a0D (95\u00a0% CI, 0.64-0.72). Significantly greater differences were observed in hypermetropes than myopes (1.30\u2009\u00b1\u20090.90\u00a0D versus 0.46\u2009\u00b1\u20090.68\u00a0D, p\u2009<\u20090.001). Moderate hypermetropes (2 to 5\u00a0D) demonstrated significantly greater refractive error than mild (0.5 to 2\u00a0D) or severe (>5\u00a0D) hypermetropes (1.71\u2009\u00b1\u20091.18\u00a0D versus 1.19\u2009\u00b1\u20090.74\u00a0D and 1.16\u2009\u00b1\u20091.08\u00a0D respectively, p\u2009<\u20090.001)." ], "LABELS": [ "PURPOSE", "METHODS", "RESULTS" ], "MESHES": [ "Adolescent", "Cyclopentolate", "Female", "Humans", "Hyperopia", "Male", "Military Personnel", "Mydriatics", "Myopia", "Prospective Studies", "Pupil", "Refraction, Ocular", "Retinoscopy", "Young Adult" ], "YEAR": "2016", "reasoning_required_pred": "maybe", "reasoning_free_pred": "yes", "final_decision": "maybe", "LONG_ANSWER": "Young hypermetropic adults possessed +1 to +2\u00a0D of latent hypermetropia. In contrast, young myopic adults revealed pseudomyopia of -0.5\u00a0D. Cycloplegic refraction should be performed in young hypermetropic adults complaining of various signs of asthenopia." }, "15530261": { "QUESTION": "Does screening or surveillance for primary hepatocellular carcinoma with ultrasonography improve the prognosis of patients?", "CONTEXTS": [ "The purpose of this paper is to evaluate the efficacy of ultrasonographic screening for primary hepatocellular carcinoma.", "A total of 680 eligible cases were classified into three groups (surveillance, opportunistic, and symptomatic groups) according to their initial exposure. We used survival time, tumor morphology, and T staging as prognostic outcomes. The outcomes of screened/unscreened and sur veillance/nonsur veillance were compared with the use of the logistic regression model.", "The adjusted odds ratios for the screened group versus the unscreened group, with 1-, 2-, and 3-year survival time being used as outcomes, were 0.33 (95% confidence interval [CI], 0.21-0.52), 0.33 (95% CI, 0.21-0.53), and 0.37 (95% CI, 0.23-0.61), respectively. The adjusted odds ratios for surveillance versus nonsurveillance were 0.58 (95% CI, 0.35-0.97), 0.45 (95% CI, 0.27-0.74), and 0.44 (95% CI, 0.26-0.73). The odds ratios were even smaller when tumor morphology or T stage was taken as the main outcome. All these results were statistically significant. There were significant gradient relationships between prognostic outcomes and extent of screening history." ], "LABELS": [ "PURPOSE", "PATIENTS AND METHODS", "RESULTS" ], "MESHES": [ "Adult", "Aged", "Carcinoma, Hepatocellular", "Female", "Humans", "Liver Neoplasms", "Male", "Mass Screening", "Middle Aged", "Odds Ratio", "Population Surveillance", "Prognosis", "Registries", "Survival Analysis", "Taiwan", "Ultrasonography" ], "YEAR": null, "reasoning_required_pred": "yes", "reasoning_free_pred": "yes", "final_decision": "yes", "LONG_ANSWER": "The significant impact of ultrasonographic screening on mortality reduction was demonstrated. These findings strongly suggest that early detection of hepatocellular carcinoma by ultrasound may improve the prognosis of patients with hepatocellular carcinoma." }, "24160268": { "QUESTION": "\"Would a man smell a rose then throw it away?", "CONTEXTS": [ "Breast cancer is the most common malignancy afflicting women, and the most common cancer overall in Jordan. A woman's decision to go for screening is influenced by her social support network. This study aims to explore Jordanian men's individual and contextual perspectives on women's breast cancer and their own role in the breast health of the females within their families.", "An explorative qualitative design was used to purposively recruit 24 married men aged 27 to 65\u00a0years (median 43\u00a0years) from four governorates in Jordan. Data in the form of interviews transcriptions was subjected to qualitative content analysis.", "Three themes were identified: a) Supporting one's wife; b) Marital needs and obligations; c) Constrained by a culture of destiny and shame. The first theme was built on men's feelings of responsibility for the family's health and well-being, their experiences of encouraging their wives to seek health care and their providing counselling and instrumental support. The second theme emerged from men's views about other men's rejection of a wife inflicted by breast cancer, their own perceptions of diminished femininity due to mastectomy and their own concerns about protecting the family from the hereditary risk of breast cancer. The third theme was seen in men's perception of breast cancer as an inevitable act of God that is far away from one's own family, in associating breast cancer with improper behaviour and in their readiness to face the culture of Eib (shame)." ], "LABELS": [ "BACKGROUND", "METHODS", "RESULTS" ], "MESHES": [ "Adult", "Aged", "Attitude to Health", "Breast Neoplasms", "Early Detection of Cancer", "Female", "Humans", "Jordan", "Male", "Mastectomy", "Men", "Middle Aged", "Qualitative Research", "Social Support", "Spouses" ], "YEAR": "2013", "reasoning_required_pred": "maybe", "reasoning_free_pred": "maybe", "final_decision": "maybe", "LONG_ANSWER": "Jordanian men perceive themselves as having a vital role in supporting, guiding and encouraging their wives to follow breast cancer early detection recommendations. Breast health awareness campaigns could involve husbands to capitalize on family support." }, "21823940": { "QUESTION": "Department of Transportation vs self-reported data on motor vehicle collisions and driving convictions for stroke survivors: do they agree?", "CONTEXTS": [ "Research on stroke survivors' driving safety has typically used either self-reports or government records, but the extent to which the 2 may differ is not known. We compared government records and self-reports of motor vehicle collisions and driving convictions in a sample of stroke survivors.", "The 56 participants were originally recruited for a prospective study on driving and community re-integration post-stroke; the study population consisted of moderately impaired stroke survivors without severe communication disorders who had been referred for a driving assessment. The driving records of the 56 participants for the 5 years before study entry and the 1-year study period were acquired with written consent from the Ministry of Transportation of Ontario (MTO), Canada. Self-reports of collisions and convictions were acquired via a semistructured interview and then compared with the MTO records.", "Forty-three participants completed the study. For 7 (13.5%) the MTO records did not match the self-reports regarding collision involvement, and for 9 (17.3%) the MTO records did not match self-reports regarding driving convictions. The kappa coefficient for the correlation between MTO records and self-reports was 0.52 for collisions and 0.47 for convictions (both in the moderate range of agreement). When both sources of data were consulted, up to 56 percent more accidents and up to 46 percent more convictions were identified in the study population in the 5 years before study entry compared to when either source was used alone." ], "LABELS": [ "OBJECTIVE", "METHODS", "RESULTS" ], "MESHES": [ "Accidents, Traffic", "Aged", "Automobile Driving", "Female", "Government Agencies", "Humans", "Male", "Middle Aged", "Ontario", "Prospective Studies", "Records as Topic", "Reproducibility of Results", "Safety", "Self Report", "Stroke", "Survivors" ], "YEAR": "2011", "reasoning_required_pred": "no", "reasoning_free_pred": "no", "final_decision": "no", "LONG_ANSWER": "In our population of stroke survivors, self-reports of motor vehicle collisions and driving convictions differed from government records. In future studies, the use of both government and self-reported data would ensure a more accurate picture of driving safety post-stroke." }, "27217036": { "QUESTION": "Neoadjuvant Imatinib in Locally Advanced Gastrointestinal stromal Tumours, Will Kit Mutation Analysis Be a Pathfinder?", "CONTEXTS": [ "Longer duration of neoadjuvant (NA) imatinib\u00a0(IM) used for locally advanced (LA) gastrointestinal stromal tumours (GIST) is not based on biology of the tumour reflected by kit mutation analysis.", "LA or locally recurrent (LR) GIST treated with NA IM from May 2008 to March 2015 from a prospective database were included in\u00a0the analysis. Archived formalin-fixed paraffin-embedded tissues (FFPE) were used for testing KIT exons 9, 11, 13 and 17 by PCR.", "One hundred twenty-five patients with LA or LR GIST were treated with NA IM. Forty-five patients (36\u00a0%) had undergone c-kit mutation testing. Exon 11 was seen in 25 patients (55.5\u00a0%), 3 with exon 9 (6.7\u00a0%) and 2 with exon 13 (4.4\u00a0%). Twelve were wild type (26.6\u00a0%) and \u00a03 (6.7 %) were declared uninterpretable. Response rate (RR) for the exon 11 mutants was higher than the non-exon 11 mutant group (84 vs. 40\u00a0%, p\u2009=\u20090.01). Disease stabilization rate (DSR) rates were also higher in the exon 11 subgroup than non-exon 11 group (92 vs. 75\u00a0%). Eighty-four per cent exon 11 and 75\u00a0% non-exon 11 mutants were surgical candidates. Patients undergoing surgery had significantly improved event free survival (EFS) (p\u2009<\u20090.001) compared to patients not undergoing surgery, with the same trend seen in OS (p\u2009=\u20090.021). Patients with a SD on response to NA IM had a lower EFS (p\u2009=\u20090.076) and OS compared to patients achieving CR/PR. There were no differences between the various exon variants in terms of outcomes and responses" ], "LABELS": [ "INTRODUCTION", "MATERIAL AND METHODS", "RESULTS" ], "MESHES": [ "Adult", "Aged", "Antineoplastic Agents", "Female", "Gastrointestinal Stromal Tumors", "Humans", "Imatinib Mesylate", "Male", "Middle Aged", "Mutation", "Neoadjuvant Therapy", "Young Adult" ], "YEAR": "2016", "reasoning_required_pred": "yes", "reasoning_free_pred": "yes", "final_decision": "yes", "LONG_ANSWER": "Upfront evaluation of kit mutation status may help us in delineating separate treatment strategies for potentially biologically different tumours and assessing the correct timing of surgery for this subset of GIST." }, "18565233": { "QUESTION": "Does the lipid-lowering peroxisome proliferator-activated receptors ligand bezafibrate prevent colon cancer in patients with coronary artery disease?", "CONTEXTS": [ "Epidemiologic studies have suggested that hypertriglyceridemia and insulin resistance are related to the development of colon cancer. Nuclear peroxisome proliferator-activated receptors (PPAR), which play a central role in lipid and glucose metabolism, had been hypothesized as being involved in colon cancerogenesis. In animal studies the lipid-lowering PPAR ligand bezafibrate suppressed colonic tumors. However, the effect of bezafibrate on colon cancer development in humans is unknown. Therefore, we proposed to investigate a possible preventive effect of bezafibrate on the development of colon cancer in patients with coronary artery disease during a 6-year follow-up.", "Our population included 3011 patients without any cancer diagnosis who were enrolled in the randomized, double blind Bezafibrate Infarction Prevention (BIP) Study. The patients received either 400 mg of bezafibrate retard (1506 patients) or placebo (1505 patients) once a day. Cancer incidence data were obtained by matching a subject's identification numbers with the National Cancer Registry. Each matched record was checked for correct identification.", "Development of new cancer (all types) was recorded in 177 patients: in 79 (5.25%) patients from the bezafibrate group vs. 98 (6.51%) from the placebo group. Development of colon cancer was recorded in 25 patients: in 8 (0.53%) patients from the bezafibrate group vs. 17 (1.13%) from the placebo group, (Fisher's exact test: one side p = 0.05; two side p = 0.07). A difference in the incidence of cancer was only detectable after a 4 year lag and progressively increased with continued follow-up. On multivariable analysis the colon cancer risk in patients who received bezafibrate tended to be lower with a hazard ratio of 0.47 and 95% confidence interval 0.2-1.1." ], "LABELS": [ "BACKGROUND", "METHODS", "RESULTS" ], "MESHES": [ "Aged", "Bezafibrate", "Colonic Neoplasms", "Coronary Artery Disease", "Dose-Response Relationship, Drug", "Female", "Follow-Up Studies", "Humans", "Hypolipidemic Agents", "Incidence", "Kaplan-Meier Estimate", "Ligands", "Longitudinal Studies", "Male", "Middle Aged", "Multivariate Analysis", "Peroxisome Proliferator-Activated Receptors", "Randomized Controlled Trials as Topic" ], "YEAR": "2008", "reasoning_required_pred": "yes", "reasoning_free_pred": "yes", "final_decision": "yes", "LONG_ANSWER": "Our data, derived from patients with coronary artery disease, support the hypothesis regarding a possible preventive effect of bezafibrate on the development of colon cancer." }, "19694846": { "QUESTION": "Does self-efficacy mediate the relationship between transformational leadership behaviours and healthcare workers' sleep quality?", "CONTEXTS": [ "Although there is evidence for the influential role of transformational leadership on health outcomes, researchers have used either attitude outcomes (e.g. job satisfaction) or softer health measures, such as general well-being. Specific measures of well-being such as sleep quality have not been used, despite its association with working conditions.", "A longitudinal design was used to collect data from Danish healthcare workers at time 1 in 2005 (n = 447) and 18 months later at time 2 in 2007 (n = 274). Structural equation modelling was used to investigate the relationships between transformational leadership, self-efficacy and sleep quality at both time points independently (cross-sectionally) and longitudinally.", "For all constructs, time 2 measures were influenced by the baseline level. Direct relationships between transformational leadership and sleep quality were found. This relationship was negative cross-sectionally at both time points, but positive between baseline and follow-up. The relationship between leadership and employees' sleep quality was not mediated by employees' self-efficacy." ], "LABELS": [ "BACKGROUND", "METHODS", "RESULTS" ], "MESHES": [ "Adult", "Denmark", "Female", "Humans", "Job Satisfaction", "Leadership", "Longitudinal Studies", "Male", "Models, Theoretical", "Nurses' Aides", "Nursing Staff, Hospital", "Self Efficacy", "Sleep", "Sleep Initiation and Maintenance Disorders", "Surveys and Questionnaires" ], "YEAR": "2009", "reasoning_required_pred": "no", "reasoning_free_pred": "maybe", "final_decision": "maybe", "LONG_ANSWER": "Our results indicate that training managers in transformational leadership behaviours may have a positive impact on healthcare workers' health over time. However, more research is needed to examine the mechanisms by which transformational leadership brings about improved sleep quality; self-efficacy was not found to be the explanation." }, "27184293": { "QUESTION": "Does microbial contamination influence the success of the hematopoietic cell transplantation outcomes?", "CONTEXTS": [ "Microbial contamination can be a marker for faulty process and is assumed to play an important role in the collection of hematopoietic progenitor cell (HPC) and infusion procedure. We aimed to determine the microbial contamination rates and evaluate the success of hematopoietic cell transplantation (HCT) in patients who received contaminated products.PATIENTS-", "We analyzed microbial contamination records of HPC grafts between 2012 and 2015, retrospectively. Contamination rates of autologous donors were evaluated for at three steps: at the end of mobilization, following processing with dimethyl sulfoxide, and just before stem cell infusion. Grafts of allogeneic donors were assessed only before HCT.", "A total of 445 mobilization procedures were carried out on 333 (167 autologous and 166 allogeneic) donors. The microbiological contamination of peripheral blood (323/333 donations) and bone marrow (10/333 donations) products were analyzed. Bacterial contamination was detected in 18 of 1552 (1.15 %) culture bottles of 333 donors. During the study period 248 patients underwent HCT and among these patients microbial contamination rate on sample basis was 1.3 % (16/1212). Microbial contamination detected in nine patients (7 autologous; 2 allogeneic). In 8 of 9 patients, a febrile neutropenic attack was observed. The median day for the neutropenic fever was 4 days (0-9). None of the patients died within the post-transplant 30 days who received contaminated products." ], "LABELS": [ "INTRODUCTION", "METHODS", "RESULT" ], "MESHES": [ "Adolescent", "Adult", "Aged", "Allografts", "Autografts", "Female", "Hematologic Neoplasms", "Hematopoietic Stem Cell Mobilization", "Hematopoietic Stem Cell Transplantation", "Hematopoietic Stem Cells", "Humans", "Male", "Middle Aged" ], "YEAR": "2016", "reasoning_required_pred": "no", "reasoning_free_pred": "maybe", "final_decision": "maybe", "LONG_ANSWER": "The use of contaminated products with antibiotic prophylaxis may be safe in terms of the first day of fever, duration of fever, neutrophil, platelet engraftment and duration of hospitalization." }, "19459018": { "QUESTION": "Very high serum CA 19-9 levels: a contraindication to pancreaticoduodenectomy?", "CONTEXTS": [ "From 2000 to 2007, 344 patients underwent pancreatoduodenectomy for PA. Fifty-three patients (elevated group) had preoperatively elevated serum CA 19-9 levels (>400 IU/ml) after resolution of obstructive jaundice. Of these, 27 patients had high levels (400-899 IU/ml (HL)) and 26 patients had very high levels>or=900 IU/ml (VHL). Fifty patients with normal preoperative serum CA 19-9 levels (<37 IU/ml) comprised the control group.", "Median survival of the control group (n = 50) versus elevated group (n = 53) was 22 versus 15 months (p = 0.02) and overall 3-year survival was 32% versus 14% (p = 0.03). There was no statistical difference in the median and 3-year overall survival between patients with HL and VHL. Patients in the elevated group who normalized their CA 19-9 levels after surgery (n = 11) had a survival equivalent to patients in the control group." ], "LABELS": [ "METHODS", "RESULTS" ], "MESHES": [ "Adenocarcinoma", "Aged", "Aged, 80 and over", "Biomarkers, Tumor", "CA-19-9 Antigen", "Contraindications", "Female", "Humans", "Male", "Middle Aged", "Pancreatic Neoplasms", "Pancreaticoduodenectomy", "Postoperative Period", "Predictive Value of Tests", "Preoperative Period", "Prognosis", "Survival Analysis", "Treatment Outcome" ], "YEAR": "2009", "reasoning_required_pred": "yes", "reasoning_free_pred": "no", "final_decision": "no", "LONG_ANSWER": "Patients who normalized their CA19-9 levels postoperatively had equivalent survival to patients with normal preoperative CA 19-9 levels. Preoperative serum CA 19-9 level by itself should not preclude surgery in patients who have undergone careful preoperative staging." }, "22042121": { "QUESTION": "Perioperative care in an animal model for training in abdominal surgery: is it necessary a preoperative fasting?", "CONTEXTS": [ "Demonstrate that the rabbit may be used in the training of surgery, in addition to present its perioperative care.", "Thirty two animals, with age and weight, respectively, from 3 to 5.5 months old and 3000 to 4200 grams, were undergone different periods of pre-operative fasting, exclusive intramuscular anesthesia (ketamine+xylazine), laparotomy with total gastrectomy and total splenectomy. It was dosed the pre-operative (initial) and post-surgical (end) serum blood glucose, in addition to quantify the gastric content after the resection of the part.", "The anesthetical-surgical procedure presented a mortality rate of 3.125% (1:32) and a morbidity rate of 6.25% (2:32). It was evidenced an initial mean blood glucose = 199.4 mg/dl and the end = 326.1 mg/dl. In spite of extended fasting (minimum of 2 hours for the absolute fasting and maximum of 8.5 hours for liquids, and 20.5 hours for solids) all animals presented at the end of the surgical procedure any gastric content and a blood glucose increase. Those with fasting for liquids and solids when compared to the quantity of solid gastric content, presented a moderate negative degree of correlation." ], "LABELS": [ "PURPOSE", "METHODS", "RESULTS" ], "MESHES": [ "Animals", "Blood Glucose", "Fasting", "Gastrectomy", "Gastric Emptying", "Laparotomy", "Male", "Models, Animal", "Perioperative Care", "Preoperative Care", "Prospective Studies", "Rabbits", "Splenectomy" ], "YEAR": "2011", "reasoning_required_pred": "no", "reasoning_free_pred": "no", "final_decision": "no", "LONG_ANSWER": "The rabbit is a good model to be used in training of surgery, with a low morbi-mortality, able to be anesthetized intramuscularly, with no need of pre-operative fasting and does not present hypoglycemia even with the extended fasting period." }, "16100194": { "QUESTION": "Are physicians aware of the side effects of angiotensin-converting enzyme inhibitors?", "CONTEXTS": [ "Angiotensin-converting enzyme inhibitors (ACE-I) are considered safe, but they are associated with characteristic side effects, namely cough and angioedema, usually requiring discontinuation. We perceived that referrals for these side effects have become more and more frequent; therefore, we evaluated the degree of knowledge on the safety of ACE-I in different medical categories.", "A questionnaire (13 questions) on side effects of ACE-I was posted to physicians.", "Everyday clinical practice.", "Cardiologists, allergists, and general practitioners (GPs) from the National Healthcare System.", "Three hundred twelve physicians were contacted, and 154 returned questionnaires that could be analyzed. Of the 154 physicians (mean age, 45 years) 48 were cardiologists, 52 were GPs, and 54 were allergists. The percentage of correct answers was low: 31.9% for cardiologists, 40% for GPs, and 33% for allergists. Thus, GPs provided a significantly higher percentage of correct answers with respect to the remaining categories (p = 0.05). The lower rate of correct answers (0 to 15.9%) concerned the time of onset of cough and the action to take. Cardiologists seemed to be less aware of the fact that angiotensin receptor blockers (sartans) can cross-react with ACE-I." ], "LABELS": [ "STUDY OBJECTIVE", "DESIGN", "SETTING", "PARTICIPANTS", "MEASUREMENT AND RESULTS" ], "MESHES": [ "Adult", "Allergy and Immunology", "Angiotensin-Converting Enzyme Inhibitors", "Cardiology", "Family Practice", "Health Knowledge, Attitudes, Practice", "Humans", "Middle Aged", "Practice Patterns, Physicians'", "Surveys and Questionnaires" ], "YEAR": "2005", "reasoning_required_pred": "no", "reasoning_free_pred": "no", "final_decision": "no", "LONG_ANSWER": "Overall, there was a poor knowledge of the side effects of ACE-I. This may account for the increased referrals for chronic cough and angioedema." }, "25480629": { "QUESTION": "Residual fundus or neofundus after laparoscopic sleeve gastrectomy: is fundectomy safe and effective as revision surgery?", "CONTEXTS": [ "Up to 30 % of patients who have undergone laparoscopic sleeve gastrectomy require revision surgery for inadequate weight loss, weight regain, and/or the development of severe upper gastrointestinal symptoms. The aim of this retrospective study was to evaluate the safety and efficacy of laparoscopic fundectomy (LF) in cases of a residual fundus/neofundus development regarding GERD symptoms.", "The study group comprised 19 patients (17 female; mean BMI 35.4 kg/m(2)) divided into 2 groups. Group A (n = 10) patients with severe GERD and evidence of residual fundus/neofundus, Hiatal hernia with good results in terms of weight loss. Group B (n = 9) patients with severe GERD, a residual fundus/neofundus, inadequate weight loss or weight regain. Fundectomy was indicated when a residual fundus/neofundus was associated with severe GERD symptoms. The presence of a residual fundus/neofundus was assessed by a barium swallow and/or multislice computed tomography.", "No mortality or intra-operative complications occurred. Five postoperative complications occurred: 2 cases of bleeding, 1 mid-gastric stenosis and 2 leaks (10.5 %). All patients experienced improvements in their GERD symptoms and stopped PPI treatment. Group B exhibited an additional %EWL of 53.4 % at 24 months." ], "LABELS": [ "INTRODUCTION", "METHODS", "RESULTS" ], "MESHES": [ "Female", "Gastrectomy", "Gastric Fundus", "Gastroesophageal Reflux", "Humans", "Laparoscopy", "Male", "Postoperative Complications", "Reoperation", "Retrospective Studies" ], "YEAR": "2015", "reasoning_required_pred": "yes", "reasoning_free_pred": "maybe", "final_decision": "maybe", "LONG_ANSWER": "LF and cruroplasty is feasible and has good results in terms of GERD symptoms control and additional weight loss. The high rate of postoperative complications observed in this series remains a matter of concern. A re-sleeve procedure might be considered as an alternative to RYGB/DS conversion restricted to selected patients." }, "17453263": { "QUESTION": "Are physician estimates of asthma severity less accurate in black than in white patients?", "CONTEXTS": [ "Racial differences in asthma care are not fully explained by socioeconomic status, care access, and insurance status. Appropriate care requires accurate physician estimates of severity. It is unknown if accuracy of physician estimates differs between black and white patients, and how this relates to asthma care disparities.", "We hypothesized that: 1) physician underestimation of asthma severity is more frequent among black patients; 2) among black patients, physician underestimation of severity is associated with poorer quality asthma care.", "We conducted a cross-sectional survey among adult patients with asthma cared for in 15 managed care organizations in the United States. We collected physicians' estimates of their patients' asthma severity. Physicians' estimates of patients' asthma as being less severe than patient-reported symptoms were classified as underestimates of severity.", "Frequency of underestimation, asthma care, and communication.", "Three thousand four hundred and ninety-four patients participated (13% were black). Blacks were significantly more likely than white patients to have their asthma severity underestimated (OR = 1.39, 95% CI 1.08-1.79). Among black patients, underestimation was associated with less use of daily inhaled corticosteroids (13% vs 20%, p<.05), less physician instruction on management of asthma flare-ups (33% vs 41%, p<.0001), and lower ratings of asthma care (p = .01) and physician communication (p = .04)." ], "LABELS": [ "BACKGROUND", "OBJECTIVE", "DESIGN, SETTING AND PATIENTS", "MEASUREMENTS", "RESULTS" ], "MESHES": [ "Adult", "African Americans", "Asthma", "Clinical Competence", "Cohort Studies", "Cross-Sectional Studies", "European Continental Ancestry Group", "Female", "Humans", "Male", "Middle Aged", "Odds Ratio", "Physician-Patient Relations", "Quality of Health Care", "Severity of Illness Index" ], "YEAR": "2007", "reasoning_required_pred": "no", "reasoning_free_pred": "yes", "final_decision": "yes", "LONG_ANSWER": "Biased estimates of asthma severity may contribute to racially disparate asthma care. Interventions to improve physicians' assessments of asthma severity and patient-physician communication may minimize racial disparities in asthma care." }, "18322741": { "QUESTION": "Does laparoscopic surgery decrease the risk of atrial fibrillation after foregut surgery?", "CONTEXTS": [ "Atrial fibrillation, which occurs in 12% of all major foregut surgeries, can prolong hospital stay and increase morbidity. Minimally invasive techniques in foregut surgery have been suggested to cause less tissue trauma. We examined the factors associated with new-onset atrial fibrillation after foregut surgery at our institution.", "We retrospectively examined the records of 154 adult patients who underwent major foregut surgery which included esophagectomy, partial or total gastrectomy, redo Heller myotomy, redo or transthoracic fundoplications. Univariate and multivariate logistic regression analysis with standard modeling techniques were performed to determine risk factors for new-onset atrial fibrillation.", "Of the 154 patients, 14 patients developed new-onset atrial fibrillation with a higher mean age of 67.1 years (+/-8.8 years) versus 56.4 years (+/-14.1 years) (p = 0.006). Laparoscopic (p = 0.004) and nonthoracic surgeries (p = 0.01) were associated with lower risk of atrial fibrillation. Patients with atrial fibrillation had received more fluid (6.5 +/- 2.8 liters versus 5.3 +/- 2.0 liters) and had longer operations (370 +/- 103 min versus 362 +/- 142 min), none of which were statistically significant. The average intensive care length of stay of patients was longer: 7.5 +/- 6.8 days versus 4.0 +/- 7.1 days (p = 0.004). Multivariate analysis revealed an association of atrial fibrillation with age (OR 1.08, 95% CI 1.02-1.14, p = 0.01), and laparoscopic surgery (OR 0.09, 95% CI 0.01-0.95, p = 0.04) after adjusting for surgery type." ], "LABELS": [ "BACKGROUND", "METHODS", "RESULTS" ], "MESHES": [ "Adult", "Age Factors", "Aged", "Atrial Fibrillation", "Cohort Studies", "Digestive System Surgical Procedures", "Female", "Humans", "Laparoscopy", "Length of Stay", "Male", "Middle Aged", "Retrospective Studies", "Risk Factors" ], "YEAR": "2009", "reasoning_required_pred": "yes", "reasoning_free_pred": "yes", "final_decision": "yes", "LONG_ANSWER": "Laparoscopic surgery is associated with lower risk of atrial fibrillation in foregut surgery. Development of atrial fibrillation is associated with increased length of intensive care stay. We recommend a prospective trial to confirm our findings." }, "19106867": { "QUESTION": "The Main Gate Syndrome: a new format in mass-casualty victim \"surge\" management?", "CONTEXTS": [ "Recent suicide bombings pose the novel problem for Trauma Centers of the massive simultaneous arrival of many gravely wounded patients.", "We report the experience of the French-German Military Trauma Group, a Level 2 Trauma Center, in Afghanistan during the wave of suicide bombings in February 2007.", "Fourteen casualties were received. A first triage was carried out by the U S Army Level I group prior to evacuation. A second surgical triage was carried out with systematic ultrasound exam. Four cases (ISS>25) were re-categorized and underwent emergency surgical procedures." ], "LABELS": [ "INTRODUCTION", "METHODS", "RESULTS" ], "MESHES": [ "Abdominal Injuries", "Adult", "Afghanistan", "Blast Injuries", "Bombs", "Emergency Medical Services", "Extremities", "Humans", "Male", "Mass Casualty Incidents", "Middle Aged", "Reproducibility of Results", "Rescue Work", "Retrospective Studies", "Syndrome", "Thoracic Injuries", "Trauma Severity Indices", "Treatment Outcome", "Wounds, Penetrating", "Young Adult" ], "YEAR": null, "reasoning_required_pred": "maybe", "reasoning_free_pred": "yes", "final_decision": "yes", "LONG_ANSWER": "Suicide bombing in crowded locations near an evacuation hospital may overwhelm the medical resources of the receiving center. It has been referred to as \"The Main Gate Syndrome.\" We introduced the novel concept of a semi-evacuation hospital or receiving center where a second surgical triage was carried out. These exceptional circumstances require open-minded flexibility, a tailored approach, and close cooperation between surgeons and anesthetists to share experience, opinions, and ideas. In the setting of mass casualties, emergency ultrasound exam was shown to be a valuable and effective tool by virtue of its mobility, reproducibility, and immediate results." }, "25691513": { "QUESTION": "Do communication disorders extend to musical messages?", "CONTEXTS": [ "Effective musical communication requires conveyance of the intended message in a manner perceptible to the receiver. Communication disorders that impair transmitting or decoding of structural features of music (e.g., pitch, timbre) and/or symbolic representation may result in atypical musical communication, which can have a negative impact on music therapy interventions.", "This study compared recognition of symbolic representation of emotions or movements in music by two groups of children with different communicative characteristics: severe to profound hearing loss (using cochlear implants [CI]) and autism spectrum disorder (ASD). Their responses were compared to those of children with typical-development and normal hearing (TD-NH). Accuracy was examined as a function of communicative status, emotional or movement category, and individual characteristics.", "Participants listened to recorded musical excerpts conveying emotions or movements and matched them with labels. Measures relevant to auditory and/or language function were also gathered.", "There was no significant difference between the ASD and TD-NH groups in identification of musical emotions or movements. However, the CI group was significantly less accurate than the other two groups in identification of both emotions and movements. Mixed effects logistic regression revealed different patterns of accuracy for specific emotions as a function of group." ], "LABELS": [ "BACKGROUND", "OBJECTIVE", "METHODS", "RESULTS" ], "MESHES": [ "Adolescent", "Auditory Perception", "Child", "Child Development Disorders, Pervasive", "Cochlear Implantation", "Communication Disorders", "Deafness", "Emotions", "Female", "Humans", "Male", "Music" ], "YEAR": "2015", "reasoning_required_pred": "yes", "reasoning_free_pred": "maybe", "final_decision": "yes", "LONG_ANSWER": "Conveyance of emotions or movements through music may be decoded differently by persons with different types of communication disorders. Because music is the primary therapeutic tool in music therapy sessions, clinicians should consider these differential abilities when selecting music for clinical interventions focusing on emotions or movement." }, "11035130": { "QUESTION": "Do patients with rheumatoid arthritis established on methotrexate and folic acid 5 mg daily need to continue folic acid supplements long term?", "CONTEXTS": [ "It is postulated that some aspects of methotrexate toxicity may be related to its action as an anti-folate. Folic acid (FA) is often given as an adjunct to methotrexate therapy, but there is no conclusive proof that it decreases the toxicity of methotrexate and there is a theoretical risk that it may decrease the efficacy of methotrexate.", "To look at the effect of stopping FA supplementation in UK rheumatoid arthritis (RA) patients established on methotrexate<20 mg weekly and FA 5 mg daily, to report all toxicity (including absolute changes in haematological and liver enzyme indices) and to report changes in the efficacy of methotrexate.", "In a prospective, randomized, double-blind, placebo-controlled study, 75 patients who were established on methotrexate<20 mg weekly and FA 5 mg daily were asked to stop their FA and were randomized to one of two groups: placebo or FA 5 mg daily. Patients were evaluated for treatment toxicity and efficacy before entry and then at intervals of 3 months for 1 yr.", "Overall, 25 (33%) patients concluded the study early, eight (21%) in the group remaining on FA and 17 (46%) in the placebo group (P = 0.02). Two patients in the placebo group discontinued because of neutropenia. At 9 months there was an increased incidence of nausea in the placebo group (45 vs. 7%, P = 0.001). The placebo group had significantly lower disease activity on a few of the variables measured, but these were probably not of clinical significance." ], "LABELS": [ "BACKGROUND", "OBJECTIVES", "METHODS", "RESULTS" ], "MESHES": [ "Aged", "Antirheumatic Agents", "Arthritis, Rheumatoid", "Double-Blind Method", "Drug Administration Schedule", "Female", "Folic Acid", "Humans", "Male", "Methotrexate", "Middle Aged", "Prospective Studies", "Treatment Outcome" ], "YEAR": "2000", "reasoning_required_pred": "yes", "reasoning_free_pred": "yes", "final_decision": "yes", "LONG_ANSWER": "It is important to continue FA supplementation over the long term in patients on methotrexate and FA in order to prevent them discontinuing treatment because of mouth ulcers or nausea and vomiting. Our data suggest that FA supplementation is also helpful in preventing neutropenia, with very little loss of efficacy of methotrexate." }, "19409117": { "QUESTION": "Can infundibular height predict the clearance of lower pole calyceal stone after extracorporeal shockwave lithotripsy?", "CONTEXTS": [ "To evaluate the efficacy of extracorporeal shock wave lithotripsy (SWL) on lower calyceal calculi in relation to the renal anatomical factors and determine which of these factors can be used to select patients who will benefit from SWL.", "We analyzed retrospectively 78 patients with single radiopaque lower calyceal stones treated with SWL. The patients were evaluated 3 months after lithotripsy with a simple abdominal X-ray and a kidney ultrasound scan. The success of the treatment, removal of all fragments, was correlated with renal anatomical factors measured in the pre-treatment intravenous urography: infundibulopelvic angle, lower infundibulum width, lower infundibulum length, ratio length/width, infundibulum height, and number of minor calyces in the lower calyceal group.", "Three months after SWL treatment, 39 patients were stone-free (NR group) and 39 had residual fragments (R group). Both groups presented no differences in relation to infundibulopelvic angle, width and length of the lower calyceal infundibulum, length/width ratio of the lower infundibulum or number of lower calyces. Height of the infundibulum, described as the distance between the line passing through the lowest part of the calyx containing the calculus and the highest point of the lower lip of renal pelvis, was the only parameter in which significant differences (p = 0.002) were found between the NR and R groups." ], "LABELS": [ "PURPOSE", "MATERIALS AND METHODS", "RESULTS" ], "MESHES": [ "Female", "Humans", "Kidney Calculi", "Kidney Calices", "Lithotripsy", "Logistic Models", "Male", "Middle Aged", "ROC Curve", "Retrospective Studies", "Treatment Outcome" ], "YEAR": null, "reasoning_required_pred": "yes", "reasoning_free_pred": "yes", "final_decision": "yes", "LONG_ANSWER": "Lower Infundibular height could be a good measurement tool for deciding which patients with lower calyceal lithiasis would benefit from SWL treatment. Height of less than 22 mm suggests a good outcome from lithotripsy." }, "21900017": { "QUESTION": "Does lung ischemia and reperfusion have an impact on coronary flow?", "CONTEXTS": [ "Ischemia-reperfusion (IR) injury remains a major cause of early morbidity and mortality after lung transplantation with poorly documented extrapulmonary repercussions. To determine the hemodynamic effect due to lung IR injury, we performed a quantitative coronary blood-flow analysis in a swine model of in situ lung ischemia and reperfusion.", "In 14 healthy pigs, blood flow was measured in the ascending aorta, left anterior descending (LAD), circumflex (Cx), right coronary artery (RCA), right common carotid artery (RCCA), and left internal mammary artery (LIMA), along with left-and right-ventricular pressures (LVP and RVP), aortic pressure (AoP), and pulmonary artery pressure (PAP). Cardiac Troponin (cTn), interleukin 6 and 10 (IL-6 and IL-10), and tumor necrosis factor A (TNF-A) were measured in coronary sinus blood samples. The experimental (IR) group (n=10) underwent 60 min of lung ischemia followed by 60 min of reperfusion by clamping and releasing the left pulmonary hilum. Simultaneous measurements of all parameters were made at baseline and during IR. The control group (n=4) had similar measurements without lung IR.", "In the IR group, total coronary flow (TCF=LAD+Cx+RCA blood-flow) decreased precipitously and significantly from baseline (113\u00b141 ml min\"1) during IR (p<0.05), with the lowest value observed at 60 min of reperfusion (-37.1%, p<0.003). Baseline cTn (0.08\u00b10.02 ng ml(-1)) increased during IR and peaked at 45 min of reperfusion (+138%, p<0.001). Baseline IL-6 (9.2\u00b12.17 pg ml(-1)) increased during IR and peaked at 60 min of reperfusion (+228%, p<0.0001). Significant LVP drop at 5 min of ischemia (p<0.05) was followed by a slow return to baseline at 45 min of ischemia. A second LVP drop occurred at reperfusion (p<0.05) and persisted. Conversely, RVP increased throughout ischemia (p<0.05) and returned toward baseline during reperfusion. Coronary blood flow and hemodynamic profile remained unchanged in the control group. IL-10 and TNF-A remained below the measurable range for both the groups." ], "LABELS": [ "OBJECTIVE", "METHODS", "RESULTS" ], "MESHES": [ "Animals", "Coronary Circulation", "Cytokines", "Disease Models, Animal", "Hemodynamics", "Inflammation Mediators", "Lung", "Male", "Reperfusion Injury", "Sus scrofa" ], "YEAR": "2012", "reasoning_required_pred": "yes", "reasoning_free_pred": "yes", "final_decision": "yes", "LONG_ANSWER": "In situ lung IR has a marked negative impact on coronary blood flow, hemodynamics, and inflammatory profile. In addition, to the best of our knowledge, this is the first study where coronary blood flow is directly measured during lung IR, revealing the associated increased cardiac risk." }, "16968876": { "QUESTION": "Is a patient's self-reported health-related quality of life a prognostic factor for survival in non-small-cell lung cancer patients?", "CONTEXTS": [ "The aim of this prognostic factor analysis was to investigate if a patient's self-reported health-related quality of life (HRQOL) provided independent prognostic information for survival in non-small cell lung cancer (NSCLC) patients.", "Pretreatment HRQOL was measured in 391 advanced NSCLC patients using the EORTC QLQ-C30 and the EORTC Lung Cancer module (QLQ-LC13). The Cox proportional hazards regression model was used for both univariate and multivariate analyses of survival. In addition, a bootstrap validation technique was used to assess the stability of the outcomes.", "The final multivariate Cox regression model retained four parameters as independent prognostic factors for survival: male gender with a hazard ratio (HR) = 1.32 (95% CI 1.03-1.69; P = 0.03); performance status (0 to 1 versus 2) with HR = 1.63 (95% CI 1.04-2.54; P = 0.032); patient's self-reported score of pain with HR= 1.11 (95% CI 1.07-1.16; P<0.001) and dysphagia with HR = 1.12 (95% CI 1.04-1.21; P = 0.003). A 10-point shift worse in the scale measuring pain and dysphagia translated into an 11% and 12% increased in the likelihood of death respectively. A risk group categorization was also developed." ], "LABELS": [ "BACKGROUND", "PATIENTS AND METHODS", "RESULTS" ], "MESHES": [ "Adult", "Aged", "Carcinoma, Non-Small-Cell Lung", "Europe", "Female", "Health Status", "Humans", "Male", "Middle Aged", "Multivariate Analysis", "Prognosis", "Quality of Life", "Regression Analysis", "Survival Analysis" ], "YEAR": "2006", "reasoning_required_pred": "maybe", "reasoning_free_pred": "yes", "final_decision": "maybe", "LONG_ANSWER": "The results suggest that patients' self-reported HRQOL provide independent prognostic information for survival. This finding supports the collection of such data in routine clinical practice." }, "18540901": { "QUESTION": "Transient tachypnea of the newborn (TTN): a role for polymorphisms in the beta-adrenergic receptor (ADRB) encoding genes?", "CONTEXTS": [ "DNA was collected for genotyping from 73 term newborns suffering from TTN and 55 healthy controls from a Caucasian cohort.", "TTN infants were more likely to be male (70% vs. 49%; p<0.05), had a lower mean birthweight (3120 +/- 450 vs. 3396 +/- 504 g; p<0.001) and gestational age (GA) (38.4 +/- 1.2 vs. 39.4 +/- 1.3 weeks; p<0.001) and were more often delivered by caesarean section (CS) (71% vs. 26%; p<0.001). The beta1Ser49Gly polymorphism differed significantly between cases and controls. Multivariate analysis provided beta1Gly49 homozygotes with higher risk for TTN (OR 18.5; 95%CI 1.5-229; p = 0.023) than beta1Ser49 allele carrier. Further analysis showed significant association of T-47C, A46G, C79G and C491T (TACC) haplotype in ADRB2 gene with TTN (p = 0.048)." ], "LABELS": [ "METHODS", "RESULTS" ], "MESHES": [ "Apgar Score", "Case-Control Studies", "DNA", "Female", "Genotype", "Gestational Age", "Haplotypes", "Humans", "Infant, Newborn", "Male", "Polymorphism, Single Nucleotide", "Receptors, Adrenergic, beta", "Respiration Disorders", "Time Factors" ], "YEAR": "2008", "reasoning_required_pred": "yes", "reasoning_free_pred": "yes", "final_decision": "yes", "LONG_ANSWER": "We conclude that beta1Gly49 homozygosity and TACC haplotype of ADRB2 gene, both loss-of-function genetic variations, may predispose to TTN." }, "27448572": { "QUESTION": "Is duration of psychological treatment for depression related to return into treatment?", "CONTEXTS": [ "There is increasing pressure on mental health providers to reduce the duration of treatments, while retaining level of quality and effectiveness. The risk is that the population is underserved and therefore needs new treatment episodes. The primary aim of this study was to investigate whether duration of treatment and return into mental health care were related.", "This study examined Dutch patients with an initial treatment episode in 2009 or 2010 in specialized mental health settings for depressive disorder (N\u00a0=\u00a085,754). Follow-up data about treatment episodes were available up until 2013. The data set included demographic (age, gender), and clinical factors (comorbidity with other DSM-IV Axis; scores on the 'Global Assessment of Functioning'). Cox regression analyses were used to assess whether duration of treatment and relapse into mental health care were related.", "The majority of patients did not return into mental health care (86\u00a0%). Patients with a shorter duration of treatment (5-250\u00a0min; 251-500\u00a0min and 751-1000\u00a0min) were slightly more likely to return (reference group:>1000\u00a0min) (HR 1.19 95\u00a0% CI 1.13-1.26; HR 1.11 95\u00a0% CI 1.06-1.17; HR 1.18 95\u00a0% CI 1.11-1.25), adjusted for demographic and clinical variables." ], "LABELS": [ "PURPOSE", "METHODS", "RESULTS" ], "MESHES": [ "Adult", "Depression", "Depressive Disorder", "Diagnostic and Statistical Manual of Mental Disorders", "Female", "Humans", "Male", "Mental Health Services", "Middle Aged", "Psychotherapy", "Recurrence", "Time Factors" ], "YEAR": "2016", "reasoning_required_pred": "yes", "reasoning_free_pred": "yes", "final_decision": "yes", "LONG_ANSWER": "The results suggest that a longer duration of treatment may prevent return into mental health care in some groups. However, because of the design of the study, no causal inference can be drawn. Further research, preferably in a RCT, is needed to determine whether the trend towards lower intensity treatments is associated with repeated mental health care use." }, "17621202": { "QUESTION": "Does shaving the incision site increase the infection rate after spinal surgery?", "CONTEXTS": [ "A prospective randomized clinical study.", "To determine whether shaving the incision site before spinal surgery causes postsurgical infection.", "Spine surgeons usually shave the skin of the incision site immediately before surgery is performed. However, evidence from some surgical series suggests that presurgical shaving may increase the postsurgical infection rate. To our knowledge, no previously published studies have addressed this issue.", "A total of 789 patients scheduled to undergo spinal surgery were randomly allocated into 2 groups: those in whom the site of operation was shaved immediately before surgery (shaved group; 371 patients) and the patients in whom presurgical shaving was not performed (unshaved group; 418 patients). The mean duration of anesthesia and the infection rates in both groups were recorded and compared.", "The duration of anesthesia did not differ in the 2 groups (P>0.05). A postoperative infection developed in 4 patients in the shaved group and in 1 patient in the nonshaved group (P<0.01)." ], "LABELS": [ "STUDY DESIGN", "OBJECTIVE", "SUMMARY OF BACKGROUND DATA", "METHODS", "RESULTS" ], "MESHES": [ "Adult", "Anti-Bacterial Agents", "Dermatologic Surgical Procedures", "Double-Blind Method", "Female", "Humans", "Male", "Middle Aged", "Neurosurgical Procedures", "Preoperative Care", "Prospective Studies", "Skin", "Spinal Diseases", "Surgical Wound Infection" ], "YEAR": "2007", "reasoning_required_pred": "yes", "reasoning_free_pred": "maybe", "final_decision": "maybe", "LONG_ANSWER": "The shaving of the incision site immediately before spinal surgery may increase the rate of postoperative infection." }, "25987398": { "QUESTION": "The influence of atmospheric pressure on aortic aneurysm rupture--is the diameter of the aneurysm important?", "CONTEXTS": [ "The rate of aortic aneurysm rupture correlates with the aneurysm's diameter, and a higher rate of rupture is observed in patients with larger aneurysms. According to the literature, contradictory results concerning the relationship between atmospheric pressure and aneurysm size have been reported.AIM: In this paper, we assessed the influence of changes in atmospheric pressure on abdominal aneurysm ruptures in relationship to the aneurysm's size.", "The records of 223 patients with ruptured abdominal aneurysms were evaluated. All of the patients had been admitted to the department in the period 1997-2007 from the Silesia region. The atmospheric pressures on the day of the rupture and on the days both before the rupture and between the rupture events were compared. The size of the aneurysm was also considered in the analysis.", "There were no statistically significant differences in pressure between the days of rupture and the remainder of the days within an analysed period. The highest frequency of the admission of patients with a ruptured aortic aneurysm was observed during periods of winter and spring, when the highest mean values of atmospheric pressure were observed; however, this observation was not statistically confirmed. A statistically non-significant trend towards the higher rupture of large aneurysms (>7 cm) was observed in the cases where the pressure increased between the day before the rupture and the day of the rupture. This trend was particularly pronounced in patients suffering from hypertension (p = 0.1)." ], "LABELS": [ "BACKGROUND", "METHODS", "RESULTS" ], "MESHES": [ "Aged", "Aged, 80 and over", "Aortic Aneurysm, Abdominal", "Aortic Rupture", "Atmospheric Pressure", "Female", "Humans", "Male", "Middle Aged", "Poland", "Retrospective Studies", "Seasons" ], "YEAR": "2015", "reasoning_required_pred": "maybe", "reasoning_free_pred": "no", "final_decision": "maybe", "LONG_ANSWER": "The results of this study do not support the hypothesis that there is a direct link between atmospheric pressure values and abdominal aortic aneurysm ruptures." }, "16816043": { "QUESTION": "Do French lay people and health professionals find it acceptable to breach confidentiality to protect a patient's wife from a sexually transmitted disease?", "CONTEXTS": [ "To determine under what conditions lay people and health professionals find it acceptable for a physician to breach confidentiality to protect the wife of a patient with a sexually transmitted disease (STD).", "In a study in France, breaching confidentiality in 48 scenarios were accepted by 144 lay people, 10 psychologists and 7 physicians. The scenarios were all possible combinations of five factors: severity of the disease (severe, lethal); time taken to discuss this with (little time, much time); intent to inform the spouse about the disease (none, one of these days, immediately); intent to adopt protective behaviours (no intent, intent); and decision to consult an expert in STDs (yes, no), 2 x 2 x 3 x 2 x 2. The importance and interactions of each factor were determined, at the group level, by performing analyses of variance and constructing graphs.", "The concept of breaching confidentiality to protect a wife from her husband's STD was favoured much more by lay people and psychologists than by physicians (mean ratings 11.76, 9.28 and 2.90, respectively, on a scale of 0-22). The patient's stated intentions to protect his wife and to inform her of the disease had the greatest impact on acceptability. A cluster analysis showed groups of lay participants who found breaching confidentiality \"always acceptable\" (n = 14), \"depending on the many circumstances\" (n = 87), requiring \"consultation with an expert\" (n = 30) and \"never acceptable (n = 13)\"." ], "LABELS": [ "OBJECTIVE", "METHODS", "RESULTS" ], "MESHES": [ "Adolescent", "Adult", "Attitude of Health Personnel", "Attitude to Health", "Cluster Analysis", "Confidentiality", "Female", "France", "Humans", "Intention", "Interpersonal Relations", "Male", "Middle Aged", "Severity of Illness Index", "Sexual Behavior", "Sexually Transmitted Diseases", "Spouses", "Time Factors" ], "YEAR": "2006", "reasoning_required_pred": "maybe", "reasoning_free_pred": "maybe", "final_decision": "maybe", "LONG_ANSWER": "Most people in France are influenced by situational factors when deciding if a physician should breach confidentiality to protect the spouse of a patient infected with STD." }, "12920330": { "QUESTION": "Do somatic complaints predict subsequent symptoms of depression?", "CONTEXTS": [ "Evidence suggests substantial comorbidity between symptoms of somatization and depression in clinical as well as nonclinical populations. However, as most existing research has been retrospective or cross-sectional in design, very little is known about the specific nature of this relationship. In particular, it is unclear whether somatic complaints may heighten the risk for the subsequent development of depressive symptoms.", "We report findings on the link between symptoms of somatization (assessed using the SCL-90-R) and depression 5 years later (assessed using the CES-D) in an initially healthy cohort of community adults, based on prospective data from the RENO Diet-Heart Study.", "Gender-stratified multiple regression analyses revealed that baseline CES-D scores were the best predictors of subsequent depressive symptoms for men and women. Baseline scores on the SCL-90-R somatization subscale significantly predicted subsequent self-reported symptoms of depressed mood 5 years later, but only in women. However, somatic complaints were a somewhat less powerful predictor than income and age." ], "LABELS": [ "BACKGROUND", "METHODS", "RESULTS" ], "MESHES": [ "Adult", "Aged", "Comorbidity", "Demography", "Depression", "Female", "Humans", "Male", "Middle Aged", "Primary Health Care", "Prospective Studies", "Somatoform Disorders" ], "YEAR": null, "reasoning_required_pred": "yes", "reasoning_free_pred": "maybe", "final_decision": "maybe", "LONG_ANSWER": "Our findings suggest that somatic complaints may represent one, but not necessarily the most important, risk factor for the subsequent development of depressive symptoms in women in nonclinical populations. The results also highlight the importance of including social variables in studies on women's depression as well as conducting additional research to further examine predictors of depressive symptoms in men." }, "23497210": { "QUESTION": "Are women with major depression in pregnancy identifiable in population health data?", "CONTEXTS": [ "Although record linkage of routinely collected health datasets is a valuable research resource, most datasets are established for administrative purposes and not for health outcomes research. In order for meaningful results to be extrapolated to specific populations, the limitations of the data and linkage methodology need to be investigated and clarified. It is the objective of this study to investigate the differences in ascertainment which may arise between a hospital admission dataset and a dispensing claims dataset, using major depression in pregnancy as an example. The safe use of antidepressants in pregnancy is an ongoing issue for clinicians with around 10% of pregnant women suffer from depression. As the birth admission will be the first admission to hospital during their pregnancy for most women, their use of antidepressants, or their depressive condition, may not be revealed to the attending hospital clinicians. This may result in adverse outcomes for the mother and infant.", "Population-based de-identified data were provided from the Western Australian Data Linkage System linking the administrative health records of women with a delivery to related records from the Midwives' Notification System, the Hospital Morbidity Data System and the national Pharmaceutical Benefits Scheme dataset. The women with depression during their pregnancy were ascertained in two ways: women with dispensing records relating to dispensed antidepressant medicines with an WHO ATC code to the 3rd level, pharmacological subgroup, 'N06A Antidepressants'; and, women with any hospital admission during pregnancy, including the birth admission, if a comorbidity was recorded relating to depression.", "From 2002 to 2005, there were 96698 births in WA. At least one antidepressant was dispensed to 4485 (4.6%) pregnant women. There were 3010 (3.1%) women with a comorbidity related to depression recorded on their delivery admission, or other admission to hospital during pregnancy. There were a total of 7495 pregnancies identified by either set of records. Using data linkage, we determined that these records represented 6596 individual pregnancies. Only 899 pregnancies were found in both groups (13.6% of all cases). 80% of women dispensed an antidepressant did not have depression recorded as a comorbidity on their hospital records. A simple capture-recapture calculation suggests the prevalence of depression in this population of pregnant women to be around 16%." ], "LABELS": [ "BACKGROUND", "METHODS", "RESULTS" ], "MESHES": [ "Adult", "Antidepressive Agents", "Australia", "Databases, Factual", "Depressive Disorder, Major", "Female", "Hospital Records", "Humans", "Longitudinal Studies", "Medical Record Linkage", "Medical Records Systems, Computerized", "Pregnancy", "Pregnancy Complications", "Prevalence" ], "YEAR": "2013", "reasoning_required_pred": "no", "reasoning_free_pred": "no", "final_decision": "no", "LONG_ANSWER": "No single data source is likely to provide a complete health profile for an individual. For women with depression in pregnancy and dispensed antidepressants, the hospital admission data do not adequately capture all cases." }, "21952349": { "QUESTION": "Remote ischemic postconditioning: does it protect against ischemic damage in percutaneous coronary revascularization?", "CONTEXTS": [ "Myocardial damage that is associated with percutaneous coronary intervention (PCI) partially affects the results of the procedure, and is related to medium-term cardiovascular death. Remote postischemic conditioning might reduce the myocardial lesions that are associated with PCI, but perhaps less so in diabetics. The aim of this study was to evaluate the protective effect of remote postischemic conditioning in patients undergoing elective PCI for stable angina or non-ST elevation acute coronary syndrome with troponin<1 ng/ml at the time of randomization.", "This randomized single-blinded single-center clinical trial involved 320 patients undergoing elective PCI who were randomized to either receive three 5-min cycles of ischemia by inflation of a cuff on the non-dominant arm to 200 mm Hg (remote postischemic conditioning) or to placebo (uninflated cuff). The primary outcome variable was the maximum increase in troponin in the first 24 h. The secondary outcome variable was readmission due to heart failure or cardiovascular mortality after 1 year of follow-up. In addition, a diabetic population was studied." ], "LABELS": [ "OBJECTIVE", "METHODS" ], "MESHES": [ "Analysis of Variance", "Angioplasty, Balloon, Coronary", "Confidence Intervals", "Coronary Angiography", "Elective Surgical Procedures", "Female", "Humans", "Ischemic Postconditioning", "Kaplan-Meier Estimate", "Logistic Models", "Male", "Middle Aged", "Multivariate Analysis", "Myocardial Infarction", "Myocardial Ischemia", "Patient Selection", "Prognosis", "Prospective Studies", "Reference Values", "Risk Assessment", "Severity of Illness Index", "Single-Blind Method", "Survival Analysis", "Treatment Outcome" ], "YEAR": "2011", "reasoning_required_pred": "maybe", "reasoning_free_pred": "yes", "final_decision": "yes", "LONG_ANSWER": "This clinical trial evaluated the possible reduction in intervention-related myocardial damage that was attributable to remote postischemic conditioning." }, "21712147": { "QUESTION": "Does combining antiretroviral agents in a single dosage form enhance quality of life of HIV/AIDS patients?", "CONTEXTS": [ "Combining various antiretroviral agents into one single dosage form has been a strategy to reduce pill burden and enhance medication adherence among human immunodeficiency virus /AIDS (HIV/AIDS) patients.", "This is a cost-utility study from a health care system's perspective comparing coformulated fixed dose (FXD) strategy versus multiple free dose combination (FRC) in antiretroviral therapy.", "The Medical Expenditure Panel Survey (MEPS) was used to identify HIV/AIDS patients with \u22652 active antiretroviral medications. Patients on FXD were matched in 1:1 ratio with the FRC group using propensity scores. All medical costs excluding those paid by patients and families were included. Utility was measured using SF-6D scores from the SF-12 questionnaire. Incremental cost-utility ratios (ICURs) were calculated using the mean annual estimates. A cost-effectiveness acceptability curve was determined using a Monte Carlo probabilistic simulation technique.", "Nine FXD antiretroviral formulations approved by the U.S. Food and Drug Administration by 2005 was included in this study. One hundred seventy HIV/AIDS patients with \u22652 antiretroviral agents were identified from the MEPS database, of which 53% (n=92) were on FXD formulation. On matching, 70 patients from FXD had a match from the FRC group. No differences in sociodemographic and health status variables were observed between the matched groups. The mean annual cost was $15,766.15 for FXD patients and $11,875.21 for FRC patients. The mean utility gained by using FXD over FRC was 0.085; however, this difference was not statistically significant. The ICUR for the FXD treatment over FRC treatment was $45,540.49/quality-adjusted life years (QALYs). Probabilistic sensitivity analysis showed FXD to dominate FRC (>50% probability of being cost-effective) above the $40,000 threshold." ], "LABELS": [ "BACKGROUND", "OBJECTIVES", "METHOD", "RESULTS" ], "MESHES": [ "Adolescent", "Adult", "Aged", "Anti-Retroviral Agents", "Cost-Benefit Analysis", "Drug Combinations", "Drug Costs", "Female", "HIV Infections", "Humans", "Male", "Middle Aged", "Quality of Life", "Quality-Adjusted Life Years", "Young Adult" ], "YEAR": null, "reasoning_required_pred": "yes", "reasoning_free_pred": "no", "final_decision": "no", "LONG_ANSWER": "Although the cost-effectiveness of a single-pill strategy was within the acceptable willingness-to-pay threshold, the QALY difference were minimal. Further research is recommended to explore the long-term impact of the strategy." }, "25311479": { "QUESTION": "The inverse equity hypothesis: does it apply to coverage of cancer screening in middle-income countries?", "CONTEXTS": [ "It is uncertain whether the inverse equity hypothesis-the idea that new health interventions are initially primarily accessed by the rich, but that inequalities narrow with diffusion to the poor-holds true for cancer screening in low and middle income countries (LMICs).This study examines the relationship between overall coverage and economic inequalities in coverage of cancer screening in four middle-income countries.", "Secondary analyses of cross-sectional data from the WHO study on Global Ageing and Adult Health in China, Mexico, Russia and South Africa (2007-2010). Three regression-based methods were used to measure economic inequalities: (1) Adjusted OR; (2) Relative Index of Inequality (RII); and (3) Slope Index of Inequality.", "Coverage for breast cancer screening was 10.5% in South Africa, 19.3% in China, 33.8% in Russia and 43% in Mexico, and coverage for cervical cancer screening was 24% in South Africa, 27.2% in China, 63.7% in Mexico and 81.5% in Russia. Economic inequalities in screening participation were substantially lower or non-existent in countries with higher aggregate coverage, for both breast cancer screening (RII: 14.57 in South Africa, 4.90 in China, 2.01 in Mexico, 1.04 in Russia) and cervical cancer screening (RII: 3.60 in China, 2.47 in South Africa, 1.39 in Mexico, 1.12 in Russia)." ], "LABELS": [ "BACKGROUND", "METHODS", "RESULTS" ], "MESHES": [ "Adult", "Aged", "Breast Neoplasms", "China", "Cross-Cultural Comparison", "Cross-Sectional Studies", "Developing Countries", "Early Detection of Cancer", "Female", "Health Services Accessibility", "Humans", "Mexico", "Middle Aged", "Regression Analysis", "Russia", "Social Class", "South Africa", "Uterine Cervical Neoplasms" ], "YEAR": "2015", "reasoning_required_pred": "maybe", "reasoning_free_pred": "yes", "final_decision": "yes", "LONG_ANSWER": "Economic inequalities in breast and cervical cancer screening are low in LMICs with high screening coverage. These findings are consistent with the inverse equity hypothesis and indicate that high levels of equity in cancer screening are feasible even in countries with high income inequality." }, "20602784": { "QUESTION": "Identification of racial disparities in breast cancer mortality: does scale matter?", "CONTEXTS": [ "This paper investigates the impact of geographic scale (census tract, zip code, and county) on the detection of disparities in breast cancer mortality among three ethnic groups in Texas (period 1995-2005). Racial disparities were quantified using both relative (RR) and absolute (RD) statistics that account for the population size and correct for unreliable rates typically observed for minority groups and smaller geographic units. Results were then correlated with socio-economic status measured by the percentage of habitants living below the poverty level.", "African-American and Hispanic women generally experience higher mortality than White non-Hispanics, and these differences are especially significant in the southeast metropolitan areas and southwest border of Texas. The proportion and location of significant racial disparities however changed depending on the type of statistic (RR versus RD) and the geographic level. The largest proportion of significant results was observed for the RD statistic and census tract data. Geographic regions with significant racial disparities for African-Americans and Hispanics frequently had a poverty rate above 10.00%." ], "LABELS": [ "BACKGROUND", "RESULTS" ], "MESHES": [ "Adult", "African Americans", "Age Distribution", "Aged", "Breast Neoplasms", "Cause of Death", "Censuses", "Confidence Intervals", "Cross-Sectional Studies", "Databases, Factual", "European Continental Ancestry Group", "Female", "Health Knowledge, Attitudes, Practice", "Health Status Disparities", "Hispanic Americans", "Humans", "Incidence", "Logistic Models", "Middle Aged", "Odds Ratio", "Risk Assessment", "Socioeconomic Factors", "Survival Analysis", "Texas" ], "YEAR": "2010", "reasoning_required_pred": "yes", "reasoning_free_pred": "yes", "final_decision": "yes", "LONG_ANSWER": "This study investigates both relative and absolute racial disparities in breast cancer mortality between White non-Hispanic and African-American/Hispanic women at the census tract, zip code and county levels. Analysis at the census tract level generally led to a larger proportion of geographical units experiencing significantly higher mortality rates for minority groups, although results varied depending on the use of the relative versus absolute statistics. Additional research is needed before general conclusions can be formulated regarding the choice of optimal geographic regions for the detection of racial disparities." }, "12836106": { "QUESTION": "Does the leukocyte count correlate with the severity of injury?", "CONTEXTS": [ "Injury severity score (ISS), Glasgow coma score (GCS), and revised trauma score (RTS) are the most frequently used methods to evaluate the severity of injury in blunt trauma patients. ISS is too complicated to assess easily and GCS and RTS are easy to assess but somewhat subjective. White blood cell count (WBC) is an easy, quick and objective test. This study was performed to evaluate the significance of the WBC count at presentation in the blunt trauma patients.", "713 blunt trauma patients, who were admitted to the Uludag University Medical Center Emergency Department between 01.04.2000-31.12.2000, were retrospectively evaluated in terms of ISS, GCS, RTS and white blood cell count at presentation.", "Statistical analysis revealed that WBC was correlated positively with ISS, but negatively with GCS and RTS." ], "LABELS": [ "BACKGROUND", "METHODS", "RESULTS" ], "MESHES": [ "Glasgow Coma Scale", "Humans", "Injury Severity Score", "Leukocyte Count", "Predictive Value of Tests", "Trauma Severity Indices", "Wounds, Nonpenetrating" ], "YEAR": "2003", "reasoning_required_pred": "yes", "reasoning_free_pred": "yes", "final_decision": "yes", "LONG_ANSWER": "The leukocyte count at presentation can be used as an adjunct in the evaluation of the severity of injury in blunt trauma patients." }, "25588461": { "QUESTION": "Can transcranial direct current stimulation be useful in differentiating unresponsive wakefulness syndrome from minimally conscious state patients?", "CONTEXTS": [ "Disorders of consciousness (DOC) diagnosis relies on the presence or absence of purposeful motor responsiveness, which characterizes the minimally conscious state (MCS) and the unresponsive wakefulness syndrome (UWS), respectively. Functional neuroimaging studies have raised the question of possible residual conscious awareness also in clinically-defined UWS patients. The aim of our study was to identify electrophysiological parameters, by means of a transcranial magnetic stimulation approach, which might potentially express the presence of residual networks sustaining fragmentary behavioral patterns, even when no conscious behavior can be observed.", "We enrolled 25 severe DOC patients, following post-anoxic or traumatic brain injury and 20 healthy individuals (HC) as control group. Baseline electrophysiological evaluation evidenced, in comparison to HC, a partial preservation of cortical effective connectivity and excitability in clinically defined MCS, whereas these components were absent in clinically defined UWS. Then, we applied an anodal transcranial direct current stimulation (a-tDCS) protocol over the orbitofrontal cortex.", "a-tDCS was able to boost cortical connectivity and excitability in all HC, MCS, and to unmask such excitability/connectivity in some UWS patients." ], "LABELS": [ "PURPOSE", "METHODS", "RESULT" ], "MESHES": [ "Adult", "Aged", "Consciousness Disorders", "Diagnosis, Differential", "Female", "Humans", "Male", "Middle Aged", "Neural Pathways", "Persistent Vegetative State", "Prefrontal Cortex", "Transcranial Direct Current Stimulation" ], "YEAR": "2015", "reasoning_required_pred": "yes", "reasoning_free_pred": "yes", "final_decision": "yes", "LONG_ANSWER": "a-tDCS could be useful in identifying residual connectivity markers in clinically-defined UWS, who may lack of purposeful behavior as a result of a motor-output failure." }, "19299238": { "QUESTION": "Aromatase inhibitor-related musculoskeletal symptoms: is preventing osteoporosis the key to eliminating these symptoms?", "CONTEXTS": [ "Aromatase inhibitors (AIs) are an effective treatment for postmenopausal women with hormone receptor-positive breast cancer. However, patients receiving AIs report a higher incidence of musculoskeletal symptoms and bone fractures; the mechanism and risk factors for this correlation are not well studied. The aim of this study was to correlate these musculoskeletal symptoms and bone fractures in patients receiving AIs with bone mineral density (BMD), previous tamoxifen use, and administration of calcium/bisphosphonate (Ca/Bis).", "We reviewed charts of 856 patients with hormone receptor-positive nonmetastatic breast cancer seen at our institution between January 1999 and October 2007. A total of 316 patients met the inclusion criteria of treatment with one of the AIs for>or = 3 months and availability of a dualenergy X-ray absorptiometry (DEXA) during this treatment. Arthralgia, generalized bone pain and/or myalgia, bone fracture after beginning AIs, any tamoxifen treatment, and Ca/Bis therapy were recorded.", "Our study demonstrates a significant association between symptoms and DEXA-BMD results (P<.001). Similarly, the group receiving tamoxifen before AIs had fewer patients with arthralgia or generalized bone pain/myalgia or bone fracture (P<.001). Furthermore, the group receiving AIs plus Ca/Bis had more patients without musculoskeletal symptoms and had fewer fractures. Finally, the group receiving steroidal AIs compared with nonsteroidal AIs had more patients with arthralgia or generalized bone pain and/or myalgia, and bone fractures (P<.001)." ], "LABELS": [ "BACKGROUND", "PATIENTS AND METHODS", "RESULTS" ], "MESHES": [ "Absorptiometry, Photon", "Aged", "Antineoplastic Agents, Hormonal", "Aromatase Inhibitors", "Bone Density", "Breast Neoplasms", "Calcium Phosphates", "Diphosphonates", "Female", "Humans", "Middle Aged", "Musculoskeletal Diseases", "Osteoporosis", "Retrospective Studies", "Tamoxifen" ], "YEAR": "2009", "reasoning_required_pred": "yes", "reasoning_free_pred": "yes", "final_decision": "yes", "LONG_ANSWER": "Patients on AIs who develop osteoporosis are at increased risk of musculoskeletal symptoms and bone fracture. Comedication with Ca/Bis reduces the likelihood for osteoporosis and musculoskeletal symptoms. Patients who received tamoxifen before AIs were less likely to develop AI-related musculoskeletal symptoms. We recommend that patients on AIs should be offered Ca/Bis to reduce the incidence of musculoskeletal symptoms and fracture, especially if patients are receiving steroidal AI and/or did not receive tamoxifen before AIs." }, "22706226": { "QUESTION": "Could different follow-up modalities play a role in the diagnosis of asymptomatic endometrial cancer relapses?", "CONTEXTS": [ "To determine current practice and to assess the value of routine follow-up procedures for endometrial cancer surveillance. To discuss whether such procedures are feasible and effective to identify asymptomatic recurrences and describe the pattern of relapse detected by procedures.", "The records of 282 consecutive women with recurrent endometrial cancer treated from 1986 to 2005 were retrospectively collected in 8 Italian institutions. Primary disease, clinical history, and recurrence features and data were analyzed.", "Thirty-five (12.4%) of 282 patients had recurrence in vaginal vault, 51 patients (18.0%) had recurrence in central pelvis, 14 patients (4.9%) had recurrence in pelvic wall, and 39 patients (13.8%) had recurrence in lymph nodes. One-hundred twenty-eight patients (45.3%) showed a distant relapse, whereas 15 patients (5.3%) developed both distant relapse and local relapse. The site of relapse influenced survival because the patients with vaginal vault recurrences lived significantly longer than the patients with recurrences in other sites. Eighty (28.4%) of the 282 patients became symptomatic and anticipated the scheduled visit, 37 (13.1 %) of the patients reported their symptoms during the follow-up meeting, and 165 (58.5 %) of the patients were asymptomatic and the diagnostic path was introduced by a planned visit or examination. Among the asymptomatic patients, the first procedure that led to further examinations was clinical visit alone for 60 (36.4%) of 165 patients, imaging for 103 patients (62.4%), and cytologic examination for 2 patients (1.2%). Symptoms at recurrence can predict survival: patients with an asymptomatic recurrence had a median survival time from relapse of 35 months versus 13 months if they had a symptomatic repetition (P = 0.0001)." ], "LABELS": [ "OBJECTIVE", "METHODS", "RESULTS" ], "MESHES": [ "Adult", "Aged", "Aged, 80 and over", "Carcinoma", "Endometrial Neoplasms", "Female", "Humans", "Italy", "Middle Aged", "Neoplasm Recurrence, Local", "Retrospective Studies" ], "YEAR": "2012", "reasoning_required_pred": "maybe", "reasoning_free_pred": "maybe", "final_decision": "maybe", "LONG_ANSWER": "Follow-up after endometrial cancer treatment varies in Italy. In this retrospective study, women with asymptomatic recurrence have shown a better clinical outcome compared with those with symptomatic relapse. The optimal approach is actually unknown, and guidelines comparing follow-up protocols have not been established. Prospective cost-effectiveness studies are needed." }, "12913878": { "QUESTION": "Locoregional opening of the rodent blood-brain barrier for paclitaxel using Nd:YAG laser-induced thermo therapy: a new concept of adjuvant glioma therapy?", "CONTEXTS": [ "Nd:YAG laser-induced thermo therapy (LITT) of rat brains is associated with blood-brain barrier (BBB) permeability changes. We address the question of whether LITT-induced locoregional disruption of the BBB could possibly allow a locoregional passage of chemotherapeutic agents into brain tissue to treat malignant glioma.STUDY DESIGN/", "CD Fischer rats were subject to LITT of the left forebrain. Disruption of the BBB was analyzed using Evans blue and immunohistochemistry (IH). Animals were perfused with paclitaxel, and high-pressure liquid chromatography (HPLC) was employed to analyze the content of paclitaxel in brain and plasma samples.", "LITT induces an opening of the BBB as demonstrated by locoregional extravasation of Evans blue, C3C, fibrinogen, and IgM. HPLC proved the passage of paclitaxel across the disrupted BBB." ], "LABELS": [ "BACKGROUND AND OBJECTIVES", "MATERIALS AND METHODS", "RESULTS" ], "MESHES": [ "Administration, Topical", "Animals", "Antineoplastic Agents, Phytogenic", "Blood-Brain Barrier", "Brain Neoplasms", "Chemotherapy, Adjuvant", "Glioma", "Hyperthermia, Induced", "Laser Therapy", "Neurosurgical Procedures", "Paclitaxel", "Permeability", "Rats", "Rats, Inbred F344" ], "YEAR": "2003", "reasoning_required_pred": "yes", "reasoning_free_pred": "yes", "final_decision": "yes", "LONG_ANSWER": "LITT induces a locoregional passage of chemotherapeutic agents into the brain tissue. This is of potential interest for the treatment of brain tumors." }, "16769333": { "QUESTION": "Preoperative tracheobronchoscopy in newborns with esophageal atresia: does it matter?", "CONTEXTS": [ "Despite surgical refinements, perioperative use of tracheobronchoscopy (TBS) as part of surgical approach to esophageal atresia (EA) is still controversial. The purpose of this study was to evaluate the influence of preoperative TBS in newborns with EA in preventing complications and improving diagnosis and surgical treatment.", "In the period ranging from 1997 to 2003, 62 patients with EA underwent preoperative TBS. The procedure was carried out with flexible bronchoscope maintaining spontaneous breathing. When a wide carinal fistula was found, this was mechanically occluded by Fogarty catheter and cannulated with rigid bronchoscopy. Type of EA, surgical procedure variations caused by TBS, and associated anomalies not easily detectable were recorded.", "Before TBS, the Gross classification of the 62 patients was as follows: type A, 9 patients; type B, none; type C, 51 patients. At TBS, however, 3 of 9 type A patients had an unsuspected proximal fistula (type B). These 3 patients, plus the 2 with H-type fistula, were repaired through a cervical approach. In 4 patients, previously undetected malformations of the respiratory tree (2 aberrant right upper bronchus and 2 hypoplastic bronchi) were found at TBS. Carinal fistulas in 14 type C patients were occluded by Fogarty catheter to improve ventilation during repair. No complications were observed. Overall, TBS was clinically useful in 28 (45.2%) of 62 patients, including 15 (24.2%) of 62 infants in whom it was crucial in modifying the surgical approach." ], "LABELS": [ "PURPOSE", "METHODS", "RESULTS" ], "MESHES": [ "Bronchoscopy", "Digestive System Surgical Procedures", "Endoscopy", "Esophageal Atresia", "Female", "Humans", "Infant, Newborn", "Male", "Postoperative Complications", "Preoperative Care", "Respiratory Tract Fistula", "Retrospective Studies", "Trachea" ], "YEAR": "2006", "reasoning_required_pred": "yes", "reasoning_free_pred": "yes", "final_decision": "yes", "LONG_ANSWER": "Tracheobronchoscopy is a useful and safe procedure and should be recommended in tertiary centers for babies with EA before surgical repair." }, "22497340": { "QUESTION": "Is horizontal semicircular canal ocular reflex influenced by otolith organs input?", "CONTEXTS": [ "To clarify whether horizontal canal ocular reflex is influenced by otolith organs input.", "The subjects were seven healthy humans. The right ear was stimulated using ice-water. Each subject was kept in a left-ear-down position for 20 s and then repositioned to a prone position, a right-ear-down position and a supine position with 20 s intervals. Nystagmus was analysed using three-dimensional video-oculography.", "Eye movements in the supine position and the prone position were not in a symmetric fashion. Nystagmus in the left-ear-down position and the right-ear-down position were not symmetric either. These phenomena indicate that the axis of the eyeball rotation was affected by the shift of the direction of gravity exerted on the head." ], "LABELS": [ "OBJECTIVE", "METHODS", "RESULTS" ], "MESHES": [ "Adult", "Head Movements", "Humans", "Male", "Nystagmus, Physiologic", "Otolithic Membrane", "Physical Stimulation", "Prone Position", "Reference Values", "Reflex, Vestibulo-Ocular", "Semicircular Canals", "Supine Position", "Young Adult" ], "YEAR": "2012", "reasoning_required_pred": "yes", "reasoning_free_pred": "yes", "final_decision": "yes", "LONG_ANSWER": "Otolith organs input influences the axis of horizontal semicircular canal ocular reflex; therefore, the plane of compensatory eye movements induced by the horizontal canal stimulation is not always parallel to the canal." }, "10223070": { "QUESTION": "Is perforation of the appendix a risk factor for tubal infertility and ectopic pregnancy?", "CONTEXTS": [ "To critically assess the evidence that appendiceal perforation is a risk factor for subsequent tubal infertility or ectopic pregnancy.", "Epidemiologic studies investigating the relationship between appendectomy and infertility or ectopic pregnancy were identified by searching the MEDLINE database from 1966 to 1997. Appropriate citations were also extracted from a manual search of the bibliographies of selected papers.", "Twenty-three articles were retrieved. Only 4 presented original data including comparisons to a nonexposed control group and they form the basis for this study.", "Because the raw data or specific techniques of data analysis were not always explicitly described, indices of risk for exposure were extracted from the data as presented and were analysed without attempting to convert them to a common measure.", "Articles were assessed according to the criteria of the Evidence-Based Medicine Working Group for evaluating articles on harm. Review of the literature yielded estimates of the risk of adverse fertility outcomes ranging from 1.6 (95% confidence interval [CI] 1.1 to 2.5) for ectopic pregnancy after an appendectomy to 4.8 (95% CI 1.5 to 14.9) for tubal infertility from perforation of the appendix. Recall bias, and poor adjustment for confounding variables in some reports, weakened the validity of the studies." ], "LABELS": [ "OBJECTIVE", "DATA SOURCES", "STUDY SELECTION", "DATA EXTRACTION", "DATA SYNTHESIS" ], "MESHES": [ "Appendicitis", "Causality", "Evidence-Based Medicine", "Female", "Humans", "Infertility, Female", "Intestinal Perforation", "Pregnancy", "Pregnancy, Ectopic", "Research Design", "Risk Factors" ], "YEAR": "1999", "reasoning_required_pred": "yes", "reasoning_free_pred": "no", "final_decision": "maybe", "LONG_ANSWER": "The methodologic weaknesses of the studies do not permit acceptance of increased risk of tubal pregnancy or infertility as a consequence of perforation of the appendix, so a causal relationship cannot be supported by the data currently available. Only a well-designed case-control study with unbiased ascertainment of exposure and adjustment for confounding variables will provide a definitive answer." }, "8566975": { "QUESTION": "Serovar specific immunity to Neisseria gonorrhoeae: does it exist?", "CONTEXTS": [ "To determine whether the host immune response to gonorrhoea provides limited serovar specific protection from reinfection.", "508 episodes of gonorrhoea diagnosed at a city centre genitourinary medicine clinic including 22 patients with multiple infections over a 4 year period.", "Patients with recurrent gonococcal infection were analysed with respect to the initial and subsequent serovars isolated.", "No significant difference was seen in the prevalence of serovars isolated following a repeat infection compared with those without repeat infections. The site of the initial infection did not appear to influence the subsequent serovar isolated." ], "LABELS": [ "OBJECTIVE", "SUBJECTS", "METHODS", "RESULTS" ], "MESHES": [ "Adolescent", "Adult", "Antibodies, Bacterial", "Antibody Specificity", "Female", "Gonorrhea", "Humans", "Male", "Neisseria gonorrhoeae", "Recurrence", "Serotyping", "Sexual Behavior" ], "YEAR": "1995", "reasoning_required_pred": "no", "reasoning_free_pred": "no", "final_decision": "no", "LONG_ANSWER": "We found no evidence of serovar specific immunity in our population. It remains possible that populations with a higher prevalence of gonorrhoea and more frequent infections may have a quantitatively greater immune response." }, "25186850": { "QUESTION": "May student examiners be reasonable substitute examiners for faculty in an undergraduate OSCE on medical emergencies?", "CONTEXTS": [ "To compare the effect of student examiners (SE) to that of faculty examiners (FE) on examinee performance in an OSCE as well as on post-assessment evaluation in the area of emergency medicine management.", "An OSCE test-format (seven stations: Advanced Cardiac Life Support (ACLS), Basic Life Support (BLS), Trauma-Management (TM), Pediatric-Emergencies (PE), Acute-Coronary-Syndrome (ACS), Airway-Management (AM), and Obstetrical-Emergencies (OE)) was administered to 207 medical students in their third year of training after they had received didactics in emergency medicine management. Participants were randomly assigned to one of the two simultaneously run tracks: either with SE (n = 110) or with FE (n = 98). Students were asked to rate each OSCE station and to provide their overall OSCE perception by means of a standardized questionnaire. The independent samples t-test was used and effect sizes were calculated (Cohens d).", "Students achieved significantly higher scores for the OSCE stations \"TM\", \"AM\", and \"OE\" as well as \"overall OSCE score\" in the SE track, whereas the station score for \"PE\" was significantly higher for students in the FE track. Mostly small effect sizes were reported. In the post-assessment evaluation portion of the study, students gave significant higher ratings for the ACS station and \"overall OSCE evaluation\" in the FE track; also with small effect sizes." ], "LABELS": [ "OBJECTIVES", "METHODS", "RESULTS" ], "MESHES": [ "Adult", "Clinical Competence", "Education, Medical, Undergraduate", "Educational Measurement", "Emergency Medicine", "Faculty, Medical", "Female", "Humans", "Male", "Medical History Taking", "Patient Simulation", "Physical Examination", "Students, Medical", "Young Adult" ], "YEAR": "2015", "reasoning_required_pred": "yes", "reasoning_free_pred": "yes", "final_decision": "yes", "LONG_ANSWER": "It seems quite admissible and justified to encourage medical students to officiate as examiners in undergraduate emergency medicine OSCE formative testing, but not necessarily in summative assessment evaluations." }, "9427037": { "QUESTION": "Are endothelial cell patterns of astrocytomas indicative of grade?", "CONTEXTS": [ "The most common primary brain tumors in children and adults are of astrocytic origin. Classic histologic grading schemes for astrocytomas have included evaluating the presence or absence of nuclear abnormalities, mitoses, vascular endothelial proliferation, and tumor necrosis.", "We evaluated the vascular pattern of 17 astrocytoma surgical specimens (seven from children and 10 from adults), and four normal brains obtained at autopsy, utilizing antibody to glial fibrillary acidic protein (GFAP) and von Willebrand factor (vWF) utilizing confocal microscopy. A modified WHO classification was used.", "All tumor cases showed cells positive for GFAP. Control tissues showed a few, widely separated vessels. Pilocytic astrocytomas (four cases) showed lacy clusters of small-to-medium sized vessels, with intact vessel wall integrity. Diffuse, low grade astrocytoma (three cases) showed a staining pattern similar to control tissue; intermediate grade (one case), anaplastic astrocytoma (three cases) and gliobastoma multiforme (six cases) showed an increased vessel density with multiple small vessels (glomeruloid clusters), some with prominent intimal hyperplasia, loss of vessel wall integrity, and with numerous vWF-positive single cells/microvessels within the tumor substance." ], "LABELS": [ "BACKGROUND", "MATERIALS AND METHODS", "RESULTS" ], "MESHES": [ "Adolescent", "Adult", "Astrocytoma", "Child, Preschool", "Endothelium, Vascular", "Female", "Fluorescent Antibody Technique, Indirect", "Glial Fibrillary Acidic Protein", "Humans", "Infant", "Male", "von Willebrand Factor" ], "YEAR": null, "reasoning_required_pred": "yes", "reasoning_free_pred": "yes", "final_decision": "yes", "LONG_ANSWER": "Evaluation of astrocytomas utilizing antibody to vWF and confocal microscopy aids in the grading of these neoplasms." }, "28359277": { "QUESTION": "Do healthier lifestyles lead to less utilization of healthcare resources?", "CONTEXTS": [ "Governments are urged to determine methods to control the use of medical resources and curb the rise of healthcare costs. The question is, do health behaviors have an impact on the use of medical resources? This study aims to identify and understand the difference in the number of outpatient visits and health examinations based on various health behaviors and to determine whether patients seek medical care for illness from the same physicians.", "This study used the dataset derived from the Department of Budget, Accounting and Statistics of Kaohsiung, Taiwan in 2005. Persons older than 15\u00a0years were surveyed using an on-site questionnaire. A total of 2911 persons were enrolled in this study. Independent t-tests, chi-square tests, one-way ANOVA, multiple linear regression and binominal logistic regression were used in the data analysis.", "The regression model for the frequency of doctor visits, health examinations, and whether the same physician is sought for medical care has demonstrated significant correlations with gender, age and education-level variables. Four health behaviors (i.e., exercise habits, dietary habits, regular blood pressure measurement, drinking habits) exhibited a significant correlation with healthcare utilization (P<0.05)." ], "LABELS": [ "BACKGROUND", "METHODS", "RESULTS" ], "MESHES": [ "Adult", "Aged", "Alcohol Drinking", "Exercise", "Female", "Health Behavior", "Health Care Costs", "Health Services Accessibility", "Healthy Lifestyle", "Humans", "Logistic Models", "Male", "Middle Aged", "Patient Acceptance of Health Care", "Preventive Health Services", "Surveys and Questionnaires", "Taiwan" ], "YEAR": "2017", "reasoning_required_pred": "yes", "reasoning_free_pred": "maybe", "final_decision": "no", "LONG_ANSWER": "Healthy lifestyles lead to an increase in the utilization of preventive health services. However, there is not much significantly reducing the number of outpatient visits in people with health behaviors. Specifically, people with regular exercise habits and who take their blood pressure measurement regularly have an increased number of outpatient visits. It is suggested that more available and accessible health consultation services be provided to inculcate in the general public the importance of maintaining a healthy lifestyle." }, "11411430": { "QUESTION": "Antral follicle assessment as a tool for predicting outcome in IVF--is it a better predictor than age and FSH?", "CONTEXTS": [ "The purpose of this study is to determine if baseline antral follicle assessment may serve as additional information in predicting in vitro fertilization outcome.", "Prospective, descriptive preliminary study of in vitro fertilization outcome. From July 1998 to July 1999, 224 patients underwent antral follicle assessment (follicle 2-6 mm in diameter) on baseline of the planned, stimulated in vitro fertilization cycle. The outcomes were analyzed with respect to antral follicle assessment (6), basal cycle day 3 follicle stimulated hormone (10 IU/L) and maternal age (35 years).", "The clinical pregnancy rate was significantly higher in the group with baseline antral follicle>6 compared to that in the group with antral follicle6 compared to that in the group with antral follicleor = 6 (33% vs. 1%, respectively)." ], "LABELS": [ "PURPOSE", "METHODS", "RESULTS" ], "MESHES": [ "Adult", "Age Factors", "Female", "Fertilization in Vitro", "Follicle Stimulating Hormone", "Humans", "Logistic Models", "Male", "Ovarian Follicle", "Ovulation Induction", "Pilot Projects", "Predictive Value of Tests", "Pregnancy", "Pregnancy Outcome", "Prospective Studies", "Ultrasonography" ], "YEAR": "2001", "reasoning_required_pred": "maybe", "reasoning_free_pred": "yes", "final_decision": "maybe", "LONG_ANSWER": "In vitro fertilization outcome is strongly correlated with both maternal ages, basal cycle, day 3 follicle, stimulated hormone, and antral follicle assessment. Antral follicle assessment was a better predictor of in vitro fertilization outcome than were age or follicle stimulated hormone. Antral follicle assessment may provide a marker for ovarian age that is distinct from chronological age or hormonal markers." }, "16956164": { "QUESTION": "Do all ethnic groups in New Zealand exhibit socio-economic mortality gradients?", "CONTEXTS": [ "First, to establish whether a deprivation gradient in all-cause mortality exists for all ethnic groups within New Zealand; second, if such gradients do exist, whether their absolute slopes are the same; and third, if such gradients exist, what impact the unequal deprivation distributions of the different ethnic groups have on the observed ethnic inequalities in life expectancy at birth.", "Abridged lifetables for the period 1999-2003 were constructed using standard demographic methods for each of four ethnic groups (Asian, Pacific, Maori and European) by NZDep2001 quintile and sex. Gradients were estimated by fitting generalised linear models to the quintile-specific life expectancy estimates for each ethnic group (by sex). The contribution of variation in deprivation distributions to inter-ethnic inequalities in life expectancy was estimated by re-weighting the quintile-specific mortality rates for each ethnic group using weights derived from the European deprivation distribution and recalculating the lifetable.", "All four ethnic groups exhibit deprivation gradients in all-cause mortality (life expectancy). Maori show the steepest gradients, with slopes approximately 25% steeper than those of Europeans for both males and females. By contrast, gradients among Asian and Pacific peoples are shallower than those of their European counterparts." ], "LABELS": [ "OBJECTIVES", "METHOD", "RESULTS" ], "MESHES": [ "Adolescent", "Adult", "Ethnic Groups", "Female", "Humans", "Male", "Middle Aged", "Mortality", "New Zealand", "Social Class", "Vital Statistics" ], "YEAR": "2006", "reasoning_required_pred": "yes", "reasoning_free_pred": "yes", "final_decision": "yes", "LONG_ANSWER": "While socio-economic gradients in health exist among all ethnic groups, they are relatively shallow among Pacific and (especially) Asian peoples. For these ethnic groups, caution should be exercised in applying deprivation or other socio-economic measures as proxy indicators of need for health services." }, "15687156": { "QUESTION": "Can normal knee kinematics be restored with unicompartmental knee replacement?", "CONTEXTS": [ "Unicompartmental replacement can be an alternative to tibial osteotomy in younger, active patients with unicompartmental knee disease. In unicompartmental replacement, the other compartments and knee ligaments are largely untouched. Therefore, it was hypothesized that the knee kinematics after unicompartmental replacement may also be unchanged. To test this hypothesis, knee kinematics and quadriceps tension were recorded before and after replacement with a unicompartmental design and then with a tricompartmental design.", "Six human cadaver knees were tested before implantation, after implantation with a bicruciate-retaining unicompartmental knee prosthesis, and after implantation with a posterior cruciate-retaining tricompartmental knee prosthesis. The unicompartmental prosthesis was initially implanted, and it was then revised to a total condylar knee replacement. The knee kinematics were measured with use of an electromagnetic tracking device while the knee was put through dynamic simulated stair-climbing under peak flexion moments of approximately 40 N-m. Quadriceps tension was also measured for all three conditions.", "No significant differences in tibial axial rotation were noted between the intact and unicompartmental conditions. However, tricompartmental replacement significantly affected tibial axial rotation (p = 0.001). Femoral rollback was not significantly affected by either unicompartmental or tricompartmental arthroplasty. Quadriceps tension was also similar among all three conditions." ], "LABELS": [ "BACKGROUND", "METHODS", "RESULTS" ], "MESHES": [ "Aged", "Aged, 80 and over", "Arthroplasty, Replacement, Knee", "Biomechanical Phenomena", "Cadaver", "Compartment Syndromes", "Female", "Humans", "Knee Joint", "Knee Prosthesis", "Leg", "Male", "Muscle, Skeletal", "Prosthesis Design", "Range of Motion, Articular" ], "YEAR": "2005", "reasoning_required_pred": "no", "reasoning_free_pred": "no", "final_decision": "no", "LONG_ANSWER": "In this in vitro cadaver study, the tricompartmental replacement significantly changed knee kinematics while the unicompartmental replacement preserved normal knee kinematics." }, "28127977": { "QUESTION": "Are serum leptin levels a prognostic factor in advanced lung cancer?", "CONTEXTS": [ "There are 71 previously untreated patients with cytological or histological evidence of primary lung cancer who were admitted to the oncology department between November 2013 and August 2014. Forty-five healthy individuals with age, sex and BMI matching the lung cancer patients, were recruited to take part in the study as a control group. Leptin levels were measured quantitatively by using a microELISA kit.", "The serum leptin levels at diagnosis were significantly lower in lung cancer patients than those in control subjects (4.75\u00b14.91 ng/ml, 9.67\u00b18.02 ng/ml; p<0.001). We did not find any significant difference in leptin values related to clinicopathological parameters such as ECOG PS, weight loss, histological type, disease stage and TNM classification. Nevertheless, we demonstrated a significant correlation between serum leptin levels and BMI in lung cancer patients (correlation coefficient: 0.303; p>0.010). The analysis of serum leptin values did not show any association with the overall survival of the patients." ], "LABELS": [ "MATERIAL AND METHODS", "RESULTS" ], "MESHES": [ "Aged", "Biomarkers, Tumor", "Body Mass Index", "Female", "Humans", "Leptin", "Lung Neoplasms", "Male", "Middle Aged", "Neoplasm Staging", "Prognosis", "Reference Values", "Statistics as Topic" ], "YEAR": "2017", "reasoning_required_pred": "no", "reasoning_free_pred": "no", "final_decision": "no", "LONG_ANSWER": "Our results showed that the serum leptin level has no prognostic indications in advanced lung cancer patients. Leptin is decreased in lung cancer, and there is lack of correlation with tumour\u2011related factors including prognosis. Therefore, leptin is not a useful clinical marker in lung cancer (Tab. 2, Fig. 2, Ref. 22)." }, "16809243": { "QUESTION": "Is fetal gender associated with emergency department visits for asthma during pregnancy?", "CONTEXTS": [ "To investigate if fetal gender (1) affects the risk of having an emergency department (ED) visit for asthma; and (2) is associated with adverse pregnancy outcomes among women who had at least one visit to the ED for asthma during pregnancy.", "We linked two provincial administrative databases containing records on in-patient deliveries and ED visits. The study sample included women who delivered a live singleton baby between April 2003 and March 2004. Pregnant women who made at least one ED visit for asthma were counted as cases and the rest of the women as control subjects. We performed a multivariable analysis using logistic regression to model the risk of having an ED visit for asthma, with fetal gender being one of the predictors. In addition, a series of multivariable logistic regressions were also constructed separately for cases and controls for the following adverse delivery outcomes: low birth weight baby, preterm delivery, and delivery via Caesarian section.", "Among 109,173 live singleton deliveries, 530 women had visited ED due to asthma during pregnancy. While having an ED visit for asthma was positively associated with teenage pregnancy, low income, and presence of pregnancy-induced hypertension, it was not associated with fetal gender (OR 1.01, 95% CI 0.85-1.19). Fetal gender was not a significant predictor of adverse pregnancy outcomes among women who had an asthma ED visit during pregnancy." ], "LABELS": [ "BACKGROUND", "METHODS", "RESULTS" ], "MESHES": [ "Adolescent", "Adult", "Ambulatory Care", "Asthma", "Case-Control Studies", "Chi-Square Distribution", "Emergency Service, Hospital", "Female", "Gestational Age", "Humans", "Incidence", "Male", "Maternal Age", "Parity", "Pregnancy", "Pregnancy Complications", "Pregnancy Outcome", "Pregnancy, High-Risk", "Prenatal Care", "Probability", "Registries", "Retrospective Studies", "Risk Assessment", "Sex Determination Analysis", "Sex Distribution", "Sex Factors" ], "YEAR": "2006", "reasoning_required_pred": "no", "reasoning_free_pred": "no", "final_decision": "no", "LONG_ANSWER": "Fetal gender does not affect the risk of having an ED visit for asthma during pregnancy, and it is not associated with adverse pregnancy outcomes among women who had an asthma-related ED during pregnancy." }, "17276182": { "QUESTION": "Stretch-sensitive KCNQ1 mutation A link between genetic and environmental factors in the pathogenesis of atrial fibrillation?", "CONTEXTS": [ "This study sought to evaluate mutations in genes encoding the slow component of the cardiac delayed rectifier K+ current (I(Ks)) channel in familial atrial fibrillation (AF).", "Although AF can have a genetic etiology, links between inherited gene defects and acquired factors such as atrial stretch have not been explored.", "Mutation screening of the KCNQ1, KCNE1, KCNE2, and KCNE3 genes was performed in 50 families with AF. The effects of mutant protein on cardiac I(Ks) activation were evaluated using electrophysiological studies and human atrial action potential modeling.", "One missense KCNQ1 mutation, R14C, was identified in 1 family with a high prevalence of hypertension. Atrial fibrillation was present only in older individuals who had developed atrial dilation and who were genotype positive. Patch-clamp studies of wild-type or R14C KCNQ1 expressed with KCNE1 in CHO cells showed no statistically significant differences between wild-type and mutant channel kinetics at baseline, or after activation of adenylate cyclase with forskolin. After exposure to hypotonic solution to elicit cell swelling/stretch, mutant channels showed a marked increase in current, a leftward shift in the voltage dependence of activation, altered channel kinetics, and shortening of the modeled atrial action potential duration." ], "LABELS": [ "OBJECTIVES", "BACKGROUND", "METHODS", "RESULTS" ], "MESHES": [ "Action Potentials", "Adult", "Aged", "Aged, 80 and over", "Atrial Fibrillation", "Cohort Studies", "Female", "Heart Atria", "Humans", "KCNQ1 Potassium Channel", "Male", "Middle Aged", "Mutation, Missense", "Pedigree", "Potassium Channels, Voltage-Gated" ], "YEAR": "2007", "reasoning_required_pred": "yes", "reasoning_free_pred": "yes", "final_decision": "yes", "LONG_ANSWER": "These data suggest that the R14C KCNQ1 mutation alone is insufficient to cause AF. Rather, we suggest a model in which a \"second hit\", such as an environmental factor like hypertension, which promotes atrial stretch and thereby unmasks an inherited defect in ion channel kinetics (the \"first hit\"), is required for AF to be manifested. Such a model would also account for the age-related increase in AF development." }, "21123461": { "QUESTION": "Are adult body circumferences associated with height?", "CONTEXTS": [ "Weight scales as height squared, which is an observation that forms the basis of body mass index (weight/height(2)). If, and how, circumferences, including waist circumference (WC) and hip circumference (HC), scale to height remains unclear, but this is an important consideration when developing normative ranges or applying WC/height and HC/height as risk indexes.", "The study aim was to examine the scaling of weight, WC, and HC to height in NHANES (National Health and Nutrition Examination Survey) III participants.", "Subjects were adult non-Hispanic white, non-Hispanic black, and Mexican American men (n = 7422) and nonpregnant women (n = 7999) who had complete demographic and anthropometric data. In addition to height, allometric models were developed for each measure that controlled for age, race, and self-reported health status.", "After adjustment for age and race, weight scaled to height in men and women with mean (\u00b1SEE) powers of 2.29 \u00b1 0.11 and 1.80 \u00b1 0.07, respectively (both P<0.001). Although univariate circumference-height models were weak or nonsignificant, when adjusted for age and race WC and HC scaled to height with powers of 0.76 \u00b1 0.08 and 0.45 \u00b1 0.05, respectively, in men and 0.80 \u00b1 0.05 and 0.53 \u00b1 0.04, respectively, in women (all P<0.001). Age- and race-adjusted incremental increases in circumferences ranged from 0.2 to 0.5 cm per centimeter increase in height. Both WC/height and HC/height scaled negatively to height in men and women, and WC/HC scaled negatively to height in women only (all P<0.001). Health status-adjusted models were similar." ], "LABELS": [ "BACKGROUND", "OBJECTIVE", "DESIGN", "RESULTS" ], "MESHES": [ "Adult", "Anthropometry", "Body Height", "Body Weight", "Female", "Hip", "Humans", "Male", "Nutrition Surveys", "Obesity", "Reference Values", "Waist Circumference" ], "YEAR": "2011", "reasoning_required_pred": "yes", "reasoning_free_pred": "yes", "final_decision": "yes", "LONG_ANSWER": "Circumferences and related ratios scale significantly to height, notably after adjustment for age and race, across subjects who are representative of the US population. These observations have implications for the clinical and epidemiologic use of these anthropometric measures and indexes." }, "22428608": { "QUESTION": "Would corrected QT dispersion predict left ventricular hypertrophy in hypertensive patients?", "CONTEXTS": [ "We explored whether QT corrected dispersion (QTcD) can identify left ventricular hypertrophy (LVH) in hypertensives.", "We enrolled 100 hypertensive patients (study group) and 30 normotensive subjects (control group). Echocardiography was performed to measure left ventricular mass and left ventricular mass index. Electrocardiogram was performed to measure QTcD.", "LVH was present in 42 patients (42%) of the study group, none among controls. Hypertensive patients had significantly greater indices of LVH and QTcD compared with controls (p<0.001 for all). Similarly, among hypertensive patients, those with LVH had a significantly greater QTcD compared with those without (p<0.001). Pearson's correlation coefficient test demonstrated strongly positive correlations between QTcD and the indices of LVH (p<0.001 for all). Analysis of the receiver operating characteristic curves identified 60 ms as the optimal cut-off value of QTcD that best predicts LVH in hypertensives. Using this value, QTcD was able to predict LVH with a sensitivity of 92.9% and specificity 98.2%." ], "LABELS": [ "AIMS", "METHODS", "RESULTS" ], "MESHES": [ "Adult", "Aged", "Cross-Sectional Studies", "Echocardiography", "Electrocardiography", "Female", "Humans", "Hypertension", "Hypertrophy, Left Ventricular", "Male", "Middle Aged", "Myocardial Contraction", "Predictive Value of Tests", "Risk Factors", "Sensitivity and Specificity", "Statistics, Nonparametric" ], "YEAR": "2012", "reasoning_required_pred": "yes", "reasoning_free_pred": "yes", "final_decision": "yes", "LONG_ANSWER": "QTcD is significantly increased in hypertensive patients with LVH compared with those without, being strongly correlated with the indices of LVH. A QTcD cut-off value of 60 ms predicted LVH in hypertensive patients with a high sensitivity and specificity." }, "27858166": { "QUESTION": "Traumatic aortic injury: does the anatomy of the aortic arch influence aortic trauma severity?", "CONTEXTS": [ "Traumatic aortic injury (TAI) is a rare but life-threatening type of injury. We investigate whether the anatomy of the aortic arch influences the severity of aortic injury.", "This is a retrospective study of twenty-two cases treated with TEVAR for TAI in our department from 2009 to 2014. Aortic injury was assessed in accordance with the recommendations of the Society of Vascular Surgery. We measured the aortic arch angle and the aortic arch index, based on the initial angio-CT scan, in each of the analyzed cases.", "The mean aortic arch index and mean aortic arch angle were 6.8\u00a0cm and 58.3\u00b0, respectively, in the type I injury group; 4.4\u00a0cm and 45.9\u00b0 in the type III group; 3.3\u00a0cm and 37\u00b0 in the type IV group. There were substantial differences in both the aortic arch index and the aortic arch angle of the type III and IV groups. A multivariate analysis confirmed that the aortic arch angle was significantly associated with the occurrence of type III damage (OR 1.5; 95% CI 1.03-2.2)." ], "LABELS": [ "PURPOSE", "METHODS", "RESULTS" ], "MESHES": [ "Adult", "Aorta", "Aorta, Thoracic", "Endovascular Procedures", "Female", "Humans", "Male", "Middle Aged", "Multivariate Analysis", "Retrospective Studies", "Stents", "Thoracic Injuries", "Tomography, X-Ray Computed", "Trauma Severity Indices", "Treatment Outcome", "Young Adult" ], "YEAR": "2017", "reasoning_required_pred": "yes", "reasoning_free_pred": "yes", "final_decision": "yes", "LONG_ANSWER": "The severity of TAI is influenced by the sharpness of the aortic arch. There is an inverse relationship between the severity of aortic injury and the aortic arch index." }, "23791827": { "QUESTION": "Inpatient versus outpatient management of neutropenic fever in gynecologic oncology patients: is risk stratification useful?", "CONTEXTS": [ "This study aimed to evaluate the utility of risk stratification of gynecologic oncology patients with neutropenic fever (NF).", "A retrospective chart review of gynecologic cancer patients admitted with NF from 2007 to 2011 was performed, wherein demographic, oncologic, and NF characteristics (hospitalization length, complications, and death) were collected. The Multinational Association for Supportive Care in Cancer (MASCC) risk index score was calculated; low risk was considered \u2265 21. SAS 9.2 was used for statistical analyses.", "Eighty-three patients met the study criteria. Most (92%) were Caucasian and had advanced stage disease (71%). Primary tumors were 58% ovary, 35% endometrium, and 6% cervix. All patients were receiving chemotherapy on admission (72% for primary, 28% for recurrent disease). Forty-eight percent had a positive culture, and most (58%) positive cultures were urine. Seventy-six percent of patients were considered low risk. High-risk patients were more likely to have a severe complication (10% versus 50%, p=0.0003), multiple severe complications (3% versus 20%, p=0.0278), ICU admission (2% versus 40%, p<0.0001), overall mortality (2% versus 15%, p=0.0417), and death due to neutropenic fever (0% versus 15%, p=0.0124). MASCC had a positive predictive value of 50% and negative predictive value of 90%. The median MASCC score for all patients was 22 (range, 11-26), but the median MASCC score for those with death or a severe complication was 17 (range, 11-24)." ], "LABELS": [ "OBJECTIVE", "METHODS", "RESULTS" ], "MESHES": [ "Adult", "Aged", "Aged, 80 and over", "Ambulatory Care", "Anti-Bacterial Agents", "Antineoplastic Agents", "Female", "Fever", "Genital Neoplasms, Female", "Hospitalization", "Humans", "Middle Aged", "Neutropenia", "Predictive Value of Tests", "Prognosis", "Retrospective Studies", "Risk Assessment", "Severity of Illness Index" ], "YEAR": "2013", "reasoning_required_pred": "yes", "reasoning_free_pred": "yes", "final_decision": "yes", "LONG_ANSWER": "Based on this pilot data, MASCC score appears promising in determining suitability for outpatient management of NF in gynecologic oncology patients. Prospective study is ongoing to confirm safety and determine impact on cost." }, "18243752": { "QUESTION": "Should chest wall irradiation be included after mastectomy and negative node breast cancer?", "CONTEXTS": [ "This study aims to evaluate local failure patterns in node negative breast cancer patients treated with post-mastectomy radiotherapy including internal mammary chain only.", "Retrospective analysis of 92 internal or central-breast node-negative tumours with mastectomy and external irradiation of the internal mammary chain at the dose of 50 Gy, from 1994 to 1998.", "Local recurrence rate was 5 % (five cases). Recurrence sites were the operative scare and chest wall. Factors associated with increased risk of local failure were age24 hours).", "We operated on 613 children, including 97 neonates. Intraoperative extubation was achieved in 71% of the cases and early extubation (\u2264 24 hours) was achieved in 89% of the cases. The overall mortality was 1.5% (9 of 613 patients). Early extubation was associated with lower mortality (1% vs 9%, P<.001) and a lower rate of reintubation (4% vs 23%, P<.001) compared with delayed extubation. Notably, 63% of the neonates were extubated within 24 hours, including 67% of arterial switch operations and 54% of total anomalous pulmonary venous return repairs. Norwood operations were the only procedure in which no patient was extubated within the first 24 hours. Multivariable logistic regression demonstrated that the predictors of delayed extubation included preoperative mechanical ventilation, weight<5 kg, a longer procedure time, and the need for postoperative inotrope support. Implementation of an early extubation strategy was associated with low rates of complications (5.1 per 10 procedures), short lengths of intensive care unit stay (median, 1 day; interquartile range, 1-3), and short hospital stays (median, 4 days; interquartile range, 3-6)." ], "LABELS": [ "OBJECTIVE", "METHODS", "RESULTS" ], "MESHES": [ "Age Factors", "Airway Extubation", "Cardiac Surgical Procedures", "Feasibility Studies", "Female", "Heart Defects, Congenital", "Humans", "Infant", "Infant, Newborn", "Intensive Care Units, Pediatric", "Length of Stay", "Logistic Models", "Male", "Multivariate Analysis", "Odds Ratio", "Retrospective Studies", "Risk Factors", "Time Factors", "Treatment Outcome" ], "YEAR": "2014", "reasoning_required_pred": "maybe", "reasoning_free_pred": "yes", "final_decision": "yes", "LONG_ANSWER": "Most children undergoing congenital heart surgery can be extubated in the operating room. Most neonates, including many undergoing complex procedures, can be extubated within the first 24 hours after surgery. Early extubation was associated with low morbidity rates and short lengths of intensive care unit and hospital stays." }, "22023714": { "QUESTION": "Does delivery mode affect women's postpartum quality of life in rural China?", "CONTEXTS": [ "To explore the impact of delivery mode on women's postpartum quality of life in rural China and probe factors influencing postnatal quality of life.", "Childbirth significantly affects puerpera's physical, psychological and social domains of quality of life. Under the circumstance of increasing high caesarean section rate in rural China, the impact of delivery mode on postnatal quality of life remains unclear.", "Cross-sectional study design.", "Women residing in rural areas and in their 0-12 months after childbirth from 30 rural townships participated in a household survey. A structured questionnaire was used to evaluate women's socio-demographic characteristics, previous pregnant experiences, foetal characteristics and use of maternal health services. The scale for rural postnatal quality of life was adopted to assess postnatal quality of life from six dimensions: physical complaints and pain, sleep and energy, sex satisfaction, interpersonal communication, self-evaluated living stress and perceived life satisfaction.", "The overall caeserean section rate was 70\u00b70% (962/1375), and most of them (59\u00b77%) were selected by maternal request. None of six dimensions and total score of quality of life displayed significant difference between women with normal delivery and cesaerean section. It was found that postnatal home visit related to good postnatal quality of life and lower husband education level, male gender of infant were associated with poor quality of life." ], "LABELS": [ "AIMS AND OBJECTIVES", "BACKGROUND", "DESIGN", "METHODS", "RESULTS" ], "MESHES": [ "Adult", "China", "Delivery, Obstetric", "Female", "Humans", "Pilot Projects", "Postpartum Period", "Quality of Life", "Rural Population", "Surveys and Questionnaires" ], "YEAR": "2012", "reasoning_required_pred": "no", "reasoning_free_pred": "no", "final_decision": "no", "LONG_ANSWER": "Delivery mode did not affect postpartum quality of life in rural China. Socio-cultural determinants may contribute more in influencing postnatal quality of life." }, "16046584": { "QUESTION": "Menopausal hormone therapy and irregular endometrial bleeding: a potential role for uterine natural killer cells?", "CONTEXTS": [ "Irregular bleeding affects many users of combined menopausal hormone therapy (HT) and commonly leads to invasive and expensive investigations to exclude underlying malignancy. In most cases no abnormality is found.", "The main objective of this study was to explore the role of uterine natural killer (uNK) cells and their regulatory cytokine IL-15 in irregular bleeding in HT users.", "This was a prospective observational study conducted between 2002 and 2004.", "The study was conducted in a tertiary referral menopause clinic at King Edward Memorial Hospital, Western Australia.", "Patients included 117 postmenopausal women taking combined HT.", "Outpatient endometrial biopsies were taken during and outside bleeding episodes.", "The relationship between endometrial uNK cells (CD56+) and bleeding patterns was measured. We also addressed the impact of HT exposure on uNK cell populations, the relationship between endometrial IL-15 expression and uNK cell populations, and killer Ig like receptor genotype in subjects with irregular bleeding.", "Endometrial CD56+ uNK cells were significantly increased in biopsies obtained during bleeding episodes (P<0.001), compared with HT users with no bleeding. The highest level of IL-15 expression was also seen in biopsies taken during bleeding. No clear relationship between killer Ig like receptor genotype and bleeding on HT was observed." ], "LABELS": [ "CONTEXT", "OBJECTIVE", "DESIGN", "SETTING", "PATIENTS", "INTERVENTIONS", "MAIN OUTCOME MEASURES", "RESULTS" ], "MESHES": [ "CD56 Antigen", "Endometrium", "Estradiol", "Estrogen Replacement Therapy", "Estrogens, Conjugated (USP)", "Female", "Genotype", "Hemorrhage", "Humans", "Immunohistochemistry", "Interleukin-15", "Killer Cells, Natural", "Lymphocyte Count", "Menopause", "Middle Aged", "Receptors, Immunologic", "Uterus" ], "YEAR": "2005", "reasoning_required_pred": "yes", "reasoning_free_pred": "yes", "final_decision": "yes", "LONG_ANSWER": "Little is known about the mechanisms underlying irregular bleeding in HT users. This is the first report of uNK cells and their association with regulating cytokines in postmenopausal endometrium and demonstrates a possible mechanism by which HT may induce irregular bleeding." }, "16155169": { "QUESTION": "Percutaneous ethanol injection for benign cystic thyroid nodules: is aspiration of ethanol-mixed fluid advantageous?", "CONTEXTS": [ "We evaluated the differences between percutaneous ethanol injection with and without aspiration of ethanol-mixed fluid for treatment of benign cystic thyroid nodules.", "We examined 60 patients with benign cystic thyroid nodules confirmed by fine-needle aspiration biopsy and divided them into 2 groups according to nonaspiration (group A, n = 30) or aspiration (group B, n = 30) of ethanol-mixed fluid after intracystic ethanol injection. We evaluated in both groups the complete disappearance of the cystic portion of the thyroid nodule on follow-up ultrasonography (first follow-up ultrasonography; mean, 4.6 months in group A; mean, 4.4 months in group B) (chi-square test), side effects or complications during and after the procedure (chi-square test), and the total procedure time (Student t test).", "Most patients showed complete disappearance of the cystic portion of the thyroid nodule (group A, n = 29; group B, n = 28), and they revealed no recurrence on follow-up ultrasonography. There was no statistical difference in the success rates between group A and group B (P>.05). Pain, the most common side effect, and other mild side effects or complications occurred in small numbers of patients in each group, but there was no significant difference in side effects or complications between the 2 groups (P>.05), except for intracystic hemorrhage (P<.05) and the complaint of all group B patients due to a double puncture (P<.001). The total procedure time was nearly double in group B than in group A because of the additional procedures, such as complete evacuation of the ethanol-mixed fluid and the 10-minute compression." ], "LABELS": [ "BACKGROUND AND PURPOSE", "METHODS", "RESULTS" ], "MESHES": [ "Adult", "Aged", "Cysts", "Ethanol", "Female", "Humans", "Injections, Intralesional", "Male", "Middle Aged", "Sclerotherapy", "Suction", "Thyroid Nodule", "Treatment Outcome", "Ultrasonography" ], "YEAR": "2005", "reasoning_required_pred": "no", "reasoning_free_pred": "no", "final_decision": "no", "LONG_ANSWER": "Percutaneous ethanol injection without aspiration of ethanol-mixed fluid seems to be the preferable method of treatment of benign cystic thyroid nodules from the perspective of both the physician and the patient." }, "19398929": { "QUESTION": "Can the growth rate of a gallbladder polyp predict a neoplastic polyp?", "CONTEXTS": [ "Cholecystectomy for GB polyps that are larger than 10 mm is generally recommended because of the high probability of neoplasm. In contrast, a follow-up strategy is preferred for GB polyps smaller than 10 mm. However, there are no treatment guidelines for polyps that grow in size during the follow-up period.STUDY: We retrospectively investigated 145 patients with GB polyps who underwent at least 1 ultrasonographic follow-up examination over an interval greater than 6 months, before cholecystectomy at Samsung medical center, South Korea, from 1994 to 2007. The growth rate was determined based on the change in size per time interval between 2 ultrasonographic examinations (mm/mo).", "The median age of the patients was 48 years (range: 25 to 75). One hundred twenty-five non-neoplastic polyps and 20 neoplastic polyps were found. Neoplastic polyps were more frequently found in patients older than 60 years, those with hypertension, a polyp size greater than 10 mm, and a rapid growth rate greater than 0.6 mm/mo. On multivariate analysis, however, the growth rate was not related to the neoplastic nature of a polyp, but older age (>60 y) and large size (>10 mm) were significantly associated with neoplastic polyps." ], "LABELS": [ "BACKGROUND", "RESULTS" ], "MESHES": [ "Adult", "Age Factors", "Aged", "Chi-Square Distribution", "Cholecystectomy", "Disease Progression", "Female", "Gallbladder Diseases", "Gallbladder Neoplasms", "Humans", "Logistic Models", "Male", "Middle Aged", "Odds Ratio", "Polyps", "Precancerous Conditions", "Republic of Korea", "Retrospective Studies", "Risk Assessment", "Risk Factors", "Time Factors", "Ultrasonography" ], "YEAR": "2009", "reasoning_required_pred": "no", "reasoning_free_pred": "yes", "final_decision": "no", "LONG_ANSWER": "Patient's age (>60 y) and large polyp size (>10 mm) were significant predictive factors for neoplastic GB polyps. GB polyps less than 10 mm in diameter do not require surgical intervention simply because they grow." }, "28143468": { "QUESTION": "Are performance measurement systems useful?", "CONTEXTS": [ "Prior literature identified the use of Performance Measurement Systems (PMS) as crucial in addressing improved processes of care. Moreover, a strategic use of PMS has been found to enhance quality, compared to non-strategic use, although a clear understanding of this linkage is still to be achieved. This paper deals with the test of direct and indirect models related to the link between the strategic use of PMS and the level of improved processes in health care organizations. Indirect models were mediated by the degree of perceived managerial discretion.", "A PLS analysis on a survey of 97 Italian managers working for health care organizations in the Lombardy region was conducted. The response rate was 77.6%.", "The strategic use of PMS in health care organizations directly and significantly (p\u2009<\u20090.001) enhances performance in terms of improved processes. Perceived managerial discretion is positively and significantly (p\u2009<\u20090.001) affected by the strategic use of PMS, whereas the mediation effect is non-significant." ], "LABELS": [ "BACKGROUND", "METHODS", "RESULTS" ], "MESHES": [ "Administrative Personnel", "Female", "Health Knowledge, Attitudes, Practice", "Humans", "Italy", "Male", "Quality Indicators, Health Care", "Surveys and Questionnaires" ], "YEAR": "2017", "reasoning_required_pred": "yes", "reasoning_free_pred": "yes", "final_decision": "yes", "LONG_ANSWER": "This study contributes to the literature investigating the design and implementation of a non-financial measurement tool, such as the non-financial information included into a balanced scorecard (BSC), in health care organizations. Managers in health care organizations can benefit from the strategic use of PMS to effectively allocate their time to strategic opportunities and threats, which might arise and affect organizational, output-related performance, such as improving processes." }, "9483814": { "QUESTION": "Does para-cervical block offer additional advantages in abortion induction with gemeprost in the 2nd trimester?", "CONTEXTS": [ "Uterus-specific synthetic Prostaglandin analogues (gemeprost, sulproston etc.) have been widely employed for termination of pregnancy in the second trimester. Since paracervical anaesthesia may be useful during this procedure, we investigated in this prospective randomised study its impact on the clinical course of abortion and pain especially in the late first and second stage of labour.", "20 women scheduled for elective abortion (fetal reasons) between the 16th and 23rd week of gestation were to be given 1 mg gemeprost vaginally every 6 hours. They were allocated at random: 10 women received only Pethidin intravenously and Butylscopolamine rectally, another 10 women were additionally treated by paracervical anaesthesia (2 x 10 ml 0.5% Bupivacain solution) at a cervical dilatation of 2-3 cm.", "A median of 3 gemeprost applications were administered in both groups. In the group without paracervical anaesthesia the median induction to abortion interval was 20 hours (range: 8-44 hours), 13 hours (range: 8-36 hours, NS) resulting for the paracervical anaesthesia group. The intervals from the last application of prostaglandin until abortion and from 3 cm cervical dilatation to abortion were slightly, but not significantly shorter in the paracervical anaesthesia group. The requirement of Butylscopolamine was higher in the latter group (p<0.05). The requirement of Pethidin and the intensity of pain (measured by pain scale according to Huskisson) especially in the late first stage of labour were not statistically different between both groups. Side effects of paracervical anaesthesia did not occur." ], "LABELS": [ "UNLABELLED", "PATIENTS AND METHODS", "RESULTS" ], "MESHES": [ "Abortifacient Agents, Nonsteroidal", "Abortion, Eugenic", "Adolescent", "Adult", "Alprostadil", "Anesthesia, Local", "Anesthesia, Obstetrical", "Bupivacaine", "Cervix Uteri", "Female", "Humans", "Pain Measurement", "Pregnancy", "Pregnancy Trimester, Second", "Prospective Studies" ], "YEAR": "1997", "reasoning_required_pred": "yes", "reasoning_free_pred": "yes", "final_decision": "yes", "LONG_ANSWER": "Paracervical anaesthesia is a method for analgesia during second trimester abortion with a low rate of side effects. It can shorten the duration of last period of second trimester abortion in some cases but has no impact on the perception of pain nor requirement of analgesics and so with only limited benefit in second trimester abortion with vaginal gemeprost." }, "22720085": { "QUESTION": "Does insulin resistance drive the association between hyperglycemia and cardiovascular risk?", "CONTEXTS": [ "Several studies have shown associations between hyperglycemia and risk of cardiovascular disease (CVD) and mortality, yet glucose-lowering treatment does little to mitigate this risk. We examined whether associations between hyperglycemia and CVD risk were explained by underlying insulin resistance.", "In 60 middle-aged individuals without diabetes we studied the associations of fasting plasma glucose, 2-hour post oral glucose tolerance test plasma glucose, insulin sensitivity as well as body fat percentage with CVD risk. Insulin sensitivity was measured as the glucose infusion rate during a euglycemic hyperinsulinemic clamp, body fat percentage was measured by dual X-ray absorptiometry, and CVD risk was estimated using the Framingham risk score. Associations of fasting plasma glucose, 2-hour plasma glucose, insulin sensitivity and body fat percentage with the Framingham risk score were assessed in linear regression models.", "Both fasting and 2-hour plasma glucose levels were associated with higher Framingham risk score (fasting glucose: r(2) = 0.21; 2-hour glucose: r(2) = 0.24; P<0.001 for both), and insulin sensitivity with lower Framingham risk score (r(2) = 0.36; P<0.001). However, adjustment for insulin sensitivity and 2-hour glucose made the effect of fasting glucose non-significant (P = 0.060). Likewise, when adjusting for insulin sensitivity and fasting glucose, the association between 2-hour glucose and Framingham risk score disappeared (P = 0.143). In contrast, insulin sensitivity was still associated with Framingham risk score after adjusting for glucose levels (P<0.001). Body fat was not associated with Framingham risk score when taking insulin sensitivity into account (P = 0.550)." ], "LABELS": [ "BACKGROUND", "METHODS", "RESULTS" ], "MESHES": [ "Adult", "Blood Glucose", "Cardiovascular Diseases", "Female", "Humans", "Hyperglycemia", "Insulin Resistance", "Male", "Middle Aged", "Risk Factors" ], "YEAR": "2012", "reasoning_required_pred": "yes", "reasoning_free_pred": "yes", "final_decision": "yes", "LONG_ANSWER": "The association between plasma glucose levels and CVD risk is mainly explained by insulin resistance, which raises the question of whether glucose lowering per se without changes in the processes that underlie hyperglycemia should be the sole clinical paradigm in the treatment of type 2 diabetes or its prevention." }, "24748473": { "QUESTION": "Are there risk factors that increase the rate of staple line leakage in patients undergoing primary sleeve gastrectomy for morbid obesity?", "CONTEXTS": [ "Laparoscopic sleeve gastrectomy (LSG) is currently being performed with increasing frequency worldwide. It offers an excellent weight loss and resolution of comorbidities in the short term with a very low incidence of complications. However, the ever present risk of a staple line leak is still a major concern.", "Since 2005, data from obese patients that undergo bariatric procedures in Germany are prospectively registered in an online database and analyzed at the Institute of Quality Assurance in Surgical Medicine. For the current analysis, all patients that had undergone primary sleeve gastrectomy for morbid obesity within a 7-year period were considered.", "Using the GBSR, data from 5.400 LSGs were considered for analysis. Staple line leak rate decreased during the study period from 6.5 to 1.4 %. Male gender, higher BMI, concomitant sleep apnea, conversion to laparotomy, longer operation time, use of both buttresses and oversewing, and the occurrence of intraoperative complications were associated with a significantly higher leakage rate. On multivariate analysis, operation time and year of procedure only had a significant impact on staple line leak rate." ], "LABELS": [ "BACKGROUND", "METHODS", "RESULTS" ], "MESHES": [ "Adolescent", "Adult", "Aged", "Bariatric Surgery", "Body Mass Index", "Comorbidity", "Female", "Gastrectomy", "Germany", "Humans", "Intraoperative Complications", "Laparoscopy", "Male", "Middle Aged", "Obesity, Morbid", "Operative Time", "Postoperative Complications", "Risk Factors", "Surgical Stapling", "Weight Loss", "Young Adult" ], "YEAR": "2014", "reasoning_required_pred": "yes", "reasoning_free_pred": "yes", "final_decision": "yes", "LONG_ANSWER": "The results of the current study demonstrated that there are factors that increase the risk of a leakage which would enable surgeons to define risk groups, to more carefully select patients, and to offer a closer follow-up during the postoperative course with early recognition and adequate treatment. All future efforts should be focused on a further reduction of serious complications to make the LSG a widely accepted and safer procedure." }, "15588538": { "QUESTION": "Chronic functional somatic symptoms: a single syndrome?", "CONTEXTS": [ "Reliable longitudinal data of patients with functional somatic symptoms in general practice are lacking.", "To identify distinctive features in patients with chronic functional somatic symptoms, and to determine whether these symptoms support the hypothesis of the existence of specific somatic syndromes.", "Observational study, with a comparison control group.", "Four primary care practices affiliated with the University of Nijmegen in the Netherlands.", "One hundred and eighty-two patients diagnosed between 1998 and 2002 as having chronic functional somatic symptoms and 182 controls matched by age, sex, socioeconomic status, and practice were included. Data on comorbidity, referrals, diagnostic tests, and hospital admissions over a period of 10 years prior to the diagnosis were collected. Medication use and number of visits to the general practitioner (GP) were extracted from the moment computerised registration was started.", "In the 10 years before the diagnosis of chronic functional somatic symptoms, significantly more patients than controls presented functional somatic symptoms in at least two body systems, and used more somatic and psychotropic drugs. They visited the GP twice as much, statistically had significantly more psychiatric morbidity, and were referred more often to mental health workers and somatic specialists. The number of patients undergoing diagnostic tests was higher for patients with chronic functional somatic symptoms than for controls, but hospital admissions rates were equal." ], "LABELS": [ "BACKGROUND", "AIMS", "DESIGN OF STUDY", "SETTING", "METHOD", "RESULTS" ], "MESHES": [ "Adolescent", "Adult", "Aged", "Aged, 80 and over", "Case-Control Studies", "Child", "Chronic Disease", "Female", "Hospitalization", "Humans", "Male", "Middle Aged", "Netherlands", "Patient Acceptance of Health Care", "Referral and Consultation", "Somatoform Disorders", "Syndrome" ], "YEAR": "2004", "reasoning_required_pred": "no", "reasoning_free_pred": "no", "final_decision": "no", "LONG_ANSWER": "Patients with chronic functional somatic symptoms have a great diversity of functional somatic symptoms. They use more somatic and psychotropic drugs than controls in the years before diagnosis. Moreover, they show high rates of referrals and psychiatric morbidity. The diversity of symptoms of patients with chronic functional somatic symptoms supports the concept that symptoms do not cluster in well defined distinct syndromes. Therefore, patients with chronic functional somatic symptoms should preferably not be classified into medical subspecialty syndromes." }, "22211919": { "QUESTION": "Actinobaculum schaalii, a cause of urinary tract infections in children?", "CONTEXTS": [ "Urine samples were examined by wet smear microscopy, incubated in 5% CO(2) for 1-2 days, and species-specific real-time polymerase chain reaction (PCR) for A. schaalii was performed.", "In 5 of the 29 screened urines, A. schaalii was found only by real-time PCR in quantities equivalent to \u2265 10(4) -10(5) CFU/mL. In addition, A. schaalii was found in quantities equivalent to \u2265 10(6) CFU/mL by both culture and PCR in two children with a urinary tract infection and large numbers of leucocytes in the urine." ], "LABELS": [ "METHODS", "RESULTS" ], "MESHES": [ "Adolescent", "Child, Preschool", "Female", "Gram-Positive Bacteria", "Gram-Positive Bacterial Infections", "Humans", "Infant", "Male", "Retrospective Studies", "Urinary Tract Infections" ], "YEAR": "2012", "reasoning_required_pred": "yes", "reasoning_free_pred": "maybe", "final_decision": "maybe", "LONG_ANSWER": "Actinobaculum schaalii is CO(2)-dependent. Therefore, if there are clinical symptoms and/or a negative culture despite the presence of leucocytes in the urine, Gram staining and incubation in 5% CO(2) or species-specific real-time PCR should be performed to identify A. schaalii." }, "24433626": { "QUESTION": "Prevalence of chronic conditions among Medicare Part A beneficiaries in 2008 and 2010: are Medicare beneficiaries getting sicker?", "CONTEXTS": [ "Medicare beneficiaries who have chronic conditions are responsible for a disproportionate share of Medicare fee-for-service expenditures. The objective of this study was to analyze the change in the health of Medicare beneficiaries enrolled in Part A (hospital insurance) between 2008 and 2010 by comparing the prevalence of 11 chronic conditions.", "We conducted descriptive analyses using the 2008 and 2010 Chronic Conditions Public Use Files, which are newly available from the Centers for Medicare and Medicaid Services and have administrative (claims) data on 100% of the Medicare fee-for-service population. We examined the data by age, sex, and dual eligibility (eligibility for both Medicare and Medicaid).", "Medicare Part A beneficiaries had more chronic conditions on average in 2010 than in 2008. The percentage increase in the average number of chronic conditions was larger for dual-eligible beneficiaries (2.8%) than for nondual-eligible beneficiaries (1.2%). The prevalence of some chronic conditions, such as congestive heart failure, ischemic heart disease, and stroke/transient ischemic attack, decreased. The deterioration of average health was due to other chronic conditions: chronic kidney disease, depression, diabetes, osteoporosis, rheumatoid arthritis/osteoarthritis. Trends in Alzheimer's disease, cancer, and chronic obstructive pulmonary disease showed differences by sex or dual eligibility or both." ], "LABELS": [ "INTRODUCTION", "METHODS", "RESULTS" ], "MESHES": [ "Aged", "Aged, 80 and over", "Chronic Disease", "Female", "Health Services Research", "Humans", "Insurance Claim Review", "Male", "Medicare Part A", "Middle Aged", "Prevalence", "Time Factors", "United States" ], "YEAR": "2014", "reasoning_required_pred": "yes", "reasoning_free_pred": "maybe", "final_decision": "yes", "LONG_ANSWER": "Analyzing the prevalence of 11 chronic conditions by using Medicare claims data provides a monitoring tool that can guide health care providers and policy makers in devising strategies to address chronic conditions and rising health care costs." }, "9542484": { "QUESTION": "Does successful completion of the Perinatal Education Programme result in improved obstetric practice?", "CONTEXTS": [ "To determine whether successful completion of the Perinatal Education Programme (PEP) improves obstetric practice.", "The three midwife obstetric units (MOUs) in a health district of Mpumalanga were included in the study. Two MOUs enrolled in the PEP and the third did not. A 'before-and-after' study design was used to assess any changes in practice, and to monitor whether any changes occurred in the district during the time of the study; data were also collected at the third MOU. Data were collected by scoring of the obstetric files after the patient had delivered.", "We ascertained whether the obstetric history, syphilis testing, blood group testing, haemoglobin measurement and uterine growth assessment were performed during antenatal care along with whether appropriate action was taken. For intrapartum care, estimation of fetal weight, the performance of pelvimetry, blood pressure monitoring, urine testing, evaluation of head above pelvis, fetal heart rate monitoring, monitoring of contractions and plotting of cervical dilatation, and whether the appropriate actions were taken, were assessed.", "Eight of the 13 midwives at the two MOUs completed the PEP and all demonstrated an improvement in knowledge. Case notes of 303 patients from the various clinics were studied. There was no change in the referral patterns of any of the clinics during the study period. The obstetric history was well documented, but in no group was there a satisfactory response to a detected problem; appropriate action was taken in between 0% and 12% of cases. Syphilis testing was performed in 56-82% of cases, with no difference between the groups. The haemoglobin level was measured in only 4-15% of patients, with no difference before or after completion of the PEP. Where a problem in uterine growth was detected, an appropriate response occurred in 0-8% of patients and no difference before or after completion of the PEP was ascertained. In all groups, estimation of fetal weight and pelvimetry were seldom performed, the urine and fetal heart rate documentation were moderately well done and the blood pressure monitoring, assessment of head above pelvis, monitoring of contractions and plotting of cervical dilatation were usually performed. No differences before or after the PEP were detected. Where problems were detected, appropriate actions taken during labour improved, but not significantly." ], "LABELS": [ "OBJECTIVE", "METHOD", "OUTCOME MEASURES", "RESULTS" ], "MESHES": [ "Africa", "Female", "Humans", "Midwifery", "Perinatal Care", "Pregnancy", "Prenatal Care", "Rural Population" ], "YEAR": "1998", "reasoning_required_pred": "no", "reasoning_free_pred": "no", "final_decision": "no", "LONG_ANSWER": "Completion of the obstetric manual of the PEP improved the knowledge of the midwives but no alteration in practice was detected." }, "17008699": { "QUESTION": "Do patients with localized prostate cancer treatment really want more aggressive treatment?", "CONTEXTS": [ "Examine whether patients with prostate cancer choose the more aggressive of two radiotherapeutic options, whether this choice is reasoned, and what the determinants of the choice are.", "One hundred fifty patients with primary prostate cancer (T(1-3)N(0)M(0)) were informed by means of a decision aid of two treatment options: radiotherapy with 70 Gy versus 74 Gy. The latter treatment is associated with more cure and more toxicity. The patients were asked whether they wanted to choose, and if so which treatment they preferred. They also assigned importance weights to the probability of various outcomes, such as survival, cure and adverse effects. Patients who wanted to choose their own treatment (n = 119) are described here.", "The majority of these patients (75%) chose the lower radiation dose. Their choice was highly consistent (P3 times a week) and 71 (65.14%) had a drink occasionally (once a month or less). Only 25 residents (23%) of the cohort had a complete and up-to-date immunization status. The immunization gap was basically against measles, mumps, rubella (MMR) and diphtheria, tetanus, poliomyelitis (dT Polio). Ninety-nine residents (90.83%) had full immunization against hepatitis B with an adequate response in 78 residents (71.56%)." ], "LABELS": [ "BACKGROUND", "METHODS", "RESULTS" ], "MESHES": [ "Adult", "Feeding Behavior", "Female", "Health Behavior", "Humans", "Internship and Residency", "Lebanon", "Male", "Overweight", "Physicians", "Smoking", "Vaccination" ], "YEAR": null, "reasoning_required_pred": "no", "reasoning_free_pred": "no", "final_decision": "no", "LONG_ANSWER": "This study showed that our residents did not always have a healthy lifestyle especially when it comes to physical activity and eating habits. They also lacked an adequate vaccination. Interventions should take place in order to promote healthy life style and to improve their vaccination status." }, "18714572": { "QUESTION": "Does vaginal intraepithelial neoplasia have the same evolution as cervical intraepithelial neoplasia?", "CONTEXTS": [ "Vaginal intraepithelial neoplasia is a little known disease which could be related to risk factors different from simple HPV infections.", "To ascertain wheter vaginal lesions have a natural history similar to cervical lesions.MATERIALS &", "A retrospective study to identify patients with vaginal lesions and synchronous cervical lesions through biopsy. The rate of mild cervical lesions (koilocytosis, warts, CIN I with and without koilocytosis) was compared with the rate of severe cervical lesions (CIN II and III, cervical carcinoma) in patients with mild vaginal lesions (warts and koilocytosis, and low-grade VAIN) and in patients with severe vaginal lesions (high-grade VAIN). Using koilocytosis as a marker, the rate of \"active\" cervical lesions was compared with the rate of \"non active\" cervical lesions in patients with \"active\" versus \"non active\" vaginal lesions. Finally, the rates of mild and severe cervical lesions were compared among each group of VAIN (low-grade, high-grade, with or without koilocytosis).", "In patients with mild vaginal lesions, mild cervical lesions were significantly more frequent than severe cervical lesions. In patients with \"active\" vaginal lesions the rate of \"active\" cervical lesions was significantly higher than \"non active\" cervical lesions. The differences in rates of mild cervical lesions and severe cervical lesions among patients with high-grade VAIN and low-grade VAIN (with and without koilocytosis) were not significant." ], "LABELS": [ "BACKGROUND", "OBJECTIVE", "METHODS", "RESULTS" ], "MESHES": [ "Biopsy", "Cervical Intraepithelial Neoplasia", "Female", "Humans", "Neoplasms, Multiple Primary", "Retrospective Studies", "Uterine Cervical Neoplasms", "Vaginal Neoplasms" ], "YEAR": "2008", "reasoning_required_pred": "maybe", "reasoning_free_pred": "maybe", "final_decision": "maybe", "LONG_ANSWER": "These data suggest that CIN and VAIN may have some common features in certain cases, i.e., if an HPV infection is proved." }, "2224269": { "QUESTION": "Should general practitioners call patients by their first names?", "CONTEXTS": [ "To assess the acceptability to patients of the use of patients' first names by doctors and doctors' first names by patients in general practice.", "An administered questionnaire survey.", "5 General practices in Lothian.", "475 Patients consulting 30 general practitioners.", "Response by patients to questionnaire on attitude to use of first names.", "Most of the patients either liked (223) or did not mind (175) being called by their first names. Only 77 disliked it, most of whom were aged over 65. Most patients (324) did not, however, want to call the doctor by his or her first name." ], "LABELS": [ "OBJECTIVE", "DESIGN", "SETTING", "PATIENTS", "MAIN OUTCOME MEASURE", "RESULTS" ], "MESHES": [ "Adolescent", "Adult", "Age Factors", "Aged", "Attitude", "Female", "Humans", "Male", "Middle Aged", "Physician-Patient Relations", "Physicians, Family", "Social Class" ], "YEAR": "1990", "reasoning_required_pred": "no", "reasoning_free_pred": "yes", "final_decision": "yes", "LONG_ANSWER": "General practitioners should consider using patients' first names more often, particularly with younger patients." }, "8910148": { "QUESTION": "Transesophageal echocardiographic assessment of left ventricular function in brain-dead patients: are marginally acceptable hearts suitable for transplantation?", "CONTEXTS": [ "The brain-dead donor supply has become one of the criteria limiting the performance of heart transplantation. Conventional screening criteria are too limiting and exclude suitable heart donors. Echocardiography is now widely available and is a reliable tool to assess left ventricular dysfunction in brain-dead donors. Yet few data are available on the degree of left ventricular dysfunction where a transplantation is possible.", "Fifty-five potential brain-dead heart donors (age 38 +/- 11 years) were prospectively evaluated by transesophageal echocardiography (TEE) before harvesting. Fractional area change (FAC) was used to assess left ventricular function in potential brain-dead donors. Transplanted hearts were evaluated on the fifth postoperative day. The transplantation was considered a success if the recipient was alive, not retransplanted, without an assistance device or an epinephrine infusion of more than 1 mg/h and showed an ejection fraction above 40%.", "Of the 55 potential heart donors, 20 exhibited an FAC of less than 50%. Forty hearts were harvested, 36 of which were successfully transplanted. Nine patients had an FAC below 50% (group H2) and 27 had an FAC over 50% (group H1). Four patients died: 2 from hemorrhage (FAC>50% in donors); 1 from right and one from left ventricular dysfunction (FAC<50% in donors). The FAC increased significantly from 51 +/- 15% to 57 +/- 11% in 18 hearts that underwent TEE in donors and afterwards in recipients. Overall actuarial survival was 86.2% versus 64.6% at 1 and 2 years in group H1 and group H2, respectively (p = NS)." ], "LABELS": [ "BACKGROUND", "METHODS", "RESULTS" ], "MESHES": [ "Adolescent", "Adult", "Brain Death", "Echocardiography, Transesophageal", "Female", "Heart Transplantation", "Humans", "Male", "Middle Aged", "Prospective Studies", "Tissue Donors", "Ventricular Function, Left" ], "YEAR": "1996", "reasoning_required_pred": "yes", "reasoning_free_pred": "yes", "final_decision": "yes", "LONG_ANSWER": "TEE is useful to assess left ventricular function in potential brain-dead donors. An FAC less than 50% is present in 36% of potential heart donors. Because left ventricular dysfunction is often reversible shortly after transplantation, an FAC below 50% may not necessarily preclude the use of hearts for transplantation." }, "23899611": { "QUESTION": "Attenuation of ischemia/reperfusion-induced ovarian damage in rats: does edaravone offer protection?", "CONTEXTS": [ "Twenty-eight female Sprague Dawley rats were allocated randomly to 4 groups. The sham group (group 1) was only subjected to catheter insertion, not to pneumoperitoneum. Group 2 received a 1 mg/kg dose of 0.9% sodium chloride by the intraperitoneal route for 10 min before pneumoperitoneum. Groups 3 and 4 received 6 and 12 mg/kg edaravone, respectively, by the intraperitoneal route for 10 min before pneumoperitoneum. After 60 min of pneumoperitoneum, the gas was deflated. Immediately after the reperfusion period, both ovaries were excised for histological scoring, caspase-3 immunohistochemistry and biochemical evaluation including glutathione (GSH) and malondialdehyde (MDA) levels. Also, total antioxidant capacity (TAC) was measured in plasma samples to evaluate the antioxidant effect of edaravone.", "Ovarian sections in the saline group revealed higher scores for follicular degeneration and edema (p<0.0001) when compared with the sham group. Administration of different doses of edaravone in rats significantly prevented degenerative changes in the ovary (p<0.0001). Caspase-3 expression was only detected in the ovarian surface epithelium in all groups, and there was a significant difference between the treatment groups and the saline group (p<0.0001). Treatment of rats with edaravone reduced caspase-3 expression in a dose-dependent manner. Moreover, biochemical measurements of oxidative stress markers (MDA, GSH and TAC) revealed that prophylactic edaravone treatment attenuated oxidative stress induced by I/R injury." ], "LABELS": [ "METHODS", "RESULTS" ], "MESHES": [ "Animals", "Antipyrine", "Caspase 3", "Female", "Free Radical Scavengers", "Glutathione", "Immunohistochemistry", "Malondialdehyde", "Ovary", "Rats", "Rats, Sprague-Dawley", "Reperfusion Injury" ], "YEAR": "2013", "reasoning_required_pred": "yes", "reasoning_free_pred": "yes", "final_decision": "yes", "LONG_ANSWER": "These results indicate that prophylactic treatment with edaravone prevents I/R-induced ovarian damage during pneumoperitoneum in an experimental rat model." }, "10548670": { "QUESTION": "Does the National Institutes of Health Stroke Scale favor left hemisphere strokes?", "CONTEXTS": [ "The National Institutes of Health Stroke Scale (NIHSS) is a valid, reproducible scale that measures neurological deficit. Of 42 possible points, 7 points are directly related to measurement of language compared with only 2 points related to neglect.", "We examined the placebo arm of the NINDS t-PA stroke trial to test the hypothesis that the total volume of cerebral infarction in patients with right hemisphere strokes would be greater than the volume of cerebral infarction in patients with left hemisphere strokes who have similar NIHSS scores. The volume of stroke was determined by computerized image analysis of CT films and CT images stored on computer tape and optical disks. Cube-root transformation of lesion volume was performed for each CT. Transformed lesion volume was analyzed in a logistic regression model to predict volume of stroke by NIHSS score for each hemisphere. Spearman rank correlation was used to determine the relation between the NIHSS score and lesion volume.", "The volume for right hemisphere stroke was statistically greater than the volume for left hemisphere strokes, adjusting for the baseline NIHSS (P<0. 001). For each 5-point category of the NIHSS score<20, the median volume of right hemisphere strokes was approximately double the median volume of left hemisphere strokes. For example, for patients with a left hemisphere stroke and a 24-hour NIHSS score of 16 to 20, the median volume of cerebral infarction was 48 mL (interquartile range 14 to 111 mL) as compared with 133 mL (interquartile range 81 to 208 mL) for patients with a right hemisphere stroke (P<0.001). The median volume of a right hemisphere stroke was roughly equal to the median volume of a left hemisphere stroke in the next highest 5-point category of the NIHSS. The Spearman rank correlation between the 24-hour NIHSS score and 3-month lesion volume was 0.72 for patients with left hemisphere stroke and 0.71 for patients with right hemisphere stroke." ], "LABELS": [ "BACKGROUND AND PURPOSE", "METHODS", "RESULTS" ], "MESHES": [ "Brain", "Cerebral Infarction", "Compact Disks", "Fibrinolytic Agents", "Follow-Up Studies", "Humans", "Image Processing, Computer-Assisted", "Language", "Logistic Models", "National Institutes of Health (U.S.)", "Neurologic Examination", "Placebos", "Stroke", "Thrombolytic Therapy", "Tissue Plasminogen Activator", "Tomography, X-Ray Computed", "United States" ], "YEAR": "1999", "reasoning_required_pred": "no", "reasoning_free_pred": "yes", "final_decision": "yes", "LONG_ANSWER": "For a given NIHSS score, the median volume of right hemisphere strokes is consistently larger than the median volume of left hemisphere strokes. The clinical implications of our finding need further exploration." }, "26907557": { "QUESTION": "Can a Novel Surgical Approach to the Temporomandibular Joint Improve Access and Reduce Complications?", "CONTEXTS": [ "This clinical study investigated whether the vascular-guided multilayer preauricular approach (VMPA) to the temporomandibular joint (TMJ) could improve access and decrease complications.", "This retrospective evaluation consisted of a consecutive series of patients who underwent TMJ surgeries through the VMPA from January through December 2013. Patients with a history of TMJ surgery were excluded. Clinical data, including operating times, subjective complaints of incision scars, functional conditions of the auriculotemporal nerve and facial nerve, and other complications, were recorded and analyzed. All patients in this study were followed for at least 6\u00a0months.", "All patients (606 joints) had successful TMJ surgeries through the VMPA. All incisions healed favorably with an uneventful recovery. No patient developed permanent weakness of the facial nerve or other severe complications." ], "LABELS": [ "PURPOSE", "PATIENTS AND METHODS", "RESULTS" ], "MESHES": [ "Adolescent", "Adult", "Aged", "Child", "Child, Preschool", "Female", "Humans", "Male", "Middle Aged", "Oral Surgical Procedures", "Postoperative Complications", "Retrospective Studies", "Temporomandibular Joint Disorders", "Treatment Outcome" ], "YEAR": "2016", "reasoning_required_pred": "yes", "reasoning_free_pred": "yes", "final_decision": "yes", "LONG_ANSWER": "The VMPA can provide direct access and favorable visibility to the TMJ region and yield good esthetic and functional results. The VMPA can be considered the approach of choice for common TMJ surgeries." }, "20871246": { "QUESTION": "Young-Burgess classification of pelvic ring fractures: does it predict mortality, transfusion requirements, and non-orthopaedic injuries?", "CONTEXTS": [ "The objectives of this study were to evaluate the ability of the Young-Burgess classification system to predict mortality, transfusion requirements, and nonorthopaedic injuries in patients with pelvic ring fractures and to determine whether mortality rates after pelvic fractures have changed over time.", "Retrospective review.", "Level I trauma center.", "One thousand two hundred forty-eight patients with pelvic fractures during a 7-year period.", "None.", "Mortality at index admission, transfusion requirement during first 24 hours, and presence of nonorthopaedic injuries as a function of Young-Burgess pelvic classification type. Mortality compared with historic controls.", "Despite a relatively large sample size, the ability of the Young-Burgess system to predict mortality only approached statistical significance (P = 0.07, Kruskal-Wallis). The Young-Burgess system differentiated transfusion requirements--lateral compression Type 3 (LC3) and anteroposterior compression Types 2 (APC2) and 3 (APC3) fractures had higher transfusion requirements than did lateral compression Type 1 (LC1), anteroposterior compression Type 1 (APC1), and vertical shear (VS) (P<0.05)--but was not as useful at predicting head, chest, or abdomen injuries. Dividing fractures into stable and unstable types allowed the system to predict mortality rates, abdomen injury rates, and transfusion requirements. Overall mortality in the study group was 9.1%, unchanged from original Young-Burgess studies 15 years previously (P = 0.3)." ], "LABELS": [ "OBJECTIVES", "DESIGN", "SETTING", "PATIENTS", "INTERVENTION", "MAIN OUTCOME MEASUREMENTS", "RESULTS" ], "MESHES": [ "Blood Transfusion", "Comorbidity", "Fractures, Compression", "Humans", "Maryland", "Pelvic Bones", "Predictive Value of Tests", "Retrospective Studies", "Survival Rate", "Tomography, X-Ray Computed", "Trauma Centers", "Trauma Severity Indices" ], "YEAR": "2010", "reasoning_required_pred": "no", "reasoning_free_pred": "maybe", "final_decision": "maybe", "LONG_ANSWER": "The Young-Burgess system is useful for predicting transfusion requirements. For the system to predict mortality or nonorthopaedic injuries, fractures must be divided into stable (APC1, LC1) and unstable (APC2, APC3, LC2, LC3, VS, combined mechanism of injury) types. LC1 injuries are very common and not always benign (overall mortality rate, 8.2%)." }, "22876568": { "QUESTION": "Is vitamin D deficiency a feature of pediatric celiac disease?", "CONTEXTS": [ "Celiac disease (CD) is an autoimmune enteropathy characterized by villus atrophy and malabsorption of essential nutrients. Vitamin D deficiency has been described in autoimmune diseases, but its status in prepubertal children with CD has not been adequately studied.", "To determine the vitamin D status of prepubertal children with CD.", "A retrospective study of prepubertal children aged 3-12 years with CD (n=24) who were compared to prepubertal, non-CD children of the same age (n=50). Children were included in the study if they had a diagnosis of CD by intestinal biopsy, and were not on a gluten-free diet (GFD). Patients were excluded if they had diseases of calcium or vitamin D metabolism, or were receiving calcium or vitamin D supplementation or had other autoimmune diseases. All subjects had their serum 25-hydroxyvitamin D [25(OH)D] level measured.", "There was no difference in 25(OH)D level between the CD and non-CD children (27.58 +/- 9.91 versus 26.20 +/- 10.45, p = 0.59). However, when the patients were subdivided into obese and non-obese groups, the non-obese CD patients had a significantly higher 25(OH)D level than the obese normal children (28.39 +/- 10.26 versus 21.58 +/- 5.67, p = 0.009). In contrast, there was no difference in 25(OH)D level between non-obese CD patients and non-obese normal children (28.39 +/- 10.26 versus 30.64 +/-12.08, p = 0.52). The season of 25(OH)D measurement was not a significant confounder (p =0.7)." ], "LABELS": [ "BACKGROUND", "OBJECTIVE", "STUDY DESIGN", "RESULTS" ], "MESHES": [ "Biopsy", "Celiac Disease", "Child", "Child, Preschool", "Female", "Humans", "Intestinal Absorption", "Intestinal Mucosa", "Male", "Retrospective Studies", "Vitamin D", "Vitamin D Deficiency" ], "YEAR": "2012", "reasoning_required_pred": "maybe", "reasoning_free_pred": "no", "final_decision": "no", "LONG_ANSWER": "Our data showed no difference in 25(OH) D levels between normal children and those with CD when adjusted for body mass index." }, "15280782": { "QUESTION": "Is unsafe sexual behaviour increasing among HIV-infected individuals?", "CONTEXTS": [ "The number of new diagnoses of HIV infection is rising in the northwestern hemisphere and it is becoming increasingly important to understand the mechanisms behind this trend.", "To evaluate whether reported unsafe sexual behaviour among HIV- infected individuals is changing over time.", "Participants in the Swiss HIV Cohort Study were asked about their sexual practices every 6 months for 3 years during regular follow-up of the cohort beginning on 1 April 2000.", ": Logistic regression models were fit using generalized estimating equations assuming a constant correlation between responses from the same individual.", "At least one sexual behaviour questionnaire was obtained for 6545 HIV-infected individuals and the median number of questionnaires completed per individual was five. There was no evidence of an increase in reported unsafe sex over time in this population [odds ratio (OR), 1.0; 95% confidence interval (CI), 0.96-1.05]. Females (OR, 1.38; 95% CI, 1.19-1.60), 15-30 year olds (OR, 1.26; 95% CI, 1.09-1.47), those with HIV-positive partners (OR, 12.58; 95% CI, 10.84-14.07) and those with occasional partners (OR, 3.25; 95% CI, 2.87-3.67) were more likely to report unsafe sex. There was no evidence of a response bias over time, but individuals were less willing to leave questions about their sexual behaviour unanswered or ambiguous (OR, 0.93; 95% CI, 0.90-0.97)." ], "LABELS": [ "BACKGROUND", "OBJECTIVE", "DESIGN", "METHODS", "RESULTS" ], "MESHES": [ "Adolescent", "Adult", "Age Factors", "Female", "HIV Infections", "HIV Seropositivity", "Humans", "Longitudinal Studies", "Male", "Risk Factors", "Risk-Taking", "Sex Factors", "Sexual Behavior", "Sexual Partners", "Substance-Related Disorders" ], "YEAR": "2004", "reasoning_required_pred": "no", "reasoning_free_pred": "no", "final_decision": "no", "LONG_ANSWER": "There was no evidence of a trend in unsafe sex behaviour over time. However, several subgroups were identified as being more likely to report unsafe sex and should be targeted for specific interventions." }, "17502203": { "QUESTION": "Cholestasis associated with small bowel atresia: do we always need to investigate?", "CONTEXTS": [ "Cholestasis occurs frequently in patients with small bowel atresia (SBA) and is often attributed to prolonged parental nutrition. When severe or prolonged, patients may undergo unnecessary intensive or invasive investigation. We characterized cholestasis and analyzed the pertinence of investigating this patient population.", "With Research Ethics Board approval, patients with SBA between 1996 and 2005 were retrospectively reviewed. Demographics, location of atresia, operative findings, complications, investigations, resumption of feeding, duration of prolonged parental nutrition, and follow-up information were examined. Cholestasis was evaluated for incidence, severity, and evolution.", "Fifty-five patients (29 male, 26 female), with a median gestational age and birth weight of 36 weeks and 2025 g, respectively, were reviewed. Care was withdrawn for 2 patients before repair. For the remaining 53 patients, SBA were duodenal atresia in 18, jejunoileal atresia in 32, and multiple atresia in 3. Of 53, 24 (45%) patients developed cholestasis postoperatively (direct/total bilirubin>20%). All patients with short bowel (4) and 60% (6/10) of patients with a delay of enteral feeding more than 14 days postoperatively had cholestasis. Ten patients (36%) proceeded with in-depth evaluations for cholestasis, with 8 (28%) undergoing liver biopsy. No patient had biliary atresia. No deaths were related to isolated cholestasis/cirrhosis. Cholestasis resolved spontaneously in all the survivors." ], "LABELS": [ "PURPOSE", "METHODS", "RESULTS" ], "MESHES": [ "Cholestasis", "Female", "Gestational Age", "Humans", "Incidence", "Infant, Newborn", "Infant, Premature", "Intestinal Atresia", "Intestine, Small", "Male", "Parenteral Nutrition", "Retrospective Studies", "Risk Factors" ], "YEAR": "2007", "reasoning_required_pred": "maybe", "reasoning_free_pred": "no", "final_decision": "no", "LONG_ANSWER": "Small bowel atresia is frequently associated with postoperative cholestasis that will resolve with time. We recommend a more selective and expectant approach to SBA-associated cholestasis to minimize unnecessary investigations." }, "26370095": { "QUESTION": "Are financial incentives cost-effective to support smoking cessation during pregnancy?", "CONTEXTS": [ "To investigate the cost-effectiveness of up to \u00a3400 worth of financial incentives for smoking cessation in pregnancy as an adjunct to routine health care.", "Cost-effectiveness analysis based on a Phase II randomized controlled trial (RCT) and a cost-utility analysis using a life-time Markov model.", "The RCT was undertaken in Glasgow, Scotland. The economic analysis was undertaken from the UK National Health Service (NHS) perspective.", "A total of 612 pregnant women randomized to receive usual cessation support plus or minus financial incentives of up to \u00a3400 vouchers (US $609), contingent upon smoking cessation.", "Comparison of usual support and incentive interventions in terms of cotinine-validated quitters, quality-adjusted life years (QALYs) and direct costs to the NHS.", "The incremental cost per quitter at 34-38 weeks pregnant was \u00a31127 ($1716).This is similar to the standard look-up value derived from Stapleton&West's published ICER tables, \u00a31390 per quitter, by looking up the Cessation in Pregnancy Incentives Trial (CIPT) incremental cost (\u00a3157) and incremental 6-month quit outcome (0.14). The life-time model resulted in an incremental cost of \u00a317 [95% confidence interval (CI)\u2009=\u2009-\u00a393, \u00a3107] and a gain of 0.04 QALYs (95% CI\u2009=\u2009-0.058, 0.145), giving an ICER of \u00a3482/QALY ($734/QALY). Probabilistic sensitivity analysis indicates uncertainty in these results, particularly regarding relapse after birth. The expected value of perfect information was \u00a330 million (at a willingness to pay of \u00a330\u2009000/QALY), so given current uncertainty, additional research is potentially worthwhile." ], "LABELS": [ "AIMS", "DESIGN", "SETTING", "PARTICIPANTS", "MEASUREMENTS", "FINDINGS" ], "MESHES": [ "Cost-Benefit Analysis", "Female", "Health Promotion", "Humans", "Markov Chains", "Motivation", "Pregnancy", "Pregnancy Complications", "Prenatal Care", "Quality-Adjusted Life Years", "Scotland", "Smoking", "Smoking Cessation", "Smoking Prevention" ], "YEAR": "2016", "reasoning_required_pred": "maybe", "reasoning_free_pred": "yes", "final_decision": "yes", "LONG_ANSWER": "Financial incentives for smoking cessation in pregnancy are highly cost-effective, with an incremental cost per quality-adjusted life years of \u00a3482, which is well below recommended decision thresholds." }, "10354335": { "QUESTION": "Can medical students contribute to quality assurance programmes in day surgery?", "CONTEXTS": [ "Health care delivery has undertaken a major shift from inpatient management to ambulatory surgical care with increasing emphasis on quality assurance (QA) processes. Educational opportunities for medical undergraduate programmes are being sought in the day surgery environment. Our study was undertaken to explore ways in which senior medical students can actively contribute to QA processes as part of an undergraduate day surgery educational programme.", "Health care delivery has undertaken a major shift from inpatient management to ambulatory surgical care with increasing emphasis on quality assurance (QA) processes. Educational opportunities for medical undergraduate programmes are being sought in the day surgery environment. Our study was undertaken to explore ways in which senior medical students can actively contribute to the QA processes as part of an undergraduate day surgery educational programme.", "Fifty-nine final year medical students followed allocated patients with common surgical conditions through all phases of the day surgery process. Students kept records about each case in a log book and also presented their cases at weekly Problem Based Learning tutorials. An audit of student log books and review of tutorial records was conducted for the 1996 and 1997 academic years, in order to evaluate student contribution to QA.", "Students followed 621 cases, representing a sampling of 14. 1% day surgery cases. Categories of problems highlighted by students included inappropriate patient and procedure selection, inadequate pain management, discharge, communication and resource issues. Students made a number of recommendations including the development of multilingual videotapes and patient information sheets for non-English speaking patients, avoidance of bilateral surgical procedures and improved links with local medical officers. They also developed new guidelines and protocols." ], "LABELS": [ "CONTEXT AND OBJECTIVES", "SUBJECTS AND METHODS", "SUBJECTS AND METHODS", "RESULTS" ], "MESHES": [ "Ambulatory Surgical Procedures", "Australia", "Humans", "Quality Assurance, Health Care", "Students, Medical" ], "YEAR": "1999", "reasoning_required_pred": "yes", "reasoning_free_pred": "yes", "final_decision": "yes", "LONG_ANSWER": "Our study confirms that students are able to identify QA issues and propose solutions. We recommend that students have a formally recognized place in day surgery QA programmes, to close the QA loop and to adequately prepare them for medical practice in the 21st century." }, "26452334": { "QUESTION": "Measurement of head and neck paragangliomas: is volumetric analysis worth the effort?", "CONTEXTS": [ "The aim of this study was to assess the reproducibility of different measurement methods and define the most workable technique for measuring head and neck paragangliomas, to determine the best method for evaluating tumour growth. The evaluation of tumour growth is vital for a 'wait-and-scan' policy, a management strategy that became increasingly important.", "Method comparison study.", "Thirty tumours, including carotid body, vagal body, jugulotympanic tumours and conglomerates of multiple tumours, were measured in duplicate, using linear dimensions, manual area tracing and an automated segmentation method.", "Reproducibility was assessed using the Bland-Altman method.", "The smallest detectable difference using the linear dimension method was 11% for carotid body and 27% for vagal body tumours, compared with 17% and 20% for the manual area tracing method. Due to the irregular shape of paragangliomas in the temporal bone and conglomerates, the manual area tracing method showed better results in these tumours (26% and 8% versus 54% and 47%). The linear dimension method was significantly faster (median 4.27 versus 18.46 minutes, P<0.001). The automatic segmentation method yielded smallest detectable differences between 39% and 75%, and although fast (2.19 \u00b1 1.49 minutes), it failed technically." ], "LABELS": [ "OBJECTIVES", "STUDY DESIGN", "SETTING AND PARTICIPANTS", "MAIN OUTCOME MEASURES", "RESULTS" ], "MESHES": [ "Female", "Head and Neck Neoplasms", "Humans", "Image Interpretation, Computer-Assisted", "Magnetic Resonance Angiography", "Male", "Paraganglioma", "Reproducibility of Results", "Tumor Burden" ], "YEAR": "2016", "reasoning_required_pred": "yes", "reasoning_free_pred": "yes", "final_decision": "yes", "LONG_ANSWER": "Due to a relatively good reproducibility, fast and easy application, we found the linear dimension method to be the most pragmatic approach for evaluation of growth of carotid and vagal body paragangliomas. For jugulotympanic tumours, the preferred method is manual area tracing. However, volumetric changes of these tumours may be of less clinical importance than changes in relation to surrounding anatomical structures." }, "15065192": { "QUESTION": "Three-dimensional ultrasound-validated large-core needle biopsy: is it a reliable method for the histological assessment of breast lesions?", "CONTEXTS": [ "The use of three-dimensional (3D) ultrasound may help to determine the exact position of the needle during breast biopsy, thereby reducing the number of core samples that are needed to achieve a reliable histological diagnosis. The aim of this study was to demonstrate the efficacy of 3D ultrasound-validated large-core needle biopsy (LCNB) of the breast.", "A total of 360 core needle biopsies was obtained from 169 breast lesions in 146 patients. Additional open breast biopsy was performed in 111 women (127/169 breast lesions); the remaining 42 lesions were followed up for at least 24 months. 3D ultrasound visualization of the needle in the postfiring position was used to classify the biopsy as central, marginal or outside the lesion. Based on this classification it was decided whether another sample had to be obtained.", "A median of two core samples per lesion provided for all the lesions a sensitivity for malignancy of 96.9%, specificity of 100%, false-positive rate of 0% and false-negative rate of 3.1%, and for the excised lesions a sensitivity of 96.5%, specificity of 100%, false-positive rate of 0%, false-negative rate of 3.5% and an underestimation rate of 3.4%." ], "LABELS": [ "OBJECTIVE", "METHODS", "RESULTS" ], "MESHES": [ "Biopsy, Needle", "Breast Diseases", "Breast Neoplasms", "False Negative Reactions", "False Positive Reactions", "Female", "Humans", "Needles", "Predictive Value of Tests", "Ultrasonography, Mammary" ], "YEAR": "2004", "reasoning_required_pred": "yes", "reasoning_free_pred": "yes", "final_decision": "yes", "LONG_ANSWER": "3D ultrasound validation of the postfiring needle position is an efficient adjunct to ultrasound-guided LCNB. The advantages of 3D ultrasound validation are likely to include a reduction in the number of core samples needed to achieve a reliable histological diagnosis (and a possible reduction in the risk of tumor cell displacement), reduced procedure time and lower costs." }, "25488308": { "QUESTION": "Can bone thickness and inter-radicular space affect miniscrew placement in posterior mandibular sites?", "CONTEXTS": [ "The proper angle of miniscrew insertion is important for cortical anchorage, patient safety, and biomechanical control. The purposes of this study are to report the alveolar process thickness and inter-radicular space in the posterior region of the mandible, to assess the impact of different miniscrew insertion angle protocols, and to identify differences between the genders or types of malocclusion.", "In this retrospective study, 100 individuals were selected for orthodontic treatment at a radiology clinic. Cone-beam computed tomography data were imported into 3-dimensional software. The predictor variable was the location in the mandible and insertion angle. The demographic variables collected included age, gender, and malocclusion (Angle Classes I and II). The primary outcome variables were bone thickness and inter-radicular space. The inter-radicular spaces were evaluated 5 mm from the cement-enamel junction. The bone thicknesses were taken at 45\u00b0, 60\u00b0, and 90\u00b0 in relation to the alveolar ridge, simulating a miniscrew insertion. These factors were evaluated for sexual dimorphism and malocclusion (Angle Classes I and II). Sexual dimorphism and malocclusion were evaluated with t tests. To compare the inter-radicular space and the thickness of bone between areas, an analysis of variance for repeated measures was used.", "The sample was composed of 100 patients with a mean age of 17.4 \u00b1 6.74 years. There were 61 female and 39 male patients and 60 Class I and 40 Class II molar relationships. The inter-radicular space ranged from 2.46 to 3.31 mm, and alveolar bone thickness ranged from 8.01 to 13.77 mm. The thickness tended to decrease with the increase in insertion angle from 45\u00b0 to 90\u00b0. No significant differences between the genders or types of malocclusion were found." ], "LABELS": [ "PURPOSE", "MATERIALS AND METHODS", "RESULTS" ], "MESHES": [ "Adolescent", "Adult", "Bone Screws", "Child", "Cone-Beam Computed Tomography", "Humans", "Mandible", "Orthodontics", "Retrospective Studies", "Young Adult" ], "YEAR": "2015", "reasoning_required_pred": "no", "reasoning_free_pred": "no", "final_decision": "no", "LONG_ANSWER": "The results of this study suggest that the safest areas for the placement of miniscrews are between the first and second premolars and between the first and second molars, regardless of the angle of insertion." }, "22449464": { "QUESTION": "Do general practice selection scores predict success at MRCGP?", "CONTEXTS": [ "Selection into general practice training is undertaken using a competency based approach. The clear advantage of this approach over traditional methods has been demonstrated through evaluation of its validity and reliability. However, the relationship between selection and performance in the Royal College of General Practitioner examinations (MRCGP) has yet to be explored. The MRCGP comprises of an applied knowledge test (AKT), a clinical skills assessment (CSA) and workplace-based assessments (WPBA).AIM: To explore the predictive validity of general practice selection scores using the AKT and CSA elements of the MRCGP as a final outcome measure.", "This study carried out a retrospective analysis of 101 trainees from the Wales Deanery who were successfully selected on to general practice training in 2007. Selection data consisted of an overall selection score as well as scores from each individual stage of selection. Correlation was used to explore associations between selection scores and examination scores.", "The score for overall performance at selection achieved statistically significant correlation with examination performance (r = 0.491 for the AKT and r = 0.526 for the CSA, P<0.01)." ], "LABELS": [ "BACKGROUND", "METHODS", "RESULTS" ], "MESHES": [ "Achievement", "Clinical Competence", "Female", "General Practice", "Humans", "Internship and Residency", "Male", "Retrospective Studies", "School Admission Criteria", "United Kingdom" ], "YEAR": "2012", "reasoning_required_pred": "yes", "reasoning_free_pred": "yes", "final_decision": "yes", "LONG_ANSWER": "The general practice selection process is predictive of future performance in the MRCGP." }, "26222664": { "QUESTION": "The Deformity Angular Ratio: Does It Correlate With High-Risk Cases for Potential Spinal Cord Monitoring Alerts in Pediatric 3-Column Thoracic Spinal Deformity Corrective Surgery?", "CONTEXTS": [ "A retrospective analysis.", "The purpose of this study was to determine whether the deformity angular ratio (DAR) can reliably assess the neurological risks of patients undergoing deformity correction.", "Identifying high-risk patients and procedures can help ensure that appropriate measures are taken to minimize neurological complications during spinal deformity corrections. Subjectively, surgeons look at radiographs and evaluate the riskiness of the procedure. However, 2 curves of similar magnitude and location can have significantly different risks of neurological deficit during surgery. Whether the curve spans many levels or just a few can significantly influence surgical strategies. Lenke et al have proposed the DAR, which is a measure of curve magnitude per level of deformity.", "The data from 35 pediatric spinal deformity correction procedures with thoracic 3-column osteotomies were reviewed. Measurements from preoperative radiographs were used to calculate the DAR. Binary logistic regression was used to model the relationship between DARs (independent variables) and presence or absence of an intraoperative alert (dependent variable).", "In patients undergoing 3-column osteotomies, sagittal curve magnitude and total curve magnitude were associated with increased incidence of transcranial motor evoked potential changes. Total DAR greater than 45\u00b0 per level and sagittal DAR greater than 22\u00b0 per level were associated with a 75% incidence of a motor evoked potential alert, with the incidence increasing to 90% with sagittal DAR of 28\u00b0 per level." ], "LABELS": [ "STUDY DESIGN", "OBJECTIVE", "SUMMARY OF BACKGROUND DATA", "METHODS", "RESULTS" ], "MESHES": [ "Electromyography", "Evoked Potentials, Motor", "Evoked Potentials, Somatosensory", "Humans", "Intraoperative Neurophysiological Monitoring", "Kyphosis", "Osteotomy", "Radiography", "Retrospective Studies", "Risk Assessment", "Spinal Cord Injuries", "Spinal Curvatures", "Spine" ], "YEAR": "2015", "reasoning_required_pred": "yes", "reasoning_free_pred": "yes", "final_decision": "yes", "LONG_ANSWER": "In patients undergoing 3-column osteotomies for severe spinal deformities, the DAR was predictive of patients developing intraoperative motor evoked potential alerts. Identifying accurate radiographical, patient, and procedural risk factors in the correction of severe deformities can help prepare the surgical team to improve safety and outcomes when carrying out complex spinal corrections." }, "22617083": { "QUESTION": "Does age moderate the effect of personality disorder on coping style in psychiatric inpatients?", "CONTEXTS": [ "To examine age-related differences in the relationship between personality and coping strategies in an Australian population of psychiatric inpatients.", "Consenting eligible adults (N=238) from 18-100 years of age consecutively admitted to inpatient psychiatry units were assessed using the SCID I and II, the Coping Orientations to Problems Experienced Scale (COPE), the Brief Psychiatric Rating Scale (BPRS), the Global Assessment of Functioning Scale (GAF), the Social and Occupational Functioning Assessment Scale (SOFAS), the 12 Item Short-Form Heath Survey (SF12), the Sarason Social Support Questionnaire, and the NEO Five Factor Inventory (NEO-FFI) (cognitively impaired, and non-English speaking patients were excluded).", "Older adults reported less symptomatology than younger patients and younger patients described more personality dysfunction than older patients. As assessed by the COPE, older adults reported lower levels of dysfunctional coping strategies than younger adults. Personality traits, social supports, gender, and age predicted coping strategies, while Axis I diagnosis, education, personality disorder, and symptom severity were not significant predictors of coping strategies." ], "LABELS": [ "OBJECTIVE", "METHOD", "RESULTS" ], "MESHES": [ "Adaptation, Psychological", "Adolescent", "Adult", "Age Factors", "Aged", "Aged, 80 and over", "Aging", "Australia", "Female", "Hospitals, Psychiatric", "Humans", "Male", "Middle Aged", "Multivariate Analysis", "Personality Disorders", "Prospective Studies", "Regression Analysis", "Sex Factors", "Social Support" ], "YEAR": "2012", "reasoning_required_pred": "yes", "reasoning_free_pred": "yes", "final_decision": "yes", "LONG_ANSWER": "This study found that influences on coping were multifactorial and moderated by age. These factors have implications for interventions designed to enhance coping strategies." }, "9767546": { "QUESTION": "Autoxidation products of both carbohydrates and lipids are increased in uremic plasma: is there oxidative stress in uremia?", "CONTEXTS": [ "Advanced glycation end products (AGEs), formed by non-enzymatic glycation and oxidation (glycoxidation) reactions, have been implicated in the pathogenesis of several diseases, including normoglycemic uremia. AGE research in uremia has focused on the accumulation of carbohydrate-derived adducts generated by the Maillard reaction. Recent studies, however, have demonstrated that one AGE, the glycoxidation product carboxymethyllysine (CML), could be derived not only from carbohydrates but also from oxidation of polyunsaturated fatty acids in vitro, raising the possibility that both carbohydrate and lipid autoxidation might be increased in uremia.", "To address this hypothesis, we applied gas chromatography-mass spectrometry and high performance liquid chromatography to measure protein adducts formed in uremic plasma by reactions between carbonyl compounds and protein amino groups: pentosidine derived from carbohydrate-derived carbonyls, malondialdehyde (MDA)-lysine derived from lipid-derived carbonyls, and CML originating possibly from both sources.", "All three adducts were elevated in uremic plasma. Plasma CML levels were mainly (>95%) albumin bound. Their levels were not correlated with fructoselysine levels and were similar in diabetic and non-diabetic patients on hemodialysis, indicating that their increase was not driven by glucose. Pentosidine and MDA-lysine were also increased in plasma to the same extent in diabetic and non-diabetic hemodialysis patients. Statistical analysis indicated that plasma levels of CML correlated weakly (P<0.05) with those of pentosidine and MDA-lysine, but that pentosidine and MDA-lysine varied independently (P>0.5)." ], "LABELS": [ "BACKGROUND", "METHODS", "RESULTS" ], "MESHES": [ "Adult", "Aged", "Arginine", "Carbohydrates", "Case-Control Studies", "Glycation End Products, Advanced", "Humans", "Lipid Peroxidation", "Lipids", "Lysine", "Malondialdehyde", "Middle Aged", "Oxidation-Reduction", "Oxidative Stress", "Uremia" ], "YEAR": "1998", "reasoning_required_pred": "no", "reasoning_free_pred": "yes", "final_decision": "yes", "LONG_ANSWER": "These data suggest that the increased levels of AGEs in blood, and probably in tissues, reported in uremia implicate a broad derangement in non-enzymatic biochemistry involving alterations in autoxidation of both carbohydrates and lipids." }, "21214884": { "QUESTION": "Can 'high-risk' human papillomaviruses (HPVs) be detected in human breast milk?", "CONTEXTS": [ "Using polymerase chain reaction techniques, we evaluated the presence of HPV infection in human breast milk collected from 21 HPV-positive and 11 HPV-negative mothers.", "Of the 32 studied human milk specimens, no 'high-risk' HPV 16, 18, 31, 33, 35, 39, 45, 51, 52, 56, 58 or 58 DNA was detected." ], "LABELS": [ "METHODS", "RESULTS" ], "MESHES": [ "Adult", "Alphapapillomavirus", "Case-Control Studies", "DNA, Viral", "Female", "Humans", "Infant", "Infectious Disease Transmission, Vertical", "Milk, Human", "Papillomavirus Infections", "Polymerase Chain Reaction", "Risk Assessment", "Young Adult" ], "YEAR": "2011", "reasoning_required_pred": "no", "reasoning_free_pred": "no", "final_decision": "no", "LONG_ANSWER": "This preliminary case-control study indicates the absence of mucosal 'high-risk' HPV types in human breast milk." }, "23321509": { "QUESTION": "Quaternary cytoreductive surgery in ovarian cancer: does surgical effort still matter?", "CONTEXTS": [ "To evaluate surgical outcome and survival benefit after quaternary cytoreduction (QC) in epithelial ovarian cancer (EOC) relapse.", "We systematically evaluated all consecutive patients undergoing QC in our institution over a 12-year period (October 2000-January 2012). All relevant surgical and clinical outcome parameters were systematically assessed.", "Forty-nine EOC patients (median age: 57; range: 28-76) underwent QC; in a median of 16 months (range:2-142) after previous chemotherapy. The majority of the patients had an initial FIGO stage III (67.3%), peritoneal carcinomatosis (77.6%) and no ascites (67.3%). At QC, patients presented following tumour pattern: lower abdomen 85.7%; middle abdomen 79.6% and upper abdomen 42.9%. Median duration of surgery was 292 min (range: a total macroscopic tumour clearance could be achieved. Rates of major operative morbidity and 30-day mortality were 28.6% and 2%, respectively.Mean follow-up from QC was 18.41 months (95% confidence interval (CI):12.64-24.18) and mean overall survival (OS) 23.05 months (95% CI: 15.5-30.6). Mean OS for patients without vs any tumour residuals was 43 months (95% CI: 26.4-59.5) vs 13.4 months (95% CI: 7.42-19.4); P=0.001. Mean OS for patients who received postoperative chemotherapy (n=18; 36.7%) vs those who did not was 40.5 months (95% CI: 27.4-53.6) vs 12.03 months (95% CI: 5.9-18.18); P<0.001.Multivariate analysis indentified multifocal tumour dissemination to be of predictive significance for incomplete tumour resection, higher operative morbidity and lower survival, while systemic chemotherapy subsequent to QC had a protective significant impact on OS. No prognostic impact had ascites, platinum resistance, high grading and advanced age." ], "LABELS": [ "BACKGROUND", "METHODS", "RESULTS" ], "MESHES": [ "Adult", "Aged", "Chemotherapy, Adjuvant", "Female", "Humans", "Middle Aged", "Neoplasm Invasiveness", "Neoplasms, Glandular and Epithelial", "Ovarian Neoplasms", "Prognosis", "Recurrence", "Survival Rate", "Treatment Outcome" ], "YEAR": "2013", "reasoning_required_pred": "yes", "reasoning_free_pred": "yes", "final_decision": "yes", "LONG_ANSWER": "Even in this highly advanced setting of the third EOC relapse, maximal therapeutic effort combining optimal surgery and chemotherapy appear to significantly prolong survival in a selected patients 'group'." }, "16827975": { "QUESTION": "Chemotherapy and survival in advanced non-small cell lung carcinoma: is pneumologists' skepticism justified?", "CONTEXTS": [ "Few studies have assessed whether the advantage chemotherapy has been shown to have in treating advanced non-small lung carcinoma in clinical trials is transferrable to normal health care activity. This could explain the skepticism of a large number of pneumologists towards this treatment. The objective of our study was to analyze prognostic factors related to survival and to see whether cytostatic treatment was an independent predictor.", "Patients enrolled in the study had been diagnosed with non-small cell carcinoma in stages IV or IIIB with pleural or N2-N3 involvement and with a performance status of 2 or below according to the Eastern Cooperative Oncology Group (ECOG). Survival was analyzed with regard to the following variables: age, sex, comorbidity, weight loss, laboratory test results, histological type, ECOG score, TNM staging, and treatment. The Student t test, the chi(2) test, the Kaplan-Meier method, the log-rank test, and Cox regression analysis were used in the statistical analysis.", "We enrolled 190 patients (157 men and 33 women) with a mean (SD) age of 61.75 (10.85) years (range, 33-85 years). Of these patients, 144 received cytostatic treatment and 46 palliative treatment. The median survival was 31 weeks and was related to absence of weight loss (hazard ratio [HR], 1.73; 95% confidence interval [CI], 1.26-2.39; P=.001), cytostatic treatment (HR, 1.85; 95% CI, 1.25-2.76; P=.002), and ECOG score of 0 to 1 (HR, 2.84; 95% CI, 1.62-5.00; P=.0001). In patients with ECOG scores of 0 to 1, weight loss and treatment were significant prognostic factors. Survival in the ECOG 2 group was 15 weeks for patients undergoing cytostatic treatment and 11 weeks for patients with symptomatic treatment." ], "LABELS": [ "OBJECTIVE", "PATIENTS AND METHODS", "RESULTS" ], "MESHES": [ "Aged", "Carcinoma, Non-Small-Cell Lung", "Disease Progression", "Female", "Humans", "Lung Neoplasms", "Male", "Middle Aged", "Neoplasm Staging", "Prognosis", "Pulmonary Medicine", "Retrospective Studies", "Survival Analysis", "Survival Rate" ], "YEAR": "2006", "reasoning_required_pred": "yes", "reasoning_free_pred": "no", "final_decision": "no", "LONG_ANSWER": "In normal clinical practice, chemotherapy significantly prolongs survival in patients with performance status of less than 2, more time being gained if there is no associated weight loss. We conclude that the reluctance shown by many pneumologists toward using this treatment is not entirely justified." }, "18179827": { "QUESTION": "Does topical ropivacaine reduce the post-tonsillectomy morbidity in pediatric patients?", "CONTEXTS": [ "To determine whether post-operative administration of topical ropivacaine hydrochloride decreases morbidity following adenotonsillectomy.", "Prospective, randomized, double-blind clinical trial.", "University referral center; ENT Department.", "Fourty one children, aged 4-16 years, undergoing tonsillectomy.", "Patients received 1.0% ropivacaine hydrochloride soaked swabs packed in their tonsillar fossae while the control group received saline-soaked swabs. Mc Grath's face scale was used to compare the two groups in respect of pain control. Chi-square and two-tailed unpaired Student's t-tests or Mann-Whitney-U-tests were used to compare the two independent groups. As 10 we made 11 comparison between groups, for Bonferroni correction, p<0.005 was accepted as statistically significant.", "Only first hour there was no significant pain-relieving effect seen in the ropivacaine group (p>0.05). The other hours and days there were statistically significance between the two groups (p<0.001). Also, the other post-operative parameters such as nausea, fever, vomiting, odor, bleeding, otalgia and trismus were not statistically different between the two groups. There were no complications associated with ropivacaine hydrochloride. No patients in this study suffered systemic side effects related to the use of this medication." ], "LABELS": [ "OBJECTIVES", "STUDY DESIGN", "SETTING", "PARTICIPANTS", "METHODS", "RESULTS" ], "MESHES": [ "Administration, Topical", "Adolescent", "Amides", "Anesthetics, Local", "Child", "Child, Preschool", "Double-Blind Method", "Female", "Humans", "Male", "Pain Measurement", "Pain, Postoperative", "Prospective Studies", "Time Factors", "Tonsillectomy" ], "YEAR": "2008", "reasoning_required_pred": "yes", "reasoning_free_pred": "yes", "final_decision": "yes", "LONG_ANSWER": "Locally 1.0% ropivacaine administration significantly relieves the pain of pediatric tonsillectomy and, it is a safe and effective method. High concentrations of ropivaciane may produce clinically significant pain relief. It is more effective to reduce of post-operative analgesic requirement after first hour." }, "14872327": { "QUESTION": "Is pain a clinically relevant problem in general adult psychiatry?", "CONTEXTS": [ "To study the prevalence of pain and risk factors for pain in psychiatric patients in a psychiatric hospital.", "Using a questionnaire we investigated in a cross-sectional study the prevalence of pain, duration of pain, impairment and unfitness for work due to pain in 106 patients primarily diagnosed with a psychiatric disorder in the field of general adult psychiatry. Potential risk factors were explored.", "The point prevalence of pain was about 50%, the 6-month prevalence 75.5% and the 12-month prevalence 76.5%. The patients' most frequent complaints were low back pain, headache and shoulder and neck pain. Patients with affective disorders most frequently had pain complaints, followed by those with neurotic, stress-related and somatoform disorders and those with psychotic disorders such as schizophrenia, schizotypic and delusional disorders. Almost 10% of all patients reported pain continuing at least 3 months in the past year. Impairment and unfitness for work were related to specific psychiatric diagnosis. Statistically significant risk factors for pain were depression (OR=6.05) and the number of past admissions to psychiatric hospitals (OR=3.609)." ], "LABELS": [ "OBJECTIVE", "METHODS", "RESULTS" ], "MESHES": [ "Adult", "Cross-Sectional Studies", "Germany", "Hospitals, Psychiatric", "Humans", "Incidence", "Mental Disorders", "Pain", "Pain Measurement", "Prevalence", "Psychiatry", "Surveys and Questionnaires" ], "YEAR": "2004", "reasoning_required_pred": "yes", "reasoning_free_pred": "yes", "final_decision": "yes", "LONG_ANSWER": "We found evidence that pain can be a significant clinical problem in psychiatric patients which seems to be underestimated in psychiatry. The investigated patients in general adult psychiatry are characterized by specific risk factors different from clinical subpopulations of other disciplines." }, "24809662": { "QUESTION": "Does concomitant anterior/apical repair during midurethral sling improve the overactive bladder component of mixed incontinence?", "CONTEXTS": [ "Midurethral sling (MUS) can improve overactive bladder (OAB) symptoms. It is unclear if anterior/apical prolapse (AA) repair provides additional benefit. We hypothesized that women with mixed urinary incontinence (MUI) experience greater improvement in the OAB component of their symptoms after concomitant MUS and AA repair compared with MUS alone.", "This is a retrospective cohort study of women with bothersome MUI (defined by objective stress test and validated questionnaire) undergoing MUS alone (\"MUS-only\") or concomitant MUS and AA repair (\"MUS + AA\"). Our primary outcome was the Overactive Bladder Questionnaire Symptom Severity (OAB-q SS) change score 6\u00a0weeks after surgery.", "Of 151 women, 67 (44\u00a0%) underwent MUS-only and 84 (56\u00a0%) underwent MUS + AA. The MUS-only cohort was younger and had less severe baseline prolapse (p\u2009<\u20090.05 for both). Postoperative complications (predominantly UTI) occurred in 35 (23\u00a0%) patients and were similar between cohorts. For all subjects mean OAB-q SS scores significantly improved postoperatively (p\u2009<\u20090.05). Our primary outcome, OAB-q SS change score, showed no significant differences between cohorts (30\u2009\u00b1\u200926 MUS-only vs 25\u2009\u00b1\u200925 MUS + AA, p\u2009=\u20090.20), indicating similar improvements in OAB symptoms. Multivariate linear regression analysis revealed no difference in OAB-q SS change score between cohorts; however, OAB-q SS change scores were lower for women with a postoperative complication (\u03b2\u2009=\u2009-19, 95\u00a0% CI -31 to -6; p\u2009<\u20090.01)." ], "LABELS": [ "INTRODUCTION AND HYPOTHESIS", "METHODS", "RESULTS" ], "MESHES": [ "Aged", "Female", "Gynecologic Surgical Procedures", "Humans", "Middle Aged", "Pelvic Organ Prolapse", "Retrospective Studies", "Suburethral Slings", "Treatment Outcome", "Urinary Bladder, Overactive", "Urinary Incontinence" ], "YEAR": "2014", "reasoning_required_pred": "no", "reasoning_free_pred": "no", "final_decision": "no", "LONG_ANSWER": "In women with bothersome MUI, concomitant AA repair does not result in additional improvement in OAB symptoms over MUS alone. Patients with postoperative complications exhibit less improvement in OAB symptoms." }, "26485091": { "QUESTION": "Does the use of atypical antipsychotics as adjunctive therapy in depression result in cost savings?", "CONTEXTS": [ "Several atypical antipsychotics (AAPs) are used as second-line agents for treatment resistant depression. AAPs can be expensive compared to other treatment options and can cause several side effects.", "To estimate healthcare costs and utilization of AAPs compared to other second-line agents.", "Observational study using Medicaid claims data (2006-2011). Subjects were depression-diagnosed adult members with at least two prescriptions of antidepressant medications followed by a second-line agent. Gamma generalized linear models (GLM) produced estimates of the difference in mean expenditures among treatment groups after adjusting for individual baseline characteristics using propensity scores. Negative binomial models produced estimates of the difference in number of hospitalizations and emergency department (ED) visits.", "A total of 3910 members received second-line treatment. Treatment groups were AAPs (n\u2009=\u20092211), augmentation agents other than AAPs (n\u2009=\u20091008), and antidepressant switching (n\u2009=\u2009691). AAPs resulted in higher mean adjusted pharmacy costs and higher mean adjusted total mental health-related costs. Mean adjusted total healthcare costs and number of inpatient and ED visits were not different among treatments." ], "LABELS": [ "BACKGROUND", "OBJECTIVES", "METHODS", "RESULTS" ], "MESHES": [ "Adult", "Antipsychotic Agents", "Cost Savings", "Depressive Disorder", "Drug Therapy, Combination", "Female", "Health Expenditures", "Humans", "Male", "Medicaid", "Middle Aged", "United States", "Young Adult" ], "YEAR": "2016", "reasoning_required_pred": "no", "reasoning_free_pred": "no", "final_decision": "no", "LONG_ANSWER": "The results show no evidence that AAPs used as second-line treatment for depression results in overall cost savings or lower inpatient and ED visits compared to other treatment strategies." }, "12805495": { "QUESTION": "Can patients be anticoagulated after intracerebral hemorrhage?", "CONTEXTS": [ "Warfarin increases both the likelihood and the mortality of intracerebral hemorrhage (ICH), particularly in patients with a history of prior ICH. In light of this consideration, should a patient with both a history of ICH and a clear indication for anticoagulation such as nonvalvular atrial fibrillation be anticoagulated? In the absence of data from a clinical trial, we used a decision-analysis model to compare the expected values of 2 treatment strategies-warfarin and no anticoagulation-for such patients.", "We used a Markov state transition decision model stratified by location of hemorrhage (lobar or deep hemispheric). Effectiveness was measured in quality-adjusted life years (QALYs). Data sources included English language literature identified through MEDLINE searches and bibliographies from selected articles, along with empirical data from our own institution. The base case focused on a 69-year-old man with a history of ICH and newly diagnosed nonvalvular atrial fibrillation.", "For patients with prior lobar ICH, withholding anticoagulation therapy was strongly preferred, improving quality-adjusted life expectancy by 1.9 QALYs. For patients with prior deep hemispheric ICH, withholding anticoagulation resulted in a smaller gain of 0.3 QALYs. In sensitivity analyses for patients with deep ICH, anticoagulation could be preferred if the risk of thromboembolic stroke is particularly high." ], "LABELS": [ "BACKGROUND AND PURPOSE", "METHODS", "RESULTS" ], "MESHES": [ "Aged", "Anticoagulants", "Aspirin", "Atrial Fibrillation", "Cerebral Hemorrhage", "Cohort Studies", "Decision Support Techniques", "Fibrinolytic Agents", "Humans", "Intracranial Thrombosis", "Male", "Markov Chains", "Quality-Adjusted Life Years", "Risk", "Risk Assessment", "Sensitivity and Specificity", "Warfarin" ], "YEAR": "2003", "reasoning_required_pred": "maybe", "reasoning_free_pred": "maybe", "final_decision": "maybe", "LONG_ANSWER": "Survivors of lobar ICH with atrial fibrillation should not be offered long-term anticoagulation. Similarly, most patients with deep hemispheric ICH and atrial fibrillation should not receive anticoagulant therapy. However, patients with deep hemispheric ICH at particularly high risk for thromboembolic stroke or low risk of ICH recurrence might benefit from long-term anticoagulation." }, "19925761": { "QUESTION": "Diagnostic and therapeutic ureteroscopy: is dilatation of ureteral meatus always necessary?", "CONTEXTS": [ "To assess the feasibility and safety of diagnostic or therapeutic semirigid ureteroscopy without ureteral meatus dilatation.", "A comparative, retrospective study was conducted of patients undergoing ureteroscopy from January 2000 to May 2008. For data analysis purposes, the population was divided into two groups based on whether ureteroscopy had been performed with (Group 1) or without (Group 2) ureteral meatus dilatation. Variables recorded included age, sex, type of procedure, surgical diagnosis, passive or active dilatation, number of stones, stone location, stone diameter, peroperative and postoperative complications, internal urinary diversion after the procedure, therapeutic success rate, operating time, and hospital stay duration. A 8-9.8 Fr Wolf semirigid ureteroscope was used. Descriptive statistics of the population and cohorts were performed, providing medians, quartiles, and limit values for non-normally distributed interval variables, and absolute and relative frequencies for categorical variables. Shapiro-Wilk's, Mann-Whitney's U, Chi-square, and Fisher's exact tests were used for statistical analysis. A value of p 2 alpha\u20090.05. No response to load was seen in either T1DM or control group over the 4-days post exercise." ], "LABELS": [ "BACKGROUND", "METHODS", "RESULTS" ], "MESHES": [ "Achilles Tendon", "Adult", "Case-Control Studies", "Diabetes Mellitus, Type 1", "Female", "Humans", "Male", "Middle Aged", "Running", "Weight-Bearing" ], "YEAR": "2015", "reasoning_required_pred": "no", "reasoning_free_pred": "no", "final_decision": "no", "LONG_ANSWER": "Active individuals with T1DM do not have a heightened Achilles tendon response to load, which suggests no increased risk of tendon injury. We cannot extrapolate these findings to sedentary individuals with T1DM." }, "12612531": { "QUESTION": "Fragility of the esophageal mucosa: a pathognomonic endoscopic sign of primary eosinophilic esophagitis?", "CONTEXTS": [ "Primary eosinophilic esophagitis, a chronic inflammatory disorder of the esophagus, evokes recurrent dysphagia. Endoscopy is often unremarkable, and no consensus exists regarding management of resultant dysphagia. The response of a series of patients with primary eosinophilic esophagitis to dilation is reported together with a description of a possibly pathognomonic sign: fragile esophageal mucosa, for which the term \"cr\u00eape-paper\" mucosa is introduced.", "Five men underwent endoscopy because of dysphagia confirmed (clinically, endoscopically, and histologically) to be caused by primary eosinophilic esophagitis and were treated by bouginage.", "All patients had extremely fragile, inelastic, and delicate mucosa, which tore easily even with minor trauma. After the procedure, patients remained asymptomatic for 3 to 24 months." ], "LABELS": [ "BACKGROUND", "METHODS", "OBSERVATIONS" ], "MESHES": [ "Adult", "Deglutition Disorders", "Dilatation", "Eosinophilia", "Esophagitis", "Esophagoscopy", "Esophagus", "Humans", "Male", "Middle Aged", "Mucous Membrane" ], "YEAR": "2003", "reasoning_required_pred": "yes", "reasoning_free_pred": "yes", "final_decision": "yes", "LONG_ANSWER": "Primary eosinophilic esophagitis is characterized by fragile esophageal mucosa that readily tears in response to minor trauma during otherwise uneventful diagnostic endoscopy. This \"cr\u00eape-paper\" sign may alert endoscopists to the presence of the disease when other mucosal alterations are lacking. Dilation is effective for patients with symptoms with minimal morbidity, despite development of disquieting lesions in response to the procedure." }, "26194560": { "QUESTION": "Does cup-cage reconstruction with oversized cups provide initial stability in THA for osteoporotic acetabular fractures?", "CONTEXTS": [ "The incidence of acetabular fractures in osteoporotic patients is increasing. Immediate total hip arthroplasty (THA) has potential advantages, but achieving acetabular component stability is challenging and, at early followup, reported revision rates for loosening are high.QUESTIONS/", "This study measured acetabular component stability and the initial surface contact achieved between the acetabular component and unfractured region of the pelvis after THA using an oversized acetabular component and cup-cage reconstruction.", "Between November 2011 and November 2013, we treated 40 acute acetabular fractures in patients older than 70 years of age. Of these, 12 (30%) underwent immediate THA using an oversized acetabular component with screws inserted only into the ilium and a cup-cage construct. Postoperatively all patients were mobilized without weightbearing restrictions. Indications for immediate THA after acetabular fractures were displaced articular comminution deemed unreducible. Eleven of the 12 were prospectively studied to evaluate the initial stability of the reconstructions using radiostereometric analysis. One of the patients died of a pulmonary embolism after surgery, and the remaining 10 (median age, 81 years; range, 72-86 years) were studied. Of these, five were analyzed at 1 year and five were analyzed at 2 years. Acetabular component migration was defined as acceptable if less than the limits for primary THA that predict later loosening (1.76 mm of proximal migration and 2.53\u00b0 of sagittal rotation). The contact surface between the acetabular component and ilium in direct continuity with the sacroiliac joint, and the ischium and pubis in direct continuity with the symphysis pubis, was measured on postoperative CT scans.", "At 1 year the median proximal migration was 0.83 mm (range, 0.09-5.13 mm) and sagittal rotation was 1.3\u00b0 (range, 0.1\u00b0-7.4\u00b0). Three of the 10 components had migration above the suggested limits for primary THA at 1 year postoperatively. The contact surface achieved at surgery between the acetabular component and pelvis ranged from 11 to 17 cm(2) (15%-27% of each component)." ], "LABELS": [ "BACKGROUND", "PURPOSES", "METHODS", "RESULTS" ], "MESHES": [ "Acetabulum", "Aged", "Aged, 80 and over", "Arthroplasty, Replacement, Hip", "Biomechanical Phenomena", "Bone Density", "Female", "Hip Fractures", "Hip Prosthesis", "Humans", "Joint Instability", "Male", "Osteoporotic Fractures", "Prospective Studies", "Prosthesis Design", "Prosthesis Failure", "Radiography", "Risk Factors", "Time Factors", "Treatment Outcome" ], "YEAR": "2015", "reasoning_required_pred": "maybe", "reasoning_free_pred": "yes", "final_decision": "yes", "LONG_ANSWER": "The majority of acetabular components in this cohort were stable despite the small contact surface achieved between the component and pelvic bone. Three of 10 migrated in excess of the limits that predict later loosening in primary THA but it remains to be seen whether these limits apply to this selected group of frail osteoporotic patients. We continue to use this technique routinely to treat patients with the same indications, but since the analysis of these data we have added screw fixation of the acetabular component to the ischial tuberosity and the superior pubic ramus." }, "11079675": { "QUESTION": "Pulmonary valve replacement in adults late after repair of tetralogy of fallot: are we operating too late?", "CONTEXTS": [ "The purpose of this study is to evaluate right ventricular (RV) volume and function after pulmonary valve replacement (PVR) and to address the issue of optimal surgical timing in these patients.", "Chronic pulmonary regurgitation (PR) following repair of tetralogy of Fallot (TOF) leads to RV dilation and an increased incidence of sudden cardiac death in adult patients.", "We studied 25 consecutive adult patients who underwent PVR for significant PR late after repair of TOF. Radionuclide angiography was performed in all at a mean of 8.2 months (+/- 8 months) before PVR and repeated at a mean of 28.0 months (+/- 22.8 months) after the operation. Right ventricular (RV) end-systolic volume (RVESV), RV end-diastolic volume (RVEDV) and RV ejection fraction (RVEF) were measured.", "Mean RVEDV, RVESV and RVEF remained unchanged after PVR (227.1 ml versus 214.9 ml, p = 0.74; 157.4 ml versus 155.4 ml, p = 0.94; 35.6% versus 34.7%, p = 0.78, respectively). Of the 10 patients with RVEF>or = 0.40 before PVR, 5 patients (50%) maintained a RVEF>or = 0.40 following PVR, whereas only 2 out of 15 patients (13%) with pre-operative values<0.40 reached an RVEF>or = 0.40 postoperatively (p<0.001)." ], "LABELS": [ "OBJECTIVES", "BACKGROUND", "METHODS", "RESULTS" ], "MESHES": [ "Adolescent", "Adult", "Child", "Child, Preschool", "Female", "Heart Valve Prosthesis", "Humans", "Male", "Pulmonary Valve", "Pulmonary Valve Insufficiency", "Radionuclide Angiography", "Survival Rate", "Tetralogy of Fallot", "Time Factors", "Ventricular Function, Right" ], "YEAR": "2000", "reasoning_required_pred": "no", "reasoning_free_pred": "yes", "final_decision": "yes", "LONG_ANSWER": "Right ventricular recovery following PVR for chronic significant pulmonary regurgitation after repair of TOF may be compromised in the adult population. In order to maintain adequate RV contractility, pulmonary valve implant in these patients should be considered before RV function deteriorates." }, "21459725": { "QUESTION": "Xanthogranulomatous cholecystitis: a premalignant condition?", "CONTEXTS": [ "Xanthogranulomatous cholecystitis (XGC) is an uncommon variant of chronic cholecystitis, characterized by marked thickening of the gallbladder wall and dense local adhesions. It often mimics a gallbladder carcinoma (GBC), and may coexist with GBC, leading to a diagnostic dilemma. Furthermore, the premalignant nature of this entity is not known. This study was undertaken to assess the p53, PCNA and beta-catenin expression in XGC in comparison to GBC and chronic inflammation.", "Sections from paraffin-embedded blocks of surgically resected specimens of GBC (69 cases), XGC (65), chronic cholecystitis (18) and control gallbladder (10) were stained with the monoclonal antibodies to p53 and PCNA, and a polyclonal antibody to beta-catenin. p53 expression was scored as the percentage of nuclei stained. PCNA expression was scored as the product of the percentage of nuclei stained and the intensity of the staining (1-3). A cut-off value of 80 for this score was taken as a positive result. Beta-catenin expression was scored as type of expression-membranous, cytoplasmic or nuclear staining.", "p53 mutation was positive in 52% of GBC cases and 3% of XGC, but was not expressed in chronic cholecystitis and control gallbladders. p53 expression was lower in XGC than in GBC (P<0.0001). PCNA expression was seen in 65% of GBC cases and 11% of XGC, but not in chronic cholecystitis and control gallbladders. PCNA expression was higher in GBC than XGC (P=0.0001), but there was no significant difference between the XGC, chronic cholecystitis and control gallbladder groups. Beta-catenin expression was positive in the GBC, XGC, chronic cholecystitis and control gallbladder groups. But the expression pattern in XGC, chronic cholecystitis and control gallbladders was homogenously membranous, whereas in GBC the membranous expression pattern was altered to cytoplasmic and nuclear." ], "LABELS": [ "BACKGROUND", "METHODS", "RESULTS" ], "MESHES": [ "Cholecystitis", "Gallbladder Neoplasms", "Granuloma", "Humans", "Immunohistochemistry", "Precancerous Conditions", "Proliferating Cell Nuclear Antigen", "Tumor Suppressor Protein p53", "Xanthomatosis", "beta Catenin" ], "YEAR": "2011", "reasoning_required_pred": "no", "reasoning_free_pred": "no", "final_decision": "no", "LONG_ANSWER": "The expression of p53, PCNA and beta-catenin in XGC was significantly different from GBC and similar to chronic cholecystitis, thus indicating the inflammatory nature of XGC and may not support a premalignant nature of the lesion." }, "21172844": { "QUESTION": "Does TDP-43 type confer a distinct pattern of atrophy in frontotemporal lobar degeneration?", "CONTEXTS": [ "To determine whether TDP-43 type is associated with distinct patterns of brain atrophy on MRI in subjects with pathologically confirmed frontotemporal lobar degeneration (FTLD).", "In this case-control study, we identified all subjects with a pathologic diagnosis of FTLD with TDP-43 immunoreactive inclusions (FTLD-TDP) and at least one volumetric head MRI scan (n = 42). In each case we applied published criteria for subclassification of FTLD-TDP into FTLD-TDP types 1-3. Voxel-based morphometry was used to compare subjects with each of the different FTLD-TDP types to age- and gender-matched normal controls (n = 30). We also assessed different pathologic and genetic variants within, and across, the different types.", "Twenty-two subjects were classified as FTLD-TDP type 1, 9 as type 2, and 11 as type 3. We identified different patterns of atrophy across the types with type 1 showing frontotemporal and parietal atrophy, type 2 predominantly anterior temporal lobe atrophy, and type 3 predominantly posterior frontal atrophy. Within the FTLD-TDP type 1 group, those with a progranulin mutation had significantly more lateral temporal lobe atrophy than those without. All type 2 subjects were diagnosed with semantic dementia. Subjects with a pathologic diagnosis of FTLD with motor neuron degeneration had a similar pattern of atrophy, regardless of whether they were type 1 or type 3." ], "LABELS": [ "OBJECTIVE", "METHODS", "RESULTS" ], "MESHES": [ "Adult", "Aged", "Aged, 80 and over", "Atrophy", "Brain", "Case-Control Studies", "DNA-Binding Proteins", "Female", "Frontal Lobe", "Frontotemporal Lobar Degeneration", "Humans", "Magnetic Resonance Imaging", "Male", "Middle Aged", "Parietal Lobe", "Risk Factors", "Temporal Lobe" ], "YEAR": "2010", "reasoning_required_pred": "yes", "reasoning_free_pred": "no", "final_decision": "yes", "LONG_ANSWER": "Although there are different patterns of atrophy across the different FTLD-TDP types, it appears that genetic and pathologic factors may also affect the patterns of atrophy." }, "24946973": { "QUESTION": "Is oncoplastic surgery a contraindication for accelerated partial breast radiation using the interstitial multicatheter brachytherapy method?", "CONTEXTS": [ "To evaluate accelerated partial breast irradiation (APBI) in patients after oncoplastic surgery for early breast cancer.", "A retrospective analysis of 136 breasts of 134 patients, who received breast-conserving oncoplastic surgery for low-risk breast cancer between 2002 and 2010 in the Universities of Vienna and Luebeck followed by adjuvant APBI applying total doses of pulse dose rate of 50.4\u00a0Gy or high-dose rate (HDR) of 32\u00a0Gy over 4\u00a0days. Target volume definition was performed by the use of surgical-free margin data, related to intraoperatively fixed clip positions, pre- and postoperative imaging, and palpation.", "At the time of data acquisition, 131 of 134 patients were alive. The median follow-up time was 39\u00a0months (range, 4-106 months). After high-dose rate treatment, 3 of 89 patients showed systemic progress after a mean follow-up of 47\u00a0months (range, 19-75 months) and 2 patients had a different quadrant in-breast tumor after 27 and 35\u00a0months. One patient died 7\u00a0months after treatment of unknown causes. After pulse dose rate treatment, 1 of 45 patients had a local recurrence after 42\u00a0months and 1 patient died because of another cause after 13\u00a0months. We observed mild fibrosis in 27 breasts, telangiectasia in 6, hyperpigmentation in 14 cases, and keloid formation in\u00a01." ], "LABELS": [ "PURPOSE", "METHODS AND MATERIALS", "RESULTS" ], "MESHES": [ "Adult", "Aged", "Aged, 80 and over", "Brachytherapy", "Breast", "Breast Neoplasms", "Carcinoma, Ductal, Breast", "Carcinoma, Intraductal, Noninfiltrating", "Carcinoma, Lobular", "Catheters", "Feasibility Studies", "Female", "Follow-Up Studies", "Humans", "Kaplan-Meier Estimate", "Mastectomy, Segmental", "Middle Aged", "Neoplasm Recurrence, Local", "Radiotherapy Dosage", "Radiotherapy Planning, Computer-Assisted", "Radiotherapy, Adjuvant", "Retrospective Studies", "Treatment Outcome" ], "YEAR": null, "reasoning_required_pred": "maybe", "reasoning_free_pred": "maybe", "final_decision": "maybe", "LONG_ANSWER": "These preliminary results suggest the feasibility of multicatheter APBI after oncoplastic breast-conserving surgery in selected low-risk breast cancer patients; however, special attention to target volume definition is needed. Further prospective investigations with long follow-up are needed to define the real value of the procedure." }, "17335331": { "QUESTION": "Diagnostic characteristics of child bipolar I disorder: does the \"Treatment of Early Age Mania (team)\" sample generalize?", "CONTEXTS": [ "To examine the representativeness of a randomized controlled trial (RCT) sample versus one obtained by consecutive new case ascertainment, for subjects with child bipolar I disorder.", "Subjects (N = 247) were outpatients who participated in either the National Institute of Mental Health-funded Phenomenology and Course of Pediatric Bipolar Disorders study or the Treatment of Early Age Mania (TEAM) study. Both studies required that subjects have current DSM-IV bipolar I disorder (manic or mixed phase) and a Children's Global Assessment Scale (CGAS) score370 consecutive days in our NICU in 2012. Overall, our CLABSI rates decreased from 4.1 per 1000 line days in 2009 (13 infections; 3163 line days) to 0.94 in 2013 (2 infections; 2115 line days), which represents a 77% reduction over a 5-year period. In the first quarter of 2013, there was a brief increase in CLABSI rate to 3.3 per 1000 line days; after a series of interventions, the CLABSI rate was maintained at zero for>600 days. Ongoing training, surveillance, and vigilance with catheter insertion and maintenance practices and improved documentation were identified as key drivers for success." ], "LABELS": [ "BACKGROUND AND OBJECTIVE", "METHODS", "RESULTS" ], "MESHES": [ "Bacteremia", "Catheter-Related Infections", "Catheterization, Central Venous", "Guideline Adherence", "Humans", "Infant, Newborn", "Time Factors" ], "YEAR": "2015", "reasoning_required_pred": "yes", "reasoning_free_pred": "maybe", "final_decision": "yes", "LONG_ANSWER": "High-quality training, strict compliance with evidence-based guidelines, and thorough documentation is associated with significant reductions in CLABSIs. Mindful organizing may lead to a better understanding of what goes into a unit's ability to handle peak demands and sustain extraordinary performance in the long-term." }, "15670262": { "QUESTION": "Is gastric electrical stimulation superior to standard pharmacologic therapy in improving GI symptoms, healthcare resources, and long-term health care benefits?", "CONTEXTS": [ "Severe upper gastrointestinal (GI) motor disorders, including gastroparesis (GP), can consume significant health care resources. Many patients are refractory to traditional drug therapy.", "To compare symptoms, healthcare resource utilization and costs in two groups of patients with the symptoms of GP: those treated via gastric electrical stimulation (GES) and those treated with traditional pharmacological agents in an intensive outpatient program (MED).", "A long-term comparison of patients with devices (n = 9) vs intensive medical therapy (n = 9).", "A total of 18 eligible patients with the symptoms of GP reported for 1-year baseline and long-term treatment for 3 years.", "Patients with the symptoms of GP were treated by a GES or intensive medical therapy (MED).", "GP Symptoms, healthcare resource utilization using investigator-derived independent outcome measure score (IDIOMS) and total hospital (inpatient and outpatient) billing costs.", "Gastrointestinal symptoms were significantly different from baseline (F = 3.03, P<0.017) with GP patients treated via GES showing more sustained improvement over 36 months than those treated via MED. Healthcare resource usage, measured via the IDIOMS, significantly improved at 12, 24 and 36 month follow-up for GES patients (F = 10.49, P<0.001), compared with patients receiving medical therapy, who demonstrated further deterioration. GP patients treated via GES also proved superior to medical therapy at 24 and 36 months with regard to decreased costs (F = 4.85, P<0.001). Within group comparisons indicated significantly reduced hospital days for both patient groups; however, no statistical differences were noted between groups in terms of hospital days. Three of nine patients in the MED group died primarily from i.v. access related problems; none of the GES patients died." ], "LABELS": [ "CONTEXT", "OBJECTIVE", "DESIGN", "SETTING AND PATIENTS", "INTERVENTIONS", "MAIN OUTCOME MEASURES", "RESULTS" ], "MESHES": [ "Adult", "Cost-Benefit Analysis", "Electric Stimulation Therapy", "Female", "Gastrointestinal Diseases", "Gastroparesis", "Health Resources", "Humans", "Long-Term Care", "Male", "Nausea", "Prospective Studies", "Quality of Life", "Treatment Outcome", "Vomiting" ], "YEAR": "2005", "reasoning_required_pred": "yes", "reasoning_free_pred": "yes", "final_decision": "yes", "LONG_ANSWER": "We conclude that GES is more effective in improving long-term GI symptoms and costs, and decreasing use of healthcare resources than intensive medical therapy, in this sample of patients with the symptoms of GP followed for 3 years. Certain patients with GP form a high-risk group in terms of costs, quality of life, morbidity and mortality." }, "22900881": { "QUESTION": "Should pulp chamber pulpotomy be seen as a permanent treatment?", "CONTEXTS": [ "Seventeen patients, aged 7-54\u00a0years (mean of 37.2\u00a0year), were treated by pulpotomy and filling with ProRoot MTA(\u00ae) in premolar or molar teeth with vital pulps and without clinical evidence of irreversible pulpitis. The patients were then followed up for 12 to 24\u00a0months and the teeth then assessed by clinical and radiographic examination. Statistical analysis was performed with Kaplan-Meier survival probability statistics to estimate the survival of the treated teeth.", "At 24\u00a0months, the survival rate without any complementary treatment was estimated to be 82%. Two of the 17 treated teeth required root canal treatment for pain control and one for prosthetic reasons." ], "LABELS": [ "METHODOLOGY", "RESULTS" ], "MESHES": [ "Adolescent", "Adult", "Aluminum Compounds", "Bicuspid", "Calcium Compounds", "Child", "Composite Resins", "Crowns", "Dental Caries", "Dental Materials", "Dental Pulp", "Dental Pulp Cavity", "Dental Pulp Test", "Drug Combinations", "Female", "Follow-Up Studies", "Humans", "Inlays", "Male", "Middle Aged", "Molar", "Oxides", "Pain Measurement", "Post and Core Technique", "Pulp Capping and Pulpectomy Agents", "Pulpotomy", "Radiography", "Silicates", "Survival Rate", "Young Adult" ], "YEAR": "2013", "reasoning_required_pred": "yes", "reasoning_free_pred": "no", "final_decision": "no", "LONG_ANSWER": "Under the conditions of this study, pulpotomy offered a viable alternative to root canal treatment for teeth with vital pulps in the short term. However, there is insufficient clinical evidence to consider this technique for the treatment of every permanent tooth. Nevertheless, it should be considered as a potential alternative approach to be further developed for future applications." }, "11413427": { "QUESTION": "Does transverse apex coincide with coronal apex levels (regional or global) in adolescent idiopathic scoliosis?", "CONTEXTS": [ "Cross-sectional.", "To identify the regional and global apexes of curves in adolescent idiopathic scoliosis and to compare the levels of those with the most rotated vertebral levels on computed tomography scans.", "The terminology regarding the terms and definitions had been arbitrary until being refined and standardized by the Scoliosis Research Society Working Group on Three-Dimensional Terminology of Spinal Deformity. Apical vertebra or disc is defined as the most laterally deviated vertebra or disc in a scoliosis curve, but the most rotated vertebra (or disc) has not been included in this terminology. One study suggested that the most rotated vertebral level was always located at the apex.", "Thirty-three structural curves of 25 consecutive patients scheduled for surgery for thoracic or thoracolumbar scoliosis were analyzed with standing anteroposterior radiographs and computed tomography scans covering the curve apexes and pelvis. Thoracic and lumbar curves were evaluated separately for all Type II curves. Vertebral rotations were normalized by the rotation of the pelvis. The most rotated vertebral (or disc) levels (transverse apex) were compared with the regional and global apex levels (vertebra or disc) (coronal apexes) of the corresponding curves separately.", "Regional and global apexes were at the same level in 18 (54.5%) curves, and within half a level in another 15 (45.4%), and the regional apex was one level higher in two curves (95% confidence levels: -0.82, +0.88). Comparison of the most rotated levels with regional and global apex levels revealed a higher variability, extending up to two levels for the global apex (95% confidence levels: -1.19, +1.54 levels for the global and -1.0, +1.41 levels for the regional apexes)." ], "LABELS": [ "STUDY DESIGN", "OBJECTIVES", "SUMMARY OF BACKGROUND DATA", "METHODS", "RESULTS" ], "MESHES": [ "Adolescent", "Humans", "Lumbar Vertebrae", "Rotation", "Scoliosis", "Thoracic Vertebrae", "Tomography, X-Ray Computed" ], "YEAR": "2001", "reasoning_required_pred": "no", "reasoning_free_pred": "no", "final_decision": "no", "LONG_ANSWER": "This study demonstrated that the regional or global apex of a given curve is the most rotated level in only a minority of the curves. The most rotated level may be as far as two levels from the global apex and one level from the regional apex." }, "9602458": { "QUESTION": "Does the Child Health Computing System adequately identify children with cerebral palsy?", "CONTEXTS": [ "This paper assesses the usefulness of the Child Health Computing System as a source of information about children with cerebral palsy.", "A comparative survey of information held on the Child Health Computing System (CHCS) and the Northern Ireland Cerebral Palsy Register (NICPR) in one Health and Social Services Board in Northern Ireland was carried out. The sample comprised children with cerebral palsy aged 5-9 years.", "Of the 135 cases recorded on the NICPR, 47 per cent were not found on the CHCS; the majority of these children had no computer record of any medical diagnosis. Of the 82 cases recorded on the CHCS, 10 (12 per cent) were not found on the NICPR; five of these cases (6 per cent) were found on follow-up not to have CP." ], "LABELS": [ "BACKGROUND", "METHODS", "RESULTS" ], "MESHES": [ "Cerebral Palsy", "Child", "Child Welfare", "Child, Preschool", "Data Collection", "Databases, Factual", "False Positive Reactions", "Health Status Indicators", "Humans", "Northern Ireland", "Prevalence", "Registries" ], "YEAR": "1998", "reasoning_required_pred": "no", "reasoning_free_pred": "no", "final_decision": "no", "LONG_ANSWER": "Unless improvements are made in case ascertainment, case validation and recording activities, the evidence suggests that the CHCS will not be able to provide the same quality of information for needs assessment and surveillance of very low birthweight infants in relation to cerebral palsy as is provided by a specialist case register." }, "24434052": { "QUESTION": "Are we seeing the effects of public awareness campaigns?", "CONTEXTS": [ "The last 20 years has seen a marked improvement in skin cancer awareness campaigns. We sought to establish whether this has affected the presenting Breslow thickness of malignant melanoma in the South West.", "This is a retrospective study looking at the first presentation of melanomas from 2003 to 2011. Data was accessed using the local online melanoma database.", "A total of 2001 new melanomas presented from 2003 to 2012 (Male:Female = 1:1.062). The average yearly number of melanomas was 200.1 (range = 138-312). The mean age was 62.5 years (range 12-99). Data was analysed using a Chi\u00b2 test. For 0-1 mm melanomas, there is a significant difference in the observed versus expected values over the 10 years (p = 0.0018). There is an increasing proportion of 0-1 mm (thin) melanomas presenting year on year, with a positive linear trend. This is very statistically significant (p<0.0001). The 1-2 mm melanomas are decreasing in proportion with a negative linear trend (p = 0.0013). The 2-4 mm are also decreasing in proportion (p = 0.0253). There is no significant change in the thick>4 mm melanomas (p = 0.1456)." ], "LABELS": [ "INTRODUCTION", "METHOD", "RESULTS" ], "MESHES": [ "Adolescent", "Adult", "Aged", "Aged, 80 and over", "Child", "England", "Female", "Health Knowledge, Attitudes, Practice", "Health Promotion", "Humans", "Male", "Melanoma", "Middle Aged", "Patient Acceptance of Health Care", "Retrospective Studies", "Skin Neoplasms", "Young Adult" ], "YEAR": "2014", "reasoning_required_pred": "yes", "reasoning_free_pred": "maybe", "final_decision": "maybe", "LONG_ANSWER": "The proportion of thin 0-1 mm melanomas presenting in South West England has significantly increased from 2003 to 2012. There is no significant change in the thick>4 mm melanomas. This may be a result of increased public awareness due to effective public health campaigns which has significant prognostic and financial implications." }, "17598882": { "QUESTION": "Is breast cancer prognosis inherited?", "CONTEXTS": [ "A genetic component is well established in the etiology of breast cancer. It is not well known, however, whether genetic traits also influence prognostic features of the malignant phenotype.", "We carried out a population-based cohort study in Sweden based on the nationwide Multi-Generation Register. Among all women with breast cancer diagnosed from 1961 to 2001, 2,787 mother-daughter pairs and 831 sister pairs with breast cancer were identified; we achieved complete follow-up and classified 5-year breast cancer-specific prognosis among proband (mother or oldest sister) into tertiles as poor, intermediary, or good. We used Kaplan-Meier estimates of survival proportions and Cox models to calculate relative risks of dying from breast cancer within 5 years depending on the proband's outcome.", "The 5-year survival proportion among daughters whose mothers died within 5 years was 87% compared to 91% if the mother was alive (p = 0.03). Among sisters, the corresponding proportions were 70% and 88%, respectively (p = 0.001). After adjustment for potential confounders, daughters and sisters of a proband with poor prognosis had a 60% higher 5-year breast cancer mortality compared to those of a proband with good prognosis (hazard ratio [HR], 1.6; 95% confidence interval [CI], 1.2 to 2.2; p for trend 0.002). This association was slightly stronger among sisters (HR, 1.8; 95% CI, 1.0 to 3.4) than among daughters (HR, 1.6; 95% CI, 1.1 to 2.3)." ], "LABELS": [ "INTRODUCTION", "METHODS", "RESULTS" ], "MESHES": [ "Adult", "Aged", "Breast Neoplasms", "Cause of Death", "Cohort Studies", "Female", "Follow-Up Studies", "Humans", "Middle Aged", "Nuclear Family", "Prognosis", "Registries", "Survival Analysis", "Survivors", "Sweden", "Time Factors" ], "YEAR": "2007", "reasoning_required_pred": "yes", "reasoning_free_pred": "yes", "final_decision": "yes", "LONG_ANSWER": "Breast cancer prognosis of a woman predicts the survival in her first-degree relatives with breast cancer. Our novel findings suggest that breast cancer prognosis might be inherited." }, "21190419": { "QUESTION": "Does nuchal translucency thickness in the first trimester predict GDM onset during pregnancy?", "CONTEXTS": [ "This study was planned to evaluate whether increased nuchal translucency (NT) thickness in the first trimester of gestation can be related to onset of gestational diabetes mellitus (GDM) during pregnancy.", "From January 2006 to August 2008, a group of 678 singleton pregnancies who had developed GDM has been selected as a study group among a total of 3966 pregnant women who had undergone first trimester screening for aneuploidies at 11-14 weeks of gestation. A group of 420 single pregnant women with physiological pregnancy were enrolled as control group. Both fetal structural and karyotype's anomalies were excluded in the two groups. NT was mesured by a Fetal Medicine Foundation certificated operator; GDM was diagnosed at 24-28 weeks of gestation following Carpenter and Coustan criteria. In the analyses of continuos variables, study and control group were compared by Student's t-test and Anova test.", "There was no significative difference (p = 0.585) between NT values in the study (mean = 1.56) and control group (mean = 1.54)." ], "LABELS": [ "BACKGROUND AND AIMS", "METHODS", "RESULTS" ], "MESHES": [ "Adult", "Diabetes, Gestational", "Early Diagnosis", "Female", "Glucose Tolerance Test", "Hospitals, University", "Humans", "Nuchal Translucency Measurement", "Predictive Value of Tests", "Pregnancy", "Pregnancy Trimester, First", "Pregnancy Trimester, Second", "Retrospective Studies" ], "YEAR": "2011", "reasoning_required_pred": "no", "reasoning_free_pred": "no", "final_decision": "no", "LONG_ANSWER": "NT thickness does not show a significative increase in those women who subsequently develop GDM. Therefore, NT assessment does not prove to be an useful ultrasound parameter for predicting GDM onset during pregnancy." }, "16403186": { "QUESTION": "Are the arginine vasopressin V1a receptor microsatellites related to hypersexuality in children with a prepubertal and early adolescent bipolar disorder phenotype?", "CONTEXTS": [ "To examine family-based transmission of the number of 5' flanking arginine vasopressin V1a receptor (AVPR1A) microsatellites, which include [(GATA)(14)] and complex [(CT)(4)-TT-(CT)(8)-(GT)(24)]repeats, in probands with a prepubertal and early adolescent bipolar disorder phenotype (PEA-BP). Preferential transmission of the number of AVPR1A microsatellite repeats to hypersexual and uninhibited people-seeking probands was hypothesized, based on reports from preclinical work in the literature.", "Probands were 83 participants in an ongoing controlled study of PEA-BP. The PEA-BP phenotype was defined by DSM-IV mania with at least one of the cardinal symptoms of mania (elation and/or grandiosity) to avoid diagnosing mania only by symptoms that overlapped with those for attention-deficit hyperactivity disorder (ADHD). Comprehensive assessment of the probands included separate Washington University in St. Louis Kiddie Schedule for Affective Disorders and Schizophrenia (WASH-U-KSADS) interviews of parents about their children and of children about themselves. Hypersexuality and uninhibited people-seeking were assessed from the corresponding WASH-U-KSADS items. Microsatellite genotyping of the AVPR1A repeats was conducted using fluorescently labeled primers and detected by laser-induced fluorescence. Alleles were determined with the assistance of semi-automated allele-calling software. There were 32 complete, biological trios (28 informative families) for the GATA repeat and 34 complete, biological trios (30 informative families) for the complex repeat. Data were analyzed using case-control and family-based association methods.", "Preferential transmission of AVPR1A GATA or complex repeats was not significant for hypersexuality or uninhibited people-seeking, using the transmission disequilibrium test. Similarly, case-control analyses found no significant associations between hypersexuality or uninhibited people-seeking and the number of AVPR1A GATA or complex repeats. For p<0.05, there was about 80% power to detect odds ratios of 5.0 and 4.0 (in the family-based analyses) and 3.5 and 2.6 (in the case-control analyses), for allele frequencies of 0.1 and 0.5, respectively." ], "LABELS": [ "OBJECTIVE", "METHODS", "RESULTS" ], "MESHES": [ "Adolescent", "Alleles", "Bipolar Disorder", "Case-Control Studies", "Child", "Female", "Humans", "Linkage Disequilibrium", "Male", "Microsatellite Repeats", "Phenotype", "Psychiatric Status Rating Scales", "Receptors, Vasopressin", "Sexual Dysfunctions, Psychological" ], "YEAR": "2005", "reasoning_required_pred": "no", "reasoning_free_pred": "no", "final_decision": "no", "LONG_ANSWER": "Preferential transmission of AVPR1A to hypersexual or uninhibited people-seeking probands was not supported." }, "11888773": { "QUESTION": "Stage I non-small cell lung carcinoma: really an early stage?", "CONTEXTS": [ "We review our results on surgical treatment of patients with stage I non-small cell lung carcinoma and we attempted to clarify the prognostic significance of some surgical--pathologic variables.", "From 1993 to 1999, 667 patients received curative lung resection and complete hilar and mediastinal lymphadenectomy for non-small cell lung cancer. Of these, there were 436 Stage I disease (65%), of whom 144 T1N0 and 292 T2N0. No patients had pre- or postoperative radio- or chemotherapy. Prognostic significance of the following independent variables was tested using univariate (log-rank) and multivariate (Cox proportional-hazards) analysis: type of resection (sublobar vs lobectomy vs pneumonectomy), histology (squamous cell vs adenocarcinoma), tumour size (3cm), histologic vascular invasion, visceral pleura involvement, positive bronchial resection margin, general T status.", "Overall 5-year survival was 63%. In both univariate and multivariate survival analysis, significant prognostic factors were histology (adenocarcinoma 65% vs squamous cell carcinoma 51%), tumour size (3cm 46%), and the presence of negative resection margin. Five-year survival by general T status was 66% in T1N0 vs 55% in T2N0 disease (P=0.19)." ], "LABELS": [ "OBJECTIVE", "METHODS", "RESULTS" ], "MESHES": [ "Carcinoma, Non-Small-Cell Lung", "Female", "Humans", "Lung", "Lung Neoplasms", "Male", "Middle Aged", "Neoplasm Recurrence, Local", "Neoplasm Staging", "Prognosis", "Proportional Hazards Models", "Retrospective Studies", "Survival Rate" ], "YEAR": "2002", "reasoning_required_pred": "yes", "reasoning_free_pred": "no", "final_decision": "no", "LONG_ANSWER": "Despite advances in early diagnosis and surgical technique, 5-year survival of stage I non-small cell lung carcinoma remains low as compared to survival of other solid organ neoplasm. Tumour size52 yrs of age) in a cohort of severely injured trauma patients for which significant variation in postinjury care is minimized.", "Secondary data analysis of an ongoing prospective multicenter cohort study.", "Academic, level I trauma and intensive care unit centers.", "Blunt-injured adults with hemorrhagic shock.", "None.", "Separate Cox proportional hazard regression models were formulated based on all patients to evaluate the effects of gender on mortality, multiple organ failure, and nosocomial infection, after controlling for all important confounders. These models were then used to characterize the effect of gender in young and old age groups. Overall mortality, multiple organ failure, and nosocomial infection rates for the entire cohort (n = 1,036) were 20%, 40%, and 45%, respectively. Mean Injury Severity Score was 32 +/- 14 (mean +/- SD). Men (n = 680) and women (n = 356) were clinically similar except that men required higher crystalloid volumes, more often had a history of alcoholism and liver disease, and had greater ventilatory and intensive care unit requirements. Female gender was independently associated with a 43% and 23% lower risk of multiple organ failure and nosocomial infection, respectively. Gender remained an independent risk factor in young and old subgroup analysis, with the protection afforded by female gender remaining unchanged." ], "LABELS": [ "OBJECTIVE", "DESIGN", "SETTING", "PATIENTS", "INTERVENTIONS", "MEASUREMENTS AND MAIN RESULTS" ], "MESHES": [ "Abbreviated Injury Scale", "Adult", "Age Factors", "Cause of Death", "Cohort Studies", "Cross Infection", "Estrogens", "Female", "Glasgow Coma Scale", "Hospital Mortality", "Humans", "Injury Severity Score", "Male", "Menopause", "Middle Aged", "Multiple Organ Failure", "Multiple Trauma", "Proportional Hazards Models", "Prospective Studies", "Regression Analysis", "Sex Factors", "Shock, Hemorrhagic", "Survival Analysis", "Wounds, Nonpenetrating" ], "YEAR": "2008", "reasoning_required_pred": "yes", "reasoning_free_pred": "no", "final_decision": "no", "LONG_ANSWER": "The independent protective effect of female gender on multiple organ failure and nosocomial infection rates remains significant in both premenopausal and postmenopausal women when compared with similarly aged men. This is contrary to previous experimental studies and the known physiologic sex hormone changes that occur after menopause in women. These results suggest that factors other than sex hormones may be responsible for gender-based differences after injury." }, "14551704": { "QUESTION": "Can communication with terminally ill patients be taught?", "CONTEXTS": [ "Communication with terminally ill patients is a main responsibility of physicians. However, many physicians feel insufficiently prepared for this task. Models of courses resulting in improvements of communicative skills of participants have been published mainly in the Anglo-American literature. This study describes the realization of a 2-day course model based on the experiences of the first three courses of this kind in Rhineland-Palatinate, and analyzes changes of participants' communication behavior.", "After each seminary, an evaluation form concerning participants' satisfaction with the course was filled in. Furthermore, all course participants received a questionnaire at the beginning and at the end of the course, as well as 3 months afterwards. The participants were asked to assess their own sense of security in seven different communication settings on a visual analog scale, and to specify perceived changes in their communication behavior 3 months after the course.", "The first three courses were attended by 31 participants. Course evaluation revealed high satisfaction scores with methods as well as with clarity and relevance of the contents. Self-assessment of participants showed a growing sense of security in different communication settings. Important increases could be demonstrated for communicating a diagnosis of cancer with good or less good prognosis, recurrence of cancer or a far progressive cancer disease without curative approach. 3 months after the course, participants described multiple changes indicating increased sensibility and professionalism in communication behavior." ], "LABELS": [ "BACKGROUND AND PURPOSE", "METHODS", "RESULTS" ], "MESHES": [ "Communication", "Curriculum", "Humans", "Medical Oncology", "Physician-Patient Relations", "Surveys and Questionnaires", "Terminally Ill" ], "YEAR": "2003", "reasoning_required_pred": "yes", "reasoning_free_pred": "yes", "final_decision": "yes", "LONG_ANSWER": "The realized communication skills courses resulted in relevant changes in communication behaviour and self-confidence of participants. Communication with terminally ill patients can be taught." }, "23234860": { "QUESTION": "Delays in time to primary treatment after a diagnosis of breast cancer: does it impact survival?", "CONTEXTS": [ "Treatment delays in breast cancer are generally thought to affect prognosis but the impact on survival remains unclear. Indicators for breast cancer care include time to primary treatment. The purpose of this study was to evaluate whether time to primary treatment (TPT) in breast cancer impacts survival.", "A total of 648 breast cancer patients treated in the University Malaya Medical Center (UMMC), Malaysia between 2004 and 2005 were included in the study. TPT was calculated from the date of pathological diagnosis to the date of primary treatment. Mortality data was obtained from the National Registry of Births and Deaths. Last date of follow-up was November 2010.", "Median TPT was 18 days. Majority 508 (69.1%) of the patients received treatment within 30 days after diagnosis. The majority was surgically treated. Ethnicity (p=0.002) and stage at presentation (p=0.007) were significantly associated with delayed TPT. Malay ethnicity had delayed TPT compared to the Chinese; Hazard Ratio (HR) 1.9 (Confidence Interval (CI) 1.237, 2.987). Delayed TPT did not affect overall survival on univariate and multivariate analyses." ], "LABELS": [ "OBJECTIVE", "METHOD", "RESULTS" ], "MESHES": [ "Adult", "Aged", "Breast Neoplasms", "Female", "Humans", "Malaysia", "Middle Aged", "Neoplasm Staging", "Retrospective Studies", "Survival Analysis", "Time Factors" ], "YEAR": "2013", "reasoning_required_pred": "no", "reasoning_free_pred": "no", "final_decision": "no", "LONG_ANSWER": "Time to primary treatment after a diagnosis of breast cancer had no impact on overall survival. Further studies on care before diagnosis are important in drawing up meaningful quality indicators." }, "22532370": { "QUESTION": "Are increased carotid artery pulsatility and resistance indexes early signs of vascular abnormalities in young obese males?", "CONTEXTS": [ "To provide insight into the factors by which obesity in itself may directly lead to early arterial damage, we aimed to determine early sonographic markers of obesity-related vascular dysfunction in young obese males.", "Thirty-five young obese males and 23 age-matched healthy male volunteers were recruited into the study. Common carotid artery pulsatility index and resistance index were calculated from blood flow velocities curves obtained by pulsed Doppler ultrasonography.", "The mean pulsatility index, resistance index, body mass index, waist circumference, systolic and diastolic blood pressure, homeostasis model assessment for insulin resistance, plasma fasting glucose, insulin, C-peptide, triglycerides, low-density lipoprotein cholesterol, and high-sensitivity C-reactive protein were statistically higher in obese subjects than in healthy controls." ], "LABELS": [ "PURPOSE", "METHODS", "RESULTS" ], "MESHES": [ "Adolescent", "Adult", "Blood Flow Velocity", "Blood Glucose", "Blood Pressure", "Body Mass Index", "C-Peptide", "C-Reactive Protein", "Carotid Arteries", "Case-Control Studies", "Humans", "Insulin", "Insulin Resistance", "Lipoproteins, LDL", "Male", "Obesity", "Pulsatile Flow", "Statistics, Nonparametric", "Triglycerides", "Ultrasonography, Doppler, Pulsed", "Vascular Resistance", "Waist Circumference" ], "YEAR": null, "reasoning_required_pred": "yes", "reasoning_free_pred": "yes", "final_decision": "yes", "LONG_ANSWER": "Our results suggest that depressed vessel compliance and increased vascular resistance are features of young, obese, normotensive subjects independently of and in addition to cardiovascular risk factors. As changes in arterial wall properties may be incipient in young obese subjects, future studies will be required to demonstrate whether early intervention such as diet and exercise in this population can improve vascular functions." }, "22656647": { "QUESTION": "Are acceptance rates of a national preventive home visit programme for older people socially imbalanced?", "CONTEXTS": [ "Preventive home visits are offered to community dwelling older people in Denmark aimed at maintaining their functional ability for as long as possible, but only two thirds of older people accept the offer from the municipalities. The purpose of this study is to investigate 1) whether socioeconomic status was associated with acceptance of preventive home visits among older people and 2) whether municipality invitational procedures for the preventive home visits modified the association.", "The study population included 1,023 community dwelling 80-year-old individuals from the Danish intervention study on preventive home visits. Information on preventive home visit acceptance rates was obtained from questionnaires. Socioeconomic status was measured by financial assets obtained from national registry data, and invitational procedures were identified through the municipalities. Logistic regression analyses were used, adjusted by gender.", "Older persons with high financial assets accepted preventive home visits more frequently than persons with low assets (adjusted OR = 1.5 (CI95%: 1.1-2.0)). However, the association was attenuated when adjusted by the invitational procedures. The odds ratio for accepting preventive home visits was larger among persons with low financial assets invited by a letter with a proposed date than among persons with high financial assets invited by other procedures, though these estimates had wide confidence intervals." ], "LABELS": [ "BACKGROUND", "METHODS", "RESULTS" ], "MESHES": [ "Aged, 80 and over", "Cross-Sectional Studies", "Denmark", "Female", "Financing, Personal", "Geriatric Assessment", "Health Services for the Aged", "Healthcare Disparities", "Home Care Services", "House Calls", "Humans", "Logistic Models", "Male", "Patient Acceptance of Health Care", "Physicians, Family", "Prevalence", "Preventive Health Services", "Program Evaluation", "Residence Characteristics", "Sex Distribution", "Social Class", "Surveys and Questionnaires" ], "YEAR": "2012", "reasoning_required_pred": "yes", "reasoning_free_pred": "yes", "final_decision": "yes", "LONG_ANSWER": "High socioeconomic status was associated with a higher acceptance rate of preventive home visits, but the association was attenuated by invitational procedures. The results indicate that the social inequality in acceptance of publicly offered preventive services might decrease if municipalities adopt more proactive invitational procedures." }, "22683044": { "QUESTION": "Does open access publishing increase the impact of scientific articles?", "CONTEXTS": [ "Some studies suggest that open access articles are more often cited than non-open access articles. However, the relationship between open access and citations count in a discipline such as intensive care medicine has not been studied to date. The present article analyzes the effect of open access publishing of scientific articles in intensive care medicine journals in terms of citations count.", "We evaluated a total of 161 articles (76% being non-open access articles) published in Intensive Care Medicine in the year 2008. Citation data were compared between the two groups up until April 30, 2011. Potentially confounding variables for citation counts were adjusted for in a linear multiple regression model.", "The median number (interquartile range) of citations of non-open access articles was 8 (4-12) versus 9 (6-18) in the case of open access articles (p=0.084). In the highest citation range (>8), the citation count was 13 (10-16) and 18 (13-21) (p=0.008), respectively. The mean follow-up was 37.5 \u00b1 3 months in both groups. In the 30-35 months after publication, the average number (mean \u00b1 standard deviation) of citations per article per month of non-open access articles was 0.28 \u00b1 0.6 versus 0.38 \u00b1 0.7 in the case of open access articles (p=0.043). Independent factors for citation advantage were the Hirsch index of the first signing author (\u03b2=0.207; p=0.015) and open access status (\u03b2=3.618; p=0.006)." ], "LABELS": [ "OBJECTIVE", "METHODS", "RESULTS" ], "MESHES": [ "Access to Information", "Critical Care", "Journal Impact Factor", "Periodicals as Topic", "Publishing" ], "YEAR": "2013", "reasoning_required_pred": "yes", "reasoning_free_pred": "yes", "final_decision": "yes", "LONG_ANSWER": "Open access publishing and the Hirsch index of the first signing author increase the impact of scientific articles. The open access advantage is greater for the more highly cited articles, and appears in the 30-35 months after publication." }, "11566686": { "QUESTION": "In vivo visualization of pyloric mucosal hypertrophy in infants with hypertrophic pyloric stenosis: is there an etiologic role?", "CONTEXTS": [ "Infantile hypertrophic pyloric stenosis (IHPS) is a common condition which presents in infants at 2-12 weeks of postnatal life, and whose cause remains obscure. Multiple associated abnormalities have been recognized within the external hypertrophied pyloric muscle layer, but the internal component of the pyloric mucosa has received scant attention in the literature to date. Our purpose in this study was to show that pyloric mucosal redundancy is a constant finding in infants with IHPS, to discuss its possible cause, and to explore the hypothesis of a relationship between pyloric mucosal redundancy and the development of IHPS.", "We identified 102 consecutive infants with surgically confirmed IHPS and determined the thickness of the pyloric mucosa compared with the thickness of the surrounding hypertrophied muscle. Fifty-one infants who did not have pyloric stenosis served as controls.", "Mean mucosal thickness in patients with IHPS approximated mean muscle thickness, with a ratio of 0.89. In infants with IHPS, the pyloric mucosa constitutes approximately one third of the cross-sectional diameter of the pyloric mass and fills and obstructs the pyloric canal." ], "LABELS": [ "OBJECTIVE", "MATERIALS AND METHODS", "RESULTS" ], "MESHES": [ "Female", "Gastric Mucosa", "Humans", "Hypertrophy", "Infant", "Infant, Newborn", "Male", "Pyloric Stenosis", "Pylorus" ], "YEAR": "2001", "reasoning_required_pred": "yes", "reasoning_free_pred": "yes", "final_decision": "yes", "LONG_ANSWER": "Mucosal redundancy is a constant associated finding in IHPS. Although the origin of the redundancy and a cause-and-effect relationship are difficult to establish, our findings support the hypothesis that hypergastrinemia may be implicated in the pathogenesis of IHPS, and suggest that mucosal thickening could be implicated as one of the initiating factors in its development." }, "15502995": { "QUESTION": "Does the early adopter of drugs exist?", "CONTEXTS": [ "To analyse associations between indicators for adoption of new drugs and to test the hypothesis that physicians' early adoption of new drugs is a personal trait independent of drug groups.", "In a population-based cohort study using register data, we analysed the prescribing of new drugs by Danish general practitioners. Angiotensin-II antagonists, triptans, selective cyclo-oxygenase-2 antagonists and esomeprazol were used in the assessment. As indicators of new drug uptake, we used adoption time, cumulative incidence, preference proportion, incidence rate and prescription cost and volume. For each measure, we ranked the general practices. Ranks were pair-wise plotted, and Pearson's correlation coefficient ( r) was calculated. Next, we analysed the correlation between ranks across different drug classes.", "For all indicators, the general practitioners' adoption of one group of drugs was poorly associated with adoption of others ( ror = 20 pack-years) was 3.73 (3.12- 4.45), 1.98 (1.73-2.27) for cough, 1.79 (1.55-2.08) for dyspnea, 1.95 (1.70-2.34) for sputum, and 2.59 (2.26-2.97) for wheeze. Respiratory symptoms were reported by 92% of smokers with AO, 86% smokers with restriction, 76% smokers with normal spirometry, and 73% of nonsmokers. Sensitivity (92% vs 90%), specificity (19% vs 22%), positive (47% vs 40%) and negative (75% vs 80%) predictive values for the presence of one or more symptoms were similar between smokers and all subjects." ], "LABELS": [ "BACKGROUND", "METHODS", "RESULTS" ], "MESHES": [ "Aged", "Female", "Humans", "Logistic Models", "Male", "Middle Aged", "Predictive Value of Tests", "Prevalence", "Pulmonary Disease, Chronic Obstructive", "Radiography, Thoracic", "Respiratory Function Tests", "Risk Factors", "Sensitivity and Specificity", "Smoking", "Spirometry", "Surveys and Questionnaires" ], "YEAR": "2010", "reasoning_required_pred": "yes", "reasoning_free_pred": "no", "final_decision": "no", "LONG_ANSWER": "COPD is underdiagnosed in the United States. Symptoms are frequent in subjects with AO and increase their risk for COPD, but add little beyond age and smoking history to the predictive value of spirometry. In view of the high prevalence of symptoms and their poor predictive value, a simpler and more effective approach would be to screen older smokers." }, "19664156": { "QUESTION": "Search engine as a diagnostic tool in difficult immunological and allergologic cases: is Google useful?", "CONTEXTS": [ "Web search engines are an important tool in communication and diffusion of knowledge. Among these, Google appears to be the most popular one: in August 2008, it accounted for 87% of all web searches in the UK, compared with Yahoo's 3.3%. Google's value as a diagnostic guide in general medicine was recently reported. The aim of this comparative cross-sectional study was to evaluate whether searching Google with disease-related terms was effective in the identification and diagnosis of complex immunological and allergic cases.", "Forty-five case reports were randomly selected by an independent observer from peer-reviewed medical journals. Clinical data were presented separately to three investigators, blinded to the final diagnoses. Investigator A was a Consultant with an expert knowledge in Internal Medicine and Allergy (IM&A) and basic computing skills. Investigator B was a Registrar in IM&A. Investigator C was a Research Nurse. Both Investigators B and C were familiar with computers and search engines. For every clinical case presented, each investigator independently carried out an Internet search using Google to provide a final diagnosis. Their results were then compared with the published diagnoses.", "Correct diagnoses were provided in 30/45 (66%) cases, 39/45 (86%) cases, and in 29/45 (64%) cases by investigator A, B, and C, respectively. All of the three investigators achieved the correct diagnosis in 19 cases (42%), and all of them failed in two cases." ], "LABELS": [ "BACKGROUND", "METHODS", "RESULTS" ], "MESHES": [ "Humans", "Hypersensitivity", "Immune System Diseases", "Information Storage and Retrieval", "Internet" ], "YEAR": "2009", "reasoning_required_pred": "maybe", "reasoning_free_pred": "yes", "final_decision": "yes", "LONG_ANSWER": "This Google-based search was useful to identify an appropriate diagnosis in complex immunological and allergic cases. Computing skills may help to get better results." }, "23356465": { "QUESTION": "Uniformity of evidence-based treatments in practice?", "CONTEXTS": [ "Various factors contribute to the effective implementation of evidence-based treatments (EBTs). In this study, cognitive processing therapy (CPT) was administered in a Veterans Affairs (VA) posttraumatic stress disorder (PTSD) specialty clinic in which training and supervision were provided following VA implementation guidelines. The aim was to (a) estimate the proportion of variability in outcome attributable to therapists and (b) identify characteristics of those therapists who produced better outcomes.", "We used an archival database of veterans (n = 192) who completed 12 sessions of CPT by therapists (n = 25) who were trained by 2 nationally recognized trainers, 1 of whom also provided weekly group supervision. Multilevel modeling was used to estimate therapist effects, with therapists treated as a random factor. The supervisor was asked to retrospectively rate each therapist in terms of perceived effectiveness based on supervision interactions. Using single case study design, the supervisor was interviewed to determine what criteria she used to rate the therapists and emerging themes were coded.", "When initial level of severity on the PTSD Checklist (PCL; McDonald&Calhoun, 2010; Weathers, Litz, Herman, Huska,&Keane, 1993) was taken into account, approximately 12% of the variability in the PCL at the end of treatment was due to therapists. The trainer, blind to the results, identified the following characteristics and actions of effective therapists: effectively addressing patient avoidance, language used in supervision, flexible interpersonal style, and ability to develop a strong therapeutic alliance." ], "LABELS": [ "OBJECTIVE", "METHOD", "RESULTS" ], "MESHES": [ "Adult", "Aged", "Clinical Competence", "Cognitive Therapy", "Evidence-Based Practice", "Female", "Humans", "Interviews as Topic", "Male", "Middle Aged", "Severity of Illness Index", "Stress Disorders, Post-Traumatic", "Treatment Outcome", "United States", "United States Department of Veterans Affairs", "Veterans", "Wisconsin", "Young Adult" ], "YEAR": "2013", "reasoning_required_pred": "no", "reasoning_free_pred": "yes", "final_decision": "yes", "LONG_ANSWER": "This study adds to the growing body of literature documenting the importance of the individual therapist as an important factor in the change process." }, "22972546": { "QUESTION": "Do approved doctors and medical referees in the UK agree when assessing a seafarer's fitness?", "CONTEXTS": [ "The seafaring industry remains a hazardous occupation that requires sophisticated systems of risk and fitness assessment. This study aims to investigate the extent of agreement between Approved Doctors (ADs) and Medical Referees (MRs) when they assess a seafarer's fitness.", "Between 2003 and 2009 a total of 232,878 seafarer medical examinations were carried out by ADs, of which 465 were considered by the MRs because the seafarer appealed against the AD's decision. The extent of agreement between ADs and MRs was studied.", "Two hundred and sixty-eight (58%) cases seen by the ADs were classed as category 4 \"permanently unfit\"; the referees only placed 85 (18%) of them in this category. On the other hand, 252 (54%) cases seen by the MRs were classed as category 2 \"fit with restrictions\", while the ADs had only placed 111 (24%) in this category. The overall agreement between the assessors (AD vs. MR) was poor (Kappa K = 0.18).", "For cardiovascular diseases and for mental ill-health, access to additional information by the MR was the commonest reason for changing the fitness category, but for all other conditions factors such as the experience and knowledge of the MRs or their different interpretation of the standards were the most frequent reasons for a change to fitness category or to restrictions." ], "LABELS": [ "INTRODUCTION", "MATERIAL AND METHODS", "RESULTS", "DISCUSSION" ], "MESHES": [ "Adult", "Cardiovascular Diseases", "Evidence-Based Practice", "Humans", "Male", "Mental Disorders", "Middle Aged", "Naval Medicine", "Occupational Medicine", "Physical Examination", "Physical Fitness", "Practice Patterns, Physicians'", "Referral and Consultation", "Reproducibility of Results", "Risk Assessment", "Ships", "United Kingdom", "Young Adult" ], "YEAR": "2012", "reasoning_required_pred": "no", "reasoning_free_pred": "no", "final_decision": "no", "LONG_ANSWER": "This study found that there was poor agreement between the AD's decision and the subsequent MR's decision regarding the fitness of those seafarers who decided to appeal against the AD's initial assessment. The reasons for this are considered." }, "21850494": { "QUESTION": "Hepatorenal syndrome: are we missing some prognostic factors?", "CONTEXTS": [ "Hepatorenal syndrome (HRS) is the functional renal failure associated with advanced cirrhosis and has also been described in fulminant hepatic failure. Without liver transplantation its prognosis is dismal. Our study included patients with type 1 HRS associated with cirrhosis, who were not liver transplant candidates.AIM: To identify variables associated with improved survival.", "Sixty-eight patients fulfilled the revised Ascites Club Criteria for type 1 HRS. None of them was suitable for liver transplantation. All the patients were treated with combinations of: albumin, midodrine and octreotide, pressors, and hemodialysis.", "Median survival was 13 days for the whole group. Survival varied with the end-stage liver disease (ESLD) etiology: autoimmune, 49 days, cardiac cirrhosis, 22 days, idiopathic, 15.5 days, viral, 15 days, hepatitis C and alcohol, 14.5 days, alcohol 8 days, and neoplasia 4 days (p = 0.048). Survival of HRS associated with alcoholic liver disease versus other etiologies was not statistically significant (p = 0.1). Increased serum creatinine (p = 0.02) and urinary sodium 6-10 mEq/l (p = 0.027) at the initiation of therapy were prognostic factors for mortality. HRS treatment modalities (p = 0.73), use of dialysis (p = 0.56), dialysis modality (p = 0.35), use of vasopressors (p = 0.26), pre-existing renal disease (p = 0.49), gender (p = 0.90), and age (p = 0.57) were not associated with survival." ], "LABELS": [ "BACKGROUND", "METHODS", "RESULTS" ], "MESHES": [ "Albumins", "Alcoholism", "Autoimmune Diseases", "Creatinine", "End Stage Liver Disease", "Female", "Hepatitis C", "Hepatorenal Syndrome", "Humans", "Liver Cirrhosis", "Male", "Middle Aged", "Midodrine", "Octreotide", "Prognosis", "Renal Dialysis", "Retrospective Studies", "Sodium", "Survival Rate" ], "YEAR": "2012", "reasoning_required_pred": "yes", "reasoning_free_pred": "yes", "final_decision": "yes", "LONG_ANSWER": "We report for the first time ESLD etiology as a prognostic factor for survival. The renal function (expressed as serum creatinine) and urinary Na (<5 mEq/l) at the time of diagnosis were found to be associated with survival, suggesting that early treatment might increase survival." }, "26778755": { "QUESTION": "Vaginal dose assessment in image-guided brachytherapy for cervical cancer: Can we really rely on dose-point evaluation?", "CONTEXTS": [ "Although dose-volume parameters in image-guided brachytherapy have become a standard, the use of posterior-inferior border of the pubic symphysis (PIBS) points has been recently proposed in the reporting of vaginal doses. The aim was to evaluate their pertinence.", "Nineteen patients who received image-guided brachytherapy after concurrent radiochemotherapy were included. Per treatment, CT scans were performed at Days 2 and 3, with reporting of the initial dwell positions and times. Doses delivered to the PIBS points were evaluated on each plan, considering that they were representative of one-third of the treatment. The movements of the applicator according to the PIBS point were analysed.", "Mean prescribed doses at PIBS -2, PIBS, PIBS +2 were, respectively, 2.23 \u00b1 1.4, 6.39 \u00b1 6.6, and 31.85 \u00b1 36.06 Gy. Significant differences were observed between the 5 patients with vaginal involvement and the remaining 14 at the level of PIBS +2 and PIBS: +47.60 Gy and +7.46 Gy, respectively (p = 0.023 and 0.03). The variations between delivered and prescribed doses at PIBS points were not significant. However, at International commission on radiation units and measurements rectovaginal point, the delivered dose was decreased by 1.43 \u00b1 2.49 Gy from the planned dose (p = 0.019). The delivered doses at the four points were strongly correlated with the prescribed doses with R(2) ranging from 0.93 to 0.95. The movements of the applicator in regard of the PIBS point assessed with the Digital Imaging and Communications in Medicine coordinates were insignificant." ], "LABELS": [ "PURPOSE", "METHODS AND MATERIALS", "RESULTS" ], "MESHES": [ "Brachytherapy", "Chemoradiotherapy", "Female", "Humans", "Pubic Symphysis", "Radiation Dosage", "Radiotherapy, Image-Guided", "Tomography, X-Ray Computed", "Uterine Cervical Neoplasms", "Vagina" ], "YEAR": null, "reasoning_required_pred": "yes", "reasoning_free_pred": "maybe", "final_decision": "maybe", "LONG_ANSWER": "The doses evaluated at PIBS points are not impacted by intrafractional movements. PIBS and PIBS +2 dose points allow distinguishing the plans of patients with vaginal infiltration. Further studies are needed to correlate these parameters with vaginal morbidity." }, "20187289": { "QUESTION": "Prescriptions as a proxy for asthma in children: a good choice?", "CONTEXTS": [ "Stock et al. (Eur Respir J 25:47-53, 2005) recently estimated asthma prevalence in Germany using claims data on prescriptions and hospital diagnoses and found high prevalence peaks in infants. Our objective was to critically assess and discuss various aspects of identifying children with asthma using prescription data.", "We replicated the selection procedure of Stock et al. using data on 290,919 children aged 0-17 years insured in the Gm\u00fcnder ErsatzKasse (GEK) in 2005. Asthma prevalence was also estimated in a sample of 17,641 children aged 0-17 years participating in the German Health Interview and Examination Survey for Children and Adolescents (KiGGS) from 2003 to 2006.", "In children aged 0-4 years insured in the GEK, prevalences were found to range from 11.7 to 17.7% for boys and from 7.2 to 11.1% for girls when the criteria of Stock et al. were applied. A steady decline in prevalences was observed in older age groups. Asthma prevalence estimated in the KiGGS data showed a quite different distribution. In the age group 0-4 years, prevalences were found to range from 0 to 2.6% in boys and from 0 to 1.0% in girls; in children>4 years, prevalences were found to increase with increasing age." ], "LABELS": [ "PURPOSE", "METHODS", "RESULTS" ], "MESHES": [ "Adolescent", "Age Distribution", "Age Factors", "Anti-Asthmatic Agents", "Asthma", "Child", "Child, Preschool", "Drug Prescriptions", "Drug Utilization", "Female", "Germany", "Health Care Surveys", "Humans", "Infant", "Infant, Newborn", "Insurance, Pharmaceutical Services", "Male", "Prevalence", "Reproducibility of Results", "Time Factors" ], "YEAR": "2010", "reasoning_required_pred": "yes", "reasoning_free_pred": "no", "final_decision": "no", "LONG_ANSWER": "When additional validation studies were taken into account, asthma medications were found to be prescribed not only for asthma but also for other respiratory diseases. In addition, not all children with current asthma had prescriptions. We therefore conclude that asthma medications are therefore not a good proxy for the disease." }, "27615402": { "QUESTION": "Does the familial transmission of drinking patterns persist into young adulthood?", "CONTEXTS": [ "Parental drinking has been shown to be associated with offspring drinking. However, the relationship appears to be more complex than often assumed and few studies have tracked it over longer time periods.", "To explore the long-term (10-year) transmission of familial drinking during adolescence to offspring drinking patterns in young adulthood.", "Swedish longitudinal study, assessing the relationship between familial drinking in 2000 and offspring drinking in 2010 using simultaneous quantile regression analysis (n=744).DATA: Data on familial drinking was gathered from the Swedish level-of-living surveys (LNU) and from partner LNU in 2000 while data on offspring drinking in young adulthood was gathered from LNU 2010. Drinking among offspring, parents and potential stepparents was measured through identical quantity-frequency indices referring to the past 12 months in 2010 and 2000 respectively.", "Young adults whose families were abstainers in 2000 drank substantially less across quintiles in 2010 than offspring of non-abstaining families. The difference, however, was not statistically significant between quintiles of the conditional distribution. Actual drinking levels in drinking families were not at all or weakly associated with drinking in offspring. Supplementary analyses confirmed these patterns." ], "LABELS": [ "BACKGROUND", "AIMS", "DESIGN", "RESULTS" ], "MESHES": [ "Adolescent", "Adult", "Alcohol Drinking", "Family", "Female", "Follow-Up Studies", "Humans", "Longitudinal Studies", "Male", "Parent-Child Relations", "Parents", "Surveys and Questionnaires", "Sweden", "Young Adult" ], "YEAR": "2016", "reasoning_required_pred": "no", "reasoning_free_pred": "maybe", "final_decision": "maybe", "LONG_ANSWER": "The association between familial drinking and offspring drinking in young adulthood exhibits clear non-linear trends. Changes in the lower part of the familial drinking distribution are strongly related to drinking in young adults, but the actual levels of drinking in drinking families appear less important in shaping the drinking patterns of the offspring in young adulthood." }, "20297950": { "QUESTION": "Proof of concept study: does fenofibrate have a role in sleep apnoea syndrome?", "CONTEXTS": [ "To investigate the effect of fenofibrate on sleep apnoea indices.", "Proof-of-concept study comprising a placebo run-in period (1 week, 5 weeks if fibrate washout was required) and a 4-week randomized, double-blind treatment period. Thirty-four subjects (mean age 55 years, body mass index 34 kg/m 2 , fasting triglycerides 3.5 mmol/L) with diagnosed sleep apnoea syndrome not treated with continuous positive airways pressure were enrolled and randomized to once daily treatment with fenofibrate (145 mg NanoCrystal(R) tablet) or placebo. Overnight polysomnography, computerized attention/vigilance tests and blood sampling for measurement of lipids, insulin, fasting plasma glucose and fibrinogen were performed at the end of each study period.", "NCT00816829.", "As this was an exploratory study, a range of sleep variables were evaluated. The apnoea/hypopnoea index (AHI) and percentage of time spent with arterial oxygen saturation (SpO(2))<90% were relevant as they have been evaluated in other clinical trials. Other variables included total apnoeas, hypopnoeas and oxygen desaturations, and non-cortical micro-awakenings related to respiratory events per hour.", "Fenofibrate treatment significantly reduced the percentage of time with SpO(2)<90% (from 9.0% to 3.5% vs. 10.0% to 11.5% with placebo, p = 0.007), although there was no significant change in the AHI (reduction vs. control 14% (95%CI -47 to 40%, p = 0.533). Treatment reduced obstructive apnoeas (by 44%, from 18.5 at baseline to 15.0 at end of treatment vs. 29.0 to 30.5 on placebo, p = 0.048), and non-cortical micro-awakenings per hour (from 23.5 to 18.0 vs. 24.0 to 25.0 with placebo, p = 0.004). Other sleep variables were not significantly influenced by fenofibrate.", "Exploratory study in patients with mild to moderate sleep apnoea, limited treatment duration; concomitant hypnotic treatment (35%); lack of correction for multiplicity of testing." ], "LABELS": [ "OBJECTIVE", "METHODS", "CLINICAL TRIAL REGISTRATION", "MAIN OUTCOME MEASURES", "RESULTS", "KEY LIMITATIONS" ], "MESHES": [ "Attention", "Double-Blind Method", "Female", "Fenofibrate", "Humans", "Hypolipidemic Agents", "Lipids", "Male", "Middle Aged", "Placebos", "Sleep", "Sleep Apnea Syndromes" ], "YEAR": "2010", "reasoning_required_pred": "yes", "reasoning_free_pred": "yes", "final_decision": "yes", "LONG_ANSWER": "The consistent direction of change in sleep indices in this proof-of-concept study may support further investigation of fenofibrate in moderate to severe sleep apnoea syndrome." }, "23735520": { "QUESTION": "Can mental imagery functional magnetic resonance imaging predict recovery in patients with disorders of consciousness?", "CONTEXTS": [ "To determine the potential prognostic value of using functional magnetic resonance imaging (fMRI) to identify patients with disorders of consciousness, who show potential for recovery.", "Observational study.", "Unit for acute rehabilitation care.", "Patients (N=22) in a vegetative state (VS; n=10) and minimally conscious state (MCS; n=12) during the first 200 days after the initial incident.", "Not applicable.", "Further course on the Coma Recovery Scale-Revised.", "Participants performed a mental imagery fMRI paradigm. They were asked to alternately imagine playing tennis and navigating through their home. In 14 of the 22 examined patients (VS, n=5; MCS, n=9), a significant activation of the regions of interest (ROIs) of the mental imagery paradigm could be found. All 5 patients with activation of a significant blood oxygen level dependent signal, who were in a VS at the time of the fMRI examination, reached at least an MCS at the end of the observation period. In contrast, 5 participants in a VS who failed to show activation in ROIs, did not (sensitivity 100%, specificity 100%). Six of 9 patients in an MCS with activation in ROIs emerged from an MCS. Of 3 patients in an MCS who did not show activation, 2 patients stayed in an MCS and 1 patient emerged from the MCS (sensitivity 85%, specificity 40%)." ], "LABELS": [ "OBJECTIVE", "DESIGN", "SETTING", "PARTICIPANTS", "INTERVENTIONS", "MAIN OUTCOME MEASURE", "RESULTS" ], "MESHES": [ "Adolescent", "Adult", "Aged", "Consciousness", "Female", "Humans", "Magnetic Resonance Imaging", "Male", "Middle Aged", "Persistent Vegetative State", "Prognosis", "Recovery of Function", "Rehabilitation Centers", "Young Adult" ], "YEAR": "2013", "reasoning_required_pred": "yes", "reasoning_free_pred": "yes", "final_decision": "yes", "LONG_ANSWER": "The fMRI paradigm mental imagery displays a high concordance with the further clinical course of patients in a VS. All 5 patients in a VS who showed significant activation of ROIs had a favorable further course until the end of the observation period. We therefore propose the term \"functional minimally conscious state\" for these patients. They may benefit from rehabilitation treatment. In cases where no significant activation was seen, the method has no prognostic value. Prediction of the clinical course of patients in an MCS by fMRI was considerably less accurate than in patients in a VS." }, "10759659": { "QUESTION": "The nurse cystoscopist: a feasible option?", "CONTEXTS": [ "To compare the accuracy achieved by a trained urology nurse practitioner (UNP) and consultant urologist in detecting bladder tumours during flexible cystoscopy.", "Eighty-three patients underwent flexible cystoscopy by both the UNP and consultant urologist, each unaware of the other's findings. Before comparing the findings, each declared whether there was tumour or any suspicious lesion requiring biopsy.", "Of 83 patients examined by flexible cystoscopy, 26 were found to have a tumour or a suspicious lesion. One tumour was missed by the UNP and one by the urologist; each tumour was minute. Analysis using the chance-corrected proportional agreement (Kappa) was 0.94, indicating very close agreement." ], "LABELS": [ "OBJECTIVES", "PATIENTS AND METHODS", "RESULTS" ], "MESHES": [ "Cystoscopy", "Evaluation Studies as Topic", "Feasibility Studies", "Humans", "Liability, Legal", "Nurse Practitioners", "Sensitivity and Specificity", "Urinary Bladder Neoplasms", "Urology" ], "YEAR": "2000", "reasoning_required_pred": "yes", "reasoning_free_pred": "yes", "final_decision": "yes", "LONG_ANSWER": "A UNP can be trained to perform cystoscopy and detect suspicious lesions as accurately as can a consultant urologist. Legal and training issues in implementation are important." }, "25592625": { "QUESTION": "Is cardiovascular evaluation necessary prior to and during beta-blocker therapy for infantile hemangiomas?", "CONTEXTS": [ "Although consensus guidelines for pretreatment evaluation and monitoring of propranolol therapy in patients with infantile hemangiomas (IH) have been formulated, little is known about the cardiovascular side effects.", "We sought to analyze cardiovascular evaluations in patients with IH at baseline and during treatment with an oral beta-blocker.", "Data from 109 patients with IH were retrospectively analyzed. Patient and family history, pretreatment electrocardiogram (ECG), heart rate, and blood pressure were evaluated before initiation of beta-blocker therapy. Blood pressure and standardized questionnaires addressing side effects were evaluated during treatment.", "Questionnaire analyses (n = 83) identified 3 cases with a family history of cardiovascular disease in first-degree relatives. ECG findings were normal in each case and no serious complication of therapy occurred. ECG abnormalities were found in 6.5% of patients but there were no contraindications to beta-blocker therapy and no major complications. Hypotension in 9 patients did not require therapy adjustment. In all, 88 parents (81%) reported side effects during beta-blocker treatment.", "The relatively small patient cohort is a limitation." ], "LABELS": [ "BACKGROUND", "OBJECTIVES", "METHODS", "RESULTS", "LIMITATIONS" ], "MESHES": [ "Adrenergic beta-Antagonists", "Cardiovascular Diseases", "Child, Preschool", "Cohort Studies", "Drug Monitoring", "Electrocardiography", "Female", "Hemangioma", "Humans", "Infant", "Infant, Newborn", "Male", "Propranolol", "Retrospective Studies", "Skin Neoplasms" ], "YEAR": "2015", "reasoning_required_pred": "no", "reasoning_free_pred": "no", "final_decision": "no", "LONG_ANSWER": "Pretreatment ECG is of limited value for patients with an unremarkable cardiovascular history and a normal heart rate and blood pressure. Hypotension may occur during treatment." }, "12595848": { "QUESTION": "Is specialty care associated with improved survival of patients with congestive heart failure?", "CONTEXTS": [ "Implementation of the complex treatment strategies that have been shown to improve survival of patients with congestive heart failure (CHF) may require certain expertise. We wanted to examine the association between pattern of outpatient care and survival of patients with CHF.", "In a retrospective cohort study conducted with national Veterans Health Administration (VHA) databases, we examined the association between the pattern of outpatient care and survival in 11,661 patients discharged from VA hospitals between October 1, 1991, and September 30, 1992, with the primary diagnosis of CHF (cohort 1). Patients were divided into 4 groups, on the basis of their pattern of outpatient care over a 12-month period after discharge: 1) general medicine clinic visits only (GM-only); 2) cardiology clinic visits only (CARD-only); 3) general medicine and cardiology (MIXED) clinic visits; and 4) neither general medicine nor cardiology clinic visits (no-GM/CARD). We used the Cox proportional hazards model to evaluate 1-year survival, controlling for clinical and demographic factors. Consistency of our results was examined by performing identical analysis on a cohort of patients discharged from VHA hospitals between October 1, 1994, and September 30, 1995 (cohort 2, n = 10,141).", "The overall 1-year mortality rate was 23% in the primary cohort. The unadjusted mortality rate was highest for patients in the no-GM/CARD follow up (29%) and lowest for patients in the MIXED group (19%). By use of the MIXED group as reference and adjusting for important clinical and demographic factors, the risk of death (risk ratio [95% CI]) was 1.12 (0.94-1.34) in the CARD-only group, 1.26 (1.15-1.38) in the GM-only group, and 1.48 (1.28-1.72) in the no-GM/CARD group. Cohort-2 results were consistent with cohort 1 for most covariates, and significant survival differences were again found between GM-only and the MIXED group (1.25 [1.14-1.37])." ], "LABELS": [ "BACKGROUND", "METHODS", "RESULTS" ], "MESHES": [ "Aged", "Cardiology", "Clinical Competence", "Cohort Studies", "Databases, Factual", "Family Practice", "Health Services Accessibility", "Heart Failure", "Hospitals, Veterans", "Humans", "Male", "Outcome Assessment (Health Care)", "Outpatient Clinics, Hospital", "Patient Discharge", "Prognosis", "Proportional Hazards Models", "Survival Rate", "United States" ], "YEAR": "2003", "reasoning_required_pred": "yes", "reasoning_free_pred": "yes", "final_decision": "yes", "LONG_ANSWER": "We found an improved survival associated with cardiologist care and a mixture of general practitioner and cardiologist care compared with general practitioner care. The pattern of outpatient care may therefore be important for the survival of patients with CHF." }, "20064872": { "QUESTION": "Can the prognosis of polymyalgia rheumatica be predicted at disease onset?", "CONTEXTS": [ "To identify the features of PMR that may predict the duration of steroid therapy, the occurrence of relapses and the late development of GCA.", "Prospective cohort study of 176 patients with PMR, followed up for 5 years. Baseline factors associated with the duration of steroids therapy were identified using Cox regression. Predictors of relapse and the late development of GCA were identified using binary logistic regression.", "A total of 176 patients with PMR were included, of whom 124 stopped steroids within 5 years. The probability of stopping steroids within 5 years was independently reduced by an elevated plasma viscosity (PV) [hazard ratio (HR) = 0.49; 95% CI 0.29, 0.82 for a PV>or = 2.00 mPa s compared with a PV15 mg prednisolone (HR = 0.63; 95% CI 0.41, 0.97; P = 0.036). Either of these independently reduced the chances of stopping steroids within a given time interval between 27 and 51%. No significant predictors of relapse were identified. Predictors of late GCA on univariable analysis were female sex [odds ratio (OR) = 8.16; 95% CI 1.06, 63.13; P = 0.044], HLA-DRB1*0101 or -*0401 alleles (OR = 4.95; 95% CI 1.05, 23.34; P = 0.043), PV>or = 2.00 mPa s compared with PV15 mg (OR = 4.53; 95% CI 1.61, 12.79; P = 0.004)." ], "LABELS": [ "OBJECTIVE", "METHODS", "RESULTS" ], "MESHES": [ "Aged", "Aged, 80 and over", "Anti-Inflammatory Agents", "Female", "Follow-Up Studies", "Giant Cell Arteritis", "HLA Antigens", "Humans", "Male", "Middle Aged", "Polymyalgia Rheumatica", "Predictive Value of Tests", "Prednisolone", "Prognosis", "Prospective Studies", "Regression Analysis", "Steroids" ], "YEAR": "2010", "reasoning_required_pred": "no", "reasoning_free_pred": "yes", "final_decision": "yes", "LONG_ANSWER": "A higher PV in PMR increases the risk of prolonged steroid therapy and late GCA. Female sex and particular HLA alleles may increase the risk of late GCA. Starting patients on>15 mg prednisolone is associated with a prolonged steroid duration." }, "23588461": { "QUESTION": "Should ascitis volume and anthropometric measurements be estimated in hospitalized alcoholic cirrotics?", "CONTEXTS": [ "Ascitis and undernutrition are frequent complications of cirrhosis, however ascitis volume and anthropometric assessment are not routinely documented or considered in prognostic evaluation. In a homogeneous cohort followed during two years these variables were scrutinized, aiming to ascertain relevance for longterm outcome.", "Population (N = 25, all males with alcoholic cirrhosis) was recruited among patients hospitalized for uncomplicated ascitis. Exclusion criteria were refractory or tense ascitis, cancer, spontaneous bacterial peritonitis, bleeding varices and critical illness. Measurements included ultrasonographically estimated ascitis volume, dry body mass index/BMI , upper arm anthropometrics, hematologic counts and liver function tests.", "Population (age 48.3 \u00b1 11.3 years, BMI 21.1 \u00b1 3.5 kg/m\u00b2, serum albumin 2.5 \u00b1 0.8 g/dL) was mostly in the Child-Pugh C category (77.8%) but clinically stable. During the follow-up period of 22.6 \u00b1 3.8 months, additional hospitalizations numbered 1.7 \u00b1 1.0 and more than one quarter succumbed. Admission ascitis volume corresponded to 7.1 \u00b1 3.6 L and dry BMI to 18.3 \u00b1 3.5 kg/m\u00b2. Child Pugh index was relevant for both mortality and rehospitalization. Nevertheless, similar matches for mortality were documented with ascitis volume and dry BMI, and arm circumference below the 5th percentile was highly significantly associated with rehospitalization." ], "LABELS": [ "INTRODUCTION", "METHODS", "RESULTS" ], "MESHES": [ "Adult", "Anthropometry", "Ascites", "Ascitic Fluid", "Blood Chemical Analysis", "Cohort Studies", "Female", "Hospitalization", "Humans", "Liver Cirrhosis, Alcoholic", "Male", "Middle Aged", "Recurrence", "Treatment Outcome" ], "YEAR": null, "reasoning_required_pred": "yes", "reasoning_free_pred": "maybe", "final_decision": "yes", "LONG_ANSWER": "A greater association than hitherto acknowledged, between ascitis volume and anthropometric measurements from one side, and long-term rehospitalization and mortality from the other, was demonstrated in male stable alcoholic cirrhotics. Further studies with alcoholic and other modalities of cirrhosis including women are recommended." }, "18019905": { "QUESTION": "The use of audit to identify maternal mortality in different settings: is it just a difference between the rich and the poor?", "CONTEXTS": [ "To illustrate how maternal mortality audit identifies different causes of and contributing factors to maternal deaths in different settings in low- and high-income countries and how this can lead to local solutions in reducing maternal deaths.", "Descriptive study of maternal mortality from different settings and review of data on the history of reducing maternal mortality in what are now high-income countries.", "Kalabo district in Zambia, Farafenni division in The Gambia, Onandjokwe district in Namibia, and the Netherlands.", "Population of rural areas in Zambia and The Gambia, peri-urban population in Namibia and nationwide data from The Netherlands.", "Data from facility-based maternal mortality audits from three African hospitals and data from the latest confidential enquiry in The Netherlands.", "Maternal mortality ratio (MMR), causes (direct and indirect) and characteristics.", "MMR ranged from 10 per 100,000 (the Netherlands) to 1540 per 100,000 (The Gambia). Differences in causes of deaths were characterized by HIV/AIDS in Namibia, sepsis and HIV/AIDS in Zambia, (pre-)eclampsia in the Netherlands and obstructed labour in The Gambia." ], "LABELS": [ "OBJECTIVE", "DESIGN", "SETTINGS", "POPULATION", "METHODS", "MAIN OUTCOME MEASURES", "RESULTS" ], "MESHES": [ "Africa", "Cause of Death", "Female", "Hospital Mortality", "Humans", "Maternal Mortality", "Medical Audit", "Netherlands" ], "YEAR": "2007", "reasoning_required_pred": "no", "reasoning_free_pred": "no", "final_decision": "no", "LONG_ANSWER": "Differences in maternal mortality are more than just differences between the rich and poor. Acknowledging the magnitude of maternal mortality and harnessing a strong political will to tackle the issues are important factors. However, there is no single, general solution to reduce maternal mortality, and identification of problems needs to be promoted through audit, both national and local." }, "25859857": { "QUESTION": "Could the extent of lymphadenectomy be modified by neoadjuvant chemotherapy in cervical cancer?", "CONTEXTS": [ "The effect of neoadjuvant chemotherapy (NACT) on topographical distribution patterns of lymph node metastasis in cervical cancer was unknown.", "Patients with FIGO stage IB1-IIB who underwent radical surgery with or without NACT were enrolled (3527 patients). A matched-case comparison design was used to compare the effects of NACT on lymph node metastasis.", "We analyzed groups of 167 and 140 patients who were diagnosed with lymph node metastasis in the matched primary surgery group and NACT group, respectively, and no significant difference was observed (p = 0.081). The incidence of lymph node metastasis was significantly decreased in the NACT-responsive group compared to the non-responsive group (18.4% vs. 38.6%, P<0.001). The metastatic rates for every lymph node group also declined in the NACT-responsive group except for the deep inguinal and the para-aortic lymph node groups. Clinical response, deep stromal, parametrial and lymph vascular invasions were independent risk factors for lymph node metastasis in the NACT group. Furthermore, deep stromal invasion and lymph vascular invasion, but not the response to NACT, were independently associated with upper LNM. The number of lymph nodes involved, response to NACT, tumor histology and a positive vaginal margin were independent prognostic factors affecting DFS or OS rates in node-positive patients treated with NACT plus radical surgery." ], "LABELS": [ "BACKGROUND", "METHODS", "RESULTS" ], "MESHES": [ "Adult", "Aged", "Antineoplastic Combined Chemotherapy Protocols", "Female", "Humans", "Lymph Node Excision", "Lymph Nodes", "Lymphatic Metastasis", "Middle Aged", "Neoadjuvant Therapy", "Neoplasm Staging", "Retrospective Studies", "Risk Factors", "Treatment Outcome", "Uterine Cervical Neoplasms", "Young Adult" ], "YEAR": "2015", "reasoning_required_pred": "yes", "reasoning_free_pred": "no", "final_decision": "no", "LONG_ANSWER": "The frequency and topographic distribution of LNM are not modified by NACT, and clinical non-responders showed more involved LNs. A systemic and extensive lymphadenectomy should be performed in patients treated with NACT plus surgery regardless of the response to NACT." }, "26418796": { "QUESTION": "Do Wound Cultures Give Information About the Microbiology of Blood Cultures in Severe Burn Patients?", "CONTEXTS": [ "Blood stream infection (BSI) and the subsequent development of sepsis are among the most common infection complications occurring in severe burn patients. This study was designed to evaluate the relationship between the burn wound flora and BSI pathogens.", "Documentation of all bacterial and fungal wound and blood isolates from severe burn patients hospitalized in the burn unit and intensive care unit was obtained from medical records retrieved retrospectively from a computerized, hospital-wide database over a 13-year period. All data were recorded in relation to the Ryan score.", "Of 195 severe burn patients, 88 had at least 1 BSI episode. Transmission of the same pathogen from wound to blood was documented in 30% of the patients, with a rising BSI frequency as the Ryan score increased. There were a total of 263 bacteremic episodes in 88 study patients, 44% of blood isolates were documented previously in wound cultures, and transmission of the same pathogen from wound to blood was noted in 65% of bacteremic patients." ], "LABELS": [ "BACKGROUND", "METHODS", "RESULTS" ], "MESHES": [ "Adult", "Aged", "Anti-Bacterial Agents", "Bacteremia", "Burn Units", "Burns", "Chi-Square Distribution", "Cohort Studies", "Confidence Intervals", "Databases, Factual", "Female", "Fungemia", "Humans", "Injury Severity Score", "Israel", "Male", "Microbial Sensitivity Tests", "Middle Aged", "Retrospective Studies", "Risk Assessment", "Severity of Illness Index", "Wound Infection" ], "YEAR": "2016", "reasoning_required_pred": "yes", "reasoning_free_pred": "yes", "final_decision": "yes", "LONG_ANSWER": "When there is clinical suspicion of sepsis, appropriate empirical systemic antibiotic therapy should be broad spectrum and should rely on the susceptibility of the organisms from recent cultures of the burn wound surface, until the blood cultures results are completed." }, "22825590": { "QUESTION": "Are behavioural risk factors to be blamed for the conversion from optimal blood pressure to hypertensive status in Black South Africans?", "CONTEXTS": [ "Longitudinal cohort studies in sub-Saharan Africa are urgently needed to understand cardiovascular disease development. We, therefore, explored health behaviours and conventional risk factors of African individuals with optimal blood pressure (BP) (\u2264 120/80 mm Hg), and their 5-year prediction for the development of hypertension.", "The Prospective Urban Rural Epidemiology study in the North West Province, South Africa, started in 2005 and included African volunteers (n = 1994; aged>30 years) from a sample of 6000 randomly selected households in rural and urban areas.", "At baseline, 48% of the participants were hypertensive (\u2265 140/90 mmHg). Those with optimal BP (n = 478) were followed at a success rate of 70% for 5 years (213 normotensive, 68 hypertensive, 57 deceased). Africans that became hypertensive smoked more than the normotensive individuals (68.2% vs 49.8%), and they also had a greater waist circumference [ratio of geometric means of 0.94 cm (95% CI: 0.86-0.99)] and greater amount of \u03b3-glutamyltransferase [0.74 U/l (95% CI: 0.62-0.88)]at baseline. The 5-year change in BP was independently explained by baseline \u03b3-glutamyltransferase [R(2) = 0.23, \u03b2 = 0.13 U/l (95% CI: 0.01-0.19)]. Alcohol intake also predicted central systolic BP and carotid cross-sectional wall area (CSWA) at follow-up. Waist circumference was another predictor of BP changes [\u03b2 = 0.18 cm (95% CI: 0.05-0.24)]and CSWA. HIV infection was inversely associated with increased BP." ], "LABELS": [ "BACKGROUND", "METHODS", "RESULTS" ], "MESHES": [ "African Continental Ancestry Group", "Anthropometry", "Biomarkers", "Blood Pressure", "C-Reactive Protein", "Chi-Square Distribution", "Creatinine", "Female", "Health Behavior", "Humans", "Hypertension", "Linear Models", "Lipids", "Male", "Middle Aged", "Predictive Value of Tests", "Prospective Studies", "Risk Factors", "South Africa", "gamma-Glutamyltransferase" ], "YEAR": "2012", "reasoning_required_pred": "yes", "reasoning_free_pred": "yes", "final_decision": "yes", "LONG_ANSWER": "During the 5 years, 24% of Africans with optimal BP developed hypertension. The surge in hypertension in Africa is largely explained by modifiable risk factors. Public health strategies should focus aggressively on lifestyle to prevent a catastrophic burden on the national health system." }, "18832500": { "QUESTION": "Systematic use of patient-rated depression severity monitoring: is it helpful and feasible in clinical psychiatry?", "CONTEXTS": [ "The gap between evidence-based treatments and routine care has been well established. Findings from the Sequenced Treatments Alternatives to Relieve Depression (STAR*D) emphasized the importance of measurement-based care for the treatment of depression as a key ingredient for achieving response and remission; yet measurement-based care approaches are not commonly used in clinical practice.", "The Nine-Item Patient Health Questionnaire (PHQ-9) for monitoring depression severity was introduced in 19 diverse psychiatric practices. During the one-year course of the project the helpfulness and feasibility of implementation of PHQ-9 in these psychiatric practices were studied. The project was modeled after the Institute for Healthcare Improvement Breakthrough Series. Two of the 19 practices dropped out during the course of the project.", "By the conclusion of the study, all remaining 17 practices had adopted PHQ-9 as a routine part of depression care in their practice. On the basis of responses from 17 psychiatrists from those practices, PHQ-9 scores influenced clinical decision making for 93% of 6,096 patient contacts. With the additional information gained from the PHQ-9 score, one or more treatment changes occurred during 40% of these clinical contacts. Changing the dosage of antidepressant medication and adding another medication were the most common treatment changes recorded by psychiatrists, followed by starting or increasing psychotherapy and by switching or initiating antidepressants. In 3% of the patient contacts, using the PHQ-9 led to additional suicide risk assessment." ], "LABELS": [ "OBJECTIVE", "METHODS", "RESULTS" ], "MESHES": [ "Depression", "Female", "Humans", "Male", "Mass Screening", "Middle Aged", "Patient Participation", "Psychiatry", "Severity of Illness Index", "Surveys and Questionnaires", "United States" ], "YEAR": "2008", "reasoning_required_pred": "yes", "reasoning_free_pred": "yes", "final_decision": "yes", "LONG_ANSWER": "The study findings suggest that adopting measurement-based care, such as using the PHQ-9, is achievable, even in practices with limited resources." }, "18955431": { "QUESTION": "Treadmill training post stroke: are there any secondary benefits?", "CONTEXTS": [ "To explore the secondary benefits of treadmill training for people in the chronic stage of recovery from stroke.", "Modified random assignment, matched-pair control group design with repeated measures.", "Outpatient stroke centre.", "Twenty individuals post first stroke who acknowledged walking slower than pre stroke. Participants matched by side of hemiparesis and motor impairment.", "Twelve 20-minute sessions of walking on a treadmill or weekly phone call.", "Depression (Beck Depression Index), mobility and social participation (Stroke Impact Scale 3.0 subscales) were assessed initially, at the end of 12 treatments (four weeks) and six weeks later.", "No significant difference was found between groups for any dependent measure. The ANOVA to investigate main effects in each group found no significant findings in the control group; however in the treatment group significant improvements over time for depression (P = 0.005, P<0.001), mobility (P = 0.008) and social participation (P = 0.004) were demonstrated." ], "LABELS": [ "OBJECTIVE", "DESIGN", "SETTING", "PARTICIPANTS", "INTERVENTIONS", "MAIN OUTCOME MEASURES", "RESULTS" ], "MESHES": [ "Aged", "Analysis of Variance", "Depression", "Exercise Therapy", "Exercise Tolerance", "Female", "Gait", "Humans", "Male", "Middle Aged", "Paresis", "Pilot Projects", "Quality of Life", "Recovery of Function", "Severity of Illness Index", "Social Behavior", "Stroke", "Stroke Rehabilitation", "Surveys and Questionnaires", "Treatment Outcome", "Walking" ], "YEAR": null, "reasoning_required_pred": "yes", "reasoning_free_pred": "yes", "final_decision": "yes", "LONG_ANSWER": "A task-specific intervention designed to improve gait speed may potentially provide secondary benefits by positively impacting depression, mobility and social participation for people post stroke." }, "9550200": { "QUESTION": "Does lunar position influence the time of delivery?", "CONTEXTS": [ "To study the relationship between lunar position and the day of delivery; to investigate the synodic distribution of spontaneous deliveries, especially in relation to the presence of a full moon.", "Retrospective analysis of 1248 spontaneous full-term deliveries in three-year period (36 lunar months), setted at Department of Obstetrics and Gynaecology, Civil Hospital, Fano (Marche, Italy), using circular statistics techniques.", "A connection between the distribution of spontaneous full-term deliveries and the lunar month was found. The effect of the phases of the moon seems to be particularly relevant in multiparae and plurigravidae; in these cases, the mean day of delivery corresponds to the first or second day after the full moon." ], "LABELS": [ "OBJECTIVE", "STUDY DESIGN", "RESULTS" ], "MESHES": [ "Confidence Intervals", "Delivery, Obstetric", "Female", "Humans", "Moon", "Parity", "Pregnancy", "Pregnancy, Multiple", "Retrospective Studies" ], "YEAR": "1998", "reasoning_required_pred": "yes", "reasoning_free_pred": "yes", "final_decision": "yes", "LONG_ANSWER": "In this paper the effect of lunar phases on the time of delivery is shown. This influence seems to be especially relevant in the case of multiparae and plurigravidae. Nevertheless, it is too weak to allow for prediction regarding the days with the highest frequency of deliveries." }, "10135926": { "QUESTION": "Is oral endotracheal intubation efficacy impaired in the helicopter environment?", "CONTEXTS": [ "Patients transported by helicopter often require advanced airway management. The purpose of this study was to determine whether or not the in-flight environment of air medical transport in a BO-105 helicopter impairs the ability of flight nurses to perform oral endotracheal intubation.", "The study was conducted in an MBB BO-105 helicopter.", "Flight nurses performed three manikin intubations in each of the two study environments: on an emergency department stretcher and in-flight in the BO-105 helicopter.", "The mean time required for in-flight intubation (25.9 +/- 10.9 seconds) was significantly longer than the corresponding time (13.2 +/- 2.8 seconds) required for intubation in the control setting (ANOVA, F = 38.7, p<.001). All intubations performed in the control setting were placed correctly in the trachea; there were two (6.7%) esophageal intubations in the in-flight setting. The difference in appropriate endotracheal intubation between the two settings was not significant (chi 2 = 0.3; p>0.05)." ], "LABELS": [ "INTRODUCTION", "SETTING", "METHODS", "RESULTS" ], "MESHES": [ "Air Ambulances", "Analysis of Variance", "Data Collection", "Emergency Nursing", "Humans", "Intubation, Intratracheal", "North Carolina", "Time and Motion Studies", "Transportation of Patients", "Treatment Outcome" ], "YEAR": "1994", "reasoning_required_pred": "yes", "reasoning_free_pred": "yes", "final_decision": "yes", "LONG_ANSWER": "Oral endotracheal intubation in the in-flight setting of the BO-105 helicopter takes approximately twice as long as intubation in a ground setting. The results support pre-flight intubation of patients who appear likely to require urgent intubation during air medical transport in the BO-105 helicopter." }, "11296674": { "QUESTION": "Prostatic syndrome and pleural effusion: are they different diseases?", "CONTEXTS": [ "To report an uncommon association of prostate and lung cancer.", "The characteristics of both tumors, their association with tumors in other sites and the time of presentation are analyzed.", "Both tumors were in the advanced stages. Metastatic carcinoma of the prostate was discarded due to the form of presentation." ], "LABELS": [ "OBJECTIVE", "METHODS", "RESULTS" ], "MESHES": [ "Adenocarcinoma", "Aged", "Humans", "Lung Neoplasms", "Male", "Neoplasms, Multiple Primary", "Pleural Effusion, Malignant", "Prostatic Neoplasms" ], "YEAR": null, "reasoning_required_pred": "yes", "reasoning_free_pred": "maybe", "final_decision": "maybe", "LONG_ANSWER": "Although the association of prostate and lung cancer is uncommon, the possibility of synchronous tumors should be considered in patients with urinary and pulmonary symptoms suggestive of neoplasm. It is important to determine if the lesion is a metastasis, since the prognosis depends on the second tumor." }, "24139705": { "QUESTION": "Telemedicine and type 1 diabetes: is technology per se sufficient to improve glycaemic control?", "CONTEXTS": [ "Each patient received a smartphone with an insulin dose advisor (IDA) and with (G3 group) or without (G2 group) the telemonitoring/teleconsultation function. Patients were classified as \"high users\" if the proportion of \"informed\" meals using the IDA exceeded 67% (median) and as \"low users\" if not. Also analyzed was the respective impact of the IDA function and teleconsultations on the final HbA1c levels.", "Among the high users, the proportion of informed meals remained stable from baseline to the end of the study 6months later (from 78.1\u00b121.5% to 73.8\u00b125.1%; P=0.107), but decreased in the low users (from 36.6\u00b129.4% to 26.7\u00b128.4%; P=0.005). As expected, HbA1c improved in high users from 8.7% [range: 8.3-9.2%] to 8.2% [range: 7.8-8.7%]in patients with (n=26) vs without (n=30) the benefit of telemonitoring/teleconsultation (-0.49\u00b10.60% vs -0.52\u00b10.73%, respectively; P=0.879). However, although HbA1c also improved in low users from 9.0% [8.5-10.1] to 8.5% [7.9-9.6], those receiving support via teleconsultation tended to show greater improvement than the others (-0.93\u00b10.97 vs -0.46\u00b11.05, respectively; P=0.084)." ], "LABELS": [ "METHODS", "RESULTS" ], "MESHES": [ "Adult", "Blood Glucose", "Cell Phone", "Diabetes Mellitus, Type 1", "Female", "Glycated Hemoglobin A", "Humans", "Hypoglycemic Agents", "Insulin", "Insulin Infusion Systems", "Internet", "Male", "Patient Compliance", "Reminder Systems", "Remote Consultation", "Self Care", "Software", "Telemedicine" ], "YEAR": "2014", "reasoning_required_pred": "yes", "reasoning_free_pred": "yes", "final_decision": "yes", "LONG_ANSWER": "The Diabeo system improved glycaemic control in both high and low users who avidly used the IDA function, while the greatest improvement was seen in the low users who had the motivational support of teleconsultations." }, "16253970": { "QUESTION": "Is controlled ovarian stimulation in intrauterine insemination an acceptable therapy in couples with unexplained non-conception in the perspective of multiple pregnancies?", "CONTEXTS": [ "Controlled ovarian stimulation (COS) with intrauterine insemination (IUI) is a common treatment in couples with unexplained non-conception. Induction of multifollicular growth is considered to improve pregnancy outcome, but it contains an increased risk of multiple pregnancies and ovarian hyperstimulation syndrome. In this study the impact of the number of follicles (>14 mm) on the ongoing pregnancy rate (PR) and multiple PR was evaluated in the first four treatment cycles.", "A retrospective cohort study was performed in all couples with unexplained non-conception undergoing COS-IUI in the Academic Hospital of Maastricht. The main outcome measure was ongoing PR. Secondary outcomes were ongoing multiple PR, number of follicles of>or=14 mm, and order of treatment cycle.", "Three hundred couples were included. No significant difference was found in ongoing PR between women with one, two, three or four follicles respectively (P=0.54), but in women with two or more follicles 12/73 pregnancies were multiples. Ongoing PR was highest in the first treatment cycle and declined significantly with increasing cycle order (P=0.006), while multiple PR did not change." ], "LABELS": [ "BACKGROUND", "METHODS", "RESULTS" ], "MESHES": [ "Cohort Studies", "Female", "Humans", "Insemination, Artificial", "Netherlands", "Ovulation Induction", "Patient Satisfaction", "Pregnancy", "Pregnancy Outcome", "Pregnancy, Multiple", "Retrospective Studies" ], "YEAR": "2006", "reasoning_required_pred": "maybe", "reasoning_free_pred": "no", "final_decision": "no", "LONG_ANSWER": "In COS-IUI for unexplained non-conception, induction of more than one follicle did not improve the ongoing PR, but increased the risk of multiple pregnancies. Multiple PR remained high in the first four cycles with multifollicular stimulation. Therefore, in order to reduce the number of multiple pregnancies, in all IUI cycles for unexplained non-conception monofollicular growth should be aimed at." }, "14745753": { "QUESTION": "Is half-dose contrast-enhanced three-dimensional MR angiography sufficient for the abdominal aorta and pelvis?", "CONTEXTS": [ "To evaluate the usefulness of half-dose contrast-enhanced magnetic resonance (MR) angiography for depicting the abdominal aorta and its major branches.", "A total of 72 consecutive patients were randomly assigned to one of four groups that underwent MR angiography after receiving different concentrations (original or diluted to 50%) and total amounts (single or half-dose) of gadolinium chelate injected at different rates (1 or 0.5 mL/second). The signal-to-noise ratio (SNR) and contrast-to-noise ratio (CNR) of the abdominal aorta and of the common and external iliac arteries were calculated, and two blinded readers rated the respective image qualities.", "The SNR and CNR of the abdominal aorta and the common iliac artery in the 0.5 mL/second groups were statistically significantly lower than those in the 1 mL/second groups. The differences in overall image quality across the four groups were not statistically significant." ], "LABELS": [ "PURPOSE", "MATERIALS AND METHODS", "RESULTS" ], "MESHES": [ "Analysis of Variance", "Aorta, Abdominal", "Contrast Media", "Dose-Response Relationship, Drug", "Female", "Gadolinium DTPA", "Humans", "Imaging, Three-Dimensional", "Injections, Intravenous", "Magnetic Resonance Angiography", "Middle Aged", "Pelvis", "Phantoms, Imaging", "Statistics, Nonparametric" ], "YEAR": "2004", "reasoning_required_pred": "yes", "reasoning_free_pred": "yes", "final_decision": "yes", "LONG_ANSWER": "Half-dose MR angiography using diluted contrast medium injected at a rate of 1 mL/second depicted the abdominal aorta and its branches as clearly as using a full single dose." }, "27078715": { "QUESTION": "Digital Tomosynthesis: A Viable Alternative to Noncontrast Computed Tomography for the Follow-Up of Nephrolithiasis?", "CONTEXTS": [ "Digital tomosynthesis (DT) is a new X-ray-based imaging technique that allows image enhancement with minimal increase in radiation exposure. The purpose of this study was to compare DT with noncontrast computed tomography (NCCT) and to evaluate its potential role for the follow-up of patients with nephrolithiasis in a nonemergent setting.", "A retrospective review of patients with nephrolithiasis at our institution that underwent NCCT and DT from July 2012 to September 2013 was performed. Renal units (RUs) that did not undergo treatment or stone passage were randomly assigned to two blinded readers, who recorded stone count, size area (mm(2)), maximum stone length (mm), and location, for both DT and NCCT. Mean differences per RU were compared. Potential variables affecting stone detection rate, including stone size and body mass index (BMI), were evaluated. Interobserver agreement was determined using the intraclass correlation coefficient to measure the consistency of measurements made by the readers.", "DT and NCCT demonstrated similar stone detection rates in terms of stone counts and stone area mm(2). Of the 79 RUs assessed, 41 RUs showed exact stone counts on DT and NCCT. The mean difference in stone area was 16.5\u2009mm(2) (-4.6 to 38.5), p\u2009=\u20090.121. The mean size of the largest stone on NCCT and DT was 9.27 and 8.87\u2009mm, respectively. Stone size and BMI did not cause a significant difference in stone detection rates. Interobserver agreement showed a strong correlation between readers and adequate reproducibility." ], "LABELS": [ "OBJECTIVE", "METHODS", "RESULTS" ], "MESHES": [ "Costs and Cost Analysis", "Follow-Up Studies", "Humans", "Kidney Calculi", "Lithotripsy", "Observer Variation", "Radiographic Image Enhancement", "Radiography", "Random Allocation", "Reproducibility of Results", "Retrospective Studies", "Tomography, X-Ray Computed" ], "YEAR": "2016", "reasoning_required_pred": "yes", "reasoning_free_pred": "yes", "final_decision": "yes", "LONG_ANSWER": "We found DT to be a comparable imaging modality to NCCT for the detection of intrarenal stones, without a significant effect from stone size and BMI and adequate reproducibility between multiple readers. DT appears to be an ideal alternative for following patients with nephrolithiasis due to its acceptable stone detection rates, low radiation exposure, and decreased cost compared to NCCT." }, "15939071": { "QUESTION": "High cumulative insulin exposure: a risk factor of atherosclerosis in type 1 diabetes?", "CONTEXTS": [ "Since insulin therapy might have an atherogenic effect, we studied the relationship between cumulative insulin dose and atherosclerosis in type 1 diabetes. We have focused on patients with type 1 diabetes instead of type 2 diabetes to minimise the effect of insulin resistance as a potential confounder.", "An observational study was performed in 215 subjects with type 1 diabetes treated with multiple insulin injection therapy. Atherosclerosis was assessed by measurement of carotid intima-media thickness (CIMT).", "The cumulative dose of regular insulin showed a positive and significant relation with CIMT: increase of 21 microm in CIMT per S.D. of insulin use (95% CI: 8-35 adjusted for gender and age), which remained unchanged after adjustment for duration of diabetes, HbA1c, BMI, pulse pressure, physical activity and carotid lumen diameter. A similar relation was found for intermediate-acting insulin: 15.5 microm per S.D. (2-29), which was no longer present after further adjustment." ], "LABELS": [ "BACKGROUND", "METHODS", "RESULTS" ], "MESHES": [ "Adult", "Arteriosclerosis", "Carotid Arteries", "Diabetes Mellitus, Type 1", "Dose-Response Relationship, Drug", "Female", "Humans", "Hypoglycemic Agents", "Insulin", "Male", "Middle Aged", "Multivariate Analysis", "Risk Factors", "Tunica Intima", "Tunica Media", "Ultrasonography" ], "YEAR": "2005", "reasoning_required_pred": "yes", "reasoning_free_pred": "yes", "final_decision": "yes", "LONG_ANSWER": "These findings provide evidence that a high cumulative dose of regular insulin is a risk factor for atherosclerosis." }, "11926574": { "QUESTION": "Are hepatitis G virus and TT virus involved in cryptogenic chronic liver disease?", "CONTEXTS": [ "Hepatitis G virus can cause chronic infection in man but the role of this agent in chronic liver disease is poorly understood. Little is known about the relation of another newly discovered agent, the TT virus, with chronic liver disease.AIM: To investigate the rate of infection with hepatitis G virus and TT virus in patients with cryptogenic chronic liver disease.", "A total of 23 subjects with chronically raised alanine transaminase and a liver biopsy in whom all known causes of liver disease had been excluded, and 40 subjects with hepatitis C virus-related chronic liver disease.", "Evaluation of anti-hepatitis G virus by enzyme immunoassay. Hepatitis G virus-RNA by polymerase chain reaction with primers from the 5' NC and NS5a regions. TT virus-DNA by nested polymerase chain reaction with primers from the ORF1 region. Results. Hepatitis G virus-RNA was detected in 4 out of 23 patients with cryptogenic chronic hepatitis and in 6 out of 40 with hepatitis C virus chronic hepatitis (17.4% vs 15% p=ns). At least one marker of hepatitis G virus infection (hepatitis G virus-RNA and/or anti-hepatitis G virus, mostly mutually exclusive) was present in 6 out of 23 patients with cryptogenic hepatitis and 16 out of 40 with hepatitis C virus liver disease (26. 1% vs 40% p=ns). T virus-DNA was present in serum in 3 subjects, 1 with cryptogenic and 2 with hepatitis C virus-related chronic liver disease. Demographic and clinical features, including stage and grade of liver histology, were comparable between hepatitis G virus-infected and uninfected subjects. Severe liver damage [chronic hepatitis with fibrosis or cirrhosis) were significantly more frequent in subjects with hepatitis C virus liver disease." ], "LABELS": [ "BACKGROUND", "PATIENTS", "METHODS" ], "MESHES": [ "Adult", "Alanine Transaminase", "DNA Virus Infections", "Female", "Flaviviridae Infections", "GB virus C", "Hepatitis, Chronic", "Hepatitis, Viral, Human", "Humans", "Liver", "Male", "Middle Aged", "Reverse Transcriptase Polymerase Chain Reaction", "Torque teno virus" ], "YEAR": "2002", "reasoning_required_pred": "yes", "reasoning_free_pred": "no", "final_decision": "no", "LONG_ANSWER": "In Southern Italy, hepatitis G virus infection is widespread among patients with chronic hepatitis, independently of parenteral risk factors. Its frequency in subjects with cryptogenic liver disease parallels that observed in hepatitis C virus chronic liver disease, thus ruling out an aetiologic role of hepatitis G virus. TT virus infection is uncommon in patients with cryptogenic or hepatitis C virus-related liver disease who do not have a history of parenteral exposure." }, "18594195": { "QUESTION": "Do older patients who refuse to participate in a self-management intervention in the Netherlands differ from older patients who agree to participate?", "CONTEXTS": [ "Refusal of patients to participate in intervention programs is an important problem in clinical trials but, in general, researchers devote relatively little attention to it. In this article, a comparison is made between patients who, after having been invited, agreed to participate in a self-management intervention (participants) and those who refused (refusers). Compared with other studies of refusers, relatively more information could be gathered with regard to both their characteristics and reasons for refusing, because all potential participants were invited personally.", "Older patients from a Dutch outpatient clinic were invited to participate in a self-management intervention, and their characteristics were assessed. Demographic data were collected, as well as data on physical functioning and lack of emotional support. People who refused to participate were asked to give their reasons for refusing.", "Of the 361 patients invited, 267 (74%) refused participation. These refusers were more restricted in their mobility, lived further away from the location of the intervention, and had a partner more often than did the participants. No differences were found in level of education, age or gender. The main reasons given by respondents for refusing to participate were lack of time, travel distance, and transport problems." ], "LABELS": [ "BACKGROUND AND AIMS", "METHODS", "RESULTS" ], "MESHES": [ "Aged", "Female", "Humans", "Logistic Models", "Male", "Motor Activity", "Netherlands", "Patients", "Refusal to Participate", "Self Care", "Surveys and Questionnaires" ], "YEAR": "2008", "reasoning_required_pred": "yes", "reasoning_free_pred": "yes", "final_decision": "yes", "LONG_ANSWER": "As in many studies, the refusal rate in this study is high, and seems to be related to physical mobility restrictions, travel distance and, partly, to availability of emotional support. These findings may be used to make the recruitment process more effective - for example, by offering transport to the location of the intervention." }, "25614468": { "QUESTION": "Preoperative locoregional staging of gastric cancer: is there a place for magnetic resonance imaging?", "CONTEXTS": [ "The aim of this study was to prospectively compare the diagnostic performance of magnetic resonance imaging (MRI), multidetector computed tomography (MDCT) and endoscopic ultrasonography (EUS) in the preoperative locoregional staging of gastric cancer.", "This study had Institutional Review Board approval, and informed consent was obtained from all patients. Fifty-two patients with biopsy-proven gastric cancer underwent preoperative 1.5-T MRI, 64-channel MDCT and EUS. All images were analysed blind, and the results were compared with histopathological findings according to the seventh edition of the TNM classification. After the population had been divided on the basis of the local invasion (T1-3 vs T4a-b) and nodal involvement (N0 vs N+), sensitivity, specificity, positive and negative predictive value, and accuracy were calculated and diagnostic performance measures were assessed using the McNemar test.", "For T staging, EUS showed higher sensitivity (94%) than MDCT and MRI (65 and 76%; p = 0.02 and p = 0.08). MDCT and MRI had significantly higher specificity (91 and 89%) than EUS (60%) (p = 0.0009 and p = 0.003). Adding MRI to MDCT or EUS did not result in significant differences for sensitivity. For N staging, EUS showed higher sensitivity (92%) than MRI and MDCT (69 and 73%; p = 0.01 and p = 0.02). MDCT showed better specificity (81%) than EUS and MRI (58 and 73%; p = 0.03 and p = 0.15)." ], "LABELS": [ "BACKGROUND", "METHODS", "RESULTS" ], "MESHES": [ "Adult", "Aged", "Aged, 80 and over", "Endosonography", "Female", "Humans", "Magnetic Resonance Imaging", "Male", "Middle Aged", "Multidetector Computed Tomography", "Multimodal Imaging", "Neoplasm Staging", "Preoperative Care", "Prospective Studies", "Sensitivity and Specificity", "Stomach Neoplasms" ], "YEAR": "2016", "reasoning_required_pred": "no", "reasoning_free_pred": "no", "final_decision": "no", "LONG_ANSWER": "Our prospective study confirmed the leading role of EUS and MDCT in the staging of gastric cancer and did not prove, at present, the value of the clinical use of MRI." }, "19653482": { "QUESTION": "Do familiar teammates request and accept more backup?", "CONTEXTS": [ "The present study investigated factors that explain when and why different groups of teammates are more likely to request and accept backup from one another when needed in an environment characterized by extreme time pressure and severe consequences of error: commercial air traffic control (ATC).", "Transactive memory theory states that teammates develop consensus regarding the distribution of their relative expertise as well as confidence in that expertise over time and that this facilitates coordination processes. The present study investigated whether this theory could help to explain between-team differences in requesting and accepting backup when needed.", "The present study used cross-sectional data collected from 51 commercial ATC teams. Hypotheses were tested using multiple regression analysis.", "Teammates with greater experience working together requested and accepted backup from one another more than those with lesser experience working together. Teammate knowledge consensus and perceived team efficacy appear to have mediated this relationship." ], "LABELS": [ "OBJECTIVE", "BACKGROUND", "METHOD", "RESULTS" ], "MESHES": [ "Aviation", "Efficiency, Organizational", "Humans", "Institutional Management Teams", "Interprofessional Relations", "Memory", "Professional Competence", "Regression Analysis", "Task Performance and Analysis" ], "YEAR": "2009", "reasoning_required_pred": "yes", "reasoning_free_pred": "yes", "final_decision": "yes", "LONG_ANSWER": "Transactive memory theory extends to high-stress environments in which members' expertise is highly overlapping. Teammates' shared mental models about one another increase the likelihood that they will request and accept backup." }, "11380492": { "QUESTION": "Transsphenoidal pituitary surgery in Cushing's disease: can we predict outcome?", "CONTEXTS": [ "To assess the results of transsphenoidal pituitary surgery in patients with Cushing's disease over a period of 18 years, and to determine if there are factors which will predict the outcome.", "Sixty-nine sequential patients treated surgically by a single surgeon in Newcastle upon Tyne between 1980 and 1997 were identified and data from 61 of these have been analysed.", "Retrospective analysis of outcome measures.", "Patients were divided into three groups (remission, failure and relapse) depending on the late outcome of their treatment as determined at the time of analysis, i.e. 88 months (median) years after surgery. Remission is defined as biochemical reversal of hypercortisolism with re-emergence of diurnal circadian rhythm, resolution of clinical features and adequate suppression on low-dose dexamethasone testing. Failure is defined as the absence of any of these features. Relapse is defined as the re-emergence of Cushing's disease more than one year after operation. Clinical features such as weight, sex, hypertension, associated endocrine disorders and smoking, biochemical studies including preoperative and postoperative serum cortisol, urine free cortisol, serum ACTH, radiological, histological and surgical findings were assessed in relation to these three groups to determine whether any factors could reliably predict failure or relapse after treatment.", "Of the 61 patients included in this study, 48 (78.7%) achieved initial remission and 13 (21.3%) failed treatment. Seven patients suffered subsequent relapse (range 22-158 months) in their condition after apparent remission, leaving a final group of 41 patients (67.2%) in the remission group. Tumour was identified at surgery in 52 patients, of whom 38 achieved remission. In comparison, only 3 of 9 patients in whom no tumour was identified achieved remission. This difference was significant (P = 0.048). When both radiological and histological findings were positive, the likelihood of achieving remission was significantly higher than if both modalities were negative (P = 0.038). There were significant differences between remission and failure groups when 2- and 6-week postoperative serum cortisol levels (P = 0.002 and 0.001, respectively) and 6-week postoperative urine free cortisol levels (P = 0.026) were compared. This allowed identification of patients who failed surgical treatment in the early postoperative period. Complications of surgery included transitory DI in 13, transitory CSF leak in 8 and transitory nasal discharge and cacosmia in 3. Twelve of 41 patients required some form of hormonal replacement therapy despite achieving long-term remission. Thirteen patients underwent a second operation, of whom 5 achieved remission." ], "LABELS": [ "OBJECTIVE", "PATIENTS", "DESIGN", "MAIN OUTCOME MEASURES", "RESULTS" ], "MESHES": [ "Adolescent", "Adult", "Aged", "Child", "Cushing Syndrome", "Female", "Humans", "Hydrocortisone", "Male", "Middle Aged", "Pituitary Gland", "Pituitary Neoplasms", "Postoperative Complications", "Recurrence", "Reoperation", "Retrospective Studies", "Treatment Failure", "Treatment Outcome" ], "YEAR": "2001", "reasoning_required_pred": "yes", "reasoning_free_pred": "yes", "final_decision": "yes", "LONG_ANSWER": "Transsphenoidal pituitary surgery is a safe method of treatment in patients with Cushing's disease. Operative findings, radiological and histological findings, together with early postoperative serum cortisol and urine free cortisol estimates may identify failures in treatment. Alternative treatment might then be required for these patients. Because of the risk of late relapse, patients require life-long follow-up." }, "16296668": { "QUESTION": "Can bedside assessment reliably exclude aspiration following acute stroke?", "CONTEXTS": [ "To investigate the ability of a bedside swallowing assessment to reliably exclude aspiration following acute stroke.", "Consecutive patients admitted within 24 h of stroke onset to two hospitals.", "A prospective study. Where possible, all patients had their ability to swallow assessed on the day of admission by both a doctor and a speech and language therapist using a standardized proforma. A videofluoroscopy examination was conducted within 3 days of admission.", "94 patients underwent videofluoroscopy; 20 (21%) were seen to be aspirating, although this was not detected at the bedside in 10. In 18 (22%) of the patients the speech and language therapist considered the swallow to be unsafe. In the medical assessment, 39 patients (41%) had an unsafe swallow. Bedside assessment by a speech and language therapist gave a sensitivity of 47%, a specificity of 86%, positive predictive value (PPV) of 50% and a negative predictive value (NPV) of 85% for the presence of aspiration. Multiple logistic regression was used to identify the optimum elements of the bedside assessments for predicting the presence of aspiration. A weak voluntary cough and any alteration in conscious level gave a sensitivity of 75%, specificity of 72%, PPV of 41% and NPV of 91% for aspiration." ], "LABELS": [ "OBJECTIVE", "SUBJECTS", "METHODS", "RESULTS" ], "MESHES": [ "Acute Disease", "Aged", "Aged, 80 and over", "Deglutition Disorders", "Female", "Fluoroscopy", "Geriatric Assessment", "Humans", "Male", "Middle Aged", "Pneumonia, Aspiration", "Predictive Value of Tests", "Prospective Studies", "Sensitivity and Specificity", "Stroke", "Videotape Recording" ], "YEAR": "1998", "reasoning_required_pred": "yes", "reasoning_free_pred": "no", "final_decision": "no", "LONG_ANSWER": "Bedside assessment of swallowing lacks the necessary sensitivity to be used as a screening instrument in acute stroke, but there are concerns about the use of videofluoroscopy as a gold standard. The relative importance of aspiration and bedside assessment in predicting complications and outcome needs to be studied." }, "19520213": { "QUESTION": "Are UK radiologists satisfied with the training and support received in suspected child abuse?", "CONTEXTS": [ "A list of telephone numbers of UK hospitals with a radiology department was obtained from the Royal College of Radiologists. One hundred hospitals were then randomly selected for inclusion in the survey. An 18-item questionnaire was successfully administered to consultant radiologists from 84 departments.", "Sixty-one percent of departments had a named radiologist to report their skeletal surveys, 16% assigned surveys to a random radiologist, and 23% referred them elsewhere. Only 52% of departments had a dedicated paediatric radiologist, thus in a significant proportion of departments (25%) initial reports on skeletal surveys for physical abuse were provided by non-paediatric radiologists. Fifteen percent did not have ready access to a paediatric radiology opinion. Sixty-one percent thought that the service could be improved. Expert evidence was provided by 5% of respondents. Seventy-three percent would never consider providing expert evidence, even if given adequate radiology and/or legal training." ], "LABELS": [ "MATERIALS AND METHODS", "RESULTS" ], "MESHES": [ "Attitude of Health Personnel", "Bone and Bones", "Child", "Child Abuse", "Clinical Competence", "Education, Medical, Continuing", "Humans", "Medical Staff, Hospital", "Radiography", "Radiology", "United Kingdom" ], "YEAR": "2009", "reasoning_required_pred": "no", "reasoning_free_pred": "no", "final_decision": "no", "LONG_ANSWER": "The survey shows significant dissatisfaction amongst consultant radiologists with the current service, confirms a low number of paediatric radiologists taking on this work, and suggests the potential to increase numbers of radiology child abuse experts by 27% if given improved training and support. Appropriate service and education strategies should be implemented." }, "14992556": { "QUESTION": "Artefacts in 24-h pharyngeal and oesophageal pH monitoring: is simplification of pH data analysis feasible?", "CONTEXTS": [ "Ambulatory 24-h dual-channel pharyngeal and oesophageal pH monitoring is the standard test for measuring gastro-oesophageal and gastropharyngeal reflux. Artefacts caused by the intake of food may result in falsely positive gastropharyngeal reflux, which necessitates a manual review of 24-h pH data. The purpose of the study was to investigate the influence of meals and whether leaving out meals affected the reliability of the test.", "Patients referred for otolaryngological complaints, suspected to have been caused by gastro-oesophageal reflux, underwent 24-h dual-channel pH monitoring. The raw unprocessed pH data were corrected by visual inspection of the 24-h tracings (corrected data), by leaving out meals or meals plus a 2-h postprandrial period.", "The raw pH data were substantially influenced by artefacts of food intake and pseudoreflux. Data obtained by leaving out meals agreed best with manually corrected data. Many of the falsely positive reflux episodes could be removed, thereby inducing a 9%-18% chance of undetected reflux. When examining the fraction of time supine, manually corrected data and data leaving out meals were fully concordant and detected 79% of patients with gastropharyngeal reflux. However, leaving out meals plus a 2-h postprandrial period resulted in 21%-50% falsely negative tests." ], "LABELS": [ "BACKGROUND", "METHODS", "RESULTS" ], "MESHES": [ "Adult", "Aged", "Aged, 80 and over", "Artifacts", "Eating", "Esophagus", "Feasibility Studies", "Female", "Gastroesophageal Reflux", "Humans", "Hydrogen-Ion Concentration", "Male", "Middle Aged", "Monitoring, Ambulatory", "Pharynx", "Postprandial Period", "Prospective Studies", "Reproducibility of Results", "Statistics as Topic" ], "YEAR": "2004", "reasoning_required_pred": "yes", "reasoning_free_pred": "maybe", "final_decision": "maybe", "LONG_ANSWER": "Leaving out the period of intake of meals and beverages from the raw pH data might be the second best test after the time-consuming visual correction with a small chance of undetected gastropharyngeal reflux. For scientific purposes and when in doubt, it remains necessary to review the computer-generated data manually to discover every gastropharyngeal reflux event." }, "19931500": { "QUESTION": "Can the condition of the cell microenvironment of mediastinal lymph nodes help predict the risk of metastases in non-small cell lung cancer?", "CONTEXTS": [ "The aim of this study was to analyze the properties of the immune cell microenvironment of regional lymph nodes (LNs) positive for lung cancer.", "Twenty-four patients operated on for stages T1 and T2 of the NSCLC, were enrolled in the study. Peripheral blood and LN tissue were obtained from different lymph node sites and levels. As a control, LN tissue was taken from patients diagnosed with emphysema or pneumothorax. The cells from randomly chosen LN were tested by multi-color flow cytometry. Separate portions of LN were snap-frozen and examined for the presence of cytokeratin positive cells (CK). Propensity for apoptosis, level of TCR zeta chain expression of T cells and the number and maturation status of dendritic cells were confronted with the presence of CK-positive cells.", "The presence of metastases correlated with the downregulation of TCR zeta, especially CD8(+) T cells. The most striking feature was the reduction in the number of myeloid CD11c(+) dendritic cells in the LN of patients with LN metastases. This could be a reflection of the immunodeficient state observed in lung cancer patients. Even in the absence of metastases in the regional LN, the same type of changes in the LN microenvironment were observed in those LN located nearer the primary tumor." ], "LABELS": [ "BACKGROUND", "METHODS", "RESULTS" ], "MESHES": [ "Aged", "Carcinoma, Non-Small-Cell Lung", "Cell Separation", "Dendritic Cells", "Female", "Flow Cytometry", "Humans", "Lung Neoplasms", "Lymph Nodes", "Lymphatic Metastasis", "Male", "Mediastinum", "Middle Aged", "Neoplasm Staging", "T-Lymphocyte Subsets", "T-Lymphocytes" ], "YEAR": "2009", "reasoning_required_pred": "yes", "reasoning_free_pred": "yes", "final_decision": "yes", "LONG_ANSWER": "The preliminary results of this study suggest that this approach may be helpful as an independent tumor staging factor. It is also worth noting that part of the staging process could also be based on features describing the immune cells in the peripheral blood." }, "18378554": { "QUESTION": "Are wandering and physically nonaggressive agitation equivalent?", "CONTEXTS": [ "The authors examined equivalence of wandering and physically nonaggressive agitation (PNA) as concepts.", "A cross-sectional correlational design was used.", "Participants were recruited from 22 nursing homes and 6 assisted living facilities in two states.", "Ambulatory residents meeting DSM-IV criteria for dementia (N = 181) were studied.", "Video-tapes for up to twelve 20-minute observations per participant were coded for wandering using an empirically derived taxonomy of ambulation patterns. Separate raters coded the same tapes for six PNA behaviors on the agitation behavior mapping instrument.", "Most participants (73.5%) wandered; all showed PNA behaviors. Factor analyses yielded an one-factor solution for wandering (explained variance = 43.66%) and a two-factor solution for PNA (explained variance = 53.45%). Overall wandering correlated significantly with PNA Factor 1 (df =179, r = 0.68, p<0.001) and Factor 2, but at a lower value (df = 179, r = 0.26, p<0.01)." ], "LABELS": [ "OBJECTIVE", "DESIGN", "SETTING", "PARTICIPANTS", "MEASUREMENTS", "RESULTS" ], "MESHES": [ "Aged", "Assisted Living Facilities", "Cross-Sectional Studies", "Homes for the Aged", "Humans", "Motor Activity", "Nursing Homes", "Patient Selection", "Psychomotor Agitation", "Videotape Recording", "Walking" ], "YEAR": "2008", "reasoning_required_pred": "no", "reasoning_free_pred": "no", "final_decision": "no", "LONG_ANSWER": "Findings depict wandering and PNA as overlapping, but nonequivalent phenomena. Evidence supporting construct validity of wandering was more robust than that for PNA. Results have implications for accuracy in scientific and clinical detection and labeling of wandering and agitation." }, "17306983": { "QUESTION": "Is size-reducing ascending aortoplasty with external reinforcement an option in modern aortic surgery?", "CONTEXTS": [ "Enlargement of the ascending aorta is often combined with valvular, coronary, or other cardiac diseases. Reduction aortoplasty can be an optional therapy; however, indications regarding the diameter of aorta, the history of dilatation (poststenosis, bicuspid aortic valve), or the intraoperative management (wall excision, reduction suture, external reinforcement) are not established.", "In a retrospective study between 1997 and 2005, we investigated 531 patients operated for aneurysm or ectasia of the ascending aorta (diameter: 45-76mm). Of these, in 50 patients, size-reducing ascending aortoplasty was performed. External reinforcement with a non-coated dacron prosthesis was added in order to stabilize the aortic wall.", "Aortoplasty was associated with aortic valve replacement in 47 cases (35 mechanical vs 12 biological), subvalvular myectomy in 29 cases, and CABG in 13 cases. The procedure was performed with low hospital mortality (2%) and a low postoperative morbidity. Computertomographic and echocardiographic diameters were significantly smaller after reduction (55.8+/-9mm down to 40.51+/-6.2mm (CT), p<0.002; 54.1+/-6.7mm preoperatively down to 38.7+/-7.1mm (echocardiography), p<0.002), with stable performance in long-term follow-up (mean follow-up time: 70 months)." ], "LABELS": [ "OBJECTIVE", "METHODS", "RESULTS" ], "MESHES": [ "Adult", "Aged", "Aged, 80 and over", "Aorta", "Aortic Aneurysm", "Aortic Diseases", "Cardiac Surgical Procedures", "Coronary Artery Bypass", "Dilatation, Pathologic", "Female", "Heart Valve Prosthesis Implantation", "Humans", "Male", "Middle Aged", "Postoperative Care", "Retrospective Studies", "Treatment Outcome" ], "YEAR": "2007", "reasoning_required_pred": "yes", "reasoning_free_pred": "yes", "final_decision": "yes", "LONG_ANSWER": "As demonstrated in this study, size reduction of the ascending aorta using aortoplasty with external reinforcement is a safe procedure with excellent long-term results. It is a therapeutic option in modern aortic surgery in patients with poststenotic dilatation of the aorta without impairment of the sinotubular junction of the aortic valve and root." }, "26879871": { "QUESTION": "Does depression diagnosis and antidepressant prescribing vary by location?", "CONTEXTS": [ "Studies have linked ethnic differences in depression rates with neighbourhood ethnic density although results have not been conclusive. We looked at this using a novel approach analysing whole population data covering just over one million GP patients in four London boroughs.", "Using a dataset of GP records for all patients registered in Lambeth, Hackney, Tower Hamlets and Newham in 2013 we investigated new diagnoses of depression and antidepressant use for: Indian, Pakistani, Bangladeshi, black Caribbean and black African patients. Neighbourhood effects were assessed independently of GP practice using a cross-classified multilevel model.", "Black and minority ethnic groups are up to four times less likely to be newly diagnosed with depression or prescribed antidepressants compared to white British patients. We found an inverse relationship between neighbourhood ethnic density and new depression diagnosis for some groups, where an increase of 10% own-ethnic density was associated with a statistically significant (p<0.05) reduced odds of depression for Pakistani [odds ratio (OR) 0.81, 95% confidence interval (CI) 0.70-0.93], Indian (OR 0.88, CI 0.81-0.95), African (OR 0.88, CI 0.78-0.99) and Bangladeshi (OR 0.94, CI 0.90-0.99) patients. Black Caribbean patients, however, showed the opposite effect (OR 1.26, CI 1.09-1.46). The results for antidepressant use were very similar although the corresponding effect for black Caribbeans was no longer statistically significant (p = 0.07)." ], "LABELS": [ "BACKGROUND", "METHOD", "RESULTS" ], "MESHES": [ "Adult", "African Continental Ancestry Group", "Antidepressive Agents", "Datasets as Topic", "Depressive Disorder", "Drug Prescriptions", "Ethnic Groups", "European Continental Ancestry Group", "Female", "Humans", "London", "Male", "Middle Aged", "Primary Health Care", "Regression Analysis", "Residence Characteristics", "Young Adult" ], "YEAR": "2016", "reasoning_required_pred": "maybe", "reasoning_free_pred": "yes", "final_decision": "yes", "LONG_ANSWER": "New depression diagnosis and antidepressant use was shown to be less likely in areas of higher own-ethnic density for some, but not all, ethnic groups." }, "24336869": { "QUESTION": "Can routinely collected ambulance data about assaults contribute to reduction in community violence?", "CONTEXTS": [ "The 'law of spatiotemporal concentrations of events' introduced major preventative shifts in policing communities. 'Hotspots' are at the forefront of these developments yet somewhat understudied in emergency medicine. Furthermore, little is known about interagency 'data-crossover', despite some developments through the Cardiff Model. Can police-ED interagency data-sharing be used to reduce community-violence using a hotspots methodology?", "12-month (2012) descriptive study and analysis of spatiotemporal clusters of police and emergency calls for service using hotspots methodology and assessing the degree of incident overlap. 3775 violent crime incidents and 775 assault incidents analysed using spatiotemporal clustering with k-means++ algorithm and Spearman's rho.", "Spatiotemporal location of calls for services to the police and the ambulance service are equally highly concentrated in a small number of geographical areas, primarily within intra-agency hotspots (33% and 53%, respectively) but across agencies' hotspots as well (25% and 15%, respectively). Datasets are statistically correlated with one another at the 0.57 and 0.34 levels, with 50% overlap when adjusted for the number of hotspots. At least one in every two police hotspots does not have an ambulance hotspot overlapping with it, suggesting half of assault spatiotemporal concentrations are unknown to the police. Data further suggest that more severely injured patients, as estimated by transfer to hospital, tend to be injured in the places with the highest number of police-recorded crimes." ], "LABELS": [ "BACKGROUND", "METHODS", "RESULTS" ], "MESHES": [ "Algorithms", "Ambulances", "Cluster Analysis", "Cooperative Behavior", "Data Collection", "Emergency Service, Hospital", "England", "Humans", "Police", "Violence", "Wounds and Injuries" ], "YEAR": "2015", "reasoning_required_pred": "yes", "reasoning_free_pred": "maybe", "final_decision": "maybe", "LONG_ANSWER": "A hotspots approach to sharing data circumvents the problem of disclosing person-identifiable data between different agencies. Practically, at least half of ambulance hotspots are unknown to the police; if causal, it suggests that data sharing leads to both reduced community violence by way of prevention (such as through anticipatory patrols or problem-oriented policing), particularly of more severe assaults, and improved efficiency of resource deployment." }, "16962519": { "QUESTION": "Volume change of uterine myomas during pregnancy: do myomas really grow?", "CONTEXTS": [ "To estimate changes in uterine myoma volume during pregnancy.", "Review of departmental electronic perinatal database and medical records. Canadian Task Force Classification II-3.", "Obstetrical ultrasound unit in an academic tertiary care center.", "One hundred-seven patients diagnosed with uterine myomas during pregnancy and who had two or more obstetrical ultrasounds in different periods of pregnancy.", "We analyzed the change in volume of uterine myomas between the first half of pregnancy (up until 19 weeks), third quarter (20-30 weeks), and last quarter (31 weeks to term). The volume of largest uterine myoma was calculated using the formula Volume (mm3)=Pi/6x(length mm)x(width mm)x(height mm).", "The mean age of the population was 31+/-6 years. Between the first and the second study periods, the percentage of uterine myomas that decreased in size was 55.1% (95% CI: 43-66), with a mean decrease in volume of 35%+/-4%; while the percentage of uterine myomas that enlarged was 44.9% (95% CI: 34-56), with a mean increase in volume of 69%+/-11%. Between the second and the third study periods, 75% (95% CI: 56-87) became smaller, with a mean decrease in volume of 30%+/-3%; while 25% (95% CI: 13-43) enlarged, with a mean increase in volume of 102%+/-62%." ], "LABELS": [ "STUDY OBJECTIVE", "DESIGN", "SETTING", "PATIENTS", "INTERVENTIONS", "MEASUREMENTS AND MAIN RESULTS" ], "MESHES": [ "Adult", "Female", "Humans", "Leiomyoma", "Longitudinal Studies", "Pregnancy", "Pregnancy Complications, Neoplastic", "ROC Curve", "Retrospective Studies", "Tumor Burden", "Ultrasonography, Prenatal", "Uterine Neoplasms" ], "YEAR": null, "reasoning_required_pred": "no", "reasoning_free_pred": "no", "final_decision": "no", "LONG_ANSWER": "Contrary to common belief, we found that uterine myomas commonly decrease in volume over the course of pregnancy." }, "17704864": { "QUESTION": "Is laparoscopic adrenalectomy safe and effective for adrenal masses larger than 7 cm?", "CONTEXTS": [ "Laparoscopic adrenalectomy (LA) has become the gold standard treatment for small (less than 6 cm) adrenal masses. However, the role of LA for large-volume (more than 6 cm) masses has not been well defined. Our aim was to evaluate, retrospectively, the outcome of LA for adrenal lesions larger than 7 cm.", "18 consecutive laparoscopic adrenalectomies were performed from 1996 to 2005 on patients with adrenal lesions larger than 7 cm.", "The mean tumor size was 8.3 cm (range 7-13 cm), the mean operative time was 137 min, the mean blood loss was 182 mL (range 100-550 mL), the rate of intraoperative complications was 16%, and in three cases we switched from laparoscopic procedure to open surgery." ], "LABELS": [ "BACKGROUND", "PATIENTS AND METHODS", "RESULTS" ], "MESHES": [ "Adolescent", "Adrenal Gland Neoplasms", "Adrenalectomy", "Adult", "Aged", "Aged, 80 and over", "Female", "Humans", "Laparoscopy", "Male", "Middle Aged" ], "YEAR": "2008", "reasoning_required_pred": "yes", "reasoning_free_pred": "yes", "final_decision": "yes", "LONG_ANSWER": "LA for adrenal masses larger than 7 cm is a safe and feasible technique, offering successful outcome in terms of intraoperative and postoperative morbidity, hospital stay and cosmesis for patients; it seems to replicate open surgical oncological principles demonstrating similar outcomes as survival rate and recurrence rate, when adrenal cortical carcinoma were treated. The main contraindication for this approach is the evidence, radiologically and intraoperatively, of local infiltration of periadrenal tissue." }, "18307476": { "QUESTION": "Upstream solutions: does the supplemental security income program reduce disability in the elderly?", "CONTEXTS": [ "The robust relationship between socioeconomic factors and health suggests that social and economic policies might substantially affect health, while other evidence suggests that medical care, the main focus of current health policy, may not be the primary determinant of population health. Income support policies are one promising avenue to improve population health. This study examines whether the federal cash transfer program to poor elderly, the Supplemental Security Income (SSI) program, affects old-age disability.", "This study uses the 1990 and 2000 censuses, employing state and year fixed-effect models, to test whether within-state changes in maximum SSI benefits over time lead to changes in disability among people aged sixty-five and older.", "Higher benefits are linked to lower disability rates. Among all single elderly individuals, 30 percent have mobility limitations, and an increase of $100 per month in the maximum SSI benefit caused the rate of mobility limitations to fall by 0.46 percentage points. The findings were robust to sensitivity analyses. First, analyses limited to those most likely to receive SSI produced larger effects, but analyses limited to those least likely to receive SSI produced no measurable effect. Second, varying the disability measure did not meaningfully alter the findings. Third, excluding the institutionalized, immigrants, individuals living in states with exceptionally large benefit changes, and individuals living in states with no SSI supplements did not change the substantive conclusions. Fourth, Medicaid did not confound the effects. Finally, these results were robust for married individuals." ], "LABELS": [ "CONTEXT", "METHODS", "FINDINGS" ], "MESHES": [ "Activities of Daily Living", "Aged", "Aged, 80 and over", "Censuses", "Disabled Persons", "Female", "Humans", "Income", "Male", "Medicaid", "Models, Econometric", "Policy Making", "Social Security", "United States" ], "YEAR": "2008", "reasoning_required_pred": "yes", "reasoning_free_pred": "yes", "final_decision": "yes", "LONG_ANSWER": "Income support policy may be a significant new lever for improving population health, especially that of lower-income persons. Even though the findings are robust, further analyses are needed to confirm their reliability. Future research should examine a variety of different income support policies, as well as whether a broader range of social and economic policies affect health." }, "15151701": { "QUESTION": "Profiling quality of care: Is there a role for peer review?", "CONTEXTS": [ "We sought to develop a more reliable structured implicit chart review instrument for use in assessing the quality of care for chronic disease and to examine if ratings are more reliable for conditions in which the evidence base for practice is more developed.", "We conducted a reliability study in a cohort with patient records including both outpatient and inpatient care as the objects of measurement. We developed a structured implicit review instrument to assess the quality of care over one year of treatment. 12 reviewers conducted a total of 496 reviews of 70 patient records selected from 26 VA clinical sites in two regions of the country. Each patient had between one and four conditions specified as having a highly developed evidence base (diabetes and hypertension) or a less developed evidence base (chronic obstructive pulmonary disease or a collection of acute conditions). Multilevel analysis that accounts for the nested and cross-classified structure of the data was used to estimate the signal and noise components of the measurement of quality and the reliability of implicit review.", "For COPD and a collection of acute conditions the reliability of a single physician review was quite low (intra-class correlation = 0.16-0.26) but comparable to most previously published estimates for the use of this method in inpatient settings. However, for diabetes and hypertension the reliability is significantly higher at 0.46. The higher reliability is a result of the reviewers collectively being able to distinguish more differences in the quality of care between patients (p<0.007) and not due to less random noise or individual reviewer bias in the measurement. For these conditions the level of true quality (i.e. the rating of quality of care that would result from the full population of physician reviewers reviewing a record) varied from poor to good across patients." ], "LABELS": [ "BACKGROUND", "METHODS", "RESULTS" ], "MESHES": [ "Acute Disease", "Chronic Disease", "Cohort Studies", "Continuity of Patient Care", "Diabetes Mellitus", "Disease Management", "Evidence-Based Medicine", "Health Services Misuse", "Humans", "Hypertension", "Internal Medicine", "Los Angeles", "Medical Records", "Michigan", "Observer Variation", "Outcome and Process Assessment (Health Care)", "Peer Review, Health Care", "Primary Health Care", "Pulmonary Disease, Chronic Obstructive", "Quality Assurance, Health Care", "Veterans" ], "YEAR": "2004", "reasoning_required_pred": "maybe", "reasoning_free_pred": "yes", "final_decision": "yes", "LONG_ANSWER": "For conditions with a well-developed quality of care evidence base, such as hypertension and diabetes, a single structured implicit review to assess the quality of care over a period of time is moderately reliable. This method could be a reasonable complement or alternative to explicit indicator approaches for assessing and comparing quality of care. Structured implicit review, like explicit quality measures, must be used more cautiously for illnesses for which the evidence base is less well developed, such as COPD and acute, short-course illnesses." }, "26399179": { "QUESTION": "Eyelid-parotid metastasis: do we screen for coexisting masses?", "CONTEXTS": [ "To report three cases illustrating that it is not unusual for a primary eyelid tumour to metastasise to the parotid gland and vice versa.", "Two patients with malignant parotid tumours underwent radical parotidectomy and presented subsequently with eyelid lesions. Biopsy showed that both eyelid lesions were histologically similar to the primary parotid tumour. A third patient was noted to have ipsilateral upper eyelid and parotid gland tumours. Histology and immunocytochemistry were used to differentiate the primary tumour and the metastasis." ], "LABELS": [ "OBJECTIVE", "CASE REPORTS" ], "MESHES": [ "Adult", "Aged", "Breast Neoplasms", "Carcinoma, Squamous Cell", "Chemoradiotherapy", "Diagnosis, Differential", "Eyelid Neoplasms", "Female", "Humans", "Male", "Middle Aged", "Neoplasm Staging", "Parotid Neoplasms", "Reconstructive Surgical Procedures", "Surgical Procedures, Operative" ], "YEAR": "2015", "reasoning_required_pred": "yes", "reasoning_free_pred": "yes", "final_decision": "yes", "LONG_ANSWER": "These cases illustrate that tumours involving eyelids and parotid glands can present simultaneously or sequentially, and either of these structures could be the focus of primary or metastatic tumour. The important message for oculoplastic and parotid surgeons is to routinely assess both the periocular and parotid area when patients present with a mass in either structure." }, "21889895": { "QUESTION": "Will CT ordering practices change if we educate residents about the potential effects of radiation exposure?", "CONTEXTS": [ "The aim of this study was to determine if educating residents about the potential effects of radiation exposure from computed tomographic (CT) imaging alters ordering patterns. This study also explored whether referring physicians are interested in radiation education and was an initial effort to address their CT ordering behavior.", "Two to four months after a radiologist's lecture on the potential effects of radiation exposure related to CT scans, urology and orthopedic residents were surveyed regarding the number and types of CT scans they ordered, the use of alternative imaging modalities, and whether they used the lecture information to educate patients.", "Twenty-one resident lecture attendants completed the survey. The number of CT scans ordered after the lecture stayed constant for 90% (19 of 21) and decreased for 10% (two of 21). The types of CT scans ordered changed after the lecture for 14% (three of 21). Thirty-three percent (seven of 21) reported increases in alternative imaging after the lecture, including 24% (five of 21) reporting increases in magnetic resonance imaging and 19% (four of 21) reporting increases in ultrasound. Patients directed questions about radiation exposure to 57% (12 of 21); 38% (eight of 21) used the lecture information to educate patients. Referring physicians were interested in the topic, and afterward, other physician groups requested radiation education lectures." ], "LABELS": [ "RATIONALE AND OBJECTIVES", "MATERIALS AND METHODS", "RESULTS" ], "MESHES": [ "Academic Medical Centers", "Education, Medical, Graduate", "Humans", "Internship and Residency", "Patient Education as Topic", "Practice Patterns, Physicians'", "Radiation Dosage", "Radiation Protection", "Radiology", "Risk", "Tomography, X-Ray Computed" ], "YEAR": "2011", "reasoning_required_pred": "yes", "reasoning_free_pred": "no", "final_decision": "no", "LONG_ANSWER": "Most clinicians did not change their CT scan ordering after receiving education about radiation from a radiologist. Radiation education allowed clinicians to discuss CT benefits and risks with their patients and to choose appropriate CT protocols. Referring physician groups are interested in this topic, and radiologists should be encouraged to give radiation lectures to them." }, "21569408": { "QUESTION": "Does context matter for the relationship between deprivation and all-cause mortality?", "CONTEXTS": [ "A growing body of research emphasizes the importance of contextual factors on health outcomes. Using postcode sector data for Scotland (UK), this study tests the hypothesis of spatial heterogeneity in the relationship between area-level deprivation and mortality to determine if contextual differences in the West vs. the rest of Scotland influence this relationship. Research into health inequalities frequently fails to recognise spatial heterogeneity in the deprivation-health relationship, assuming that global relationships apply uniformly across geographical areas. In this study, exploratory spatial data analysis methods are used to assess local patterns in deprivation and mortality. Spatial regression models are then implemented to examine the relationship between deprivation and mortality more formally.", "The initial exploratory spatial data analysis reveals concentrations of high standardized mortality ratios (SMR) and deprivation (hotspots) in the West of Scotland and concentrations of low values (coldspots) for both variables in the rest of the country. The main spatial regression result is that deprivation is the only variable that is highly significantly correlated with all-cause mortality in all models. However, in contrast to the expected spatial heterogeneity in the deprivation-mortality relationship, this relation does not vary between regions in any of the models. This result is robust to a number of specifications, including weighting for population size, controlling for spatial autocorrelation and heteroskedasticity, assuming a non-linear relationship between mortality and socio-economic deprivation, separating the dependent variable into male and female SMRs, and distinguishing between West, North and Southeast regions. The rejection of the hypothesis of spatial heterogeneity in the relationship between socio-economic deprivation and mortality complements prior research on the stability of the deprivation-mortality relationship over time." ], "LABELS": [ "BACKGROUND", "RESULTS" ], "MESHES": [ "Adolescent", "Adult", "Aged", "Child", "Child, Preschool", "Demography", "Female", "Health Status Disparities", "Humans", "Infant", "Infant, Newborn", "Male", "Middle Aged", "Mortality", "Poverty Areas", "Residence Characteristics", "Scotland", "Socioeconomic Factors", "Young Adult" ], "YEAR": "2011", "reasoning_required_pred": "yes", "reasoning_free_pred": "no", "final_decision": "no", "LONG_ANSWER": "The homogeneity we found in the deprivation-mortality relationship across the regions of Scotland and the absence of a contextualized effect of region highlights the importance of taking a broader strategic policy that can combat the toxic impacts of socio-economic deprivation on health. Focusing on a few specific places (e.g. 15% of the poorest areas) to concentrate resources might be a good start but the impact of socio-economic deprivation on mortality is not restricted to a few places. A comprehensive strategy that can be sustained over time might be needed to interrupt the linkages between poverty and mortality." }, "23025584": { "QUESTION": "Does stress increase imitation of drinking behavior?", "CONTEXTS": [ "That alcohol consumption is strongly influenced by the drinking behavior of social company has been demonstrated in observational research. However, not everyone is equally vulnerable to other people's drinking, and it is important to unravel which factors underlie these individual differences. This study focuses on the role of psychosocial stress in attempting to explain individual differences in the propensity to imitate alcohol consumption.", "With a 2 (confederate's drinking condition: alcohol vs. soda) \u00d7 2 (participant's stress condition: stress vs. no stress) experimental design, we tested whether the tendency to imitate other people's drinking was related to participants' induced stress levels. The young male adults (N = 106) were randomly assigned to each of the conditions. In each session, directly after the stress or no-stress period, confederates and participants entered a bar laboratory where we observed their drinking behavior. Prior to entering the session, confederates were instructed to drink alcohol or soda.", "Participants in both stress and no-stress conditions consumed substantially more alcohol when confederates drank alcohol than when they drank soda. There was no difference in alcohol consumed between stress and no-stress conditions. No moderating effect of stress on the tendency to drink along with peers was found." ], "LABELS": [ "BACKGROUND", "METHODS", "RESULTS" ], "MESHES": [ "Adolescent", "Adult", "Alcohol Drinking", "Ethanol", "Humans", "Imitative Behavior", "Male", "Social Behavior", "Stress, Psychological", "Young Adult" ], "YEAR": "2013", "reasoning_required_pred": "no", "reasoning_free_pred": "no", "final_decision": "no", "LONG_ANSWER": "Generally, it appears that among young male adults, imitation of alcohol consumption is a robust phenomenon not dependent on individual stress levels." }, "23587089": { "QUESTION": "School food policy at Dutch primary schools: room for improvement?", "CONTEXTS": [ "Schools can play an important role in the prevention of obesity, e.g. by providing an environment that stimulates healthy eating habits and by developing a food policy to provide such an environment. The effectiveness of a school food policy is affected by the content of the policy, its implementation and its support by parents, teachers and principals. The aim of this study is to detect opportunities to improve the school food policy and/or implementation at Dutch primary schools. Therefore, this study explores the school food policy and investigates schools' (teachers and principals) and parents' opinion on the school food policy.", "Data on the schools' perspective of the food policy was collected from principals and teachers by means of semi-structured interviews. In total 74 principals and 72 teachers from 83 Dutch primary schools were interviewed. Data on parental perceptions about the school food policy were based on a cross-sectional survey among 1,429 parents from the same schools.", "Most principals (87.1%) reported that their school had a written food policy; however in most cases the rules were not clearly defined. Most of the principals (87.8%) believed that their school paid sufficient attention to nutrition and health. Teachers and principals felt that parents were primarily responsible to encourage healthy eating habits among children, while 49.8% of the parents believed that it is also a responsibility of the school to foster healthy eating habits among children. Most parents reported that they appreciated the school food policy and comply with the food rules. Parents' opinion on the enforcement of the school food policy varied: 28.1% believed that the school should enforce the policy more strongly, 32.1% was satisfied, and 39.8% had no opinion on this topic." ], "LABELS": [ "BACKGROUND", "METHODS", "RESULTS" ], "MESHES": [ "Attitude to Health", "Cross-Sectional Studies", "Faculty", "Female", "Humans", "Male", "Netherlands", "Nutrition Policy", "Obesity", "Parents", "Qualitative Research", "Schools", "Surveys and Questionnaires" ], "YEAR": "2013", "reasoning_required_pred": "yes", "reasoning_free_pred": "yes", "final_decision": "yes", "LONG_ANSWER": "Dutch primary schools could play a more important role in fostering healthy eating habits among children. The school food policy could be improved by clearly formulating food rules, simplifying supervision of the food rules, and defining how to enforce the food rules. In addition, the school food policy will only influence children's dietary behaviour if both the school and the parents support the policy." }, "15539888": { "QUESTION": "Is the atopy patch test with house dust mites specific for atopic dermatitis?", "CONTEXTS": [ "The atopy patch test (APT), namely the patch test with aeroallergens, is regarded as specific for patients with atopic dermatitis (AD), but small numbers of positive APT were reported in the past also in atopic subjects without dermatitis and in healthy persons.", "The aim of this study was to evaluate the response to the APT with house dust mites (HDM) in subjects nonaffected by AD and to compare the outcomes observed in these cases with those pointed out in AD patients, evaluating also the differences between two allergen extracts manufactured at different purifications and concentrations.", "Forty-seven atopic subjects without eczema (AWE), 33 nonatopic (NA) subjects and 77 adult AD patients were patch tested with an extract of purified bodies of HDM at 20% and with another extract of whole bodies of HDM at 30%, the latter corresponding to 300 microg/g of Der p 1. The reproducibility of APT was also tested in 8 AD patients, in 37 AWE subjects and in 19 NA subjects.", "Positive responses with extract at 20% were observed in 29 (37.7%) AD, in 5 (10.6%) AWE and in 4 (12.1%) NA subjects. The APT with HDM at 30% was positive in 32 (41.6%) AD, 9 (19.1%) AWE and 4 (12.1%) NA persons. The rates of positivity and the intensity scores of responses were significantly different between AD and non-AD subjects (p<0.01). The reproducibility of the APT in the three groups was satisfactory." ], "LABELS": [ "BACKGROUND", "OBJECTIVE", "METHODS", "RESULTS" ], "MESHES": [ "Adolescent", "Adult", "Allergens", "Animals", "Case-Control Studies", "Dermatitis, Atopic", "Female", "Humans", "Male", "Middle Aged", "Mites", "Patch Tests", "Reference Values", "Risk Assessment", "Sensitivity and Specificity", "Severity of Illness Index" ], "YEAR": "2004", "reasoning_required_pred": "yes", "reasoning_free_pred": "yes", "final_decision": "yes", "LONG_ANSWER": "These observations lead to conclude that the APT with HDM is positive also in non-AD subjects but it is probably more specific for AD." }, "23096188": { "QUESTION": "Juvenile osteochondritis dissecans: is it a growth disturbance of the secondary physis of the epiphysis?", "CONTEXTS": [ "The primary physis is responsible for longitudinal bone growth. Similarly, epiphysial growth relies on endochondral ossification from the circumferential secondary physeal [corrected]. injury can result in disruption of normal ossification. The cause of juvenile osteochondritis dissecans (OCD) remains elusive. We hypothesized that juvenile OCD results from an insult affecting endochondral ossification from the secondary physis. The purpose of our study was to evaluate the MRI appearance of the distal femoral epiphysis-particularly the secondary physis-of children with juvenile OCD and to compare these findings with the MRI findings of unaffected children.", "Knee MRI examinations of 30 children (age range, 8 years 8 months to 13 years 4 months) with OCD and 30 matched control patients were evaluated for skeletal maturity; location of the OCD lesion, if present; secondary physeal [corrected] continuity; overlying chondroepiphysial integrity, contour, and width; signal intensity of subchondral bone; and secondary physeal [corrected]conspicuity. Variables were compared using chi-square tests.", "All children were skeletally immature. Condylar lesions were medial in 24 knees and lateral in six knees. All were in the middle one third, posterior one third, or middle and posterior thirds in the sagittal plane. The majority of lesions spanned the intercondylar and middle one third of the femoral condyle in the coronal plane (73%). There was a significant difference between secondary physeal [corrected] disruption in juvenile OCD condyles compared with unaffected condyles (p<0.001) and control condyles (p<0.001). Compared with unaffected and control condyles, the OCD group showed chondroepiphysial widening (p<0.001) and subchondral bone edema (p<0.001) on MRI. Neither chondroepiphysial integrity nor chondroepiphysial contour was significantly different between groups (p = 0.21, p = 0.31, respectively)." ], "LABELS": [ "OBJECTIVE", "MATERIALS AND METHODS", "RESULTS" ], "MESHES": [ "Adolescent", "Case-Control Studies", "Chi-Square Distribution", "Child", "Epiphyses", "Female", "Humans", "Knee Joint", "Magnetic Resonance Imaging", "Male", "Osteochondritis Dissecans" ], "YEAR": "2012", "reasoning_required_pred": "maybe", "reasoning_free_pred": "yes", "final_decision": "yes", "LONG_ANSWER": "MRI of children with OCD consistently showed secondary physis disruption, overlying chondroepiphysial widening, and subchondral bone edema. We suggest that disruption of normal endochondral ossification may be associated with juvenile OCD." }, "21881325": { "QUESTION": "Do preoperative statins reduce atrial fibrillation after coronary artery bypass grafting?", "CONTEXTS": [ "Recent studies have demonstrated that statins have pleiotropic effects, including anti-inflammatory effects and atrial fibrillation (AF) preventive effects. The objective of this study was to assess the efficacy of preoperative statin therapy in preventing AF after coronary artery bypass grafting (CABG).", "221 patients underwent CABG in our hospital from 2004 to 2007. 14 patients with preoperative AF and 4 patients with concomitant valve surgery were excluded from this study. Patients were divided into two groups to examine the influence of statins: those with preoperative statin therapy (Statin group, n = 77) and those without it (Non-statin group, n = 126). In addition, patients were divided into two groups to determine the independent predictors for postoperative AF: those with postoperative AF (AF group, n = 54) and those without it (Non-AF group, n = 149). Patient data were collected and analyzed retrospectively.", "The overall incidence of postoperative AF was 26%. Postoperative AF was significantly lower in the Statin group compared with the Non-statin group (16% versus 33%, p = 0.005). Multivariate analysis demonstrated that independent predictors of AF development after CABG were preoperative statin therapy (odds ratio [OR]0.327, 95% confidence interval [CI] 0.107 to 0.998, p = 0.05) and age (OR 1.058, 95% CI 1.004 to 1.116, p = 0.035)." ], "LABELS": [ "OBJECTIVE", "METHODS", "RESULTS" ], "MESHES": [ "Aged", "Analysis of Variance", "Atrial Fibrillation", "Chi-Square Distribution", "Coronary Artery Bypass", "Drug Administration Schedule", "Female", "Humans", "Hydroxymethylglutaryl-CoA Reductase Inhibitors", "Japan", "Male", "Middle Aged", "Odds Ratio", "Retrospective Studies", "Risk Assessment", "Risk Factors", "Time Factors", "Treatment Outcome" ], "YEAR": "2011", "reasoning_required_pred": "yes", "reasoning_free_pred": "yes", "final_decision": "yes", "LONG_ANSWER": "Our study indicated that preoperative statin therapy seems to reduce AF development after CABG." }, "19542542": { "QUESTION": "Is there a uniform basal endometrial gene expression profile during the implantation window in women who became pregnant in a subsequent ICSI cycle?", "CONTEXTS": [ "To understand which genes are really involved in the implantation process, we planned to study the gene basal expression profile during the window of implantation (WOI) of patients who became pregnant in a subsequent ICSI cycle.", "Women attending their first ICSI cycle at ANDROS Day Surgery for severe male factor infertility were included in the study. An endometrial biopsy was performed during the WOI, in one of the last two cycles before the ICSI cycle. Forty-seven selected gene profiles were analyzed using Low Density Array technology. Only biopsies from women who subsequently became pregnant were evaluated, to exclude any bias regarding embryo viability and embryo transfer difficulties.", "Fifteen patients were included in the analysis as they became pregnant after ICSI procedure. Four of 47 selected genes were excluded from the analysis. Of the 43 genes analyzed, only 6 genes (VEGFA, PLA2G2A, ALPL, LIF, NNMT and STC1) showed a statistically uniform expression among patients who subsequently became pregnant. For all the other genes analyzed there were considerable differences in their expression levels amongst women who subsequently became pregnant." ], "LABELS": [ "BACKGROUND", "METHODS", "RESULTS" ], "MESHES": [ "Adult", "Embryo Implantation", "Endometrium", "Female", "Gene Expression Profiling", "Humans", "Oligonucleotide Array Sequence Analysis", "Pregnancy", "Sperm Injections, Intracytoplasmic" ], "YEAR": "2009", "reasoning_required_pred": "yes", "reasoning_free_pred": "maybe", "final_decision": "maybe", "LONG_ANSWER": "Our data suggest that very few genes, which change their expression level during the WOI, show a quantitative homogeneous expression in endometrially-receptive patients. In conclusion, in this study only six genes showed a homogeneous expression, and are probably involved in embryo implantation mechanisms." }, "16872243": { "QUESTION": "Can decisional algorithms replace global introspection in the individual causality assessment of spontaneously reported ADRs?", "CONTEXTS": [ "In this study, an expert panel assessed causality of adverse reports by using the WHO global introspection (GI) method. The same reports were independently assessed using 15 published algorithms. The causality assessment level 'possible' was considered the lower limit for a report to be considered to be drug related. For a given algorithm, sensitivity was determined by the proportion of reports simultaneously classified as drug related by the algorithm and the GI method. Specificity was measured as the proportion of reports simultaneously considered non-drug related. The analysis was performed for the total sample and within serious or unexpected events.", "Five hundred adverse reports were studied. Algorithms presented high rates of sensitivity (average of 93%, positive predictive value of 89%) and low rates of specificity (average of 7%, negative predictive value of 31%)." ], "LABELS": [ "METHOD", "RESULTS" ], "MESHES": [ "Adverse Drug Reaction Reporting Systems", "Algorithms", "Decision Support Techniques", "Drug-Related Side Effects and Adverse Reactions", "Evaluation Studies as Topic", "Gastrointestinal Diseases", "Humans", "Reproducibility of Results", "Risk Factors", "Skin Diseases", "World Health Organization" ], "YEAR": "2006", "reasoning_required_pred": "no", "reasoning_free_pred": "no", "final_decision": "no", "LONG_ANSWER": "Decisional algorithms are sensitive methods for the detection of ADRs, but they present poor specificity. A reference method was not identified. Algorithms do not replace GI and are not definite alternatives in the individual causality assessment of suspected ADRs." }, "15141797": { "QUESTION": "Can we rely on arterial line sampling in performing activated plasma thromboplastin time after cardiac surgery?", "CONTEXTS": [ "Arterial catheters are routinely used to sample blood for clotting studies in most cardiothoracic intensive care units. The clotting profile in surgical bleeding after cardiac surgery influences further management. Aspiration and discard of a certain amount of blood from the line, prior to sampling, are assumed to clear heparin contamination. We have investigated this assumption through analysis of the clotting profile by simultaneous arterial line and peripheral venous samples.", "The morning following cardiac surgery, simultaneous arterial line and peripheral venous blood samples were taken for activated plasma thromboplastin time (APTT) ratio and international normalized ratio (INR) in 49 randomly selected patients. Also, a thromboelastogram analysis (TEG) (n = 7) was made. A survey of 22 UK cardiothoracic intensive care units was carried out to determine the practice for the withdrawal of blood for clotting studies.", "The median arterial APTT ratio value was 1.32 +/- 0.52 as compared to the median peripheral APTT ratio value which was 1.1 +/- 0.24 (P<0.001). INR values were statistically similar by both routes. Heparin contamination was confirmed by TEG which revealed that the R-value for arterial catheter blood samples without heparinase in the cup was higher (406.00 +/- 64.44 s) compared with the value for arterial samples with heparinase in the cup (318.28 +/- 47.26s, P<0.05). The survey of 22 UK cardiothoracic intensive care units showed that heparinized arterial lines were by far the commonest ports used for blood withdrawal for the measurement of APTT ratio results." ], "LABELS": [ "BACKGROUND AND OBJECTIVE", "METHODS", "RESULTS" ], "MESHES": [ "Blood Coagulation Tests", "Blood Specimen Collection", "Cardiac Surgical Procedures", "Catheterization, Peripheral", "Catheters, Indwelling", "Data Collection", "Heparin", "Humans", "Intensive Care Units", "International Normalized Ratio", "Partial Thromboplastin Time", "Thrombelastography", "United Kingdom" ], "YEAR": "2004", "reasoning_required_pred": "yes", "reasoning_free_pred": "no", "final_decision": "no", "LONG_ANSWER": "Samples withdrawn from heparinized arterial lines cannot be relied upon for APTT ratio results." }, "25394614": { "QUESTION": "Does timing of initial surfactant treatment make a difference in rates of chronic lung disease or mortality in premature infants?", "CONTEXTS": [ "To compare two treatment strategies in preterm infants with or at risk of respiratory distress syndrome: early surfactant administration (within one hour of birth) versus late surfactant administration, in a geographically defined population.", "The primary outcome was chronic lung disease (CLD) and mortality before/at 36 weeks. Secondary outcomes included: duration of mechanical ventilation and continuous positive airway pressure (CPAP), post-natal steroids for CLD and major neonatal morbidities.", "Premature infants born at 22-32 weeks' gestation between January 2006 and December 2009.", "Ten neonatal intensive care units (NICUs) in New South Wales (NSW) and Australian Capital Territory (ACT), Australia.", "Retrospective analysis of prospectively collected data from the regional NICU database in NSW and ACT.", "Of the 2170 infants who received surfactant, 1182 (54.5%) and 988 (45.5%) received early and late surfactant, respectively. The early surfactant group was less mature (27.1\u2009\u00b1\u20092.1 versus 29.4\u2009\u00b1\u20092.1 weeks) and had more CLD and mortality (40.2% versus 20.0%). The multivariable analysis showed early surfactant to be associated with less duration of ventilation, longer duration of CPAP and longer hospital stay but had little or no impact on CLD/mortality." ], "LABELS": [ "OBJECTIVE", "OUTCOME", "SUBJECTS", "SETTING", "DESIGN", "RESULTS" ], "MESHES": [ "Australian Capital Territory", "Chronic Disease", "Female", "Humans", "Infant, Newborn", "Male", "New South Wales", "Pulmonary Surfactants", "Respiratory Distress Syndrome, Newborn", "Treatment Outcome" ], "YEAR": "2016", "reasoning_required_pred": "yes", "reasoning_free_pred": "maybe", "final_decision": "maybe", "LONG_ANSWER": "Early surfactant administration is associated with shorter duration of ventilation but does not appear to be significantly protective against CLD/mortality among premature infants. This may support the growing evidence for consideration of CPAP as an alternative to routine intubation and early surfactant administration. Further investigation from large randomized clinical trials is warranted to confirm these results." }, "10381996": { "QUESTION": "Clinician assessment for acute chest syndrome in febrile patients with sickle cell disease: is it accurate enough?", "CONTEXTS": [ "To determine whether the use of empiric chest radiography (CXR) is of significant value in detecting clinically unsuspected acute chest syndrome (ACS) in febrile patients with sickle cell disease (SCD).", "Patients with SCD presenting to the emergency department and hematology clinic with temperature greater than or equal to 38 degrees C were prospectively evaluated using a physician-completed questionnaire. The questionnaire included inquiries into the patient's physical signs and symptoms and the physician's clinical impression for the presence of ACS. The questionnaire was completed before obtaining CXR results in all patients.", "Seventy-three patients with SCD with 96 febrile events were evaluated over a 1-year period. Twenty-four percent (23/96) of the patients had CXR evidence of ACS. On the basis of the questionnaire data, 61% (14/23) of ACS cases were not clinically suspected by the evaluating physician before obtaining CXR. Comparing the patients with and without ACS revealed that, with the exception of splinting (4/23 [17%] versus 0/73 [0%]), no symptom or physical examination finding helped to identify which patients had ACS. Fifty-seven percent of patients with ACS had completely normal findings on physical examination. The presentation of patients with clinically detected versus clinically unsuspected ACS also did not differ significantly. Length of hospitalization, oxygen use, and need for transfusion were the same in both the unsuspected and detected ACS groups. Overall physician sensitivity for predicting ACS was only 39%, and diagnostic accuracy did not improve significantly with increasing levels of pediatric training." ], "LABELS": [ "STUDY OBJECTIVE", "METHODS", "RESULTS" ], "MESHES": [ "Acute Disease", "Adolescent", "Anemia, Sickle Cell", "Blood Transfusion", "Child", "Child, Preschool", "Diagnosis, Differential", "Emergency Treatment", "Female", "Fever", "Humans", "Infant", "Length of Stay", "Male", "Oxygen Inhalation Therapy", "Physical Examination", "Pneumonia", "Prospective Studies", "Radiography", "Reproducibility of Results", "Single-Blind Method", "Surveys and Questionnaires", "Syndrome" ], "YEAR": "1999", "reasoning_required_pred": "no", "reasoning_free_pred": "no", "final_decision": "no", "LONG_ANSWER": "ACS is common in patients with SCD who present with fever and was grossly underestimated by evaluating physicians. History and physical examination appear to be of little value in defining which febrile patients require CXR. In view of the mortality and morbidity associated with ACS, empiric CXR should be considered when evaluating a febrile patient with SCD." }, "26134053": { "QUESTION": "Outcome Feedback within Emergency Medicine Training Programs: An Opportunity to Apply the Theory of Deliberate Practice?", "CONTEXTS": [ "Outcome feedback is the process of learning patient outcomes after their care within the emergency department. We conducted a national survey of Canadian Royal College emergency medicine (EM) residents and program directors to determine the extent to which active outcome feedback and follow-up occurred. We also compared the perceived educational value of outcome feedback between residents and program directors.", "We distributed surveys to all Royal College-accredited adult and pediatric EM training programs using a modified Dillman method. We analyzed the data using student's t-test for continuous variables and Fisher's exact test for categorical variables.", "We received 210 completed surveys from 260 eligible residents (80.8%) and 21 of 24 program directors (87.5%) (overall 81.3%). Mandatory active outcome feedback was not present in any EM training program for admitted or discharged patients (0/21). Follow-up was performed electively by 89.4% of residents for patients admitted to the hospital, and by 44.2% of residents for patients discharged home. A majority of residents (76.9%) believed that patient follow-up should be mandatory compared to 42.9% of program directors (p=0.002). The perceived educational value of outcome feedback was 5.8/7 for residents and 5.1/7 for program directors (difference 0.7; p=0.002) based on a seven-point Likert scale (1=not important; 7=very important)." ], "LABELS": [ "OBJECTIVES", "METHODS", "RESULTS" ], "MESHES": [ "Adult", "Canada", "Child", "Clinical Competence", "Curriculum", "Emergency Medicine", "Female", "Humans", "Internship and Residency", "Male", "Models, Educational", "Surveys and Questionnaires" ], "YEAR": "2015", "reasoning_required_pred": "yes", "reasoning_free_pred": "maybe", "final_decision": "maybe", "LONG_ANSWER": "While Canadian EM training programs do not mandate follow-up, it is performed electively by the majority of residents surveyed. Residents place a significantly greater educational value on outcome feedback than their program directors, and believe that follow-up should be a mandatory component of EM residencies." }, "25274085": { "QUESTION": "Can multiple SNP testing in BRCA2 and BRCA1 female carriers be used to improve risk prediction models in conjunction with clinical assessment?", "CONTEXTS": [ "Several single nucleotide polymorphisms (SNPs) at different loci have been associated with breast cancer susceptibility, accounting for around 10% of the familial component. Recent studies have found direct associations between specific SNPs and breast cancer in BRCA1/2 mutation carriers. Our aim was to determine whether validated susceptibility SNP scores improve the predictive ability of risk models in comparison/conjunction to other clinical/demographic information.", "Female BRCA1/2 carriers were identified from the Manchester genetic database, and included in the study regardless of breast cancer status or age. DNA was extracted from blood samples provided by these women and used for gene and SNP profiling. Estimates of survival were examined with Kaplan-Meier curves. Multivariable Cox proportional hazards models were fit in the separate BRCA datasets and in menopausal stages screening different combinations of clinical/demographic/genetic variables. Nonlinear random survival forests were also fit to identify relevant interactions. Models were compared using Harrell's concordance index (1 - c-index).", "548 female BRCA1 mutation carriers and 523 BRCA2 carriers were identified from the database. Median Kaplan-Meier estimate of survival was 46.0 years (44.9-48.1) for BRCA1 carriers and 48.9 (47.3-50.4) for BRCA2. By fitting Cox models and random survival forests, including both a genetic SNP score and clinical/demographic variables, average 1 - c-index values were 0.221 (st.dev. 0.019) for BRCA1 carriers and 0.215 (st.dev. 0.018) for BRCA2 carriers." ], "LABELS": [ "BACKGROUND", "METHODS", "RESULTS" ], "MESHES": [ "BRCA1 Protein", "BRCA2 Protein", "Breast Neoplasms", "Female", "Genetic Testing", "Heterozygote", "Humans", "Middle Aged", "Polymorphism, Single Nucleotide", "Risk Assessment", "Survival Analysis" ], "YEAR": "2014", "reasoning_required_pred": "yes", "reasoning_free_pred": "no", "final_decision": "no", "LONG_ANSWER": "Random survival forests did not yield higher performance compared to Cox proportional hazards. We found improvement in prediction performance when coupling the genetic SNP score with clinical/demographic markers, which warrants further investigation." }, "12098035": { "QUESTION": "Does a special interest in laparoscopy affect the treatment of acute cholecystitis?", "CONTEXTS": [ "We tested the hypothesis that the treatment of patients with acute cholecystitis (AC) would be improved under the care of laparoscopic specialists.", "The records of patients undergoing cholecystectomy for AC from 1 January 1996 to 31 December 1998 were reviewed retrospectively. Of 170 patients, 48 were cared for by three laparoscopic specialists (LS group), whereas 122 were treated by nine general surgeons who perform only laparoscopic cholecystectomy (LC) (GS group). The rates of successful LC, complications, and length of hospital stay were compared. Multivariate analysis was used to control for baseline differences.", "The patients in the GS group were older (median age, 63 vs 53 years; p = 0.01). In all, 31 LS patients (65%), as compared with 44 GS patients (36%), had successful laparoscopic treatment (p = 0.001). The operating time was the same (median, 70 min). The proportion of patients with postoperative complications was similar in the two groups (37% in the GS vs 31% in the LS group; p = 0.6). The median postoperative hospital stay (3 vs 5 days; p<0.01) was shorter in the LS group. On logistic regression analysis, significant predictors of a successful laparoscopic operation included LS group (p<0.01) and age (p = 0). Predictors of prolonged length of hospital stay were age (p<0.01) and comorbidity score (p<0.01), with LS group status not a significant factor (p = 0.21)." ], "LABELS": [ "BACKGROUND", "METHODS", "RESULTS" ], "MESHES": [ "Acute Disease", "Adult", "Age Factors", "Aged", "Aged, 80 and over", "Cholecystectomy, Laparoscopic", "Cholecystitis", "Humans", "Intraoperative Complications", "Laparoscopy", "Length of Stay", "Logistic Models", "Middle Aged", "Multivariate Analysis", "Postoperative Complications", "Predictive Value of Tests", "Retrospective Studies", "Risk Factors", "Time Factors", "Treatment Outcome", "Workload" ], "YEAR": "2002", "reasoning_required_pred": "yes", "reasoning_free_pred": "yes", "final_decision": "yes", "LONG_ANSWER": "Patients with AC are more likely to undergo successful LC if cared for by a surgeon with an interest in laparoscopy. However, length of hospital stay is influenced more by patient factors in a multivariate model." }, "12963175": { "QUESTION": "Can progression of valvar aortic stenosis be predicted accurately?", "CONTEXTS": [ "It was the aim of the present study to elaborate criteria for the assessment of rapid hemodynamic progression of valvar aortic stenosis. These criteria are of special importance when cardiac surgery is indicated for other reasons but the established criteria for aortic valve replacement are not yet fulfilled. Such aspects of therapeutic planing were mostly disregarded in the past so that patients had to undergo cardiac reoperation within a few years.", "Hemodynamic, echocardiographic, and clinical data of 169 men and 88 women with aortic stenosis, aged 55.2 +/- 15.7 years at their first and 63.4 +/- 15.6 years at their second cardiac catheterization, were analyzed.", "The progression rate of aortic valve obstruction was found to be dependent on the degree of valvar calcification ([VC] scoring 0 to III) and to be exponentially correlated with the aortic valve opening area (AVA) at initial catheterization. Neither age nor sex of the patient nor etiology of the valvar obstruction significantly influence the progression of aortic stenosis. If AVA decreases below 0.75 cm(2) with a present degree of VC = 0, or AVA of 0.8 with VC of I, AVA of 0.9 with VC of II, or AVA of 1.0 with VC of III, it is probable that aortic stenosis will have to be operated upon in the following years." ], "LABELS": [ "BACKGROUND", "METHODS", "RESULTS" ], "MESHES": [ "Adolescent", "Adult", "Aged", "Aged, 80 and over", "Aortic Valve Stenosis", "Child", "Disease Progression", "Female", "Follow-Up Studies", "Humans", "Male", "Middle Aged", "Predictive Value of Tests", "Reproducibility of Results" ], "YEAR": "2003", "reasoning_required_pred": "yes", "reasoning_free_pred": "yes", "final_decision": "yes", "LONG_ANSWER": "The present data indicate that for clinical purposes and planning of valvar surgery the progression of asymptomatic aortic stenosis can be sufficiently predicted by the present aortic valve opening area and the degree of valvar calcification." }, "21865668": { "QUESTION": "Does automatic transmission improve driving behavior in older drivers?", "CONTEXTS": [ "Most older drivers continue to drive as they age. To maintain safe and independent transport, mobility is important for all individuals, but especially for older drivers.", "The objective of this study was to investigate whether automatic transmission, compared with manual transmission, may improve the driving behavior of older drivers.", "In total, 31 older drivers (mean age 75.2 years) and 32 younger drivers - used as a control group (mean age 39.2 years) - were assessed twice on the same fixed route; once in a car with manual transmission and once in a car with automatic transmission. The cars were otherwise identical. The driving behavior was assessed with the Ryd On-Road Assessment driving protocol. Time to completion of left turns (right-hand side driving) and the impact of a distraction task were measured.", "The older group had more driving errors than the younger group, in both the manual and the automatic transmission car. However, and contrary to the younger drivers, automatic transmission improved the older participants' driving behavior as demonstrated by safer speed adjustment in urban areas, greater maneuvering skills, safer lane position and driving in accordance with the speed regulations." ], "LABELS": [ "BACKGROUND", "OBJECTIVE", "METHOD", "RESULTS" ], "MESHES": [ "Adult", "Aged", "Aged, 80 and over", "Aging", "Automobile Driving", "Automobiles", "Humans", "Middle Aged", "Motor Skills", "Task Performance and Analysis" ], "YEAR": "2012", "reasoning_required_pred": "yes", "reasoning_free_pred": "yes", "final_decision": "yes", "LONG_ANSWER": "Switching to automatic transmission may be recommended for older drivers as a means to maintain safe driving and thereby the quality of their transport mobility." }, "16510651": { "QUESTION": "Birth characteristics and risk of low intellectual performance in early adulthood: are the associations confounded by socioeconomic factors in adolescence or familial effects?", "CONTEXTS": [ "In this study we investigated whether the association between measures of fetal growth restriction and intellectual performance was mediated by socioeconomic or familial factors.", "This was a population-based cohort study of 357,768 Swedish males born as singletons without congenital malformations between 1973 and 1981. The main outcome measure was intellectual performance at military conscription.", "Compared with men born with appropriate birth weight for gestational age, men born light for gestational age suffered an increased risk of low intellectual performance after adjustment for maternal and socioeconomic factors. The increase in risk of low intellectual performance related to a decrease in birth weight for gestational age was similar between families and within families. Men born short or with a small head circumference for gestational age were also at increased risk of low intellectual performance, both when adjusting for maternal and socioeconomic factors and within families." ], "LABELS": [ "OBJECTIVE", "METHODS", "RESULTS" ], "MESHES": [ "Adolescent", "Birth Weight", "Family Characteristics", "Female", "Fetal Growth Retardation", "Growth", "Humans", "Infant, Newborn", "Infant, Small for Gestational Age", "Intelligence", "Male", "Pregnancy", "Risk Factors", "Socioeconomic Factors", "Sweden" ], "YEAR": "2006", "reasoning_required_pred": "yes", "reasoning_free_pred": "no", "final_decision": "no", "LONG_ANSWER": "We found that all of the studied dimensions of restricted fetal growth are independently associated with increased risks of low intellectual performance and that these associations are only partly mediated by socioeconomic or familial factors." }, "24901580": { "QUESTION": "Is scintigraphy a guideline method in determining amputation levels in diabetic foot?", "CONTEXTS": [ "In this study, we aimed to evaluate the potential use of a 3-phase bone scintigraphy method to determine the level of amputation on treatment cost, morbidity and mortality, reamputation rates, and the duration of hospitalization in diabetic foot.", "Thirty patients who were admitted to our clinic between September 2008 and July 2009, with diabetic foot were included. All patients were evaluated according to age, gender, diabetes duration, 3-phase bone scintigraphy, Doppler ultrasound, amputation/reamputation levels, and hospitalization periods. Patients underwent 3-phase bone scintigraphy using technetium-99m methylene diphosphonate, and the most distal site of the region displaying perfusion during the perfusion and early blood flow phase was marked as the amputation level. Amputation level was determined by 3-phase bone scintigraphy, Doppler ultrasound, and inspection of the infection-free clear region during surgery.", "The amputation levels of the patients were as follows: finger in six (20%), ray amputation in five (16.6%), transmetatarsal in one (3.3%), Lisfranc in two (6.6%), Chopart in seven (23.3%), Syme in one (3.3%), below-the-knee in six (20%), above the knee in one (3.3%), knee disarticulation in one (3.3%), and two patients underwent amputation at other centers. After primary amputation, reamputation was performed on seven patients, and one patient was treated with debridement for wound site problems. No mortality was encountered during study." ], "LABELS": [ "BACKGROUND", "METHODS", "RESULTS" ], "MESHES": [ "Aged", "Aged, 80 and over", "Amputation", "Cohort Studies", "Decision Making", "Diabetic Foot", "Female", "Follow-Up Studies", "Humans", "Imaging, Three-Dimensional", "Male", "Middle Aged", "Practice Guidelines as Topic", "Preoperative Care", "Prospective Studies", "Radionuclide Imaging", "Risk Assessment", "Sensitivity and Specificity", "Severity of Illness Index", "Technetium Tc 99m Sestamibi", "Treatment Outcome", "Wound Healing" ], "YEAR": "2014", "reasoning_required_pred": "yes", "reasoning_free_pred": "yes", "final_decision": "yes", "LONG_ANSWER": "We conclude that 3-phase bone scintigraphy prior to surgery could be a useful method to determine the amputation level in a diabetic foot. We conclude that further, comparative, more comprehensive, long-term, and controlled studies are required." }, "19468282": { "QUESTION": "Is determination between complete and incomplete traumatic spinal cord injury clinically relevant?", "CONTEXTS": [ "Prospective multicenter longitudinal cohort study.", "To validate the prognostic value of the acute phase sacral sparing measurements with regard to chronic phase-independent ambulation in patients with traumatic spinal cord injury (SCI).", "European Multicenter Study of Human Spinal Cord Injury (EM-SCI).", "In 432 patients, acute phase (0-15 days) American Spinal Injury Association (ASIA)/International Spinal Cord Society neurological standard scale (AIS) grades, ASIA sacral sparing measurements, which are S4-5 light touch (LT), S4-5 pin prick (PP), anal sensation and voluntary anal contraction; and chronic phase (6 or 12 months) indoor mobility Spinal Cord Independence Measure (SCIM) measurements were analyzed. Calculations of positive and negative predictive values (PPV/NPV) as well as univariate and multivariate logistic regressions were performed in all four sacral sparing criteria. The area under the receiver-operating characteristic curve (AUC) ratios of all regression equations was calculated.", "To achieve independent ambulation 1-year post injury, a normal S4-5 PP score showed the best PPV (96.5%, P<0.001, 95% confidence interval (95% CI): 87.9-99.6). Best NPV was reported in the S4-5 LT score (91.7%, P<0.001, 95% CI: 81.6-97.2). The use of the combination of only voluntary anal contraction and the S4-5 LT and PP sensory scores (AUC: 0.906, P<0.001, 95% CI: 0.871-0.941) showed significantly better (P<0.001, 95% CI: 0.038-0.128) discriminating results in prognosticating 1-year independent ambulation than with the use of currently used distinction between complete and incomplete SCI (AUC: 0.823, P<0.001, 95% CI: 0.781-0.864)." ], "LABELS": [ "STUDY DESIGN", "OBJECTIVE", "SETTING", "METHODS", "RESULTS" ], "MESHES": [ "Adolescent", "Adult", "Aged", "Aged, 80 and over", "Anal Canal", "Cohort Studies", "Diagnosis, Differential", "Disability Evaluation", "Female", "Humans", "Male", "Middle Aged", "Neurologic Examination", "Paralysis", "Predictive Value of Tests", "Prospective Studies", "Reproducibility of Results", "Sacrum", "Severity of Illness Index", "Somatosensory Disorders", "Spinal Cord", "Spinal Cord Injuries", "Young Adult" ], "YEAR": "2009", "reasoning_required_pred": "no", "reasoning_free_pred": "maybe", "final_decision": "maybe", "LONG_ANSWER": "Out of the four sacral sparing criteria, the acute phase anal sensory score measurements do not contribute significantly to the prognosis of independent ambulation. The combination of the acute phase voluntary anal contraction and the S4-5 LT and PP scores, predicts significantly better chronic phase-independent ambulation outcomes than the currently used distinction between complete and incomplete SCI." }, "21481154": { "QUESTION": "Improvements in survival of gynaecological cancer in the Anglia region of England: are these an effect of centralisation of care and use of multidisciplinary management?", "CONTEXTS": [ "Our hypothesis is that the adoption of Department of Health (DH) guidance has led to an improvement in outcome in gynaecological cancer survival.", "In 1999 the DH in England introduced the Improving Outcomes in Gynaecological Cancer guidance, advising case management by multidisciplinary teams with surgical concentration in specialist hospitals. This guidance was rapidly adopted in the East of England, with a population of 2.5 million.", "The population of the Anglia Cancer Network was approximately 2.3 million.", "From 1996 to 2003, details of 3406 cases of gynaecological cancer were identified in the Anglia region of England. Survival analysis was performed by Cox proportional hazards regression, relative to cases diagnosed in 1996.", "Primary endpoint was survival.", "The survival rates for cases diagnosed between 1996 and 1999 were broadly the same across the time period, with a marked improvement taking place in 2000, and continuing to 2003 (HR 0.71, 95% CI 0.64-0.79, comparing 2000-03 with 1996-99 diagnoses), for all gynaecological sites combined. Adjustment for treatments or method of case follow-up did not attenuate these improvements. There was a concurrent change towards major surgery being performed in specialist centres from 2000." ], "LABELS": [ "OBJECTIVE", "SETTING", "POPULATION", "METHODS", "MAIN OUTCOME MEASURE", "RESULTS" ], "MESHES": [ "England", "Female", "Genital Neoplasms, Female", "Health Services Accessibility", "Humans", "Patient Care Team", "Regional Medical Programs", "Survival Rate" ], "YEAR": "2012", "reasoning_required_pred": "yes", "reasoning_free_pred": "yes", "final_decision": "yes", "LONG_ANSWER": "The adoption of the 1999 guidance on gynaecological cancer, which included multidisciplinary case management and centralisation of surgery, resulted in a marked step-change improvement in survival of gynaecological cancer in an area of eastern England in 2000." }, "21398266": { "QUESTION": "Do symptoms matter when considering patients for phase I clinical trials?", "CONTEXTS": [ "Older adults (OA) with advanced cancer (AC) undergoing phase I clinical trials (PICT) have poor prognosis. There are no studies which describe symptoms experienced by OA.", "Retrospective chart review of PICT participants>60 years. OA were compared by age (>65 vs 60-65) and by number of symptoms (>3 vs \u22643).", "N = 56. Mean age = 67.09; 48.21% female. Median life-expectancy = 5 months (interquartile range = 2-9 months); 80.36% had pain; of those 64% without pain scale. Most did not have interdisciplinary professionals or hospice referrals. Older adults with>3 symptoms had more admissions (37.5% vs 14.29%; P = .0335), complications (46.43% vs 16.07%; P = .0026), and greater decline in functional status (24 participants>3 symptoms vs 8; P = .0173). There were no significant differences comparing OA by age." ], "LABELS": [ "BACKGROUND", "METHODS", "RESULTS" ], "MESHES": [ "Aged", "Clinical Protocols", "Clinical Trials, Phase I as Topic", "Female", "Health Status", "Humans", "Life Expectancy", "Male", "Middle Aged", "Neoplasms", "Pain", "Patient Admission", "Patient Selection", "Pilot Projects", "Quality of Life", "Research Design", "Retrospective Studies" ], "YEAR": "2011", "reasoning_required_pred": "yes", "reasoning_free_pred": "yes", "final_decision": "yes", "LONG_ANSWER": "Older adults enrolled in PICT with more symptoms may sacrifice QOL for experimental treatment." }, "25371231": { "QUESTION": "Is vitamin D insufficiency or deficiency related to the development of osteochondritis dissecans?", "CONTEXTS": [ "The aetiology of osteochondritis dissecans is still unclear. The aim of this prospective pilot study was to analyse whether vitamin D insufficiency, or deficiency, might be a contributing etiological factor in the development of an OCD lesion.", "The serum level of vitamin D3 in 23 consecutive patients (12 male and 11 female) suffering from a stage III, or stages III and IV, OCD lesion (mostly stage III) admitted for surgery was measured.", "The patients' mean age was 31.3\u00a0years and most of them already exhibited closed epiphyseal plates. In the majority of patients (18/23), a distinct vitamin D3 deficiency was found, two patients were vitamin D3-insufficient and, in three patients, the vitamin D3 level reached the lowest normal value." ], "LABELS": [ "PURPOSE", "METHODS", "RESULTS" ], "MESHES": [ "Adolescent", "Adult", "Aged", "Child", "Female", "Humans", "Male", "Middle Aged", "Osteochondritis Dissecans", "Pilot Projects", "Prospective Studies", "Vitamin D Deficiency", "Young Adult" ], "YEAR": "2016", "reasoning_required_pred": "yes", "reasoning_free_pred": "maybe", "final_decision": "maybe", "LONG_ANSWER": "These first data show that a vitamin D3 deficiency rather than an insufficiency may be involved in the development of OCD lesions. Probably, with a vitamin D3 substitution, the development of an advanced OCD stage could be avoided. Further analyses, including morphological analyses regarding a possible osteomalacia, and examination of the PTH and other determinants of the bone metabolism, should be undertaken to either confirm or refute these data." }, "23252468": { "QUESTION": "Do episodic migraineurs selectively attend to headache-related visual stimuli?", "CONTEXTS": [ "To assess pain-related attentional biases among individuals with episodic migraine.", "Prior studies have examined whether chronic pain patients selectively attend to pain-related stimuli in the environment, but these studies have produced largely mixed findings and focused primarily on patients with chronic musculoskeletal pain. Limited research has implicated attentional biases among chronic headache patients, but no studies have been conducted among episodic migraineurs, who comprise the overwhelming majority of the migraine population.", "This was a case-control, experimental study. Three hundred and eight participants (mean age\u2009=\u200919.2 years [standard deviation\u2009=\u20093.3]; 69.5% female; 36.4% minority), consisting of 84 episodic migraineurs, diagnosed in accordance with International Classification of Headache Disorders (2(nd) edition) criteria using a structured diagnostic interview, and 224 non-migraine controls completed a computerized dot probe task to assess attentional bias toward headache-related pictorial stimuli. The task consisted of 192 trials and utilized 2 emotional-neutral stimulus pairing conditions (headache-neutral and happy-neutral).", "No within-group differences for reaction time latencies to headache vs happy conditions were found among those with episodic migraine or among the non-migraine controls. Migraine status was unrelated to attentional bias indices for both headache (F [1,306]\u2009=\u20090.56, P\u2009=\u2009.45) and happy facial stimuli (F [1,306]\u2009=\u20090.37, P\u2009=\u2009.54), indicating a lack of between-group differences. Lack of within- and between-group differences was confirmed with repeated measures analysis of variance." ], "LABELS": [ "OBJECTIVE", "BACKGROUND", "METHODS", "RESULTS" ], "MESHES": [ "Adolescent", "Attention", "Bias", "Case-Control Studies", "Disability Evaluation", "Emotions", "Female", "Headache", "Humans", "Male", "Migraine Disorders", "Photic Stimulation", "Reaction Time", "Surveys and Questionnaires", "Young Adult" ], "YEAR": "2013", "reasoning_required_pred": "no", "reasoning_free_pred": "no", "final_decision": "no", "LONG_ANSWER": "In light of the large sample size and prior pilot testing of presented images, results suggest that episodic migraineurs do not differentially attend to headache-related facial stimuli. Given modest evidence of attentional biases among chronic headache samples, these findings suggest potential differences in attentional processing between chronic and episodic headache subforms." }, "22440363": { "QUESTION": "Mitral replacement or repair for functional mitral regurgitation in dilated and ischemic cardiomyopathy: is it really the same?", "CONTEXTS": [ "This was a study to compare the results of mitral valve (MV) repair and MV replacement for the treatment of functional mitral regurgitation (MR) in advanced dilated and ischemic cardiomyopathy (DCM).", "One-hundred and thirty-two patients with severe functional MR and systolic dysfunction (mean ejection fraction 0.32 \u00b1 0.078) underwent mitral surgery in the same time frame. The decision to replace rather than repair the MV was taken when 1 or more echocardiographic predictors of repair failure were identified at the preoperative echocardiogram. Eighty-five patients (64.4%) received MV repair and 47 patients (35.6%) received MV replacement. Preoperative characteristics were comparable between the 2 groups. Only ejection fraction was significantly lower in the MV repair group (0.308 \u00b1 0.077 vs 0.336 \u00b1 0.076, p = 0.04).", "Hospital mortality was 2.3% for MV repair and 12.5% for MV replacement (p = 0.03). Actuarial survival at 2.5 years was 92 \u00b1 3.2% for MV repair and 73 \u00b1 7.9% for MV replacement (p = 0.02). At a mean follow-up of 2.3 years (median, 1.6 years), in the MV repair group LVEF significantly increased (from 0.308 \u00b1 0.077 to 0.382 \u00b1 0.095, p<0.0001) and LV dimensions significantly decreased (p = 0.0001). On the other hand, in the MV replacement group LVEF did not significantly change (from 0.336 \u00b1 0.076 to 0.31 \u00b1 0.11, p = 0.56) and the reduction of LV dimensions was not significant. Mitral valve replacement was identified as the only predictor of hospital (odds ratio, 6; 95% confidence interval, 1.1 to 31; p = 0.03) and overall mortality (hazard ratio, 3.1; 95% confidence interval, 1.1 to 8.9; p = 0.02)." ], "LABELS": [ "BACKGROUND", "METHODS", "RESULTS" ], "MESHES": [ "Aged", "Cardiomyopathy, Dilated", "Echocardiography", "Female", "Heart Valve Prosthesis Implantation", "Hospital Mortality", "Humans", "Male", "Middle Aged", "Mitral Valve", "Mitral Valve Insufficiency", "Myocardial Ischemia", "Retrospective Studies", "Ventricular Function, Left" ], "YEAR": "2012", "reasoning_required_pred": "no", "reasoning_free_pred": "no", "final_decision": "no", "LONG_ANSWER": "In patients with advanced dilated and ischemic cardiomyopathy and severe functional MR, MV replacement is associated with higher in-hospital and late mortality compared with MV repair. Therefore, mitral repair should be preferred whenever possible in this clinical setting." }, "12068831": { "QUESTION": "Do nontriploid partial hydatidiform moles exist?", "CONTEXTS": [ "To study whether nontriploid partial hydatidiform moles truly exist.", "We conducted a reevaluation of pathology and ploidy in 19 putative nontriploid partial hydatidiform moles using standardized histologic diagnostic criteria and repeat flow cytometric testing by the Hedley technique.", "On review of the 19 moles, 53% (10/19) were diploid nonpartial moles (initially pathologically misclassified), and 37% (7/19) were triploid partial moles (initial ploidy misclassifications). One additional case (5%) was a diploid early complete mole (initially pathologically misclassified)." ], "LABELS": [ "OBJECTIVE", "STUDY DESIGN", "RESULTS" ], "MESHES": [ "Boston", "Diagnostic Errors", "Female", "Flow Cytometry", "Histological Techniques", "Humans", "Hydatidiform Mole", "Polyploidy", "Pregnancy", "Uterine Neoplasms" ], "YEAR": "2002", "reasoning_required_pred": "no", "reasoning_free_pred": "no", "final_decision": "no", "LONG_ANSWER": "Nontriploid partial moles probably do not exist: careful reevaluation of putative specimens will probably uncover pathologic or ploid errors in almost all cases." }, "20530150": { "QUESTION": "Is cholecystectomy really an indication for concomitant splenectomy in mild hereditary spherocytosis?", "CONTEXTS": [ "Children referred with symptomatic gallstones complicating HS between April 1999 and April 2009 were prospectively identified and reviewed retrospectively. During this period, the policy was to undertake concomitant splenectomy only if indicated for haematological reasons and not simply because of planned cholecystectomy.", "A total of 16 patients (mean age 10.4, range 3.7 to 16 years, 11 women) with HS and symptomatic gallstones underwent cholecystectomy. Three patients subsequently required a splenectomy for haematological reasons 0.8-2.5 years after cholecystectomy; all three splenectomies were performed laparoscopically. There were no postoperative complications in the 16 patients; postoperative hospital stay was 1-3 days after either cholecystectomy or splenectomy. The 13 children with a retained spleen remain under regular review by a haematologist (median follow-up 4.6, range 0.5 to 10.6 years) and are well and transfusion independent." ], "LABELS": [ "METHODS", "RESULTS" ], "MESHES": [ "Adolescent", "Child", "Child, Preschool", "Cholecystectomy", "Female", "Follow-Up Studies", "Gallstones", "Humans", "Laparoscopy", "Male", "Retrospective Studies", "Spherocytosis, Hereditary", "Splenectomy", "Treatment Outcome", "Unnecessary Procedures" ], "YEAR": "2010", "reasoning_required_pred": "no", "reasoning_free_pred": "maybe", "final_decision": "no", "LONG_ANSWER": "The advice to perform a concomitant splenectomy in children with mild HS undergoing cholecystectomy for symptomatic gallstones needs revisiting. In the era of minimal access surgery, the need for splenectomy in such children should be judged on its own merits." }, "21618245": { "QUESTION": "Does surgery or radiation therapy impact survival for patients with extrapulmonary small cell cancers?", "CONTEXTS": [ "Extrapulmonary small cell carcinomas (EPSCC) are rare tumors where therapy remains poorly defined. We sought to determine the impact of surgical extirpation and radiation therapy for outcomes of EPSCC.", "The Surveillance, Epidemiology, and End Results (SEER) database was queried for patients with EPSCC which were further categorized by site and evaluated for survival by specific treatment strategy.", "We identified 94,173 patients with small cell carcinoma of which 88,605 (94.1%) and 5,568 (5.9%) had pulmonary small cell carcinoma and EPSCC, respectively. EPSCC patients were subdivided by site with the following proportions: genitourinary (24.1%), gastrointestinal (22.1%), head and neck (7.1%), breast (4%), and miscellaneous (42.7%). Overall EPSSC and specifically gastrointestinal disease had significantly improved median, 5- and 10-year survival with surgery and/or radiation for all stages and sizes. For all EPSCCs multivariate analysis revealed age (>50), gender (female), stage (regional, distant), radiation, and surgery to be independent predictors of survival." ], "LABELS": [ "BACKGROUND AND OBJECTIVES", "METHODS", "RESULTS" ], "MESHES": [ "Aged", "Brachytherapy", "Breast Neoplasms", "Combined Modality Therapy", "Female", "Follow-Up Studies", "Gastrointestinal Neoplasms", "Head and Neck Neoplasms", "Humans", "Lung Neoplasms", "Male", "Small Cell Lung Carcinoma", "Survival Rate", "Treatment Outcome", "Urogenital Neoplasms" ], "YEAR": "2011", "reasoning_required_pred": "yes", "reasoning_free_pred": "yes", "final_decision": "yes", "LONG_ANSWER": "Although outcomes for EPSCC remains poor, both surgery and radiation is shown to significantly improve median, 5- and 10-year survival rates. EPSCC patients who are potential candidates for surgical resection or radiation therapy may benefit from these treatments." }, "26085176": { "QUESTION": "MR Diagnosis of Bone Metastases at 1.5 T and 3 T: Can STIR Imaging Be Omitted?", "CONTEXTS": [ "To date, no prospective comparative study of the diagnostic value of STIR versus T1-weighted (T1w) sequences at both 1.5 T and 3 T has been performed with special focus on the detectability of bone metastases.", "212 oncological patients had a whole-body MRI at 1.5 T and/or at 3 T. The standard protocol comprised STIR and T1w sequences. All patients who showed typical signs of bone metastases were included in the study. Evaluation of the images was performed by the calculation of the number of metastases by three independent readers and by visual assessment on a 4-point scale.", "86 patients fulfilled the inclusion criteria. The total number of metastases was significantly higher on T1w than on STIR images at both field strengths (p<0.05). T1w revealed a sensitivity of 99.72% (3 T) and 100.00% (1.5 T) versus STIR with 70.99 % (3 T) and 79.34 % (1.5 T). In 53% (38/72) of all patients, STIR detected fewer bone metastases in comparison with T1w at 3\u200aT. At 1.5 T, STIR showed inferior results in 37.5 % (18/48) of all patients. Qualitative analysis indicated a significantly better lesion conspicuity, lesion delineation and an improved image quality on T1w compared to STIR imaging at both field strengths (p<0.05) with similar results for T1w at 1.5 T and 3 T, but inferior results for STIR especially at 3 T." ], "LABELS": [ "OBJECTIVE", "MATERIALS AND METHODS", "RESULTS" ], "MESHES": [ "Aged", "Bone Neoplasms", "Bone and Bones", "Female", "Humans", "Image Enhancement", "Magnetic Resonance Imaging", "Male", "Middle Aged", "Prospective Studies", "Sensitivity and Specificity", "Spinal Neoplasms", "Spine", "Whole Body Imaging" ], "YEAR": "2015", "reasoning_required_pred": "yes", "reasoning_free_pred": "yes", "final_decision": "yes", "LONG_ANSWER": "The whole-body MRI protocol for the detection of bone metastases could safely be limited to the T1w sequence in adults, especially at 3 T. There is no need for an additional STIR sequence. These initial results will have a major impact on the department's workflow if confirmed by larger studies as they will help reduce examination time and therefore save financial resources." }, "18158048": { "QUESTION": "Histologic evaluation of the testicular remnant associated with the vanishing testes syndrome: is surgical management necessary?", "CONTEXTS": [ "There is controversy surrounding the optimal management of the testicular remnant associated with the vanishing testes syndrome. Some urologists advocate the need for surgical exploration, whereas others believe this is unnecessary. These differing opinions are based on the variable reports of viable germ cell elements found within the testicular remnants. To better understand the pathology associated with this syndrome and the need for surgical management, we reviewed our experience regarding the incidence of viable germ cell elements within the testicular remnant.", "An institutional review board-approved, retrospective review was performed of all consecutive patients undergoing exploration for a nonpalpable testis at Eastern Virginia Medical School and Geisinger Medical Center between 1994 and 2006. Patients who were found to have spermatic vessels and a vas deferens exiting a closed internal inguinal ring were included in this analysis.", "Fifty-six patients underwent removal of the testicular remnant. Patient age ranged from 11 to 216 months. In 8 of the specimens (14%), we identified viable germ cell elements. In an additional 4 patients (7%), we identified seminiferous tubules without germ cell elements." ], "LABELS": [ "OBJECTIVES", "METHODS", "RESULTS" ], "MESHES": [ "Child, Preschool", "Cryptorchidism", "Germ Cells", "Humans", "Infant", "Male", "Seminiferous Tubules", "Testis" ], "YEAR": "2007", "reasoning_required_pred": "maybe", "reasoning_free_pred": "yes", "final_decision": "yes", "LONG_ANSWER": "In our review, we identified that a significant number of testicular remnants associated with the vanishing testes syndrome can harbor viable germ cell elements or seminiferous tubules. The exact fate of these residual elements remains unknown; however, there may exist the potential for malignant transformation. Given the potential for malignant degeneration, we believe that these remnants should be removed." }, "21256734": { "QUESTION": "Does pain intensity predict a poor opioid response in cancer patients?", "CONTEXTS": [ "A secondary analysis of one-hundred-sixty-seven patients referred for treatment of cancer-related pain was conducted. Pain intensity at admission was recorded and patients were divided in three categories of pain intensity: mild, moderate and severe. Patients were offered a treatment with opioid dose titration, according to department policy. Data regarding opioid doses and pain intensity were collected after dose titration was completed. Four levels of opioid response were considered: (a) good pain control, with minimal opioid escalation and without relevant adverse effects; (b) good pain control requiring more aggressive opioid escalation, for example doubling the doses in four days; (c) adequate pain control associated with the occurrence of adverse effects; (d) poor pain control with adverse effects.", "Seventy-six, forty-four, forty-one and six patients showed a response a, b, c, and d, respectively. No correlation between baseline pain intensity categories and opioid response was found. Patients with response 'b' and 'd' showed higher values of OEImg." ], "LABELS": [ "METHODS", "RESULTS" ], "MESHES": [ "Aged", "Analgesics, Opioid", "Analysis of Variance", "Female", "Humans", "Karnofsky Performance Status", "Male", "Middle Aged", "Neoplasms", "Pain", "Pain Measurement", "Prospective Studies", "Treatment Outcome" ], "YEAR": "2011", "reasoning_required_pred": "no", "reasoning_free_pred": "no", "final_decision": "no", "LONG_ANSWER": "Baseline pain intensity does not predict the outcome after an appropriate opioid titration. It is likely that non-homogeneous pain treatment would have biased the outcome of a previous work." }, "28027677": { "QUESTION": "Do prerecorded lecture VODcasts affect lecture attendance of first-yearpre-clinical Graduate Entry to Medicine students?", "CONTEXTS": [ "There is increasing concern amongst educators that the provision of recorded lectures may reduce student attendance of live lectures. We therefore sought to determine if the provision of prerecorded lecture video podcasts (VODcasts) to first-year Graduate Entry to Medicine (GEM) students, affected attendance at 21 Physiology lectures within three separate pre-clinical modules.", "Data on lecture attendance, utilization of VODcasts, and whether VODcasts should replace live lectures were drawn from three surveys conducted in academic years 2014-2015 and 2015-2016 on all first-year GEM students in two first-year pre-clinical modules where prerecorded Physiology VODcasts were available for viewing or downloading prior to scheduled live lectures.", "A total of 191/214 (89%) students responded to the three surveys, with 84.3% of students attending all 21 lectures in the study. Only 4% of students missed more than one lecture in each of the three lecture series, with 79% indicating that VODcasts should not replace lectures." ], "LABELS": [ "BACKGROUND", "METHODS", "RESULTS" ], "MESHES": [ "Adult", "Education, Medical, Undergraduate", "Female", "Humans", "Internet", "Male", "Surveys and Questionnaires", "Teaching", "Videotape Recording", "Young Adult" ], "YEAR": "2017", "reasoning_required_pred": "no", "reasoning_free_pred": "no", "final_decision": "no", "LONG_ANSWER": "Therefore, we conclude that the attendance of pre-clinical GEM students at live lectures is not significantly impacted upon by the provision of lecture VODcasts, with most students viewing them as useful revision tools rather than as a replacement for live lectures." }, "17595200": { "QUESTION": "Is there an intrauterine influence on obesity?", "CONTEXTS": [ "It has been suggested that increasing obesity levels in young women lead to intrauterine environments that, in turn, stimulate increased obesity among their offspring, generating an intergenerational acceleration of obesity levels. If this mechanism is important, the association of maternal body mass index (BMI) with offspring BMI should be stronger than the association of paternal with offspring BMI.", "To compare the relative strengths of association of maternal and paternal BMI with offspring BMI at age 7.5, taking into account the possible effect of non-paternity.", "We compared strength of association for maternal-offspring and paternal-offspring BMI for 4654 complete parent-offspring trios in the Avon Longitudinal Study of Parents and Children (ALSPAC), using unstandardised and standardised regression analysis. We carried out a sensitivity analysis to investigate the influence of non-paternity on these associations.", "The strength of association between parental BMI and offspring BMI at age 7.5 was similar for both parents. Taking into account correlations between maternal and paternal BMI, performing standardised rather than unstandardised regression and carrying out a sensitivity analysis for non-paternity emphasised the robustness of the general similarity of the associations. The associations between high parental BMI (top decile) and offspring BMI are also similar for both parents." ], "LABELS": [ "BACKGROUND", "OBJECTIVE", "METHODS", "RESULTS" ], "MESHES": [ "Adult", "Body Mass Index", "Child", "Child, Preschool", "Cross-Sectional Studies", "Fathers", "Female", "Health Surveys", "Humans", "Longitudinal Studies", "Male", "Mothers", "Obesity", "Parent-Child Relations", "Pregnancy", "United Kingdom" ], "YEAR": "2007", "reasoning_required_pred": "no", "reasoning_free_pred": "no", "final_decision": "no", "LONG_ANSWER": "Comparison of mother-offspring and father-offspring associations for BMI suggests that intergenerational acceleration mechanisms do not make an important contribution to levels of childhood BMI within the population. Associations at later ages and for different components of body composition now require study." }, "26548832": { "QUESTION": "Assessing Patient Reported Outcomes Measures via Phone Interviews Versus Patient Self-Survey in the Clinic: Are We Measuring the Same Thing?", "CONTEXTS": [ "Longitudinally following patients requires a full-time employee (FTE)-dependent data inflow infrastructure. There are efforts to capture patient-reported outcomes (PROs) by the use of non-FTE-dependent methodologies. In this study, we set out to assess the reliability of PRO data captured via FTE-dependent compared with non-FTE-dependent methodologies.", "A total of 119 adult patients (65 men) who underwent 1-and 2-level lumbar fusions at Duke University Medical Center were enrolled in this prospective study. Enrollment criteria included available demographic, clinical, and PRO data. All patients completed 2 sets of questionnaires--the first a phone interviews and the second a self-survey. There was at least a 2-week period between the phone interviews and self-survey. Questionnaires included the Oswestry Disability Index (ODI), the visual analog scale for back pain (VAS-BP), and the visual analog scale for leg pain (VAS-LP). Repeated-measures analysis of variance was used to compare the reliability of baseline PRO data captured.", "A total of 39.49% of patients were smokers, 21.00% had diabetes, and 11.76% had coronary artery disease; 26.89% reported history of anxiety disorder, and 28.57% reported history of depression. A total of 97.47% of patients had a high-school diploma or General Education Development, and 49.57% attained a 4-year college degree or postgraduate degree. We observed a high correlation between baseline PRO data captured between FTE-dependent versus non-FTE dependent methodologies (ODI: r = -0.89, VAS-BP: r = 0.74, VAS-LP: r = 0.70). There was no difference in PROs of baseline pain and functional disability between FTE-dependent and non-FTE-dependent methodologies: baseline ODI (FTE-dependent: 47.73 \u00b1 16.77 [mean \u00b1 SD] vs. non-FTE-dependent: 45.81 \u00b1 12.11, P = 0.39), VAS-LP (FTE-dependent: 6.13 \u00b1 2.78 vs. non-FTE-dependent: 6.46 \u00b1 2.79, P = 0.36) and VAS-BP (FTE-dependent: 6.33 \u00b1 2.90 vs. non-FTE-dependent: 6.53 \u00b1 2.48, P = 0.57)." ], "LABELS": [ "BACKGROUND", "METHODS", "RESULTS" ], "MESHES": [ "Adult", "Aged", "Disability Evaluation", "Female", "Health Personnel", "Humans", "Lumbosacral Region", "Male", "Middle Aged", "Pain", "Pain Measurement", "Patient Outcome Assessment", "Prospective Studies", "Reproducibility of Results", "Risk Factors", "Socioeconomic Factors", "Spinal Fusion", "Surveys and Questionnaires", "Telephone" ], "YEAR": "2016", "reasoning_required_pred": "yes", "reasoning_free_pred": "yes", "final_decision": "yes", "LONG_ANSWER": "Our study suggests that there is great reliability between PRO data captured between FTE-dependent and non-FTE-dependent methodologies." }, "15879722": { "QUESTION": "Is cytokeratin immunoreactivity useful in the diagnosis of short-segment Barrett's oesophagus in Korea?", "CONTEXTS": [ "Cytokeratin 7/20 staining has been reported to be helpful in diagnosing Barrett's oesophagus and gastric intestinal metaplasia. However, this is still a matter of some controversy.", "To determine the diagnostic usefulness of cytokeratin 7/20 immunostaining for short-segment Barrett's oesophagus in Korea.", "In patients with Barrett's oesophagus, diagnosed endoscopically, at least two biopsy specimens were taken from just below the squamocolumnar junction. If goblet cells were found histologically with alcian blue staining, cytokeratin 7/20 immunohistochemical stains were performed. Intestinal metaplasia at the cardia was diagnosed whenever biopsy specimens taken from within 2 cm below the oesophagogastric junction revealed intestinal metaplasia. Barrett's cytokeratin 7/20 pattern was defined as cytokeratin 20 positivity in only the superficial gland, combined with cytokeratin 7 positivity in both the superficial and deep glands.", "Barrett's cytokeratin 7/20 pattern was observed in 28 out of 36 cases (77.8%) with short-segment Barrett's oesophagus, 11 out of 28 cases (39.3%) with intestinal metaplasia at the cardia, and nine out of 61 cases (14.8%) with gastric intestinal metaplasia. The sensitivity and specificity of Barrett's cytokeratin 7/20 pattern were 77.8 and 77.5%, respectively." ], "LABELS": [ "BACKGROUND", "OBJECTIVE", "METHODS", "RESULTS" ], "MESHES": [ "Adult", "Aged", "Barrett Esophagus", "Biomarkers", "Biopsy", "Cardia", "Esophagoscopy", "Female", "Humans", "Intermediate Filament Proteins", "Keratin-20", "Keratin-7", "Keratins", "Male", "Metaplasia", "Middle Aged", "Sensitivity and Specificity", "Stomach" ], "YEAR": "2005", "reasoning_required_pred": "yes", "reasoning_free_pred": "yes", "final_decision": "yes", "LONG_ANSWER": "Barrett's cytokeratin 7/20 pattern can be a useful marker for the diagnosis of short-segment Barrett's oesophagus, although the false positive or false negative rate is approximately 25%." }, "27554179": { "QUESTION": "Is routine dissection of the station 9 lymph nodes really necessary for primary lung cancer?", "CONTEXTS": [ "Mediastinal lymph node dissection is an essential component of lung cancer surgery; however, choosing mediastinal lymph nodes stations to be dissected is subjective. We carried out this research to investigate the need for dissection of station 9 lymph nodes during lung cancer surgery.", "Patients with primary lung cancer who underwent radical surgery between 2010 and 2014 were retrospectively reviewed. Clinical, pathologic, and prognosis data were obtained and analyzed.", "A total number of 1397 patients were included in this research. The metastasis rate of station 9 was 3.45%, which was significantly lower than other mediastinal stations. This metastasis rate was significantly correlated with pT stage, the lobe where the tumor was located, metastasis status of intrapulmonary lymph nodes, pTNM stage, and most of the other mediastinal lymph node stations. In males or ground glass opacity (GGO) patients, the metastasis of station 9 nodes was more unlikely to occur, even though there was no statistical significance. The staging results of most patients (99.63%) would not be impaired, even if station 9 nodes were not dissected, and the prognostic analysis showed that the metastasis status of station 9 had no significant influence on survival." ], "LABELS": [ "OBJECTIVES", "METHODS", "RESULTS" ], "MESHES": [ "Adult", "Aged", "Female", "Humans", "Lung Neoplasms", "Lymph Node Excision", "Lymphatic Metastasis", "Male", "Middle Aged", "Prognosis", "Retrospective Studies" ], "YEAR": "2016", "reasoning_required_pred": "no", "reasoning_free_pred": "no", "final_decision": "no", "LONG_ANSWER": "The metastasis rate of station 9 lymph nodes was significantly lower than other mediastinal stations in lung cancer patients. The metastasis status of station 9 had no significant influence on tumor staging or prognosis. Routine dissection of station 9 lymph nodes may not be necessary, especially in patients with a low T stage, upper or middle lobe tumors, or without intrapulmonary lymph node metastasis." }, "15381614": { "QUESTION": "Cutaneous melanoma in a multiethnic population: is this a different disease?", "CONTEXTS": [ "Cutaneous melanoma in nonwhite persons has a manifestation and a prognosis that are different than those of cutaneous melanoma in white persons.", "Case series.", "Tertiary care university-affiliated community medical center located in a multiethnic state in which white persons are a minority of the population.", "Consecutive series of 357 patients with melanoma seen between January 1994 and August 2003.", "Ethnicity, age, sex, primary site, tumor thickness, nodal status, stage at diagnosis, and survival.", "There were 208 men and 149 women who ranged in age from 15 to 93 years (mean, 58 years). Twenty-two patients initially had unknown primary sites. Of these 357 patients, 67 (18.7%) were nonwhite. There was no statistically significant difference in the age (P =.10) or sex (P =.57) distribution of these 2 populations. Nonwhite patients at initial diagnosis had thicker tumors (P =.002), more frequently had ulcerated primary tumors (P<.001), more frequently had positive nodes (P =.004), and were at a more advanced stage (P =.002) than their white counterparts. The anatomic distribution between the 2 populations was significantly different (P<.001), with a high incidence of melanoma on the sole and subungual locations and a substantially less frequent occurrence on the head and neck, trunk, and extremities in the nonwhite population when compared with the white population. The overall survival rate of the nonwhite patients was significantly worse than that of the white patients, but when stratified by stage at initial diagnosis, there was no difference in outcome." ], "LABELS": [ "HYPOTHESIS", "DESIGN", "SETTING", "PATIENTS", "MAIN OUTCOME MEASURES", "RESULTS" ], "MESHES": [ "Adolescent", "Adult", "Aged", "Aged, 80 and over", "Chi-Square Distribution", "Combined Modality Therapy", "Female", "Humans", "Male", "Melanoma", "Middle Aged", "Phenotype", "Proportional Hazards Models", "Skin Neoplasms" ], "YEAR": "2004", "reasoning_required_pred": "maybe", "reasoning_free_pred": "maybe", "final_decision": "maybe", "LONG_ANSWER": "In a multiethnic population, cutaneous melanoma in nonwhite persons is unusual but not rare. Although the diagnoses are distinctly different at initial examinations, suggesting a potential biological component, stage-for-stage outcomes are similar between white and nonwhite persons and suggest a need for early diagnostic interventions with unusual pigmented lesions in nonwhite persons." }, "22012962": { "QUESTION": "Marital status, living arrangement and mortality: does the association vary by gender?", "CONTEXTS": [ "Men appear to benefit more from being married than women with respect to mortality in middle age. However, there is some uncertainty about gender differences in mortality risks in older individuals, widowed, divorced and single individuals and about the impact of living arrangements.", "Longitudinal data with 1990 census records being linked to mortality data up to 2005 were used (Swiss National Cohort). The sample comprised all residents over age 44 years in Switzerland (n=2,440,242). All-cause mortality HRs for marital status and living arrangements were estimated by Cox regression for men and women and different age groups with adjustment for education and socio-professional category.", "The benefit of being married was stronger for men than for women; however, mortality patterns were similar, with higher mortality in divorced and single individuals compared with widowed individuals (<80 years). After adjustment for living arrangements, the gender difference by marital status disappeared. Stratification by living arrangement revealed that mortality risks were highest for 45-64-year-old divorced (HR 1.72 (95% CI 1.67 to 1.76)) and single men (HR 1.67 (95% CI 1.63 to 1.71)) who lived alone. In women of the same age, the highest mortality risk was observed for those who were single and living with a partner (HR 1.70 (95% CI 1.58 to 1.82)). In older age groups, the impact of marital status decreased." ], "LABELS": [ "BACKGROUND", "METHODS", "RESULTS" ], "MESHES": [ "Aged", "Aged, 80 and over", "Female", "Humans", "Male", "Marital Status", "Middle Aged", "Mortality", "Proportional Hazards Models", "Residence Characteristics", "Switzerland" ], "YEAR": "2012", "reasoning_required_pred": "maybe", "reasoning_free_pred": "yes", "final_decision": "yes", "LONG_ANSWER": "Evaluation of living arrangements is crucial for identifying and explaining gender differences in mortality risks by marital status. The impact of living alone and living with a partner seems to be different in men and women." }, "18948835": { "QUESTION": "Does somatostatin confer insulinostatic effects of neuromedin u in the rat pancreas?", "CONTEXTS": [ "Neuromedin U (NmU) is a neuropeptide with anorexigenic activity. Two receptor subtypes (NmUR1 and NmUR2) confer the effects of NmU on target cells. We have recently demonstrated that NmU reduces insulin secretion from isolated pancreatic islets. Aim of our current study is to investigate the role of somatostatin at mediating the effects of NmU on insulin secretion.", "Expression of NmU in the pancreas was detected by immunohistochemistry. Insulin and somatostatin secretion from in situ perfused rat pancreas and isolated pancreatic islets was measured by radioimmunoassay. The paracrine effects of somatostatin within pancreatic islets were blocked by cyclosomatostatin, a somatostatin receptor antagonist.", "Receptor subtype NmUR1, but not NmUR2, was expressed in the endocrine pancreas, predominantly in the periphery. Neuromedin U reduced insulin secretion from in situ perfused rat pancreas and stimulated somatostatin secretion from isolated pancreatic islets. Neuromedin U stimulated somatostatin secretion at both physiological and supraphysiological glucose concentrations. Cyclosomatostatin increased insulin secretion and reduced NmU-induced inhibition of insulin secretion." ], "LABELS": [ "OBJECTIVES", "METHODS", "RESULTS" ], "MESHES": [ "Animals", "Insulin", "Neuropeptides", "Pancreas", "Rats", "Rats, Wistar", "Receptors, Neurotransmitter", "Somatostatin" ], "YEAR": "2009", "reasoning_required_pred": "yes", "reasoning_free_pred": "yes", "final_decision": "yes", "LONG_ANSWER": "Neuromedin U reduces insulin and increases somatostatin secretion. Blockade of somatostatin action abolishes the inhibition of insulin secretion by NmU. The results of the study suggest that somatostatin mediates the inhibitory action of NmU on insulin secretion." }, "18435678": { "QUESTION": "Kell alloimmunization in pregnancy: associated with fetal thrombocytopenia?", "CONTEXTS": [ "Kell haemolytic disease in pregnancies has been suggested to be associated with decreased fetal platelet counts. The aim of this study was to evaluate the incidence and clinical significance of fetal thrombocytopenia in pregnancies complicated by Kell alloimmunization.", "In this retrospective cohort study, fetal platelet counts were performed in 42 pregnancies with severe Kell alloimmunization prior to the first intrauterine blood transfusion. Platelet counts from 318 first intrauterine transfusions in RhD alloimmunized pregnancies were used as controls.", "Fetal thrombocytopenia (platelet count<150 x 10(9)/l) was found in 4/42 (10%) in the Kell group and in 84/318 (26%) in the RhD group. None of the fetuses in the Kell alloimmunized pregnancies, including 15 with severe hydrops, had a clinically significant thrombocytopenia defined as a platelet count<50 x 10(9)/l. In the RhD alloimmunized pregnancies, 2/230 (1%) of the non-hydropic fetuses and 7/30 (23%) of the severely hydropic fetuses had a clinically significant thrombocytopenia." ], "LABELS": [ "BACKGROUND AND OBJECTIVES", "MATERIALS AND METHODS", "RESULTS" ], "MESHES": [ "Blood Group Incompatibility", "Cohort Studies", "Edema", "Female", "Fetus", "Humans", "Incidence", "Kell Blood-Group System", "Pregnancy", "Pregnancy Complications, Hematologic", "Prospective Studies", "Rh Isoimmunization", "Thrombocytopenia, Neonatal Alloimmune" ], "YEAR": "2008", "reasoning_required_pred": "no", "reasoning_free_pred": "no", "final_decision": "no", "LONG_ANSWER": "In contrast to fetuses with severe anaemia and hydrops due to RhD alloimmunization, fetuses with severe anaemia due to Kell alloimmunization are generally not at risk for substantial thrombocytopenia." }, "19268855": { "QUESTION": "Application of computer-aided diagnosis (CAD) in MR-mammography (MRM): do we really need whole lesion time curve distribution analysis?", "CONTEXTS": [ "The identification of the most suspect enhancing part of a lesion is regarded as a major diagnostic criterion in dynamic magnetic resonance mammography. Computer-aided diagnosis (CAD) software allows the semi-automatic analysis of the kinetic characteristics of complete enhancing lesions, providing additional information about lesion vasculature. The diagnostic value of this information has not yet been quantified.", "Consecutive patients from routine diagnostic studies (1.5 T, 0.1 mmol gadopentetate dimeglumine, dynamic gradient-echo sequences at 1-minute intervals) were analyzed prospectively using CAD. Dynamic sequences were processed and reduced to a parametric map. Curve types were classified by initial signal increase (not significant, intermediate, and strong) and the delayed time course of signal intensity (continuous, plateau, and washout). Lesion enhancement was measured using CAD. The most suspect curve, the curve-type distribution percentage, and combined dynamic data were compared. Statistical analysis included logistic regression analysis and receiver-operating characteristic analysis.", "Fifty-one patients with 46 malignant and 44 benign lesions were enrolled. On receiver-operating characteristic analysis, the most suspect curve showed diagnostic accuracy of 76.7 +/- 5%. In comparison, the curve-type distribution percentage demonstrated accuracy of 80.2 +/- 4.9%. Combined dynamic data had the highest diagnostic accuracy (84.3 +/- 4.2%). These differences did not achieve statistical significance. With appropriate cutoff values, sensitivity and specificity, respectively, were found to be 80.4% and 72.7% for the most suspect curve, 76.1% and 83.6% for the curve-type distribution percentage, and 78.3% and 84.5% for both parameters." ], "LABELS": [ "RATIONALE AND OBJECTIVES", "MATERIALS AND METHODS", "RESULTS" ], "MESHES": [ "Algorithms", "Artificial Intelligence", "Breast Neoplasms", "Humans", "Image Enhancement", "Image Interpretation, Computer-Assisted", "Magnetic Resonance Imaging", "Male", "Middle Aged", "Pattern Recognition, Automated", "Reproducibility of Results", "Sensitivity and Specificity" ], "YEAR": "2009", "reasoning_required_pred": "no", "reasoning_free_pred": "maybe", "final_decision": "no", "LONG_ANSWER": "The integration of whole-lesion dynamic data tends to improve specificity. However, no statistical significance backs up this finding." }, "12238307": { "QUESTION": "Vaccine protection in the elderly: are Austrian seniors adequately protected by vaccinations?", "CONTEXTS": [ "The aim of the study was to evaluate, if elderly persons are sufficiently protected against infectious diseases by vaccination.", "300 elderly (>60 years) and 300 young (<35 years) persons from five Austrian cities were recruited according to the criteria of a field study. Antibody concentrations against tetanus, diphtheria, tickborne encephalitis and influenza were assessed by ELISA or by haemagglutination inhibition test. Disease and vaccination histories were recorded.", "The results of the study demonstrate that protection against infectious diseases was frequently insufficient in the elderly. This was partly due to the fact that old persons were not vaccinated according to recommended strategies. However, low antibody concentration and a short duration of protective humoral immunity were also observed in many elderly persons in spite of regular vaccination. This was not only the case in frail, but also in healthy elderlies." ], "LABELS": [ "OBJECTIVE", "PROBANDS AND METHODS", "RESULTS" ], "MESHES": [ "Aged", "Aged, 80 and over", "Antibodies", "Austria", "Communicable Diseases", "Enzyme-Linked Immunosorbent Assay", "Female", "Frail Elderly", "Geriatric Assessment", "Health Services Needs and Demand", "Hemagglutination Inhibition Tests", "Humans", "Immune Tolerance", "Immunization Schedule", "Male", "Middle Aged", "Vaccination" ], "YEAR": "2002", "reasoning_required_pred": "no", "reasoning_free_pred": "no", "final_decision": "no", "LONG_ANSWER": "The data demonstrate that vaccination has a relatively weak and short-lasting effect in old age. The results of the study should stimulate discussions about strategies how vaccinations can be made more effective in old age. Improved campaigns, shortened vaccination intervals as well as the design of novel vaccines tailored to fulfill the specific demands of the aging immune system are imaginable." }, "20684175": { "QUESTION": "Vitamin D supplementation and regulatory T cells in apparently healthy subjects: vitamin D treatment for autoimmune diseases?", "CONTEXTS": [ "Epidemiological data show significant associations of vitamin D deficiency and autoimmune diseases. Vitamin D may prevent autoimmunity by stimulating naturally occurring regulatory T cells.", "To elucidate whether vitamin D supplementation increases Tregs frequency (%Tregs) within circulating CD4+ T cells.", "We performed an uncontrolled vitamin D supplementation trial among 50 apparently healthy subjects including supplementation of 140,000 IU at baseline and after 4 weeks (visit 1). The final follow-up visit was performed 8 weeks after the baseline examination (visit 2). Blood was drawn at each study visit to determine 25-hydroxyvitamin D levels and %Tregs. Tregs were characterized as CD4+CD25++ T cells with expression of the transcription factor forkhead box P3 and low or absent expression of CD127.", "Forty-six study participants (65% females, mean age +/- SD 31 +/- 8 years) completed the trial. 25(OH)D levels increased from 23.9 +/- 12.9 ng/ml at baseline to 45.9 +/- 14.0 ng/ml at visit 1 and 58.0 +/- 15.1 ng/ml at visit 2. %Tregs at baseline were 4.8 +/- 1.4. Compared to baseline levels we noticed a significant increase of %Tregs at study visit 1 (5.9 +/- 1.7, P<0.001) and 2 (5.6 +/- 1.6, P<0.001)." ], "LABELS": [ "BACKGROUND", "OBJECTIVES", "METHODS", "RESULTS" ], "MESHES": [ "Adult", "C-Reactive Protein", "CD4 Lymphocyte Count", "CD4-Positive T-Lymphocytes", "Calcium", "Dietary Supplements", "Female", "Follow-Up Studies", "Forkhead Transcription Factors", "Humans", "Immunologic Factors", "Interleukin-2 Receptor alpha Subunit", "Interleukin-7 Receptor alpha Subunit", "Male", "Pilot Projects", "T-Lymphocytes, Regulatory", "Vitamin D", "Vitamins" ], "YEAR": "2010", "reasoning_required_pred": "yes", "reasoning_free_pred": "yes", "final_decision": "yes", "LONG_ANSWER": "Vitamin D supplementation was associated with significantly increased %Tregs in apparently healthy individuals. This immunomodulatory effect of vitamin D might underlie the associations of vitamin D deficiency and autoimmune diseases. Hence, our finding provides a rationale for further studies to investigate vitamin D effects on autoimmunological processes." }, "27216167": { "QUESTION": "Gynecological cancer alarm symptoms: is contact with specialist care associated with lifestyle and socioeconomic status?", "CONTEXTS": [ "The aim of this study was to determine the proportion of patients who were referred to specialist care after reporting gynecological cancer alarm symptoms to their general practitioner. We sought to investigate whether contact with specialist care was associated with lifestyle factors or socioeconomic status.", "Nationwide population-based prospective cohort study in Denmark, based on a random sample of 51 090 women aged 20 years or older from the general population. A web-based questionnaire regarding gynecological alarm symptoms and lifestyle was distributed to the invited individuals. Data about contact with specialist care were obtained from the National Patient Register and the National Health Insurance Service Registry, whereas information about socioeconomic status was collected from Statistics Denmark. Main outcome measures were percentages of patients having contact with specialist care and odds ratios (ORs) for associations between specialist care contact, lifestyle factors and socioeconomic status.", "The study included 25 866 nonpregnant women; 2957 reported the onset of at least one gynecological cancer alarm symptom, and 683 of these (23.1%) reported symptoms to their general practitioner. The proportion of individuals having contact with specialist care ranged from 39.3% (pain during intercourse) to 47.8% (bleeding during intercourse). Individuals with higher educational level had significantly higher odds of contact with a specialist (OR 1.86, 95% CI 1.17-2.95)." ], "LABELS": [ "INTRODUCTION", "MATERIAL AND METHODS", "RESULTS" ], "MESHES": [ "Adult", "Cohort Studies", "Coitus", "Denmark", "Educational Status", "Female", "General Practitioners", "Genital Neoplasms, Female", "Humans", "Life Style", "Middle Aged", "Pain", "Patient Acceptance of Health Care", "Pelvic Pain", "Postmenopause", "Referral and Consultation", "Social Class", "Surveys and Questionnaires", "Uterine Hemorrhage" ], "YEAR": "2016", "reasoning_required_pred": "yes", "reasoning_free_pred": "yes", "final_decision": "yes", "LONG_ANSWER": "Educational level influences contact with specialist care among patients with gynecological cancer alarm symptoms. Future studies should investigate inequalities in access to the secondary healthcare system." }, "19852337": { "QUESTION": "Does elective re-siting of intravenous cannulae decrease peripheral thrombophlebitis?", "CONTEXTS": [ "Peripheral venous thrombophlebitis (PVT) is a common complication of intravenous cannulation, occurring in about 30% of patients. We evaluated the effect of elective re-siting of intravenous cannulae every 48 hours on the incidence and severity of PVT in patients receiving intravenous fluids/drugs.", "We randomized 42 patients who were admitted for major abdominal surgery to either the control or study group (n = 21 in either group). Informed consent was obtained from all of them. Cannulae in the control group were removed only if the site became painful, the cannula got dislodged or there were signs and symptoms suggestive of PVT, namely pain, erythema, swelling, excessive warmth or a palpable venous cord. Cannulae in the study group were changed and re-sited electively every 48 hours. All the patients were examined every 24 hours for signs and symptoms of PVT at the current and previous sites of infusion.", "The incidence of PVT was 100% (21/21) in the control group and only 9.5% (2/21) in the study group (p<0.0001). The severity of PVT was also less in the study group compared with that in the control group. Day-wise correlation of the incidence of PVT showed that 82.6% of the episodes of PVT occurred on day 3." ], "LABELS": [ "BACKGROUND", "METHODS", "RESULTS" ], "MESHES": [ "Adult", "Aged", "Catheterization, Peripheral", "Elective Surgical Procedures", "Female", "Humans", "Infusions, Intravenous", "Male", "Middle Aged", "Prospective Studies", "Thrombophlebitis" ], "YEAR": null, "reasoning_required_pred": "yes", "reasoning_free_pred": "yes", "final_decision": "yes", "LONG_ANSWER": "Elective re-siting of intravenous cannulae every 48 hours results in a significant reduction in the incidence and severity of PVT. We recommend that this should be adopted as standard practice in managing all patients who require prolonged intravenous therapy." }, "22680064": { "QUESTION": "Can third trimester ultrasound predict the presentation of the first twin at delivery?", "CONTEXTS": [ "To determine the ability of early sonogram to predict the presentation of twin A at birth.", "A retrospective cohort study was conducted on all twin pregnancies evaluated at our Fetal Evaluation Unit from 2007 to 2009. Sonogram records were reviewed for the presentation of twin A at seven gestational age intervals and inpatient medical records were reviewed for the presentation of twin A at delivery. The positive predictive value, sensitivity, and specificity of presentation as determined by ultrasound, at each gestational age interval, for the same presentation at delivery were calculated.", "Two hundred and thirty-eight twin pregnancies met inclusion criteria. A total of 896 ultrasounds were reviewed. The positive predictive value of cephalic presentation of twin A as determined by ultrasound for the persistence of cephalic presentation at delivery reached 95% after 28 weeks gestation. The positive predictive value for noncephalic presentation as established by sonogram for noncephalic at delivery was>90% after 32 weeks gestation." ], "LABELS": [ "OBJECTIVE", "METHODS", "RESULTS" ], "MESHES": [ "Adult", "Birth Order", "Birth Weight", "Cohort Studies", "Delivery, Obstetric", "Female", "Humans", "Infant, Newborn", "Labor Presentation", "Predictive Value of Tests", "Pregnancy", "Pregnancy Trimester, Third", "Pregnancy, Twin", "Retrospective Studies", "Twins", "Ultrasonography, Prenatal" ], "YEAR": "2012", "reasoning_required_pred": "yes", "reasoning_free_pred": "yes", "final_decision": "yes", "LONG_ANSWER": "The presentation of the first twin at delivery can be determined by sonogram by the 32nd week of gestation in over 90% of twin pregnancies." }, "25417760": { "QUESTION": "Risk factors for major depression during midlife among a community sample of women with and without prior major depression: are they the same or different?", "CONTEXTS": [ "Women's vulnerability for a first lifetime-onset of major depressive disorder (MDD) during midlife is substantial. It is unclear whether risk factors differ for first lifetime-onset and recurrent MDD. Identifying these risk factors can provide more focused depression screening and earlier intervention. This study aims to evaluate whether lifetime psychiatric and health histories, personality traits, menopausal status and factors that vary over time, e.g. symptoms, are independent risk factors for first-onset or recurrent MDD across 13 annual follow-ups.", "Four hundred and forty-three women, aged 42-52 years, enrolled in the Study of Women's Health Across the Nation in Pittsburgh and participated in the Mental Health Study. Psychiatric interviews obtained information on lifetime psychiatric disorders at baseline and on occurrences of MDD episodes annually. Psychosocial and health-related data were collected annually. Cox multivariable analyses were conducted separately for women with and without a MDD history at baseline.", "Women without lifetime MDD at baseline had a lower risk of developing MDD during midlife than those with a prior MDD history (28% v. 59%) and their risk profiles differed. Health conditions prior to baseline and during follow-ups perception of functioning (ps<0.05) and vasomotor symptoms (VMS) (p = 0.08) were risk factors for first lifetime-onset MDD. Being peri- and post-menopausal, psychological symptoms and a prior anxiety disorder were predominant risk factors for MDD recurrence." ], "LABELS": [ "BACKGROUND", "METHOD", "RESULTS" ], "MESHES": [ "Adult", "Depressive Disorder, Major", "Female", "Health Status", "Humans", "Longitudinal Studies", "Menopause", "Middle Aged", "Pennsylvania", "Personality", "Psychiatric Status Rating Scales", "Recurrence", "Risk Factors" ], "YEAR": "2015", "reasoning_required_pred": "no", "reasoning_free_pred": "maybe", "final_decision": "no", "LONG_ANSWER": "The menopausal transition warrants attention as a period of vulnerability to MDD recurrence, while health factors and VMS should be considered important risk factors for first lifetime-onset of MDD during midlife." }, "22365295": { "QUESTION": "Totally implantable venous access device placement by interventional radiologists: are prophylactic antibiotics necessary?", "CONTEXTS": [ "To determine the rate of early infection for totally implantable venous access devices (TIVADs) placed without antibiotic prophylaxis.", "A list of patients who underwent TIVAD placement in 2009 was obtained from the patient archiving and communication system (PACS). This list was cross-referenced to all patients who underwent TIVAD removal from January 1, 2009, through January 30, 2010, to identify TIVADs that were removed within 30 days of placement. Retrospective chart review was performed to record patient demographics, including age, sex, cancer diagnosis, and indication for removal. Concurrent antibiotic therapy, chemotherapy, and laboratory data before and within 30 days of placement were recorded. Central line-associated bloodstream infections (CLABSIs) were identified using U.S. Centers for Disease Control and Prevention (CDC) criteria.", "There were 1,183 ports placed and 13 removed. CLABSIs occurred in seven (0.6%) patients within 30 days of placement. At the time of TIVAD placement, 81 (7%) patients were receiving antibiotics incidental to the procedure. One patient who received an antibiotic the day of implantation developed a CLABSI. Chemotherapy was administered to 148 (13%) patients on the day of placement." ], "LABELS": [ "PURPOSE", "MATERIAL AND METHODS", "RESULTS" ], "MESHES": [ "Adolescent", "Adult", "Aged", "Aged, 80 and over", "Antibiotic Prophylaxis", "Catheter-Related Infections", "Catheterization, Central Venous", "Catheters, Indwelling", "Device Removal", "Equipment Design", "Female", "Guideline Adherence", "Humans", "Male", "Middle Aged", "Practice Guidelines as Topic", "Radiography, Interventional", "Retrospective Studies", "Risk Assessment", "Risk Factors", "Time Factors", "Treatment Outcome", "Young Adult" ], "YEAR": "2012", "reasoning_required_pred": "no", "reasoning_free_pred": "no", "final_decision": "no", "LONG_ANSWER": "The rate of early infection without antibiotic prophylaxis before TIVAD placement in the interventional radiology suite is<1%. Based on these data, use of prophylactic antibiotics for TIVAD placement is not recommended." }, "15125825": { "QUESTION": "Starting insulin in type 2 diabetes: continue oral hypoglycemic agents?", "CONTEXTS": [ "To evaluate the effects of insulin 30/70 twice daily or bedtime isophane (NPH) insulin plus continued sulfonylurea and metformin in patients with type 2 diabetes in primary care.", "Open-label, randomized trial.", "Persons younger than 76 years with type 2 diabetes whose disease had not been controlled with oral hypoglycemic agents alone. A total of 64 insulin-naive patients treated with maximal feasible dosages of sulfonylurea and metformin (baseline glycosylated hemoglobin [HbA1c]=8.5%) were randomly assigned to insulin monotherapy (IM group; n=31) or insulin in addition to unchanged oral hypoglycemic medication (IC group; n=33) for 12 months. Insulin doses were adjusted to obtain fasting glucose<7.0 mmol/L and postprandial glucose<10.0 mmol/L.", "Outcome measures included HbA1c, treatment failure, weight, hypoglycemic events and symptoms, satisfaction with treatment, general well-being, and fear of injecting insulin and testing.", "HbA1c improved from 8.3% to 7.6% in the IC group, and from 8.8% to 7.6% in the IM group (P=NS). The IC group had 24% treatment failures, compared with 2% in the IM group (P=.09). Patients in the IC group had less weight gain than those in the IM group (1.3 vs 4.2 kg; P=.01), and they reported fewer hypoglycemic events (2.7 vs 4.3; P=.02). Increased satisfaction with treatment was equal in the 2 groups, and general well-being improved by 3.0 points more in the IC group (P=.05). Fear of self-injecting and self-testing did not differ." ], "LABELS": [ "OBJECTIVE", "STUDY DESIGN", "POPULATION", "OUTCOMES MEASURED", "RESULTS" ], "MESHES": [ "Analysis of Variance", "Diabetes Mellitus, Type 2", "Drug Therapy, Combination", "Female", "Glycated Hemoglobin A", "Humans", "Hypoglycemic Agents", "Insulin", "Insulin, Isophane", "Male", "Metformin", "Middle Aged", "Sulfonylurea Compounds" ], "YEAR": "2004", "reasoning_required_pred": "yes", "reasoning_free_pred": "yes", "final_decision": "yes", "LONG_ANSWER": "Bedtime NPH insulin added to maximal therapy with sulfonylurea and metformin is an effective, simple, well-tolerated approach for patients with uncontrolled type 2 diabetes." }, "24652474": { "QUESTION": "Clinical identifiers for early-stage primary/idiopathic adhesive capsulitis: are we seeing the real picture?", "CONTEXTS": [ "Adhesive capsulitis is often difficult to diagnose in its early stage and to differentiate from other common shoulder disorders.", "The aim of this study was to validate any or all of the 8 clinical identifiers of early-stage primary/idiopathic adhesive capsulitis established in an earlier Delphi study.", "This was a cross-sectional study.", "Sixty-four patients diagnosed with early-stage adhesive capsulitis by a physical therapist or medical practitioner were included in the study. Eight active and 8 passive shoulder movements and visual analog scale pain scores for each movement were recorded prior to and immediately following an intra-articular injection of corticosteroid and local anesthetic. Using the local anesthetic as the reference standard, pain relief of \u226570% for passive external rotation was deemed a positive anesthetic response (PAR).", "Sixteen participants (25%) demonstrated a PAR. Univariate logistic regression identified that of the proposed identifiers, global loss of passive range of movement (odds ratio [OR]=0.26, P=.03), pain at the end of range of all measured active movements (OR=0.06, P=.02), and global loss of passive glenohumeral movements (OR=0.23, P=.02) were associated with a PAR. Following stepwise removal of the variables, pain at the end of range of all measured active movements remained the only identifier but was associated with reduced odds of a PAR.", "The lack of a recognized reference standard for diagnosing early-stage adhesive capsulitis remains problematic in all related research." ], "LABELS": [ "BACKGROUND", "OBJECTIVE", "DESIGN", "METHODS", "RESULTS", "LIMITATIONS" ], "MESHES": [ "Adult", "Anesthetics, Local", "Arthralgia", "Bursitis", "Cross-Sectional Studies", "Female", "Humans", "Male", "Middle Aged", "Movement", "Pain Measurement", "Range of Motion, Articular", "Rotation", "Shoulder Joint" ], "YEAR": "2014", "reasoning_required_pred": "maybe", "reasoning_free_pred": "no", "final_decision": "no", "LONG_ANSWER": "None of the clinical identifiers for early-stage adhesive capsulitis previously proposed by expert consensus have been validated in this study. Clinicians should be aware that commonly used clinical identifiers may not be applicable to this stage." }, "24947183": { "QUESTION": "Does delaying surgery in immature adolescent idiopathic scoliosis patients with progressive curve, lead to addition of fusion levels?", "CONTEXTS": [ "To analyze the changes in the curve extent, pattern and the fusion level in adolescent idiopathic scoliosis (AIS) patients who undergo delayed surgery instead of early surgery.", "Thirty-five immature AIS patients whose radiographs demonstrated an initial primary curve of more than 40\u00b0 with a subsequent increase of 10\u00b0 before attaining skeletal maturity with brace were enrolled. The initial and the final radiographs taken before surgery were compared to assess the changes in curve extent, pattern and the fusion levels as recommended by King's, Lenke's and Suk's guidelines.", "The average age of 35 AIS patients was 12.7 \u00b1 1.6 years. The time interval between initial and final radiography was 39.3 \u00b1 20.2 months and the degree of progress of the primary curve was 13 \u00b1 9.7\u00b0. Fusion levels changed in 33 (94.2%), 33 (94.2%) and 32 (91.4%) patients according to King's, Lenke's and Suk's guidelines, respectively. Curve pattern was changed in 2 (5.7%), 12 (34.3%) and 10 (28.6) patients by King's, Lenke's and Suk's guidelines. The mean number of levels requiring fusion increased from 9.4 \u00b1 2.1 at initial visit to 11.1 \u00b1 1.8 at the final follow-up using King's guidelines, 9.7 \u00b1 2.2-11.6 \u00b1 2.0 as per Lenke's guidelines and 9.1 \u00b1 2.0-11.5 \u00b1 2.3 when fusion was planned using Suk's guidelines (p<0.001 in all guidelines)." ], "LABELS": [ "PURPOSE", "METHODS", "RESULTS" ], "MESHES": [ "Adolescent", "Braces", "Child", "Disease Progression", "Female", "Humans", "Male", "Scoliosis", "Spinal Fusion", "Time-to-Treatment", "Treatment Outcome" ], "YEAR": "2014", "reasoning_required_pred": "yes", "reasoning_free_pred": "yes", "final_decision": "yes", "LONG_ANSWER": "Delay of surgery in immature AIS patients whose Cobb's angle exceed 40\u00b0 initially and showing subsequent progression of the curve, of more than 10\u00b0 can lead to alterations in the curve pattern and the need for increase in fusion levels." }, "25779009": { "QUESTION": "Bactericidal activity of 3 cutaneous/mucosal antiseptic solutions in the presence of interfering substances: Improvement of the NF EN 13727 European Standard?", "CONTEXTS": [ "There is no standard protocol for the evaluation of antiseptics used for skin and mucous membranes in the presence of interfering substances. Our objective was to suggest trial conditions adapted from the NF EN 13727 standard, for the evaluation of antiseptics used in gynecology and dermatology.", "Three antiseptic solutions were tested in vitro: a chlorhexidine-benzalkonium (CB) combination, a hexamidine-chlorhexidine-chlorocresol (HCC) combination, and povidone iodine (P). The adaptation of trial conditions to the standard involved choosing dilutions, solvent, and interfering substances. The activity of solutions was assessed on the recommended strains at concentrations of 97% (pure solution), 50%, and 10% (diluted solution), and 1%. A logarithmic reduction \u2265 5 was expected after 60seconds of contact, to meet requirements of bactericidal activity.", "HCC did not present any bactericidal activity except on P. aeruginosa at a concentration of 97%. P was not bactericidal on E. hirae at any concentration and on S. aureus at 97%. CB had the most homogeneous bactericidal activity with a reduction>5 log on the 4 bacterial strains at concentrations of 97%, 50% and 10%." ], "LABELS": [ "OBJECTIVE", "METHODS", "RESULTS" ], "MESHES": [ "Animals", "Anti-Infective Agents, Local", "Benzalkonium Compounds", "Benzamidines", "Cattle", "Chlorhexidine", "Cresols", "Dose-Response Relationship, Drug", "Drug Combinations", "Drug Interactions", "Enterococcus", "Erythrocytes", "Escherichia coli", "Europe", "Hand Disinfection", "Humans", "Inorganic Chemicals", "Microbial Sensitivity Tests", "Mucous Membrane", "Osmolar Concentration", "Povidone-Iodine", "Pseudomonas aeruginosa", "Serum Albumin, Bovine", "Skin", "Solutions", "Staphylococcus aureus" ], "YEAR": "2015", "reasoning_required_pred": "no", "reasoning_free_pred": "maybe", "final_decision": "maybe", "LONG_ANSWER": "Adapting the NF EN 13727 standard allowed assessing the 3 tested solutions: only CB was bactericidal in dirty conditions. This study proved the possibility of validating antiseptic choice in vitro, in current practice conditions, for adjunctive treatment of skin and mucous membranes disorders, primarily of bacterial origin or with a potential of superinfection." }, "17105833": { "QUESTION": "Empiric treatment of uncomplicated urinary tract infection with fluoroquinolones in older women in Israel: another lost treatment option?", "CONTEXTS": [ "Current guidelines for the treatment of uncomplicated urinary tract infection (UTI) in women recommend empiric therapy with antibiotics for which local resistance rates do not exceed 10-20%. We hypothesized that resistance rates of Escherichia coli to fluoroquinolones may have surpassed this level in older women in the Israeli community setting.", "To identify age groups of women in which fluoroquinolones may no longer be appropriate for empiric treatment of UTI.", "Resistance rates for ofloxacin were calculated for all cases of uncomplicated UTI diagnosed during the first 5 months of 2005 in a managed care organization (MCO) in Israel, in community-dwelling women aged 41-75 years. The women were without risk factors for fluoroquinolone resistance. Uncomplicated UTI was diagnosed with a urine culture positive for E. coli. The data set was stratified for age, using 5 year intervals, and stratum-specific resistance rates (% and 95% CI) were calculated. These data were analyzed to identify age groups in which resistance rates have surpassed 10%.", "The data from 1291 urine cultures were included. The crude resistance rate to ofloxacin was 8.7% (95% CI 7.4 to 10.2). Resistance was lowest among the youngest (aged 41-50 y) women (3.2%; 95% CI 1.11 to 5.18), approached 10% in women aged 51-55 years (7.1%; 95% CI 3.4 to 10.9), and reached 19.86% (95% CI 13.2 to 26.5) among the oldest women (aged 56-75 y)." ], "LABELS": [ "BACKGROUND", "OBJECTIVES", "METHODS", "RESULTS" ], "MESHES": [ "Adolescent", "Adult", "Age Factors", "Aged", "Drug Resistance, Bacterial", "Empirical Research", "Escherichia coli", "Escherichia coli Infections", "Female", "Fluoroquinolones", "Humans", "Israel", "Middle Aged", "Practice Guidelines as Topic", "Urinary Tract Infections" ], "YEAR": "2006", "reasoning_required_pred": "yes", "reasoning_free_pred": "maybe", "final_decision": "maybe", "LONG_ANSWER": "Physicians who opt to treat UTI in postmenopausal women empirically should consider prescribing drugs other than fluoroquinolones. Concomitant longitudinal surveillance of both antibiotic utilization patterns and uropathogen resistance rates should become routine practice in this managed-care organization." }, "27338535": { "QUESTION": "Do risk calculators accurately predict surgical site\u00a0occurrences?", "CONTEXTS": [ "Current risk assessment models for surgical site occurrence (SSO) and surgical site infection (SSI) after open ventral hernia repair (VHR) have limited external validation. Our aim was to determine (1) whether existing models stratify patients into groups by risk and (2) which model best predicts the rate of SSO and SSI.", "Patients who underwent open VHR and were followed for at least 1\u00a0mo were included. Using two data sets-a retrospective multicenter database (Ventral Hernia Outcomes Collaborative) and a single-center prospective database (Prospective)-each patient was assigned a predicted risk with each of the following models: Ventral Hernia Risk Score (VHRS), Ventral Hernia Working Group (VHWG), Centers for Disease Control and Prevention Wound Class, and Hernia Wound Risk Assessment Tool (HW-RAT). Patients in the Prospective database were also assigned a predicted risk from the American College of Surgeons National Surgical Quality Improvement Program (ACS-NSQIP). Areas under the receiver operating characteristic curve (area under the curve [AUC]) were compared to assess the predictive accuracy of the models for SSO and SSI. Pearson's chi-square was used to determine which models were able to risk-stratify patients into groups with significantly differing rates of actual SSO and SSI.", "The Ventral Hernia Outcomes Collaborative database (n\u00a0=\u00a0795) had an overall SSO and SSI rate of 23% and 17%, respectively. The AUCs were low for SSO (0.56, 0.54, 0.52, and 0.60) and SSI (0.55, 0.53, 0.50, and 0.58). The VHRS (P\u00a0=\u00a00.01) and HW-RAT (P\u00a0<\u00a00.01) significantly stratified patients into tiers for SSO, whereas the VHWG (P\u00a0<\u00a00.05) and HW-RAT (P\u00a0<\u00a00.05) stratified for SSI. In the Prospective database (n\u00a0=\u00a088), 14% and 8% developed an SSO and SSI, respectively. The AUCs were low for SSO (0.63, 0.54, 0.50, 0.57, and 0.69) and modest for SSI (0.81, 0.64, 0.55, 0.62, and 0.73). The ACS-NSQIP (P\u00a0<\u00a00.01) stratified for SSO, whereas the VHRS (P\u00a0<\u00a00.01) and ACS-NSQIP (P\u00a0<\u00a00.05) stratified for SSI. In both databases, VHRS, VHWG, and Centers for Disease Control and Prevention overestimated risk of SSO and SSI, whereas HW-RAT and ACS-NSQIP underestimated risk for all groups." ], "LABELS": [ "INTRODUCTION", "METHODS", "RESULTS" ], "MESHES": [ "Adult", "Aged", "Databases, Factual", "Decision Support Techniques", "Female", "Follow-Up Studies", "Herniorrhaphy", "Humans", "Male", "Middle Aged", "Prognosis", "ROC Curve", "Retrospective Studies", "Risk Assessment", "Risk Factors", "Surgical Wound Infection" ], "YEAR": "2016", "reasoning_required_pred": "no", "reasoning_free_pred": "no", "final_decision": "no", "LONG_ANSWER": "All five existing predictive models have limited ability to risk-stratify patients and accurately assess risk of SSO. However, both the VHRS and ACS-NSQIP demonstrate modest success in identifying patients at risk for SSI. Continued model refinement is needed to improve the two highest performing models (VHRS and ACS-NSQIP) along with investigation to determine whether modifications to perioperative management based on risk stratification can improve outcomes." }, "27690714": { "QUESTION": "Can EMS Providers Provide Appropriate Tidal Volumes in a Simulated Adult-sized Patient with a Pediatric-sized Bag-Valve-Mask?", "CONTEXTS": [ "In the prehospital setting, Emergency Medical Services (EMS) professionals rely on providing positive pressure ventilation with a bag-valve-mask (BVM). Multiple emergency medicine and critical care studies have shown that lung-protective ventilation protocols reduce morbidity and mortality. Our primary objective was to determine if a group of EMS professionals could provide ventilations with a smaller BVM that would be sufficient to ventilate patients. Secondary objectives included 1) if the pediatric bag provided volumes similar to lung-protective ventilation in the hospital setting and 2) compare volumes provided to the patient depending on the type of airway (mask, King tube, and intubation).", "Using a patient simulator of a head and thorax that was able to record respiratory rate, tidal volume, peak pressure, and minute volume via a laptop computer, participants were asked to ventilate the simulator during six 1-minute ventilation tests. The first scenario was BVM ventilation with an oropharyngeal airway in place ventilating with both an adult- and pediatric-sized BVM, the second scenario had a supraglottic airway and both bags, and the third scenario had an endotracheal tube and both bags. Participants were enrolled in convenience manner while they were on-duty and the research staff was able to travel to their stations. Prior to enrolling, participants were not given any additional training on ventilation skills.", "We enrolled 50 providers from a large, busy, urban fire-based EMS agency with 14.96 (SD = 9.92) mean years of experience. Only 1.5% of all breaths delivered with the pediatric BVM during the ventilation scenarios were below the recommended tidal volume. A greater percentage of breaths delivered in the recommended range occurred when the pediatric BVM was used (17.5% vs 5.1%, p<0.001). Median volumes for each scenario were 570.5mL, 664.0mL, 663.0mL for the pediatric BMV and 796.0mL, 994.5mL, 981.5mL for the adult BVM. In all three categories of airway devices, the pediatric BVM provided lower median tidal volumes (p<0.001)." ], "LABELS": [ "INTRODUCTION", "METHODS", "RESULTS" ], "MESHES": [ "Adult", "Emergency Medical Services", "Female", "Humans", "Male", "Manikins", "Middle Aged", "Respiration, Artificial", "Respiratory Insufficiency", "Resuscitation", "Tidal Volume" ], "YEAR": null, "reasoning_required_pred": "yes", "reasoning_free_pred": "yes", "final_decision": "yes", "LONG_ANSWER": "The study suggests that ventilating an adult patient is possible with a smaller, pediatric-sized BVM. The tidal volumes recorded with the pediatric BVM were more consistent with lung-protective ventilation volumes." }, "21346501": { "QUESTION": "Can students' scores on preclerkship clinical performance examinations predict that they will fail a senior clinical performance examination?", "CONTEXTS": [ "This study was designed to determine whether preclerkship performance examinations could accurately identify medical students at risk for failing a senior clinical performance examination (CPE).", "This study used a retrospective case-control, multiyear design, with contingency table analyses, to examine the performance of 412 students in the classes of 2005 to 2010 at a midwestern medical school. During their second year, these students took four CPEs that each used three standardized patient (SP) cases, for a total of 12 cases. The authors correlated each student's average year 2 case score with the student's average case score on a senior (year 4) CPE. Contingency table analysis was carried out using performance on the year 2 CPEs and passing/failing the senior CPE. Similar analyses using each student's United States Medical Licensing Examination (USMLE) Step 1 scores were also performed. Sensitivity, specificity, odds ratio, and relative risk were calculated for two year 2 performance standards.", "Students' low performances relative to their class on the year 2 CPEs were a strong predictor that they would fail the senior CPE. Their USMLE Step 1 scores also correlated with their performance on the senior CPE, although the predictive values for these scores were considerably weaker." ], "LABELS": [ "PURPOSE", "METHOD", "RESULTS" ], "MESHES": [ "Case-Control Studies", "Chi-Square Distribution", "Clinical Clerkship", "Clinical Competence", "Education, Medical, Undergraduate", "Educational Measurement", "Educational Status", "Humans", "Illinois", "Licensure", "Patient Simulation", "Predictive Value of Tests", "Retrospective Studies", "Risk Factors", "Sensitivity and Specificity", "United States" ], "YEAR": "2011", "reasoning_required_pred": "yes", "reasoning_free_pred": "yes", "final_decision": "yes", "LONG_ANSWER": "Under the conditions of this study, preclerkship (year 2) CPEs strongly predicted medical students at risk for failing a senior CPE. This finding opens the opportunity for remediation of deficits prior to or during clerkships." }, "24964725": { "QUESTION": "Recovery Outcome Measures: Is There a Place for Culture, Attitudes, and Faith?", "CONTEXTS": [ "Utilization of the Recovery Knowledge Inventory (RKI) and Recovery Attitudes Questionnaire (RAQ) in southeastern Australia raised questions about the RAQ, including links between attitudes, faith, and culture in supporting the recovery journey. These questions are particularly important when considered in the context of people with mental illness who live in secular multicultural societies." ], "LABELS": [ "PURPOSE" ], "MESHES": [ "Australia", "Cultural Competency", "Health Knowledge, Attitudes, Practice", "Humans", "Mental Disorders", "Outcome Assessment (Health Care)", "Parish Nursing", "Psychiatric Nursing", "Surveys and Questionnaires" ], "YEAR": "2015", "reasoning_required_pred": "yes", "reasoning_free_pred": "yes", "final_decision": "yes", "LONG_ANSWER": "This paper discusses the cultural appropriateness of the RAQ in Australian settings, and identifies the need to develop rigorous, inclusive recovery outcome measures." }, "18708308": { "QUESTION": "Can surgeon familiarization with current evidence lead to a change in practice?", "CONTEXTS": [ "Despite evidence against its utility, many surgeons continue to employ prophylactic nasogastric decompression in elective colonic resection. This study aimed to establish whether an easy and practical intervention, mailing out a summary of current evidence to surgeons, can change surgeons practice to bring it more in line with current evidence.", "The use of prophylactic nasogastric (NG) decompression in elective colonic resections was documented for the 2 consecutive months of October and November, 2004 at the Royal Alexandra Hospital (RAH). A one page summary of recent evidence concerning this practice was then mailed to all general surgeons at that institution. A similar second review was carried out for the months of January and February, 2005. The two periods were compared with regards to prophylactic NG use.", "Twenty two patients underwent elective colonic resections during the months of October and November, 2004. Twenty one patients underwent such procedures in January and February, 2005. Seven out of the 22 cases in the first group (the pre-intervention block) received prophylactic NG decompression. Five out of the 21 cases in the second group (the post-intervention block) received prophylactic NG decompression. The difference in prophylactic NG use between the two groups was not statistically significant." ], "LABELS": [ "BACKGROUND", "METHODS", "RESULTS" ], "MESHES": [ "Aged", "Aged, 80 and over", "Alberta", "Attitude of Health Personnel", "Chi-Square Distribution", "Colectomy", "Elective Surgical Procedures", "Evidence-Based Medicine", "Female", "Follow-Up Studies", "Health Care Surveys", "Humans", "Intubation, Gastrointestinal", "Male", "Middle Aged", "Outcome Assessment (Health Care)", "Postoperative Complications", "Practice Patterns, Physicians'", "Probability", "Retrospective Studies", "Risk Assessment", "Statistics, Nonparametric", "Treatment Outcome" ], "YEAR": "2008", "reasoning_required_pred": "no", "reasoning_free_pred": "no", "final_decision": "no", "LONG_ANSWER": "This study has shown that mailing out a summary of current evidence to surgeons concerning a certain issue is not sufficient to lead to a change in practice." }, "11759976": { "QUESTION": "Advanced epithelial ovarian carcinoma in Thai women: should we continue to offer second-look laparotomy?", "CONTEXTS": [ "To determine survival among patients with epithelial ovarian carcinoma (EOC) who underwent a second-look laparotomy (SLL) and those refusing the procedure. Also to analyze factor(s) influencing the survival of the patients.", "Medical records were reviewed of patients with advanced EOC who were clinically free of disease after primary surgery and platinum-based chemotherapy between January 1, 1992, and December 31, 1998. All of them were offered SLL. Measurement outcomes include patient survival and disease-free survival.", "There were 50 patients with clinically complete remission after chemotherapy. Sixteen patients underwent SLL, and thirty-four patients refused the procedure (NSLL). Seven patients (43.8%) were reported to have positive SLL. After the median follow-up time of 35 months, 12 patients had died, and 5 patients were lost to follow-up. The median survival time for patients with SLL was about 60 months. Five-year survival rates of patients in the SLL, and NSLL groups were 37 per cent (95%CI = 7%-69%), and 88 per cent (95%CI = 65%-96%) respectively (P<0.001). The median time to relapse was about 25 months for patients with negative SLL. Five-year disease-free survival rates of patients in the negative SLL, and NSLL groups were 28 per cent (95%CI = 4%-59%), and 54 per cent (95%CI = 34%-70%) respectively (P=0.251). By Cox regression analysis, tumor grade was the only significant prognostic factor influencing patients' survival (HR = 6, 95%CI of HR = 1.2-34.2)." ], "LABELS": [ "OBJECTIVE", "METHOD AND MATERIAL", "RESULTS" ], "MESHES": [ "Chemotherapy, Adjuvant", "Disease-Free Survival", "Female", "Humans", "Incidence", "Neoplasm Recurrence, Local", "Ovarian Neoplasms", "Proportional Hazards Models", "Second-Look Surgery", "Survival Rate", "Thailand" ], "YEAR": "2001", "reasoning_required_pred": "maybe", "reasoning_free_pred": "no", "final_decision": "no", "LONG_ANSWER": "The second-look laparotomy doesn't have a favorable impact on overall and disease-free survival. Tumor grade is the only independent prognostic variable for survival of the patients." }, "16319544": { "QUESTION": "Is high-sensitivity C-reactive protein associated with carotid atherosclerosis in healthy Koreans?", "CONTEXTS": [ "There is a positive association between chronic inflammation and the risk of cardiovascular disease, but whether there is an association between C-reactive protein (CRP) and carotid atherosclerosis is controversial. We investigated the relationship between high-sensitivity CRP (hsCRP) levels and carotid intima-media thickness (IMT) in healthy Koreans.", "We measured hsCRP levels, the carotid IMT, and conventional cardiovascular risk factors including obesity parameters, blood pressure, lipid profiles, insulin resistance, and smoking habits in 820 volunteers (35-79 years old) in a cross-sectional study.", "Higher hsCRP quartile groups had higher mean IMTs, as compared with the lowest quartile (P<0.001 for the trend across quartiles). However, after adjustment for age, the relationship between hsCRP level and IMT was substantially weaker (P = 0.018). After additional adjustments for conventional cardiovascular risk factors, no significant association was observed (P = 0.548). The unadjusted risk for a high carotid IMT value (>or = 1.0 mm) was also positively related to hsCRP quartile, but this relationship was not significant after adjustment for age and other cardiovascular risk factors." ], "LABELS": [ "BACKGROUND", "DESIGN", "RESULTS" ], "MESHES": [ "Adult", "Aged", "Biomarkers", "C-Reactive Protein", "Carotid Artery Diseases", "Carotid Artery, Common", "Cross-Sectional Studies", "Female", "Humans", "Korea", "Male", "Middle Aged", "Nephelometry and Turbidimetry", "Prevalence", "Reference Values", "Retrospective Studies", "Risk Factors", "Tunica Intima", "Ultrasonography" ], "YEAR": "2005", "reasoning_required_pred": "no", "reasoning_free_pred": "no", "final_decision": "no", "LONG_ANSWER": "Both hsCRP levels and the carotid IMT were strongly correlated with conventional cardiovascular risk factors, but there was no independent association between hsCRP levels and carotid IMT in healthy Korean adults." }, "20674150": { "QUESTION": "Validation of the 2009 TNM version in a large multi-institutional cohort of patients treated for renal cell carcinoma: are further improvements needed?", "CONTEXTS": [ "A new edition of the TNM was recently released that includes modifications for the staging system of kidney cancers. Specifically, T2 cancers were subclassified into T2a and T2b (10 cm), tumors with renal vein involvement or perinephric fat involvement were classified as T3a cancers, and those with adrenal involvement were classified as T4 cancers.", "Our aim was to validate the recently released edition of the TNM staging system for primary tumor classification in kidney cancer.", "Our multicenter retrospective study consisted of 5339 patients treated in 16 academic Italian centers.", "Patients underwent either radical or partial nephrectomy.", "Univariable and multivariable Cox regression models addressed cancer-specific survival (CSS) after surgery.", "In the study, 1897 patients (35.5%) were classified as pT1a, 1453 (27%) as pT1b, 437 (8%) as pT2a, 153 (3%) as pT2b, 1059 (20%) as pT3a, 117 (2%) as pT3b, 26 (0.5%) as pT3c, and 197 (4%) as pT4. At a median follow-up of 42 mo, 786 (15%) had died of disease. In univariable analysis, patients with pT2b and pT3a tumors had similar CSS, as did patients with pT3c and pT4 tumors. Moreover, both pT3a and pT3b stages included patients with heterogeneous outcomes. In multivariable analysis, the novel classification of the primary tumor was a powerful independent predictor of CSS (p for trend<0.0001). However, the substratification of pT1 tumors did not retain an independent predictive role. The major limitations of the study are retrospective design, lack of central pathologic review, and the small number of patients included in some substages." ], "LABELS": [ "BACKGROUND", "OBJECTIVE", "DESIGN, SETTING, AND PARTICIPANTS", "INTERVENTION", "MEASUREMENTS", "RESULTS AND LIMITATIONS" ], "MESHES": [ "Aged", "Carcinoma, Renal Cell", "Cohort Studies", "Female", "Humans", "Kidney Neoplasms", "Male", "Middle Aged", "Neoplasm Staging", "Retrospective Studies" ], "YEAR": "2010", "reasoning_required_pred": "yes", "reasoning_free_pred": "yes", "final_decision": "yes", "LONG_ANSWER": "The recently released seventh edition of the primary tumor staging system for kidney tumors is a powerful predictor of CSS. However, some of the substages identified by the classification have overlapping prognoses, and other substages include patients with heterogeneous outcomes. The few modifications included in this edition may have not resolved the most critical issues in the previous version." }, "25336163": { "QUESTION": "Are interstitial fluid concentrations of meropenem equivalent to plasma concentrations in critically ill patients receiving continuous renal replacement therapy?", "CONTEXTS": [ "To describe the interstitial fluid (ISF) and plasma pharmacokinetics of meropenem in patients on continuous venovenous haemodiafiltration (CVVHDF).", "This was a prospective observational pharmacokinetic study. Meropenem (500 mg) was administered every 8 h. CVVHDF was targeted as a 2-3 L/h exchange using a polyacrylonitrile filter with a surface area of 1.05 m2 and a blood flow rate of 200 mL/min. Serial blood (pre- and post-filter), filtrate/dialysate and ISF concentrations were measured on 2 days of treatment (Profiles A and B). Subcutaneous tissue ISF concentrations were determined using microdialysis.", "A total of 384 samples were collected. During Profile A, the comparative median (IQR) ISF and plasma peak concentrations were 13.6 (12.0-16.8) and 40.7 (36.6-45.6) mg/L and the trough concentrations were 2.6 (2.4-3.4) and 4.9 (3.5-5.0) mg/L, respectively. During Profile B, the ISF trough concentrations increased by \u223c40%. Meropenem ISF penetration was estimated at 63% (60%-69%) and 69% (65%-74%) for Profiles A and B, respectively, using comparative plasma and ISF AUCs. For Profile A, the plasma elimination t1/2 was 3.7 (3.3-4.0) h, the volume of distribution was 0.35 (0.25-0.46) L/kg, the total clearance was 4.1 (4.1-4.8) L/h and the CVVHDF clearance was 2.9 (2.7-3.1) L/h." ], "LABELS": [ "OBJECTIVES", "PATIENTS AND METHODS", "RESULTS" ], "MESHES": [ "Aged", "Anti-Bacterial Agents", "Area Under Curve", "Critical Illness", "Extracellular Fluid", "Female", "Hemodiafiltration", "Humans", "Intensive Care Units", "Male", "Middle Aged", "Plasma", "Prospective Studies", "Renal Replacement Therapy", "Thienamycins" ], "YEAR": "2015", "reasoning_required_pred": "no", "reasoning_free_pred": "no", "final_decision": "no", "LONG_ANSWER": "This is the first known report of concurrent plasma and ISF concentrations of a meropenem antibiotic during CVVHDF. We observed that the ISF concentrations of meropenem were significantly lower than the plasma concentrations, although the present dose was appropriate for infections caused by intermediately susceptible pathogens (MIC\u22644 mg/L)." }, "25887165": { "QUESTION": "Does Sensation Return to the Nasal Tip After Microfat Grafting?", "CONTEXTS": [ "Patients usually complain about numbness in the nasal tip after microfat injections. The present study evaluated the severity of the numbness in the nasal tip after the procedure.", "To address the research question, a prospective study of young women was designed and performed at the Beijing Anzhen Hospital. Time was the primary predictor variable. The nasal tip sensation, which was evaluated using objective and subjective assessments, was used as the primary outcome variable. The McNemar-Bowker test (time vs nasal tip sensation) was used to detect statistical significance.", "A total of 30 young women (age 20.04 \u00b1 3.63 years) were recruited for the present study. The preoperative mean touch threshold value was 3.60 units. One week after the injection, the women experienced a decrease in the touch threshold value by 2.50 units. However, the sensation recovered gradually during the follow-up period (1.51 units at week 2, 2.39 units at week 4, 3.01 units at week 8, and 3.35 units at week 12). Significant differences were detected between multiple different measurement points (P<.05). The percentage of those experiencing paresthesia after the microfat injections also gradually diminished to none." ], "LABELS": [ "PURPOSE", "PATIENTS AND METHODS", "RESULTS" ], "MESHES": [ "Abdominal Fat", "Adipose Tissue", "Adult", "Cohort Studies", "Female", "Follow-Up Studies", "Humans", "Hypesthesia", "Middle Aged", "Nose", "Postoperative Complications", "Prospective Studies", "Recovery of Function", "Rhinoplasty", "Sensory Thresholds", "Touch", "Transplant Donor Site", "Young Adult" ], "YEAR": "2015", "reasoning_required_pred": "yes", "reasoning_free_pred": "yes", "final_decision": "yes", "LONG_ANSWER": "Postoperative numbness occurs in most patients receiving nasal microfat injections. Partial to complete recovery of nasal tip sensation can be expected to occur over a 3-month period." }, "12947068": { "QUESTION": "Do older men benefit from curative therapy of localized prostate cancer?", "CONTEXTS": [ "Prior decision-analytic models are based on outdated or suboptimal efficacy, patient preference, and comorbidity data. We estimated life expectancy (LE) and quality-adjusted life expectancy (QALE) associated with available treatments for localized prostate cancer in men aged>/= 65 years, adjusting for Gleason score, patient preferences, and comorbidity.", "We evaluated three treatments, using a decision-analytic Markov model: radical prostatectomy (RP), external beam radiotherapy (EBRT), and watchful waiting (WW). Rates of treatment complications and pretreatment incontinence and impotence were derived from published studies. We estimated treatment efficacy using three data sources: cancer registry cohort data, pooled case series, and modern radiotherapy studies. Utilities were obtained from 141 prostate cancer patients and from published studies.", "For men with well-differentiated tumors and few comorbidities, potentially curative therapy (RP or EBRT) prolonged LE up to age 75 years but did not improve QALE at any age. For moderately differentiated cancers, potentially curative therapy resulted in LE and QALE gains up to age 75 years. For poorly differentiated disease, potentially curative therapy resulted in LE and QALE gains up to age 80 years. Benefits of potentially curative therapy were restricted to men with no worse than mild comorbidity. When cohort and pooled case series data were used, RP was preferred over EBRT in all groups but was comparable to modern radiotherapy." ], "LABELS": [ "PURPOSE", "METHODS", "RESULTS" ], "MESHES": [ "Aged", "Aged, 80 and over", "Comorbidity", "Decision Support Techniques", "Humans", "Life Expectancy", "Male", "Markov Chains", "Postoperative Complications", "Prostatectomy", "Prostatic Neoplasms", "Quality of Life", "Radiotherapy", "Treatment Outcome" ], "YEAR": "2003", "reasoning_required_pred": "yes", "reasoning_free_pred": "yes", "final_decision": "yes", "LONG_ANSWER": "Potentially curative therapy results in significantly improved LE and QALE for older men with few comorbidities and moderately or poorly differentiated localized prostate cancer. Age should not be a barrier to treatment in this group." }, "16776337": { "QUESTION": "Pituitary apoplexy: do histological features influence the clinical presentation and outcome?", "CONTEXTS": [ "A retrospective analysis of a contemporary series of patients with pituitary apoplexy was performed to ascertain whether the histopathological features influence the clinical presentation or the outcome.", "A retrospective analysis was performed in 59 patients treated for pituitary apoplexy at the University of Virginia Health System, Charlottesville, Virginia, or Groote Schuur Hospital, University of Cape Town, South Africa. The patients were divided into two groups according to the histological features of their disease: one group with infarction alone, comprising 22 patients; and the other with hemorrhagic infarction and/or frank hemorrhage, comprising 37 patients. The presenting symptoms, clinical features, endocrinological status, and outcome were compared between the two groups." ], "LABELS": [ "OBJECT", "METHODS" ], "MESHES": [ "Adenoma", "Female", "Follow-Up Studies", "Hemorrhage", "Humans", "Infarction", "Male", "Middle Aged", "Pituitary Apoplexy", "Pituitary Gland", "Pituitary Neoplasms", "Retrospective Studies", "Treatment Outcome" ], "YEAR": "2006", "reasoning_required_pred": "yes", "reasoning_free_pred": "yes", "final_decision": "yes", "LONG_ANSWER": "The patients who presented with histological features of pituitary tumor infarction alone had less severe clinical features on presentation, a longer course prior to presentation, and a better outcome than those presenting with hemorrhagic infarction or frank hemorrhage. The endocrine replacement requirements were similar in both groups." }, "22668852": { "QUESTION": "Do African American women require fewer calories to maintain weight?", "CONTEXTS": [ "The high prevalence of obesity in African American (AA) women may result, in part, from a lower resting metabolic rate (RMR) than non-AA women. If true, AA women should require fewer calories than non-AA women to maintain weight. Our objective was to determine in the setting of a controlled feeding study, if AA women required fewer calories than non-AA women to maintain weight.", "This analysis includes 206 women (73% AA), aged 22-75 years, who participated in the Dietary Approaches to Stop Hypertension (DASH) trial-a multicenter, randomized, controlled, feeding study comparing the effects of 3 dietary patterns on blood pressure in individuals with prehypertension or stage 1 hypertension. After a 3-week run-in, participants were randomized to 1 of 3 dietary patterns for 8 weeks. Calorie intake was adjusted during feeding to maintain stable weight. The primary outcome of this analysis was average daily calorie (kcal) intake during feeding.", "AA women had higher baseline weight and body mass index than non-AA women (78.4 vs 72.4 kg, P<.01; 29.0 vs 27.6 kg/m(2), P<.05, respectively). During intervention feeding, mean (SD) kcal was 2168 (293) in AA women and 2073 (284) in non-AA women. Mean intake was 94.7 kcal higher in AA women than in non-AA women (P<.05). After adjustment for potential confounders, there was no difference in caloric intake between AA and non-AA women (\u0394 = -2.8 kcal, P = .95)." ], "LABELS": [ "BACKGROUND", "MATERIALS AND METHODS", "RESULTS" ], "MESHES": [ "Adult", "African Americans", "Aged", "Basal Metabolism", "Blood Pressure", "Body Composition", "Body Mass Index", "Body Weight", "Diet", "Energy Intake", "Female", "Humans", "Hypertension", "Middle Aged", "Obesity", "Young Adult" ], "YEAR": "2012", "reasoning_required_pred": "no", "reasoning_free_pred": "no", "final_decision": "no", "LONG_ANSWER": "These results do not support the view that AA women are at greater risk for obesity because they require fewer calories to maintain weight." }, "25251991": { "QUESTION": "Emergency double-balloon enteroscopy combined with real-time viewing of capsule endoscopy: a feasible combined approach in acute overt-obscure gastrointestinal bleeding?", "CONTEXTS": [ "There are few data concerning emergency double-balloon enteroscopy (DBE) and its usefulness in the management of severe acute obscure gastrointestinal bleeding (OGIB). The aim of this retrospective study was to evaluate emergency DBE and capsule endoscopy (CE) in patients with overt OGIB, analyzing the feasibility of this combined approach.", "Emergency DBE in patients with overt OGIB was defined as performance within 24\u2009h of symptom onset. We reported 27 patients (16 men, mean age: 64.6\u2009\u00b1\u200917.9 years) with overt severe bleeding who underwent 29 emergency DBE (22 anterograde, 7 retrograde). Of 27 patients, 16 (59.3%) underwent CE with real time (RT) viewing.", "Patients were diagnosed with the following: Dieulafoy's lesion (DL; n\u2009=\u200911, 40.7%), angioectasia (n\u2009=\u20097, 25.9%), tumors (n\u2009=\u20094, 14.8%), diverticulum (n\u2009=\u20093, 11.1%), ulcers (n\u2009=\u20092, 7.4%). We diagnosed 23 lesions amenable to endoscopic hemostasis and successfully treated 21 of them (77.8%). DL detection rate was statistically higher in the emergency DBE group than in OGIB patients with DBE done 24\u2009h after symptom onset (40.7% vs 0.9%, respectively, P\u2009<\u20090.001). Combined approach with RT viewing by CE correctly modified DBE management in four patients (25%)." ], "LABELS": [ "BACKGROUND AND AIM", "METHODS", "RESULTS" ], "MESHES": [ "Acute Disease", "Aged", "Aged, 80 and over", "Capsule Endoscopy", "Combined Modality Therapy", "Double-Balloon Enteroscopy", "Emergencies", "Feasibility Studies", "Female", "Follow-Up Studies", "Gastrointestinal Hemorrhage", "Humans", "Male", "Middle Aged", "Patient Safety", "Retrospective Studies", "Risk Assessment", "Severity of Illness Index", "Treatment Outcome" ], "YEAR": "2015", "reasoning_required_pred": "yes", "reasoning_free_pred": "yes", "final_decision": "yes", "LONG_ANSWER": "Emergency DBE is feasible, safe and effective in acute OGIB and may avoid major surgery, diagnosing and successfully treating most patients. Combined approach with RT viewing by CE is especially useful to identify recurrent bleeding vascular lesions such as DL that may be easily misdiagnosed by non-emergency DBE." }, "7497757": { "QUESTION": "Cardiopulmonary bypass temperature does not affect postoperative euthyroid sick syndrome?", "CONTEXTS": [ "To determine if temperature during cardiopulmonary bypass (CPB) has an effect on perioperative and postoperative thyroid function.", "Prospective study comparing thyroid function during and after hypothermic and normothermic CPB.", "Cardiac surgical unit at a university-affiliated hospital.", "Twelve patients scheduled to undergo cardiac operations with normothermic (n = 6) or hypothermic (n = 6) CPB.", "Blood was analyzed for serum concentration of total thyroxine (TT4), total triiodothyronine (TT3), free T3 (fT3), reverse T3 (rT3), and thyroid stimulating hormone (TSH) preoperatively, 60 min after CPB was initiated, 30 min after discontinuing CPB, and on postoperative days (POD) 1, 3, and 5.", "Patients who underwent either cold (26 degrees +/- 5 degrees C) or warm (35 degrees +/- 1 degree C) CPB were comparable with regard to age, body weight, duration of CPB, cross-clamp time, use of inotropes, total heparin dose, and length of hospital stay. Incidence of postoperative myocardial infarction, congestive heart failure, and death were similar. In both groups, TT4 and TT3 were reduced below baseline values beginning with CPB and persisting for up to 5 days after CPB (p<0.05), free T3 was reduced for up to 3 days after CPB (p<0.05), mean serum rT3 was elevated on POD 1 and POD 3 (p<0.05), and TSH remained unchanged." ], "LABELS": [ "STUDY OBJECTIVE", "DESIGN", "SETTING", "PATIENTS", "INTERVENTIONS", "MEASUREMENTS AND RESULTS" ], "MESHES": [ "Body Temperature", "Cardiac Surgical Procedures", "Cardiopulmonary Bypass", "Euthyroid Sick Syndromes", "Humans", "Middle Aged", "Postoperative Complications", "Thyrotropin", "Thyroxine", "Triiodothyronine", "Triiodothyronine, Reverse" ], "YEAR": "1995", "reasoning_required_pred": "maybe", "reasoning_free_pred": "no", "final_decision": "no", "LONG_ANSWER": "The results of this study suggest that normothermic CPB does not prevent the development of the \"euthyroid sick syndrome\" during and after CPB. Despite these changes in thyroid function, most patients in both groups had a normal postoperative recovery." }, "8262881": { "QUESTION": "Body dysmorphic disorder: does it have a psychotic subtype?", "CONTEXTS": [ "Although body dysmorphic disorder (BDD) is classified in DSM-III-R as a nonpsychotic somatoform disorder, controversy exists as to whether BDD can present with psychotic features. If it can, this raises the possibility that its DSM-III-R psychotic counterpart-delusional disorder, somatic type--may not be a separate disorder. The purpose of this study was to determine whether patients with nonpsychotic BDD (defined according to DSM-III-R criteria, i.e., with maintenance of some insight) were different from patients with psychotic BDD (those whose preoccupation was without insight and of delusional intensity).", "Fifty consecutive patients meeting DSM-III-R criteria A and C for BDD were assessed with a semistructured interview and the Structured Clinical Interview for DSM-III-R (SCID). Family histories of psychiatric disorders were blindly assessed. The 24 patients with nonpsychotic BDD were compared with the 26 patients with psychotic BDD with respect to demographics, phenomenology, course of illness, associated features, comorbid psychiatric disorders, family history, and treatment response.", "Patients with psychotic BDD displayed a significantly higher rate of lifetime DSM-III-R psychotic disorder diagnoses than patients with nonpsychotic BDD. However, the two groups did not differ significantly on most other variables examined. For instance, both psychotic and nonpsychotic patients displayed significant morbidity; high comorbidity with mood, anxiety, and psychoactive substance use disorders; and apparent preferential response to serotonin reuptake inhibitors rather than to non-serotonin reuptake blocking antidepressants or antipsychotics." ], "LABELS": [ "BACKGROUND", "METHOD", "RESULTS" ], "MESHES": [ "Adult", "Aged", "Body Image", "Comorbidity", "Delusions", "Diagnosis, Differential", "Female", "Humans", "Male", "Mental Disorders", "Middle Aged", "Psychiatric Status Rating Scales", "Somatoform Disorders", "Terminology as Topic", "Treatment Outcome" ], "YEAR": "1993", "reasoning_required_pred": "no", "reasoning_free_pred": "yes", "final_decision": "yes", "LONG_ANSWER": "Body dysmorphic disorder may have a closely related psychotic subtype that significantly overlaps with, or may even be the same disorder as, the BDD variant of delusional disorder, somatic type. Inclusion of a psychotic subtype for BDD should be considered for future editions of DSM." }, "11943048": { "QUESTION": "Does receipt of hospice care in nursing homes improve the management of pain at the end of life?", "CONTEXTS": [ "To compare analgesic management of daily pain for dying nursing home residents enrolled and not enrolled in Medicare hospice.", "Retrospective, comparative cohort study.", "Over 800 nursing homes in Kansas, Maine, Mississippi, New York, and South Dakota.", "A subset of residents with daily pain near the end of life taken from a matched cohort of hospice (2,644) and nonhospice (7,929) nursing home residents who had at least two resident assessments (Minimum Data Sets (MDSs)) completed, their last between 1992 and 1996, and who died before April 1997. The daily pain subset consisted of 709 hospice and 1,326 nonhospice residents.", "Detailed drug use data contained on the last MDS before death were used to examine analgesic management of daily pain. Guidelines from the American Medical Directors Association (AMDA) were used to identify analgesics not recommended for use in managing chronic pain in long-term care settings. The study outcome, regular treatment of daily pain, examined whether patients received any analgesic, other than those not recommended by AMDA, at least twice a day for each day of documented daily pain (i.e., 7 days before date of last MDS).", "Fifteen percent of hospice residents and 23% of nonhospice residents in daily pain received no analgesics (odds ratio (OR) = 0.57, 95% confidence interval (CI) = 0.45-0.74). A lower proportion of hospice residents (21%) than of nonhospice residents (29%) received analgesics not recommended by AMDA (OR = 0.65, 95% CI =0.52-0.80). Overall, acetaminophen (not in combination with other drugs) was used most frequently for nonhospice residents (25% of 1,673 prescriptions), whereas morphine derivatives were used most frequently for hospice residents (30% of 1,058 prescriptions). Fifty-one percent of hospice residents and 33% of nonhospice residents received regular treatment for daily pain. Controlling for clinical confounders, hospice residents were twice as likely as nonhospice residents to receive regular treatment for daily pain (adjusted odds ratio = 2.08, 95% CI = 1.68-2.56)." ], "LABELS": [ "OBJECTIVES", "DESIGN", "SETTING", "PARTICIPANTS", "MEASUREMENTS", "RESULTS" ], "MESHES": [ "Aged", "Analgesics", "Cohort Studies", "Drug Utilization", "Female", "Homes for the Aged", "Hospice Care", "Humans", "Male", "Nursing Homes", "Pain", "Retrospective Studies" ], "YEAR": "2002", "reasoning_required_pred": "yes", "reasoning_free_pred": "yes", "final_decision": "yes", "LONG_ANSWER": "Findings suggest that analgesic management of daily pain is better for nursing home residents enrolled in hospice than for those not enrolled in hospice.The prescribing practices portrayed by this study reveal that many dying nursing home residents in daily pain are receiving no analgesic treatment or are receiving analgesic treatment inconsistent with AMDA and other pain management guidelines. Improving the analgesic management of pain in nursing homes is essential if high-quality end-of-life care in nursing homes is to be achieved." }, "20306735": { "QUESTION": "Fulfilling human resources development goal in West Africa: can the training of ophthalmologist diplomates be improved?", "CONTEXTS": [ "To ascertain the perspectives of Trainee Ophthalmologist Diplomats (TOD) on the Ophthalmic Diploma Training (ODT) in West Africa with a view to improving the programme.", "A survey of set 2005 TOD on ODT was carried out in Ghana, 2006.", "The trainees included 10 (83.35%) males and two (16.7%) females whose ages ranged between thirty-two and fifty-one years. The sponsors of the trainees included Sight Savers International, five (41.7%); Christian Blind Mission International, three (25.0%); Eye Foundation, Lagos, Nigeria two (16.7%); Ministry of Defence Nigeria, one (8.3%); and Health Authority Ghana, one (8.3%). Nine trainees (75.0%) felt the programme was well structured, training allowances were adequate eight (66.7%) and inadequate four (33.3%). Eleven (91.7%) trainees would work wherever they were posted; ten (83.3%) trainees had sense of fulfillment and three (25%) would like to proceed for residency training. All trainees were at least good in chalazion surgery and treatment of common medical eye conditions. Majority were at least good in eye surgery like cataract, eleven (91.7%); trabeculectomy nine (75.0%); pterygium 10 (83.3%); eyelid, eight (66.7%); destructive 11 (91.6%) and refraction 9 (75.0%). Some trainees' perceived problems included inadequate sponsorship (33.3%), short duration of the course four (33.3%) and poor accommodation facility two (16.7%). However, trainees' suggested increase in training posts, four (33.3); training allowance three (25.0%); and incentives for trainers/training hospitals two (16.7%)." ], "LABELS": [ "OBJECTIVE", "METHODS", "RESULTS" ], "MESHES": [ "Clinical Competence", "Education, Medical, Continuing", "Female", "Ghana", "Health Manpower", "Humans", "Male", "Medical Staff, Hospital", "Nigeria", "Ophthalmology", "Staff Development", "Surveys and Questionnaires" ], "YEAR": "2009", "reasoning_required_pred": "yes", "reasoning_free_pred": "yes", "final_decision": "yes", "LONG_ANSWER": "The ODT programme ensures ophthalmic manpower for secondary eye care level despite challenges. The stakeholders should look into problems facing the programme in order to improve it." }, "23414523": { "QUESTION": "Does binge drinking during early pregnancy increase the risk of psychomotor deficits?", "CONTEXTS": [ "The potential effects of binge drinking during pregnancy on child motor function have only been assessed in a few, small studies. We aimed to examine the effects of binge alcohol consumption during early pregnancy, including number of binge episodes and timing of binge drinking, on child motor function at age 5.", "We performed a prospective follow-up study of 678 women and their children sampled from the Danish National Birth Cohort based on maternal alcohol consumption during pregnancy. At 5 years of age, the children were tested with the Movement Assessment Battery for Children. Parental education, maternal IQ, prenatal maternal smoking, the child's age at testing, sex of child, and tester were considered core confounders, while the full model also controlled for prenatal maternal average alcohol intake, maternal age and prepregnancy body mass index, parity, home environment, postnatal parental smoking, health status, participation in organized sport, and indicators for hearing and vision impairment.", "There were no systematic or significant differences in motor function between children of mothers reporting isolated episodes of binge drinking and children of mothers with no binge episodes. No association was observed with respect to the number of binge episodes (maximum of 12) and timing of binge drinking." ], "LABELS": [ "BACKGROUND", "METHODS", "RESULTS" ], "MESHES": [ "Adult", "Alcohol Drinking", "Binge Drinking", "Child, Preschool", "Denmark", "Female", "Follow-Up Studies", "Humans", "Male", "Pregnancy", "Pregnancy Trimester, First", "Prenatal Exposure Delayed Effects", "Prospective Studies", "Psychomotor Disorders", "Risk Factors" ], "YEAR": "2013", "reasoning_required_pred": "no", "reasoning_free_pred": "no", "final_decision": "no", "LONG_ANSWER": "In this study, we found no systematic association between isolated episodes of binge drinking during early pregnancy and child motor function at age 5." }, "11882828": { "QUESTION": "Does tranexamic acid reduce desmopressin-induced hyperfibrinolysis?", "CONTEXTS": [ "Desmopressin releases tissue-type plasminogen activator, which augments cardiopulmonary bypass--associated hyperfibrinolysis, causing excessive bleeding. Combined use of desmopressin with prior administration of the antifibrinolytic drug tranexamic acid may decrease fibrinolytic activity and might improve postoperative hemostasis.", "This prospective randomized study was carried out with 100 patients undergoing coronary artery bypass operations between April 1999 and November 2000 in G\u00fclhane Military Medical Academy. Patients were divided into 2 groups. Desmopressin (0.3 microg/kg) was administrated just after cardiopulmonary bypass and after protamine infusion in group 1 (n = 50). Both desmopressin and tranexamic acid (before the skin incision at a loading dose of 10 mg/kg over 30 minutes and followed by 12 hours of 1 mg.kg(-1).h(-1)) were administrated in group 2 (n = 50).", "Significantly less drainage was noted in group 2 (1010 +/- 49.9 mL vs 623 +/- 41.3 mL, P =.0001). Packed red blood cells were transfused at 2.1 +/- 0.5 units per patient in group 1 versus 0.9 +/- 0.3 units in group 2 (P =.0001). Fresh frozen plasma was transfused at 1.84 +/- 0.17 units per patient in group 1 versus 0.76 +/- 0.14 units in group 2 (P =.0001). Only 24% of patients in group 2 required donor blood or blood products compared with 74% of those in the isolated desmopressin group (group 1, P =.00001). Group 1 and group 2 findings were as follows: postoperative fibrinogen, 113 +/- 56.3 mg/dL versus 167 +/- 45.8 mg/dL (P =.0001); fibrin split product, 21.2 +/- 2.3 ng/mL versus 13.5 +/- 3.4 ng/mL (P =.0001); and postoperative hemoglobin level, 7.6 plus minus 1.2 g/dL versus 9.1 plus minus 1.2 g/dL (P =.0001)." ], "LABELS": [ "OBJECTIVE", "METHODS", "RESULTS" ], "MESHES": [ "Aged", "Antifibrinolytic Agents", "Blood Loss, Surgical", "Blood Transfusion", "Cardiopulmonary Bypass", "Coronary Artery Bypass", "Deamino Arginine Vasopressin", "Drug Interactions", "Female", "Fibrinolysis", "Hemostasis, Surgical", "Hemostatics", "Humans", "Male", "Middle Aged", "Prospective Studies", "Tranexamic Acid" ], "YEAR": "2002", "reasoning_required_pred": "yes", "reasoning_free_pred": "yes", "final_decision": "yes", "LONG_ANSWER": "Tranexamic acid administration significantly reduces desmopressin and bypass-induced hyperfibrinolysis. Combined use of tranexamic acid and desmopressin decreases both postoperative blood loss and transfusion requirement." }, "24939676": { "QUESTION": "Does ultrasound-scored synovitis depend on the pharmacokinetics of subcutaneous anti-TNF agents in patients with rheumatoid arthritis?", "CONTEXTS": [ "The aim of this study was to investigate the influence of the pharmacokinetics of s.c. anti-TNF agents on the grade of US-detected synovitis in RA patients.", "Fifty RA patients were prospectively recruited from the Biologic Therapy Unit of our hospital. Inclusion criteria were being in treatment with s.c. anti-TNF agents and having had neither changes in therapy nor local corticosteroid injections in the previous 3 months. Patients underwent clinical, laboratory [28-joint DAS (DAS28) and Simplified Disease Activity Index (SDAI)]and US assessment at two time points, i.e. at peak plasma drug concentration and at trough plasma drug concentration. US assessments were performed blindly to the anti-TNF agent, the administration time and the clinical and laboratory data. Twenty-eight joints were investigated for the presence and grade (0-3) of B-mode synovitis and synovial power Doppler signal. Global indices for B-mode synovitis (BSI) and Doppler synovitis (DSI) were calculated for 12 joints and for wrist-hand-ankle-foot joints. B-mode US remission was defined as a BSI<1 and Doppler US remission as a DSI<1.", "There were no significant differences between the clinical, laboratory and B-mode and Doppler US parameters at peak time and trough time (P = 0.132-0.986). There were no significant differences between the proportion of patients with active disease and those in remission according to DAS28, SDAI, B-mode US and Doppler US at peak time and trough time assessments (P = 0.070-1)." ], "LABELS": [ "OBJECTIVE", "METHODS", "RESULTS" ], "MESHES": [ "Adult", "Aged", "Aged, 80 and over", "Antirheumatic Agents", "Arthritis, Rheumatoid", "Female", "Follow-Up Studies", "Humans", "Male", "Middle Aged", "Prognosis", "Prospective Studies", "Severity of Illness Index", "Synovitis", "Tumor Necrosis Factor-alpha", "Ultrasonography, Doppler", "Young Adult" ], "YEAR": "2014", "reasoning_required_pred": "no", "reasoning_free_pred": "no", "final_decision": "no", "LONG_ANSWER": "Our results suggested that s.c. anti-TNF pharmacokinetics do not significantly influence US-scored synovitis in RA patients." }, "17076590": { "QUESTION": "Counter sampling combined with medical provider education: do they alter prescribing behavior?", "CONTEXTS": [ "To observe if medical providers alter their prescribing patterns of three relatively expensive categories of medications provided as samples by manufacturers (focus medications) when they receive additional education from pharmacists concerning the appropriate use of lower cost alternatives (counter samples) that are made available to dispense.", "Pretest, post-test with a control group.", "Two rural, private care clinics in southeastern Idaho providing immediate care services.", "Eight medical providers at a clinic where interventions were employed (active intervention group) and seven medical providers in a clinic where no interventions occurred (control group).", "Medical providers in the active intervention group had: 1) education from pharmacists concerning the appropriate use of lower-cost alternatives compared with expensive focus medications 2) counter samples and patient sample handouts available to dispense to patients at their own discretion.", "The percentage of the total yearly prescriptions for nonsteroidal anti-inflammatory drugs (NSAIDs), antihistamines, and acid-relief medications that consisted of focus-COX-2 NSAIDs, nonsedating antihistamines, and proton pump inhibitors (PPIs), respectively.", "The prescribing behavior of medical providers in the active intervention and control groups were significantly different at baseline in all three categories of focus medications. This suggested that the results should focus on changes across the two years of the study within the intervention and control groups rather than across the two groups. Medical providers in the intervention group significantly decreased the use of COX-2 NSAID prescriptions relative to total NSAID prescriptions following active intervention (38.9% in year 1 versus 23.7% in year 2, P<0.05). Over the same two time periods, a nonstatistically significant decrease in COX-2 NSAID prescribing was seen at the control site (67.5% versus 62%, P>0.05). Education and counter sampling did not stop medical providers from significantly increasing the total yearly prescriptions for antihistamines and acid-relief medications that consisted of focus-nonsedating antihistamines (86.7% versus 93.1%, P<0.05) and PPIs (68.9% versus 86.2%, P<0.05). Statistically significant increases in the prescribing of focus-nonsedating antihistamines (77.9% versus 98.3%, P<0.05) and PPIs (77.5% versus 91.4%, P<0.05) were also observed in the control group." ], "LABELS": [ "OBJECTIVE", "DESIGN", "SETTING", "PARTICIPANTS", "INTERVENTIONS", "MAIN OUTCOME MEASURES", "RESULTS" ], "MESHES": [ "Anti-Inflammatory Agents, Non-Steroidal", "Cost Savings", "Drug Costs", "Drug Utilization", "Education, Medical, Continuing", "Histamine H1 Antagonists, Non-Sedating", "Humans", "Pharmaceutical Services", "Practice Patterns, Physicians'", "Proton Pump Inhibitors", "Rural Health Services" ], "YEAR": "2006", "reasoning_required_pred": "yes", "reasoning_free_pred": "maybe", "final_decision": "maybe", "LONG_ANSWER": "Education by pharmacists, combined with access to counter samples, may or may not have an effect on medical provider prescribing, depending on the category of medication targeted for cost control." }, "21745056": { "QUESTION": "Global Longitudinal Pathway: has medical education curriculum influenced medical students' skills and attitudes toward culturally diverse populations?", "CONTEXTS": [ "The Pathway represents a longitudinal program for medical students, consisting of both domestic and international experiences with poor populations. A previous study reported no significant attitudinal changes toward the medically indigent between Pathway and non-Pathway students.", "The purpose of this study was to investigate and differentiate the skills and attitudes of Pathway and non-Pathway students in working with culturally diverse populations by conducting quantitative and qualitative analyses.", "Selected items from a cultural assessment were analyzed using independent t-tests and a proportional analysis using approximation of the binomial distribution. In addition, a qualitative assessment of non-Pathway and Pathway students was conducted.", "A statistically significant difference was found at the end of Years 2, 3, and 4 regarding student confidence ratings, and qualitative results had similar findings." ], "LABELS": [ "BACKGROUND", "PURPOSE", "METHODS", "RESULTS" ], "MESHES": [ "Attitude", "Clinical Competence", "Cultural Competency", "Cultural Diversity", "Curriculum", "Education, Medical", "Female", "Focus Groups", "Humans", "Male", "Massachusetts" ], "YEAR": null, "reasoning_required_pred": "yes", "reasoning_free_pred": "yes", "final_decision": "yes", "LONG_ANSWER": "Clear and distinct differences between the two studied groups were found indicating the root of this increased confidence may have developed due to exposure to the Pathway program." }, "22979954": { "QUESTION": "Failed IUD insertions in community practice: an under-recognized problem?", "CONTEXTS": [ "The data analysis was conducted to describe the rate of unsuccessful copper T380A intrauterine device (IUD) insertions among women using the IUD for emergency contraception (EC) at community family planning clinics in Utah.", "These data were obtained from a prospective observational trial of women choosing the copper T380A IUD for EC. Insertions were performed by nurse practitioners at two family planning clinics in order to generalize findings to the type of service setting most likely to employ this intervention. Adjuvant measures to facilitate difficult IUD insertions (cervical anesthesia, dilation, pain medication, and use of ultrasound guidance) were not utilized. The effect of parity on IUD insertion success was determined using exact logistic regression models adjusted for individual practitioner failure rates.", "Six providers performed 197 IUD insertion attempts. These providers had a mean of 14.1 years of experience (range 1-27, S.D. \u00b112.5). Among nulliparous women, 27 of 138 (19.6%) IUD insertions were unsuccessful. In parous women, 8 of 59 IUD insertions were unsuccessful (13.6%). The adjusted odds ratio (aOR) showed that IUD insertion failure was more likely in nulliparous women compared to parous women (aOR=2.31, 95% CI 0.90-6.52, p=.09)." ], "LABELS": [ "BACKGROUND", "METHODS", "RESULTS" ], "MESHES": [ "Adolescent", "Adult", "Ambulatory Care Facilities", "Community Health Services", "Contraception, Postcoital", "Equipment Failure", "Female", "Humans", "Intrauterine Devices, Copper", "Nurse Practitioners", "Odds Ratio", "Parity", "Pregnancy", "Prospective Studies", "Treatment Failure", "Utah", "Young Adult" ], "YEAR": "2013", "reasoning_required_pred": "yes", "reasoning_free_pred": "yes", "final_decision": "yes", "LONG_ANSWER": "The high rate of unsuccessful IUD insertions reported here, particularly for nulliparous women, suggests that the true insertion failure rate of providers who are not employing additional tools for difficult insertions may be much higher than reported in clinical trials. Further investigation is necessary to determine if this is a common problem and, if so, to assess if the use of adjuvant measures will reduce the number of unsuccessful IUD insertions." }, "22955530": { "QUESTION": "Type II supracondylar humerus fractures: can some be treated nonoperatively?", "CONTEXTS": [ "The range of injury severity that can be seen within the category of type II supracondylar humerus fractures (SCHFs) raises the question whether some could be treated nonoperatively. However, the clinical difficulty in using this approach lies in determining which type II SCHFs can be managed successfully without a surgical intervention.", "We reviewed clinical and radiographic information on 259 pediatric type II SCHFs that were enrolled in a prospective registry of elbow fractures. The characteristics of the patients who were treated without surgery were compared with those of patients who were treated surgically. Treatment outcomes, as assessed by the final clinical and radiographic alignment, range of motion of the elbow, and complications, were compared between the groups to define clinical and radiographic features that related to success or failure of nonoperative management.", "During the course of treatment, 39 fractures were found to have unsatisfactory alignment with nonoperative management and were taken for surgery. Ultimately, 150 fractures (57.9%) were treated nonoperatively, and 109 fractures (42.1%) were treated surgically. At final follow-up, outcome measures of change in carrying angle, range of motion, and complications did not show clinically significant differences between treatment groups. Fractures without rotational deformity or coronal angulation and with a shaft-condylar angle of>15 degrees were more likely to be associated with successful nonsurgical treatment. A scoring system was developed using these features to stratify the severity of the injury. Patients with isolated extension deformity, but none of the other features, were more likely to complete successful nonoperative management." ], "LABELS": [ "BACKGROUND", "METHODS", "RESULTS" ], "MESHES": [ "Adolescent", "Algorithms", "Child", "Child, Preschool", "Female", "Follow-Up Studies", "Humans", "Humeral Fractures", "Infant", "Male", "Orthopedic Procedures", "Range of Motion, Articular", "Registries", "Retrospective Studies", "Trauma Severity Indices", "Treatment Outcome" ], "YEAR": null, "reasoning_required_pred": "yes", "reasoning_free_pred": "yes", "final_decision": "yes", "LONG_ANSWER": "This study suggests that some of the less severe pediatric type II SCHFs can be successfully treated without surgery if close follow-up is achieved. Fractures with initial rotational deformity, coronal malalignment, and significant extension of the distal fragment are likely to fail a nonoperative approach. An algorithm using the initial radiographic characteristics can aid in distinguishing groups." }, "18507507": { "QUESTION": "The promise of specialty pharmaceuticals: are they worth the price?", "CONTEXTS": [ "Specialty pharmaceuticals have evolved beyond their status as niche drugs designed to treat rare conditions and are now poised to become the standard of care in a wide variety of common chronic illnesses. Due in part to the cost of these therapies, payers are increasingly demanding evidence of their value. Determining the value of these medications is hampered by a lack of robust pharmacoeconomic data.", "To outline emerging strategies and case study examples for the medical and pharmacy benefits management of specialty pharmaceuticals.", "The promise of specialty pharmaceuticals: increased life expectancy, improved quality of life, enhanced workplace productivity, decreased burden of disease, and reduced health care spending comes at a significant cost. These agents require special handling, administration, patient education, clinical support, and risk mitigation. Additionally, specialty drugs require distribution systems that ensure appropriate patient selection and data collection. With the specialty pharmaceutical pipeline overflowing with new medicines and an aging population increasingly relying on these novel treatments to treat common diseases, the challenge of managing the costs associated with these agents can be daunting. Aided by sophisticated pharmacoeconomic models to assess value, the cost impacts of these specialty drugs can be appropriately controlled." ], "LABELS": [ "BACKGROUND", "OBJECTIVE", "SUMMARY" ], "MESHES": [ "Chronic Disease", "Drug Costs", "Humans", "Managed Care Programs", "Pharmaceutical Preparations", "Rare Diseases", "United States" ], "YEAR": "2008", "reasoning_required_pred": "maybe", "reasoning_free_pred": "yes", "final_decision": "yes", "LONG_ANSWER": "Current evidence suggests that when used in targeted patient populations, specialty pharmaceuticals may represent a good health care value." }, "17323047": { "QUESTION": "Does invasive diagnosis of nosocomial pneumonia during off-hours delay treatment?", "CONTEXTS": [ "We examined whether invasive lung-specimen collection-to-treatment times for intensive care unit patients with suspected ventilator-associated pneumonia (VAP) differ with to the work shift during which specimens were collected. We compared weekday day shifts and off-hours (from 6:30 p.m. to 8:29 a.m. the next day for night shifts, from Saturday 1:00 p.m. to Monday 8:29 a.m. for weekends, and from 8:30 a.m. to 8:29 a.m. the following morning for public holidays).", "Single-center, observational study in the intensive care unit in an academic teaching hospital.", "101 patients who developed 152 episodes of bacteriologically confirmed VAP.", "Of the 152 VAP episodes 66 were diagnosed during off-hours. Neither more bronchoscopy complications nor more inappropriate initial antimicrobial treatments for patients were observed between day and off-hour shifts. Indeed, the overall time from brochoalveolar lavage to antibiotic administration was shorter for off-hours than day-shifts due to shorter specimen collection-to-antibiotic prescription times, but antibiotic prescription-to-administration times were the same." ], "LABELS": [ "OBJECTIVE", "DESIGN AND SETTING", "PATIENTS AND PARTICIPANTS", "MEASUREMENTS AND RESULTS" ], "MESHES": [ "Anti-Bacterial Agents", "Bronchoalveolar Lavage Fluid", "Bronchoscopy", "Cross Infection", "Humans", "Intensive Care Units", "Pneumonia, Ventilator-Associated", "Time Factors" ], "YEAR": "2007", "reasoning_required_pred": "no", "reasoning_free_pred": "no", "final_decision": "no", "LONG_ANSWER": "An invasive strategy based on bronchoscopy to diagnose VAP was not associated with a longer time to first appropriate antibiotic administration when clinical suspicion of VAP occurs during off-hours." }, "24851767": { "QUESTION": "Undescended testes: does age at orchiopexy affect survival of the testis?", "CONTEXTS": [ "The optimal age at which to perform orchiopexy for cryptorchidism has long been debated. The aim of this study was to determine if age at orchiopexy affected testicular atrophy.", "A retrospective review of patients undergoing orchiopexy from 2000 to 2010 was conducted. An individual testis, rather than patient, was used as the dependent variable. A total of 349 testicles from 1126 charts (ICD-9=752.51) were identified. Primary study outcome was testicular survival without atrophy.", "Mean follow up for the study was 25 months. There was postoperative atrophy in 27 testes (7.7%). Intraabdominal testicle was independently associated with increased postsurgical atrophy (p<0.0001). The odds of postsurgical atrophy were 15.66 times higher for an abdominal vs. inguinal location (95% CI: 5.5-44.6). Testicular atrophy was highest for orchiopexy at ages 13-24 months (n=16 of 133, 12%) vs. those less than 13 months (n=3 of 64, 5%), and those greater than 24 months (n=8 of 152, 5%) (p=0.0024). After adjusting for location, age was not statistically significant with postsurgical atrophy (p=0.055)." ], "LABELS": [ "PURPOSE", "METHODS", "RESULTS" ], "MESHES": [ "Age Factors", "Atrophy", "Child, Preschool", "Cryptorchidism", "Follow-Up Studies", "Humans", "Infant", "Male", "Orchiopexy", "Postoperative Complications", "Retrospective Studies", "Risk Factors" ], "YEAR": "2014", "reasoning_required_pred": "no", "reasoning_free_pred": "no", "final_decision": "no", "LONG_ANSWER": "From this study we conclude that there is no increase in testicular atrophy in patients less than 13 months." }, "12006913": { "QUESTION": "Do lipids, blood pressure, diabetes, and smoking confer equal risk of myocardial infarction in women as in men?", "CONTEXTS": [ "Studies on coronary risk factors in men and women are mainly based on mortality data and few compare results of both sexes with consistent study design and diagnostic criteria. This study assesses the major risk factors for coronary events in men and women from the Reykjavik Study.", "Within a prospective, population-based cohort study individuals without history of myocardial infarction were identified and the relative risk of baseline variables was assessed in relation to verified myocardial infarction or coronary death during follow-up.", "Of the 9681 women and 8888 men who attended risk assessment from 1967-1991, with follow-up period of up to 28 years, 706 women and 1700 men suffered a non-fatal myocardial infarction or coronary death.", "Serum cholesterol was a significant risk factor for both sexes, with hazard ratios (HR) decreasing with age. Systolic blood pressure was a stronger risk factor for women as was ECG-confirmed left ventricular hypertrophy (women HR 2.89, 95% confidence interval [CI] 1.67-5.01; men HR 1.11 [CI 0.86-1.43]). Fasting blood glucose>or =6.7 mmol/L identified significantly higher risk for women (HR 2.65) than men (HR 2.08) as did self-reported diabetes. Triglyceride risk was significantly higher for women and decreased significantly with age. Smoking increased risk two- to five-fold, increasing with dose, for women, which was significantly higher than the doubling in risk for men." ], "LABELS": [ "BACKGROUND", "DESIGN", "METHODS", "RESULTS" ], "MESHES": [ "Adult", "Age Factors", "Biomarkers", "Blood Glucose", "Blood Pressure", "Cohort Studies", "Diabetes Complications", "Diabetes Mellitus", "Electrocardiography", "Endpoint Determination", "Female", "Follow-Up Studies", "Humans", "Hypertrophy, Left Ventricular", "Iceland", "Lipids", "Male", "Middle Aged", "Multivariate Analysis", "Myocardial Infarction", "Prospective Studies", "Risk Factors", "Sex Factors", "Smoking", "Women's Health" ], "YEAR": "2002", "reasoning_required_pred": "no", "reasoning_free_pred": "no", "final_decision": "no", "LONG_ANSWER": "This large study of the major risk factors compared between the sexes demonstrates similar relative risk of myocardial infarction associated with cholesterol for both sexes, however, the relative risk is higher in women for many other risk factors such as smoking, diabetes, elevated triglycerides and left ventricular hypertrophy." }, "8245806": { "QUESTION": "Does family practice at residency teaching sites reflect community practice?", "CONTEXTS": [ "Family medicine has aspired to train residents and conduct research in settings that closely resemble community practice. The purpose of this study was to compare the patient characteristics of the ambulatory teaching centers of a consortium of seven community-based university-affiliated family practice residency programs in northeast Ohio with the National Ambulatory Medical Care Survey (NAMCS) results for family physicians (FPs) and general practitioners (GPs).", "Ninety-eight faculty and resident physicians at the residency training site of the Northeastern Ohio Universities College of Medicine collected data on all ambulatory patient visits (N = 1498) for one randomly chosen week between July 1, 1991, and June 30, 1992. We compared these data with patient visits reported in the 1990 NAMCS for FPs and GPs.", "The residency training sites saw slightly more children, women, blacks, and Medicare and Medicaid patients. The most common reason for an office visit in both populations was an undifferentiated symptom. Fifteen of the top 20 \"reason for visit\" codes were identical, as were 14 of the top 20 diagnoses. More preventive and therapeutic services were offered or performed at our residency training sites but fewer diagnostic services were performed. There were fewer consultations requested at our residency training sites but similar hospitalization rates for patients. The mean duration of visit differed by only 1 minute." ], "LABELS": [ "BACKGROUND", "METHODS", "RESULTS" ], "MESHES": [ "Adolescent", "Adult", "Aged", "Ambulatory Care", "Child", "Child, Preschool", "Family Practice", "Female", "Humans", "Internship and Residency", "Male", "Middle Aged", "Office Visits", "Ohio", "Patients", "Reimbursement Mechanisms", "Time Factors" ], "YEAR": "1993", "reasoning_required_pred": "yes", "reasoning_free_pred": "yes", "final_decision": "yes", "LONG_ANSWER": "The residency training sites of the Northeastern Ohio Universities College of Medicine provide patient care opportunities similar to those found in a national survey of family and general practitioners." }, "17691856": { "QUESTION": "Midwives' competence: is it affected by working in a rural location?", "CONTEXTS": [ "Rising health care costs and the need to consolidate expertise in tertiary services have led to the centralisation of services. In the UK, the result has been that many rural maternity units have become midwife-led. A key consideration is that midwives have the skills to competently and confidently provide maternity services in rural areas, which may be geographically isolated and where the midwife may only see a small number of pregnant women each year. Our objective was to compare the views of midwives in rural and urban settings, regarding their competence and confidence with respect to 'competencies' identified as being those which all professionals should have in order to provide effective and safe care for low-risk women.", "This was a comparative questionnaire survey involving a stratified sample of remote and rural maternity units and an ad hoc comparison group of three urban maternity units in Scotland. Questionnaires were sent to 82 midwives working in remote and rural areas and 107 midwives working in urban hospitals with midwife-led units.", "The response rate from midwives in rural settings was considerably higher (85%) than from midwives in the urban areas (60%). Although the proportion of midwives who reported that they were competent was broadly similar in the two groups, there were some significant differences regarding specific competencies. Midwives in the rural group were more likely to report competence for breech delivery (p = 0.001), while more urban midwives reported competence in skills such as intravenous fluid replacement (p<0.001) and initial and discharge examination of the newborn (p<0.001). Both groups reported facing barriers to continuing professional development; however, more of the rural group had attended an educational event within the last month (p<0.001). Lack of time was a greater barrier for urban midwives (p = 0.02), whereas distance to training was greater for rural midwives (p = 0.009). Lack of motivation or interest was significantly higher in urban units (p = 0.006)." ], "LABELS": [ "INTRODUCTION", "METHOD", "RESULTS" ], "MESHES": [ "Attitude of Health Personnel", "Clinical Competence", "Education, Nursing, Continuing", "Female", "Health Care Surveys", "Humans", "Midwifery", "Pregnancy", "Rural Health Services", "Scotland", "Urban Health Services" ], "YEAR": null, "reasoning_required_pred": "yes", "reasoning_free_pred": "maybe", "final_decision": "maybe", "LONG_ANSWER": "It is often assumed that midwives in rural areas where there are fewer deliveries, will be less competent and confident in their practice. Our exploratory study suggests that the issue of competence is far more complex and deserves further attention." }, "15222284": { "QUESTION": "The effective orifice area/patient aortic annulus area ratio: a better way to compare different bioprostheses?", "CONTEXTS": [ "The aim of this prospective, randomized study was to compare the hemodynamic performance of the Medtronic Mosaic and Edwards Perimount bioprostheses in the aortic position, and to evaluate prosthesis-specific differences in valve sizing and valve-size labeling.", "Between August 2000 and September 2002, 139 patients underwent isolated aortic valve replacement (AVR) with the Mosaic (n = 67) or Perimount (n = 72) bioprosthesis. Intraoperatively, the internal aortic annulus diameter was measured by insertion of a gauge (Hegar dilator), while prosthesis size was determined by using the original sizers. Transthoracic echocardiography was performed to determine hemodynamic and dimensional data. As the aim of AVR is to achieve a maximal effective orifice area (EOA) within a given aortic annulus, the ratio of EOA to patient aortic annulus area was calculated, the latter being based on annulus diameter measured intraoperatively.", "Operative mortality was 2.2% (Mosaic 3.0%; Perimount 1.4%; p = NS). Upsizing (using a prosthesis larger in labeled valve size than the patient's measured internal aortic annulus diameter) was possible in 28.4% of Mosaic patients and 8.3% of Perimount patients. The postoperative mean systolic pressure gradient ranged from 10.5 to 22.2 mmHg in the Mosaic group, and from 9.4 to 12.6 mmHg in the Perimount group; it was significantly lower for 21 and 23 Perimount valves than for 21 and 23 Mosaic valves. The EOA ranged from 0.78 to 2.37 cm2 in Mosaic patients, and from 0.95 to 2.12 cm2 in Perimount patients. When indexing EOA by calculating the ratio of EOA to patient aortic annulus area to adjust for variables such as patient anatomy and valve dimensions, there was no significant difference between the two bioprostheses." ], "LABELS": [ "BACKGROUND AND AIM OF THE STUDY", "METHODS", "RESULTS" ], "MESHES": [ "Aged", "Aged, 80 and over", "Aortic Valve", "Aortic Valve Insufficiency", "Aortic Valve Stenosis", "Bioprosthesis", "Blood Pressure", "Female", "Heart Valve Prosthesis", "Heart Valve Prosthesis Implantation", "Humans", "Male", "Middle Aged", "Product Labeling", "Prospective Studies", "Prosthesis Design", "Prosthesis Fitting" ], "YEAR": "2004", "reasoning_required_pred": "no", "reasoning_free_pred": "yes", "final_decision": "yes", "LONG_ANSWER": "Comparisons of absolute EOA values grouped by the manufacturers' valve sizes are misleading because of specific differences in geometric dimensions. The EOA:patient aortic annulus area ratio provides a new hemodynamic index which may facilitate objective comparisons between different valve types." }, "25501465": { "QUESTION": "Evaluation of pediatric VCUG at an academic children's hospital: is the radiographic scout image necessary?", "CONTEXTS": [ "There is heterogeneity in how pediatric voiding cystourethrography (VCUG) is performed. Some institutions, including our own, obtain a radiographic scout image prior to contrast agent instillation.", "To demonstrate that the radiographic scout image does not augment VCUG interpretation or contribute management-changing information but nonetheless carries a non-negligible effective dose.", "We evaluated 181 children who underwent VCUG in 2012, with an age breakdown of less than 1\u00a0year (56 children), 1-5 years (66 children), 6-10 years (43 children) and 11-18 years (16 children), with a mean age of 4.0\u00a0years. We investigated patient demographics, clinical indication for the examination, scout image findings and estimated effective radiation dose, as well as overall exam findings and impression.", "No clinically significant or management-changing findings were present on scout images, and no radiopaque urinary tract calculi or concerning incidental finding was identified. Scout image estimated effective radiation dose averaged 0.09\u00a0mSv in children younger than 1\u00a0y, 0.09\u00a0mSv in children age 1-5, 0.13\u00a0mSv in children age 6-10 and 0.18\u00a0mSv in children age 11-18. Total fluoroscopy time per examination averaged 36.7\u00a0s (range 34.8-39.6\u00a0s for all age group averages). Evaluation of known or suspected vesicoureteral reflux (VUR) and urinary tract infection (UTI) were the most common clinical indications, stated in 40.9% and 37.0% of exams, respectively." ], "LABELS": [ "BACKGROUND", "OBJECTIVE", "MATERIALS AND METHODS", "RESULTS" ], "MESHES": [ "Academic Medical Centers", "Adolescent", "Child", "Child, Preschool", "Female", "Fluoroscopy", "Hospitals, Pediatric", "Humans", "Infant", "Male", "Retrospective Studies", "Vesico-Ureteral Reflux" ], "YEAR": "2015", "reasoning_required_pred": "no", "reasoning_free_pred": "no", "final_decision": "no", "LONG_ANSWER": "Although the estimated effective dose is low for VCUG radiographic scout images, this step did not augment VCUG interpretation or contribute management-changing information. This step should be omitted or substituted to further reduce dose in pediatric VCUG." }, "16414216": { "QUESTION": "Are endometrial polyps from pre-menopausal women similar to post-menopausal women?", "CONTEXTS": [ "Do endometrial polyps from pre- and post-menopausal women have similar immunohistochemical expression of oestrogen and progesterone receptors (ER, PR) and markers of cellular proliferation/apoptosis (Ki67 and Bcl-2).", "Prospective cohort study. Non-parametric statistical analysis was used.", "Polyps recruited from women attending an out-patient hysteroscopy clinic in a UK district general hospital.", "Fourteen pre-menopausal and 16 post-menopausal women who presented with abnormal bleeding with endometrial polyps.", "Immunohistochemical staining was performed on endometrial polyps.", "Significant differences or correlations between hormone receptor expression (oestrogen and progesterone) and cell growth indices (Ki67 and Bcl-2).", "Endometrial polyps from pre- and post-menopausal women had significant differences in their expression of hormone receptors and Ki67. However, polyps from both groups of women had similarly increased levels of Bcl-2, an inhibitor of apoptosis." ], "LABELS": [ "OBJECTIVE", "DESIGN", "SETTING", "PATIENTS", "INTERVENTIONS", "MAIN OUTCOME MEASURES", "RESULTS" ], "MESHES": [ "Adult", "Cohort Studies", "Endometrial Neoplasms", "Female", "Humans", "Immunohistochemistry", "Middle Aged", "Polyps", "Postmenopause", "Premenopause", "Prospective Studies", "Receptors, Estrogen", "Receptors, Progesterone" ], "YEAR": "2006", "reasoning_required_pred": "yes", "reasoning_free_pred": "no", "final_decision": "no", "LONG_ANSWER": "Pre- and post-menopausal polyps exhibit differing hormone receptor and proliferation markers, presumably a result of their hormonal milieu. However, both groups appear to have lost the usual control mechanisms for apoptotic regulation, this appears to be responsible for their growth." }, "19155657": { "QUESTION": "Does accompanying metabolic syndrome contribute to heart dimensions in hypertensive patients?", "CONTEXTS": [ "Metabolic syndrome (MetS) is associated with increased risk for cardiovascular events. We evaluated heart dimensions in hypertensive patients with MetS.", "The study included 75 hypertensive patients (34 males, 41 females; mean age 51+/-9 years) without coronary artery disease. Patients were evaluated in two groups depending on the presence or absence of MetS. Age- and gender-matched 20 healthy subjects (9 males, 11 females; mean age 50+/-5 years) comprised the control group. The diagnosis of MetS was based on the presence of at least three of five MetS criteria. Hypertension was defined as arterial blood pressure exceeding 140/85 mmHg on three consecutive measurements or the use of antihypertensive drugs. Echocardiographic measurements included interventricular septal thickness, left ventricular internal diameter, posterior wall thickness, aortic diameter, left atrial diameter, relative wall thickness, and left ventricular mass.", "Metabolic syndrome was present in 32 hypertensive patients (42.7%; 18 males, 14 females). The mean number of MetS criteria was 2.6+/-1.0 in the hypertensive group. Compared to the control group, patients with or without MetS exhibited significantly increased interventricular septum and posterior wall thickness, left atrial diameter, relative wall thickness, and left ventricular mass (p<0.05). The only significant difference between the two patient groups was that MetS was associated with a greater left atrial diameter (p=0.019). Left atrial diameter was correlated with the number of MetS criteria (r=0.51; p<0.001)." ], "LABELS": [ "OBJECTIVES", "STUDY DESIGN", "RESULTS" ], "MESHES": [ "Cardiovascular Diseases", "Case-Control Studies", "Echocardiography", "Female", "Humans", "Hypertension", "Hypertrophy, Left Ventricular", "Male", "Metabolic Syndrome", "Middle Aged", "Risk Factors", "Ventricular Remodeling" ], "YEAR": "2008", "reasoning_required_pred": "yes", "reasoning_free_pred": "maybe", "final_decision": "maybe", "LONG_ANSWER": "Left ventricular dimensions are not influenced by MetS. Rather than MetS, hypertension is primarily responsible for changes in left ventricular dimensions. However, left atrial enlargement is more prominent in patients with MetS, suggesting that each MetS criterion contributes to left ventricular diastolic dysfunction." }, "27050489": { "QUESTION": "The Prevalence of Incidentally Detected Idiopathic Misty Mesentery on\u00a0Multidetector Computed Tomography: Can Obesity Be the Triggering Cause?", "CONTEXTS": [ "Misty mesentery appearance is commonly reported in daily practice, usually as a secondary finding of various pathological entities, but sometimes it is encountered as an isolated finding that cannot be attributed to any other disease entity. We aimed to assess the prevalence of cases with incidentally detected idiopathic misty mesentery on computed tomography (CT) and to summarize the pathologies leading to this appearance.", "Medical records and initial and follow-up CT features of patients with misty mesentery appearance between January 2011 and January 2013 were analysed. The study included cases with no known cause of misty mesentery according to associated CT findings, clinical history, or biochemical manifestations, and excluded patients with diseases known to cause misty mesentery, lymph nodes greater than a short-axis diameter of 5\u00a0mm, discrete mesenteric masses, or bowel wall thickening.", "There were a total of 561 patients in whom misty mesentery appearance was depicted on abdominopelvic CT scans. A total of 80 cases were found to have isolated incidental idiopathic misty mesentery, giving a prevalence of 7%. The common indication for CT examination was abdominal pain. There was a slight female predominance (51.3%). 67.5% of all patients were classified as obese and 17.5% as overweight." ], "LABELS": [ "PURPOSE", "METHODS", "RESULTS" ], "MESHES": [ "Adult", "Body Mass Index", "Female", "Humans", "Incidental Findings", "Male", "Mesentery", "Middle Aged", "Multidetector Computed Tomography", "Obesity", "Prevalence" ], "YEAR": "2016", "reasoning_required_pred": "yes", "reasoning_free_pred": "yes", "final_decision": "yes", "LONG_ANSWER": "The results of the present study show that idiopathic incidental misty mesentery appearance has a significant prevalence. Also, the high body mass index of these patients and the growing evidence of obesity-induced inflammatory changes in adipose tissue are suggestive of an association between obesity and misty mesentery appearance on CT." }, "24977765": { "QUESTION": "Are pediatric concussion patients compliant with discharge instructions?", "CONTEXTS": [ "Concussions are commonly diagnosed in pediatric patients presenting to the emergency department (ED). The primary objective of this study was to evaluate compliance with ED discharge instructions for concussion management.", "A prospective cohort study was conducted from November 2011 to November 2012 in a pediatric ED at a regional Level 1 trauma center, serving 35,000 pediatric patients per year. Subjects were aged 8 years to 17 years and were discharged from the ED with a diagnosis of concussion. Exclusion criteria included recent (past 3 months) diagnosis of head injury, hospital admission, intracranial injury, skull fracture, suspected nonaccidental trauma, or preexisting neurologic condition. Subjects were administered a baseline survey in the ED and were given standardized discharge instructions for concussion by the treating physician. Telephone follow-up surveys were conducted at 2 weeks and 4 weeks after ED visit.", "A total of 150 patients were enrolled. The majority (67%) of concussions were sports related. Among sports-related concussions, soccer (30%), football (11%), lacrosse (8%), and basketball (8%) injuries were most common. More than one third (39%) reported return to play (RTP) on the day of the injury. Physician follow-up was equivalent for sport and nonsport concussions (2 weeks, 58%; 4 weeks, 64%). Sports-related concussion patients were more likely to follow up with a trainer (2 weeks, 25% vs. 10%, p = 0.06; 4 weeks, 29% vs. 8%, p<0.01). Of the patients who did RTP or normal activities at 2 weeks (44%), more than one third (35%) were symptomatic, and most (58%) did not receive medical clearance. Of the patients who had returned to activities at 4 weeks (64%), less than one quarter (23%) were symptomatic, and most (54%) received medical clearance." ], "LABELS": [ "BACKGROUND", "METHODS", "RESULTS" ], "MESHES": [ "Adolescent", "Athletic Injuries", "Basketball", "Brain Concussion", "Child", "Emergency Service, Hospital", "Female", "Humans", "Male", "Patient Compliance", "Patient Discharge Summaries", "Prospective Studies", "Racquet Sports", "Soccer" ], "YEAR": "2014", "reasoning_required_pred": "no", "reasoning_free_pred": "yes", "final_decision": "yes", "LONG_ANSWER": "Pediatric patients discharged from the ED are mostly compliant with concussion instructions. However, a significant number of patients RTP on the day of injury, while experiencing symptoms or without medical clearance." }, "19130332": { "QUESTION": "Is the zeolite hemostatic agent beneficial in reducing blood loss during arterial injury?", "CONTEXTS": [ "Uncontrolled hemorrhage is the leading cause of fatality. The aim of this study was to evaluate the effect of zeolite mineral (QuikClot - Advanced Clotting Sponge [QC-ACS]) on blood loss and physiological variables in a swine extremity arterial injury model.", "Sixteen swine were used. Oblique groin incision was created and a 5 mm incision was made. The animals were allocated to: control group (n: 6): Pressure dressing was applied with manual pressure over gauze sponge; or QC group (n: 10): QC was directly applied over lacerated femoral artery. Mean arterial pressure, blood loss and physiological parameters were measured during the study period.", "Application of QC led to a slower drop in blood pressure. The control group had a significantly higher increase in lactate within 60 minutes. The mean prothrombin time in the control group was significantly increased at 60 minutes. The application of QC led to decreased total blood loss. The QC group had significantly higher hematocrit levels. QC application generated a significant heat production. There were mild edematous and vacuolar changes in nerve samples." ], "LABELS": [ "BACKGROUND", "METHODS", "RESULTS" ], "MESHES": [ "Administration, Topical", "Animals", "Bandages", "Blood Pressure", "Disease Models, Animal", "Femoral Artery", "Hematocrit", "Hemorrhage", "Hemostatics", "Random Allocation", "Specific Pathogen-Free Organisms", "Survival Rate", "Swine", "Zeolites" ], "YEAR": "2009", "reasoning_required_pred": "yes", "reasoning_free_pred": "maybe", "final_decision": "yes", "LONG_ANSWER": "According to the physiological parameters, we observed that zeolite tends to reduce blood loss, however could not stop bleeding completely. We believe that further clinical trials are needed to conclude that zeolite could be used in the routine practice." }, "23949294": { "QUESTION": "Treatment as prevention in resource-limited settings: is it feasible to maintain HIV viral load suppression over time?", "CONTEXTS": [ "Recently, there has been increasing interest in the role of \"treatment as prevention\" (TasP). Some of the questions regarding TasP strategies arise from the perceived difficulties in achieving and maintaining viral load (VL) suppression over time and the risk of emergence of viral resistance that could compromise future treatment options. This study was conducted to assess these questions in a resource-limited setting.", "We performed a retrospective observational study of HIV-infected patients diagnosed in the pre-HAART era on follow-up at a private center from Buenos Aires, Argentina. Socio-demographic, clinical, and laboratory data were extracted from clinical charts. Analyses were performed to test for potential associations of selected variables with current virologic failure or use of third-line drugs.", "Of 619 patients on follow-up, 82 (13.2%) were diagnosed in the pre-HAART era. At the time of our study, 79 (96.3%) patients were on HAART, with a median duration of 14 years (IQR 12-15) of therapy, and exposure to mono or dual nucleoside reverse transcriptase inhibitors regimens in 47.8% of cases.\u00a0Sixty-nine patients (87.3%) had undetectable VL, 37 (46.8%) never presented virologic failure, and 19 (24.1%) experienced only one failure. Thirteen patients (16.5%) were receiving third-line ART regimens, with an average of 2.7-fold more virologic failures than those on first- or second-line regimens (p = 0.007)." ], "LABELS": [ "INTRODUCTION", "METHODOLOGY", "RESULTS" ], "MESHES": [ "Adolescent", "Adult", "Anti-HIV Agents", "Argentina", "Chemoprevention", "Cohort Studies", "Developing Countries", "Drug Resistance, Viral", "Female", "HIV", "HIV Infections", "Humans", "Male", "Retrospective Studies", "Treatment Failure", "Viral Load", "Young Adult" ], "YEAR": "2013", "reasoning_required_pred": "yes", "reasoning_free_pred": "yes", "final_decision": "yes", "LONG_ANSWER": "Maintaining viral load suppression over time in resource-limited-settings is feasible." }, "18926458": { "QUESTION": "Are octogenarians at high risk for carotid endarterectomy?", "CONTEXTS": [ "Several prospective randomized trials have proved carotid endarterectomy to be safe and effective for both symptomatic and asymptomatic patients younger than 80 years of age. Recently, carotid artery stenting (CAS) has been approved for use in selected high-risk patients. It has been proposed that being an octogenarian places patients in this high-risk category.", "All patients between the ages of 80 to 89 years undergoing carotid endarterectomy during a 12-year period were included in the study. Information included indications for carotid endarterectomy, associated risk factors, length of stay, and hospital course. Perioperative morbidity and mortality, including neurologic events and myocardial infarction, were recorded.", "A total of 103 carotid endarterectomies were performed in 95 octogenarians. Procedures were performed on 59 men and 36 women. Indications for operation included symptomatic carotid stenosis in 44 patients (43%) and asymptomatic carotid stenosis in 59 (57%). Associated risk factors included diabetes mellitus (17%), hypertension (76%), coronary artery disease (28%), hyperlipidemia (39%), and history of smoking (42%). There were 4 perioperative neurologic complications, which included 1 transient ischemic attack (0.97%), 2 minor strokes (1.94%), and 1 major stroke (0.97%). There were no deaths." ], "LABELS": [ "BACKGROUND", "STUDY DESIGN", "RESULTS" ], "MESHES": [ "Age Factors", "Aged, 80 and over", "Carotid Stenosis", "Endarterectomy, Carotid", "Female", "Humans", "Male", "Morbidity" ], "YEAR": "2008", "reasoning_required_pred": "yes", "reasoning_free_pred": "no", "final_decision": "no", "LONG_ANSWER": "Combined end points for adverse events are acceptable in the octogenarian. Carotid endarterectomy remains the gold standard for treatment of extracranial carotid disease in all age groups. Age alone should not place patients in the high-risk category for carotid endarterectomy." }, "10749257": { "QUESTION": "Has the mammography quality standards act affected the mammography quality in North Carolina?", "CONTEXTS": [ "The United States Food and Drug Administration implemented federal regulations governing mammography under the Mammography Quality Standards Act (MQSA) of 1992. During 1995, its first year in implementation, we examined the impact of the MQSA on the quality of mammography in North Carolina.", "All mammography facilities were inspected during 1993-1994, and again in 1995. Both inspections evaluated mean glandular radiation dose, phantom image evaluation, darkroom fog, and developer temperature. Two mammography health specialists employed by the North Carolina Division of Radiation Protection performed all inspections and collected and codified data.", "The percentage of facilities that met quality standards increased from the first inspection to the second inspection. Phantom scores passing rate was 31.6% versus 78.2%; darkroom fog passing rate was 74.3% versus 88.5%; and temperature difference passing rate was 62.4% versus 86.9%." ], "LABELS": [ "OBJECTIVE", "MATERIALS AND METHODS", "RESULTS" ], "MESHES": [ "Mammography", "North Carolina", "United States", "United States Food and Drug Administration" ], "YEAR": "2000", "reasoning_required_pred": "yes", "reasoning_free_pred": "yes", "final_decision": "yes", "LONG_ANSWER": "In 1995, the first year that the MQSA was in effect, there was a significant improvement in the quality of mammography in North Carolina. This improvement probably resulted from facilities' compliance with federal regulations." }, "17610439": { "QUESTION": "Do hospitals provide lower quality care on weekends?", "CONTEXTS": [ "To examine the effect of a weekend hospitalization on the timing and incidence of intensive cardiac procedures, and on subsequent expenditures, mortality and readmission rates for Medicare patients hospitalized with acute myocardial infarction (AMI).", "The primary data are longitudinal, administrative claims for 922,074 elderly, non-rural, fee-for-service Medicare beneficiaries hospitalized with AMI from 1989 to 1998. Annual patient-level cohorts provide information on ex ante health status, procedure use, expenditures, and health outcomes.", "The patient is the primary unit of analysis. I use ordinary least squares regression to estimate the effect of weekend hospitalization on rates of cardiac catheterization, angioplasty, and bypass surgery (in various time periods subsequent to the initial hospitalization), 1-year expenditures and rates of adverse health outcomes in various periods following the AMI admission.", "Weekend AMI patients are significantly less likely to receive immediate intensive cardiac procedures, and experience significantly higher rates of adverse health outcomes. Weekend admission leads to a 3.47 percentage point reduction in catheterization at 1 day, a 1.52 point reduction in angioplasty, and a 0.35 point reduction in by-pass surgery (p<.001 in all cases). The primary effect is delayed treatment, as weekend-weekday procedure differentials narrow over time from the initial hospitalization. Weekend patients experience a 0.38 percentage point (p<.001) increase in 1-year mortality and a 0.20 point (p<.001) increase in 1-year readmission with congestive heart failure." ], "LABELS": [ "OBJECTIVE", "DATA SOURCES", "STUDY DESIGN", "PRINCIPAL FINDINGS" ], "MESHES": [ "Aged", "Female", "Health Expenditures", "Health Status", "Hospital Mortality", "Hospitalization", "Humans", "Insurance Claim Review", "Male", "Medicare", "Myocardial Infarction", "Patient Readmission", "Quality of Health Care", "Time Factors", "Treatment Outcome" ], "YEAR": "2007", "reasoning_required_pred": "yes", "reasoning_free_pred": "yes", "final_decision": "yes", "LONG_ANSWER": "Weekend hospitalization leads to delayed provision of intensive procedures and elevated 1-year mortality for elderly AMI patients. The existence of measurable differences in treatments raises questions regarding the efficacy of a single input regulation (e.g., mandated nurse staffing ratios) in enhancing the quality of weekend care. My results suggest that targeted financial incentives might be a more cost-effective policy response than broad regulation aimed at improving quality." }, "26859535": { "QUESTION": "Screening History Among Women with Invasive Cervical Cancer in an Academic Medical Center: Will We Miss Cancers Following Updated Guidelines?", "CONTEXTS": [ "Updated guidelines for the screening and management of cervical cancer in the United States recommend starting Papanicolaou (Pap) testing at age 21 and screening less frequently with less aggressive management for abnormalities. We sought to examine updated Pap test screening guidelines and how they may affect the detection of invasive cervical cancer, especially among women<30 years of age.", "Patients diagnosed at Brigham and Women's Hospital with invasive cervical cancer between 2002 and 2012 were retrospectively identified. Prior screening history was obtained and patients were divided into two groups based on age<30 years or age \u226530 years. The two groups were then compared with respect to demographics, pathological findings, and time to diagnosis.", "A total of 288 patients with invasive cervical carcinoma were identified. Among these patients, 109 had adequate information on prior screening history. Invasive adenocarcinoma (IAC) was diagnosed in 37 (33.94%) patients, whereas 64 (58.72%) patients were diagnosed with invasive squamous cell carcinoma (ISCC). The remaining eight patients were diagnosed with other types of cancers of the cervix. A total of 13 patients were younger than 30 while 96 patients were 30 or older. The mean time from normal Pap to diagnosis of IAC was 15 months in patients younger than 30 years of age compared to 56 months in patients aged 30 and older (p\u2009<\u20090.001). The mean time from normal Pap to diagnosis of ISCC was 38 months in patients younger than 30 years of age and 82 months in patients aged 30 and older (p\u2009=\u20090.018)." ], "LABELS": [ "OBJECTIVE", "MATERIALS AND METHODS", "RESULTS" ], "MESHES": [ "Academic Medical Centers", "Adenocarcinoma", "Adult", "Aged", "Female", "Humans", "Mass Screening", "Massachusetts", "Middle Aged", "Papanicolaou Test", "Practice Guidelines as Topic", "Retrospective Studies", "Risk Factors", "Uterine Cervical Neoplasms", "Vaginal Smears" ], "YEAR": "2016", "reasoning_required_pred": "no", "reasoning_free_pred": "no", "final_decision": "no", "LONG_ANSWER": "In this small retrospective study, updated Pap test screening guidelines would not have missed invasive cancer on average among screened women age 30 and older. However, young patients aged 21-29 years may be at increased risk of developing IAC of the cervix between the recommended screening intervals." }, "25446909": { "QUESTION": "Oral mucocele/ranula: Another human immunodeficiency virus-related salivary gland disease?", "CONTEXTS": [ "To describe clinical characteristics of oral mucoceles/ranulas, with a focus on human immunodeficiency virus (HIV)-related salivary gland diseases.", "A descriptive and clinical study, with review of patient data.", "We reviewed 113 referred cases of oral mucocele. The following anatomical sites were identified: lip, tongue, and floor of the mouth (simple ranulas), as well as plunging ranulas. The age and gender data of the patients with oral mucoceles were recorded. The HIV status of the patients and other information were reviewed.", "There were 30 (26.5%) males and 83 (73.5%) females. Most patients were below 30 years of age, with the peak frequency in the first and second decade. Ranula (simple and plunging) represented 84.1% of the mucocele locations. Mucocele on the lips represented 10.6%. Seventy-two (63.7%) patients were HIV positive; and 97.2% of them had ranulas. Thirty-eight (33.6%) patients presented with plunging ranulas; and 92.1% of them were HIV positive, compared with two patients presenting with plunging ranulas in the HIV-negative group. These results strongly suggest that an HIV-positive patient is statistically (P<0.001) more at risk of presenting with not only a simple, but also a plunging ranula type." ], "LABELS": [ "HYPOTHESIS", "STUDY DESIGN", "MATERIAL AND METHODS", "RESULTS" ], "MESHES": [ "Adolescent", "Adult", "Child", "Child, Preschool", "Diagnosis, Differential", "Female", "Follow-Up Studies", "HIV", "HIV Infections", "Humans", "Infant", "Infant, Newborn", "Male", "Middle Aged", "Mouth Diseases", "Mucocele", "Prevalence", "Ranula", "Salivary Gland Diseases", "South Africa", "Young Adult" ], "YEAR": "2015", "reasoning_required_pred": "yes", "reasoning_free_pred": "yes", "final_decision": "yes", "LONG_ANSWER": "This study presents a different clinical picture of oral mucoceles/ranulas, as observed in HIV-positive patients. Additionally, it suggests a possible clinical link between the two pathologies. The authors strongly support the suggestion that oral mucocele/ranula is an HIV-related salivary gland disease." }, "17220021": { "QUESTION": "Is there an increase in the incidence of gbs carrier rates among pregnant women in northern Israel?", "CONTEXTS": [ "Group B Streptococci (GBS) asymptomatically colonize the vaginal or rectal areas of about 20% of pregnant women (4-40%). About 50% of infants to mothers with GBS colonization also become colonized at rectal, umbilical or oral sites. GBS is a leading bacterial cause of neonatal illness and death. The present prevalence rate of GBS carriers among parturients in the western Galilee in Israel is unknown.AIM: A prospective study of the GBS carrier rate according to origin and gestational age in the western Galilee in Israel.", "A prospective study including 700 pregnant women. All women were screened for carriage of GBS by vaginal and rectal cultures.", "Sixteen percent of the parturients were found to be GBS colonized. The prevalence of GBS was 13.7% in Jewish women and 19% in Arab women, P=0.038. The women were also divided into two groups according to the gestational age one group included 414 women in 24-37 weeks gestation, and the other group included 286 women in term pregnancy. No difference was found in the rate of GBS carriers between the two gestational age groups." ], "LABELS": [ "BACKGROUND", "METHODS", "RESULTS" ], "MESHES": [ "Adolescent", "Adult", "Carrier State", "Female", "Humans", "Israel", "Pregnancy", "Pregnancy Complications, Infectious", "Prevalence", "Prospective Studies", "Rectum", "Streptococcal Infections", "Streptococcus agalactiae", "Vagina" ], "YEAR": "2006", "reasoning_required_pred": "no", "reasoning_free_pred": "yes", "final_decision": "yes", "LONG_ANSWER": "In the present study we found a significant increase in the incidence of GBS colonization in pregnant women in northern Israel. We also found an increased rate of GBS carriers in Arab women relative to Jewish women." }, "10158597": { "QUESTION": "Does a dedicated discharge coordinator improve the quality of hospital discharge?", "CONTEXTS": [ "To evaluate the effectiveness of the role of a discharge coordinator whose sole responsibility was to plan and coordinate the discharge of patients from medical wards.", "An intervention study in which the quality of discharge planning was assessed before and after the introduction of a discharge coordinator. Patients were interviewed on the ward before discharge and seven to 10 days after being discharged home.", "The three medical wards at the Homerton Hospital in Hackney, East London.", "600 randomly sampled adult patients admitted to the medical wards of the study hospital, who were resident in the district (but not in institutions), were under the care of physicians (excluding psychiatry), and were discharged home from one of the medical wards. The sampling was conducted in three study phases, over 18 months.", "Phase I comprised base line data collection; in phase II data were collected after the introduction of the district discharge planning policy and a discharge form (checklist) for all patients; in phase III data were collected after the introduction of the discharge coordinator.", "The quality and out come of discharge planning. Readmission rates, duration of stay, appropriateness of days of care, patients' health and satisfaction, problems after discharge, and receipt of services.", "The discharge coordinator resulted in an improved discharge planning process, and there was a reduction in problems experienced by patients after discharge, and in perceived need for medical and healthcare services. There was no evidence that the discharge coordinator resulted in a more timely or effective provision of community services after discharge, or that the appropriateness or efficiency of bed use was improved." ], "LABELS": [ "OBJECTIVE", "DESIGN", "SETTING", "PATIENTS", "INTERVENTIONS", "MAIN MEASURES", "RESULTS" ], "MESHES": [ "Activities of Daily Living", "Health Status Indicators", "Humans", "Length of Stay", "London", "Patient Discharge", "Patient Readmission", "Patient Satisfaction", "Program Evaluation", "Quality Assurance, Health Care", "Social Work", "State Medicine" ], "YEAR": "1996", "reasoning_required_pred": "yes", "reasoning_free_pred": "yes", "final_decision": "yes", "LONG_ANSWER": "The introduction of a discharge coordinator improved the quality of discharge planning, but at additional cost." }, "21198823": { "QUESTION": "Can dobutamine stress echocardiography induce cardiac troponin elevation?", "CONTEXTS": [ "Elevation of cardiac troponin (cTn) is considered specific for myocardial damage. Elevated cTn and echocardiogrpahic documentation of wall motion abnormalities (WMAs) that were recorded after extreme physical effort raise the question whether dobutamine stress echo (DSE), can also induce elevation of troponin.", "we prospective enrolled stable patients (age>18 years) referred to DSE. The exam was performed under standardized conditions. Blood samples for cTnI were obtained at baseline and 18-24 hours after the test. We aimed to compare between the clinical and echocardiographic features of patients with elevated cTnI and those without cTnI elevations.", "Fifty-seven consecutive patients were included. The average age was 64.4 \u00b1 10.7, 73% of the patients were males, and nearly half of the patients were known to have ischemic heart disease. Two of the patients were excluded due to technical difficulty. No signs of ischemia were recorded in 25 (45.4%). Among the patients with established ischemia on DSE, 12 (22%) had mild ischemia, 13 (23.6%) had moderate and 5 (9%) had severe ischemia. Angiography was performed in 13 (26%) of the patients, of which 7 had PCI and one was referred to bypass surgery. None of the patients had elevated cTnI 18-24 hours after the DSE." ], "LABELS": [ "BACKGROUND", "METHODS", "RESULTS" ], "MESHES": [ "Dobutamine", "Echocardiography", "Exercise Test", "Female", "Humans", "Male", "Middle Aged", "Myocardial Infarction", "Troponin T", "Vasodilator Agents" ], "YEAR": "2011", "reasoning_required_pred": "no", "reasoning_free_pred": "no", "final_decision": "no", "LONG_ANSWER": "Our results indicate that there is no elevation of cTn despite the occurrence of significant WMAs on DSE. We conclude that cTnI cannot be used as an additional diagnostic tool during pharmacological stress test performed to evaluate the presence and severity of ischemia." }, "17259061": { "QUESTION": "Cigarettes and cinema: does parental restriction of R-rated movie viewing reduce adolescent smoking susceptibility?", "CONTEXTS": [ "To examine the relationship between exposure to pro-smoking messages in media and susceptibility to smoking adoption among middle school students. The hypothesis that parental restriction of R-rated movie viewing is associated with lower adolescent smoking susceptibility was tested.", "A sample of 1687 6th-, 7th-, and 8th-grade students from four Wisconsin middle schools were surveyed about their use of cigarettes, exposure to smoking in media, their views of smoking, and peer smoking behaviors.", "An index of smoking susceptibility was created using measures of cigarette use and future intention to smoke. A zero-order correlation for parental restriction of R-rated movie viewing and smoking susceptibility showed a strong association (r = -.36, p<.001). A hierarchical logistic regression yielded odds ratios (ORs) for being susceptible to or having tried smoking for three levels of parental R-rated movie restriction. Results show that compared to full restriction, respondents with partial or no restriction were more likely to be susceptible to smoking (partial restriction: OR = 2.1, 95% CI = 1.5-2.8; no restriction: OR = 3.3, 95% CI = 2.3-4.6), when controlling for demographic factors, and family and friend smoking. Analyses using a measure of smoking prevalence as the dependent variable yielded similar results (partial restriction: OR = 1.5, 95% CI = 1.0-2.2; no restriction: OR = 2.5, 95% CI = 1.7-3.7)." ], "LABELS": [ "PURPOSE", "METHODS", "RESULTS" ], "MESHES": [ "Adolescent", "Adolescent Behavior", "Female", "Humans", "Imitative Behavior", "Logistic Models", "Male", "Motion Pictures", "Parent-Child Relations", "Prevalence", "Smoking", "Smoking Prevention", "Surveys and Questionnaires", "Wisconsin" ], "YEAR": "2007", "reasoning_required_pred": "yes", "reasoning_free_pred": "yes", "final_decision": "yes", "LONG_ANSWER": "Parental restriction of R-rated movie viewing is associated with both lower adolescent smoking susceptibility and lower smoking rates." }, "18375490": { "QUESTION": "Are laboratories reporting serum quantitative hCG results correctly?", "CONTEXTS": [ "Human chorionic gonadotropin (hCG) is a heterodimeric glycoprotein hormone that exists in multiple forms. Immunoassays commonly used in clinical laboratories measure intact hCG, total beta hCG (intact hCG + hCG free beta-subunit), and/or hCG free beta-subunit. Measurement of serum concentrations of hCG is useful for confirmation and monitoring of pregnancy, diagnosis of trophoblastic diseases and monitoring of the efficacy of treatment, and prenatal screening. Correctly reporting results for the various forms of hCG is clinically important.", "We prepared samples by addition of intact hCG and hCG free beta-subunit to an essentially hCG-free human serum matrix. The samples were analyzed by participant laboratories using various immunoassay methods.", "We identified errors in participant reporting of intact hCG results as total beta hCG (9.3%; 22 of 235 laboratories) and total beta hCG as intact hCG (13.1%; 8 of 61 laboratories)." ], "LABELS": [ "BACKGROUND", "METHOD", "RESULTS" ], "MESHES": [ "Chorionic Gonadotropin", "Chorionic Gonadotropin, beta Subunit, Human", "Humans", "Immunoassay", "Quality Control", "Research Design" ], "YEAR": "2008", "reasoning_required_pred": "yes", "reasoning_free_pred": "no", "final_decision": "no", "LONG_ANSWER": "Many factors contribute to the erroneous reporting of hCG results, including (a) the complexity of hCG molecule and confusion of nomenclature on the various forms of hCG; (b) laboratory personnel's lack of awareness of the distinctions of the forms of hCG and failure to recognize the specificity of assays for their measurement; (c) lack of clarity and uniformity in manufacturers' reagent labeling; and (d) most product inserts' lack of information on the specificity of each method to the various forms of hCG." }, "24142776": { "QUESTION": "Steroids in aminoglycoside-containing ear drops: do they reduce cochlear toxicity?", "CONTEXTS": [ "To determine whether betamethasone (BM) reduces the cochlear toxicity of otic gentamicin (GM) if given together.", "Controlled animal study.", "Thirty-four mice were assigned at random to receive intratympanic injections of either 0.1 % BM (11 mice), 0.3% GM (13 mice), or a combination of both (GM/BM) with benzalkonium chloride (10 mice) in the left ear (treated) and saline on the right (untreated). Six injections were given on alternate days. Auditory brainstem response thresholds were assessed at 1 month, 2 months, and>2 months.", "There was a significantly greater degree of hearing loss in the BM-treated ears compared to the untreated ears (6.48 dB hearing loss, P = .007) and in the GM-treated ears compared to untreated ears (6.59 dB hearing loss, P = .010,). However, otic GM/BM and benzalkonium chloride did not cause significant additional hearing loss compared with the untreated ears (3.56 dB hearing loss, P = .242)." ], "LABELS": [ "HYPOTHESIS", "STUDY DESIGN", "METHODS", "RESULTS" ], "MESHES": [ "Aminoglycosides", "Animals", "Benzalkonium Compounds", "Betamethasone", "Cochlea", "Evoked Potentials, Auditory, Brain Stem", "Injections", "Mice", "Mice, Inbred C57BL", "Random Allocation" ], "YEAR": "2014", "reasoning_required_pred": "yes", "reasoning_free_pred": "yes", "final_decision": "yes", "LONG_ANSWER": "Our data suggest that hearing loss caused by GM otic drops may be reduced by the inclusion of BM and benzalkonium chloride. Our finding that BM alone was associated with hearing loss suggests that the benzalkonium chloride may be the protective agent in combination otic drops." }, "18388848": { "QUESTION": "Does having a regular primary care clinician improve quality of preventive care for young children?", "CONTEXTS": [ "This study examines whether having a regular clinician for preventive care is associated with quality of care for young children, as measured by interpersonal quality ratings and content of anticipatory guidance.", "The National Survey of Early Childhood Health (NSECH), a nationally representative parent survey of health care quality for 2068 young US children fielded by the National Center for Health Statistics (NCHS).", "Bivariate and multivariate analyses evaluate associations between having a regular clinician for well child care and interpersonal quality, the content of anticipatory guidance, and timely access to care.", "In bivariate analysis, parents of children with a regular clinician for preventive care reported slightly higher interpersonal quality (69 vs. 65 on a 0-100 scale, P = 0.01). Content of anticipatory guidance received was slightly greater for children with a regular clinician (82 vs. 80 on a 0-100 scale, P = 0.03). In bivariate analysis, a regular clinician was associated with interpersonal quality only among African American and Hispanic children. In multivariate analyses, controlling for factors that could independently influence self-reports of experiences with care, interpersonal quality but not anticipatory guidance content was higher for children with a regular clinician." ], "LABELS": [ "OBJECTIVE", "DATA SOURCE", "STUDY DESIGN", "PRINCIPAL FINDINGS" ], "MESHES": [ "Child, Preschool", "Continental Population Groups", "Educational Status", "Female", "Health Services", "Health Services Accessibility", "Health Status", "Humans", "Infant", "Insurance Coverage", "Insurance, Health", "Male", "Primary Health Care", "Primary Prevention", "Quality of Health Care" ], "YEAR": "2008", "reasoning_required_pred": "maybe", "reasoning_free_pred": "yes", "final_decision": "yes", "LONG_ANSWER": "Having a regular primary care clinician is embraced in pediatrics, although team care among physicians is also widely practiced. For young children, having a regular clinician is associated with modest gains in interpersonal quality and no differences in content of anticipatory guidance. The benefit of having a regular clinician may primarily occur in interpersonal quality for subgroups of young children." }, "22251324": { "QUESTION": "Does performance in selection processes predict performance as a dental student?", "CONTEXTS": [ "This study investigated associations between the performance of dental students in each of the three components of the selection procedure [academic average, Undergraduate Medicine and Health Sciences Admission Test (UMAT) and structured interview], socio-demographic characteristics and their academic success in an undergraduate dental surgery programme.", "Longitudinal review of admissions data relating to students entering dental education at the University of Otago, New Zealand, between 2004 and 2009 was compared with academic performance throughout the dental programme.", "After controlling for variables, pre-admission academic average, UMAT scores and interview performance did not predict performance as a dental student. Class place in second year, however, was a strong predictor of class place in final year. Multivariate analysis demonstrated that the best predictors of higher class placement in the final year were New Zealand European ethnicity and domestic (rather than international) student status. Other socio-demographic characteristics were not associated with performance. These interim findings provide a sound base for the ongoing study." ], "LABELS": [ "OBJECTIVE", "MATERIALS AND METHODS", "RESULTS AND DISCUSSION" ], "MESHES": [ "Achievement", "Adult", "Chi-Square Distribution", "Clinical Competence", "College Admission Test", "Educational Measurement", "Female", "Humans", "Interviews as Topic", "Longitudinal Studies", "Male", "New Zealand", "Predictive Value of Tests", "School Admission Criteria", "Students, Dental" ], "YEAR": "2012", "reasoning_required_pred": "yes", "reasoning_free_pred": "no", "final_decision": "no", "LONG_ANSWER": "The study found important socio-demographic differences in pre-admission test scores, but those scores did not predict performance in the dental programme, whether measured in second year or in final year." }, "27896825": { "QUESTION": "Is it time to reconsider lobectomy in low-risk paediatric thyroid cancer?", "CONTEXTS": [ "Current guidelines recommend total thyroidectomy for nearly all children with well-differentiated thyroid cancer (WDTC). These guidelines, however, derive from older data accrued prior to current high-resolution imaging. We speculate that there is a subpopulation of children who may be adequately treated with lobectomy.", "Retrospective analysis of prospectively maintained database.", "Seventy-three children with WDTC treated between 2004 and 2015.", "We applied two different risk-stratification criteria to this population. First, we determined the number of patients meeting American Thyroid Association (ATA) 'low-risk' criteria, defined as disease grossly confined to the thyroid with either N0/Nx or incidental microscopic N1a disease. Second, we defined a set of 'very-low-risk' histopathological criteria, comprising unifocal tumours \u22644 cm without predefined high-risk factors, and determined the proportion of patients that met these criteria.", "Twenty-seven (37%) males and 46 (63%) females were included in this study, with a mean age of 13\u00b74 years. Ipsilateral- and contralateral multifocality were identified in 27 (37\u00b70%) and 19 (26\u00b70%) of specimens. Thirty-seven (51%) patients had lymph node metastasis (N1a = 18/N1b = 19). Pre-operative ultrasound identified all cases with clinically significant nodal disease. Of the 73 patients, 39 (53\u00b74%) met ATA low-risk criteria and 16 (21\u00b79%) met 'very-low-risk' criteria. All 'very-low-risk' patients demonstrated excellent response to initial therapy without persistence/recurrence after a mean follow-up of 36\u00b74 months." ], "LABELS": [ "OBJECTIVE", "DESIGN", "PATIENTS", "MEASUREMENTS", "RESULTS" ], "MESHES": [ "Adolescent", "Child", "Endocrine Surgical Procedures", "Female", "Humans", "Lymphatic Metastasis", "Male", "Neoplasm Recurrence, Local", "Retrospective Studies", "Risk Assessment", "Thyroid Neoplasms", "Thyroidectomy" ], "YEAR": "2017", "reasoning_required_pred": "yes", "reasoning_free_pred": "yes", "final_decision": "yes", "LONG_ANSWER": "Ultrasound and histopathology identify a substantial population that may be candidates for lobectomy, avoiding the risks and potential medical and psychosocial morbidity associated with total thyroidectomy. We propose a clinical framework to stimulate discussion of lobectomy as an option for low-risk patients." }, "22117569": { "QUESTION": "Is an advance care planning model feasible in community palliative care?", "CONTEXTS": [ "An effective advance care planning programme involves an organizational wide commitment and preparedness for health service reform to embed advance care planning into routine practice. Internationally, such programmes have been implemented predominantly in aged and acute care with more recent work in primary care.", "A multi-site action research was conducted over a 16-month period in 2007-2009 with three Victorian community palliative care services. Using mixed method data collection strategies to assess feasibility, we conducted a baseline audit of staff and clients; analysed relevant documents (client records, policies, procedures and quality improvement strategies) pre-implementation and post-implementation and conducted key informant interviews (n\u2003=\u20039).", "Three community palliative care services: one regional and two metropolitan services in Victoria, Australia.", "The services demonstrated that it was feasible to embed the Model into their organizational structures. Advance care planning conversations and involvement of family was an important outcome measure rather than completion rate of advance care planning documents in community settings. Services adapted and applied their own concept of community, which widened the impact of the model. Changes to quality audit processes were essential to consolidate the model into routine palliative care practice." ], "LABELS": [ "BACKGROUND", "METHODS", "SETTINGS AND PARTICIPANTS", "RESULTS" ], "MESHES": [ "Adult", "Advance Care Planning", "Advance Directives", "Aged", "Aged, 80 and over", "Community Health Services", "Female", "Humans", "Male", "Middle Aged", "Models, Organizational", "Palliative Care", "Program Evaluation", "Rural Population", "Terminal Care", "Urban Population", "Victoria" ], "YEAR": "2012", "reasoning_required_pred": "yes", "reasoning_free_pred": "yes", "final_decision": "yes", "LONG_ANSWER": "An advance care planning model is feasible for community palliative care services. Quality audit processes are an essential component of the Model with documentation of advance care planning discussion established as an important outcome measure." }, "25787073": { "QUESTION": "Do preoperative serum C-reactive protein levels predict the definitive pathological stage in patients with clinically localized prostate cancer?", "CONTEXTS": [ "The serum C-reactive protein (CRP) level correlates with the clinical prognosis in patients with kidney, penile and metastatic castration-resistant prostate cancer (PC). We prospectively evaluated the preoperative CRP level as a predictive marker for an advanced tumor stage or high-grade cancer in patients with clinically localized PC.", "The study evaluated 629 patients with clinically localized PC who underwent radical prostatectomy between 2010 and 2013. Exclusion criteria were signs of systemic infection, symptoms of an autoimmune disease or neoadjuvant androgen deprivation.", "Poorly differentiated PC tends to be more common in patients with elevated CRP levels (15.5 vs. 9.5%, p = 0.08). Analogously, patients with a Gleason score \u22658 PC had significantly higher median CRP levels than those with a Gleason score \u22647 PC (1.9 vs. 1.2 mg/l, p = 0.03). However, neither uni- nor multivariate analysis showed an association between the preoperative CRP level and the presence of a locally advanced tumor stage, lymph node metastases or a positive surgical margin. CRP also failed to correlate with the initial PSA level and the clinical tumor-associated findings. Moreover, multivariate analysis relativized the association between an elevated CRP level and poor tumor differentiation." ], "LABELS": [ "PURPOSE", "METHODS", "RESULTS" ], "MESHES": [ "Adenocarcinoma", "Adult", "Aged", "Aged, 80 and over", "C-Reactive Protein", "Digital Rectal Examination", "Humans", "Lymphatic Metastasis", "Male", "Middle Aged", "Neoplasm Grading", "Neoplasm Staging", "Neoplasm, Residual", "Predictive Value of Tests", "Preoperative Period", "Prospective Studies", "Prostate-Specific Antigen", "Prostatectomy", "Prostatic Neoplasms" ], "YEAR": "2015", "reasoning_required_pred": "no", "reasoning_free_pred": "no", "final_decision": "no", "LONG_ANSWER": "In patients with clinically localized PC, CRP does not appear to possess the predictive value and it was shown to have patients with other tumor entities or advanced PC." }, "14631523": { "QUESTION": "Sub-classification of low-grade cerebellar astrocytoma: is it clinically meaningful?", "CONTEXTS": [ "The objectives were to identify prognostic factors for the survival of children with cerebellar astrocytoma, and to evaluate the reproducibility and prognostic value of histological sub-classification and grading.", "Children aged 0-14 years treated in Denmark for a cerebellar astrocytoma in the period 1960-1984 were included and followed until January 2001 or until their death. The histological specimens from each patient were reviewed for revised grading and classification according to three different classification schemes: the WHO, the Kernohan and the Daumas-Duport grading systems.", "The overall survival rate was 81% after a follow-up time of 15-40 years. The significant positive prognostic factors for survival were \"surgically gross-total removal\" of the tumour at surgery and location of the tumour in the cerebellum proper as opposed to location in the fourth ventricle. No difference in survival time was demonstrated when we compared pilocytic astrocytoma and fibrillary astrocytoma. Moreover, we found that the Kernohan and the WHO classification systems had no predictive value and that the Daumas-Duport system is unsuitable as a prognostic tool for low-grade posterior fossa astrocytomas." ], "LABELS": [ "OBJECTIVES", "METHODS", "RESULTS" ], "MESHES": [ "Adolescent", "Astrocytoma", "Cerebellar Neoplasms", "Child", "Child, Preschool", "Female", "Follow-Up Studies", "Histological Techniques", "Humans", "Infant", "Infant, Newborn", "Male", "Neurologic Examination", "Predictive Value of Tests", "Prognosis", "Retrospective Studies", "Survival Rate", "Time Factors", "World Health Organization", "alpha-Crystallin B Chain" ], "YEAR": "2003", "reasoning_required_pred": "no", "reasoning_free_pred": "no", "final_decision": "no", "LONG_ANSWER": "Discordant observations due to interobserver variability make histological sub-classification of low-grade cerebellar astrocytomas in children insufficient for predicting prognosis and biological behaviour. Similar survival rates in a population of paediatric low-grade cerebellar astrocytomas of grades I and II indicate that tumour grade has no prognostic significance within this group of patients. \"Surgically gross-total removal\", especially if the tumour is located in the fourth ventricle is of the highest importance for long-term survival. Histological sub-classification of the tumours has no predictive value." }, "26717802": { "QUESTION": "After CLASS--Is a voluntary public insurance program a realistic way to meet the long-term support and service needs of adults with disabilities?", "CONTEXTS": [ "The CLASS Act, which was part of the Affordable Care Act of 2010, established a voluntary personal assistance services (PAS) insurance program. However, concerns about enrollment and adverse selection led to repeal of the CLASS Act in\u00a02013.", "To estimate the number of middle-aged adults interested in purchasing PAS insurance, the sociodemographic, socioeconomic and disability attributes of this population, and the maximum monthly premium they would be willing to pay for such coverage.", "A total of 13,384 adults aged 40-65 answered questions about their interest in PAS insurance in the 2011 Sample Adult National Health Interview Survey. We applied survey weights for the U.S. population and conducted logistic regression analyses to identify personal factors associated with interest in paying for the CLASS program.", "An estimated 25.8 million adults aged 40-65 (26.7%) said they would be interested in paying for a public insurance program to cover PAS benefits. However, interest in PAS insurance varied by age, race, ethnicity, region, income, disability status, and family experience with ADL assistance. Only 1.6 million adults aged 40-65 (1.8%) said they would be willing to pay $100 per month or more for coverage." ], "LABELS": [ "BACKGROUND", "OBJECTIVES", "METHODS", "RESULTS" ], "MESHES": [ "Activities of Daily Living", "Adult", "Aged", "Attitude to Health", "Disabled Persons", "Female", "Health Care Reform", "Health Services Needs and Demand", "Home Health Aides", "Humans", "Insurance Coverage", "Insurance, Health", "Logistic Models", "Long-Term Care", "Male", "Middle Aged", "Patient Protection and Affordable Care Act", "Voluntary Programs" ], "YEAR": "2016", "reasoning_required_pred": "yes", "reasoning_free_pred": "no", "final_decision": "no", "LONG_ANSWER": "While more than a quarter of the middle-aged adult population said they were interested in PAS insurance, actual participation would be highly dependent on premium rates. The current lack of publicly subsidized insurance for long-term care and personal assistance services remains a serious gap in the disability service system." }, "23810330": { "QUESTION": "Is intraoperative neuromonitoring associated with better functional outcome in patients undergoing open TME?", "CONTEXTS": [ "Intraoperative neuromonitoring (IONM) aims to control nerve-sparing total mesorectal excision (TME) for rectal cancer in order to improve patients' functional outcome. This study was designed to compare the urogenital and anorectal functional outcome of TME with and without IONM of innervation to the bladder and the internal anal sphincter.", "A consecutive series of 150 patients with primary rectal cancer were analysed. Fifteen match pairs with open TME and combined urogenital and anorectal functional assessment at follow up were established identical regarding gender, tumour site, tumour stage, neoadjuvant radiotherapy and type of surgery. Urogenital and anorectal function was evaluated prospectively on the basis of self-administered standardized questionnaires, measurement of residual urine volume and longterm-catheterization rate.", "Newly developed urinary dysfunction after surgery was reported by 1 of 15 patients in the IONM group and by 6 of 15 in the control group (p\u00a0=\u00a00.031). Postoperative residual urine volume was significantly higher in the control group. At follow up impaired anorectal function was present in 1 of 15 patients undergoing TME with IONM and in 6 of 15 without IONM (p\u00a0=\u00a00.031). The IONM group showed a trend towards a lower rate of sexual dysfunction after surgery." ], "LABELS": [ "AIMS", "METHODS", "RESULTS" ], "MESHES": [ "Aged", "Aged, 80 and over", "Anal Canal", "Autonomic Pathways", "Case-Control Studies", "Cohort Studies", "Fecal Incontinence", "Female", "Humans", "Lower Urinary Tract Symptoms", "Male", "Middle Aged", "Monitoring, Intraoperative", "Organ Sparing Treatments", "Peripheral Nerve Injuries", "Prospective Studies", "Rectal Neoplasms", "Rectum", "Sexual Dysfunction, Physiological", "Treatment Outcome", "Urinary Bladder" ], "YEAR": "2013", "reasoning_required_pred": "yes", "reasoning_free_pred": "yes", "final_decision": "yes", "LONG_ANSWER": "In this study TME with IONM was associated with significant lower rates of urinary and anorectal dysfunction. Prospective randomized trials are mandatory to evaluate the definite role of IONM in rectal cancer surgery." }, "19027440": { "QUESTION": "Can we predict which head and neck cancer survivors develop fears of recurrence?", "CONTEXTS": [ "The objectives of this study were to investigate longitudinal predictors of fear of recurrence in survivors of head and neck cancer (HNC) using Leventhal's Common Sense Model (CSM) as a framework. The research questions were as follows: (a) to what extent do HNC patients report fear of cancer recurrence? (b) To what extent are fears of recurrence manifestations of illness-related anxiety? (c) Are fears of recurrence closely related to current symptoms, disease, and treatment-related factors, or psychological/socio-demographic factors? (d) What factors are predictive of long-term fears of recurrence?", "A prospective repeat measures design was employed whereby a sample of 82 newly diagnosed HNC patients (54 males, 28 females, mean age 60 years) completed measures of fears of recurrence, illness perceptions, coping, and anxiety and depression, prior to treatment and 6-8 months after treatment (fears of recurrence only).", "A third of HNC patients at diagnosis reported relatively high levels of fears of recurrence, with 12% still reporting similar levels of fear 6-8 months after treatment. Fears of recurrence were not related to any socio-demographic factor (age, gender, ethnicity, marital status, and educational attainment) or disease, treatment, or symptom-related factors. Path analyses demonstrated that fears of cancer recurrence after treatment were directly predicted by pre-treatment fears and optimism." ], "LABELS": [ "OBJECTIVES", "METHODS", "RESULTS" ], "MESHES": [ "Adaptation, Psychological", "Attitude to Health", "Breast Neoplasms", "Fear", "Female", "Head and Neck Neoplasms", "Humans", "Male", "Personality Inventory", "Prognosis", "Recurrence", "Stress, Psychological", "Surveys and Questionnaires", "Survivors" ], "YEAR": "2008", "reasoning_required_pred": "yes", "reasoning_free_pred": "yes", "final_decision": "yes", "LONG_ANSWER": "Although components of the CSM (cognitive and emotional representations, coping strategies) were associated with fears of recurrence, optimism was found to be the strongest predictor of fear, independent of anxiety and level of fear reported prior to treatment." }, "24446763": { "QUESTION": "The secular growth acceleration: does it appear during fetal life?", "CONTEXTS": [ "To test if secular growth acceleration occurs during fetal life.", "ANOVA Kruskal-Wallis and Mann-Whitney U-test have been used for the biometric characteristics comparison of nowadays fetal population with those three decades ago and to test the hypothesis about the existence of secular growth acceleration during fetal life. For this purpose, we first calculated mean values of particular biometric parameters for the whole pregnancy. During the period 2002-2009 biparietal diameter, fetal length and abdominal circumference measurements in singleton uncomplicated pregnancies between 22 and 41 gestational weeks were obtained. Gestational age was estimated according to Naegele's rule and confirmed with an early ultrasound examination. Pregnancies with fetal cromosomopathies and malformations were excluded as well as those resulting in perinatal death.", "There were no statistically significant differences of the examined fetal biometric parameters measured by ultrasound between contemporary fetal population and those from 35 years ago." ], "LABELS": [ "OBJECTIVE", "METHODS", "RESULTS" ], "MESHES": [ "Acceleration", "Birth Weight", "Female", "Fetal Development", "Gestational Age", "Humans", "Infant, Newborn", "Population Growth", "Pregnancy", "Reference Values", "Ultrasonography, Prenatal" ], "YEAR": "2014", "reasoning_required_pred": "no", "reasoning_free_pred": "no", "final_decision": "no", "LONG_ANSWER": "Our investigation did not undoubtedly prove that significant changes of fetal biometric parameters occurred in the last three decades. It is possible that secular growth acceleration does not exist in prenatal period but also the observed time period could have been short for this phenomenon to manifest." }, "9444542": { "QUESTION": "Does hippocampal atrophy on MRI predict cognitive decline?", "CONTEXTS": [ "To investigate whether the presence of hippocampal atrophy (HCA) on MRI in Alzheimer's disease (AD) leads to a more rapid decline in cognitive function. To investigate whether cognitively unimpaired controls and depressed subjects with HCA are at higher risk than those without HCA of developing dementia.", "A prospective follow-up of subjects from a previously reported MRI study.", "Melbourne, Australia.", "Five controls with HCA and five age-matched controls without HCA, seven depressed subjects with HCA and seven without HCA, and 12 subjects with clinically diagnosed probable AD with HCA and 12 without HCA were studied. They were followed up at approximately 2 years with repeat cognitive testing, blind to initial diagnosis and MRI result.", "HCA was rated by two radiologists blind to cognitive test score results. Cognitive assessment was by the Cambridge Cognitive Examination (CAMCOG).", "No significant differences in rate of cognitive decline, mortality or progression to dementia were found between subjects with or without HCA." ], "LABELS": [ "OBJECTIVES", "DESIGN", "SETTING", "PARTICIPANTS", "MEASURES", "RESULTS" ], "MESHES": [ "Aged", "Aged, 80 and over", "Alzheimer Disease", "Atrophy", "Case-Control Studies", "Cognition Disorders", "Depression", "Disease Progression", "Female", "Follow-Up Studies", "Geriatric Assessment", "Hippocampus", "Humans", "Magnetic Resonance Imaging", "Male", "Middle Aged", "Prognosis", "Prospective Studies" ], "YEAR": "1997", "reasoning_required_pred": "no", "reasoning_free_pred": "no", "final_decision": "no", "LONG_ANSWER": "HCA was not found to be a predictor of subsequent cognitive decline in this series." }, "23389866": { "QUESTION": "Chemoradiotherapy in the management of locally advanced squamous cell carcinoma esophagus: is surgical resection required?", "CONTEXTS": [ "The present study aims to evaluate benefit of adding surgery to chemoradiotherapy alone in management of carcinoma esophagus.", "We retrospectively analyzed 45 eligible patients of squamous cell esophageal carcinoma which were enrolled from February 2008 to April 2009. All patients were treated with chemoradiotherapy (50.40 Gy with 40 mg/m(2) of weekly cisplatin). Tumor response was assessed after 6 weeks of treatment. Patients with resectable disease were subjected to surgical resection (arm A) and remaining was kept on regular clinical follow-up (arm B). Overall survival (OS) was selected as the primary endpoint. The secondary end points were disease-free survival (DFS) and clinical toxicities.", "Median follow-up was 13.6 months. Pathological complete response was seen in 60.9 % patients in arm A. In arm B, 77.3 % patients attained radiological complete response (p\u2009=\u20090.194). The median OS was 16.4 and 19.1 months (p\u2009=\u20090.388) and median DFS was 5.8 and 4.1 months (p\u2009=\u20090.347) in arm A and B, respectively. The 2-year survival probability was 39.1 and 36.4 % (p\u2009=\u20090.387) in arm A and B, respectively. The recurrence probability was 56.5 % (SE\u2009=\u20095.6 %) and 45.5 % (SE\u2009=\u20094.2 %) (p\u2009=\u20090.328) in arm A and B, respectively. The probability of loco regional recurrence was more in arm B than in arm A (p\u2009=\u20090.002)." ], "LABELS": [ "PURPOSE", "METHODS", "RESULTS" ], "MESHES": [ "Antineoplastic Combined Chemotherapy Protocols", "Carcinoma, Squamous Cell", "Combined Modality Therapy", "Esophageal Neoplasms", "Esophagectomy", "Female", "Follow-Up Studies", "Humans", "Lymphatic Metastasis", "Male", "Middle Aged", "Neoplasm Recurrence, Local", "Neoplasm Staging", "Prognosis", "Radiotherapy, Intensity-Modulated", "Retrospective Studies", "Survival Rate" ], "YEAR": "2013", "reasoning_required_pred": "yes", "reasoning_free_pred": "no", "final_decision": "no", "LONG_ANSWER": "The study suggests that there is no difference in clinical toxicity profiles or survival outcomes with either definitive chemoradiotherapy or chemoradiation followed by surgery in management of locally advanced esophageal cancer." }, "19142546": { "QUESTION": "Does quantitative left ventricular regional wall motion change after fibrous tissue resection in endomyocardial fibrosis?", "CONTEXTS": [ "We compared left ventricular regional wall motion, the global left ventricular ejection fraction, and the New York Heart Association functional class pre- and postoperatively.", "Endomyocardial fibrosis is characterized by fibrous tissue deposition in the endomyocardium of the apex and/or inflow tract of one or both ventricles. Although left ventricular global systolic function is preserved, patients exhibit wall motion abnormalities in the apical and inferoapical regions. Fibrous tissue resection in New York Heart Association FC III and IV endomyocardial fibrosis patients has been shown to decrease morbidity and mortality.", "We prospectively studied 30 patients (20 female, 30+/-10 years) before and 5+/-8 months after surgery. The left ventricular ejection fraction was determined using the area-length method. Regional left ventricular motion was measured by the centerline method. Five left ventricular segments were analyzed pre- and postoperatively. Abnormality was expressed in units of standard deviation from the mean motion in a normal reference population.", "Left ventricular wall motion in the five regions did not differ between pre- and postoperative measurements. Additionally, the left ventricular ejection fraction did not change after surgery (0.45+/-0.13% x 0.43+/-0.12% pre- and postoperatively, respectively). The New York Heart Association functional class improved to class I in 40% and class II in 43% of patients postoperatively (p<0.05)." ], "LABELS": [ "OBJECTIVES", "INTRODUCTION", "METHODS", "RESULTS" ], "MESHES": [ "Adult", "Diastole", "Endomyocardial Fibrosis", "Female", "Heart Ventricles", "Humans", "Male", "Postoperative Complications", "Prospective Studies", "Severity of Illness Index", "Statistics, Nonparametric", "Stroke Volume", "Ventricular Dysfunction, Left" ], "YEAR": "2009", "reasoning_required_pred": "no", "reasoning_free_pred": "no", "final_decision": "no", "LONG_ANSWER": "Although endomyocardial fibrosis patients have improved clinical symptoms after surgery, the global left ventricular ejection fraction and regional wall motion in these patients do not change. This finding suggests that other explanations, such as improvements in diastolic function, may be operational." }, "9107172": { "QUESTION": "Bridge experience with long-term implantable left ventricular assist devices. Are they an alternative to transplantation?", "CONTEXTS": [ "If long-term use of left ventricular assist devices (LVADs) as bridges to transplantation is successful, the issue of permanent device implantation in lieu of transplantation could be addressed through the creation of appropriately designed trials. Our medium-term experience with both pneumatically and electrically powered ThermoCardiosystems LVADs is presented to outline the benefits and limitations of device support in lieu of transplantation.", "Detailed records were kept prospectively for all patients undergoing LVAD insertion. Fifty-eight LVADs were inserted over 5 years, with a survival rate of 74%. Mean patient age was 50 years, and duration of support averaged 98 days. Although common, both preexisting infection and infection during LVAD support were not associated with increased mortality or decreased rate of successful transplantation. Thromboembolic complications were rare, occurring in only three patients (5%) despite the absence of anticoagulation. Ventricular arrhythmias were well tolerated in all patients except in cases of early perioperative right ventricular failure, with no deaths. Right ventricular failure occurred in one third of patients and was managed in a small percentage by right ventricular assist device (RVAD) support and/or inhaled nitric oxide therapy. There were no serious device malfunctions, but five graft-related hemorrhages resulted in two deaths. Finally, a variety of noncardiac surgical procedures were performed in LVAD recipients, with no major morbidity and mortality." ], "LABELS": [ "BACKGROUND", "METHODS AND RESULTS" ], "MESHES": [ "Aged", "Arrhythmias, Cardiac", "Assisted Circulation", "Cardiomyopathies", "Combined Modality Therapy", "Comorbidity", "Cost-Benefit Analysis", "Endocarditis", "Equipment Design", "Female", "Follow-Up Studies", "Heart Failure", "Heart Transplantation", "Heart-Assist Devices", "Hemorrhage", "Humans", "Infection", "Length of Stay", "Male", "Middle Aged", "Nitric Oxide", "Postoperative Complications", "Prospective Studies", "Prostheses and Implants", "Survival Analysis", "Thromboembolism" ], "YEAR": "1997", "reasoning_required_pred": "yes", "reasoning_free_pred": "yes", "final_decision": "yes", "LONG_ANSWER": "Over all, our medium-term experience with implantable LVAD support is encouraging. Although additional areas of investigation exist, improvements in patients selection and management together with device alterations that have reduced the thromboembolic incidence and facilitated patient rehabilitation lead us to believe that a prospective, randomized trial is indicated to study the role that LVADs may have as an alternative to medical management." }, "9363244": { "QUESTION": "Does occupational nuclear power plant radiation affect conception and pregnancy?", "CONTEXTS": [ "To determine the effect of occupational exposure in a nuclear power plant in Griefswald, Germany on male and female fecundity.", "The frequency of men and women exposed to ionizing radiation through work in a nuclear power plant among 270 infertile couples was retrospectively compared to a control fertile population using a pair-matched analysis. The total cumulative equivalent radiation dose was determined. In addition, the spermiograms of the male partners in both groups were compared and correlated to the degree of exposure.", "No differences were noted in the frequency of nuclear power plant exposure between sterile and fertile groups. There was a higher rate of anomalous spermiograms in nuclear power plant workers. However, abnormalities were temporary. No correlation was found between the cumulative equivalent radiation dose and abnormal spermiograms." ], "LABELS": [ "OBJECTIVES", "METHODS", "RESULTS" ], "MESHES": [ "Female", "Humans", "Infertility", "Male", "Occupational Exposure", "Power Plants", "Pregnancy", "Radiation Dosage", "Retrospective Studies", "Sperm Count", "Spermatozoa" ], "YEAR": "1995", "reasoning_required_pred": "no", "reasoning_free_pred": "yes", "final_decision": "yes", "LONG_ANSWER": "The data suggest that occupational exposure due to ionizing radiation should be investigated as a possible cause for involuntary temporary sterility and as a risk factor for early pregnancy disorders." }, "10411439": { "QUESTION": "Lloyd-Davies position with Trendelenburg--a disaster waiting to happen?", "CONTEXTS": [ "Lower limb compartment syndrome has been reported to occur after colorectal, urological, and gynecological procedures during which the patient's lower limbs are elevated for prolonged periods of time.", "We investigated lower limb perfusion in a group of patients undergoing prolonged pelvic surgery both during and immediately after surgery, using intra-arterial blood pressure monitoring, laser doppler flowmetry, and pulse oximetry.", "Use of the modified lithotomy position was not associated with any demonstrable decrease in lower limb perfusion. The addition of 15 degrees head-down tilt, however, during pelvic dissection, led to an immediate and significant drop in lower limb perfusion (P<0.05; Mann-Whitney U test). The subgroup of patients analyzed postoperatively showed a ten-fold increase (P<0.01) in perfusion that was confined to the muscle compartment with no demonstrable increase in skin perfusion or intra-arterial pedal blood pressure." ], "LABELS": [ "PURPOSE", "METHOD", "RESULTS" ], "MESHES": [ "Colectomy", "Colonic Diseases", "Compartment Syndromes", "Head-Down Tilt", "Humans", "Ischemia", "Leg", "Postoperative Complications", "Rectal Diseases" ], "YEAR": "1999", "reasoning_required_pred": "no", "reasoning_free_pred": "yes", "final_decision": "yes", "LONG_ANSWER": "The use of the modified lithotomy position during pelvic surgery is not associated with lower limb ischemia. Addition of Trendelenburg position, however, causes profound ischemia of the lower limbs, and this is followed during the recovery period by hyperperfusion that is confined to the muscle compartments, which may put patients at risk of developing lower limb compartment syndrome." }, "21166749": { "QUESTION": "Is patient outcome compromised during the initial experience with robot-assisted radical cystectomy?", "CONTEXTS": [ "\u2022 Robot-assisted radical cystectomy (RARC) remains controversial in terms of oncologic outcomes, especially during the initial experience. The purpose of this study was to evaluate the impact of initial experience of robotic cystectomy programs on oncologic outcomes and overall survival.", "\u2022 Utilizing a prospectively maintained, single institution robotic cystectomy database, we identified 164 consecutive patients who underwent RARC since November 2005. \u2022 After stratification by age group, gender, pathologic T stage, lymph node status, surgical margin status, and sequential case number; we used chi-squared analyses to correlate sequential case number to operative time, surgical blood loss, lymph node yield, and surgical margin status. \u2022 We also addressed the relationship between complications and sequential case number. We then utilized Cox proportional hazard modeling and Kaplan-Meier survival analyses to correlate variables to overall mortality.", "\u2022 Sequential case number was not significantly associated with increased incidence of complications, surgical blood loss, or positive surgical margins (P= 0.780, P= 0.548, P= 0.545). Case number was, however, significantly associated with shorter operative time and mean number of lymph nodes retrieved (P<0.001, P<0.001). \u2022 Sequential case number was not significantly associated with survival; however, tumour stage, the presence of lymph node metastases, and positive surgical margins were significantly associated with death. \u2022 Although being the largest of its kind, this was a small study with short follow-up when compared to open cystectomy series." ], "LABELS": [ "OBJECTIVE", "PATIENTS AND METHODS", "RESULTS" ], "MESHES": [ "Adult", "Age Factors", "Aged", "Aged, 80 and over", "Blood Loss, Surgical", "Chemotherapy, Adjuvant", "Clinical Competence", "Cystectomy", "Female", "Humans", "Learning Curve", "Lymphatic Metastasis", "Male", "Middle Aged", "Postoperative Complications", "Prospective Studies", "Risk Factors", "Robotics", "Survival Analysis", "Treatment Outcome", "Urinary Bladder Neoplasms", "Urology" ], "YEAR": "2011", "reasoning_required_pred": "yes", "reasoning_free_pred": "no", "final_decision": "no", "LONG_ANSWER": "\u2022 Initial experience with RARC did not affect the incidence of positive surgical margins, operative/postoperative complications, or overall survival in a single-institution series." }, "27818079": { "QUESTION": "Is the Retromandibular Transparotid Approach a Reliable Option for the Surgical Treatment of Condylar Fractures?", "CONTEXTS": [ "This study evaluated the outcomes and complications of the surgical treatment of condylar fractures by the retromandibular transparotid approach. The authors hypothesized that such an approach would be safe and reliable for the treatment of most condylar fractures.", "A retrospective evaluation of patients who underwent surgical reduction of a condylar fracture from January 2012 to December 2014 at the Clinic of Dentistry and Maxillofacial Surgery of the University Hospital of Verona (Verona, Italy) was performed. Inclusion criteria were having undergone surgical treatment of condylar fractures with a retromandibular transparotid approach and the availability of computed tomograms of the preoperative and postoperative facial skeleton with a minimum follow-up of 1\u00a0year. Static and dynamic occlusal function, temporomandibular joint health status, presence of neurologic impairments, and esthetic outcomes were evaluated in all patients.", "The sample was composed of 25 patients. Preinjury occlusion and temporomandibular joint health were restored in most patients. Esthetic outcomes were deemed satisfactory by clinicians and patients. Neither permanent neurologic impairments nor major postoperative complications were observed." ], "LABELS": [ "PURPOSE", "MATERIALS AND METHODS", "RESULTS" ], "MESHES": [ "Adolescent", "Adult", "Aged", "Female", "Humans", "Male", "Mandibular Condyle", "Mandibular Fractures", "Middle Aged", "Retrospective Studies", "Tomography, X-Ray Computed", "Young Adult" ], "YEAR": "2017", "reasoning_required_pred": "yes", "reasoning_free_pred": "yes", "final_decision": "yes", "LONG_ANSWER": "According to the results of the present study, the retromandibular transparotid approach is a viable and safe approach for the surgical treatment of condylar fractures." }, "25982163": { "QUESTION": "Appendectomy timing: Will delayed surgery increase the complications?", "CONTEXTS": [ "This study investigated whether the time from emergency room registration to appendectomy (ETA) would affect the incidence of perforation and postoperative complications in patients with acute appendicitis.", "Patients who underwent an appendectomy at the Ren-Ai branch of Taipei City Hospital between January 2010 and October 2012 were retrospectively reviewed. Their demographics, white blood cell count, C-reactive protein, body temperature, computed tomography scan usage, operation method, pathology report, postoperative complication, length of hospital stay, and ETA were abstracted. Multivariate analysis was performed to search the predictors, including ETA, of outcomes for the perforation and postoperative complication rates.", "A total of 236 patients were included in the study. Perforation occurred in 12.7% (30/236) and postoperative complications developed in 24.1% (57/236) of these patients. There were 121 patients with ETA<8 hours, 88 patients with ETA of 8-24 hours, and 27 patients with ETA>24 hours; patients with ETA>24 hours had significantly longer hospital stay. Univariate analysis showed that perforated patients were significantly older, and had higher C-reactive protein level, longer hospital stay, and higher complication rate. Patients who developed postoperative complications were significantly older, and had higher neutrophil count, less use of computed tomography, and higher open appendectomy rate. After multivariate analysis, age \u226555 years was the only predictor for perforation [odds ratio (OR) = 3.65; 95% confidence interval (CI), 1.54-8.68]; for postoperative complications, age \u226555 years (OR = 1.65; 95% CI, 1.84-3.25), perforated appendicitis (OR = 3.17; 95% CI, 1.28-7.85), and open appendectomy (OR = 3.21; 95% CI, 1.36-7.58) were associated. ETA was not a significant predictor in both analyses." ], "LABELS": [ "BACKGROUND", "METHODS", "RESULTS" ], "MESHES": [ "Adult", "Aged", "Appendectomy", "C-Reactive Protein", "Female", "Humans", "Length of Stay", "Male", "Middle Aged", "Postoperative Complications", "Retrospective Studies", "Time Factors" ], "YEAR": "2015", "reasoning_required_pred": "no", "reasoning_free_pred": "no", "final_decision": "no", "LONG_ANSWER": "In our study, it was observed that although longer ETA was associated with longer hospitalization, ETA was not correlated with postoperative complications. Our results inclined toward the position that appendectomy can be performed as a semielective surgery." }, "20353735": { "QUESTION": "Treatment of vitamin D deficiency in CKD patients with ergocalciferol: are current K/DOQI treatment guidelines adequate?", "CONTEXTS": [ "Vitamin D deficiency/insufficiency (VDDI) is common in CKD patients and may be associated with abnormal mineral metabolism. It is not clear whether the K/DOQI recommended doses of ergocalciferol are adequate for correction of VDDI and hyperparathyroidism.", "Retrospective study of 88 patients with CKD Stages 1 - 5 and baseline 25-hydroxyvitamin D level<30 ng/ml (<75 nmol/l). Patients treated with ergocalciferol as recommended by K/DOQI guidelines. Only 53 patients had elevated baseline PTH level for the CKD stage. Patients were excluded if they received vitamin D preparations other than ergocalciferol or phosphate binders. 25-hydroxyvitamin D level, intact PTH level (iPTH), and other parameters of mineral metabolism were measured at baseline and after completion of ergocalciferol course.", "88 patients with CKD were treated with ergocalciferol. Mean age 56.8 +/- 9.5 years and 41% were males. The mean (+/- SD) GFR was 28.3 +/- 16.6 ml/min. At the end of the 6-month period of ergocalciferol treatment, the mean 25-hydroxyvitamin D level increased from 15.1 +/- 5.8 to 23.3 +/- 11.8 ng/ml (37.75 +/- 14.5 to 58.25 +/- 29.5 nmol/l) (p<0.001). Treatment led to>or = 5 ng/ml (12.5 nmol/l) increases in 25-hydroxyvitamin D level in 54% of treated patients, and only 25% achieved levels>or = 30 ng/ml (75 nmol/l). Mean iPTH level decreased from 157.9 +/- 125.9 to 150.7 +/- 127.5 pg/ml (p = 0.5). Only 26% of patients had>or = 30% decrease in their iPTH level after treatment with ergocalciferol." ], "LABELS": [ "BACKGROUND", "METHODS", "RESULTS" ], "MESHES": [ "Aged", "Chi-Square Distribution", "Dose-Response Relationship, Drug", "Drug Administration Schedule", "Ergocalciferols", "Female", "Humans", "Kidney Failure, Chronic", "Male", "Middle Aged", "Parathyroid Hormone", "Patient Selection", "Practice Guidelines as Topic", "Regression Analysis", "Retrospective Studies", "Treatment Outcome", "Vitamin D", "Vitamin D Deficiency", "Vitamins" ], "YEAR": "2010", "reasoning_required_pred": "no", "reasoning_free_pred": "no", "final_decision": "no", "LONG_ANSWER": "Current K/DOQI guidelines are inadequate for correcting VDDI or secondary hyperparathyroidism in CKD patients. Future studies should examine the effects of higher or more frequent dosing of ergocalciferol on these clinical endpoints." }, "24630333": { "QUESTION": "Comparing femoral version after intramedullary nailing performed by trauma-trained and non-trauma trained surgeons: is there a difference?", "CONTEXTS": [ "As with some procedures, trauma fellowship training and greater surgeon experience may result in better outcomes following intramedullary nailing (IMN) of diaphyseal femur fractures. However, surgeons with such training and experience may not always be available to all patients. The purpose of this study is to determine whether trauma training affects the post-operative difference in femoral version (DFV) following IMN.", "Between 2000 and 2009, 417 consecutive patients with diaphyseal femur fractures (AO/OTA 32A-C) were treated via IMN. Inclusion criteria for this study included complete baseline and demographic documentation as well as pre-operative films for fracture classification and post-operative CT scanogram (per institutional protocol) for version and length measurement of both the nailed and uninjured femurs. Exclusion criteria included bilateral injuries, multiple ipsilateral lower extremity fractures, previous injury, and previous deformity. Of the initial 417 subjects, 355 patients met our inclusion criteria. Other data included in our analysis were age, sex, injury mechanism, open vs. closed fracture, daytime vs. nighttime surgery, mechanism of injury, and AO and Winquist classifications. Post-operative femoral version of both lower extremities was measured on CT scanogram by an orthopaedic trauma fellowship trained surgeon. Standard univariate and multivariate analyses were performed to determine statistically significant risk factors for malrotation between the two cohorts.", "Overall, 80.3% (288/355) of all fractures were fixed by trauma-trained surgeons. The mean post-operative DFV was 8.7\u00b0 in these patients, compared to 10.7\u00b0 in those treated by surgeons of other subspecialties. This difference was not statistically significant when accounting for other factors in a multivariate model (p>0.05). The same statistical trend was true when analyzing outcomes of only the more severe Winquist type III and IV fractures. Additionally, surgeon experience was not significantly predictive of post-operative version for either trauma or non-trauma surgeons (p>0.05 for both)." ], "LABELS": [ "INTRODUCTION", "MATERIALS AND METHODS", "RESULTS" ], "MESHES": [ "Adult", "Clinical Competence", "Fellowships and Scholarships", "Female", "Femoral Fractures", "Fracture Fixation, Intramedullary", "Humans", "Male", "Orthopedics", "Postoperative Complications", "Range of Motion, Articular", "Retrospective Studies", "Specialties, Surgical", "Surgeons", "Tomography, X-Ray Computed", "Traumatology", "Treatment Outcome" ], "YEAR": "2014", "reasoning_required_pred": "no", "reasoning_free_pred": "no", "final_decision": "no", "LONG_ANSWER": "Post-operative version or percentage of DFV>15\u00b0 did not significantly differ following IMN of diaphyseal femur fractures between surgeons with and without trauma fellowship training. However, prospective data that removes the inherent bias that the more complex cases are left for the traumatologists are required before a definitive comparison is made." }, "27096199": { "QUESTION": "Does Viral Co-Infection Influence the Severity of Acute Respiratory Infection in Children?", "CONTEXTS": [ "Multiple viruses are often detected in children with respiratory infection but the significance of co-infection in pathogenesis, severity and outcome is unclear.", "To correlate the presence of viral co-infection with clinical phenotype in children admitted with acute respiratory infections (ARI).", "We collected detailed clinical information on severity for children admitted with ARI as part of a Spanish prospective multicenter study (GENDRES network) between 2011-2013. A nested polymerase chain reaction (PCR) approach was used to detect respiratory viruses in respiratory secretions. Findings were compared to an independent cohort collected in the UK.", "204 children were recruited in the main cohort and 97 in the replication cohort. The number of detected viruses did not correlate with any markers of severity. However, bacterial superinfection was associated with increased severity (OR: 4.356; P-value = 0.005), PICU admission (OR: 3.342; P-value = 0.006), higher clinical score (1.988; P-value = 0.002) respiratory support requirement (OR: 7.484; P-value<0.001) and longer hospital length of stay (OR: 1.468; P-value<0.001). In addition, pneumococcal vaccination was found to be a protective factor in terms of degree of respiratory distress (OR: 2.917; P-value = 0.035), PICU admission (OR: 0.301; P-value = 0.011), lower clinical score (-1.499; P-value = 0.021) respiratory support requirement (OR: 0.324; P-value = 0.016) and oxygen necessity (OR: 0.328; P-value = 0.001). All these findings were replicated in the UK cohort." ], "LABELS": [ "BACKGROUND", "OBJECTIVES", "METHODS", "RESULTS" ], "MESHES": [ "Acute Disease", "Adult", "Child, Preschool", "Coinfection", "Female", "Hospitalization", "Humans", "Infant", "Infant, Newborn", "Male", "Middle Aged", "Phenotype", "Pneumococcal Vaccines", "Respiratory Tract Infections" ], "YEAR": "2016", "reasoning_required_pred": "no", "reasoning_free_pred": "no", "final_decision": "no", "LONG_ANSWER": "The presence of more than one virus in hospitalized children with ARI is very frequent but it does not seem to have a major clinical impact in terms of severity. However bacterial superinfection increases the severity of the disease course. On the contrary, pneumococcal vaccination plays a protective role." }, "22519710": { "QUESTION": "Will clinical studies elucidate the connection between the length of storage of transfused red blood cells and clinical outcomes?", "CONTEXTS": [ "The temporal pattern of the biologic mechanism linking red blood cell (RBC) storage duration with clinical outcomes is yet unknown. This study investigates how such a temporal pattern can affect the power of randomized controlled trials (RCT) to detect a relevant clinical outcome mediated by the transfusion of stored RBCs.", "This study was a computer simulation of four RCTs, each using a specific categorization of the RBC storage time. The trial's endpoint was evaluated assuming five hypothetical temporal patterns for the biologic mechanism linking RBC storage duration with clinical outcomes.", "Power of RCTs to unveil a significant association between RBC storage duration and clinical outcomes was critically dependent on a complex interaction among three factors: 1) the way the RBC storage time is categorized in the trial design, 2) the temporal pattern assumed for the RBC storage lesion, and 3) the age distribution of RBCs in the inventory from which they are picked up for transfusion. For most combinations of these factors, the power of RCTs to detect a significant treatment effect was below 80%. All the four simulated RCTs had a very low power to disclose a harmful clinical effect confined to last week of the maximum 42-day shelf life of stored RBCs." ], "LABELS": [ "BACKGROUND", "STUDY DESIGN AND METHODS", "RESULTS" ], "MESHES": [ "Blood Preservation", "Computer Simulation", "Erythrocyte Transfusion", "Erythrocytes", "Humans", "Randomized Controlled Trials as Topic", "Time Factors" ], "YEAR": "2013", "reasoning_required_pred": "no", "reasoning_free_pred": "no", "final_decision": "no", "LONG_ANSWER": "Ongoing RCTs may lack enough power to settle the issue of whether or not the transfusion of stored blood has a negative clinical impact. A precautionary reduction of the maximum storage time to 35 days is advisable." }, "12040336": { "QUESTION": "Cardiogenic shock complicating acute myocardial infarction in elderly patients: does admission to a tertiary center improve survival?", "CONTEXTS": [ "The role of early revascularization among patients with acute myocardial infarction complicated by cardiogenic shock remains controversial. Angioplasty registries, while suggesting a benefit, are subject to selection bias, and clinical trials have been underpowered to detect early benefits. If an invasive strategy is beneficial in this population, patients admitted to hospitals with onsite coronary revascularization might be expected to have a better prognosis. We sought to determine whether access to cardiovascular resources at the admitting hospital influenced the prognosis of patients with acute myocardial infarction complicated by cardiogenic shock.", "By use of the Cooperative Cardiovascular Project database (a retrospective medical record review of Medicare patients discharged with acute myocardial infarction), we identified patients aged>or =65 years whose myocardial infarction was complicated by cardiogenic shock.", "Of the 601 patients with cardiogenic shock, 287 (47.8%) were admitted to hospitals without revascularization services and 314 (52.2%) were admitted to hospitals with coronary angioplasty and coronary artery bypass surgery facilities. Clinical characteristics were similar across the subgroups. Patients admitted to hospitals with revascularization services were more likely to undergo coronary revascularization during the index hospitalization and during the first month after acute myocardial infarction. After adjustment for demographic, clinical, hospital, and treatment strategies, the presence of onsite revascularization services was not associated with a significantly lower 30-day (odds ratio 0.83, 95% CI 0.47, 1.45) or 1-year mortality (odds ratio 0.91, 95% CI 0.49, 1.72)." ], "LABELS": [ "BACKGROUND", "METHODS", "RESULTS" ], "MESHES": [ "Aged", "Angioplasty, Balloon, Coronary", "Cohort Studies", "Coronary Artery Bypass", "Coronary Care Units", "Databases, Factual", "Female", "Hospitalization", "Humans", "Male", "Myocardial Infarction", "Myocardial Revascularization", "Odds Ratio", "Prognosis", "Shock, Cardiogenic" ], "YEAR": "2002", "reasoning_required_pred": "yes", "reasoning_free_pred": "no", "final_decision": "no", "LONG_ANSWER": "In a community-based cohort, patients with acute myocardial infarction complicated by cardiogenic shock did not have significantly different adjusted 30-day and 1-year mortality, irrespective of the revascularization capabilities of the admitting hospital." }, "15918864": { "QUESTION": "Learning needs of postpartum women: does socioeconomic status matter?", "CONTEXTS": [ "Little is known about how information needs change over time in the early postpartum period or about how these needs might differ given socioeconomic circumstances. This study's aim was to examine women's concerns at the time of hospital discharge and unmet learning needs as self-identified at 4 weeks after discharge.", "Data were collected as part of a cross-sectional survey of postpartum health outcomes, service use, and costs of care in the first 4 weeks after postpartum hospital discharge. Recruitment of 250 women was conducted from each of 5 hospitals in Ontario, Canada (n = 1,250). Women who had given vaginal birth to a single live infant, and who were being discharged at the same time as their infant, assuming care of their infant, competent to give consent, and able to communicate in one of the study languages were eligible. Participants completed a self-report questionnaire in hospital; 890 (71.2%) took part in a structured telephone interview 4 weeks after hospital discharge.", "Approximately 17 percent of participants were of low socioeconomic status. Breastfeeding and signs of infant illness were the most frequently identified concerns by women, regardless of their socioeconomic status. Signs of infant illness and infant care/behavior were the main unmet learning needs. Although few differences in identified concerns were evident, women of low socioeconomic status were significantly more likely to report unmet learning needs related to 9 of 10 topics compared with women of higher socioeconomic status. For most topics, significantly more women of both groups identified learning needs 4 weeks after discharge compared with the number who identified corresponding concerns while in hospital." ], "LABELS": [ "BACKGROUND", "METHODS", "RESULTS" ], "MESHES": [ "Adolescent", "Adult", "Attitude to Health", "Breast Feeding", "Female", "Humans", "Ontario", "Patient Education as Topic", "Postpartum Period", "Socioeconomic Factors", "Surveys and Questionnaires", "Time Factors" ], "YEAR": "2005", "reasoning_required_pred": "no", "reasoning_free_pred": "yes", "final_decision": "yes", "LONG_ANSWER": "It is important to ensure that new mothers are adequately informed about topics important to them while in hospital. The findings highlight the need for accessible and appropriate community-based information resources for women in the postpartum period, especially for those of low socioeconomic status." }, "23601294": { "QUESTION": "Pitfalls in urinary stone identification using CT attenuation values: are we getting the same information on different scanner models?", "CONTEXTS": [ "Evaluate the capability of different Computed Tomography scanners to determine urinary stone compositions based on CT attenuation values and to evaluate potential differences between each model.", "241 human urinary stones were obtained and their biochemical composition determined. Four different CT scanners (Siemens, Philips, GEMS and Toshiba) were evaluated. Mean CT-attenuation values and the standard deviation were recorded separately and compared with a t-paired test.", "For all tested CT scanners, when the classification of the various types of stones was arranged according to the mean CT-attenuation values and to the confidence interval, large overlappings between stone types were highlighted. The t-paired test showed that most stone types could not be identified. Some types of stones presented mean CT attenuation values significantly different from one CT scanner to another. At 80kV, the mean CT attenuation values obtained with the Toshiba Aquilion were significantly different from those obtained with the Siemens Sensation. On the other hand, mean values obtained with the Philips Brilliance were all significantly equal to those obtained with the Siemens Sensation and with the Toshiba Aquilion. At 120kV mean CT attenuation values of uric acid, cystine and struvite stones obtained with the Philips model are significantly different from those obtained with the Siemens and the Toshiba but equal to those obtained with the GE 64." ], "LABELS": [ "INTRODUCTION", "METHODS", "RESULTS" ], "MESHES": [ "Equipment Design", "Equipment Failure Analysis", "Humans", "Radiographic Image Enhancement", "Reproducibility of Results", "Sensitivity and Specificity", "Tomography, X-Ray Computed", "Urinary Calculi" ], "YEAR": "2013", "reasoning_required_pred": "no", "reasoning_free_pred": "yes", "final_decision": "no", "LONG_ANSWER": "According to our study, there is a great variability when different brands and models of scanners are compared directly. Furthermore, the CT scan analysis and HU evaluation appears to gather insufficient information in order to characterize and identify the composition of renal stones." }, "21361755": { "QUESTION": "Laminoplasty outcomes: is there a difference between patients with degenerative stenosis and those with ossification of the posterior longitudinal ligament?", "CONTEXTS": [ "Two common causes of cervical myelopathy include degenerative stenosis and ossification of the posterior longitudinal ligament (OPLL). It has been postulated that patients with OPLL have more complications and worse outcomes than those with degenerative stenosis. The authors sought to compare the surgical results of laminoplasty in the treatment of cervical stenosis with myelopathy due to either degenerative changes or segmental OPLL.", "The authors conducted a retrospective review of 40 instrumented laminoplasty cases performed at a single institution over a 4-year period to treat cervical myelopathy without kyphosis. Twelve of these patients had degenerative cervical stenotic myelopathy ([CSM]; degenerative group), and the remaining 28 had segmental OPLL (OPLL group). The 2 groups had statistically similar demographic characteristics and number of treated levels (mean 3.9 surgically treated levels; p>0.05). The authors collected perioperative and follow-up data, including radiographic results.", "The overall clinical follow-up rate was 88%, and the mean clinical follow-up duration was 16.4 months. The mean radiographic follow-up rate was 83%, and the mean length of radiographic follow-up was 9.3 months. There were no significant differences in the estimated blood loss (EBL) or length of hospital stay (LOS) between the groups (p>0.05). The mean EBL and LOS for the degenerative group were 206 ml and 3.7 days, respectively. The mean EBL and LOS for the OPLL group were 155 ml and 4 days, respectively. There was a statistically significant improvement of more than one grade in the Nurick score for both groups following surgery (p<0.05). The Nurick score improvement was not statistically different between the groups (p>0.05). The visual analog scale (VAS) neck pain scores were similar between groups pre- and postoperatively (p>0.05). The complication rates were not statistically different between groups either (p>0.05). Radiographically, both groups lost extension range of motion (ROM) following laminoplasty, but this change was not statistically significant (p>0.05)." ], "LABELS": [ "OBJECT", "METHODS", "RESULTS" ], "MESHES": [ "Cervical Vertebrae", "Constriction, Pathologic", "Female", "Follow-Up Studies", "Humans", "Laminectomy", "Longitudinal Ligaments", "Male", "Middle Aged", "Neurodegenerative Diseases", "Ossification of Posterior Longitudinal Ligament", "Radiography", "Retrospective Studies", "Spinal Cord Diseases", "Treatment Outcome" ], "YEAR": "2011", "reasoning_required_pred": "no", "reasoning_free_pred": "no", "final_decision": "no", "LONG_ANSWER": "Patients with CSM due to either degenerative disease or segmental OPLL have similar perioperative results and neurological outcomes with laminoplasty. The VAS neck pain scores did not improve significantly with laminoplasty for either group. Laminoplasty may limit extension ROM." }, "9003088": { "QUESTION": "Immunohistochemical assessment of steroid hormone receptors in tissues of the anal canal. Implications for anal incontinence?", "CONTEXTS": [ "A prerequisite for a hormonal influence on anal continence in women is the presence of hormone receptors in the tissues of the anal canal. Using immunohistochemical techniques, we demonstrated and localized estrogen and progesterone receptors (ER, PR) in tissue sections of the anal canal.", "Thirty-five specimens of the anorectal region from 21 patients (14 women, seven men) were examined histologically for smooth muscle (present in specimens from ten females and in seven males), striated muscle (present in three females and five males), and perimuscular connective tissue (present in 12 females and seven males). Immunostaining for ER and PR was then performed by incubating with primary anti-ER and anti-PR antibody and visualization of specific antibody binding by the ABC technique with DAB as chromogen.", "Positive staining for ER and PR was seen exclusively over cell nuclei. Estrogen receptors were found in the smooth muscle cells of the internal sphincter of all females (10/10) and in four of the seven males. Staining for ER was detected in the perimuscular connective tissue of all females (12/12) and in four of the seven males. No specific staining for ER or PR was found in the nuclei of striated muscle cells of the external sphincter in males or females (n = 8). In females, about 50% of the ER-positive tissues were also found to be positive for PR. Amongst the male patients, only one exhibited staining for PR, and this was confined to the smooth muscle." ], "LABELS": [ "BACKGROUND", "METHODS", "RESULTS" ], "MESHES": [ "Anal Canal", "Connective Tissue", "Fecal Incontinence", "Female", "Humans", "Immunohistochemistry", "Male", "Muscle, Skeletal", "Muscle, Smooth", "Receptors, Estrogen", "Receptors, Progesterone" ], "YEAR": "1996", "reasoning_required_pred": "yes", "reasoning_free_pred": "yes", "final_decision": "yes", "LONG_ANSWER": "Expression of ER is a constitutional feature of the connective tissue and smooth muscle cells of the anal continence organ. Estrogen receptors and PR are not detectable in the striated muscle fibers of the external anal sphincter in either sex. The presence of ER in the stroma and smooth muscles of the anal canal suggests that these tissues are targets for estrogen. This constitutes a theoretical basis for the beneficial effects of estrogen and progestin replacement on anal continence in postmenopausal women." }, "23337545": { "QUESTION": "Is acute fibrinous and organizing pneumonia the expression of immune dysregulation?", "CONTEXTS": [ "Acute fibrinous and organizing pneumonia (AFOP) is a recently described histologic pattern of diffuse pulmonary disease. In children, all cases reported to date have been fatal. In this study, we describe the first nonfatal AFOP in a child and review the literature.", "A 10-year-old boy developed very severe aplastic anemia (VSAA) after being admitted to our hospital with a fulminant hepatic failure of unknown origin. A chest computed tomography scan revealed multiple lung nodules and a biopsy of a pulmonary lesion showed all the signs of AFOP. Infectious workup remained negative. We started immunosuppressive therapy with antithymocyte globulin and cyclosporine to treat VSAA. Subsequent chest computed tomography scans showed a considerable diminution of the lung lesions but the VSAA did not improve until we performed hematopoietic stem cell transplantation 5 months later." ], "LABELS": [ "INTRODUCTION", "DESCRIPTION" ], "MESHES": [ "Acute Disease", "Child", "Cryptogenic Organizing Pneumonia", "Hematopoietic Stem Cell Transplantation", "Humans", "Immune System Diseases", "Immunosuppressive Agents", "Male" ], "YEAR": "2013", "reasoning_required_pred": "yes", "reasoning_free_pred": "yes", "final_decision": "yes", "LONG_ANSWER": "Aplastic anemia is associated with a variety of autoimmune syndromes. The sequence of events in our patient suggests that the hepatic failure, AFOP, and the VSAA may all have been part of an autoimmune syndrome. AFOP could be the result of immune dysregulation in this pediatric case with favorable outcome after immunosuppressive therapy and hematopoietic stem cell transplantation." }, "10490564": { "QUESTION": "Hypotension in patients with coronary disease: can profound hypotensive events cause myocardial ischaemic events?", "CONTEXTS": [ "To determine whether anginal episodes might be related to extremes of hypotension in patients with ischaemic heart disease taking drugs to treat angina and heart failure.", "Observational study of patients with ischaemic heart disease attending an urban tertiary referral cardiology centre.", "A selected patient population was enrolled, having: angina on one or more hypotensive cardiovascular medications; hypotension on clinic or ambulatory measurement; and a resting ECG suitable for ambulatory monitoring. Patients had echocardiography, ambulatory blood pressure monitoring, and Holter monitoring. Hypotension induced ischaemic (HII) events were defined as episodes of ST segment ischaemia occurring at least one minute after an ambulatory blood pressure measurement (systolic/diastolic) below 100/65 mm Hg during the day, or 90/50 mm Hg at night.", "25 suitable patients were enrolled, and 107 hypotensive events were documented. 40 ST events occurred in 14 patients, of which a quarter were symptomatic. Fourteen HII events occurred in eight patients, with 13 of the 14 preceded by a fall in diastolic pressure (median diastolic pressure 57.5 mm Hg, interquartile range 11, maximum 72 mm Hg, minimum 45 mm Hg), and six preceded by a fall in systolic pressure (chi(2) = 11.9, p<0.001). ST events were significantly associated with preceding hypotensive events (chi(2) = 40.2, p<0.0001). Patients with HII events were more frequently taking multiple hypotensive drug regimens (8/8 v 9/17, chi(2) = 5.54, p = 0.022)." ], "LABELS": [ "OBJECTIVE", "DESIGN AND SETTING", "INTERVENTIONS AND OUTCOME MEASURES", "RESULTS" ], "MESHES": [ "Adrenergic beta-Antagonists", "Aged", "Antihypertensive Agents", "Atenolol", "Blood Pressure Monitoring, Ambulatory", "Celiprolol", "Chi-Square Distribution", "Coronary Disease", "Diastole", "Drug Therapy, Combination", "Echocardiography", "Electrocardiography, Ambulatory", "Female", "Humans", "Hypotension", "Iatrogenic Disease", "Male", "Metoprolol", "Middle Aged", "Myocardial Ischemia", "Sotalol" ], "YEAR": "1999", "reasoning_required_pred": "yes", "reasoning_free_pred": "yes", "final_decision": "yes", "LONG_ANSWER": "In patients with ischaemic heart disease and hypotension, symptomatic and silent ischaemia occurred in a temporally causal relation with hypotension, particularly for diastolic pressures, suggesting that patients with coronary disease may be susceptible to ischaemic events incurred as a result of low blood pressure caused by excessive hypotensive drug treatment." }, "23375036": { "QUESTION": "An HIV1/2 point of care test on sputum for screening TB/HIV co-infection in Central India - Will it work?", "CONTEXTS": [ "To determine whether the OraQuick\u00ae HIV-1/2 Assay (OraSure Technologies, Inc., Bethlehem, PA, USA) in sputum is a valid tool for HIV surveillance among TB patients.", "A cross sectional study was carried out on sputa of patients diagnosed with tuberculosis. Sputa were tested for antibodies to HIV using OraQuick\u00ae HIV-1/2 Assay (OraSure Technologies, Inc., Bethlehem, PA, USA). The results were compared with results of serum ELISA.", "Compared to serum ELISA, the OraQuick\u00ae HIV-1/2 Assay in sputum specimens reported 90% sensitivity (9/10) and 100% specificity (307/307), with a positive predictive value of 100% (95%CI: 66.37%-100.00%) and a negative predictive value of 99.68% (95%CI: 98.20%-99.99%)." ], "LABELS": [ "OBJECTIVE", "METHODS", "RESULTS" ], "MESHES": [ "Adolescent", "Adult", "Aged", "Cross-Sectional Studies", "Enzyme-Linked Immunosorbent Assay", "Female", "HIV Infections", "HIV-1", "HIV-2", "Humans", "India", "Male", "Mass Screening", "Middle Aged", "Point-of-Care Systems", "Sensitivity and Specificity", "Sputum", "Tuberculosis, Pulmonary", "Young Adult" ], "YEAR": "2013", "reasoning_required_pred": "yes", "reasoning_free_pred": "yes", "final_decision": "yes", "LONG_ANSWER": "This testing method may provide a useful strategy for conducting HIV surveillance in possible co-infected TB patients at peripheral centres. Since there is no investment on infrastructure, it may be possible for paramedical health professionals to carry out the test, particularly in areas with low HIV endemicity." }, "11555508": { "QUESTION": "Do inhaled corticosteroids affect perception of dyspnea during bronchoconstriction in asthma?", "CONTEXTS": [ "Some of the disagreements on the perception of dyspnea (PD) during bronchoconstriction in asthma patients could depend on the interrelationships among the following: (1) the influence of baseline airflow obstruction on the patient's ability to detect any further increase in airway resistance; (2) the effect of eosinophilic inflammation on the airway; (3) bronchial hyperresponsiveness (BHR); and (4) the effect of inhaled corticosteroids (ICSs).", "We hypothesized that if the inflammation of the airway wall influences to some extent and in some way the PD in asthma patients, ICSs reverse the effect of airway inflammation on the PD.", "We studied 100 asthma patients who were divided into the following four groups: patients with obstruction who were either ICS-naive (group I) or were treated with ICSs (group II); and nonobstructed patients who were either ICS-naive (group III) or were treated with ICSs (group IV). PD on the visual analog scale (VAS) was assessed during a methacholine-induced FEV(1) decrease and specifically was quantified as the VAS slope and score at an FEV(1) decrease of 5 to 20%. BHR was assessed in terms of the provocative concentration of methacholine causing a 20% fall in FEV(1) (PC(20)). Eosinophil counts in induced sputum samples also were performed. Regression analysis, univariate analysis of variance, and factor analysis were applied for statistical evaluation.", "For a 5 to 20% fall in FEV(1) from the lowest point after saline solution induction, VAS score was lowest in group II, slightly higher in group I, slightly higher still in group IV, and the highest in group III. In the patients as a whole, BHR related to PD, but age, clinical score, duration of the disease, and presence of baseline airway obstruction did not. In patients with obstruction who were treated with ICSs, eosinophil counts related to PD negatively. Factor analysis yielded the following four factors that accounted for 70% of the variance in the data: ICS; eosinophil counts; FEV(1); and PC(20) loaded on separated factors with PD loading on the same factors as PC(20). The post hoc analysis carried out dividing the patients into ICS-treated and ICS-naive, showed that in the former group eosinophil counts and BHR proved to be factors negatively associated with PD, while in the latter group eosinophil counts were positively associated with PD." ], "LABELS": [ "BACKGROUND", "OBJECTIVE", "METHODS", "RESULTS" ], "MESHES": [ "Adolescent", "Adult", "Aged", "Anthropometry", "Asthma", "Bronchial Provocation Tests", "Bronchoconstriction", "Dyspnea", "Eosinophils", "Factor Analysis, Statistical", "Female", "Glucocorticoids", "Humans", "Leukocyte Count", "Linear Models", "Male", "Middle Aged", "Vital Capacity" ], "YEAR": "2001", "reasoning_required_pred": "yes", "reasoning_free_pred": "yes", "final_decision": "yes", "LONG_ANSWER": "We have shown that eosinophilic inflammation of the airway wall may increase PD and that the association of eosinophil counts with ICSs may result in lessening the PD." }, "16778275": { "QUESTION": "Is routine chest radiography after transbronchial biopsy necessary?", "CONTEXTS": [ "Pneumothorax following flexible bronchoscopy (FB) with transbronchial biopsy (TBB) occurs in 1 to 6% of cases. Routine chest radiography (CXR) following TBB is therefore requested by most pulmonologists in an attempt to detect complications, particularly pneumothorax. The objective of this study was to determine if routine CXR after bronchoscopy and TBB is necessary.", "The study group included 350 consecutive patients who underwent FB with TBB at our institution between December 2001 and January 2004. Routine CXR was performed up to 2 h after the procedure in all cases. Additionally, the following information was recorded in all patients: sex, age, immune status, indication for bronchoscopy, total number of biopsies done, segment sampled, pulse oxygen saturation, and development of symptoms suggestive of pneumothorax.", "Pneumothorax was diagnosed radiologically in 10 patients (2.9%). Seven patients had symptoms strongly suggestive of pneumothorax prior to CXR, including four patients with large (>10%) pneumothorax. The other three patients were asymptomatic, with only minimal pneumothorax (50\u00a0years during the same period. Patients aged 41-50\u00a0years were not included in the study, to accentuate potential age-related differences. Clinicopathological characteristics, treatment, and outcome were compared between groups.", "The cohort included 330 patients, followed for a median time of 65.9\u00a0months (range 4.7-211). Several significant differences were noted. The younger group had a different ethnic composition. They had higher rates of family history of colorectal cancer (p\u00a0=\u00a00.003), hereditary colorectal cancer syndromes (p\u00a0<\u00a00.0001), and inflammatory bowel disease (p\u00a0=\u00a00.007), and a lower rate of polyps (p\u00a0<\u00a00.0001). They were more likely to present with stage III or IV disease (p\u00a0=\u00a00.001), angiolymphatic invasion, signet cell ring adenocarcinoma, and rectal tumors (p\u00a0=\u00a00.02). Younger patients more frequently received treatment. Young patients had a worse estimated 5-year disease-free survival rate (57.6\u00a0 vs. 70\u00a0%, p\u00a0=\u00a00.039), but this did not retain significance when analyzed by stage (p\u00a0=\u00a00.092). Estimated 5-year overall survival rates were 59.1 and 62.1\u00a0% in the younger and the control group, respectively (p\u00a0=\u00a00.565)." ], "LABELS": [ "BACKGROUND", "METHODS", "RESULTS" ], "MESHES": [ "Adenomatous Polyposis Coli", "Adult", "Age Factors", "Aged", "Aged, 80 and over", "Carcinoma, Signet Ring Cell", "Colonic Polyps", "Colorectal Neoplasms", "Disease-Free Survival", "Female", "Humans", "Inflammatory Bowel Diseases", "Male", "Middle Aged", "Neoplasm Invasiveness", "Neoplasm Staging", "Rectal Neoplasms", "Retrospective Studies", "Survival Rate", "Young Adult" ], "YEAR": "2016", "reasoning_required_pred": "no", "reasoning_free_pred": "yes", "final_decision": "yes", "LONG_ANSWER": "Colorectal cancer among young patients may constitute a distinct clinical entity. Further research is needed to validate our findings and define the optimal approach in this population." }, "21080127": { "QUESTION": "Can implant retention be recommended for treatment of infected TKA?", "CONTEXTS": [ "Retention treatment is reportedly associated with lower infection control rates than two-stage revision. However, the studies on which this presumption are based depend on comparisons of historical rather than concurrent controls.QUESTIONS/", "We (1) asked whether the infection control rates, number of additional procedures, length of hospital stay, and treatment duration differed between implant retention and two-stage revision treatment; and (2) identified risk factors that can contribute to failure of infection control.", "We reviewed the records of 60 patients treated for 64 infected TKA from 2002 to 2007. Twenty-eight patients (32 knees) underwent d\u00e9bridement with retention of component, and 32 patients (32 knees) were treated with component removal and two-stage revision surgery. We determined patients' demographics, type of infection, causative organisms, and outcome of treatment. Mean followup was 36 months (range, 12-84 months).", "Infection control rate was 31% in retention and 59% in the removal group after initial surgical treatment, and 81% and 91% at latest followup, respectively. Treatment duration was shorter in the retention group and there was no difference in number of additional surgeries and length of hospital stay. Type of treatment (retention versus removal) was the only factor associated with infection control; subgroup analysis in the retention group showed Staphylococcus aureus infection and polyethylene nonexchange as contributing factors for failure of infection control." ], "LABELS": [ "BACKGROUND", "PURPOSES", "METHODS", "RESULTS" ], "MESHES": [ "Adult", "Aged", "Aged, 80 and over", "Arthroplasty, Replacement, Knee", "Boston", "Chi-Square Distribution", "Debridement", "Device Removal", "Female", "Humans", "Infection Control", "Kaplan-Meier Estimate", "Knee Prosthesis", "Length of Stay", "Logistic Models", "Male", "Middle Aged", "Odds Ratio", "Patient Selection", "Polyethylene", "Prosthesis Design", "Prosthesis-Related Infections", "Reoperation", "Retrospective Studies", "Risk Assessment", "Risk Factors", "Time Factors", "Treatment Outcome" ], "YEAR": "2011", "reasoning_required_pred": "yes", "reasoning_free_pred": "maybe", "final_decision": "maybe", "LONG_ANSWER": "Although initial infection control rate was substantially lower in the retention group than the removal group, final results were comparable at latest followup. We believe retention treatment can be selectively considered for non-S. aureus infection, and when applied in selected patients, polyethylene exchange should be performed." }, "18086459": { "QUESTION": "Long-term significance of postictal psychotic episodes II. Are they predictive of interictal psychotic episodes?", "CONTEXTS": [ "The aim of this study was to determine whether postictal psychotic episodes (PIPE) are predictive of the development of interictal psychotic episodes (IPE).", "This was a retrospective study of 18 consecutive adults with a partial seizure disorder and PIPE (study group) and 36 patients with a partial seizure disorder but without PIPE (control group). These two groups were compared with respect to the likelihood of developing IPE over an 8-year follow-up period and the variables operant in the development of IPE. Statistical analyses consisted of logistic regression models to identify the variables predictive of the development of IPE. Predictors included: number and location of ictal foci, seizure type, etiology, age at seizure onset, duration of seizure disorder, MRI abnormalities, and psychiatric history prior to the index video/EEG monitoring (other than PIPE).", "Seven patients with PIPE and one control patient went on to develop an IPE. Predictors of IPE in univariate logistic regression analyses included a history of PIPE (P=0.006), male gender (P=0.028), and having bilateral ictal foci (P=0.048). Significance disappeared for all of these variables when they were entered into a multivariate analysis." ], "LABELS": [ "OBJECTIVE", "METHODS", "RESULTS" ], "MESHES": [ "Adult", "Disease Progression", "Electroencephalography", "Epilepsy", "Female", "Humans", "Longitudinal Studies", "Magnetic Resonance Imaging", "Male", "Predictive Value of Tests", "Psychiatric Status Rating Scales", "Psychotic Disorders", "Video Recording" ], "YEAR": "2008", "reasoning_required_pred": "no", "reasoning_free_pred": "yes", "final_decision": "yes", "LONG_ANSWER": "A history of PIPE may be a risk factor for the development of IPE. Yet, the disappearance of significance on multivariate analysis indicates that it is not an independent predictor." }, "11978239": { "QUESTION": "Do primary care physicians underprescribe antibiotics for peptic ulcer disease?", "CONTEXTS": [ "To determine how often primary care physicians prescribe eradication therapy for peptic ulcer disease (PUD) and nonulcer dyspepsia (NUD).", "During a 2-year period (1998-2000) we analyzed data concerning patients with PUD or NUD seen by 80 Italian primary care physicians uniformly distributed throughout the country. We classified patients as having a definitive or a presumptive diagnosis on the basis of the completeness of the diagnostic workup and interpreted the prescription of antibiotics for dyspepsia as evidence of attempted eradication of Helicobacter pylori.", "Consecutive ambulatory patients.", "The frequency with which predefined groups of patients received eradication therapy.", "Of 6866 patients, 690 (10%) received eradication therapy. Of 2162 patients with PUD, 596 (27.6%) received eradication therapy; of 4704 patients with NUD, however, only 94 (2%) received this treatment (P =.0001). A total of 341 (37.7%) of 904 PUD patients with a definitive diagnosis were given eradication therapy and 255 (20.3%) of 1258 PUD patients with a presumptive diagnosis were given therapy (P<.0001). In NUD patients, 7 of 743 (0.9%) with a definitive diagnosis received eradication therapy, while 87 (2.2%) of 3961 of those with a presumptive diagnosis were given the same therapy (P =.025)." ], "LABELS": [ "OBJECTIVE", "STUDY DESIGN", "POPULATION", "OUTCOMES MEASURED", "RESULTS" ], "MESHES": [ "Adult", "Anti-Bacterial Agents", "Drug Utilization", "Dyspepsia", "Family Practice", "Female", "Health Services Misuse", "Helicobacter Infections", "Helicobacter pylori", "Humans", "Italy", "Male", "Middle Aged", "Peptic Ulcer", "Practice Patterns, Physicians'", "Retrospective Studies" ], "YEAR": "2002", "reasoning_required_pred": "no", "reasoning_free_pred": "yes", "final_decision": "yes", "LONG_ANSWER": "While Italian primary care physicians appropriately target eradication therapy for H pylori infection in patients with peptic ulcer disease rather than nonulcer disease, the intervention was still underused in these patients. Improvements in this prescribing behavior are needed." }, "10798511": { "QUESTION": "Blunt trauma in intoxicated patients: is computed tomography of the abdomen always necessary?", "CONTEXTS": [ "Physical examination to detect abdominal injuries has been considered unreliable in alcohol-intoxicated trauma patients. Computed tomography (CT) plays the primary role in these abdominal evaluations.", "We reviewed medical records of all blunt trauma patients admitted to our trauma service from January 1, 1992, to March 31, 1998. Study patients had a blood alcohol level>or =80 mg/dL, Glasgow Coma Scale (GCS) score of 15, and unremarkable abdominal examination.", "Of 324 patients studied, 317 (98%) had CT scans negative for abdominal injury. Abdominal injuries were identified in 7 patients (2%), with only 2 (0.6%) requiring abdominal exploration. A significant association was found between major chest injury and abdominal injury." ], "LABELS": [ "BACKGROUND", "METHODS", "RESULTS" ], "MESHES": [ "Abdominal Injuries", "Adolescent", "Adult", "Aged", "Aged, 80 and over", "Alcoholic Intoxication", "Female", "Humans", "Male", "Middle Aged", "Physical Examination", "Radiography", "Risk Factors", "Trauma Centers" ], "YEAR": "2000", "reasoning_required_pred": "no", "reasoning_free_pred": "no", "final_decision": "no", "LONG_ANSWER": "The incidence of abdominal injury in intoxicated, hemodynamically stable, blunt trauma patients with a normal abdominal examination and normal mentation is low. Physical examination and attention to clinical risk factors allow accurate abdominal evaluation without CT." }, "15477551": { "QUESTION": "Chronic progressive cervical myelopathy with HTLV-I infection: Variant form of HAM/TSP?", "CONTEXTS": [ "To investigate the role of human T-lymphotrophic virus type I (HTLV-I) infection in four patients who developed slowly progressive myelopathy with abnormal MRI lesions in the cervical cord levels.", "Clinical and neuroradiologic examinations were performed, and the odds that an HTLV-I-infected individual of specified genotype, age, and provirus load had HTLV-I-associated myelopathy (HAM)/tropical spastic paraparesis (TSP) were calculated.", "Anti-HTLV-I antibodies were positive in both the serum and the CSF in all of the patients. Biopsied sample from spinal cord lesions showed inflammatory changes in Patient 1. Patient 2 had a demyelinating type of sensorimotor polyneuropathy. Two of the three patients examined showed high risk of developing HAM/TSP in virologic and immunologic aspects." ], "LABELS": [ "OBJECTIVE", "METHODS", "RESULTS" ], "MESHES": [ "Aged", "Case-Control Studies", "Chronic Disease", "Contrast Media", "Disease Progression", "Female", "Gadolinium DTPA", "Genotype", "Humans", "Immunohistochemistry", "Japan", "Magnetic Resonance Imaging", "Male", "Middle Aged", "Neck", "Paraparesis, Tropical Spastic", "Polyneuropathies", "Probability", "Spinal Cord" ], "YEAR": "2004", "reasoning_required_pred": "yes", "reasoning_free_pred": "yes", "final_decision": "yes", "LONG_ANSWER": "These four cases may belong to a variant form of HAM/TSP, predominantly involving the cervical cord levels." }, "19327500": { "QUESTION": "Gender difference in survival of resected non-small cell lung cancer: histology-related phenomenon?", "CONTEXTS": [ "It remains controversial whether there is a gender difference in survival of patients with resected non-small cell lung cancer.", "We retrospectively analyzed 2770 patients (1689 men and 1081 women) with non-small cell lung cancer who underwent pulmonary resection between 1995 and 2005 at the National Cancer Center Hospital, Tokyo. A gender difference in survival was studied in all patients, in those divided according to histology or pathologic stage, and in propensity-matched gender pairs.", "There were no differences in background, such as preoperative pulmonary function, operation procedures, or operative mortality. The proportions of adenocarcinoma and pathologic stage I in women were greater than those in men (93.6% vs 61.7% and 71.4% vs 58.6%, respectively) (P<.001). Overall 5-year survival of women was better than that of men (81% vs 70%, P<.001). In adenocarcinoma, the overall 5-year survival for women was better than that for men in pathologic stage I (95% vs 87%, P<.001) and in pathologic stage II or higher (58% vs 51%, P = .017). In non-adenocarcinoma, there was no significant gender difference in survival in pathologic stage I (P = .313) or pathologic stage II or higher (P = .770). The variables such as age, smoking status, histology, and pathologic stage were used for propensity score matching, and survival analysis of propensity score-matched gender pairs did not show a significant difference (P = .69)." ], "LABELS": [ "OBJECTIVE", "METHODS", "RESULTS" ], "MESHES": [ "Carcinoma, Non-Small-Cell Lung", "Female", "Humans", "Lung Neoplasms", "Male", "Middle Aged", "Pneumonectomy", "Retrospective Studies", "Sex Factors", "Survival Analysis" ], "YEAR": "2009", "reasoning_required_pred": "no", "reasoning_free_pred": "yes", "final_decision": "yes", "LONG_ANSWER": "Women had better survival than men; however, there was no survival advantage in propensity-matched gender pairs. A gender difference in survival was observed only in the adenocarcinoma subset, suggesting pathobiology in adenocarcinoma in women might be different from that of men." }, "24614789": { "QUESTION": "Is lumbar drainage of postoperative cerebrospinal fluid fistula after spine surgery effective?", "CONTEXTS": [ "Postoperative CSF fistulas were described in 16 of 198 patients (8%) who underwent spine surgery between 2009 and 2010. The choice of the therapeutic strategy was based on the clinical condition of the patients, taking into account the possibility to maintain the prone position continuously and the risk of morbidity due to prolonged bed rest. Six patients were treated conservatively (position prone for three weeks), ten patients were treated by positioning an external CSF lumbar drainage for ten days. The mean follow-up period was ten months.", "All patients healed their wound properly and no adverse events were recorded. Patients treated conservatively were cured in a mean period of 30 days, while patients treated with CSF drainage were cured in a mean period of 10 days." ], "LABELS": [ "METHODS", "RESULTS" ], "MESHES": [ "Cerebrospinal Fluid Rhinorrhea", "Drainage", "Dura Mater", "Female", "Fistula", "Humans", "Male", "Postoperative Complications", "Retrospective Studies", "Treatment Outcome" ], "YEAR": "2014", "reasoning_required_pred": "yes", "reasoning_free_pred": "yes", "final_decision": "yes", "LONG_ANSWER": "Lumbar drainage seems to be effective and safe both in preventing CSF fistula in cases of large dural tears and debilitated/irradiated patients and in treating CSF leaks." }, "23283159": { "QUESTION": "Is obesity a risk factor for wheezing among adolescents?", "CONTEXTS": [ "To investigate the effect of obesity at the start of adolescence on the prevalence, incidence and maintenance of chest wheezing among individuals aged 11-15 years in a birth cohort in a developing country.", "The seventh follow-up of the 1993 Pelotas birth cohort occurred in 2004 (individuals aged 10-11 years). Between January and August 2008, the eighth follow-up of the cohort was conducted. All the individuals of the original cohort who were alive (who were then adolescents aged between 14 and 15 years) were targets for the study. The International Study of Asthma and Allergies in Childhood (ISAAC) questionnaire was used to define wheezing. In addition to the body mass index (BMI), used to define obesity by the World Health Organization (WHO) criteria, we assessed skinfold thickness.", "From the original cohort, 4,349 individuals were located (85.7% follow-up rate). The prevalence of chest wheezing at 11 and 15 years were 13.5% (95% CI: 12.5%-14.5%) and 12.1% (95% CI: 11.1%-13.1%), respectively. The prevalence of wheezing at both times was 4.5% (95% CI: 3.9%-5.1%) and the incidence of wheezing was 7.5% (95% CI: 6.7%-8.3%). Independent of the effect of various confounding variables, the prevalence of wheezing at 15 years was 50% greater among obese individuals than among eutrophic individuals at 11 years (RR 1.53; 95% CI: 1.14-2.05). The greater the skinfold tertile at 11 years, the higher the prevalence of wheezing at 15 years was (p = .011). Weight status and skinfolds did not present any association with incident wheezing. After controlling for confounding factors, the risk of persistent wheezing among obese individuals at 11 years was 1.82 (95% CI: 1.30-2.54)." ], "LABELS": [ "PURPOSE", "METHODS", "RESULTS" ], "MESHES": [ "Adolescent", "Asthma", "Brazil", "Child", "Cohort Studies", "Confounding Factors (Epidemiology)", "Female", "Humans", "Incidence", "Male", "Obesity", "Prevalence", "Respiratory Sounds", "Risk Factors" ], "YEAR": "2012", "reasoning_required_pred": "no", "reasoning_free_pred": "yes", "final_decision": "yes", "LONG_ANSWER": "Since obesity at the start of adolescence is associated with asthma symptom persistence, prevention and treatment of obesity may reduce avoidable healthcare costs and disease burden." }, "16465002": { "QUESTION": "Dose end-tidal carbon dioxide measurement correlate with arterial carbon dioxide in extremely low birth weight infants in the first week of life?", "CONTEXTS": [ "To study the correlation and agreement between end-tidal carbon dioxide (EtCO2) and arterial carbon dioxide (PaCO(2)) in ventilated extremely low birth weight (ELBW) infants in the first week of life.", "Retrospective chart review of all ELBW (<1,000 g) infants admitted to a level III NICU from January 2003 to December 2003. Data collected included demographic details and simultaneous EtCO(2) (mainstream capnography) and arterial blood gas values (pH, PaCO(2), PaO(2)).", "The correlation coefficient, degree of bias with 95% confidence interval between the EtCO(2) and PaCO(2).", "There were 754 end-tidal and arterial CO(2) pairs from 31 ELBW infants (21 male and 10 female). The overall EtCO(2) values were significantly lower than PaCO(2) value. In only 89/754(11.8%) pairs, the EtCO(2) was higher than the PaCO(2). The overall bias was 5.6 +/- 6.9 mmHg (95% C.I. 5.11-6.09). The intraclass correlation coefficient was 0.81. Using EtCO2 ranges of 30 to 50 mmHg, the capnographic method was able to identify 84% of instances where PaCO(2) was between 35 (<35 = hypocarbia) and 55 mmHg (>55= hypercapnia)." ], "LABELS": [ "OBJECTIVE", "METHODS", "OUTCOME", "RESULTS" ], "MESHES": [ "Blood Gas Analysis", "Capnography", "Carbon Dioxide", "Female", "Follow-Up Studies", "Humans", "Infant, Newborn", "Infant, Very Low Birth Weight", "Male", "Respiration, Artificial", "Respiratory Distress Syndrome, Newborn", "Retrospective Studies", "Risk Assessment", "Sensitivity and Specificity", "Tidal Volume" ], "YEAR": "2006", "reasoning_required_pred": "yes", "reasoning_free_pred": "yes", "final_decision": "yes", "LONG_ANSWER": "There is good correlation and agreement between end-tidal CO(2) and arterial CO(2) in ELBW infants in the EtCO(2) range 30-50 mmHg. End-tidal CO(2) monitoring can be helpful in trending or for screening abnormal PaCO(2) values in ELBW infants in first week of life." }, "24191126": { "QUESTION": "Is CA72-4 a useful biomarker in differential diagnosis between ovarian endometrioma and epithelial ovarian cancer?", "CONTEXTS": [ "Surgical excision of ovarian endometriomas in patients desiring pregnancy has recently been criticized because of the risk of damage to healthy ovarian tissue and consequent reduction of ovarian reserve. A correct diagnosis in cases not scheduled for surgery is therefore mandatory in order to avoid unexpected ovarian cancer misdiagnosis. Endometriosis is often associated with high levels of CA125. This marker is therefore not useful for discriminating ovarian endometrioma from ovarian malignancy. The aim of this study was to establish if the serum marker CA72-4 could be helpful in the differential diagnosis between ovarian endometriosis and epithelial ovarian cancer.", "Serums CA125 and CA72-4 were measured in 72 patients with ovarian endometriomas and 55 patients with ovarian cancer.", "High CA125 concentrations were observed in patients with ovarian endometriosis and in those with ovarian cancer. A marked difference in CA72-4 values was observed between women with ovarian cancer (71.0%) and patients with endometriosis (13.8%) (P<0.0001)." ], "LABELS": [ "BACKGROUND", "METHODS", "RESULTS" ], "MESHES": [ "Adult", "Aged", "Antigens, Tumor-Associated, Carbohydrate", "CA-125 Antigen", "Carcinoma", "Diagnosis, Differential", "Endometriosis", "Female", "Humans", "Membrane Proteins", "Middle Aged", "Ovarian Neoplasms" ], "YEAR": "2013", "reasoning_required_pred": "yes", "reasoning_free_pred": "yes", "final_decision": "yes", "LONG_ANSWER": "This study suggests that CA72-4 determination can be useful to confirm the benign nature of ovarian endometriomas in women with high CA125 levels." }, "25218577": { "QUESTION": "Preservation of the PCL when performing cruciate-retaining TKA: Is the tibial tuberosity a reliable predictor of the PCL footprint location?", "CONTEXTS": [ "Reconstruction of the joint line is crucial in total knee arthroplasty (TKA). A routine height of tibial cut to maintain the natural joint line may compromise the preservation of the PCL. Since the PCL footprint is not accessible prior to tibial osteotomy, it seems beneficial to identify a reliable extraarticular anatomic landmark for predicting the PCL footprint and being visible within standard TKA approach. The fibula head predicts reliably the location of PCL footprint; however, it is not accessible during TKA. The aim of this study now was to analyze whether the tibial tuberosity can serve as a reliable referencing landmark to estimate the PCL footprint height prior to tibial cut.", "The first consecutive case series included 216 CR TKA. Standing postoperative lateral view radiographs were utilized to measure the vertical distance between tibial tuberosity and tibial osteotomy plane. In the second case series, 223 knee MRIs were consecutively analyzed to measure the vertical distance between tibial tuberosity and PCL footprint. The probability of partial or total PCL removal was calculated for different vertical distances between tibial tuberosity and tibial cutting surface.", "The vertical distance between the tibial tuberosity and tibial cut averaged 24.7 \u00b1 4 mm. The average vertical distance from tibial tuberosity to proximal and to distal PCL footprint was found to be 22 \u00b1 4.4 and 16 \u00b1 4.4 mm, respectively. Five knees were considered at 50% risk of an entire PCL removal after CR TKA." ], "LABELS": [ "PURPOSE", "METHODS", "RESULTS" ], "MESHES": [ "Arthroplasty, Replacement, Knee", "Female", "Humans", "Male", "Middle Aged", "Osteotomy", "Posterior Cruciate Ligament", "Radiography", "Tibia" ], "YEAR": "2016", "reasoning_required_pred": "yes", "reasoning_free_pred": "yes", "final_decision": "yes", "LONG_ANSWER": "Current surgical techniques of tibial preparation may result in partial or total PCL damage. Tibial tuberosity is a useful anatomical landmark to locate the PCL footprint and to predict the probability of its detachment pre-, intra-, and postoperatively. This knowledge might be useful to predict and avoid instability, consecutive pain, and dissatisfaction after TKA related to PCL insufficiency." }, "20488985": { "QUESTION": "The English antibiotic awareness campaigns: did they change the public's knowledge of and attitudes to antibiotic use?", "CONTEXTS": [ "To determine the effect of the 2008 English public antibiotic campaigns.", "English and Scottish (acting as controls) adults aged>or = 15 years were questioned face to face about their attitudes to and use of antibiotics, in January 2008 (1888) before and in January 2009 (1830) after the antibiotic campaigns.", "Among English respondents, there was a small increase in recollection of campaign posters (2009 23.7% versus 2008 19.2%; P = 0.03), but this increase was only 2.3% higher in England than in Scotland. We did not detect any improvement in either England or Scotland, or any differences between England and Scotland in the understanding of the lack of benefit of antibiotics for coughs and colds, and we found no improvement in antibiotic use. We detected a significant increase in respondents retaining leftover antibiotics. Over 20% reported discussing antibiotics with their general practitioner (GP) or nurse in the year to January 2009. The offer of a delayed antibiotic prescription was reported significantly more often by English respondents (19% versus 8% Scottish in 2009; P = 0.01), and English respondents were advised to use other remedies for coughs and colds significantly more often in the year to January 2009 (12.7% in 2009 versus 7.4% in 2008; P<0.001)." ], "LABELS": [ "OBJECTIVES", "METHODS", "RESULTS" ], "MESHES": [ "Adolescent", "Adult", "Aged", "Aged, 80 and over", "Anti-Bacterial Agents", "Bacterial Infections", "Data Collection", "England", "Female", "Health Knowledge, Attitudes, Practice", "Health Services Research", "Humans", "Male", "Middle Aged", "Scotland", "Young Adult" ], "YEAR": "2010", "reasoning_required_pred": "yes", "reasoning_free_pred": "no", "final_decision": "no", "LONG_ANSWER": "There is little evidence that the 2008 public antibiotic campaigns were effective. The use and visibility of future campaign materials needs auditing. A carefully planned approach that targets the public in GP waiting rooms and through clinicians in consultations may be a more effective way of improving prudent antibiotic use." }, "11567820": { "QUESTION": "Does increased nerve length within the treatment volume improve trigeminal neuralgia radiosurgery?", "CONTEXTS": [ "To test the hypothesis that increasing the nerve length within the treatment volume for trigeminal neuralgia radiosurgery would improve pain relief.", "Eighty-seven patients with typical trigeminal neuralgia were randomized to undergo retrogasserian gamma knife radiosurgery (75 Gy maximal dose with 4-mm diameter collimators) using either one (n = 44) or two (n = 43) isocenters. The median follow-up was 26 months (range 1-36).", "Pain relief was complete in 57 patients (45 without medication and 12 with low-dose medication), partial in 15, and minimal in another 15 patients. The actuarial rate of obtaining complete pain relief (with or without medication) was 67.7% +/- 5.1%. The pain relief was identical for one- and two-isocenter radiosurgery. Pain relapsed in 30 of 72 responding patients. Facial numbness and mild and severe paresthesias developed in 8, 5, and 1 two-isocenter patients vs. 3, 4, and 0 one-isocenter patients, respectively (p = 0.23). Improved pain relief correlated with younger age (p = 0.025) and fewer prior procedures (p = 0.039) and complications (numbness or paresthesias) correlated with the nerve length irradiated (p = 0.018)." ], "LABELS": [ "PURPOSE", "METHODS AND MATERIALS", "RESULTS" ], "MESHES": [ "Adult", "Aged", "Aged, 80 and over", "Analysis of Variance", "Double-Blind Method", "Female", "Follow-Up Studies", "Humans", "Magnetic Resonance Imaging", "Male", "Middle Aged", "Prospective Studies", "Radiosurgery", "Trigeminal Nerve", "Trigeminal Neuralgia" ], "YEAR": "2001", "reasoning_required_pred": "yes", "reasoning_free_pred": "no", "final_decision": "no", "LONG_ANSWER": "Increasing the treatment volume to include a longer nerve length for trigeminal neuralgia radiosurgery does not significantly improve pain relief but may increase complications." }, "19100463": { "QUESTION": "Is the affinity column-mediated immunoassay method suitable as an alternative to the microparticle enzyme immunoassay method as a blood tacrolimus assay?", "CONTEXTS": [ "Tacrolimus is a potent immunosuppressive drug used in organ transplantation. Because of its substantial toxic effects, narrow therapeutic index, and interindividual pharmacokinetic variability, therapeutic drug monitoring of whole-blood tacrolimus concentrations has been recommended. We investigated the comparability of the results of 2 immunoassay systems, affinity column-mediated immunoassay (ACMIA) and microparticle enzyme immunoassay (MEIA), comparing differences in the tacrolimus concentrations measured by the 2 methods in relation to the hematologic and biochemical values of hepatic and renal functions.", "A total of 154 samples from kidney or liver transplant recipients were subjected to Dimension RxL HM with a tacrolimus Flex reagent cartilage for the ACMIA method and IMx tacrolimus II for the MEIA method.", "Tacrolimus concentrations measured by the ACMIA method (n = 154) closely correlated with those measured by the MEIA method (r = 0.84). The Bland-Altman plot using concentration differences between the 2 methods and the average of the 2 methods showed no specific trends. The tacrolimus levels determined by both the MEIA method and the ACMIA method were not influenced by hematocrit levels, but the difference between the 2 methods (ACMIA - MEIA) tended to be larger in low hematocrit samples (P<.001)." ], "LABELS": [ "BACKGROUND", "METHODS", "RESULTS" ], "MESHES": [ "Bilirubin", "Chromatography, Affinity", "Creatinine", "Hematocrit", "Humans", "Immunoassay", "Immunoenzyme Techniques", "Immunosuppressive Agents", "Kidney Transplantation", "Reproducibility of Results", "Sensitivity and Specificity", "Serum Albumin", "Tacrolimus" ], "YEAR": "2008", "reasoning_required_pred": "maybe", "reasoning_free_pred": "yes", "final_decision": "yes", "LONG_ANSWER": "The ACMIA method used for a tacrolimus assay is precise and has advantages, including the lack of a required pretreatment procedure. Furthermore, it is only slightly influenced by the hematologic or biochemical status of the samples." }, "12769830": { "QUESTION": "Should tumor depth be included in prognostication of soft tissue sarcoma?", "CONTEXTS": [ "Most staging systems for soft tissue sarcoma are based on histologic malignancy-grade, tumor size and tumor depth. These factors are generally dichotomized, size at 5 cm. We believe it is unlikely that tumor depth per se should influence a tumor's metastatic capability. Therefore we hypothesized that the unfavourable prognostic importance of depth could be explained by the close association between size and depth, deep-seated tumors on average being larger than the superficial ones. When tumor size is dichotomized, this effect should be most pronounced in the large size (>5 cm) group in which the size span is larger.", "We analyzed the associations between tumor size and depth and the prognostic importance of grade, size and depth in a population-based series of 490 adult patients with soft tissue sarcoma of the extremity or trunk wall with complete, 4.5 years minimum, follow-up.", "Multivariate analysis showed no major prognostic effect of tumor depth when grade and size were taken into account. The mean size of small tumors was the same whether superficial or deep but the mean size of large and deep-seated tumors were one third larger than that of large but superficial tumors. Tumor depth influenced the prognosis in the subset of high-grade and large tumors. In this subset deep-seated tumors had poorer survival rate than superficial tumors, which could be explained by the larger mean size of the deep-seated tumors." ], "LABELS": [ "BACKGROUND", "METHODS", "RESULTS" ], "MESHES": [ "Adolescent", "Adult", "Aged", "Aged, 80 and over", "Female", "Humans", "Male", "Middle Aged", "Multivariate Analysis", "Neoplasm Staging", "Prognosis", "Sarcoma" ], "YEAR": "2003", "reasoning_required_pred": "no", "reasoning_free_pred": "no", "final_decision": "no", "LONG_ANSWER": "Most of the prognostic value of tumor depth in soft tissue sarcomas of the extremity or trunk wall can be explained by the association between tumor size and depth." }, "10331115": { "QUESTION": "Is amoxapine an atypical antipsychotic?", "CONTEXTS": [ "All currently available atypical antipsychotics have, at clinically relevant doses: i) high serotonin (5-HT)2 occupancy; ii) greater 5-HT2 than dopamine (D)2 occupancy; and iii) a higher incidence of extrapyramidal side effects when their D2 occupancy exceeds 80%. A review of pharmacologic and behavioral data suggested that amoxapine should also conform to this profile; therefore, we undertook a positron-emission tomography (PET) study of its 5-HT2 and D2 occupancy.", "Seven healthy volunteers received 50-250 mg/day of amoxapine for 5 days and then had [11C]-raclopride and [18F]-setoperone PET scans.", "5-HT2 receptors showed near saturation at doses of 100 mg/day and above. The D2 receptor occupancies showed a dose-dependent increase, never exceeding 80%; at all doses 5-HT2 occupancy exceeded D2 occupancy." ], "LABELS": [ "BACKGROUND", "METHODS", "RESULTS" ], "MESHES": [ "Adult", "Amoxapine", "Brain", "Female", "Humans", "Male", "Psychotic Disorders", "Receptors, Dopamine D2", "Receptors, Serotonin", "Tomography, Emission-Computed" ], "YEAR": "1999", "reasoning_required_pred": "yes", "reasoning_free_pred": "yes", "final_decision": "yes", "LONG_ANSWER": "PET data show that amoxapine's profile is very similar to that of the established atypical antipsychotics. These data, together with amoxapine's in vitro pharmacologic profile, effectiveness in animal models, and efficacy in psychotic depression raise the possibility of amoxapine as an \"atypical\" antipsychotic agent in the treatment of schizophrenia." }, "17483607": { "QUESTION": "Does topical N-acetylcysteine application after myringotomy cause severe otorrhea?", "CONTEXTS": [ "The effect of topical N-acetylcysteine (NAC) application was investigated on the healing of acute experimental tympanic membrane perforations.", "Twenty guinea pigs were used in this study. Under intraperitoneal ketamine anesthesia, incisional myringotomies were performed in the posterosuperior quadrant of the tympanic membranes with a straight otologic hook. The diameter of the perforations was approximately 2 mm. Perforations in both ears were treated with freshly prepared sponges soaked in either 0.1 ml 0.9% NaCl solution (10 control animals) or 0.6 mg/0.1 ml NAC (10 animals) for three consecutive days. All the tympanic membranes were examined by otomicroscopy on the third, fifth, seventh, and ninth days.", "In the control group, all the perforations were completely closed at the end of nine days. During the same period, only 40% of the perforations were completely closed in the NAC group. The remaining ears exhibited otorrhea by the third day." ], "LABELS": [ "OBJECTIVES", "MATERIALS AND METHODS", "RESULTS" ], "MESHES": [ "Acetylcysteine", "Administration, Topical", "Animals", "Anti-Inflammatory Agents, Non-Steroidal", "Guinea Pigs", "Otitis Media, Suppurative", "Severity of Illness Index", "Tympanic Membrane Perforation" ], "YEAR": "2007", "reasoning_required_pred": "yes", "reasoning_free_pred": "yes", "final_decision": "yes", "LONG_ANSWER": "N-acetylcysteine may cause severe otorrhea in the healing process of tympanic membrane perforations. Further studies including histopathological examinations are required to elucidate this condition." }, "28056802": { "QUESTION": "Is non-HDL-cholesterol a better predictor of long-term outcome in patients after acute myocardial infarction compared to LDL-cholesterol?", "CONTEXTS": [ "It has recently been shown that non-high density lipoprotein cholesterol (non-HDL-C) may be a better predictor of cardiovascular risk than low density lipoprotein cholesterol (LDL-C). Based on known ethic differences in lipid parameters and cardiovascular risk prediction, we sought to study the predictability of attaining non-HDL-C target and long-term major adverse cardiovascular event (MACE) in Thai patients after acute myocardial infarction (AMI) compared to attaining LDL-C target.", "We retrospectively obtained the data of all patients who were admitted at Maharaj Nakorn Chiang Mai hospital due to AMI during 2006-2013. The mean non-HDL-C and LDL-C during long-term follow-up were used to predict MACE at each time point. The patients were classified as target attainment if non-HDL-C\u2009<100\u00a0mg/dl and/or LDL-C\u2009<70\u00a0mg/dl. The MACE was defined as combination of all-cause death, nonfatal coronary event and nonfatal stroke.", "During mean follow-up of 2.6\u2009\u00b1\u20091.6\u00a0years among 868 patients after AMI, 34.4% achieved non-HDL-C target, 23.7% achieved LDL-C target and 21.2% experienced MACEs. LDL-C and non-HDL-C were directly compared in Cox regression model. Compared with non-HDL-C\u2009<100\u00a0mg/dl, patients with non-HDL-C of>130\u00a0mg/dl had higher incidence of MACEs (HR 3.15, 95% CI 1.46-6.80, P\u2009=\u20090.003). Surprisingly, LDL-C\u2009>100\u00a0mg/dl was associated with reduced risk of MACE as compared to LDL\u2009<70\u00a0mg/dl (HR 0.42, 95% CI 0.18-0.98, p\u2009=\u20090.046) after direct pairwise comparison with non-HDL-C level." ], "LABELS": [ "BACKGROUND", "METHODS", "RESULTS" ], "MESHES": [ "Aged", "Biomarkers", "Cardiovascular Agents", "Chi-Square Distribution", "Cholesterol", "Cholesterol, LDL", "Dyslipidemias", "Female", "Humans", "Hypolipidemic Agents", "Incidence", "Male", "Middle Aged", "Multivariate Analysis", "Non-ST Elevated Myocardial Infarction", "Percutaneous Coronary Intervention", "Proportional Hazards Models", "Retrospective Studies", "Risk Factors", "ST Elevation Myocardial Infarction", "Secondary Prevention", "Thailand", "Thrombolytic Therapy", "Time Factors", "Treatment Outcome" ], "YEAR": "2017", "reasoning_required_pred": "yes", "reasoning_free_pred": "yes", "final_decision": "yes", "LONG_ANSWER": "Non-attaining non-HDL-C goal predicted MACE at long-term follow-up after AMI whereas non-attaining LDL-C goal was not associated with the higher risk. Therefore, non-HDL-C may be a more suitable target of dyslipidemia treatment than LDL-C in patients after AMI." }, "11776681": { "QUESTION": "Is intrapartum vibroacoustic stimulation an effective predictor of fetal acidosis?", "CONTEXTS": [ "The hypothesis of this prospective study is that intrapartum vibroacoustic stimulation (VAS) is an effective predictor of fetal acidosis during labor. Various clinical conditions, such as term versus preterm gestation, first stage versus second stage of labor, and fetal heart rate (FHR) variable decelerations versus late decelerations will be tested.", "During the study period, 113 patients were studied prospectively in either active phase of first stage (n = 53) or during the second stage of labor (n = 60). They were selected from cases exhibiting moderate to severe FHR variable decelerations or late decelerations. The fetuses of study subjects received a VAS for three seconds and FHR changes were recorded. Fetal scalp blood pH or umbilical arterial blood pH was obtained within 15 minutes of VAS. The relationship between FHR responses to VAS and fetal blood pH in term and preterm gestations, the relationship of two tests (VAS and fetal blood pH) to type of FHR decelerations, and the predictability of neonatal morbidity by two tests were analyzed. Where appropriate, Fisher's exact test (p<0.05 was considered statistically different) and the odd ratio with 95% confidence intervals were used for statistical analyses.", "Excellent association between acceleration response to VAS and pH>or = 7.20, and between a negative response to VAS (no acceleration or decelerations) and pH<7.20 were found in the first stage of labor, the second stage of labor, and the combination of both stages together (p = 0.0001, OR = 10.6 [3.3-34.0]). It was observed that negative VAS responses for predicting fetal acidosis (pH<7.20) were comparable between term (>or = 37 weeks) and preterm (<37 weeks,>or = 34 weeks) fetuses. Since the preterm fetuses enrolled in the study were limited in number, it is difficult to draw adequate conclusions. The positive predictive value (PPV) of fetal acidosis was 67% in both groups of FHR variable decelerations and late decelerations, but the false negative rate of acceleration VAS response for predicting no acidosis was significantly higher in the group of late decelerations (29% vs 8%, p = 0.034). Finally, both a negative VAS response and fetal acidosis (pH<7.20) have equal predictability for neonatal morbidity. The PPV of NICU admission by a negative VAS response was two times higher than that of fetal acidosis (PPV = 61% vs 29%, p = 0.038)." ], "LABELS": [ "OBJECTIVE", "METHODS", "RESULTS" ], "MESHES": [ "Acidosis", "Acoustic Stimulation", "Blood Specimen Collection", "Female", "Fetal Blood", "Fetal Diseases", "Heart Rate, Fetal", "Humans", "Hydrogen-Ion Concentration", "Labor, Obstetric", "Pregnancy", "Prospective Studies", "Scalp", "Sensitivity and Specificity", "Vibration" ], "YEAR": "2001", "reasoning_required_pred": "maybe", "reasoning_free_pred": "yes", "final_decision": "yes", "LONG_ANSWER": "We found that intrapartum VAS was an effective predictor of fetal acidosis in cases of FHR variable decelerations, but its predictability for fetal acidosis in cases of FHR late decelerations was limited. Both VAS and fetal blood pH are good predictors of neonatal morbidity." }, "24922528": { "QUESTION": "The association of puberty and young adolescent alcohol use: do parents have a moderating role?", "CONTEXTS": [ "To explore the extent to which parent-adolescent emotional closeness, family conflict, and parental permissiveness moderate the association of puberty and alcohol use in adolescents (aged 10-14).", "Cross-sectional survey of 7631 adolescents from 231 Australian schools. Measures included pubertal status, recent (30day) alcohol use, parent-adolescent emotional closeness, family conflict, parental permissiveness of alcohol use and peer alcohol use. The analysis was based on a two-level (individuals nested within schools) logistic regression model, with main effects entered first, and interaction terms added second.", "The interaction of family factors and pubertal stage did not improve the fit of the model, so a main effect model of family factors and pubertal stage was adopted. There were significant main effects for pubertal stage with boys in middle puberty at increased odds of alcohol use, and girls in advanced puberty at increased odds of alcohol use." ], "LABELS": [ "PURPOSE", "METHODS", "RESULTS" ], "MESHES": [ "Adolescent", "Alcohol Drinking", "Australia", "Child", "Cross-Sectional Studies", "Family Conflict", "Female", "Humans", "Logistic Models", "Male", "Odds Ratio", "Parent-Child Relations", "Parents", "Puberty" ], "YEAR": "2014", "reasoning_required_pred": "yes", "reasoning_free_pred": "no", "final_decision": "no", "LONG_ANSWER": "Puberty and family factors were strong predictors of adolescent alcohol use, but family factors did not account for variation in the association of pubertal stage and alcohol use." }, "8200238": { "QUESTION": "Must early postoperative oral intake be limited to laparoscopy?", "CONTEXTS": [ "This prospective, randomized study was designed to evaluate whether or not early postoperative feeding (claimed as a unique benefit of laparoscopic surgery) is possible after laparotomy and colorectal resection.", "The trial was performed between July 1, 1992 and October 31, 1992 and included all 64 consecutive patients who underwent laparotomy with either a colonic or an ileal resection. In all cases the nasogastric tube was removed immediately after the operation. Group 1 consisted of 32 patients (age range, 15-81 years; mean, 52 years) who received a regular diet on the first postoperative morning. Group 2 consisted of 32 patients (age range, 15-87 years; mean, 52 years) who were fed in a traditional manner. Regular food was permitted after resolution of ileus as defined by resumption of bowel movements in the absence of abdominal distention, nausea, or vomiting.", "The rate of nasogastric tube reinsertion for distention with persistent vomiting was 18.7 percent (six patients) in Group 1 and 12.5 percent (four patients) in Group 2. Although vomiting was experienced more frequently by patients in Group 1 (44 percent vs. 25 percent, respectively), there was no difference between the two groups with regard to the duration of postoperative ileus (3.6 vs. 3.4 days, respectively). In the 26 patients from Group 1 who did not require nasogastric tube reinsertion, there was a trend toward shorter hospitalization (6.7 vs. 8.0 days, respectively)." ], "LABELS": [ "PURPOSE", "METHODS", "RESULTS" ], "MESHES": [ "Adolescent", "Adult", "Aged", "Aged, 80 and over", "Eating", "Female", "Humans", "Intestinal Obstruction", "Intestines", "Intubation, Gastrointestinal", "Laparoscopy", "Length of Stay", "Male", "Middle Aged", "Postoperative Care", "Postoperative Complications", "Prospective Studies", "Vomiting" ], "YEAR": "1994", "reasoning_required_pred": "no", "reasoning_free_pred": "no", "final_decision": "no", "LONG_ANSWER": "Early oral intake is possible after laparotomy and colorectal resection. Thus, the laparoscopic surgeon's claim of early tolerated oral intake may not be unique to laparoscopy." }, "10593212": { "QUESTION": "Does loss of consciousness predict neuropsychological decrements after concussion?", "CONTEXTS": [ "To investigate the importance of loss of consciousness (LOC) in predicting neuropsychological test performance in a large sample of patients with head injury.", "Retrospective comparison of neuropsychological test results for patients who suffered traumatic LOC, no LOC, or uncertain LOC.", "Allegheny General Hospital, Pittsburgh, Pennsylvania.", "The total number of patients included in this study was 383.", "Neuropsychological test measures, including the visual reproduction, digit span, and logical memory subtests of the Wechsler memory scale (revised), the Trail Making test, Wisconsin Card Sorting test, Hopkins Verbal Learning test, Controlled Oral Word Association, and the Galveston Orientation and Amnesia test (GOAT).", "No significant differences were found between the LOC, no LOC, or uncertain LOC groups for any of the neuropsychological measures used. Patients who had experienced traumatic LOC did not perform more poorly on neuropsychological testing than those with no LOC or uncertain LOC. All three groups demonstrated mildly decreased performance on formal tests of speed of information processing, attentional process, and memory." ], "LABELS": [ "OBJECTIVE", "DESIGN", "SETTING", "PATIENTS", "MAIN OUTCOME MEASURES", "RESULTS" ], "MESHES": [ "Adult", "Brain Concussion", "Female", "Glasgow Coma Scale", "Humans", "Male", "Neuropsychological Tests", "Retrospective Studies", "Unconsciousness" ], "YEAR": "1999", "reasoning_required_pred": "no", "reasoning_free_pred": "no", "final_decision": "no", "LONG_ANSWER": "The results of this study cast doubt on the importance of LOC as a predictor of neuropsychological test performance during the acute phase of recovery from mild traumatic brain injury. Neuropsychological testing procedures have been shown to be sensitive in measuring cognitive sequelae of mild traumatic brain injury (concussion) in athletes. The failure of this study to find any relationship between LOC and neuropsychological functioning in a large sample of patients with mild head trauma calls into question the assignment of primary importance to LOC in grading severity of concussion. This study also does not provide support for the use of guidelines that rely heavily on LOC in making return-to-play decisions. Continued research is necessary to determine the relative importance of markers of concussion in athletes." }, "12630042": { "QUESTION": "Does body mass index (BMI) influence morbidity and long-term survival in gastric cancer patients after gastrectomy?", "CONTEXTS": [ "The long-term survival of patients with gastric cancer is governed by various factors, such as the clinical stage of the cancer, the patient's nutritional state, and the treatment and may be governed by the volume of intraperitoneal adipose tissue. The aim of this study is to clarify the relationship between the degree of the patients' body mass index and their long-term survival.", "Gastric cancer patients who had undergone a gastrectomy with D2-lymphadenectomy and with resection A and B according to the criteria of the Japanese Research Society for Gastric Cancer Rules were subgrouped into those patients with a body mass index<0.185 (the lower body mass index group) and those patients with a body mass index>0.210 (the higher body mass index group). The patient's morbidity and long-term survival rate was retrospectively compared between the 2 groups.", "A significantly longer mean survival rate was observed for the lower body mass index group in stage 2 (1667 vs. 1322 days, P = 0.0240). Also, a significantly longer mean survival rate was observed for the higher BMI group in stage 3a (1431 vs. 943, P = 0.0071)." ], "LABELS": [ "AIMS", "METHODOLOGY", "RESULTS" ], "MESHES": [ "Adult", "Aged", "Body Mass Index", "Female", "Gastrectomy", "Humans", "Male", "Middle Aged", "Neoplasm Recurrence, Local", "Neoplasm Staging", "Postoperative Complications", "Prognosis", "Retrospective Studies", "Risk Factors", "Stomach Neoplasms", "Survival Rate", "Time Factors" ], "YEAR": null, "reasoning_required_pred": "yes", "reasoning_free_pred": "maybe", "final_decision": "maybe", "LONG_ANSWER": "The body mass index is one of the prognostic factors of stage 2 and stage 3a gastric cancer. However, it does not appear to be useful for determining the prognosis of stage 1a, 1b, 3b, and 4a gastric cancers." }, "24519615": { "QUESTION": "Does patella position influence ligament balancing in total knee arthroplasty?", "CONTEXTS": [ "In vivo comparative gap measurements were performed in three different patella positions (reduced, subluxated and everted) using offset-type-force-controlled-spreader-system.", "Prospectively, 50 knees were operated by total knee arthroplasty using a navigation-assisted gap-balancing technique. The offset-type-force-controlled-spreader-system was used for gap measurements. This commercially available instrument allows controllable tension in patella-reduced position. The mediolateral gaps of knee extension (0\u00b0) and flexion (90\u00b0) angle were recorded in three different patella positions; reduced, subluxated and everted. Any gap differences of more than 3 mm were considered as a meaningful difference. Correlation between the difference with the demographic data, preoperative radiologic alignment and intraoperative data was analysed. For statistical analysis, ANOVA and Pearson's correlation test were used.", "The gaps in patella eversion demonstrated smaller gaps both in knee extension and flexion position compared to the gaps of patella reduction position. The amount of decreased gaps was more definite in knee flexion position. Statistically significant difference was observed for the lateral gap of patella eversion compared to gap of patella reduction in knee flexion position (p<0.05). There were notable cases of variability in knee flexion position. Significant portion of 12 (24 %) knees of patella subluxation and 33 (66 %) knees of patella evertion demonstrated either increased or decreased gaps in knee flexion position compared to the gaps of patella reduction position." ], "LABELS": [ "PURPOSE", "METHODS", "RESULTS" ], "MESHES": [ "Aged", "Arthroplasty, Replacement, Knee", "Body Weights and Measures", "Female", "Genu Varum", "Humans", "Knee Joint", "Male", "Osteoarthritis, Knee", "Patella", "Patellar Ligament", "Range of Motion, Articular", "Surgery, Computer-Assisted" ], "YEAR": "2015", "reasoning_required_pred": "yes", "reasoning_free_pred": "yes", "final_decision": "yes", "LONG_ANSWER": "The gaps in patella eversion demonstrated smaller gaps both in knee extension and flexion position compared to the gaps of patella reduction position. The amount of decreased gaps was more definite in knee flexion position. Therefore, the intraoperative patellar positioning has influence on the measurement of the joint gap. Keeping the patella in reduced position is important during gap balancing." }, "24267613": { "QUESTION": "Is the advanced age a contraindication to GERD laparoscopic surgery?", "CONTEXTS": [ "In this prospective non randomized observational cohort study we have evaluated the influence of age on outcome of laparoscopic total fundoplication for GERD.", "Six hundred and twenty consecutive patients underwent total laparoscopic fundoplication for GERD. Five hundred and twenty-four patients were younger than 65 years (YG), and 96 patients were 65 years or older (EG). The following parameters were considered in the preoperative and postoperative evaluation: presence, duration, and severity of GERD symptoms, presence of a hiatal hernia, manometric and 24 hour pH-monitoring data, duration of operation, incidence of complications and length of hospital stay.", "Elderly patients more often had atypical symptoms of GERD and at manometric evaluation had a higher rate of impaired esophageal peristalsis in comparison with younger patients. The duration of the operation was similar between the two groups. The incidence of intraoperative and postoperative complications was low and the difference was not statistically significant between the two groups. An excellent outcome was observed in 93.0% of young patients and in 88.9% of elderly patients (p = NS)." ], "LABELS": [ "BACKGROUND", "METHODS", "RESULTS" ], "MESHES": [ "Adolescent", "Adult", "Age Factors", "Aged", "Aged, 80 and over", "Child", "Contraindications", "Female", "Follow-Up Studies", "Fundoplication", "Gastroesophageal Reflux", "Humans", "Laparoscopy", "Male", "Middle Aged", "Prospective Studies", "Time Factors", "Young Adult" ], "YEAR": "2013", "reasoning_required_pred": "no", "reasoning_free_pred": "no", "final_decision": "no", "LONG_ANSWER": "Laparoscopic antireflux surgery is a safe and effective treatment for GERD even in elderly patients, warranting low morbidity and mortality rates and a significant improvement of symptoms comparable to younger patients." }, "21074975": { "QUESTION": "Ultra high risk (UHR) for psychosis criteria: are there different levels of risk for transition to psychosis?", "CONTEXTS": [ "The ultra high risk (UHR) for psychosis criteria have been validated in a number of studies. However, it is not known whether particular UHR criteria (Attenuated Psychotic Symptoms (APS), Brief Limited Intermittent Psychotic Symptoms (BLIPS) or Trait vulnerability criteria), or combination of criteria, is associated with a higher risk of transition to psychosis. The current study investigated this issue over a 6-month follow-up period. We hypothesised that the risk of transition would increase in the following order: Trait aloneor = 18.", "Neutrophils were isolated from citrated blood from 17 adult blunt major trauma patients at admission (0 hrs) and 8 and 24 hrs later. Identical samples were obtained from normal laboratory volunteers (n = 9). The neutrophil count in each specimen was measured, and the number of neutrophils migrating across porous tissue culture inserts in response to defined concentrations of interleukin-8 (0, 10, 30, and 100 ng/mL) was quantitated by peroxidase assay. Neutrophil counts in the whole blood specimens obtained from those later developing ARDS were elevated significantly at admission and declined rapidly throughout the next 24 hrs. Significantly greater numbers of trauma patients' neutrophils migrated to concentrations of interleukin-8 (30 and 100 ng/mL) at each time point when compared with normal volunteers (Mann-Whitney U test, p<.05). Neutrophils isolated from major trauma patients exhibited an enhanced migratory response to high concentrations of interleukin-8 throughout the first 24 hrs of admission, in contrast to the normal physiologic attenuation of migration seen in neutrophils isolated from normal laboratory volunteers." ], "LABELS": [ "OBJECTIVE", "DESIGN", "SETTING", "PATIENTS", "MEASUREMENTS AND MAIN RESULTS" ], "MESHES": [ "Acute Disease", "Adult", "Cell Movement", "Humans", "Interleukin-8", "Leukocyte Count", "Neutrophil Infiltration", "Neutrophils", "Prospective Studies", "Respiratory Distress Syndrome, Adult", "Risk Factors", "Time Factors", "Trauma Severity Indices", "Wales", "Wounds, Nonpenetrating" ], "YEAR": "2002", "reasoning_required_pred": "yes", "reasoning_free_pred": "yes", "final_decision": "yes", "LONG_ANSWER": "These data indicate that major blunt trauma enhances the migratory capacity of circulating neutrophils. This is manifest within 2 hrs of admission and may be attributable to alteration in interleukin-8 receptor expression, affinity, or downstream signaling. In patients who later develop ARDS, initially elevated circulating neutrophil counts decrease rapidly, over the same time course. Early enhanced neutrophil migratory activity coupled with elevated pulmonary concentrations of interleukin-8 may be central to the establishment of the neutrophil infiltration that is characteristic of ARDS." }, "16392897": { "QUESTION": "BCRABL transcript detection by quantitative real-time PCR : are correlated results possible from homebrew assays?", "CONTEXTS": [ "Quantitative real-time PCR has become the predominant molecular technique to monitor BCRABL levels in response to treatment in Ph(+) leukemia patients. However, without some form of standardized methodology between laboratories, the correlation of results is difficult.", "Using TaqMan-based assays, parallel quantitative real-time PCR analysis was performed on 70 clinical specimens at Vanderbilt University Medical Center and Virginia Commonwealth University. While the same positive control cell line (K562) and quality control gene (BCR) were used, the RNA isolation technique, cDNA synthesis, BCR control cell line, and PCR primer and probe sequences were different.", "The detection of BCRABL-positive results spanned a dynamic range from 10(0) to 10(5)/100,000 cells. Forty-three samples were negative at both facilities. A Spearman rank correlation analysis was performed for the 22 BCRABL-positive paired results. The correlation coefficient, r(s), was 0.9435 (p<0.00001), suggesting a strong correlation of the results. One discordant result was obtained for consecutive samples from one patient with a low BCRABL copy number as a result of a minimal RNA yield at one laboratory." ], "LABELS": [ "BACKGROUND", "METHODS", "RESULTS" ], "MESHES": [ "Adolescent", "Adult", "Aged", "Aged, 80 and over", "Antineoplastic Agents", "Benzamides", "Bone Marrow", "Bone Marrow Transplantation", "Genes, abl", "Humans", "Imatinib Mesylate", "K562 Cells", "Leukemia, Myelogenous, Chronic, BCR-ABL Positive", "Leukemia, Myeloid", "Middle Aged", "Molecular Diagnostic Techniques", "Piperazines", "Polymerase Chain Reaction", "Precursor Cell Lymphoblastic Leukemia-Lymphoma", "Pyrimidines", "RNA, Messenger", "RNA, Neoplasm", "Sensitivity and Specificity" ], "YEAR": "2005", "reasoning_required_pred": "maybe", "reasoning_free_pred": "yes", "final_decision": "maybe", "LONG_ANSWER": "These results suggest that quantitative real-time PCR assays for BCRABL detection can be comparable between laboratories despite significant differences in methodologies if the same positive control cell line and quality control gene are used. It is imperative that some level of assay standardization be adopted between laboratories, not only for patients who are monitored at different facilities, but also for larger investigative studies in which hematologic, cytogenetic and molecular responses are to be compared." }, "19615731": { "QUESTION": "Does higher body mass index contribute to worse asthma control in an urban population?", "CONTEXTS": [ "Epidemiologic findings support a positive association between asthma and obesity.", "Determine whether obesity or increasing level of body mass index (BMI) are associated with worse asthma control in an ethnically diverse urban population.", "Cross-sectional assessment of asthma control was performed in patients with asthma recruited from primary care offices by using 4 different validated asthma control questionnaires: the Asthma Control and Communication Instrument (ACCI), the Asthma Control Test (ACT), the Asthma Control Questionnaire (ACQ), and the Asthma Therapy Assessment Questionnaire (ATAQ). Multiple linear regression analysis was performed to evaluate the association between obesity and increasing BMI level and asthma control.", "Of 292 subjects with a mean age of 47 years, the majority were women (82%) and African American (67%). There was a high prevalence of obesity with 63%, with only 15% normal weight. The mean score from all 4 questionnaires showed an average suboptimal asthma control (mean score/maximum possible score): ACCI (8.3/19), ACT (15.4/25), ACQ (2.1/6), and ATAQ (1.3/4). Regression analysis showed no association between obesity or increasing BMI level and asthma control using all 4 questionnaires. This finding persisted even after adjusting for FEV(1), smoking status, race, sex, selected comorbid illnesses, and long-term asthma controller use." ], "LABELS": [ "BACKGROUND", "OBJECTIVE", "METHODS", "RESULTS" ], "MESHES": [ "Aged", "Anti-Asthmatic Agents", "Asthma", "Body Mass Index", "Cross-Sectional Studies", "Female", "Humans", "Linear Models", "Male", "Middle Aged", "Obesity", "Prescription Drugs", "Spirometry", "Surveys and Questionnaires", "United States", "Urban Population" ], "YEAR": "2009", "reasoning_required_pred": "no", "reasoning_free_pred": "no", "final_decision": "no", "LONG_ANSWER": "Using 4 validated asthma control questionnaires, we failed to find an association between obesity and asthma control in an urban population with asthma. Weight loss may not be an appropriate strategy to improve asthma control in this population." }, "18537964": { "QUESTION": "Does a physician's specialty influence the recording of medication history in patients' case notes?", "CONTEXTS": [ "To determine the impact of a physician's specialty on the frequency and depth of medication history documented in patient medical records.", "A cross-sectional assessment of the frequency and depth of medication history information documented by 123 physicians for 900 randomly selected patients stratified across Cardiology, Chest, Dermatology, Endocrine, Gastroenterology, Haematology, Neurology, Psychiatry and Renal specialties was carried out at a 900-bed teaching hospital located in Ibadan, Nigeria.", "Four hundred and forty-three (49.2%) of the cohort were males and 457 (50.8%) were females; with mean ages 43.2 +/- 18.6 and 43.1 +/- 17.9 years respectively. Physicians' specialties significantly influenced the depth of documentation of the medication history information across the nine specialties (P<0.0001). Post hoc pair-wise comparisons with Tukey's HSD test showed that the mean scores for adverse drug reactions and adherence to medicines was highest in the Cardiology specialty; while the Chest specialty had the highest mean scores for allergy to drugs, food, chemicals and cigarette smoking. Mean scores for the use of alcohol; illicit drugs; dietary restrictions was highest for Gastroenterology, Psychiatry and Endocrine specialties respectively. Physicians' specialties also significantly influenced the frequency of documentation of the medication history across the nine specialties (P<0.0001)." ], "LABELS": [ "AIMS", "METHODS", "RESULTS" ], "MESHES": [ "Adult", "Attitude of Health Personnel", "Clinical Protocols", "Cross-Sectional Studies", "Female", "Humans", "Male", "Medical History Taking", "Medical Records", "Medicine", "Middle Aged", "Nigeria", "Physicians", "Practice Patterns, Physicians'", "Random Allocation", "Specialization" ], "YEAR": "2008", "reasoning_required_pred": "yes", "reasoning_free_pred": "yes", "final_decision": "yes", "LONG_ANSWER": "Physicians appear to document more frequently and in greater depth medication history information that may aid the diagnostic tasks in their specific specialty. Researchers and other users of medication history data documented in patients' medical records by physicians may want to take special cognizance of this phenomenon." }, "25747932": { "QUESTION": "Living in an urban environment and non-communicable disease risk in Thailand: Does timing matter?", "CONTEXTS": [ "This paper uses a life-course approach to explore whether the timing and/or duration of urban (vs rural) exposure was associated with risk factors for NCDs.", "A cross-sectional survey was conducted among health care workers in two hospitals in Thailand. Two measures of urbanicity were considered: early-life urban exposure and the proportion of urban life years. We explored four behavioral NCD risk factors, two physiological risk factors and four biological risk factors.", "Both measures of urbanicity were each independently associated with increases in all behavioral and physiological risk factors. For some biological risk factors, people spending their early life in an urban area may be more susceptible to the effect of increasing proportion of urban life years than those growing up in rural areas." ], "LABELS": [ "BACKGROUND", "METHODS", "RESULTS" ], "MESHES": [ "Adult", "Child, Preschool", "Chronic Disease", "Cross-Sectional Studies", "Humans", "Middle Aged", "Prevalence", "Risk Factors", "Thailand", "Time Factors", "Urban Health" ], "YEAR": "2015", "reasoning_required_pred": "yes", "reasoning_free_pred": "yes", "final_decision": "yes", "LONG_ANSWER": "Urbanicity was associated with increases in behavioral and physiological risk factors. However, these associations may not translate directly into increases in biological risk factors. It is likely that these biological risk factors were results of a complex interaction between both long term accumulation of exposure and early life exposures." }, "17445978": { "QUESTION": "Is renal warm ischemia over 30 minutes during laparoscopic partial nephrectomy possible?", "CONTEXTS": [ "To evaluate renal damage and impairment of renal function 1 yr after laparoscopic partial nephrectomy (LPN) with warm ischemia>30 min.", "From July 2004 to June 2005, 18 patients underwent LPN with warm ischemia time>30 min. Kidney damage markers (daily proteinuria and tubular enzymes) and renal function (serum creatinine, cystatin C, and creatinine clearances) were assessed on postoperative days 1 and 5 and at 12 mo. Glomerular filtration rate (GFR) was evaluated before surgery and at 3 mo. Renal scintigraphy was performed before the procedure, at 5 d and at 3 and 12 mo postoperatively. Statistical analysis was performed using the Student t test and logistic regression analysis.", "In terms of kidney damage and renal function markers, the statistical analysis demonstrated that at 1 yr there was complete return to the normal range and no statistical difference between the values at the various time points. The GFR was not significantly different before and 3 mo after surgery. In terms of scintigraphy of the operated kidney, the values were 48.35+/-3.82% (40-50%) before the procedure, 36.88+/-8.42 (16-50%) on postoperative day 5 (p=0.0001), 40.56+/-8.96 (20-50%) at 3 mo (p=0.003), and 42.8+/-7.2% (20-50%) 1 yr after surgery (p=0.001)." ], "LABELS": [ "OBJECTIVE", "METHODS", "RESULTS" ], "MESHES": [ "Humans", "Ischemia", "Kidney", "Laparoscopy", "Nephrectomy", "Prospective Studies" ], "YEAR": "2007", "reasoning_required_pred": "yes", "reasoning_free_pred": "no", "final_decision": "no", "LONG_ANSWER": "Our results demonstrate that kidney damage occurs during LPN when warm ischemia is>30 min. This damage is only partially reversible and efforts should be made to keep warm ischemia within 30 min." }, "19309468": { "QUESTION": "Does a febrile reaction to platelets predispose recipients to red blood cell alloimmunization?", "CONTEXTS": [ "A variable effect of inflammation on alloimmunization to transfused red blood cells (RBCs) in mice has been recently reported. We investigated whether RBC alloimmunization in humans was affected by transfusion of blood products in temporal proximity to experiencing a febrile transfusion reaction (FTR) to platelets (PLTs), an event predominantly mediated by inflammatory cytokines.", "Blood bank databases were used to identify patients who experienced an FTR or possible FTR to PLTs from August 2000 to March 2008 (FTR group). The control group of patients received a PLT transfusion on randomly selected dates without experiencing an FTR. The \"event\" was defined as the PLT transfusion that caused the FTR in the FTR group or the index PLT transfusion in the control group. The number of transfused blood products and their proximity to the event were recorded along with other recipient data. The primary endpoint was the rate of RBC alloimmunization between the two groups.", "There were 190 recipients in the FTR group and 245 in the control group. Overall, the recipients in the control group were younger and received more blood products on the day of their event and over the subsequent 10 days. The alloimmunization rate among recipients in the FTR group was higher than in the control group (8% vs. 3%, respectively; p = 0.026)." ], "LABELS": [ "BACKGROUND", "STUDY DESIGN AND METHODS", "RESULTS" ], "MESHES": [ "Adult", "Aged", "Erythrocytes", "Fever", "Humans", "Isoantibodies", "Middle Aged", "Platelet Transfusion" ], "YEAR": "2009", "reasoning_required_pred": "yes", "reasoning_free_pred": "yes", "final_decision": "yes", "LONG_ANSWER": "These preliminary data support our hypothesis that recipient inflammation may affect RBC alloimmunization in humans; however, a more detailed understanding of the pathophysiologic association between inflammation and alloimmunization is required before definitive conclusions can be reached." }, "25885219": { "QUESTION": "Maternal creatine homeostasis is altered during gestation in the spiny mouse: is this a metabolic adaptation to pregnancy?", "CONTEXTS": [ "Pregnancy induces adaptations in maternal metabolism to meet the increased need for nutrients by the placenta and fetus. Creatine is an important intracellular metabolite obtained from the diet and also synthesised endogenously. Experimental evidence suggests that the fetus relies on a maternal supply of creatine for much of gestation. However, the impact of pregnancy on maternal creatine homeostasis is unclear. We hypothesise that alteration of maternal creatine homeostasis occurs during pregnancy to ensure adequate levels of this essential substrate are available for maternal tissues, the placenta and fetus. This study aimed to describe maternal creatine homeostasis from mid to late gestation in the precocial spiny mouse.", "Plasma creatine concentration and urinary excretion were measured from mid to late gestation in pregnant (n = 8) and age-matched virgin female spiny mice (n = 6). At term, body composition and organ weights were assessed and tissue total creatine content determined. mRNA expression of the creatine synthesising enzymes arginine:glycine amidinotransferase (AGAT) and guanidinoacetate methyltransferase (GAMT), and the creatine transporter (CrT1) were assessed by RT-qPCR. Protein expression of AGAT and GAMT was also assessed by western blot analysis.", "Plasma creatine and renal creatine excretion decreased significantly from mid to late gestation (P<0.001, P<0.05, respectively). Pregnancy resulted in increased lean tissue (P<0.01), kidney (P<0.01), liver (P<0.01) and heart (P<0.05) mass at term. CrT1 expression was increased in the heart (P<0.05) and skeletal muscle (P<0.05) at term compared to non-pregnant tissues, and creatine content of the heart (P<0.05) and kidney (P<0.001) were also increased at this time. CrT1 mRNA expression was down-regulated in the liver (<0.01) and brain (<0.01) of pregnant spiny mice at term. Renal AGAT mRNA (P<0.01) and protein (P<0.05) expression were both significantly up-regulated at term, with decreased expression of AGAT mRNA (<0.01) and GAMT protein (<0.05) observed in the term pregnant heart. Brain AGAT (<0.01) and GAMT (<0.001) mRNA expression were also decreased at term." ], "LABELS": [ "BACKGROUND", "METHODS", "RESULTS" ], "MESHES": [ "Amidinotransferases", "Animals", "Blotting, Western", "Creatine", "Female", "Gene Expression Regulation", "Guanidinoacetate N-Methyltransferase", "Homeostasis", "Membrane Transport Proteins", "Murinae", "Pregnancy", "RNA, Messenger", "Reverse Transcriptase Polymerase Chain Reaction" ], "YEAR": "2015", "reasoning_required_pred": "yes", "reasoning_free_pred": "yes", "final_decision": "yes", "LONG_ANSWER": "Change of maternal creatine status (increased creatine synthesis and reduced creatine excretion) may be a necessary adjustment of maternal physiology to pregnancy to meet the metabolic demands of maternal tissues, the placenta and developing fetus." }, "26348845": { "QUESTION": "Pap smears with glandular cell abnormalities: Are they detected by rapid prescreening?", "CONTEXTS": [ "Rapid prescreening (RPS) is one of the quality assurance (QA) methods used in gynecologic cytology. The efficacy of RPS has been previously studied but mostly with respect to squamous lesions; in fact, there has been no study so far specifically looking at the sensitivity of RPS for detecting glandular cell abnormalities.", "A total of 80,565 Papanicolaou (Pap) smears underwent RPS during a 25-month period. A sample was designated as \"review for abnormality\" (R) if any abnormal cells (at the threshold of atypical squamous cells of undetermined significance/atypical glandular cells [AGC]) were thought to be present or was designated as negative (N) if none were detected. Each sample then underwent full screening (FS) and was designated as either R or N and also given a cytologic interpretation.", "The final cytologic interpretation was a glandular cell abnormality (\u2265AGC) in 107 samples (0.13%); 39 of these (36.4%) were flagged as R on RPS. Twenty-four patients (33.8%) out of 71 who had histologic follow-up were found to harbor a high-grade squamous intraepithelial lesion or carcinoma; 13 of those 24 Pap smears (54.2%) had been flagged as R on RPS. Notably, 11 AGC cases were picked up by RPS only and not by FS and represented false-negative cases; 2 of these showed endometrial adenocarcinoma on histologic follow-up." ], "LABELS": [ "BACKGROUND", "METHODS", "RESULTS" ], "MESHES": [ "Adenocarcinoma", "Adult", "Aged", "Aged, 80 and over", "Female", "Humans", "Mass Screening", "Middle Aged", "Papanicolaou Test", "Sensitivity and Specificity", "Uterine Cervical Neoplasms", "Vaginal Smears", "Young Adult" ], "YEAR": "2015", "reasoning_required_pred": "yes", "reasoning_free_pred": "yes", "final_decision": "yes", "LONG_ANSWER": "Pap smears with glandular cell abnormalities are often flagged as abnormal by RPS, and this results in a sensitivity of 36.4% (at the AGC threshold). Most importantly, some cases of AGC are detected on Pap smears by RPS only, and this demonstrates that RPS is a valuable QA method." }, "12221908": { "QUESTION": "The HELPP syndrome--evidence of a possible systemic inflammatory response in pre-eclampsia?", "CONTEXTS": [ "The principal causes of morbidity and mortality during pregnancy in Mexico, are preeclampsia/eclampsia, obstetric hemorrhage and puerperium complications; this is, 62% of maternal deaths in last years. HELLP syndrome was observed between 5 to 25% of the mortality in pregnancies of 36 weeks or less.", "To analyze patients with HELLP syndrome in ICU's (Intensive Care Unit) of a Gynecology and Obstetric Hospital, related to the abnormal hematological, hepatic and renal results with the obstetric case history and the clinical complications.", "A transversal study in patients with HELLP syndrome during 1998 and 1999 were carry out.", "Peripheral blood with Microangiopathic hemolysis, elevated liver enzymes: AST, ALT over 40 UI/L, even when were LDH lower than 600 UI/L. It was evaluated the hepatic and renal function, platelets count, microangiopathic hemolysis, arterial pressure, seizures, icteric skin color, blindness, visual disturbances, nausea, vomiting and upper quadrant right abdominal pain. In newborn we analyzed gestational age, sex, weight and APGAR. We studied for an association between maternal and biochemical variables with Correlation Pearson Test, and dependence between variables with lineal regression model.", "2878 patients with hypertensives disorders in pregnancy (11.64%). The 1.15% (n = 33) had HELLP syndrome with specific maternal mortality of 0.4 per 10,000 live birth, perinatal mortality of 1.62 per 10,000 live birth; and renal damage in 84.5%. Coefficient beta was higher between number of pregnancies to platelets count (-0.33) and creatinine clearance (-0.401)." ], "LABELS": [ "INTRODUCTION", "OBJECTIVE", "MATERIALS AND METHODS", "CASE DEFINITION", "RESULTS" ], "MESHES": [ "Abortion, Induced", "Adult", "Anemia, Hemolytic", "Cesarean Section", "Comorbidity", "Critical Care", "Cross-Sectional Studies", "Disease Susceptibility", "Female", "Humans", "Hypertension", "Infant Mortality", "Infant, Newborn", "Infant, Newborn, Diseases", "Kidney Function Tests", "Liver Diseases", "Liver Function Tests", "Male", "Maternal Age", "Maternal Mortality", "Mexico", "Parity", "Pre-Eclampsia", "Pregnancy", "Pregnancy Complications", "Socioeconomic Factors", "Systemic Inflammatory Response Syndrome", "Thrombocytopenia" ], "YEAR": "2002", "reasoning_required_pred": "yes", "reasoning_free_pred": "yes", "final_decision": "yes", "LONG_ANSWER": "We found an important renal damage, low platelets, elevated liver enzymes in women with two or more pregnancies. Then we propose there are similarities between HELLP syndrome and Systemic Inflammatory Response Syndrome (SIRS) because they could have the same pathophysiology." }, "21904069": { "QUESTION": "Spleen-preserving distal pancreatectomy with resection of the splenic vessels. Should one rely on the short gastric arteries?", "CONTEXTS": [ "Knowing the collaterals is essential for a spleen-preserving distal pancreatectomy with resection of the splenic vessels.", "To ascertain the sources of the blood supply to the spleen after a spleen-preserving distal pancreatectomy with resection of the splenic vessels.", "Perfusion of the cadaveric left gastric and right gastroepiploic arteries with methylene blue after occlusion of all the arteries except the short gastric arteries (n=10). Intraoperative color Doppler ultrasound was used for the evaluation of the hilar arterial blood flow at distal pancreatectomy (n=23) after 1) clamping of the splenic artery alone, 2) clamping of the splenic and left gastroepiploic arteries and 3) clamping of the splenic and short gastric arteries. CT angiography of the gastric and splenic vessels before and after a spleen-preserving distal pancreatectomy (n=10).", "Perfusion of the cadaveric arteries revealed no effective direct or indirect (through the submucous gastric arterial network) communication between the left gastric and the branches of the short gastric arteries. In no case did intraoperative color Doppler ultrasound detect any hilar arterial blood flow after the clamping of the splenic and left gastroepiploic arteries. The clamping of the short gastric arteries did not change the flow parameters. In none of the cases did a post-spleen-preserving distal pancreatectomy with resection of the splenic vessels CT angiography delineate the short gastric vessels supplying the spleen. In all cases, the gastroepiploic arcade was the main arterial pathway feeding the spleen." ], "LABELS": [ "CONTEXT", "OBJECTIVE", "METHODS", "RESULTS" ], "MESHES": [ "Adult", "Arteries", "Cadaver", "Female", "Gastroepiploic Artery", "Humans", "Intraoperative Period", "Male", "Middle Aged", "Models, Biological", "Organ Sparing Treatments", "Pancreatectomy", "Spleen", "Splenic Artery", "Splenic Vein", "Stomach", "Ultrasonography, Doppler, Color" ], "YEAR": "2011", "reasoning_required_pred": "yes", "reasoning_free_pred": "no", "final_decision": "no", "LONG_ANSWER": "Experimental, intra- and postoperative instrumental investigations did not show the short gastric arteries to be engaged in the blood supply to the spleen after a spleen-preserving distal pancreatectomy with resection of the splenic vessels. In all cases, the left gastroepiploic artery was the main collateral vessel." }, "14976655": { "QUESTION": "Delayed peripheral facial palsy in the stapes surgery: can it be prevented?", "CONTEXTS": [ "The aim of this study was to evaluate poststapedectomy-delayed facial palsy etiopathogenesis, risk factors, evolution, and prevention.", "Seven hundred six stapedectomies performed in 580 patients were reviewed. In all patients who developed delayed facial palsy, the dates of onset and subside of facial palsy, the anatomic and pathologic predisposing factors, and a possible history for recurrent labial herpetic lesions were considered. The House-Brackmann (H-B) grading system was used to evaluate the facial function. Virus-specific immunoglobulin (Ig) G and IgM antibodies against herpes simplex virus type 1 (HSV-1) were determined by enzyme-linked immunosorbent assay (ELISA) 3 weeks after the onset of the paralysis. The results were compared with a control group without a history of recurrent herpes labialis.", "Poststapedectomy facial palsy developed in 7 out of 706 procedures. All 7 patients referred a history of recurrent labial herpetic lesions. One patient showed a facial palsy H-B grade II, 2 a grade III, and 3 a grade IV. After acyclovir therapy, 6 subjects recovered completely, whereas 1 maintained an H-B grade II. An increased IgG antibody titer was found in 6 of the patients with delayed facial palsy and in 1 out of 7 controls. Mean IgG titer was 1:14,050 in the subjects with delayed facial palsy and 1:2,300 in controls (P<.001)." ], "LABELS": [ "PURPOSE", "MATERIALS AND METHODS", "RESULTS" ], "MESHES": [ "Acyclovir", "Adult", "Antibodies, Viral", "Antiviral Agents", "Audiometry, Pure-Tone", "Enzyme-Linked Immunosorbent Assay", "Facial Paralysis", "Female", "Herpes Labialis", "Herpesvirus 1, Human", "Humans", "Male", "Middle Aged", "Postoperative Complications", "Recurrence", "Retrospective Studies", "Risk Factors", "Stapes Surgery", "Virus Latency" ], "YEAR": null, "reasoning_required_pred": "yes", "reasoning_free_pred": "yes", "final_decision": "yes", "LONG_ANSWER": "Poststapedectomy-delayed facial palsy is likely caused by a reactivation of HSV-1, latent within the geniculate ganglion. The activation of the latent virus is more frequent in patients with a history of herpes labialis and can be prevented by an adequate acyclovir therapy." }, "22644412": { "QUESTION": "Hepatic arterial embolization for unresectable hepatocellular carcinomas: do technical factors affect prognosis?", "CONTEXTS": [ "To evaluate retrospectively whether technical factors of hepatic arterial embolization affect the prognosis of patients with hepatocellular carcinoma (HCC).", "Inclusion criteria of this study were the following: (1) patients received embolization as the initial treatment during 2003-2004, (2) Child A or B liver profile, (3) five or fewer HCCs with maximum diameter of 7 cm or smaller, and (4) no extrahepatic metastasis. Patient data were gathered from 43 centers. Prognostic factors were evaluated using univariate and multivariate analyses.", "Eight hundred fifteen patients were enrolled. The 1-, 3-, 5-, and 7-year overall survival rates were 92.0 % (95 % CI 90.1-93.9), 62.9 % (95 % CI 59.3-66.6), 39.0 % (95 % CI 35.1-43.0), and 26.7 % (95 % CI 22.6-30.8) in all patients. Univariate analysis showed a Child-Pugh class-A, alpha-fetoprotein level lower than 100 ng/ml, tumor size of 3 cm or smaller, tumor number of 3 or fewer, one-lobe tumor distribution, nodular tumor type, within the Milan criteria, stage I or II, no portal venous invasion, use of iodized oil, and selective embolization were significantly better prognostic factors. In the multivariate Cox model, the benefit to survival of selective embolization remained significant (hazard ratio 0.68; 95 % CI 0.48-0.97; p = 0.033)." ], "LABELS": [ "PURPOSE", "MATERIALS AND METHODS", "RESULTS" ], "MESHES": [ "Adult", "Aged", "Aged, 80 and over", "Carcinoma, Hepatocellular", "Chemoembolization, Therapeutic", "Disease Progression", "Female", "Hepatic Artery", "Humans", "Iodized Oil", "Liver Neoplasms", "Male", "Middle Aged", "Prognosis", "Proportional Hazards Models", "Retrospective Studies", "Survival Rate", "alpha-Fetoproteins" ], "YEAR": "2012", "reasoning_required_pred": "yes", "reasoning_free_pred": "yes", "final_decision": "yes", "LONG_ANSWER": "Selective embolization contributes to survival in patients with HCCs." }, "26419377": { "QUESTION": "Are pelvic anatomical structures in danger during arthroscopic acetabular labral repair?", "CONTEXTS": [ "The purpose of this study was to evaluate safe depth for suture anchor insertion during acetabular labral repair and to determine the neighbouring structures at risk during drilling and anchor insertion.", "Ten human cadaveric hips (six males and four females) were obtained. Acetabular labral surface was prepared and marked for right hips as 12, 1 and 3 o'clock positions, for left hips 12, 11 and 9 o'clock positions. Those were defined as anterior, anterior-superior and superior zones, respectively. These labral positions were drilled at defined zones. After measurements, depth of the bone at 10\u00b0 and 20\u00b0 drill angles on zones was compared statistically.", "Acetabular bone widths at investigated labral insertion points did not statistically differ. A total of 14 injuries in 60 penetrations occurred (23.3\u00a0%) with free drill penetrations, and no injuries occurred with stopped drill penetrations. The bone depth was gradually decreasing from 10\u00b0 to 20\u00b0 drill angles and from anterior to superior inserting zones without significant importance. The risk of perforation to the pelvic cavity started with 20\u00a0mm drill depth, and the mean depth for all insertions was calculated as 31.7\u00a0mm (SD 2.6)." ], "LABELS": [ "PURPOSE", "METHODS", "RESULTS" ], "MESHES": [ "Acetabulum", "Arthroscopy", "Cadaver", "Cartilage, Articular", "Female", "Hip Joint", "Humans", "Male", "Pelvis", "Postoperative Complications", "Safety", "Suture Anchors" ], "YEAR": "2017", "reasoning_required_pred": "no", "reasoning_free_pred": "yes", "final_decision": "yes", "LONG_ANSWER": "It is anatomically possible that some pelvic structures sustain iatrogenic injury during acetabular drilling for anchor placement. Being aware of mean pelvic vault is important in which drilling after excessive pincer rim trimming could easily decrease the depth of acetabular bone without a standard. Careful drilling by using stopped drill bit is crucial to prevent iatrogenic injury." }, "23052500": { "QUESTION": "Staging laparoscopy in patients with hepatocellular carcinoma: is it useful?", "CONTEXTS": [ "Staging laparoscopy (SL) is not regularly performed for patients with hepatocellular carcinoma (HCC). It may change treatment strategy, preventing unnecessary open exploration. An additional advantage of SL is possible biopsy of the nontumorous liver to assess fibrosis/cirrhosis. This study aimed to determine whether SL for patients with HCC still is useful.", "Patients with HCC who underwent SL between January 1999 and December 2011 were analyzed. Their demographics, preoperative imaging studies, surgical findings, and histology were assessed.", "The 56 patients (34 men and 22 women; mean age, 60 \u00b1 14 years) in this study underwent SL for assessment of extensive disease or metastases. For two patients, SL was unsuccessful because of intraabdominal adhesions. For four patients (7.1 %), SL showed unresectability because of metastases (n = 1), tumor progression (n = 1), or severe cirrhosis in the contralateral lobe (n = 2). An additional five patients did not undergo laparotomy due to disease progression detected on imaging after SL. Exploratory laparotomy for the remaining 47 patients showed 6 (13 %) additional unresectable tumors due to advanced tumor (n = 5) or nodal metastases (n = 1). Consequently, the yield of SL was 7 % (95 % confidence interval (CI), 3-17 %), and the accuracy was 27 % (95 % CI, 11-52 %). A biopsy of the contralateral liver was performed for 45 patients who underwent SL, leading to changes in management for 4 patients (17 %) with cirrhosis." ], "LABELS": [ "BACKGROUND", "METHODS", "RESULTS" ], "MESHES": [ "Carcinoma, Hepatocellular", "Female", "Humans", "Laparoscopy", "Laparotomy", "Liver Cirrhosis", "Liver Neoplasms", "Magnetic Resonance Imaging", "Male", "Middle Aged", "Neoplasm Metastasis", "Neoplasm Recurrence, Local", "Neoplasm Staging", "Tomography, X-Ray Computed" ], "YEAR": "2013", "reasoning_required_pred": "yes", "reasoning_free_pred": "no", "final_decision": "no", "LONG_ANSWER": "The overall yield of SL for HCC was 7 %, and the accuracy was 27 %. When accurate imaging methods are available and additional percutaneous liver biopsy is implemented as a standard procedure in the preoperative workup of patients with HCC, the benefit of SL will become even less." }, "26104852": { "QUESTION": "Can echocardiography and ECG discriminate hereditary transthyretin V30M amyloidosis from hypertrophic cardiomyopathy?", "CONTEXTS": [ "Hereditary transthyretin (ATTR) amyloidosis with increased left ventricular wall thickness could easily be misdiagnosed by echocardiography as hypertrophic cardiomyopathy (HCM). Our aim was to create a diagnostic tool based on echocardiography and ECG that could optimise identification of ATTR amyloidosis.", "Data were analysed from 33 patients with biopsy proven ATTR amyloidosis and 30 patients with diagnosed HCM. Conventional features from ECG were acquired as well as two dimensional and Doppler echocardiography, speckle tracking derived strain and tissue characterisation analysis. Classification trees were used to select the most important variables for differentiation between ATTR amyloidosis and HCM.", "The best classification was obtained using both ECG and echocardiographic features, where a QRS voltage>30\u2009mm was diagnostic for HCM, whereas in patients with QRS voltage<30\u2009mm, an interventricular septal/posterior wall thickness ratio (IVSt/PWt)>1.6 was consistent with HCM and a ratio<1.6 supported the diagnosis of ATTR amyloidosis. This classification presented both high sensitivity (0.939) and specificity (0.833)." ], "LABELS": [ "OBJECTIVE", "METHODS", "RESULTS" ], "MESHES": [ "Adult", "Aged", "Amino Acid Substitution", "Amyloidosis", "Biopsy", "Cardiomyopathy, Hypertrophic", "Diagnosis, Differential", "Echocardiography, Doppler", "Electrocardiography", "Female", "Gene Expression", "Heart Ventricles", "Humans", "Male", "Middle Aged", "Mutation", "Myocardium", "Prealbumin" ], "YEAR": "2015", "reasoning_required_pred": "yes", "reasoning_free_pred": "yes", "final_decision": "yes", "LONG_ANSWER": "Our study proposes an easily interpretable classification method for the differentiation between HCM and increased left ventricular myocardial thickness due to ATTR amyloidosis. Our combined echocardiographic and ECG model could increase the ability to identify ATTR cardiac amyloidosis in clinical practice." }, "9142039": { "QUESTION": "Does pediatric housestaff experience influence tests ordered for infants in the neonatal intensive care unit?", "CONTEXTS": [ "To assess the relationship between the experience of pediatric housestaff and tests ordered on infants in the neonatal intensive care unit (ICU).", "Prospective, cohort study over one full academic year.", "One academic Level III neonatal intensive care nursery.", "Data were collected prospectively on all 785 infants admitted to the neonatal ICU from July 1993 to June 1994. These infants were cared for by 14 different categorical pediatric housestaff.", "Our neonatal ICU has either a resident or an intern on-call by himself/herself at night, affording us a natural setting to compare intern vs. resident test ordering. The outcomes of interest were number of arterial blood gases, radiographs, and electrolytes ordered per infant by the on-call pediatric houseofficer, as tabulated the morning after the call night. Control variables included the severity-of-illness of the individual infant (using the Neonatal Therapeutic Intervention Scoring System), the workload of the houseofficer (number of patients, number of admissions), and supervision (rounding frequency and on-call attending). Controlling for the severity-of-illness of the infant, the workload on the call night, and supervision with multiple linear regression, we found that interns ordered significantly (p = .02) greater numbers of arterial blood gases per infant than residents, amounting to some 0.33 blood gases per infant per call night (3.22 vs. 2.89 arterial blood gases per infant per night). This increase of 0.33 blood gases per infant amounts to interns ordering $169 more arterial blood gases per call night at our institution. There was no difference between interns and residents in ordering radiographs or electrolytes." ], "LABELS": [ "OBJECTIVE", "DESIGN", "SETTING", "PATIENTS", "MEASUREMENTS AND MAIN RESULTS" ], "MESHES": [ "Blood Gas Analysis", "Clinical Competence", "Diagnostic Tests, Routine", "Hospitals, University", "Humans", "Infant, Newborn", "Intensive Care Units, Neonatal", "Internship and Residency", "Kentucky", "Pediatrics", "Practice Patterns, Physicians'", "Prospective Studies", "Severity of Illness Index", "Workload" ], "YEAR": "1997", "reasoning_required_pred": "no", "reasoning_free_pred": "yes", "final_decision": "yes", "LONG_ANSWER": "Interns order significantly more arterial blood gases per infant than junior and senior residents on-call in the neonatal ICU. Additional study is required to see if the experience of housestaff is associated with a broader array of neonatal outcomes, such as morbidity and mortality." }, "18399830": { "QUESTION": "Is robotically assisted laparoscopic radical prostatectomy less invasive than retropubic radical prostatectomy?", "CONTEXTS": [ "To evaluate whether robotically assisted laparoscopic prostatectomy (RALP) is less invasive than radical retropubic prostatectomy (RRP), as experimental studies suggest that the acute phase reaction is proportional to surgery-induced tissue damage.", "Between May and November 2006, all patients undergoing RRP or RALP in our department were prospectively assessed. Blood samples were collected 24 h before (T0), during surgery (T1), at the end of anaesthesia (T2), and 12 (T3) and 24 h after surgery (T4), and assayed for interleukin(IL)-6 and IL-1 alpha, C-reactive protein (CRP), and lactate. The Mann-Whitney U-, Student's t- and Friedman tests were used to compare continuous variables, and the Pearson chi-square and Fisher test for categorical variables, with a two-sided P<0.05 considered to indicate significance.", "In all, 35 and 26 patients were assessed for RALP and RRP, respectively; the median (interquartile range) age was 62 (56-68) and 68.5 (59.2-71.2) years, respectively (P<0.009). Baseline levels (T0) of IL-1, IL-6, CRP and lactate were comparable in both arms. IL-6, CRP and lactates levels increased during both kinds of surgery. The mean IL-6 and CPR values were higher for RRP at T1 (P = 0.01 and 0.001), T2 (P = 0.001 and<0.001), T3 (P = 0.002 and<0.001) and T4 (P<0.001 and 0.02), respectively. Lactate was higher for RRP at T2 (P = 0.001), T3 (P = 0.001) and T4 (P = 0.004), although remaining within the normal ranges. IL-1 alpha did not change at the different sample times." ], "LABELS": [ "OBJECTIVES", "PATIENTS AND METHODS", "RESULTS" ], "MESHES": [ "Acute-Phase Reaction", "Aged", "C-Reactive Protein", "Humans", "Interleukin-1alpha", "Interleukin-6", "Lactic Acid", "Laparoscopy", "Male", "Middle Aged", "Postoperative Complications", "Prospective Studies", "Prostatectomy", "Prostatic Neoplasms", "Robotics", "Statistics, Nonparametric", "Treatment Outcome" ], "YEAR": "2008", "reasoning_required_pred": "no", "reasoning_free_pred": "yes", "final_decision": "yes", "LONG_ANSWER": "This study showed for the first time that RALP induces lower tissue trauma than RRP." }, "18575014": { "QUESTION": "Nasal polyposis: is there an inheritance pattern?", "CONTEXTS": [ "Nasal Polyposis (NP) is defined as a chronic inflammatory disease of sinonasal mucosa leading to diffuse formation of benign polyps. Although family histories are frequently suggested in medical literature, no specific study focused on this point has been reported. The purpose of this study is to determine whether a hereditary factor could be implied for NP in a family where several members were affected. We included 99 members of this family.", "All patients were assessed for conditions known to be associated with the development or presence of NP. Concerning NP, patients were screened with a validated questionnaire and selected patients had a medical examination by an Ear, Nose and Throat practitioner.", "Thirteen patients had a personal history of NP without asthma, aspirin intolerance, Churg Strauss syndrome, cystic fibrosis, Young's syndrome, bare lymphocyte syndrome, or primary ciliary dyskinesia. Within this family, 19.7% of those older than 17 years were affected by NP, as compared with the national French prevalence of 2.1%." ], "LABELS": [ "BACKGROUND", "METHODS", "RESULTS" ], "MESHES": [ "Adolescent", "Adult", "Child", "Consanguinity", "Female", "France", "Genetic Linkage", "Humans", "Inheritance Patterns", "Male", "Middle Aged", "Nasal Polyps", "Paranasal Sinus Diseases", "Pedigree" ], "YEAR": "2008", "reasoning_required_pred": "yes", "reasoning_free_pred": "yes", "final_decision": "yes", "LONG_ANSWER": "Regarding the pedigree, we discuss different modes of inheritance. The presence of consanguineous unions in this family suggests the possibility of a common ancestor and thus a recessive autosomal mode of inheritance." }, "15223779": { "QUESTION": "c-Kit-dependent growth of uveal melanoma cells: a potential therapeutic target?", "CONTEXTS": [ "This study was conducted to investigate the expression and functional impact of the proto-oncogene c-kit in uveal melanoma.", "Based on immunohistochemical (IHC) study of paraffin-embedded specimens from 134 uveal melanomas and Western blot analysis on eight fresh-frozen samples the expression of c-kit in uveal melanoma was studied. Furthermore, the phosphorylation of c-kit and the impact of the tyrosine kinase inhibitor STI571 was examined in the three uveal melanoma cell lines OCM-1, OCM-3, and 92-1.", "Eighty-four of 134 paraffin-embedded samples and six of eight fresh-frozen samples expressed c-kit. c-Kit was strongly expressed and tyrosine phosphorylated in cultured uveal melanoma cells compared with cutaneous melanoma cells. Moreover, in contrast to cutaneous melanoma cell lines c-kit maintained a high phosphorylation level in serum-depleted uveal melanoma cells. No activation-related mutations in exon 11 of the KIT gene were found. On the contrary, expression of the stem cell growth factor (c-kit ligand) was detected in all three uveal melanoma cell lines, suggesting the presence of autocrine (paracrine) stimulation pathways. Treatment of uveal melanoma cell lines with STI571, which blocks c-kit autophosphorylation, resulted in cell death. The IC(50) of the inhibitory effects on c-kit phosphorylation and cell proliferation was of equal size and less than 2.5 microM." ], "LABELS": [ "PURPOSE", "METHODS", "RESULTS" ], "MESHES": [ "Adult", "Aged", "Aged, 80 and over", "Benzamides", "Blotting, Western", "Cell Division", "Female", "Humans", "Imatinib Mesylate", "Immunoenzyme Techniques", "Male", "Melanoma", "Middle Aged", "Paraffin Embedding", "Phosphorylation", "Piperazines", "Polymerase Chain Reaction", "Proto-Oncogene Proteins c-kit", "Pyrimidines", "RNA, Messenger", "Skin Neoplasms", "Tumor Cells, Cultured", "Tyrosine", "Uveal Neoplasms" ], "YEAR": "2004", "reasoning_required_pred": "yes", "reasoning_free_pred": "yes", "final_decision": "yes", "LONG_ANSWER": "The results confirm that c-kit is vastly expressed in uveal melanoma, suggest that the c-kit molecular pathway may be important in uveal melanoma growth, and point to its use as a target for therapy with STI571." }, "24487044": { "QUESTION": "Pharmacologic regimens for knee osteoarthritis prevention: can they be cost-effective?", "CONTEXTS": [ "We sought to determine the target populations and drug efficacy, toxicity, cost, and initiation age thresholds under which a pharmacologic regimen for knee osteoarthritis (OA) prevention could be cost-effective.", "We used the Osteoarthritis Policy (OAPol) Model, a validated state-transition simulation model of knee OA, to evaluate the cost-effectiveness of using disease-modifying OA drugs (DMOADs) as prophylaxis for the disease. We assessed four cohorts at varying risk for developing OA: (1) no risk factors, (2) obese, (3) history of knee injury, and (4) high-risk (obese with history of knee injury). The base case DMOAD was initiated at age 50 with 40% efficacy in the first year, 5% failure per subsequent year, 0.22% major toxicity, and annual cost of $1,000. Outcomes included costs, quality-adjusted life expectancy (QALE), and incremental cost-effectiveness ratios (ICERs). Key parameters were varied in sensitivity analyses.", "For the high-risk cohort, base case prophylaxis increased quality-adjusted life-years (QALYs) by 0.04 and lifetime costs by $4,600, and produced an ICER of $118,000 per QALY gained. ICERs>$150,000/QALY were observed when comparing the base case DMOAD to the standard of care in the knee injury only cohort; for the obese only and no risk factors cohorts, the base case DMOAD was less cost-effective than the standard of care. Regimens priced at $3,000 per year and higher demonstrated ICERs above cost-effectiveness thresholds consistent with current US standards." ], "LABELS": [ "OBJECTIVE", "DESIGN", "RESULTS" ], "MESHES": [ "Adult", "Age Factors", "Aged", "Aged, 80 and over", "Computer Simulation", "Cost-Benefit Analysis", "Female", "Humans", "Knee Injuries", "Male", "Middle Aged", "Obesity", "Osteoarthritis, Knee", "Quality-Adjusted Life Years", "Risk Factors", "Treatment Outcome" ], "YEAR": "2014", "reasoning_required_pred": "no", "reasoning_free_pred": "yes", "final_decision": "yes", "LONG_ANSWER": "The cost-effectiveness of DMOADs for OA prevention for persons at high risk for incident OA may be comparable to other accepted preventive therapies." }, "12377809": { "QUESTION": "Is anorectal endosonography valuable in dyschesia?", "CONTEXTS": [ "Dyschesia can be provoked by inappropriate defecation movements. The aim of this prospective study was to demonstrate dysfunction of the anal sphincter and/or the musculus (m.) puborectalis in patients with dyschesia using anorectal endosonography.", "Twenty consecutive patients with a medical history of dyschesia and a control group of 20 healthy subjects underwent linear anorectal endosonography (Toshiba models IUV 5060 and PVL-625 RT). In both groups, the dimensions of the anal sphincter and the m. puborectalis were measured at rest, and during voluntary squeezing and straining. Statistical analysis was performed within and between the two groups.", "The anal sphincter became paradoxically shorter and/or thicker during straining (versus the resting state) in 85% of patients but in only 35% of control subjects. Changes in sphincter length were statistically significantly different (p<0.01, chi(2) test) in patients compared with control subjects. The m. puborectalis became paradoxically shorter and/or thicker during straining in 80% of patients but in only 30% of controls. Both the changes in length and thickness of the m. puborectalis were significantly different (p<0.01, chi(2) test) in patients versus control subjects." ], "LABELS": [ "AIMS", "METHODS", "RESULTS" ], "MESHES": [ "Adolescent", "Adult", "Aged", "Aged, 80 and over", "Anal Canal", "Case-Control Studies", "Chi-Square Distribution", "Constipation", "Defecation", "Endosonography", "Female", "Humans", "Male", "Middle Aged", "Pelvic Floor", "Rectum" ], "YEAR": "2002", "reasoning_required_pred": "yes", "reasoning_free_pred": "yes", "final_decision": "yes", "LONG_ANSWER": "Linear anorectal endosonography demonstrated incomplete or even absent relaxation of the anal sphincter and the m. puborectalis during a defecation movement in the majority of our patients with dyschesia. This study highlights the value of this elegant ultrasonographic technique in the diagnosis of \"pelvic floor dyssynergia\" or \"anismus\"." }, "16195477": { "QUESTION": "Production of chemokines by perivascular adipose tissue: a role in the pathogenesis of atherosclerosis?", "CONTEXTS": [ "Obesity is associated with an increased risk for cardiovascular disease. Although it is known that white adipose tissue (WAT) produces numerous proinflammatory and proatherogenic cytokines and chemokines, it is unclear whether adipose-derived chemotactic signals affect the chronic inflammation in atherosclerosis.", "Histological examination showed that perivascular WAT (pWAT) is in close proximity to vascular walls, particularly at sites that have a tendency to develop atherosclerosis. In rodents, the amount of pWAT is markedly increased by a high-fat diet. At a functional level, supernatant from subcutaneous and pWAT strongly induced the chemotaxis of peripheral blood leukocytes. The migration of granulocytes and monocytes was mostly mediated by interleukin-8 and monocyte chemoattractant protein-1, respectively, whereas both chemokines contributed to the migration of activated T cells. Moreover, pWAT produces these chemokines, as shown by immunohistochemistry and by explant culture. The accumulation of macrophages and T cells at the interface between pWAT and the adventitia of human atherosclerotic aortas may reflect this prochemotactic activity of pWAT." ], "LABELS": [ "OBJECTIVE", "METHODS AND RESULTS" ], "MESHES": [ "Adipose Tissue", "Animals", "Aorta", "Atherosclerosis", "Cells, Cultured", "Chemokine CCL2", "Chemotaxis, Leukocyte", "Diet, Atherogenic", "Dietary Fats", "Granulocytes", "Humans", "Interleukin-8", "Monocytes", "Obesity", "Rats", "Rats, Wistar" ], "YEAR": "2005", "reasoning_required_pred": "yes", "reasoning_free_pred": "yes", "final_decision": "yes", "LONG_ANSWER": "Human pWAT has chemotactic properties through the secretion of different chemokines, and we propose that pWAT might contribute to the progression of obesity-associated atherosclerosis." }, "16678696": { "QUESTION": "Continuity of care experience of residents in an academic vascular department: are trainees learning complete surgical care?", "CONTEXTS": [ "It is widely accepted that exemplary surgical care involves a surgeon's involvement in the preoperative, perioperative, and postoperative periods. In an era of ever-expanding therapeutic modalities available to the vascular surgeon, it is important that trainees gain experience in preoperative decision-making and how this affects a patient's operative and postoperative course. The purpose of this study was to define the current experience of residents on a vascular surgery service regarding the continuity of care they are able to provide for patients and the factors affecting this experience.", "This prospective cohort study was approved by the Institutional Review Board and conducted at the University of British Columbia during January 2005. All patients who underwent a vascular procedure at either of the two teaching hospitals were included. In addition to type of case (emergent, outpatient, inpatient), resident demographic data and involvement in each patient's care (preoperative assessment, postoperative daily assessment, and follow-up clinic assessment) were recorded. Categoric data were analyzed with the chi2 test.", "The study included 159 cases, of which 65% were elective same-day admission patients, 20% were elective previously admitted patients; and 15% were emergent. The overall rate of preoperative assessment was 67%, involvement in the decision to operate, 17%; postoperative assessment on the ward, 79%; and patient follow-up in clinic, 3%. The rate of complete in-hospital continuity of care (assessing patient pre-op and post-op) was 57%. Emergent cases were associated with a significantly higher rate of preoperative assessment (92% vs 63%, P<.05). For elective cases admitted before the day of surgery compared with same-day admission patients, the rates of preoperative assessment (78% vs 58%, P<.05) and involvement in the decision to operate (16% vs 4%, P<.05) were significantly higher." ], "LABELS": [ "BACKGROUND", "METHODS", "RESULTS" ], "MESHES": [ "Ambulatory Surgical Procedures", "British Columbia", "Cohort Studies", "Continuity of Patient Care", "Curriculum", "Decision Making", "Emergencies", "Forecasting", "Hospitals, Teaching", "Humans", "Internship and Residency", "Postoperative Care", "Preoperative Care", "Prospective Studies", "Surgery Department, Hospital", "Vascular Surgical Procedures" ], "YEAR": "2006", "reasoning_required_pred": "yes", "reasoning_free_pred": "no", "final_decision": "no", "LONG_ANSWER": "The continuity-of-care experiences of vascular trainees are suboptimal. This is especially true for postoperative clinic assessment. Same-day admission surgery accounted for most of the cases and was associated with the poorest continuity of care. To provide complete surgical training in an era of changing therapeutic modalities and same-day admission surgery, vascular programs must be creative in structuring training to include adequate ambulatory experience." }, "18801797": { "QUESTION": "Household and food shopping environments: do they play a role in socioeconomic inequalities in fruit and vegetable consumption?", "CONTEXTS": [ "Fruit and vegetables are protective of a number of chronic diseases; however, their intakes have been shown to vary by socioeconomic position (SEP). Household and food shopping environmental factors are thought to contribute to these differences. To determine whether household and food shopping environmental factors are associated with fruit and vegetable (FV) intakes, and contribute to socioeconomic inequalities in FV consumption.", "Cross-sectional data were obtained by a postal questionnaire among 4333 adults (23-85 years) living in 168 neighbourhoods in the south-eastern Netherlands. Participants agreed/disagreed with a number of statements about the characteristics of their household and food shopping environments, including access, prices and quality. Education was used to characterise socioeconomic position (SEP). Main outcome measures were whether or not participants consumed fruit or vegetables on a daily basis. Multilevel logistic regression models examined between-area variance in FV consumption and associations between characteristics of the household and food shopping environments and FV consumption.", "Only a few household and food shopping environmental factors were significantly associated with fruit and vegetable consumption, and their prevalence was low. Participants who perceived FV to be expensive were more likely to consume them. There were significant socioeconomic inequalities in fruit and vegetable consumption (ORs of not consuming fruit and vegetables were 4.26 and 5.47 among the lowest-educated groups for fruit and vegetables, respectively); however, these were not explained by any household or food shopping environmental factors." ], "LABELS": [ "BACKGROUND", "METHODS", "RESULTS" ], "MESHES": [ "Adult", "Age Factors", "Aged", "Aged, 80 and over", "Commerce", "Educational Status", "Environment", "Epidemiologic Methods", "Feeding Behavior", "Food Supply", "Fruit", "Humans", "Middle Aged", "Netherlands", "Sex Factors", "Social Class", "Vegetables", "Young Adult" ], "YEAR": "2009", "reasoning_required_pred": "no", "reasoning_free_pred": "no", "final_decision": "no", "LONG_ANSWER": "Improving access to FV in the household and food shopping environments will only make a small contribution to improving population consumption levels, and may only have a limited effect in reducing socioeconomic inequalities in their consumption." }, "12153648": { "QUESTION": "Does birth center care during a woman's first pregnancy have any impact on her future reproduction?", "CONTEXTS": [ "Women's experiences of childbirth may affect their future reproduction, and the model of care affects their experiences, suggesting that a causal link may exist between model of care and future reproduction. The study objective was to examine whether the birth center model of care during a woman's first pregnancy affects whether or not she has a second baby, and on the spacing to the next birth.", "Between October 1989 and July 1993, a total of 1860 women at low medical risk in early pregnancy, who participated in a randomized controlled trial of in-hospital birth center care versus standard care, gave birth. The 1063 primiparas in the trial, 543 in the birth center group and 520 in the standard care group, were included in a secondary analysis in which women's personal identification codes were linked to the Swedish National Birth Register, which included information about their subsequent birth during the following 7 to 10 years. Time to an event curves were constructed by means of the Kaplan Meier method.", "The observation period after the first birth was on average 8.8 years in the birth center group and 8.7 years in the standard care group. No statistical difference was found between the groups in time to second birth, which was 2.85 and 2.82 years, respectively (median; log-rank 1.26; p=0.26)." ], "LABELS": [ "BACKGROUND", "METHODS", "RESULTS" ], "MESHES": [ "Adult", "Birth Intervals", "Birthing Centers", "Continuity of Patient Care", "Delivery Rooms", "Female", "Humans", "Maternal Health Services", "Models, Nursing", "Nurse Midwives", "Parity", "Patient Care Team", "Patient Satisfaction", "Pregnancy", "Registries", "Reproduction", "Surveys and Questionnaires", "Sweden" ], "YEAR": "2002", "reasoning_required_pred": "no", "reasoning_free_pred": "no", "final_decision": "no", "LONG_ANSWER": "A woman's model of care, such as birth center care, during her first pregnancy does not seem to be a sufficiently important factor to affect subsequent reproduction in Sweden." }, "12632437": { "QUESTION": "Are environmental factors important in primary systemic vasculitis?", "CONTEXTS": [ "To investigate the association between primary systemic vasculitis (PSV) and environmental risk factors.", "Seventy-five PSV cases and 273 controls (220 nonvasculitis, 19 secondary vasculitis, and 34 asthma controls) were interviewed using a structured questionnaire. Factors investigated were social class, occupational and residential history, smoking, pets, allergies, vaccinations, medications, hepatitis, tuberculosis, and farm exposure in the year before symptom onset (index year). The Standard Occupational Classification 2000 and job-exposure matrices were used to assess occupational silica, solvent, and metal exposure. Stepwise multiple logistic regression was used to calculate the odds ratio (OR) and 95% confidence interval (95% CI) adjusted for potential confounders. Total PSV, subgroups (47 Wegener's granulomatosis [WG], 12 microscopic polyangiitis, 16 Churg-Strauss syndrome [CSS]), and antineutrophil cytoplasmic antibody (ANCA)-positive cases were compared with control groups.", "Farming in the index year was significantly associated with PSV (OR 2.3 [95% CI 1.2-4.6]), with WG (2.7 [1.2-5.8]), with MPA (6.3 [1.9-21.6]), and with perinuclear ANCA (pANCA) (4.3 [1.5-12.7]). Farming during working lifetime was associated with PSV (2.2 [1.2-3.8]) and with WG (2.7 [1.3-5.7]). Significant associations were found for high occupational silica exposure in the index year (with PSV 3.0 [1.0-8.4], with CSS 5.6 [1.3-23.5], and with ANCA 4.9 [1.3-18.6]), high occupational solvent exposure in the index year (with PSV 3.4 [0.9-12.5], with WG 4.8 [1.2-19.8], and with classic ANCA [cANCA] 3.9 [1.6-9.5]), high occupational solvent exposure during working lifetime (with PSV 2.7 [1.1-6.6], with WG 3.4 [1.3-8.9], and with cANCA 3.3 [1.0-10.8]), drug allergy (with PSV 3.6 [1.8-7.0], with WG 4.0 [1.8-8.7], and with cANCA 4.7 [1.9-11.7]), and allergy overall (with PSV 2.2 [1.2-3.9], with WG 2.7 [1.4-5.7]). No other significant associations were found." ], "LABELS": [ "OBJECTIVE", "METHODS", "RESULTS" ], "MESHES": [ "Adolescent", "Adult", "Aged", "Aged, 80 and over", "Agriculture", "Case-Control Studies", "Female", "Hazardous Substances", "Hospitals, University", "Humans", "Male", "Middle Aged", "Occupational Exposure", "Risk Factors", "Surveys and Questionnaires", "United Kingdom", "Vasculitis" ], "YEAR": "2003", "reasoning_required_pred": "yes", "reasoning_free_pred": "yes", "final_decision": "yes", "LONG_ANSWER": "A significant association between farming and PSV has been identified for the first time. Results also support previously reported associations with silica, solvents, and allergy." }, "10201555": { "QUESTION": "Is low serum chloride level a risk factor for cardiovascular mortality?", "CONTEXTS": [ "Serum chloride level is routinely assayed in clinical laboratories in the management of patients with kidney disorders and with metabolic diseases. It is a biological parameter that is easily, precisely and relatively cheaply measured. The epidemiological features of serum chloride levels have not been studied before.", "For the random sample of men and women from the Belgian Interuniversity Research on Nutrition and Health aged 25-74 years, free of symptomatic coronary heart disease at baseline, serum chloride concentrations were measured, among those of other electrolytes. The cohort was followed up for 10 years with respect to subsequent cause-specific mortality.", "The results are based on observations of 4793 men and 4313 women. According to Cox regression analysis serum chloride level was one of the strongest predictors of total, cardiovascular disease (CVD) and non-CVD mortalities independently of age, body mass index, sex, smoking, systolic blood pressure, levels of total and high-density lipoprotein cholesterol, uric acid, serum creatinine and serum total proteins and intake of diuretics. This relation was proved to be independent of levels of other serum electrolytes and similar for men and women. The estimated adjusted risk ratio for CVD death for subjects with a serum chloride level500 mg/d) in adult patients with pharmacoresistant epilepsy.", "Between 2006 and 2013, all epileptic outpatients treated with high doses of ZNS were selected. Safety and efficacy were assessed based on patient and caregiver reports. Serum levels of ZNS and other concomitant antiepileptic drugs were evaluated if available.", "Nine patients (5 female): 8 focal/1 generalized pharmacoresistant epilepsy. Mean age: 34 years. Most frequent seizure type: complex partial seizures; other seizure types: generalized tonic-clonic, tonic, myoclonia. Zonisamide in polytherapy in all (100%), administered in tritherapy in 3 (33%) of 9 patients; mean dose: 633 (600-700) mg/d; efficacy (>50% seizure reduction) was observed in 5 (55%) of 9 patients. Five of 9 patients are still taking high doses of ZNS (more than 1 year). Adverse events were observed in 3 (37%) of 8 patients. Good tolerance to high doses of other antiepileptic drugs had been observed in 6 (66%) of 9 patients. Plasma levels of ZNS were only available in 2 patients; both were in the therapeutic range (34.95, 30.91) (10-40 mg/L)." ], "LABELS": [ "OBJECTIVE", "METHODS", "RESULTS" ], "MESHES": [ "Adult", "Anticonvulsants", "Dose-Response Relationship, Drug", "Drug Resistant Epilepsy", "Europe", "Female", "Humans", "Isoxazoles", "Male", "Middle Aged", "Retrospective Studies", "Treatment Outcome", "Young Adult" ], "YEAR": null, "reasoning_required_pred": "yes", "reasoning_free_pred": "yes", "final_decision": "yes", "LONG_ANSWER": "High doses of ZNS are effective and safe in pharmacoresistant epileptic patients. Therapeutic drug monitoring of ZNS may be considered at therapeutic failure." }, "18005142": { "QUESTION": "Does early migraine treatment shorten time to headache peak and reduce its severity?", "CONTEXTS": [ "To explore whether early treatment would shorten the duration of headache from headache onset to its peak and reduce headache severity at peak.", "Prior clinical studies almost exclusively focused on headache relief after dosing. No data are available on whether early intervention affects the duration from headache onset to peak and headache severity at peak.", "Adult migraineurs were enrolled in this observational study from multi-site headache clinics. Patients recorded their migraine experiences via an electronic diary over 1 month. Patients reported the time and pain severity at onset, dosing, and peak. We used a linear mixed model to evaluate the impact of the timing of treatment and to adjust for covariates and correlation of observations within subjects.", "A total of 182 patients reported 970 migraine episodes, 620 of which were treated before headaches progressed to peak. Mean time from headache onset to peak varied from 1.9 hours to 8.9 hours for patients treated within 15 minutes of onset and those who waited for 4 or more hours, respectively. However, early intervention was not associated with reduced headache severity at peak. In multivariate analysis, early treatment, use of triptans, and mild migraine headache in the past 3 months were significantly associated with shorter time from onset to headache peak. A separate model indicated that the timing of medication was not associated with the duration between dosing and headache peak, but use of triptans shortened the time from dosing to headache peak." ], "LABELS": [ "OBJECTIVE", "BACKGROUND", "METHODS", "RESULTS" ], "MESHES": [ "Adult", "Dose-Response Relationship, Drug", "Drug Administration Schedule", "Female", "Humans", "Male", "Middle Aged", "Migraine Disorders", "Multivariate Analysis", "Severity of Illness Index", "Time Factors", "Treatment Outcome", "Tryptamines" ], "YEAR": "2008", "reasoning_required_pred": "yes", "reasoning_free_pred": "yes", "final_decision": "yes", "LONG_ANSWER": "Early treatment and use of triptans may lead to shorter duration from migraine headache onset to its peak but did not alleviate headache severity at peak. This could result in decreased migraine burden by reducing total migraine headache duration." }, "11481172": { "QUESTION": "Does the manic/mixed episode distinction in bipolar disorder patients run true over time?", "CONTEXTS": [ "The authors sought to determine whether the manic/mixed episode distinction in patients with bipolar disorder runs true over time.", "Over an 11-year period, the observed distribution of manic and mixed episodes (N=1,224) for patients with three or more entries in the management information system of a community mental health center (N=241) was compared to the expected distribution determined by averaging 1,000 randomly generated simulations.", "Episodes were consistent (all manic or all mixed) in significantly more patients than would be expected by chance." ], "LABELS": [ "OBJECTIVE", "METHOD", "RESULTS" ], "MESHES": [ "Bipolar Disorder", "Community Mental Health Centers", "Computer Simulation", "Depressive Disorder", "Humans", "Management Information Systems", "Probability", "Random Allocation", "Retrospective Studies", "Statistical Distributions" ], "YEAR": "2001", "reasoning_required_pred": "no", "reasoning_free_pred": "yes", "final_decision": "yes", "LONG_ANSWER": "These data suggest a pattern of diagnostic stability over time for manic and mixed episodes in patients with bipolar disorder. Careful prospective studies of this issue are needed." }, "19230985": { "QUESTION": "Post-tonsillectomy late haemorrhage: is it a preferably night-time event?", "CONTEXTS": [ "This retrospective study was carried out in the Ear Nose Throat (ENT) Unit of Giannina Gaslini Institute, Genoa, Italy on children operated for adenotonsillectomy (AT) or tonsillectomy (T) between January 2003 and February 2008. We considered in the study all the post-tonsillectomy late haemorrhages irrespective of their severity and for each case we evaluated whether they recurred in the day-time (B) (between 9.00 a.m. and 9.00 p.m.) or in the night-time (A) (between 9.00 p.m. and 9.00 a.m.). Finally we considered the number of haemorrhages per hour in the whole day.", "Out of 3306 patients undergoing elective adenotonsillectomy or tonsillectomy, post-operative late haemorrhage occurred in 59 (1.78%). We noted that 42 episodes (71.2%) occurred in the night-time and 17 (28.8%) in the day-time. The average time from the operation was 8.4 days. A statistically significant difference (p=0.002) was found when comparing the frequencies of night-time and day-time haemorrhages. We did not observe any significant difference in the distribution per hour of the haemorrhages." ], "LABELS": [ "STUDY DESIGN AND SETTING", "RESULTS" ], "MESHES": [ "Adolescent", "Child", "Child, Preschool", "Circadian Rhythm", "Female", "Humans", "Male", "Postoperative Hemorrhage", "Prevalence", "Retrospective Studies", "Tonsillectomy" ], "YEAR": "2009", "reasoning_required_pred": "yes", "reasoning_free_pred": "yes", "final_decision": "yes", "LONG_ANSWER": "The incidence of post-tonsillectomy late haemorrhage in our study population was 1.78%. A statistically significant difference was found between night-time and day-time haemorrhages. Even though no significant distribution of haemorrhages per hour was observed, we underline that we recorded 32 (54.2%) events in 2 periods of the day: from 10 p.m. to 1 a.m. and from 6 to 9 a.m." }, "17295865": { "QUESTION": "Acute hepatitis C in Israel: a predominantly iatrogenic disease?", "CONTEXTS": [ "Acute hepatitis C virus infection in the era of universal screening of blood products has not disappeared, and is thought to be transmitted primarily via injecting drug use. A growing body of evidence supports iatrogenic transmission as an important mode of transmission. The aim of this study was to examine transmission routes and clinical characteristics in a group of patients with acute hepatitis C in Israel.", "A retrospective chart review was conducted in three different liver clinics in Israel, of all new hepatitis C patients. Patients identified as possible acute hepatitis C were re-interviewed and all other sources such as blood bank records and pre-employment check-ups reviewed in order to establish the diagnosis of acute hepatitis C infection and to identify the transmission route.", "Twenty-nine patients were found to have acute hepatitis C, representing 0.75% of all new referrals for hepatitis C. The most frequent (65%) mode of transmission was iatrogenic involving several, often minimal, procedures and clinical settings. The group in which iatrogenic transmission was suspected was older and the patients more often in monogamous relationship compared with other transmission routes groups. Injecting drug use was the second most common route of infection. Spontaneous seroconversion has occurred in approximately one third of the patients." ], "LABELS": [ "BACKGROUND AND AIMS", "METHODS", "RESULTS" ], "MESHES": [ "Acute Disease", "Adult", "Aged", "Aged, 80 and over", "Female", "Hepatitis C", "Humans", "Iatrogenic Disease", "Male", "Middle Aged", "Retrospective Studies" ], "YEAR": "2007", "reasoning_required_pred": "yes", "reasoning_free_pred": "yes", "final_decision": "yes", "LONG_ANSWER": "Acute hepatitis C in the post universal blood products screening era was found to be predominantly an iatrogenic disease in the investigated localities. This finding should direct attention and resources towards the development and implementation of preventive measures." }, "17565137": { "QUESTION": "Out of the smokescreen II: will an advertisement targeting the tobacco industry affect young people's perception of smoking in movies and their intention to smoke?", "CONTEXTS": [ "To evaluate the effect of an antismoking advertisement on young people's perceptions of smoking in movies and their intention to smoke.SUBJECTS/", "3091 cinema patrons aged 12-24 years in three Australian states; 18.6% of the sample (n = 575) were current smokers.DESIGN/", "Quasi-experimental study of patrons, surveyed after having viewed a movie. The control group was surveyed in week 1, and the intervention group in weeks 2 and 3. Before seeing the movie in weeks 2 and 3, a 30 s antismoking advertisement was shown, shot in the style of a movie trailer that warned patrons not to be sucked in by the smoking in the movie they were about to see.", "Attitude of current smokers and non-smokers to smoking in the movies; intention of current smokers and non-smokers to smoke in 12 months.", "Among non-smokers, 47.8% of the intervention subjects thought that the smoking in the viewed movie was not OK compared with 43.8% of the control subjects (p = 0.04). However, there was no significant difference among smokers in the intervention (16.5%) and control (14.5%) groups (p = 0.4). A higher percentage of smokers in the intervention group indicated that they were likely to be smoking in 12 months time (38.6%) than smokers in the control group (25.6%; p<0.001). For non-smokers, there was no significant difference in smoking intentions between groups, with 1.2% of intervention subjects and 1.6% of controls saying that they would probably be smoking in 12 months time (p = 0.54)." ], "LABELS": [ "OBJECTIVE", "SETTING", "INTERVENTION", "OUTCOMES", "RESULTS" ], "MESHES": [ "Adolescent", "Advertising as Topic", "Attitude to Health", "Female", "Humans", "Male", "Motion Pictures", "Smoking", "Smoking Prevention" ], "YEAR": "2007", "reasoning_required_pred": "no", "reasoning_free_pred": "yes", "final_decision": "yes", "LONG_ANSWER": "This real-world study suggests that placing an antismoking advertisement before movies containing smoking scenes can help to immunise non-smokers against the influences of film stars' smoking. Caution must be exercised in the type of advertisement screened as some types of advertising may reinforce smokers' intentions to smoke." }, "18041059": { "QUESTION": "Do adjuvant aromatase inhibitors increase the cardiovascular risk in postmenopausal women with early breast cancer?", "CONTEXTS": [ "Despite the advantages from using aromatase inhibitors (AIs) compared with tamoxifen for early breast cancer, an unexpectedly greater number of grade 3 and 4 cardiovascular events (CVAE) (as defined by National Cancer Institute of Canada-Common Toxicity Criteria [version 2.0] was demonstrated.", "Phase 3 randomized clinical trials (RCTs) comparing AI with tamoxifen in early breast cancer were considered eligible for this review. The event-based risk ratios (RRs) with 95% confidence intervals (95% CIs) were derived, and a test of heterogeneity was applied. Finally, absolute differences (ADs) in event rates and the number of patients needed to harm 1 patient (NNH) were determined.", "Seven eligible RCTs (19,818 patients) reported CVAE results. When considering all RCTs, the AD of the primary endpoint (CVAE) between the 2 arms (0.52%), tamoxifen versus AI, was statistically significant (RR, 1.31; 95% CI, 1.07-1.60; P= .007). This translated into an NNH value of 189 patients; when only third-generation AIs were considered, the difference (0.57%) remained significant (RR, 1.34; 95% CI, 1.09-1.63; P= .0038). Thromboembolic events were significantly more frequent in the tamoxifen arm, regardless of the strategy adopted (RR, 0.53; 95% CI, 0.42-0.65; P<.0001), without significant heterogeneity (P= .21). An AD of 1.17% and an NNH value of 85 patients were observed." ], "LABELS": [ "BACKGROUND", "METHODS", "RESULTS" ], "MESHES": [ "Aromatase Inhibitors", "Breast Neoplasms", "Cardiovascular Diseases", "Female", "Humans", "Middle Aged", "Postmenopause", "Randomized Controlled Trials as Topic", "Tamoxifen" ], "YEAR": "2008", "reasoning_required_pred": "yes", "reasoning_free_pred": "yes", "final_decision": "yes", "LONG_ANSWER": "According to the results from this meta-analysis, the risk of grade 3 and 4 CVAEs in patients who were receiving AIs was higher compared with the risk in patients who were receiving tamoxifen, and the difference reached statistical significance. However, the AD was relatively low, and from 160 to 180 patients had to be treated to produce 1 event." }, "17274051": { "QUESTION": "Metastatic carcinoma to the cervical nodes from an unknown head and neck primary site: Is there a need for neck dissection?", "CONTEXTS": [ "The aim of the study was to evaluate the outcomes and patterns of failure in patients with metastatic carcinoma to cervical lymph nodes from an unknown head and neck primary origin, who were treated curatively with radiotherapy, with or without neck dissection.", "The study included 61 patients referred to the McGill University Hospital Centers from 1987 to 2002. The median age was 57 years, with male to female ratio of 4:1. Distribution of patients by N status was as follows: N1, 16 patients (26%); N2a, 18 (30%); N2b, 13 (22%); N2c, 7 (11%); and N3, 7 (11%). Twenty patients underwent neck dissection (11 radical, 9 functional) and 41 patients had biopsy (9 fine-needle aspiration and 32 excisional biopsy). All patients received radiotherapy. The median dose to the involved node(s) was 64 Gy, and 60 Gy to the rest of the neck. Treatment of the neck was bilateral in 50 patients (82%) and ipsilateral in 11 (18%). The minimum duration of the follow-up was 12 months, with the median of 32 months.", "The 5- and 8-year overall survival for the whole population was 79% and 67%, respectively. There was no statistically significant difference in the 8-year actuarial overall survival (64.8% and 67.6%, respectively, p = .64) and local relapse-free survival (75% vs 74.5%, respectively, p = .57), among patients who had biopsy versus those who had neck dissection." ], "LABELS": [ "BACKGROUND", "METHODS", "RESULTS" ], "MESHES": [ "Biopsy, Fine-Needle", "Carcinoma", "Female", "Head and Neck Neoplasms", "Humans", "Lymph Nodes", "Lymphatic Irradiation", "Lymphatic Metastasis", "Male", "Middle Aged", "Neck Dissection", "Neoplasms, Unknown Primary", "Radiotherapy Dosage", "Survival Analysis" ], "YEAR": "2007", "reasoning_required_pred": "no", "reasoning_free_pred": "no", "final_decision": "no", "LONG_ANSWER": "In our experience, definitive radiotherapy to the neck and the potential mucosal sites, whether preceded by neck dissection or not, is effective to achieve a good local control rate in the unknown primary cancer of the head and neck. The indication for neck dissection, in particular for early nodal stage, is controversial." }, "26460153": { "QUESTION": "Cardiac reoperations in octogenarians: Do they really benefit?", "CONTEXTS": [ "We retrospectively identified 84 consecutive patients aged \u226580 years, who underwent a cardiac reoperation at the department for Cardiothoracic Surgery in the Heart&Vessel Center Bad Bevensen between January 2007 and 2013. Demographic profiles as well as operative data were analyzed, and the patients were prospectively followed. Patient's functional status and quality of life were assessed with the Barthel Index, New York Heart Association class and the short form-12 questionnaire.", "The mean age of the study group (61 men, 23 women) was 81.9\u2009\u00b1\u20091.9 years. Most redo-procedures were carried out after primary coronary artery bypass grafting (65%), primary aortic valve replacement (21%) and primary mitral valve replacement (6%). The most frequent actual surgical procedures were combined coronary artery bypass grafting and aortic valve replacement (26%), isolated coronary artery bypass grafting (19%), and isolated aortic valve replacement (19%). The mean length of hospital stay was 17\u2009\u00b1\u200915 days. In-hospital mortality counted for 32.1%. During follow up (29\u2009\u00b1\u200920 months) a further 19.0% of the patients died. The Barthel Index of the survivors was 89\u2009\u00b1\u200917 and their mean New York Heart Association class was 2\u2009\u00b1\u20091. A total of 93% of the patients were living at home. Summary scores of physical and mental health of the short form-12 questionnaire equalled those of an age- and sex-matched normative population." ], "LABELS": [ "METHODS", "RESULTS" ], "MESHES": [ "Age Factors", "Aged, 80 and over", "Cardiovascular Surgical Procedures", "Cause of Death", "Cohort Studies", "Female", "Frail Elderly", "Geriatric Assessment", "Hospital Mortality", "Humans", "Kaplan-Meier Estimate", "Male", "Prognosis", "Quality of Life", "Reoperation", "Retrospective Studies", "Risk Assessment", "Sex Factors", "Survival Analysis", "Treatment Outcome" ], "YEAR": "2016", "reasoning_required_pred": "yes", "reasoning_free_pred": "yes", "final_decision": "yes", "LONG_ANSWER": "Despite high perioperative mortality, results document a sustainable recovery of the survivors offering the prospect of a highly independent and satisfying life. Therefore, advanced age alone should not be a contraindication for redo cardiac interventions. Geriatr Gerontol Int 2016; 16: 1138-1144." }, "24061619": { "QUESTION": "Location and number of sutures placed for hiatal hernia repair during laparoscopic adjustable gastric banding: does it matter?", "CONTEXTS": [ "It has been demonstrated that hiatal hernia repair (HHR) during laparoscopic adjustable gastric banding (LAGB) decreases the rate of reoperation. However, the technical aspects (location and number of sutures) are not standardized. It is unknown whether such technical details are associated with differing rates of reoperation for band-related problems.", "A retrospective analysis was performed from a single institution, including 2,301 patients undergoing LAGB with HHR from July 1, 2007 to December 31, 2011. Independent variables were number and location of sutures. Data collected included demographics, operating room (OR) time, length of stay (LOS), follow-up time, postoperative BMI/%EWL, and rates of readmission/reoperation. Statistical analyses included ANOVA and Chi squared tests. Kaplan-Meier, log-rank, and Cox regression tests were used for follow-up data and reoperation rates, in order to account for differential length of follow-up and confounding variables.", "There was no difference in length of follow-up among all groups. The majority of patients had one suture (range 1-6; 55 %). Patients with fewer sutures had shorter OR time (1 suture 45 min vs. 4+ sutures 56 min, p<0.0001). LOS, 30-day readmission, band-related reoperation, and postop BMI/%EWL were not statistically significant. Anterior suture placement (vs. posterior vs. both) was most common (61 %). OR time was shorter in those with anterior suture (41 min vs. posterior 56 min vs. both 59 min, p<0.0001). Patients with posterior suture had a longer LOS (84 % 1 day vs. anterior 74 % 1 day vs. both 74 % 1 day, p<0.0001). There was no difference in 30-day readmission, band-related reoperation, and postoperative BMI/%EWL." ], "LABELS": [ "BACKGROUND", "METHODS", "RESULTS" ], "MESHES": [ "Adult", "Aged", "Analysis of Variance", "Female", "Follow-Up Studies", "Gastroplasty", "Hernia, Hiatal", "Herniorrhaphy", "Humans", "Laparoscopy", "Length of Stay", "Male", "Middle Aged", "Obesity, Morbid", "Postoperative Period", "Reoperation", "Retrospective Studies", "Suture Techniques", "Sutures" ], "YEAR": "2014", "reasoning_required_pred": "no", "reasoning_free_pred": "maybe", "final_decision": "no", "LONG_ANSWER": "Patients with fewer or anterior sutures have shorter OR times. However, 30-day readmission, band-related reoperation, and postoperative weight loss were unaffected by number or location of suture. The technical aspects of HHR did not appear to be associated with readmission or reoperation, and therefore a standardized approach may not be necessary." }, "26867834": { "QUESTION": "Is Alveolar Macrophage Phagocytic Dysfunction in Children With Protracted Bacterial Bronchitis a Forerunner to Bronchiectasis?", "CONTEXTS": [ "Children with recurrent protracted bacterial bronchitis (PBB) and bronchiectasis share common features, and PBB is likely a forerunner to bronchiectasis. Both diseases are associated with neutrophilic inflammation and frequent isolation of potentially pathogenic microorganisms, including nontypeable Haemophilus influenzae (NTHi), from the lower airway. Defective alveolar macrophage phagocytosis of apoptotic bronchial epithelial cells (efferocytosis), as found in other chronic lung diseases, may also contribute to tissue damage and neutrophil persistence. Thus, in children with bronchiectasis or PBB and in control subjects, we quantified the phagocytosis of airway apoptotic cells and NTHi by alveolar macrophages and related the phagocytic capacity to clinical and airway inflammation.", "Children with bronchiectasis (n = 55) or PBB (n = 13) and control subjects (n = 13) were recruited. Alveolar macrophage phagocytosis, efferocytosis, and expression of phagocytic scavenger receptors were assessed by flow cytometry. Bronchoalveolar lavage fluid interleukin (IL) 1\u03b2 was measured by enzyme-linked immunosorbent assay.", "For children with PBB or bronchiectasis, macrophage phagocytic capacity was significantly lower than for control subjects (P = .003 and P<.001 for efferocytosis and P = .041 and P = .004 for phagocytosis of NTHi; PBB and bronchiectasis, respectively); median phagocytosis of NTHi for the groups was as follows: bronchiectasis, 13.7% (interquartile range [IQR], 11%-16%); PBB, 16% (IQR, 11%-16%); control subjects, 19.0% (IQR, 13%-21%); and median efferocytosis for the groups was as follows: bronchiectasis, 14.1% (IQR, 10%-16%); PBB, 16.2% (IQR, 14%-17%); control subjects, 18.1% (IQR, 16%-21%). Mannose receptor expression was significantly reduced in the bronchiectasis group (P = .019), and IL-1\u03b2 increased in both bronchiectasis and PBB groups vs control subjects." ], "LABELS": [ "BACKGROUND", "METHODS", "RESULTS" ], "MESHES": [ "Apoptosis", "Bacterial Infections", "Bronchiectasis", "Bronchitis", "Bronchoalveolar Lavage Fluid", "Cell Line", "Child, Preschool", "Enzyme-Linked Immunosorbent Assay", "Female", "Humans", "Infant", "Macrophages, Alveolar", "Male", "Phagocytosis" ], "YEAR": "2016", "reasoning_required_pred": "yes", "reasoning_free_pred": "yes", "final_decision": "yes", "LONG_ANSWER": "A reduced alveolar macrophage phagocytic host response to apoptotic cells or NTHi may contribute to neutrophilic inflammation and NTHi colonization in both PBB and bronchiectasis. Whether this mechanism also contributes to the progression of PBB to bronchiectasis remains unknown." }, "22349635": { "QUESTION": "Do elderly patients benefit from surgery in addition to radiotherapy for treatment of metastatic spinal cord compression?", "CONTEXTS": [ "Treatment of elderly cancer patients has gained importance. One question regarding the treatment of metastatic spinal cord compression (MSCC) is whether elderly patients benefit from surgery in addition to radiotherapy? In attempting to answer this question, we performed a matched-pair analysis comparing surgery followed by radiotherapy to radiotherapy alone.", "Data from 42 elderly (age>\u200965 years) patients receiving surgery plus radiotherapy (S\u2009+\u2009RT) were matched to 84 patients (1:2) receiving radiotherapy alone (RT). Groups were matched for ten potential prognostic factors and compared regarding motor function, local control, and survival. Additional matched-pair analyses were performed for the subgroups of patients receiving direct decompressive surgery plus stabilization of involved vertebrae (DDSS, n\u2009=\u200981) and receiving laminectomy (LE, n\u2009=\u200945).", "Improvement of motor function occurred in 21% after S\u2009+\u2009RT and 24% after RT (p\u2009=\u20090.39). The 1-year local control rates were 81% and 91% (p\u2009=\u20090.44), while the 1-year survival rates were 46% and 39% (p\u2009=\u20090.71). In the matched-pair analysis of patients receiving DDSS, improvement of motor function occurred in 22% after DDSS\u2009+\u2009RT and 24% after RT alone (p\u2009=\u20090.92). The 1-year local control rates were 95% and 89% (p\u2009=\u20090.62), and the 1-year survival rates were 54% and 43% (p\u2009=\u20090.30). In the matched-pair analysis of patients receiving LE, improvement of motor function occurred in 20% after LE\u2009+\u2009RT and 23% after RT alone (p\u2009=\u20090.06). The 1-year local control rates were 50% and 92% (p\u2009=\u20090.33). The 1-year survival rates were 32% and 32% (p\u2009=\u20090.55)." ], "LABELS": [ "BACKGROUND", "PATIENTS AND METHODS", "RESULTS" ], "MESHES": [ "Aged", "Female", "Humans", "Male", "Matched-Pair Analysis", "Neoplasms", "Radiotherapy", "Spinal Cord Compression", "Survival Analysis", "Treatment Outcome" ], "YEAR": "2012", "reasoning_required_pred": "yes", "reasoning_free_pred": "no", "final_decision": "no", "LONG_ANSWER": "Elderly patients with MSCC did not benefit from surgery in addition to radiotherapy regarding functional outcome, local control of MSCC, or survival." }, "15112004": { "QUESTION": "Are WHO/UNAIDS/UNICEF-recommended replacement milks for infants of HIV-infected mothers appropriate in the South African context?", "CONTEXTS": [ "Little is known about the nutritional adequacy and feasibility of breastmilk replacement options recommended by WHO/UNAIDS/UNICEF. The study aim was to explore suitability of the 2001 feeding recommendations for infants of HIV-infected mothers for a rural region in KwaZulu Natal, South Africa specifically with respect to adequacy of micronutrients and essential fatty acids, cost, and preparation times of replacement milks.", "Nutritional adequacy, cost, and preparation time of home-prepared replacement milks containing powdered full cream milk (PM) and fresh full cream milk (FM) and different micronutrient supplements (2 g UNICEF micronutrient sachet, government supplement routinely available in district public health clinics, and best available liquid paediatric supplement found in local pharmacies) were compared. Costs of locally available ingredients for replacement milk were used to calculate monthly costs for infants aged one, three, and six months. Total monthly costs of ingredients of commercial and home-prepared replacement milks were compared with each other and the average monthly income of domestic or shop workers. Time needed to prepare one feed of replacement milk was simulated.", "When mixed with water, sugar, and each micronutrient supplement, PM and FM provided<50% of estimated required amounts for vitamins E and C, folic acid, iodine, and selenium and<75% for zinc and pantothenic acid. PM and FM made with UNICEF micronutrient sachets provided 30% adequate intake for niacin. FM prepared with any micronutrient supplement provided no more than 32% vitamin D. All PMs provided more than adequate amounts of vitamin D. Compared with the commercial formula, PM and FM provided 8-60% of vitamins A, E, and C, folic acid, manganese, zinc, and iodine. Preparations of PM and FM provided 11% minimum recommended linoleic acid and 67% minimum recommended alpha-linolenic acid per 450 ml mixture. It took 21-25 minutes to optimally prepare 120 ml of replacement feed from PM or commercial infant formula and 30-35 minutes for the fresh milk preparation. PM or FM cost approximately 20% of monthly income averaged over the first six months of life; commercial formula cost approximately 32%." ], "LABELS": [ "OBJECTIVE", "METHODS", "FINDINGS" ], "MESHES": [ "HIV Infections", "Humans", "Infant Food", "Infant, Newborn", "Infectious Disease Transmission, Vertical", "Milk Substitutes", "South Africa", "United Nations", "World Health Organization" ], "YEAR": "2004", "reasoning_required_pred": "yes", "reasoning_free_pred": "no", "final_decision": "no", "LONG_ANSWER": "No home-prepared replacement milks in South Africa meet all estimated micronutrient and essential fatty acid requirements of infants aged<6 months. Commercial infant formula is the only replacement milk that meets all nutritional needs. Revisions of WHO/UNAIDS/UNICEF HIV and infant feeding course replacement milk options are needed. If replacement milks are to provide total nutrition, preparations should include vegetable oils, such as soybean oil, as a source of linoleic and alpha-linolenic acids, and additional vitamins and minerals." }, "15954832": { "QUESTION": "Is minilaparoscopic inguinal hernia repair feasible?", "CONTEXTS": [ "Laparoscopy has rapidly emerged as the preferred surgical approach to a number of different diseases because it allows for a correct diagnosis and proper treatment. It seems to be moving toward the use of mini-instruments (5 mm or less in diameter). The aim of this paper is to illustrate retrospectively the results of an initial experience of minilaparoscopic transabdominal preperitoneal (miniTAPP) repair of groin hernia defects performed at two institutions.", "Between February 2000 and December 2003, a total of 303 patients (mean age, 45 years) underwent a miniTAPP procedure: 213 patients (70.2%) were operated on bilaterally and 90 (28.7%) for a unilateral defect, with a total of 516 hernia defects repaired. The primary endpoint was the feasibility rate for miniTAPP. The secondary endpoint was the incidence of mini-TAPP-related complications.", "No conversions to laparoscopy or an anterior open approach were required. There were no major complications, while minor complications ranged as high as 0.3%." ], "LABELS": [ "INTRODUCTION", "MATERIALS AND METHODS", "RESULTS" ], "MESHES": [ "Adolescent", "Adult", "Aged", "Digestive System Surgical Procedures", "Female", "Hernia, Inguinal", "Humans", "Laparoscopy", "Male", "Middle Aged", "Retrospective Studies" ], "YEAR": "2005", "reasoning_required_pred": "yes", "reasoning_free_pred": "yes", "final_decision": "yes", "LONG_ANSWER": "While limited by its retrospective design, the present study indicates that the minilaparoscopic approach to groin hernia repair is safe and effective, making miniTAPP a challenging alternative to laparoscopy in the approach to groin hernia repair." }, "19640728": { "QUESTION": "Surgical treatment of prosthetic valve endocarditis in patients with double prostheses: is single-valve replacement safe?", "CONTEXTS": [ "Bias against operating on patients with prosthetic valve endocarditis (PVE) who have multiple prostheses may preclude the use of life-saving valve replacement. We investigated the accuracy of the preoperative diagnosis of PVE in patients with both mitral and aortic prosthesis and the safety of single-valve replacement when only one valve seemed infected.", "Patients with a diagnosis of active PVE who had mitral and aortic prosthesis in place were assessed. We looked at the methods for diagnosis, causative agents, indication for valve replacement, operative findings and outcome.", "Twenty patients, who had both mitral and aortic prostheses and a diagnosis of PVE, were assessed. Streptococci and staphylococci caused 70% of cases. By means of echocardiography, the valves involved were: mitral (11 patients), aortic (six patients), and in three cases both prosthetic valves seemed infected. Surgery was undertaken in 17 patients (85%). The positive predictive value of transesophageal echocardiogram (TEE) for the preoperative diagnosis of the site of infection was 100%. In 13 patients, only the prosthetic valve that seemed infected was replaced. Four of these patients died within a week after the procedure. Nine patients survived the surgical procedure, completed a course of antimicrobial therapy and were followed up for 15.78 months (95% CI: 12.83-18.72). All were considered cured and relapses were not observed." ], "LABELS": [ "OBJECTIVE", "METHODS", "RESULTS" ], "MESHES": [ "Aged", "Aortic Valve", "Echocardiography, Transesophageal", "Endocarditis, Bacterial", "Female", "Heart Valve Prosthesis", "Heart Valve Prosthesis Implantation", "Humans", "Male", "Middle Aged", "Mitral Valve", "Prosthesis-Related Infections", "Retrospective Studies", "Treatment Outcome" ], "YEAR": "2010", "reasoning_required_pred": "yes", "reasoning_free_pred": "yes", "final_decision": "yes", "LONG_ANSWER": "TEE allowed a diagnosis of site involvement that did correlate with the anatomic diagnosis obtained during the operation. This fact contributed to the management of patients and was of great help in guiding the surgical intervention. Echo-oriented single-valve replacement may be a safe strategy for patients with PVE and double prostheses." }, "22154448": { "QUESTION": "Epidural analgesia for surgical treatment of peritoneal carcinomatosis: a risky technique?", "CONTEXTS": [ "To study the risks of haemodynamic instability, and the possible occurrence of spinal haematoma, meningitis and epidural abscess when epidural analgesia is performed for cytoreductive surgery and hyperthermic intraperitoneal chemotherapy (HIPEC).", "We retrospectively analyzed the data of 35 patients treated by HIPEC with oxaliplatin or cisplatin. An epidural catheter was inserted before induction of general anaesthesia. Postoperatively, a continuous epidural infusion of ropivacain, then a patient-controlled epidural analgesia were started.", "The epidural catheter was used peroperatively before HIPEC in 12 subjects (34%), and after HIPEC in 23 subjects (66%). The median dose of ropivacain given peroperatively in the epidural catheter was 40 mg (30-75). Norepinephrin was used in two subjects (6%) peroperatively (median infusion rate 0.325 \u03bcg/kg per minute [0.32-0.33]), and in four subjects (11%) in the postoperative 24 hours. No spinal haematoma, meningitis or epidural abscess were noted. Five subjects (14%) had a thrombopenia or a prothrombin time less than 60% before catheter removal. Two subjects (6%) had a leukopenia before catheter removal. No thrombopenia or blood coagulation disorders were recorded the day of catheter removal." ], "LABELS": [ "BACKGROUND", "METHODS", "RESULTS" ], "MESHES": [ "Adult", "Aged", "Analgesia, Epidural", "Analgesia, Patient-Controlled", "Anesthesia, General", "Antineoplastic Agents", "Carcinoma", "Cisplatin", "Combined Modality Therapy", "Epidural Abscess", "Female", "Hematoma, Epidural, Spinal", "Hemodynamics", "Humans", "Hyperthermia, Induced", "Male", "Meningitis", "Middle Aged", "Norepinephrine", "Organoplatinum Compounds", "Peritoneal Neoplasms", "Retrospective Studies", "Risk", "Safety", "Thrombocytopenia", "Vasoconstrictor Agents" ], "YEAR": "2012", "reasoning_required_pred": "no", "reasoning_free_pred": "no", "final_decision": "no", "LONG_ANSWER": "In this series of 35 patients, the use of epidural analgesia for HIPEC does not seem to be associated with a worse risk of haemodynamic instability, spinal haematoma, meningitis or epidural abscess. HIPEC with platinum salt is not incompatible with the safety of epidural analgesia, with an optimized fluid management peroperatively and the following of perimedullary anesthesia practice guidelines." }, "11146778": { "QUESTION": "Risk stratification in emergency surgical patients: is the APACHE II score a reliable marker of physiological impairment?", "CONTEXTS": [ "The APACHE II (Acute Physiology and Chronic Health Evaluation II) score used as an intensive care unit (ICU) admission score in emergency surgical patients is not independent of the effects of treatment and might lead to considerable bias in the comparability of defined groups of patients and in the evaluation of treatment policies. Postoperative monitoring with the APACHE II score is clinically irrelevant.", "Inception cohort study.", "Secondary referral center.", "Eighty-five consecutive emergency surgical patients admitted to the surgical ICU in 1999. The APACHE II score was calculated before surgery; after admission to the ICU; and on postoperative days 3, 7, and 10.", "APACHE II scores and predicted and observed mortality rates.", "The mean +/- SD APACHE II score of 24.2 +/- 8.3 at admission to the ICU was approximately 36% greater than the initial APACHE II score of 17.8 +/- 7.7, a difference that was highly statistically significant (P<.001). The overall mortality of 32% favorably corresponds with the predicted mortality of 34% according to the initial APACHE II score. However, the predicted mortality of 50% according to the APACHE II score at admission to the ICU was significantly different from the observed mortality rate (P =.02). In 40 long-term patients (>/=10 days in the ICU), the difference between the APACHE II scores of survivors and patients who died was statistically significant on day 10 (P =.04)." ], "LABELS": [ "HYPOTHESES", "DESIGN", "SETTING", "PATIENTS", "MAIN OUTCOME MEASURES", "RESULTS" ], "MESHES": [ "APACHE", "Aged", "Cohort Studies", "Emergency Treatment", "Female", "Humans", "Intensive Care Units", "Male", "Preoperative Care", "Risk Assessment", "Surgical Procedures, Operative", "Survival Rate", "Time Factors" ], "YEAR": "2001", "reasoning_required_pred": "yes", "reasoning_free_pred": "yes", "final_decision": "yes", "LONG_ANSWER": "For risk stratification in emergency surgical patients, it is essential to measure the APACHE II score before surgical treatment. Longitudinal APACHE II scoring reveals continuous improvement of the score in surviving patients but has no therapeutic relevance in the individual patient." }, "8199520": { "QUESTION": "Are physicians meeting the needs of family caregivers of the frail elderly?", "CONTEXTS": [ "To explore expressed needs, both formal and informal, of family caregivers of frail elderly. To evaluate roles of physicians.", "Questionnaire survey of members of the Montreal Jewish community providing care for frail elderly family members.", "Jewish community of Montreal.", "Volunteer caregivers who were caring for a family member or friend 60 years or older, who had greatest responsibility for providing physical or emotional support to an elderly person, who saw themselves as caregivers, and who could speak English or French were studied. Of 118 volunteers, 32 were excluded because they withdrew for personal reasons or because they did not meet study criteria.", "Demographic variables, functional status of the care receiver, use of home care services, and needs assessment to identify additional services.", "An average of 75.4% respondents did not use formal support services. Just under half of caregivers were dissatisfied with the attention they received from the health care system, and more than one third expressed feelings of stress, depression, guilt, and isolation." ], "LABELS": [ "OBJECTIVE", "DESIGN", "SETTING", "PARTICIPANTS", "MAIN OUTCOME MEASURES", "RESULTS" ], "MESHES": [ "Activities of Daily Living", "Adult", "Aged", "Attitude to Health", "Caregivers", "Counseling", "Family", "Female", "Frail Elderly", "Geriatric Assessment", "Health Services Accessibility", "Health Services Needs and Demand", "Health Services Research", "Home Care Services", "Humans", "Jews", "Male", "Middle Aged", "Patient Satisfaction", "Physician's Role", "Quebec", "Surveys and Questionnaires" ], "YEAR": "1994", "reasoning_required_pred": "no", "reasoning_free_pred": "no", "final_decision": "no", "LONG_ANSWER": "Hypotheses for this discontent are presented. Physicians may be uninterested in helping caregivers; even if they were receptive to counseling caregivers, they could be poorly remunerated for the types of counseling sessions that are usual for caregivers; and being a professional caregiver to family caregivers is demanding in itself." }, "9100537": { "QUESTION": "Can nonproliferative breast disease and proliferative breast disease without atypia be distinguished by fine-needle aspiration cytology?", "CONTEXTS": [ "Cytologic criteria reported to be helpful in the distinction of proliferative breast disease without atypia (PBD) from nonproliferative breast disease (NPBD) have not been rigorously tested.", "Fifty-one air-dried, Diff-Quik-stained fine-needle aspirates (FNA) of palpable breast lesions with biopsy-proven diagnoses of NPBD (34 cases) or PBD (17 cases) were reviewed. The smears were evaluated for the cellularity, size, and architectural arrangement of the epithelial groups; the presence of single epithelial cells and myoepithelial cells; and nuclear characteristics.", "The only cytologic feature found to be significantly different between PBD and NPBD was a swirling pattern of epithelial cells. A swirling pattern was noted in 13 of 17 PBD cases (76%) and 12 of 34 NPBD cases (35%) (P = 0.008)." ], "LABELS": [ "BACKGROUND", "METHODS", "RESULTS" ], "MESHES": [ "Biopsy, Needle", "Breast Diseases", "Diagnosis, Differential", "Female", "Humans", "Precancerous Conditions", "Predictive Value of Tests" ], "YEAR": "1997", "reasoning_required_pred": "yes", "reasoning_free_pred": "no", "final_decision": "no", "LONG_ANSWER": "Because there is significant overlap in cytologic features between NPBD and PBD, the distinction between the two entities is difficult in cytologic sampling." }, "26518378": { "QUESTION": "Are Reviewers' Scores Influenced by Citations to Their Own Work?", "CONTEXTS": [ "Academic medical researchers are judged by how often their publications are cited in the literature. When serving as journal reviewers, they may be more favorably disposed to manuscripts that cite their work. We investigate whether manuscripts that contain a citation to the reviewer's work receive higher evaluations than those that do not and\u00a0whether peer reviewers encourage authors to cite that reviewer's work.", "We analyzed all research manuscripts submitted in 2012 to Annals of Emergency Medicine to determine whether they contained citations to each reviewer's work. To determine whether citation affected reviewer scores, we obtained each reviewer's score of the manuscript's overall desirability (1=worst to 5=best) and used descriptive statistics and regression modeling to compare scores of cited and noncited reviewers. We also enumerated how often reviewers suggested that authors add citations to the reviewer's work or other work.", "There were 395 manuscripts and 999 corresponding reviews with an manuscript desirability score. The 83 reviews by cited reviewers (8.3%) had a mean score of 2.8 (SD 1.4); the 916 reviews by noncited reviewers (91.7%), 2.5 (1.2; \u0394=0.3; 95% confidence interval [CI] 0 to 0.6). The mean score in the 117 reviews of the noncited reviewers of\u00a0the\u00a057\u00a0manuscripts that had both cited and noncited reviewers was 2.9 (SD 1.2) compared with 2.9 (SD 1.1) for the 68 reviews by cited reviewers (\u0394=0; 95% CI -0.3 to 0.4). In the final ordinal regression model, the unadjusted OR for\u00a0the manuscript desirability score was 1.6 (95% CI 1.0 to 2.7); when adjusting for the manuscripts' mean desirability score, it was 1.4 (95% CI 0.8 to\u00a02.2), demonstrating that manuscript quality was a confounder. Authors were asked to\u00a0add a citation to the reviewer's work in 28 reviews (3%) but to others' work in 98 (10%)." ], "LABELS": [ "STUDY OBJECTIVE", "METHODS", "RESULTS" ], "MESHES": [ "Authorship", "Biomedical Research", "Emergency Medicine", "Humans", "Manuscripts, Medical as Topic", "Peer Review, Research", "Periodicals as Topic", "Publishing" ], "YEAR": "2016", "reasoning_required_pred": "yes", "reasoning_free_pred": "yes", "final_decision": "yes", "LONG_ANSWER": "In a leading specialty journal, cited reviewers gave higher scores than noncited reviewers. However, this\u00a0was likely due to their being assigned higher-quality manuscripts and not because they were cited in the manuscript. Reviewer requests that their work be cited were rare." }, "23774337": { "QUESTION": "Does the central venous pressure predict fluid responsiveness?", "CONTEXTS": [ "Despite a previous meta-analysis that concluded that central venous pressure should not be used to make clinical decisions regarding fluid management, central venous pressure continues to be recommended for this purpose.AIM: To perform an updated meta-analysis incorporating recent studies that investigated indices predictive of fluid responsiveness. A priori subgroup analysis was planned according to the location where the study was performed (ICU or operating room).", "MEDLINE, EMBASE, Cochrane Register of Controlled Trials, and citation review of relevant primary and review articles.", "Clinical trials that reported the correlation coefficient or area under the receiver operating characteristic curve (AUC) between the central venous pressure and change in cardiac performance following an intervention that altered cardiac preload. From 191 articles screened, 43 studies met our inclusion criteria and were included for data extraction. The studies included human adult subjects, and included healthy controls (n = 1) and ICU (n = 22) and operating room (n = 20) patients.", "Data were abstracted on study characteristics, patient population, baseline central venous pressure, the correlation coefficient, and/or the AUC between central venous pressure and change in stroke volume index/cardiac index and the percentage of fluid responders. Meta-analytic techniques were used to summarize the data.", "Overall 57% \u00b1 13% of patients were fluid responders. The summary AUC was 0.56 (95% CI, 0.54-0.58) with no heterogenicity between studies. The summary AUC was 0.56 (95% CI, 0.52-0.60) for those studies done in the ICU and 0.56 (95% CI, 0.54-0.58) for those done in the operating room. The summary correlation coefficient between the baseline central venous pressure and change in stroke volume index/cardiac index was 0.18 (95% CI, 0.1-0.25), being 0.28 (95% CI, 0.16-0.40) in the ICU patients, and 0.11 (95% CI, 0.02-0.21) in the operating room patients." ], "LABELS": [ "BACKGROUND", "DATA SOURCES", "STUDY SELECTION", "DATA EXTRACTION", "DATA SYNTHESIS" ], "MESHES": [ "Central Venous Pressure", "Fluid Therapy", "Hemodynamics", "Humans", "Intensive Care Units", "Respiration, Artificial" ], "YEAR": "2013", "reasoning_required_pred": "no", "reasoning_free_pred": "no", "final_decision": "no", "LONG_ANSWER": "There are no data to support the widespread practice of using central venous pressure to guide fluid therapy. This approach to fluid resuscitation should be abandoned." }, "16151770": { "QUESTION": "Memory-provoked rCBF-SPECT as a diagnostic tool in Alzheimer's disease?", "CONTEXTS": [ "Alzheimer's disease (AD) is a primary degenerative disease that progressively affects all brain functions, with devastating consequences for the patient, the patient's family and society. Rest regional cerebral blood flow (rCBF) could have a strategic role in differentiating between AD patients and normal controls, but its use for this purpose has a low discriminatory capacity. The purpose of this study was to evaluate whether the diagnostic sensitivity of rCBF single-photon emission computed tomography (SPECT) could be increased by using an episodic memory task provocation, i.e. memory-provoked rCBF-SPECT (MP-SPECT).", "Eighteen persons (73.2+/-4.8 years) with mild AD and 18 healthy elderly (69.4+/-3.9 years) were included in the study. The subjects were injected with (99m)Tc-hexamethylpropylene amine oxime (HMPAO) during memory provocation with faces and names, followed by an rCBF-SPECT study. The rCBF (99m)Tc-HMPAO SPECT images were analysed using statistical parametric mapping (SPM2). Peaks with a false discovery rate corrected value of 0.05 were considered significant.", "On MP-SPECT, the AD group showed a significant rCBF reduction in the left parietal cortex in comparison with healthy elderly. At rest, no significant group differences were seen." ], "LABELS": [ "PURPOSE", "METHODS", "RESULTS" ], "MESHES": [ "Adult", "Aged", "Alzheimer Disease", "Brain", "Brain Mapping", "Cerebrovascular Circulation", "Evoked Potentials", "Female", "Humans", "Image Interpretation, Computer-Assisted", "Male", "Memory", "Radiopharmaceuticals", "Reproducibility of Results", "Sensitivity and Specificity", "Technetium Tc 99m Exametazime", "Tomography, Emission-Computed, Single-Photon" ], "YEAR": "2006", "reasoning_required_pred": "yes", "reasoning_free_pred": "yes", "final_decision": "yes", "LONG_ANSWER": "Memory provocation increased the sensitivity of rCBF-SPECT for the detection of AD-related blood flow changes in the brain at the group level. Further studies are needed to evaluate MP-SPECT as a diagnostic tool at the individual level. If a higher sensitivity for AD at the individual level is verified in future studies, a single MP-SPECT study might be sufficient in the clinical setting." }, "26418441": { "QUESTION": "Can we ease the financial burden of colonoscopy?", "CONTEXTS": [ "Polyps identified at colonoscopy are predominantly diminutive (<5\u2009mm) with a small risk (>1%) of high-grade dysplasia or carcinoma; however, the cost of histological assessment is substantial.AIM: The aim of this study was to determine whether prediction of colonoscopy surveillance intervals based on real-time endoscopic assessment of polyp histology is accurate and cost effective.", "A prospective cohort study was conducted across a tertiary care and private community hospital. Ninety-four patients underwent colonoscopy and polypectomy of diminutive (\u22645\u2009mm) polyps from October 2012 to July 2013, yielding a total of 159 polyps. Polyps were examined and classified according to the Sano-Emura classification system. The endoscopic assessment (optical diagnosis) of polyp histology was used to predict appropriate colonoscopy surveillance intervals. The main outcome measure was the accuracy of optical diagnosis of diminutive colonic polyps against the gold standard of histological assessment.", "Optical diagnosis was correct in 105/108 (97.2%) adenomas. This yielded a sensitivity, specificity and positive and negative predictive values (with 95%CI) of 97.2% (92.1-99.4%), 78.4% (64.7-88.7%), 90.5% (83.7-95.2%) and 93% (80.9-98.5%) respectively. Ninety-two (98%) patients were correctly triaged to their repeat surveillance colonoscopy. Based on these findings, a cut and discard approach would have resulted in a saving of $319.77 per patient." ], "LABELS": [ "BACKGROUND", "METHODS", "RESULTS" ], "MESHES": [ "Adenomatous Polyps", "Adult", "Aged", "Australia", "Colonic Neoplasms", "Colonoscopy", "Cost-Benefit Analysis", "Early Detection of Cancer", "Female", "Humans", "Male", "Middle Aged", "Predictive Value of Tests", "Prospective Studies", "Sensitivity and Specificity" ], "YEAR": "2015", "reasoning_required_pred": "yes", "reasoning_free_pred": "yes", "final_decision": "yes", "LONG_ANSWER": "Endoscopists within a tertiary care setting can accurately predict diminutive polyp histology and confer an appropriate surveillance interval with an associated financial benefit to the healthcare system. However, limitations to its application in the community setting exist, which may improve with further training and high-definition colonoscopes." }, "20130378": { "QUESTION": "Is low birth weight a risk indicator for congenital cytomegalovirus infection?", "CONTEXTS": [ "Congenital cytomegalovirus infection is currently the leading cause of congenital infection in 0.2-2.2% of live births worldwide leading to variable serious sequalae. The aim of the study was to determine if low birth weight is an indicator of CMV congenital infection evidenced by detecting CMV-DNA in umbilical cord blood at the time of delivery.", "CMV-IgG and IgM antibodies and CMV-DNAemia were assessed in umbilical cord blood of two hundreds newborns, one hundred of whom had birth weight2700 gram)." ], "LABELS": [ "BACKGROUND", "METHODOLOGY", "RESULTS" ], "MESHES": [ "Antibodies, Viral", "Body Weights and Measures", "Cytomegalovirus", "Cytomegalovirus Infections", "DNA, Viral", "Fetal Blood", "Humans", "Immunoglobulin G", "Immunoglobulin M", "Infant, Low Birth Weight", "Infant, Newborn", "Infant, Newborn, Diseases", "Polymerase Chain Reaction", "Risk Factors", "Viremia" ], "YEAR": "2009", "reasoning_required_pred": "yes", "reasoning_free_pred": "no", "final_decision": "no", "LONG_ANSWER": "CMV-IgM and IgG antibodies assessment was not a potential discriminative test to identify congenitally infected newborns. In addition, low birth weight and small head circumference at birth failed to predict congenital CMV infection. CMV-DNA detection in umbilical cord blood at the time of delivery using real-time PCR of all newborns is recommended as decisive, rapid and non-invasive test." }, "11799314": { "QUESTION": "Is external palliative radiotherapy for gallbladder carcinoma effective?", "CONTEXTS": [ "Gallbladder carcinoma is characterized by delayed diagnosis, ineffective treatment and poor prognosis. Surgical resection has been thought to be the treatment of choice, while the role of radiotherapy as adjuvant or palliative treatment has not been fully clarified in the literature.", "We present the case of a 45-year-old female, with unresectable gallbladder carcinoma, grade IV, histologically diagnosed during laparotomy. The patient was treated with palliative intent with percutaneous transhepatic biliary drainage. Furthermore, she received external radiotherapy by (60)Co, using a three-field technique (anterior-posterior and right lateral). The total dose was 3,000 cGy in 10 fractions, with 300 cGy per fraction, 5 days weekly.", "The patient showed clinico-laboratory improvement and was discharged with a permanent percutaneous transhepatic endoprosthesis. During follow-up (10 and 12 months postirradiation), abdominal CTs showed no local extension of the tumor, while the patient had a good performance status. So far, 1 year after the diagnosis of gallbladder cancer she is still alive." ], "LABELS": [ "BACKGROUND", "PATIENT AND METHODS", "RESULTS" ], "MESHES": [ "Cobalt Radioisotopes", "Female", "Follow-Up Studies", "Gallbladder Neoplasms", "Humans", "Middle Aged", "Neoplasm Staging", "Palliative Care", "Radioisotope Teletherapy", "Radiotherapy Planning, Computer-Assisted", "Tomography, X-Ray Computed" ], "YEAR": "2001", "reasoning_required_pred": "yes", "reasoning_free_pred": "yes", "final_decision": "yes", "LONG_ANSWER": "External radiotherapy seems to be a safe and effective method of palliative management of gallbladder carcinoma. However, further studies are necessary to determine the role of radiotherapy in palliative or adjuvant treatment of gallbladder carcinoma." }, "20828836": { "QUESTION": "Is discordance in TEOAE and AABR outcomes predictable in newborns?", "CONTEXTS": [ "To determine the perinatal predictors of discordant screening outcomes based on a two-stage screening protocol with transient-evoked otoacoustic emissions (TEOAE) and automated auditory brainstem response (AABR).", "A cross-sectional study of infants tested with TEOAE and AABR under a hospital-based universal newborn hearing screening program in Lagos, Nigeria. Maternal and infant factors associated with discordant TEOAE and AABR outcomes were determined with multivariable logistic regression analyses adjusting for potential confounding factors.", "Of the 4718 infants enrolled under the program 1745 (36.9%) completed both TEOAE and AABR. Of this group, 1060 (60.7%) passed both TEOAE and AABR (\"true-negatives\"); 92 (5.3%) failed both TEOAE and AABR (\"true-positive\"); 571 (32.7%) failed TEOAE but passed AABR (\"false-positives\") while 22 (1.3%) passed TEOAE but failed AABR (\"false-negatives\"). Infants with false-positives were likely to be admitted into well-baby nursery (p=0.001), belong to mothers who attended antenatal care (p=0.010) or who delivered vaginally (p<0.001) compared to infants with true-negatives while infants with true-positives were also more likely to be delivered vaginally (p=0.002) or admitted into well-baby nursery (p=0.035) compared to infants with false-negatives. Infants with true-positives were significantly more likely to be delivered vaginally (p<0.001) and have severe hyperbilirubinemia (p=0.045) compared with infants with true-negatives. No association was observed between false-negatives and true-negatives. Antenatal care status, mode of delivery and nursery type were useful predictors of discordant outcomes among all infants undergoing screening (c-statistic=0.73)." ], "LABELS": [ "OBJECTIVE", "METHODS", "RESULTS" ], "MESHES": [ "Adult", "Cross-Sectional Studies", "Delivery, Obstetric", "Evoked Potentials, Auditory, Brain Stem", "False Negative Reactions", "False Positive Reactions", "Female", "Hearing Loss", "Humans", "Hyperbilirubinemia, Neonatal", "Infant, Newborn", "Logistic Models", "Male", "Neonatal Screening", "Nigeria", "Nurseries, Hospital", "Otoacoustic Emissions, Spontaneous", "Prenatal Care", "Retrospective Studies" ], "YEAR": "2010", "reasoning_required_pred": "yes", "reasoning_free_pred": "no", "final_decision": "yes", "LONG_ANSWER": "Given the available screening technologies, discordant TEOAE and AABR may be inevitable for some categories of hearing loss among apparently healthy newborns whose mothers received prenatal care. The potential limitations of perinatal morbidities as basis of targeted screening for such cases therefore merit further consideration." }, "19608436": { "QUESTION": "Chemical shift MRI: is there any contribution to morphologic evaluation of solid breast masses?", "CONTEXTS": [ "To investigate the contribution of chemical shift magnetic resonance imaging for assessment of the margins of solid breast masses by benefiting from India ink artifact.", "Eighty-eight masses in 64 patients were evaluated in T1- and T2-weighted images, dynamic contrast and chemical shift studies according to Breast Imaging Reporting and Data System magnetic resonance lexicon. Subtraction images were automatically obtained by chemical shift imaging and dynamic studies. Each sequence was scored using a scale of 1 to 5 according to its ability to demonstrate margins separate from surrounding parenchyma. Breast parenchyma was evaluated as fatty and dense. The results were compared with the histopathologic results.", "Twenty-eight (31.8%) of the lesions were localized in fatty breast, and the remaining 60 (68.2%) lesions were localized in dense breast. There were 34 (38.6%) benign and 54 (61.4%) malignant masses. In fatty breast, chemical shift subtraction and T1-weighted images were valuable both for the demonstration and differentiation of benign lesions (P<.05). None of the sequence was valuable for both the demonstration and differentiation of malignant lesions in fatty breasts (P>.05). In dense breasts, chemical shift subtraction and dynamic contrast subtraction images were valuable for both the demonstration and differentiation of benign and malignant lesions. Additional to these sequences, T2-weighted images was also valuable for benign lesions (P<.05)." ], "LABELS": [ "RATIONALE AND OBJECTIVES", "METHODS AND MATERIALS", "RESULTS" ], "MESHES": [ "Adult", "Aged", "Algorithms", "Breast Neoplasms", "Female", "Humans", "Image Enhancement", "Image Interpretation, Computer-Assisted", "Magnetic Resonance Imaging", "Middle Aged", "Reproducibility of Results", "Sensitivity and Specificity", "Subtraction Technique" ], "YEAR": "2009", "reasoning_required_pred": "yes", "reasoning_free_pred": "yes", "final_decision": "yes", "LONG_ANSWER": "Chemical shift subtraction can contribute to routine dynamic contrast subtraction in morphologic analysis particularly for the evaluation of margins of benign lesions in fatty breasts. It can also help in morphologic analysis of masses in dense breast." }, "15995461": { "QUESTION": "Do some U.S. states have higher/lower injury mortality rates than others?", "CONTEXTS": [ "This article examines the hypothesis that the six U.S. states with the highest rates of road traffic deaths (group 1 states) also had above-average rates of other forms of injury such as falling, poisoning, drowning, fire, suffocation, homicide, and suicide, and also for the retail trade and construction industries. The converse, second hypothesis, for the six states with the lowest rates of road traffic deaths (group 2 states) is also examined.", "Data for these 12 states for the period 1983 to 1995 included nine categories of unintentional and four categories of intentional injury. Seventy-four percent of the group 1 states conformed to the first hypothesis, and 85% of the group 2 states conformed to the second hypothesis." ], "LABELS": [ "BACKGROUND", "RESULTS" ], "MESHES": [ "Accidental Falls", "Accidents, Traffic", "Asphyxia", "Drowning", "Female", "Fires", "Humans", "Male", "Poisoning", "United States", "Wounds and Injuries", "Wounds, Gunshot" ], "YEAR": "2005", "reasoning_required_pred": "yes", "reasoning_free_pred": "yes", "final_decision": "yes", "LONG_ANSWER": "Group 1 states are likely to exhibit above-average rates for most other categories of injury death, whereas group 2 states are even more likely to exhibit below-average rates for most other categories of injury death." }, "23224030": { "QUESTION": "Do European people with type 1 diabetes consume a high atherogenic diet?", "CONTEXTS": [ "Individuals with type 1 diabetes have a high risk of developing cardiovascular diseases, and it has been reported that they consume a high atherogenic diet. We examined how nutrient intake and adherence to current European nutritional recommendations evolved in a large cohort of European individuals with type 1 diabetes over a period of 7 years.SUBJECTS/", "We analysed data from the EURODIAB Prospective Complications Study, a European multicentre prospective cohort study. Standardized 3-day dietary records were employed in individuals with type 1 diabetes. One thousand one hundred and two patients (553 men, 549 women, baseline age 33 \u00b1 10 years, duration 15 \u00b1 9 years) had complete nutritional data available at baseline and after 7 years. We calculated mean differences in reported nutrients over time and adjusted these for age, gender, HbA1c and BMI with ANOVA models.", "Compared to baseline, there were minor changes in nutrients. Reported protein (-0.35% energy (en), fat (-1.07% en), saturated fat (-0.25% en) and cholesterol (-7.42 mg/1000 kcal) intakes were lower, whereas carbohydrate (+1.23% en) and fibre (+0.46 g/1000 kcal) intakes were higher at the 7-year follow-up. European recommendations for adequate nutrient intakes were followed in individuals with type 1 diabetes for protein (76% at baseline and 78% at follow-up), moderately for fat (34, 40%), carbohydrate (34, 41%) and cholesterol (39, 47%), but poorly for fibre (1.4, 2.4%) and saturated fat (11, 13%)." ], "LABELS": [ "OBJECTIVES", "METHODS", "RESULTS" ], "MESHES": [ "Adolescent", "Adult", "Body Mass Index", "Body Weight", "Cardiovascular Diseases", "Cholesterol", "Cholesterol, Dietary", "Diabetes Mellitus, Type 1", "Diet Records", "Diet, Atherogenic", "Dietary Carbohydrates", "Dietary Fiber", "Dietary Proteins", "Energy Intake", "European Continental Ancestry Group", "Female", "Follow-Up Studies", "Humans", "Insulin", "Male", "Middle Aged", "Motor Activity", "Nutrition Assessment", "Nutritional Status", "Prospective Studies", "Recommended Dietary Allowances", "Young Adult" ], "YEAR": "2013", "reasoning_required_pred": "no", "reasoning_free_pred": "yes", "final_decision": "yes", "LONG_ANSWER": "European individuals with type 1 diabetes consume a high atherogenic diet as few patients met recommendations for dietary fibre and saturated fat. This study showed minor changes in dietary nutrients and energy intakes over a period of 7 years. Nutrition education needs particular focus on strategies to increase dietary fibre and reduce saturated fat to exploit their potential benefit." }, "17910536": { "QUESTION": "Adults with mild intellectual disabilities: can their reading comprehension ability be improved?", "CONTEXTS": [ "Adults with a mild intellectual disability (ID) often show poor decoding and reading comprehension skills. The goal of this study was to investigate the effects of teaching text comprehension strategies to these adults. Specific research goals were to determine (1) the effects of two instruction conditions, i.e. strategy instruction to individuals and strategy instruction in small groups in a reciprocal teaching context; (2) intervention programme effects on specific strategy tests (so-called direct effects), and possible differences between strategies; (3) (long-term) transfer effects of the programme on general reading comprehension ability; and (4) the regression of general text comprehension by the variables of technical reading, IQ, reading comprehension of sentences (RCS), and pretest and posttest scores on the strategies taught.", "In total, 38 adults (age range 20-72 years; mean age of 36 years) with ID participated in the study. IQs ranged from 45 to 69 with a mean IQ of 58. The intervention programme involved 15 weekly lessons of 1 h each, taught during 3 months. Blocks of lessons included each of Brown and Palincsar's strategies of summarizing, questioning, clarifying and predicting, as participants read and studied narrative and expository texts.", "Results indicated no significant difference between group and individual instruction conditions. Second, direct programme effects - as determined by posttest-pretest contrasts for strategy tests - were substantial, except for the questioning strategy. Third, even more substantial was the transfer effect to general text comprehension. Moreover, the results on this test were well maintained at a follow-up test. Finally, the variance of general reading comprehension ability was best explained by the test of RCS, and only moderately by the strategies trained." ], "LABELS": [ "BACKGROUND", "METHODS", "RESULTS" ], "MESHES": [ "Adult", "Aged", "Comprehension", "Education of Intellectually Disabled", "Female", "Follow-Up Studies", "Group Processes", "Humans", "Intelligence", "Male", "Middle Aged", "Netherlands", "Reading", "Transfer (Psychology)" ], "YEAR": "2007", "reasoning_required_pred": "no", "reasoning_free_pred": "yes", "final_decision": "yes", "LONG_ANSWER": "The presently used intervention programme provides a good starting point for adults with ID to become better readers." }, "12070552": { "QUESTION": "Do antibiotics decrease post-tonsillectomy morbidity?", "CONTEXTS": [ "A tonsillectomy audit was carried out and compared with other studies, to emphasize the role of antibiotics.", "This study was carried out at North West Armed Forces Hospital, Tabuk, Kingdom of Saudi Arabia, during the year January 1999 through to December 1999. This is a retrospective study of patients who had tonsillectomy with or with adenoidectomy, the topics audited included indication for surgery, grade of surgeon, method of surgery, length of hospital stay, complications and the use of postoperative antibiotics.", "A total of 185 patients underwent tonsillectomy with or without adenoidectomy. The patients age ranged between 2 years to 53 years and the majority were children. In our audit we found no difference with regard to grade of surgeons, method of hemostasis in the outcome of surgery. Moreover, postoperative antibiotics had no role in pain control, postoperative fever, secondary hemorrhage or reduction in hospital stay. The administration of analgesics on the basis of, as required, had poor pain control." ], "LABELS": [ "OBJECTIVE", "METHODS", "RESULTS" ], "MESHES": [ "Adolescent", "Adult", "Anti-Bacterial Agents", "Anti-Inflammatory Agents, Non-Steroidal", "Child", "Child, Preschool", "Female", "Humans", "Male", "Medical Audit", "Middle Aged", "Postoperative Care", "Postoperative Complications", "Retrospective Studies", "Tonsillectomy" ], "YEAR": "2002", "reasoning_required_pred": "no", "reasoning_free_pred": "no", "final_decision": "no", "LONG_ANSWER": "Post tonsillectomy antibiotics did not prove to have a role in minimizing postoperative morbidity. Moreover, analgesics given on the basis of as required had a limited value." }, "18182265": { "QUESTION": "Body diffusion-weighted MR imaging of uterine endometrial cancer: is it helpful in the detection of cancer in nonenhanced MR imaging?", "CONTEXTS": [ "In this study, the authors discussed the feasibility and value of diffusion-weighted (DW) MR imaging in the detection of uterine endometrial cancer in addition to conventional nonenhanced MR images.", "DW images of endometrial cancer in 23 patients were examined by using a 1.5-T MR scanner. This study investigated whether or not DW images offer additional incremental value to conventional nonenhanced MR imaging in comparison with histopathological results. Moreover, the apparent diffusion coefficient (ADC) values were measured in the regions of interest within the endometrial cancer and compared with those of normal endometrium and myometrium in 31 volunteers, leiomyoma in 14 patients and adenomyosis in 10 patients. The Wilcoxon rank sum test was used, with a p<0.05 considered statistically significant.", "In 19 of 23 patients, endometrial cancers were detected only on T2-weighted images. In the remaining 4 patients, of whom two had coexisting leiomyoma, no cancer was detected on T2-weighted images. This corresponds to an 83% detection sensitivity for the carcinomas. When DW images and fused DW images/T2-weighted images were used in addition to the T2-weighted images, cancers were identified in 3 of the remaining 4 patients in addition to the 19 patients (overall detection sensitivity of 96%). The mean ADC value of endometrial cancer (n=22) was (0.97+/-0.19)x10(-3)mm(2)/s, which was significantly lower than those of the normal endometrium, myometrium, leiomyoma and adenomyosis (p<0.05)." ], "LABELS": [ "OBJECTIVE", "METHODS AND MATERIALS", "RESULTS" ], "MESHES": [ "Adult", "Aged", "Contrast Media", "Diffusion Magnetic Resonance Imaging", "Endometrial Neoplasms", "Female", "Humans", "Image Enhancement", "Middle Aged", "Reproducibility of Results", "Sensitivity and Specificity", "Subtraction Technique", "Whole Body Imaging" ], "YEAR": "2009", "reasoning_required_pred": "yes", "reasoning_free_pred": "yes", "final_decision": "yes", "LONG_ANSWER": "DW imaging can be helpful in the detection of uterine endometrial cancer in nonenhanced MR imaging." }, "15841770": { "QUESTION": "Do inhaled steroids differ from cromones in terms of hospital admission rates for asthma in children?", "CONTEXTS": [ "Annual data on children aged under 16 y treated for asthma, including consumption of regular medication for asthma, numbers of hospital periods, lengths of hospitalizations and annual proportions of readmissions, were collected using patient-specific medical records from 1995 to 1999. In the Kuopio province, on average, 35.6-36.7/1000 children were on maintenance for asthma, of which 23% were receiving cromones, 51% were taking inhaled steroids and 26% were treated with cromones plus intermittent steroids. In the Oulu province, the respective prevalence was 32.7-34.9/1000, and the respective proportions were 5%, 93% and 2%.", "Total and first admissions, as well as hospital days were clearly less in the Oulu province. In the children aged>or = 6y, the average annual total admissions were 0.3/1000 (Oulu) vs 1.2/1000 (Kuopio) (p<0.001). Similarly, the first admissions were 0.2/1000 vs 1.0/1000 (p<0.001), proportions of readmissions 6.3% vs 19.3% (p<0.05), and numbers of hospital days 0.7/1000 vs 3.8/1000 (p<0.001). The differences were in the same direction, though less prominent, also among children 2-5 y of age." ], "LABELS": [ "METHODS", "RESULTS" ], "MESHES": [ "Administration, Inhalation", "Adolescent", "Anti-Inflammatory Agents", "Asthma", "Child", "Child, Preschool", "Chromones", "Female", "Hospitalization", "Humans", "Length of Stay", "Male", "Patient Admission", "Prevalence", "Retrospective Studies" ], "YEAR": "2004", "reasoning_required_pred": "yes", "reasoning_free_pred": "yes", "final_decision": "yes", "LONG_ANSWER": "Our results suggest that inhaled steroids are better than cromones in preventing admissions for asthma when two provinces with different practices for maintenance medication of steady-state asthma were compared." }, "8017535": { "QUESTION": "Substance use and HIV-related sexual behaviors among US high school students: are they related?", "CONTEXTS": [ "This study was undertaken to examine whether use of alcohol, cigarettes, marijuana, cocaine, and other illicit drugs is related to the likelihood of sexual behaviors that increase risk for human immunodeficiency virus (HIV) infection among youth.", "The 1990 national Youth Risk Behavior Survey was used to collect self-reported information about a broad range of health risk behaviors from a representative sample of 11,631 high school students in the United States.", "Students who reported no substance use were least likely to report having had sexual intercourse, having had four or more sex partners, and not having used a condom at last sexual intercourse. Adjusted for age, sex, and race/ethnicity, odds ratios for each of these sexual risk behaviors were greatest among students who had used marijuana, cocaine, or other illicit drugs. Students who had used only alcohol or cigarettes had smaller but still significant increases in the likelihood of having had sexual intercourse and of having had four or more sex partners." ], "LABELS": [ "OBJECTIVES", "METHODS", "RESULTS" ], "MESHES": [ "Adolescent", "Adolescent Behavior", "Cross-Sectional Studies", "Female", "HIV Infections", "Health Knowledge, Attitudes, Practice", "Humans", "Male", "Odds Ratio", "Risk-Taking", "Sexual Behavior", "Substance-Related Disorders", "United States" ], "YEAR": "1994", "reasoning_required_pred": "yes", "reasoning_free_pred": "yes", "final_decision": "yes", "LONG_ANSWER": "HIV prevention programs for youth should recognize that substance use may be an important indicator of risk for HIV infection and acquired immunodeficiency syndrome through its association with unsafe sexual behaviors." }, "24172579": { "QUESTION": "Does sex influence the response to intravenous thrombolysis in ischemic stroke?", "CONTEXTS": [ "Women are more likely to have a worse outcome after an acute stroke than men. Some studies have suggested that women also benefit less from intravenous thrombolysis after an acute ischemic stroke, but others found no sex differences in safety and efficacy. We aimed to evaluate differences in 3-month outcome between sexes in intravenous tissue-type plasminogen activator-treated patients registered in the Safe Implementation of Treatments in Stroke-International Stroke Thrombolysis Register.", "A total of 45 079 patients treated with intravenous alteplase were recorded from 2002 to 2011. Main outcome measures were symptomatic intracerebral hemorrhage, functional independence (modified Rankin Scale score, 0-2), and mortality at 3 months.", "Among 25 777 (57.2%) men and 19 302 (42.8%) women, we found no difference in the rate of symptomatic intracerebral hemorrhage (P=0.13), a significantly higher likelihood of functional independence at 3 months in men (P<0.0001) and a higher mortality in women when compared with men (P<0.00001). After adjustment for confounding variables, we did not observe any difference between sexes in functional outcome (odds ratio, 1.03; 95% confidence interval, 0.97-1.09; P=0.39), whereas male sex was related to a higher risk of mortality (odds ratio, 1.19; 95% confidence interval, 1.10-1.29; P=0.00003) and symptomatic intracerebral hemorrhage (odds ratio, 1.25, 95% confidence interval, 1.04-1.51; P=0.02)." ], "LABELS": [ "BACKGROUND AND PURPOSE", "METHODS", "RESULTS" ], "MESHES": [ "Administration, Intravenous", "Aged", "Aged, 80 and over", "Body Weight", "Brain Ischemia", "Female", "Fibrinolytic Agents", "Humans", "Male", "Middle Aged", "Registries", "Sex Factors", "Stroke", "Thrombolytic Therapy", "Tissue Plasminogen Activator", "Treatment Outcome" ], "YEAR": "2013", "reasoning_required_pred": "no", "reasoning_free_pred": "yes", "final_decision": "yes", "LONG_ANSWER": "Data from Safe Implementation of Treatments in Stroke-International Stroke Thrombolysis Register suggest that intravenous thrombolysis may modify the observed survival and recovery advantage for men expected in the natural course of an ischemic stroke, with a possible larger beneficial treatment effect in women when compared with men." }, "10781708": { "QUESTION": "Thrombosis prophylaxis in hospitalised medical patients: does prophylaxis in all patients make sense?", "CONTEXTS": [ "Most studies on thrombosis prophylaxis focus on postoperative venous thrombosis. In medical wards thrombosis prophylaxis is generally restricted to patients who are immobilised. Our primary aim was to investigate the incidence of venous thrombosis in a general internal ward, to assess whether more rigorous prophylaxis would be feasible.", "We investigated the incidence of venous thrombosis in patients hospitalised from 1992 to 1996 and related our findings to literature reports.", "The incidence of symptomatic venous thrombosis in internal patients during hospitalisation was 39/6332 (0.6%). Among these 39 patients, 24 had a malignancy, whereas 876 out of all 6332 patients had a known malignancy. So, the incidence in this group with cancer was 2.7% compared with 0.3% (15/5456) in the non-cancer group (relative risk for venous thrombosis due to malignancy was 10.0 (95%C.I. 5.3-18.9)." ], "LABELS": [ "BACKGROUND", "METHODS", "RESULTS" ], "MESHES": [ "Adolescent", "Adult", "Aged", "Aged, 80 and over", "Enoxaparin", "Female", "Humans", "Incidence", "Male", "Neoplasms", "Postoperative Care", "Postoperative Complications", "Retrospective Studies", "Risk Factors", "Venous Thrombosis" ], "YEAR": "2000", "reasoning_required_pred": "yes", "reasoning_free_pred": "no", "final_decision": "no", "LONG_ANSWER": "The incidence of venous thrombosis during hospitalisation in a department of general internal medicine is low and does not justify prophylaxis in all internal patients. Cancer is a strong risk factor for hospital-acquired thrombosis in the medical ward. Further studies may answer the question as to whether thrombosis prophylaxis in this subgroup is feasible." }, "26561905": { "QUESTION": "Do teleoncology models of care enable safe delivery of chemotherapy in rural towns?", "CONTEXTS": [ "To compare the dose intensity and toxicity profiles for patients undergoing chemotherapy at the Townsville Cancer Centre (TCC), a tertiary cancer centre in northern Queensland, with those for patients treated in Mount Isa, supervised by the same medical oncologists via teleoncology.", "A quasi-experimental design comparing two patient groups.", "TCC and Mount Isa Hospital, which both operate under the auspices of the Townsville Teleoncology Network (TTN).", "Eligible patients who received chemotherapy at TCC or Mt Isa Hospital between 1 May 2007 and 30 April 2012.", "Teleoncology model for managing cancer patients in rural towns.", "Dose intensity (doses, number of cycles and lines of treatment) and toxicity rates (rate of serious side effects, hospital admissions and mortality).", "Over 5 years, 89 patients received a total of 626 cycles of various chemotherapy regimens in Mount Isa. During the same period, 117 patients who received a total of 799 cycles of chemotherapy at TCC were eligible for inclusion in the comparison group. There were no significant differences between the Mount Isa and TCC patients in most demographic characteristics, mean numbers of treatment cycles, dose intensities, proportions of side effects, and hospital admissions. There were no toxicity-related deaths in either group." ], "LABELS": [ "OBJECTIVES", "DESIGN", "SETTING", "PARTICIPANTS", "INTERVENTION", "MAIN OUTCOME MEASURES", "RESULTS" ], "MESHES": [ "Adolescent", "Adult", "Aged", "Aged, 80 and over", "Antineoplastic Agents", "Female", "Humans", "Male", "Medical Oncology", "Middle Aged", "Models, Organizational", "Neoplasms", "Oncology Service, Hospital", "Queensland", "Retrospective Studies", "Rural Health Services", "Rural Population", "Telemedicine", "Young Adult" ], "YEAR": "2015", "reasoning_required_pred": "yes", "reasoning_free_pred": "yes", "final_decision": "yes", "LONG_ANSWER": "It appears safe to administer chemotherapy in rural towns under the supervision of medical oncologists from larger centres via teleoncology, provided that rural health care resources and governance arrangements are adequate." }, "17089900": { "QUESTION": "Does Paget's disease exist in India?", "CONTEXTS": [ "Paget's disease of bone has been described as a few case reports from India. The aim of the present study is to document the existence of Paget's disease (PD) in India.", "We describe demography, clinical manifestations, biochemical and radiological profile and the treatment outcome of 21 patients of PD.", "Mean (+/-SD) age of these patients at presentation was 49.2 +/- 17.6 years and the male to female ratio was 2.5:1. Common clinical manifestations included backache, headache and bone pains. Others were fracture, joint pain, deafness, gait ataxia, visual impairment and difficulty in biting. Two patients presented with hydrocephalus and one had recurrent paraparesis. Fifteen (71.4%) patients had polyostotic and six (28.6%) had monoostotic Paget's disease. More commonly involved bones were skull and spine (61.9%) followed by pelvis (38.1%), femur (33.3%), tibia (9%) and ulna (9%). Mean (+/-SD) serum alkaline phosphatase at diagnosis was 1514 +/- 1168 IU/L and nine months after treatment with bisphosphonates decreased to 454 +/- 406 IU/ L(P<0.03)." ], "LABELS": [ "OBJECTIVE", "MATERIAL AND METHODS", "RESULTS" ], "MESHES": [ "Absorptiometry, Photon", "Adolescent", "Adult", "Age Distribution", "Aged", "Back Pain", "Bone Density Conservation Agents", "Child", "Diphosphonates", "Female", "Headache", "Health Surveys", "Hospitals", "Humans", "India", "Male", "Middle Aged", "Osteitis Deformans", "Risk Assessment", "Sex Distribution" ], "YEAR": "2006", "reasoning_required_pred": "yes", "reasoning_free_pred": "yes", "final_decision": "yes", "LONG_ANSWER": "This illustrates that Paget's disease does exist in India and a high index of suspicion is required to clinch the diagnosis." }, "26536001": { "QUESTION": "Is There an Additional Value of Using Somatostatin Receptor Subtype 2a Immunohistochemistry Compared to Somatostatin Receptor Scintigraphy Uptake in Predicting Gastroenteropancreatic Neuroendocrine Tumor Response?", "CONTEXTS": [ "It is unknown whether tumoral somatostatin receptor subtype 2a (sst2a) immunohistochemistry (IHC) has additional value compared to somatostatin receptor scintigraphy (SRS) uptake using OctreoScan\u00ae in predicting response to peptide receptor radiotherapy using 177Lu-octreotate (PRRT) in patients with gastroenteropancreatic neuroendocrine tumors (GEP-NETs). The aims of this study were: (1) to establish the percentage of sst2a immunopositivity in GEP-NET samples of PRRT-treated patients, (2) to determine the relationship between best GEP-NET response using RECIST 1.0 criteria 1 year after PRRT and tumoral sst2a IHC, and (3) to compare characteristics of patients with sst2a IHC-negative and -positive tumors.", "All 73 consecutive patients were selected for PRRT based on a positive SRS. Radiological response was scored according to RECIST 1.0 criteria. sst2a status was detected on tumor samples by IHC.", "In total, 93% of GEP-NET samples showed sst2a IHC positivity. No statistically significant relationship was observed between in vitro sst2a expression and in vivo best GEP-NET response 1 year after PRRT (p = 0.47). Sex, primary tumor site, disease stage, ENETS TNM classification, Ki-67 index, highest serum chromogranin-A level, and highest neuron-specific enolase level were not significantly different between patients with negative and positive sst2a tumoral IHC with the exception of age at diagnosis (p = 0.007)." ], "LABELS": [ "BACKGROUND AND AIMS", "METHODS", "RESULTS" ], "MESHES": [ "Adult", "Aged", "Antineoplastic Agents", "Female", "Gene Expression Regulation, Neoplastic", "Humans", "Immunohistochemistry", "Intestinal Neoplasms", "Male", "Middle Aged", "Neuroendocrine Tumors", "Octreotide", "Pancreatic Neoplasms", "Radionuclide Imaging", "Receptors, Somatostatin", "Stomach Neoplasms", "Treatment Outcome" ], "YEAR": "2016", "reasoning_required_pred": "no", "reasoning_free_pred": "no", "final_decision": "no", "LONG_ANSWER": "sst2a IHC of tumor samples has no additional value compared to SRS uptake using OctreoScan\u00ae in predicting tumor response after PRRT." }, "19643525": { "QUESTION": "Can vaginal pH be measured from the wet mount slide?", "CONTEXTS": [ "To assess the accuracy of vaginal pH measurement on wet mount microscopy slides compared with direct measurements on fresh vaginal fluid. We also tested whether differences in accuracy were dependent on the sampling devices used or on the diagnosis of the vaginal infections.", "Using a cotton swab, cytobrush or wooden spatula a vaginal fluid specimen was collected from 84 consecutive women attending a vulvo-vaginitis clinic. A pH strip (pH range 4-7, Merck) was brought in contact with the vaginal fluid on the sampling device and on the glass slide after adding one droplet of saline and performing microscopy by two different people unaware of the microscopy results of the clinical exam. Values were compared by Fisher exact and Student's t-tests.", "pH measurement from microscopy slides after the addition of saline causes systematic increases of pH leading to false positive readings. This is true for all types of disturbance of the flora and infections studied, and was seen in the abnormal as well as in the normal or intermediate pH range." ], "LABELS": [ "OBJECTIVES", "STUDY DESIGN", "RESULTS" ], "MESHES": [ "Adolescent", "Adult", "Aged", "Body Fluids", "False Positive Reactions", "Female", "Humans", "Hydrogen-Ion Concentration", "Microscopy", "Middle Aged", "Sodium Chloride", "Vagina", "Vaginal Smears", "Vaginitis" ], "YEAR": "2009", "reasoning_required_pred": "no", "reasoning_free_pred": "yes", "final_decision": "no", "LONG_ANSWER": "Vaginal pH should be measured by bringing the pH strip in direct contact with fresh vaginal fluid without first adding saline." }, "26285789": { "QUESTION": "Does Molecular Genotype Provide Useful Information in the Management of Radioiodine Refractory Thyroid Cancers?", "CONTEXTS": [ "Whether mutation status should be used to guide therapy is an important issue in many cancers. We correlated mutation profile in radioiodine-refractory (RAIR) metastatic thyroid cancers (TCs) with patient outcome and response to tyrosine kinase inhibitors (TKIs), and discussed the results with other published data.", "Outcome in 82 consecutive patients with metastatic RAIR thyroid carcinoma prospectively tested for BRAF, RAS and PI3KCA mutations was retrospectively analyzed, including 55 patients treated with multikinase inhibitors.", "Papillary thyroid carcinomas (PTCs) were the most frequent histological subtype (54.9 %), followed by poorly differentiated thyroid carcinoma [PDTC] (30.5 %) and follicular thyroid carcinoma [FTC](14.6 %). A genetic mutation was identified in 23 patients (28 %) and BRAF was the most frequently mutated gene (23 %). Median progression-free survival (PFS) on first-line TKI treatment was 14.6 months (95% CI 9.9-18.4). BRAF mutation positively influenced median PFS, both in the entire TKI-treated cohort (median PFS 34.7 months versus 11.6 months; hazard ratio [HR] 0.29; 95% CI 0.09-0.98; p\u2009=\u20090.03) and in the TKI-treated PTC cohort (n\u2009=\u200922) [log-rank p\u2009=\u20090.086; HR 2.95; 95 % CI 0.81-10.70). However, in TKI-treated patients, PDTC histologic subtype was the only independent prognostic factor for PFS identified in the multivariate analysis (HR 2.36; 95% CI 1.01-5.54; p\u2009=\u20090.048)." ], "LABELS": [ "INTRODUCTION", "MATERIALS AND METHODS", "RESULTS" ], "MESHES": [ "Adenocarcinoma", "Adult", "Aged", "Aged, 80 and over", "Biomarkers, Tumor", "Carcinoma, Papillary", "Disease Management", "Female", "Follow-Up Studies", "Genotype", "Humans", "Iodine Radioisotopes", "Male", "Middle Aged", "Molecular Targeted Therapy", "Mutation", "Neoplasm Staging", "Prognosis", "Prospective Studies", "Radiation Tolerance", "Retrospective Studies", "Survival Rate", "Thyroid Neoplasms" ], "YEAR": "2016", "reasoning_required_pred": "yes", "reasoning_free_pred": "yes", "final_decision": "yes", "LONG_ANSWER": "Patients with BRAF-mutant PTC had a significantly longer PFS than BRAF wild-type when treated with TKIs. However, due to the small number of BRAF-mutant patients, further investigations are required, especially to understand the potential positive effect of BRAF mutations in RAIR TC patients while having a negative prognostic impact in RAI-sensitive PTC patients." }, "22266735": { "QUESTION": "Screening for gestational diabetes mellitus: are the criteria proposed by the international association of the Diabetes and Pregnancy Study Groups cost-effective?", "CONTEXTS": [ "The International Association of the Diabetes and Pregnancy Study Groups (IADPSG) recently recommended new criteria for diagnosing gestational diabetes mellitus (GDM). This study was undertaken to determine whether adopting the IADPSG criteria would be cost-effective, compared with the current standard of care.", "We developed a decision analysis model comparing the cost-utility of three strategies to identify GDM: 1) no screening, 2) current screening practice (1-h 50-g glucose challenge test between 24 and 28 weeks followed by 3-h 100-g glucose tolerance test when indicated), or 3) screening practice proposed by the IADPSG. Assumptions included that 1) women diagnosed with GDM received additional prenatal monitoring, mitigating the risks of preeclampsia, shoulder dystocia, and birth injury; and 2) GDM women had opportunity for intensive postdelivery counseling and behavior modification to reduce future diabetes risks. The primary outcome measure was the incremental cost-effectiveness ratio (ICER).", "Our model demonstrates that the IADPSG recommendations are cost-effective only when postdelivery care reduces diabetes incidence. For every 100,000 women screened, 6,178 quality-adjusted life-years (QALYs) are gained, at a cost of $125,633,826. The ICER for the IADPSG strategy compared with the current standard was $20,336 per QALY gained. When postdelivery care was not accomplished, the IADPSG strategy was no longer cost-effective. These results were robust in sensitivity analyses." ], "LABELS": [ "OBJECTIVE", "RESEARCH DESIGN AND METHODS", "RESULTS" ], "MESHES": [ "Cost-Benefit Analysis", "Diabetes, Gestational", "Female", "Glucose Tolerance Test", "Humans", "Mass Screening", "Pregnancy", "Quality-Adjusted Life Years" ], "YEAR": "2012", "reasoning_required_pred": "maybe", "reasoning_free_pred": "yes", "final_decision": "yes", "LONG_ANSWER": "The IADPSG recommendation for glucose screening in pregnancy is cost-effective. The model is most sensitive to the likelihood of preventing future diabetes in patients identified with GDM using postdelivery counseling and intervention." }, "15052394": { "QUESTION": "Are higher rates of depression in women accounted for by differential symptom reporting?", "CONTEXTS": [ "The gender difference in prevalence and incidence rates of depression is one of the most consistent findings in psychiatric epidemiology. We sought to examine whether any gender differences in symptom profile might account for this difference in rates.", "This study was a population-based 13-year follow-up survey of community-dwelling adults living in East Baltimore in 1981. Subjects were the continuing participants of the Baltimore Epidemiologic Catchment Area Program. Participants interviewed between 1993 and 1996 with complete data on depressive symptoms and covariates were included (n = 1727). We applied structural equations with a measurement model for dichotomous data (the MIMIC-multiple indicators, multiple causes-model) to compare symptoms between women and men, in relation to the nine symptom groups comprising the diagnostic criteria for major depression, adjusting for several potentially influential characteristics (namely, age, self-reported ethnicity, educational attainment, marital status, and employment).", "There were no significant gender differences in the self-report of depression symptoms even taking into account the higher level of depressive symptoms of women and the influence of other covariates. For example, women were no more likely to endorse sadness than were men, as evidenced by a direct effect coefficient that was not significantly different from the null [adjusted estimated direct effect of gender on report of sadness = 0.105, 95% confidence interval (-0.113, 0.323)]." ], "LABELS": [ "BACKGROUND", "METHOD", "RESULTS" ], "MESHES": [ "Adult", "Aged", "Aged, 80 and over", "Baltimore", "Catchment Area (Health)", "Depressive Disorder, Major", "Female", "Follow-Up Studies", "Humans", "Incidence", "Male", "Middle Aged", "Prevalence", "Psychometrics", "Risk Factors", "Self Disclosure", "Sex Factors", "Socioeconomic Factors", "Women's Health" ], "YEAR": "2004", "reasoning_required_pred": "no", "reasoning_free_pred": "no", "final_decision": "no", "LONG_ANSWER": "Men and women in this community sample reported similar patterns of depressive symptoms. No evidence that the presentation of depressive symptoms differs by gender was found." }, "23264436": { "QUESTION": "Follow-up of patients with new cardiovascular implantable electronic devices: are experts' recommendations implemented in routine clinical practice?", "CONTEXTS": [ "A 2008 expert consensus statement outlined the minimum frequency of follow-up of patients with cardiovascular implantable electronic devices (CIEDs).", "We studied 38 055 Medicare beneficiaries who received a new CIED between January 1, 2005, and June 30, 2009. The main outcome measure was variation of follow-up by patient factors and year of device implantation. We determined the number of patients who were eligible for and attended an in-person CIED follow-up visit within 2 to 12 weeks, 0 to 16 weeks, and 1 year after implantation. Among eligible patients, 42.4% had an initial in-person visit within 2 to 12 weeks. This visit was significantly more common among white patients than black patients and patients of other races (43.0% versus 36.8% versus 40.5%; P<0.001). Follow-up within 2 to 12 weeks improved from 40.3% in 2005 to 55.1% in 2009 (P<0.001 for trend). The rate of follow-up within 0 to 16 weeks was 65.1% and improved considerably from 2005 to 2009 (62.3%-79.6%; P<0.001 for trend). Within 1 year, 78.0% of the overall population had at least 1 in-person CIED follow-up visit." ], "LABELS": [ "BACKGROUND", "METHODS AND RESULTS" ], "MESHES": [ "African Americans", "Aftercare", "Aged", "Aged, 80 and over", "Cardiac Resynchronization Therapy", "Cardiac Resynchronization Therapy Devices", "Chi-Square Distribution", "Continuity of Patient Care", "Defibrillators, Implantable", "Electric Countershock", "Equipment Design", "European Continental Ancestry Group", "Female", "Follow-Up Studies", "Guideline Adherence", "Humans", "Male", "Medicare", "Practice Guidelines as Topic", "Practice Patterns, Physicians'", "Time Factors", "Treatment Outcome", "United States" ], "YEAR": "2013", "reasoning_required_pred": "yes", "reasoning_free_pred": "yes", "final_decision": "yes", "LONG_ANSWER": "Although most Medicare beneficiaries who received a new CIED between 2005 and 2009 did not have an initial in-person CIED follow-up visit within 2 to 12 weeks after device implantation, the rate of initial follow-up improved appreciably over time. This CIED follow-up visit was significantly more common in white patients than in patients of other races." }, "22302658": { "QUESTION": "Does limb-salvage surgery offer patients better quality of life and functional capacity than amputation?", "CONTEXTS": [ "Patients with aggressive lower extremity musculoskeletal tumors may be candidates for either above-knee amputation or limb-salvage surgery. However, the subjective and objective benefits of limb-salvage surgery compared with amputation are not fully clear.QUESTIONS/", "We therefore compared functional status and quality of life for patients treated with above-knee amputation versus limb-salvage surgery.", "We reviewed 20 of 51 patients aged 15 years and older treated with above-knee amputation or limb-salvage surgery for aggressive musculoskeletal tumors around the knee between 1994 and 2004 as a retrospective cohort study. At last followup we obtained the Physiological Cost Index, the Reintegration to Normal Living Index, SF-36, and the Toronto Extremity Salvage Score questionnaires. The minimum followup was 12 months (median, 56 months; range, 12-108 months).", "Compared with patients having above-knee amputation, patients undergoing limb-salvage surgery had superior Physiological Cost Index scores and Reintegration to Normal Living Index. The Toronto Extremity Salvage scores and SF-36 scores were similar in the two groups." ], "LABELS": [ "INTRODUCTION", "PURPOSES", "METHODS", "RESULTS" ], "MESHES": [ "Activities of Daily Living", "Adolescent", "Adult", "Aged", "Amputation", "Amputees", "Artificial Limbs", "Bone Neoplasms", "Female", "Humans", "Limb Salvage", "Lower Extremity", "Male", "Middle Aged", "Patient Selection", "Perception", "Prosthesis Fitting", "Quality of Life", "Recovery of Function", "Retrospective Studies", "Soft Tissue Neoplasms", "Surveys and Questionnaires", "Texas", "Time Factors", "Treatment Outcome", "Young Adult" ], "YEAR": "2012", "reasoning_required_pred": "yes", "reasoning_free_pred": "maybe", "final_decision": "maybe", "LONG_ANSWER": "These data suggest that limb-salvage surgery offers better gait efficiency and return to normal living compared with above-knee amputation, but does not improve the patient's perception of quality of life." }, "17894828": { "QUESTION": "Serum angiotensin-converting enzyme and frequency of severe hypoglycaemia in Type 1 diabetes: does a relationship exist?", "CONTEXTS": [ "An association has been described between elevated serum angiotensin-converting enzyme (ACE) and an increased risk of severe hypoglycaemia (SH). To ascertain whether this reported association could be replicated in a different country, it was re-examined in 300 individuals with Type 1 diabetes.", "People with Type 1 diabetes, none of whom was taking renin-angiotensin system blocking drugs, were recruited. Participants recorded the frequency with which they had experienced SH. Glycated haemoglobin (HbA(1c)) and serum ACE were measured. The difference in the incidence of SH between different quartiles of ACE activity and the relationship between serum ACE and SH were examined using non-parametric statistical tests and a negative binomial model.", "Data were obtained from 300 patients [158 male; HbA(1c) median (range) 8.2% (5.2-12.8%), median age 36 years (16-88); duration of diabetes 14.5 years (2-49)]. The incidence of SH was 0.93 episodes per patient year. The mean incidence of SH in the top and bottom quartiles of ACE activity was 0.5 and 1.7 episodes per patient year, respectively, but this difference was not statistically significant (P = 0.075). Spearman's test showed a very weak, although statistically significant, association between serum ACE level and SH incidence (r = 0.115, P = 0.047). The binomial model also showed a statistically significant (P = 0.002), but clinically weak, relationship between serum ACE and SH." ], "LABELS": [ "AIMS", "METHODS", "RESULTS" ], "MESHES": [ "Adolescent", "Adult", "Aged", "Aged, 80 and over", "Chromatography, High Pressure Liquid", "Diabetes Mellitus, Type 1", "Female", "Glycated Hemoglobin A", "Humans", "Hypoglycemia", "Incidence", "Male", "Middle Aged", "Peptidyl-Dipeptidase A", "Spectrophotometry" ], "YEAR": "2007", "reasoning_required_pred": "yes", "reasoning_free_pred": "yes", "final_decision": "yes", "LONG_ANSWER": "The present survey showed a weak relationship between serum ACE and the frequency of SH, the clinical relevance of which is unclear. This limits the proposed role for serum ACE as an index of risk for SH." }, "18096128": { "QUESTION": "Cervical spine fractures in geriatric blunt trauma patients with low-energy mechanism: are clinical predictors adequate?", "CONTEXTS": [ "Studies have identified clinical predictors to guide radiologic evaluation of the cervical spine in geriatric patients. We hypothesized that clinical predictors are not adequate in the identification of cervical spine fractures in geriatric blunt trauma patients with low-energy mechanism.", "A retrospective case-control study was performed on geriatric blunt trauma patients sustaining low-energy trauma from January 2000 to January 2006. A data form including 8 clinical predictors was completed for each group.", "There were 35 study and 64 control patients identified. Both groups were similar in age (study 83.6 vs control 81.2) and injury severity score (study 9.06 vs control 9.61). Only neck tenderness exceeded the expected occurrence in the presence of a cervical spine injury (chi(2) = 18.1, P = .001) in just 45.5% of the study group." ], "LABELS": [ "BACKGROUND", "METHODS", "RESULTS" ], "MESHES": [ "Accidental Falls", "Age Factors", "Aged", "Aged, 80 and over", "Case-Control Studies", "Cervical Vertebrae", "Female", "Geriatric Assessment", "Humans", "Incidence", "Injury Severity Score", "Male", "Physical Examination", "Predictive Value of Tests", "Probability", "Retrospective Studies", "Sensitivity and Specificity", "Spinal Fractures", "Tomography, X-Ray Computed", "Wounds, Nonpenetrating" ], "YEAR": "2008", "reasoning_required_pred": "no", "reasoning_free_pred": "no", "final_decision": "no", "LONG_ANSWER": "Clinical predictors appear inadequate for the evaluation of the cervical spine in geriatric trauma patients with low-energy mechanism." }, "12913347": { "QUESTION": "Do microbiological factors account for poor pregnancy outcome among unmarried pregnant women in Poland?", "CONTEXTS": [ "Being unmarried is a well-known risk factor for poor pregnancy outcome such as preterm delivery and intrauterine growth restriction. The aim of this prospective study was to assess the prevalence and risk of bacterial vaginosis (BV) and selected bacteria isolated from the lower genital tract and to determine the socioeconomic and microbiological characteristics that might be responsible for poor pregnancy outcome observed among unmarried pregnant women.", "The study population comprised 196 pregnant women attending 10 randomly selected outpatient maternity units in the Lodz region, central Poland. Cervicovaginal samples were obtained between 8 and 16 weeks of gestation. Based on Spiegel's criteria, gram-stained vaginal smears were examined for BV and the BV-associated flora was sought by culture. To evaluate the risk factors, relative risk ratios were calculated using EPI INFO software.", "Among 196 pregnant women, 40 (20.4%) were unmarried. BV was diagnosed among 55 (28.1%) women studied. In the univariate analysis, unmarried pregnant women were characterized by younger age, primary educational level, poor economic situation and excessive smoking during pregnancy, as compared to married women. The unmarried status was a borderline risk factor for BV (OR = 1.83, 95% CI 0.94-4.9) after adjustment for age, smoking and education. An analysis of the microbiological culture from the lower genital tract revealed that unmarried pregnant women had a higher risk for several types of pathological microflora, as compared to married women. However, this finding was significant only for Mycoplasma hominis. The independent risk factors of M. hominis were the young age of the subject and a low concentration of Lactobacillus spp." ], "LABELS": [ "OBJECTIVE", "METHODS", "RESULTS" ], "MESHES": [ "Adult", "Female", "Humans", "Lactobacillus", "Mycoplasma Infections", "Mycoplasma hominis", "Poland", "Pregnancy", "Pregnancy Complications, Infectious", "Pregnancy Outcome", "Prevalence", "Prospective Studies", "Random Allocation", "Risk Factors", "Single Person", "Vaginosis, Bacterial" ], "YEAR": null, "reasoning_required_pred": "yes", "reasoning_free_pred": "yes", "final_decision": "yes", "LONG_ANSWER": "The observed socioeconomic, demographic and microbiological differences between unmarried and married women could be responsible for the poor pregnancy outcome among unmarried pregnant women in Poland. Unmarried pregnant women should be covered by comprehensive medical care even before pregnancy. Further studies taking into account the role of psychological stress, patterns of sexual behavior and substance abuse during pregnancy could help identify the factors responsible for adverse pregnancy outcome among unmarried pregnant women." }, "24684514": { "QUESTION": "Should all patients be optimized to the same preoperative hemoglobin level to avoid transfusion in primary knee arthroplasty?", "CONTEXTS": [ "Optimization of the preoperative hemoglobin (Hb) level is an effective way to reduce allogeneic transfusion in total knee arthroplasty (TKA) though the procedure is expensive, requires close monitoring and is often inconvenient for patients with reduced mobility. Our aim was to investigate the value of preoperative Hb levels to predict transfusion and thereby tailoring Hb optimization to patient characteristics.", "All consecutive patients who undergone primary TKA in our center over 2\u00a0years, and received tranexamic acid intraoperatively, were reviewed. The adjusted association between preoperative Hb levels and transfusion was assessed by multivariate logistic regression, and the estimated probability of transfusion for individual patients was derived from the logistic model.", "Out of the 784 patients who meet the inclusion criteria, risk of transfusion was associated with poorer performance status, as measured by the America Association of Anestesiology (ASA) score III/IV (OR: 3\u00b73, P\u00a0<\u00a00\u00b7001) and lower preoperative Hb level (OR 3\u00b78 for each g/dl below 13\u00a0g/dl; P\u00a0<\u00a00\u00b7001). According to the Hb level, the estimated probability of transfusion was 0\u00b703 (range: 0\u00b703-0\u00b764) for ASA I/II patients and 0\u00b710 (range: 0\u00b710-0\u00b784) for ASA III/IV." ], "LABELS": [ "BACKGROUND AND OBJECTIVE", "MATERIALS AND METHODS", "RESULTS" ], "MESHES": [ "Adult", "Aged", "Aged, 80 and over", "Antifibrinolytic Agents", "Arthroplasty, Replacement, Knee", "Blood Transfusion", "Female", "Hemoglobins", "Humans", "Joint Diseases", "Male", "Middle Aged", "Practice Guidelines as Topic", "Preoperative Care", "Retrospective Studies", "Risk", "Tranexamic Acid" ], "YEAR": "2014", "reasoning_required_pred": "no", "reasoning_free_pred": "no", "final_decision": "no", "LONG_ANSWER": "Not all the patients undergoing TKA who receive tranexamic acid need the same preoperative Hb optimization target. Two easily available factors, such as the ASA score and the Hb level, can help individualize the Hb optimization target." }, "24793469": { "QUESTION": "Is there any relation between cervical cord plaques and discopathy in patients with multiple sclerosis?", "CONTEXTS": [ "Multiple sclerosis (MS) is the most common chronic autoimmune demyelinating disease of the central nervous system. The purpose of this study is to determine the relationship between the site of the cervical discopathy and cervical spinal cord plaque in MS patients.", "This retrospective study included all patients with a definite diagnosis of MS who were treated at an outpatient clinic between September 2004 and September 2011. All patients underwent cervical magnetic resonance imaging (MRI) for primary investigation of the disease. Cervical MRI scans were evaluated for detection of any evidence of cervical discopathy and cervical MS plaques. Any correlation between the site of the MS lesions and discopathy was recorded.", "From 536 patients who were involved in the study, 214 patients had both cervical discopathy and cervical cord plaques. In this group 148 (69.1% of patients) had cervical plaque at the same site of cervical discopathy. The number of patients with cervical cord plaque and discopathy at same site was significantly higher than those with plaque and discopathy at different sites (P<0.05)." ], "LABELS": [ "INTRODUCTION", "METHODS", "RESULTS" ], "MESHES": [ "Adolescent", "Adult", "Cervical Cord", "Female", "Humans", "Magnetic Resonance Imaging", "Male", "Middle Aged", "Multiple Sclerosis", "Retrospective Studies", "Young Adult" ], "YEAR": "2014", "reasoning_required_pred": "yes", "reasoning_free_pred": "yes", "final_decision": "yes", "LONG_ANSWER": "The study data suggests a possible correlation between cervical discopathy and cervical MS plaque." }, "23360491": { "QUESTION": "Is the processing of affective prosody influenced by spatial attention?", "CONTEXTS": [ "The present study asked whether the processing of affective prosody is modulated by spatial attention. Pseudo-words with a neutral, happy, threatening, and fearful prosody were presented at two spatial positions. Participants attended to one position in order to detect infrequent targets. Emotional prosody was task irrelevant. The electro-encephalogram (EEG) was recorded to assess processing differences as a function of spatial attention and emotional valence.", "Event-related potentials (ERPs) differed as a function of emotional prosody both when attended and when unattended. While emotional prosody effects interacted with effects of spatial attention at early processing levels (<200 ms), these effects were additive at later processing stages (>200 ms)." ], "LABELS": [ "BACKGROUND", "RESULTS" ], "MESHES": [ "Acoustic Stimulation", "Adult", "Affect", "Analysis of Variance", "Attention", "Brain Mapping", "Electroencephalography", "Evoked Potentials", "Female", "Humans", "Male", "Reaction Time", "Speech Perception", "Time Factors", "Vocabulary", "Young Adult" ], "YEAR": "2013", "reasoning_required_pred": "no", "reasoning_free_pred": "maybe", "final_decision": "maybe", "LONG_ANSWER": "Emotional prosody, therefore, seems to be partially processed outside the focus of spatial attention. Whereas at early sensory processing stages spatial attention modulates the degree of emotional voice processing as a function of emotional valence, emotional prosody is processed outside of the focus of spatial attention at later processing stages." }, "10340286": { "QUESTION": "Is there a role for leukocyte and CRP measurements in the diagnosis of acute appendicitis in the elderly?", "CONTEXTS": [ "The diagnosis of acute appendicitis is still difficult and the results are unsatisfactory in three particular patient groups: in children, in fertile-age women and in elderly patients. As our population ages, the challenge for expedient diagnosis and intervention in older age groups will become more and more significant. The present study aimed at clarifying the role of leukocyte count and C-reactive protein (CRP) measurements in the diagnosis of acute appendicitis in the elderly. In particular, are there patients with acute appendicitis but unelevated leukocyte count and CRP?", "Eighty-three consecutive elderly patients underwent appendectomy for suspected acute appendicitis. The mean leukocyte count and CRP value were calculated in patients with an uninflamed appendix (group A) and in those with acute appendicitis (group B). The percentages of patients with: (1) both values unelevated; (2) only leukocyte count elevated; (3) only CRP value elevated; (4) both values elevated were calculated within the groups A and B.", "There was no statistically significant difference in leukocyte counts or CRP values between patients with an uninflamed appendix (group A) and those with acute appendicitis (group B). When the patients were divided into the four subgroups, the most conspicuous finding was that group B (acute appendicitis, n = 73) contained no patients with both values unelevated." ], "LABELS": [ "OBJECTIVES", "METHODS", "RESULTS" ], "MESHES": [ "Acute Disease", "Aged", "Appendectomy", "Appendicitis", "C-Reactive Protein", "Case-Control Studies", "Female", "Humans", "Leukocyte Count", "Male" ], "YEAR": "1999", "reasoning_required_pred": "yes", "reasoning_free_pred": "maybe", "final_decision": "maybe", "LONG_ANSWER": "Although elevated leukocyte count and CRP value cannot effectively establish the diagnosis of acute appendicitis in the elderly, unelevated values exclude it. Accordingly, appendectomy is not recommended to be performed in an elderly patient with unelevated leukocyte count and CRP value, although clinical symptoms and signs indicate acute appendicitis." }, "22954812": { "QUESTION": "Are bipolar disorders underdiagnosed in patients with depressive episodes?", "CONTEXTS": [ "Recent reports indicate that the prevalence of bipolar disorder (BD) in patients with an acute major depressive episode might be higher than previously thought. We aimed to study systematically all patients who sought therapy for major depressive episode (MDE) within the BRIDGE study in Germany, reporting on an increased number (increased from 2 in the international BRIDGE report to 5) of different diagnostic algorithms.", "A total of 252 patients with acute MDE (DSM-IV confirmed) were examined for the existence of BD (a) according to DSM-IV criteria, (b) according to modified DSM-IV criteria (without the exclusion criterion of 'mania not induced by substances/antidepressants'), (c) according to a Bipolarity Specifier Algorithm which expands the DSM-IV criteria, (d) according to HCL-32R (Hypomania-Checklist-32R), and (e) according to a criteria-free physician's diagnosis.", "The five different diagnostic approaches yielded immensely variable prevalences for BD: (a) 11.6; (b) 24.8%; (c) 40.6%; (d) 58.7; e) 18.4% with only partial overlap between diagnoses according to the physician's diagnosis or HCL-32R with diagnoses according to the three DSM-based algorithms." ], "LABELS": [ "BACKGROUND", "METHODS", "RESULTS" ], "MESHES": [ "Adult", "Aged", "Aged, 80 and over", "Algorithms", "Antidepressive Agents", "Bipolar Disorder", "Checklist", "Comorbidity", "Cross-Sectional Studies", "Depressive Disorder, Major", "Diagnosis, Differential", "Diagnostic Errors", "Diagnostic and Statistical Manual of Mental Disorders", "Female", "Germany", "Humans", "Male", "Middle Aged", "Prevalence", "Young Adult" ], "YEAR": "2012", "reasoning_required_pred": "maybe", "reasoning_free_pred": "maybe", "final_decision": "maybe", "LONG_ANSWER": "The diagnosis of BD in patients with MDE depends strongly on the method and criteria employed. The considerable difference between criteria-free physician's diagnosis and the remaining algorithms indicate the usefulness of criteria lists within the everyday clinical setting." }, "18274917": { "QUESTION": "Prognosis of low-tone sudden deafness - does it inevitably progress to Meniere's disease?", "CONTEXTS": [ "To investigate whether low-tone SD was a precursor of Meniere's disease and whether patients with low-tone SD suffered from endolymphatic hydrops.", "This was a retrospective case review in the university hospital. A total of 184 patients with low-tone SD were divided into two groups with single and recurrent episodes. The progress, follow-up audiograms, and ECochG results of the patients were reviewed and compared with those of patients with high-tone SD and Meniere's disease.", "In all, 83 of 177 patients with low-tone SD unaccompanied by vertigo had recurrent hearing loss; 15 of the 83 developed vertiginous attacks. The remaining 94 patients had a single episode. Three of the seven patients with low-tone SD accompanied by vertigo had recurrent hearing loss; two of the three were subsequently confirmed to have Meniere's disease. The other four had a single episode. No difference in rate of progress from SD to Meniere's disease was observed among the low-tone and the high-tone SD groups. The average -SP/AP of each group with a single episode is smaller than that of other groups with recurrent episodes and Meniere's disease." ], "LABELS": [ "OBJECTIVES", "PATIENTS AND METHODS", "RESULTS" ], "MESHES": [ "Audiometry, Evoked Response", "Audiometry, Pure-Tone", "Auditory Threshold", "Disease Progression", "Endolymphatic Hydrops", "Female", "Follow-Up Studies", "Hearing Loss, Sudden", "Humans", "Male", "Meniere Disease", "Pitch Discrimination", "Recurrence", "Retrospective Studies" ], "YEAR": "2008", "reasoning_required_pred": "no", "reasoning_free_pred": "no", "final_decision": "no", "LONG_ANSWER": "We conclude that not all low-tone sudden deafness (SD) patients suffered from endolymphatic hydrops even if they had vertigo attack at the onset and that electrocochleography (ECochG) was a useful prognostic tool." }, "22350859": { "QUESTION": "Can pictorial warning labels on cigarette packages address smoking-related health disparities?", "CONTEXTS": [ "The objective of this study was to determine the most effective content of pictorial health warning labels (HWLs) and whether educational attainment moderates these effects.", "Field experiments were conducted with 529 adult smokers and 530 young adults (258 nonsmokers; 271 smokers). Participants reported responses to different pictorial HWLs printed on cigarette packages. One experiment involved manipulating textual form (testimonial narrative vs. didactic) and the other involved manipulating image type (diseased organs vs. human suffering).", "Tests of mean ratings and rankings indicated that pictorial HWLs with didactic textual forms had equivalent or significantly higher credibility, relevance, and impact than pictorial HWLs with testimonial forms. Results from mixed-effects models confirmed these results. However, responses differed by participant educational attainment: didactic forms were consistently rated higher than testimonials among participants with higher education, whereas the difference between didactic and testimonial narrative forms was weaker or not statistically significant among participants with lower education. In the second experiment, with textual content held constant, greater credibility, relevance, and impact was found for graphic imagery of diseased organs than imagery of human suffering." ], "LABELS": [ "OBJECTIVE", "METHODS", "RESULTS" ], "MESHES": [ "Adolescent", "Adult", "Aged", "Aged, 80 and over", "Educational Status", "Female", "Healthcare Disparities", "Humans", "Male", "Mexico", "Middle Aged", "Product Labeling", "Smoking", "Smoking Prevention", "Young Adult" ], "YEAR": "2012", "reasoning_required_pred": "yes", "reasoning_free_pred": "yes", "final_decision": "yes", "LONG_ANSWER": "Pictorial HWLs with didactic textual forms seem to work better than those with testimonial narratives. Future research should determine which pictorial HWL content has the greatest real-world impact among consumers from disadvantaged groups, including assessment of how HWL content should change to maintain its impact as tobacco control environments strengthen and consumer awareness of smoking-related risks increases." }, "26237424": { "QUESTION": "Does patient-prosthesis mismatch after aortic valve replacement affect survival and quality of life in elderly patients?", "CONTEXTS": [ "To evaluate the impact of patient-prosthesis mismatch (PPM) on survival, functional status, and quality of life (QoL) after aortic valve replacement (AVR) with small prosthesis size in elderly patients.", "Between January 2005 and December 2013, 152 patients with pure aortic stenosis, aged at least 75 years, underwent AVR, with a 19 or 21\u200amm prosthetic heart valve. PPM was defined as an indexed effective orifice area less than 0.85\u200acm/m. Median age was 82 years (range 75-93 years). Mean follow-up was 56 months (range 1-82 months) and was 98% complete. Late survival rate, New York Heart Association functional class, and QoL (RAND SF-36) were assessed.", "Overall, PPM was found in 78 patients (53.8%). Among them, 42 patients (29%) had an indexed effective orifice area less than 0.75\u200acm/m and 17 less than 0.65\u200acm/m (11.7%). Overall survival at 5 years was 78\u200a\u00b1\u200a4.5% and was not influenced by PPM (P\u200a=\u200aNS). The mean New York Heart Association class for long-term survivors with PPM improved from 3.0 to 1.7 (P\u200a<\u200a0.001). QoL (physical functioning 45.18\u200a\u00b1\u200a11.35, energy/fatigue 49.36\u200a\u00b1\u200a8.64, emotional well being 58.84\u200a\u00b1\u200a15.44, social functioning 61.29\u200a\u00b1\u200a6.15) was similar to that of no-PPM patients (P\u200a=\u200aNS)." ], "LABELS": [ "BACKGROUND", "METHODS", "RESULTS" ], "MESHES": [ "Aged", "Aged, 80 and over", "Aortic Valve", "Aortic Valve Stenosis", "Female", "Follow-Up Studies", "Heart Valve Prosthesis", "Heart Valve Prosthesis Implantation", "Humans", "Italy", "Male", "Quality of Life", "Ventricular Remodeling" ], "YEAR": "2016", "reasoning_required_pred": "no", "reasoning_free_pred": "no", "final_decision": "no", "LONG_ANSWER": "PPM after AVR does not affect survival, functional status, and QoL in patients aged at least 75 years. Surgical procedures, often time-consuming, contemplated to prevent PPM, may therefore be not justified in this patient subgroup." }, "15041506": { "QUESTION": "Is fear of anaphylactic shock discouraging surgeons from more widely adopting percutaneous and laparoscopic techniques in the treatment of liver hydatid cyst?", "CONTEXTS": [ "Sources of reports about laparoscopic and percutaneous treatment of liver hydatid cysts are limited to just a few countries. To address the reason behind this, we carried out a survey of 30 surgeons in northern Jordan.", "A questionnaire was distributed to collect data regarding the surgical technique preferred by each surgeon. Further information was collected from those not adopting minimal-access techniques to determine their reasons for not doing so.", "Only 3 surgeons (10%) considered laparoscopy as the first line of treatment. Of the 27 surgeons who did not consider percutaneous or laparoscopic treatment, fear of anaphylaxis and/or dissemination was the main reason given by 21 surgeons (78%) for not using minimal access techniques." ], "LABELS": [ "BACKGROUND", "METHODS", "RESULTS" ], "MESHES": [ "Adult", "Anaphylaxis", "Echinococcosis, Hepatic", "General Surgery", "Humans", "Laparoscopy", "Middle Aged", "Postoperative Complications", "Practice Patterns, Physicians'", "Surveys and Questionnaires" ], "YEAR": "2004", "reasoning_required_pred": "yes", "reasoning_free_pred": "yes", "final_decision": "yes", "LONG_ANSWER": "The seemingly exaggerated traditional fear of anaphylaxis seems to discourage surgeons from more widely adopting minimal access techniques for the treatment of hydatid cyst." }, "25280365": { "QUESTION": "Reporting and interpreting red blood cell morphology: is there discordance between clinical pathologists and clinicians?", "CONTEXTS": [ "Clinical pathologists (CPs) report RBC morphologic (RBC-M) changes to assist clinicians in prioritizing differential diagnoses. However, reporting is subjective, semiquantitative, and potentially biased. Reporting decisions vary among CPs, and reports may not be interpreted by clinicians as intended.", "The aims of this study were to survey clinicians and CPs about RBC-M terms and their clinical value, and identify areas of agreement and discordance.", "Online surveys were distributed to small animal clinicians via the Veterinary Information Network and to CPs via the ASVCP listserv. A quiz assessed understanding of RBC-M terms among respondent groups. Descriptive statistics were used to analyze responses to survey questions, and quiz scores were compared among groups.", "Analyzable responses were obtained from 1662 clinicians and 82 CPs. Both clinicians and CPs considered some terms, e.g., agglutination, useful, whereas only CPs considered other terms, e.g., ghost cells, useful. All groups interpreted certain terms, e.g., Heinz bodies, correctly, whereas some clinicians misinterpreted others, e.g., eccentrocytes. Responses revealed that CPs often do not report RBC-M they consider insignificant, when present in low numbers. Twenty-eight percent of clinicians think CPs review all blood smears while only 19% of CPs report reviewing all smears." ], "LABELS": [ "BACKGROUND", "OBJECTIVES", "METHODS", "RESULTS" ], "MESHES": [ "Animal Diseases", "Animals", "Diagnosis, Differential", "Erythrocytes", "Pathology, Veterinary", "Surveys and Questionnaires", "Veterinarians" ], "YEAR": "2014", "reasoning_required_pred": "yes", "reasoning_free_pred": "yes", "final_decision": "yes", "LONG_ANSWER": "Important differences about the clinical relevance of certain RBC-M terms exist between clinicians and CPs. Inclusion of interpretive comments on CBC reports is the clearest way to ensure that RBC-M changes are interpreted as intended by the CP. Reporting practices should be examined critically to improve communication, transparency, and ultimately medical decisions." }, "20813740": { "QUESTION": "Does \u03b2-catenin have a role in pathogenesis of sebaceous cell carcinoma of the eyelid?", "CONTEXTS": [ "48 cases of SbCC were analysed immunohistochemically using monoclonal \u03b2-catenin antibody and the results correlated with tumour size, histopathological differentiation, orbital invasion and pagetoid spread.", "Cytoplasmic overexpression of \u03b2-catenin was seen in 66% cases of SbCC which correlated positively with tumour size, orbital invasion and pagetoid spread. This correlation was found to be significant in tumour size>2 cm (p = 0.242). Nuclear staining was not observed in any of the cases." ], "LABELS": [ "METHOD", "RESULTS" ], "MESHES": [ "Adult", "Aged", "Aged, 80 and over", "Eyelid Neoplasms", "Female", "Humans", "Male", "Middle Aged", "Neoplasm Proteins", "Prognosis", "Sebaceous Gland Neoplasms", "Tumor Burden", "beta Catenin" ], "YEAR": "2011", "reasoning_required_pred": "yes", "reasoning_free_pred": "yes", "final_decision": "yes", "LONG_ANSWER": "Cytoplasmic overexpression of \u03b2-catenin was observed in the majority of the cases of SbCC of eyelid, and this correlated significantly with tumour size. The authors therefore hypothesise that \u03b2-catenin overexpression in SbCC of eyelid may be a result of Wnt/\u03b2-catenin pathway dysregulation. However, its role both in the development of sebaceous cell carcinoma of the eyelid as well as its prognosis needs to be explored further." }, "20401819": { "QUESTION": "Is ultrasound equal to X-ray in pediatric fracture diagnosis?", "CONTEXTS": [ "Ultrasound is currently not established for the diagnosis of fractures. The aim of this study was to compare ultrasound and X-ray beyond their use solely for the identification of fractures, i. e., for the detection of fracture type and dislocation for pediatric fracture diagnosis.", "Limb bones of dead young pigs served as a model for pediatric bones. The fractured bones were examined with ultrasound, X-ray, and CT, which served as the gold standard.", "162 of 248 bones were fractured. 130 fractures were identified using ultrasound, and 148 using X-ray. There were some advantages of X-ray over ultrasound in the detection of fracture type (80 correct results using X-ray, 66 correct results using ultrasound). Ultrasound, however, was superior to X-ray for dislocation identification (41 correct results using X-ray, 51 correct results using ultrasound). Both findings were not statistically significant after adjustment for multiple testing." ], "LABELS": [ "PURPOSE", "MATERIALS AND METHODS", "RESULTS" ], "MESHES": [ "Age Factors", "Animals", "Bone and Bones", "Child", "Disease Models, Animal", "Fractures, Bone", "Fractures, Closed", "Fractures, Comminuted", "Growth Plate", "Humans", "Image Processing, Computer-Assisted", "Imaging, Three-Dimensional", "Intra-Articular Fractures", "Salter-Harris Fractures", "Sensitivity and Specificity", "Swine", "Tomography, Spiral Computed", "Ultrasonography" ], "YEAR": "2010", "reasoning_required_pred": "yes", "reasoning_free_pred": "yes", "final_decision": "yes", "LONG_ANSWER": "Ultrasound not only has comparable sensitivity to that of X-ray for the identification of limb fractures but is also equally effective for the diagnosis of fracture type and dislocation. Thus, ultrasound can be used as an adequate alternative method to X-ray for pediatric fracture diagnosis." }, "7860319": { "QUESTION": "Measuring hospital mortality rates: are 30-day data enough?", "CONTEXTS": [ "We compare 30-day and 180-day postadmission hospital mortality rates for all Medicare patients and those in three categories of cardiac care: coronary artery bypass graft surgery, acute myocardial infarction, and congestive heart failure. DATA SOURCES/", "Health Care Financing Administration (HCFA) hospital mortality data for FY 1989.", "Using hospital level public use files of actual and predicted mortality at 30 and 180 days, we constructed residual mortality measures for each hospital. We ranked hospitals and used receiver operating characteristic (ROC) curves to compare 0-30, 31-180, and 0-180-day postadmission mortality.", "For the admissions we studied, we found a broad range of hospital performance when we ranked hospitals using the 30-day data; some hospitals had much lower than predicted 30-day mortality rates, while others had much higher than predicted mortality rates. Data from the time period 31-180 days postadmission yield results that corroborate the 0-30 day postadmission data. Moreover, we found evidence that hospital performance on one condition is related to performance on the other conditions, but that the correlation is much weaker in the 31-180-day interval than in the 0-30-day period. Using ROC curves, we found that the 30-day data discriminated the top and bottom fifths of the 180-day data extremely well, especially for AMI outcomes." ], "LABELS": [ "OBJECTIVE", "COLLECTION", "STUDY DESIGN", "PRINCIPAL FINDINGS" ], "MESHES": [ "Cardiology Service, Hospital", "Centers for Medicare and Medicaid Services (U.S.)", "Coronary Artery Bypass", "Forecasting", "Heart Failure", "Hospital Mortality", "Humans", "Medicare", "Myocardial Infarction", "Patient Admission", "ROC Curve", "Survival Rate", "Time Factors", "United States" ], "YEAR": "1995", "reasoning_required_pred": "yes", "reasoning_free_pred": "yes", "final_decision": "yes", "LONG_ANSWER": "Using data on cumulative hospital mortality from 180 days postadmission does not yield a different perspective from using data from 30 days postadmission for the conditions we studied." }, "25255719": { "QUESTION": "Hearing loss: an unknown complication of pre-eclampsia?", "CONTEXTS": [ "This prospective case-control study consisted of 33 patients with pre-eclampsia and 32 normotensive pregnant patients as controls. All of the subjects underwent otoscopic examinations - pure tone audiometry (0.25-16\u2009kHz) and transient evoked otoacoustic emission (1-4\u2009kHz) tests - during their third trimester of pregnancy.", "The mean ages of the patients with pre-eclampsia and the control subjects were 29.6\u2009\u00b1\u20095.7 and 28.6\u2009\u00b1\u20095.3 years, respectively. The baseline demographic characteristics, including age, gravidity, parity number, and gestational week, were similar between the two patient groups. Hearing thresholds in the right ear at 1, 4, 8, and 10\u2009kHz and in the left ear at 8 and 10\u2009kHz were significantly higher in the patients with pre-eclampsia compared to the control subjects. The degree of systolic blood pressure measured at the time of diagnosis had a deteriorating effect on hearing at 8, 10, and 12\u2009kHz in the right ear and at 10\u2009kHz in the left ear." ], "LABELS": [ "MATERIAL AND METHODS", "RESULTS" ], "MESHES": [ "Adult", "Auditory Threshold", "Blood Pressure", "Case-Control Studies", "Cochlea", "Female", "Hearing Loss, Sensorineural", "Humans", "Pre-Eclampsia", "Pregnancy", "Prospective Studies", "Risk Factors", "Systole", "Young Adult" ], "YEAR": "2015", "reasoning_required_pred": "yes", "reasoning_free_pred": "yes", "final_decision": "yes", "LONG_ANSWER": "Pre-eclampsia is a potential risk factor for cochlear damage and sensorineural hearing loss. Further studies that include routine audiological examinations are needed in these patients." }, "24352924": { "QUESTION": "Is portable ultrasonography accurate in the evaluation of Schanz pin placement during extremity fracture fixation in austere environments?", "CONTEXTS": [ "The purpose of this study was to investigate the efficacy of ultrasonography to confirm Schanz pin placement in a cadaveric model, and the interobserver repeatability of the ultrasound methodology.", "This investigation is a repeated measures cadaveric study with multiple examiners.", "Cadaveric preparation and observations were done by an orthopaedic traumatologist and resident, and two general surgery traumatologists.", "A total of 16 Schanz pins were equally placed in bilateral femora and tibiae. Four examiners took measurements of pin protrusion beyond the distal cortices using first ultrasonography and then by direct measurement after gross dissection.MAIN OUTCOME MEASURE(S): Distal Schanz pin protrusion length measurements from both ultrasonography and direct measurement post dissection.", "Schanz pin protrusion measurements are underestimated by ultrasonography (p<0.01) by an average of 10 percent over the range of 5 to 18 mm, and they display a proportional bias that increases the under reporting as the magnitude of pin protrusion increases. Ultrasound data demonstrate good linear correlation and closely represent actual protrusion values in the 5 to 12 mm range. Interobserver repeatability analysis demonstrated that all examiners were not statistically different in their measurements despite minimal familiarity with the ultrasound methodology (p>0.8)." ], "LABELS": [ "OBJECTIVE", "DESIGN", "PARTICIPANTS", "INTERVENTIONS", "RESULTS" ], "MESHES": [ "Bone Nails", "Cadaver", "External Fixators", "Femoral Fractures", "Fracture Fixation", "Humans", "Point-of-Care Systems", "Reproducibility of Results", "Tibial Fractures", "Ultrasonography" ], "YEAR": "2013", "reasoning_required_pred": "yes", "reasoning_free_pred": "yes", "final_decision": "yes", "LONG_ANSWER": "Despite the statistical imparity of pin protrusion measurement via ultrasound compared to that of gross dissection, a consideration of the clinical relevance of ultrasound measurement bias during an austere operating theatre leads to the conclusion that ultrasonography is an adequate methodology for Schanz pin protrusion measurement." }, "24973051": { "QUESTION": "Medical student education in emergency medicine: do students meet the national standards for clinical encounters of selected core conditions?", "CONTEXTS": [ "Establishing a core curriculum for undergraduate Emergency Medicine (EM) education is crucial to development of the specialty. The Clerkship Directors in Emergency Medicine (CDEM) National Curriculum Task Force recommended that all students in a 4(th)-year EM clerkship be exposed to 10 emergent clinical conditions.", "To evaluate the feasibility of encountering recommended core conditions in a clinical setting during a 4(th)-year EM clerkship.", "Students from three institutions participated in this ongoing, prospective observation study. Students' patient logs were collected during 4-week EM clerkships between July 2011 and June 2012. De-identified logs were reviewed and the number of patient encounters for each of the CDEM-identified emergent conditions was recorded. The percentage of students who saw each of the core complaints was calculated, as was the average number of core complaints seen by each.", "Data from 130 students at three institutions were captured; 15.4% of students saw all 10 conditions during their rotation, and 76.9% saw at least eight. The average number of conditions seen per student was 8.4 (range of 7.0-8.6). The percentage of students who saw each condition varied, ranging from 100% (chest pain and abdominal pain) to 31% (cardiac arrest)." ], "LABELS": [ "BACKGROUND", "OBJECTIVES", "METHODS", "RESULTS" ], "MESHES": [ "Clinical Clerkship", "Curriculum", "Education, Medical, Undergraduate", "Emergency Medicine", "Feasibility Studies", "Humans", "Prospective Studies" ], "YEAR": "2014", "reasoning_required_pred": "no", "reasoning_free_pred": "no", "final_decision": "no", "LONG_ANSWER": "Most students do not encounter all 10 conditions during patient encounters throughout a 4-week EM rotation, although most have exposure to at least eight. Certain conditions are far less likely than others to be encountered, and may need to be taught in a nonclinical setting." }, "20497146": { "QUESTION": "A model of healing of Los Angeles grades C and D reflux oesophagitis: is there an optimal time of acid suppression for maximal healing?", "CONTEXTS": [ "In patients with Los Angeles (LA) grade C or D oesophagitis, a positive relationship has been established between the duration of intragastric acid suppression and healing.AIM: To determine whether there is an apparent optimal time of intragastric acid suppression for maximal healing of reflux oesophagitis.", "Post hoc analysis of data from a proof-of-concept, double-blind, randomized study of 134 adult patients treated with esomeprazole (10 or 40 mg od for 4 weeks) for LA grade C or D oesophagitis. A curve was fitted to pooled 24-h intragastric pH (day 5) and endoscopically assessed healing (4 weeks) data using piecewise quadratic logistic regression.", "Maximal reflux oesophagitis healing rates were achieved when intragastric pH>4 was achieved for approximately 50-70% (12-17 h) of the 24-h period. Acid suppression above this threshold did not yield further increases in healing rates." ], "LABELS": [ "BACKGROUND", "METHODS", "RESULTS" ], "MESHES": [ "Adult", "Aged", "Dose-Response Relationship, Drug", "Double-Blind Method", "Esomeprazole", "Esophageal pH Monitoring", "Esophagitis, Peptic", "Female", "Humans", "Hydrogen-Ion Concentration", "Los Angeles", "Male", "Middle Aged", "Wound Healing", "Young Adult" ], "YEAR": "2010", "reasoning_required_pred": "yes", "reasoning_free_pred": "yes", "final_decision": "yes", "LONG_ANSWER": "After 4 weeks' acid-suppressive therapy for LA grade C or D oesophagitis, successful healing appears to reach a threshold above which improvements are unlikely to be achieved despite an increase in number of hours with intragastric pH>4." }, "19401574": { "QUESTION": "Gadofosveset-enhanced MR angiography of carotid arteries: does steady-state imaging improve accuracy of first-pass imaging?", "CONTEXTS": [ "To evaluate the diagnostic accuracy of gadofosveset-enhanced magnetic resonance (MR) angiography in the assessment of carotid artery stenosis, with digital subtraction angiography (DSA) as the reference standard, and to determine the value of reading first-pass, steady-state, and \"combined\" (first-pass plus steady-state) MR angiograms.", "This study was approved by the local ethics committee, and all subjects gave written informed consent. MR angiography and DSA were performed in 84 patients (56 men, 28 women; age range, 61-76 years) with carotid artery stenosis at Doppler ultrasonography. Three readers reviewed the first-pass, steady-state, and combined MR data sets, and one independent observer evaluated the DSA images to assess stenosis degree, plaque morphology and ulceration, stenosis length, and tandem lesions. Interobserver agreement regarding MR angiographic findings was analyzed by using intraclass correlation and Cohen kappa coefficients. Sensitivity, specificity, positive predictive value (PPV), and negative predictive value (NPV) were calculated by using the McNemar test to determine possible significant differences (P<.05).", "Interobserver agreement regarding all MR angiogram readings was substantial. For grading stenosis, sensitivity, specificity, PPV, and NPV were, respectively, 90%, 92%, 91%, and 91% for first-pass imaging; 95% each for steady-state imaging; and 96%, 99%, 99%, and 97% for combined imaging. For evaluation of plaque morphology, respective values were 84%, 86%, 88%, and 82% for first-pass imaging; 98%, 97%, 98%, and 97% for steady-state imaging; and 98%, 100%, 100%, and 97% for combined imaging. Differences between the first-pass, steady-state, and combined image readings for assessment of stenosis degree and plaque morphology were significant (P<.001)." ], "LABELS": [ "PURPOSE", "MATERIALS AND METHODS", "RESULTS" ], "MESHES": [ "Aged", "Angiography, Digital Subtraction", "Carotid Stenosis", "Contrast Media", "Female", "Gadolinium", "Humans", "Image Enhancement", "Magnetic Resonance Angiography", "Male", "Middle Aged", "Organometallic Compounds", "Reproducibility of Results", "Sensitivity and Specificity" ], "YEAR": "2009", "reasoning_required_pred": "yes", "reasoning_free_pred": "yes", "final_decision": "yes", "LONG_ANSWER": "Gadofosveset-enhanced MR angiography is a promising technique for imaging carotid artery stenosis. Steady-state image reading is superior to first-pass image reading, but the combined reading protocol is more accurate." }, "23571528": { "QUESTION": "Sternal skin conductance: a reasonable surrogate for hot flash measurement?", "CONTEXTS": [ "This study aims to examine the accuracy of a new sternal skin conductance (SSC) device in measuring hot flashes and to assess the acceptability of the device by women.", "Three small descriptive pilot studies were performed using two sequential prototypes of the SSC device developed by an engineering device company in the Midwest. The devices were worn either in a monitored setting for 24 hours or in an ambulatory setting for 5 weeks. During the study period, women recorded hot flashes in a prospective hot flash diary and answered questions about the acceptability of wearing the SSC device.", "The first prototype was not able to collect any analyzable skin conductance data owing to various malfunction issues, including poor conductance and battery failure. However, 16 women wore the device for 5 weeks and reported that wearing the device was acceptable, although 31% stated that it interfered with daily activities. Hot flash data from the second prototype revealed a 24% concordance rate between self-reported and device-recorded hot flashes." ], "LABELS": [ "OBJECTIVE", "METHODS", "RESULTS" ], "MESHES": [ "Female", "Galvanic Skin Response", "History, Ancient", "Hot Flashes", "Humans", "Menopause", "Middle Aged", "Monitoring, Ambulatory", "Monitoring, Physiologic", "Pilot Projects", "Prospective Studies", "Self Report", "Skin", "Skin Temperature", "Sternum", "Women's Health" ], "YEAR": "2013", "reasoning_required_pred": "no", "reasoning_free_pred": "no", "final_decision": "no", "LONG_ANSWER": "Findings from these studies support discordance between device-recorded and self-reported hot flashes. In addition, the studies reveal further limitations of SSC monitoring, including difficulties with data collection and lack of consistency in interpretation. Based on these results and other recent trials identifying issues with SSC methodology, it is time to find a better physiologic surrogate measure for hot flashes." }, "25228241": { "QUESTION": "Elephant trunk in a small-calibre true lumen for chronic aortic dissection: cause of haemolytic anaemia?", "CONTEXTS": [ "The elephant trunk technique for aortic dissection is useful for reducing false lumen pressure; however, a folded vascular prosthesis inside the aorta can cause haemolysis. The purpose of this study was to investigate whether an elephant trunk in a small-calibre lumen can cause haemolysis.", "Inpatient and outpatient records were retrospectively reviewed.", "Two cases of haemolytic anaemia after aortic surgery using the elephant trunk technique were identified from 2011 to 2013. A 64-year-old man, who underwent graft replacement of the ascending aorta for acute Stanford type A aortic dissection, presented with enlargement of the chronic dissection of the descending aorta and moderate aortic regurgitation. A two-stage surgery was scheduled. Total arch replacement with an elephant trunk in the true lumen and concomitant aortic valve replacement were performed. Postoperatively, he developed severe haemolytic anaemia because of the folded elephant trunk. The anaemia improved after the second surgery, including graft replacement of the descending aorta. Similarly, a 61-year-old man, who underwent total arch replacement for acute Stanford type A aortic dissection, presented with enlargement of the chronic dissection of the descending aorta. Graft replacement of the descending aorta with an elephant trunk inserted into the true lumen was performed. The patient postoperatively developed haemolytic anaemia because of the folded elephant trunk, which improved after additional stent grafting into the elephant trunk." ], "LABELS": [ "OBJECTIVES", "METHODS", "RESULTS" ], "MESHES": [ "Anemia, Hemolytic", "Aneurysm, Dissecting", "Aortic Aneurysm", "Aortography", "Blood Vessel Prosthesis", "Blood Vessel Prosthesis Implantation", "Chronic Disease", "Humans", "Male", "Middle Aged", "Prosthesis Design", "Reoperation", "Tomography, X-Ray Computed", "Treatment Outcome" ], "YEAR": "2014", "reasoning_required_pred": "yes", "reasoning_free_pred": "yes", "final_decision": "yes", "LONG_ANSWER": "A folded elephant trunk in a small-calibre lumen can cause haemolysis. Therefore, inserting an elephant trunk in a small-calibre true lumen during surgery for chronic aortic dissection should be avoided." }, "22348433": { "QUESTION": "Does partial expander deflation exacerbate the adverse effects of radiotherapy in two-stage breast reconstruction?", "CONTEXTS": [ "The optimum protocol for expander volume adjustment with respect to the timing and application of radiotherapy remains controversial.", "Eighteen New Zealand rabbits were divided into three groups. Metallic port integrated anatomic breast expanders of 250 cc were implanted on the back of each animal and controlled expansion was performed. Group I underwent radiotherapy with full expanders while in Group II, expanders were partially deflated immediately prior to radiotherapy. Control group did not receive radiotherapy.The changes in blood flow at different volume adjustments were investigated in Group II by laser Doppler flowmetry. Variations in the histopathologic properties of the irradiated tissues including the skin, capsule and the pocket floor, were compared in the biopsy specimens taken from different locations in each group.", "A significant increase in skin blood flow was detected in Group II with partial expander deflation. Overall, histopathologic exam revealed aggravated findings of chronic radiodermatitis (epidermal atrophy, dermal inflammation and fibrosis, neovascularisation and vascular changes as well as increased capsule thickness) especially around the lower expander pole, in Group II." ], "LABELS": [ "BACKGROUND", "METHODS", "RESULTS" ], "MESHES": [ "Animals", "Breast Implants", "Breast Neoplasms", "Dermatologic Surgical Procedures", "Female", "Magnetic Resonance Imaging", "Mammaplasty", "Mastectomy", "Rabbits", "Radiation Injuries", "Radiation Oncology", "Radiotherapy Planning, Computer-Assisted", "Skin", "Tissue Expansion", "Tissue Expansion Devices" ], "YEAR": "2012", "reasoning_required_pred": "yes", "reasoning_free_pred": "yes", "final_decision": "yes", "LONG_ANSWER": "Expander deflation immediately prior to radiotherapy, may augment the adverse effects, especially in the lower expander pole, possibly via enhanced radiosensitization due to a relative increase in the blood flow and tissue oxygenation." }, "20538207": { "QUESTION": "Should temperature be monitorized during kidney allograft preservation?", "CONTEXTS": [ "It is generally considered that kidney grafts should be preserved at 4 degrees C during cold storage. However, actual temperature conditions are not known. We decided to study the temperature levels during preservation with the Biotainer storage can and Vitalpack transport pack.", "Temperature was monitored using the Thermobouton probe during preservation of pig kidneys, in the same conditions used with human grafts. The probe recorded the temperature level every 10 minutes during four days. We compared the results found with the new storage can with results obtained in the same conditions with the storage can formerly used by our team. We also studied the best position of the probe for temperature monitoring and the influence of the amount of ice within the transport pack on the temperature level. We then monitored the temperature during the conservation of actual human kidney grafts harvested at our institution from August 2007 to May 2008.", "The temperature levels were the same regardless of the position of the probe within the transport pack. The lowest temperature was maintained during 15 hours, and the temperature level stayed below 5 degrees C for 57 hours with the new storage can. The former storage can maintained the lowest temperature level for 80 minutes, and temperature reached 5 degrees C after 10 hours 40 minutes. Temperature levels were similar when 2 or 4 kg of crushed ice were used. We observed similar results when monitoring the conservation of human grafts." ], "LABELS": [ "GOAL", "MATERIAL", "RESULTS" ], "MESHES": [ "Animals", "Body Temperature", "Cold Temperature", "Humans", "Kidney Transplantation", "Organ Preservation", "Swine" ], "YEAR": "2010", "reasoning_required_pred": "yes", "reasoning_free_pred": "no", "final_decision": "no", "LONG_ANSWER": "The new storage can affords more stable temperature levels when compared to the formerly used can. Since temperature is stable during conservation, continuous monitoring in everyday practice does not seem warranted." }, "10927144": { "QUESTION": "Can p53 alterations be used to predict tumour response to pre-operative chemo-radiotherapy in locally advanced rectal cancer?", "CONTEXTS": [ "To examine whether p53 tumour suppressor gene alterations can be used to predict tumour response to pre-operative chemo-radiation in locally advanced rectal cancer in terms of reduction in tumour size and local failure.", "p53 alterations were studied in pre-treatment biopsy specimens of rectal carcinomas from 48 patients by immunohistochemistry (IHC) and polymerase chain reaction/single strand conformation polymorphism (PCR-SSCP) gene mutation analysis. Pre-operative pelvic radiotherapy was delivered with four fields, 45 Gy to the ICRU point in 25 fractions over 5 weeks. A radio-sensitising dose of 5-fluorouracil (500 mg/m(2)) was delivered concurrently for 6 days of the 5-week schedule (days 1, 2, 3 and days 22, 23 and 24). Total meso-rectal excision was planned 4 to 6 weeks from completion of pre-operative treatment. Response to therapy was assessed by macroscopic measurement of the surgical specimen by a pathologist who was unaware of the pre-treatment tumour size or of the p53 status.", "IHC evidence of p53 protein accumulation was found in 40% of tumours, p53 gene mutation in 35% and p53 alteration (either or both changes) in 46%. The average reduction in tumour size was 53% in the group with 'wild-type' p53 (IHC-/SSCP-) and 63% in the group with altered p53 (either IHC+ or SSCP+; P=0.18). No significant differences in tumour size reduction or local failure were observed in the groups with p53 overexpression or p53 mutation compared with normal." ], "LABELS": [ "PURPOSE", "METHODS", "RESULTS" ], "MESHES": [ "Adenocarcinoma", "Adult", "Aged", "Aged, 80 and over", "Antimetabolites, Antineoplastic", "Biomarkers, Tumor", "Chemotherapy, Adjuvant", "Female", "Fluorouracil", "Follow-Up Studies", "Humans", "Male", "Middle Aged", "Preoperative Care", "Probability", "Prognosis", "Prospective Studies", "Radiotherapy, Adjuvant", "Rectal Neoplasms", "Sensitivity and Specificity", "Survival Analysis", "Treatment Outcome", "Tumor Suppressor Protein p53" ], "YEAR": "2000", "reasoning_required_pred": "yes", "reasoning_free_pred": "no", "final_decision": "no", "LONG_ANSWER": "p53 alteration detected by IHC or SSCP analysis is not a clinically useful predictor of local response to pre-operative adjuvant therapy in advanced rectal carcinoma." }, "20197761": { "QUESTION": "Is irritable bowel syndrome a diagnosis of exclusion?", "CONTEXTS": [ "Guidelines emphasize that irritable bowel syndrome (IBS) is not a diagnosis of exclusion and encourage clinicians to make a positive diagnosis using the Rome criteria alone. Yet many clinicians are concerned about overlooking alternative diagnoses. We measured beliefs about whether IBS is a diagnosis of exclusion, and measured testing proclivity between IBS experts and community providers.", "We developed a survey to measure decision-making in two standardized patients with Rome III-positive IBS, including IBS with diarrhea (D-IBS) and IBS with constipation (C-IBS). The survey elicited provider knowledge and beliefs about IBS, including testing proclivity and beliefs regarding IBS as a diagnosis of exclusion. We surveyed nurse practitioners, primary care physicians, community gastroenterologists, and IBS experts.", "Experts were less likely than nonexperts to endorse IBS as a diagnosis of exclusion (8 vs. 72%; P<0.0001). In the D-IBS vignette, experts were more likely to make a positive diagnosis of IBS (67 vs. 38%; P<0.001), to perform fewer tests (2.0 vs. 4.1; P<0.01), and to expend less money on testing (US$297 vs. $658; P<0.01). Providers who believed IBS is a diagnosis of exclusion ordered 1.6 more tests and consumed $364 more than others (P<0.0001). Experts only rated celiac sprue screening and complete blood count as appropriate in D-IBS; nonexperts rated most tests as appropriate. Parallel results were found in the C-IBS vignette." ], "LABELS": [ "OBJECTIVES", "METHODS", "RESULTS" ], "MESHES": [ "Algorithms", "Decision Making", "Diagnosis, Differential", "Female", "Gastroenterology", "Guidelines as Topic", "Health Knowledge, Attitudes, Practice", "Humans", "Irritable Bowel Syndrome", "Male", "Middle Aged", "Nurse Practitioners", "Physicians, Family", "Regression Analysis", "Surveys and Questionnaires" ], "YEAR": "2010", "reasoning_required_pred": "no", "reasoning_free_pred": "yes", "final_decision": "maybe", "LONG_ANSWER": "Most community providers believe IBS is a diagnosis of exclusion; this belief is associated with increased resource use. Experts comply more closely with guidelines to diagnose IBS with minimal testing. This disconnect suggests that better implementation of guidelines is warranted to minimize variation and improve cost-effectiveness of care." }, "21394762": { "QUESTION": "Is pelvic pain associated with defecatory symptoms in women with pelvic organ prolapse?", "CONTEXTS": [ "To investigate the significance of pelvic pain and its association with defecatory symptoms in women with pelvic organ prolapse (POP).", "This is a cross sectional study of 248 women with stage II POP or greater. Women were stratified into \"pain\" and \"no-pain\" groups based on their response to a question on the Pelvic Floor Distress Inventory short form. Associations between patient demographics, exam findings and responses to validated questionnaires were evaluated.", "In women with POP, defecatory symptoms are significantly more common in women with pelvic pain including straining with bowel movement (OR 2.4, 95% CI 1.3, 4.6), sense of incomplete emptying (OR 4.4, 95% CI 2.3, 8.2), pain with bowel movement (OR 5.3, 95% CI 1.2, 23.0) and splinting with bowel movement (OR 3.8, 95% CI 2.0, 7.5)." ], "LABELS": [ "OBJECTIVE", "STUDY DESIGN", "RESULTS" ], "MESHES": [ "Adult", "Aged", "Cross-Sectional Studies", "Defecation", "Female", "Humans", "Logistic Models", "Middle Aged", "Odds Ratio", "Pain Measurement", "Pelvic Organ Prolapse", "Pelvic Pain", "Philadelphia", "Risk Assessment", "Risk Factors", "Severity of Illness Index", "Surveys and Questionnaires" ], "YEAR": "2011", "reasoning_required_pred": "yes", "reasoning_free_pred": "yes", "final_decision": "yes", "LONG_ANSWER": "In women with POP, the symptom of pelvic pain is associated with the presence of defecatory symptoms." }, "12442934": { "QUESTION": "Does ibuprofen increase perioperative blood loss during hip arthroplasty?", "CONTEXTS": [ "To determine whether prior exposure of non-steroidal anti-inflammatory drugs increases perioperative blood loss associated with major orthopaedic surgery.", "Fifty patients scheduled for total hip replacement were allocated to two groups (double blind, randomized manner). All patients were pretreated for 2 weeks before surgery: Group 1 with placebo drug, Group 2 with ibuprofen. All patients were injected intrathecally with bupivacaine 20mg plus morphine 0.1 mg, in a total volume of 4 mL, to provide surgical anaesthesia.", "The presence of severe adverse effects caused eight patients in the ibuprofen group and six in the placebo group to terminate their participation in the trial. The perioperative blood loss increased by 45% in the ibuprofen group compared with placebo. The total (+/-SD) blood loss in the ibuprofen group was 1161 (+/-472) mL versus 796 (+/-337) mL in the placebo group." ], "LABELS": [ "BACKGROUND AND OBJECTIVE", "METHODS", "RESULTS" ], "MESHES": [ "Analgesics, Non-Narcotic", "Anesthesia, Spinal", "Anti-Inflammatory Agents, Non-Steroidal", "Arthroplasty, Replacement, Hip", "Blood Loss, Surgical", "Double-Blind Method", "Female", "Humans", "Ibuprofen", "Male", "Middle Aged", "Postoperative Hemorrhage", "Preoperative Care" ], "YEAR": "2002", "reasoning_required_pred": "yes", "reasoning_free_pred": "yes", "final_decision": "yes", "LONG_ANSWER": "Pretreatment with ibuprofen before elective total hip surgery increases the perioperative blood loss significantly. Early discontinuation of non-selective non-steroidal anti-inflammatory drugs is advised." }, "9347843": { "QUESTION": "Laparoscopic-assisted ileocolic resections in patients with Crohn's disease: are abscesses, phlegmons, or recurrent disease contraindications?", "CONTEXTS": [ "Because of the inflammatory nature of Crohn's disease, ileocolic resections are often difficult to perform, especially if an abscess, phlegmon, or recurrent disease at a previous ileocolic anastomosis is present. Our goal was to determine whether the above factors are contraindications to a successful laparoscopic-assisted ileocolic resection.", "Between 1992 and 1996, 46 laparoscopic-assisted ileocolic resections were attempted. Fourteen patients had an abscess or phlegmon treated with bowel rest before operation (group I), 10 patients had recurrent Crohn's disease at the previous ileocolic anastomosis (group II), and 22 patients had no previous operation and no phlegmon or abscess associated with their disease (group III). These groups were compared with each other and with 70 consecutive open ileocolic resections for Crohn's disease during the same time period (group IV).", "Operative blood loss and time were greater in group IV than in groups I, II, and III (245 versus 151, 131, and 195 ml, respectively, and 202 versus 152, 144, and 139 minutes, respectively). Conversion to open procedure occurred in 5 patients (group I, 1 [7%]; group II, 2 [20%]; group III, 2 [9%]). Morbidity was highest in group IV (21% versus 0%, 10%, and 10%, respectively). Only one patient died (group IV, 1%). Length of hospital stay was longest in group IV (7.9 versus 4.8, 3.9, and 4.5 days, respectively)." ], "LABELS": [ "BACKGROUND", "METHODS", "RESULTS" ], "MESHES": [ "Abdominal Abscess", "Adult", "Anastomosis, Surgical", "Cellulitis", "Colon", "Contraindications", "Crohn Disease", "Female", "Humans", "Ileum", "Laparoscopy", "Male", "Morbidity", "Postoperative Complications", "Recurrence", "Registries", "Reoperation", "Retrospective Studies", "Treatment Outcome" ], "YEAR": "1997", "reasoning_required_pred": "no", "reasoning_free_pred": "no", "final_decision": "no", "LONG_ANSWER": "The laparoscopic-assisted approach to Crohn's disease is feasible and safe with good outcomes. Co-morbid preoperative findings such as abscess, phlegmon, or recurrent disease at the previous ileocolic anastomosis are not contraindications to a successful laparoscopic-assisted ileocolic resection in select patients." }, "23072266": { "QUESTION": "Has the use of complementary and alternative medicine therapies by U.S. adults with chronic disease-related functional limitations changed from 2002 to 2007?", "CONTEXTS": [ "This study examined changes in the use of complementary and alternative medicine (CAM) therapies by U.S. adults aged 18 years or older with chronic disease-related functional limitations between 2002 and 2007.", "The study was a cross-sectional survey.SETTING/", "The study was conducted in the United States.", "The study comprised adults aged 18 years or older with chronic disease-related functional limitations.", "Data were obtained from the 2002 and 2007 U.S. National Health Interview Survey to compare the use of 22 CAM therapies (n=9313 and n=7014, respectively). Estimates were age adjusted to the year 2000 U.S. standard population.", "The unadjusted and age-standardized prevalence of overall CAM use (22 therapies comparable between both survey years) was higher in 2007 than in 2002 (30.6% versus 26.9%, p<0.001 and 34.4% versus 30.6%, p<0.001, respectively). Adults with functional limitations that included changing and maintaining body position experienced a significant increase in CAM use between 2002 and 2007 (31.1%-35.0%, p<0.01). The use of deep breathing exercises was the most prevalent CAM therapy in both 2002 and 2007 and increased significantly during this period (from 17.9% to 19.9%, p<0.05). The use of meditation, massage, and yoga also increased significantly from 2002 and 2007 (11.0%-13.5%, p<0.01; 7.0%-10.9%, p<0.0001; and 5.1% to 6.6%, p<0.05, respectively), while the use of the Atkins diet decreased (2.2%- 1.4%, p<0.01)." ], "LABELS": [ "OBJECTIVES", "DESIGN", "LOCATION", "SUBJECTS", "METHODS", "RESULTS" ], "MESHES": [ "Activities of Daily Living", "Adolescent", "Adult", "Aged", "Chronic Disease", "Complementary Therapies", "Cross-Sectional Studies", "Diet Therapy", "Female", "Health Care Surveys", "Humans", "Male", "Massage", "Middle Aged", "Mind-Body Therapies", "Patient Acceptance of Health Care", "Posture", "United States", "Young Adult" ], "YEAR": "2013", "reasoning_required_pred": "yes", "reasoning_free_pred": "yes", "final_decision": "yes", "LONG_ANSWER": "Among U.S. adults with chronic disease-related functional limitations, the overall increase in CAM use from 2002 to 2007 was significant, particularly among those with changing and maintaining body position limitations." }, "12607666": { "QUESTION": "Is extended aortic replacement in acute type A dissection justifiable?", "CONTEXTS": [ "The aim of this study was to evaluate the effectiveness of our surgical strategy for acute aortic dissection based on the extent of the dissection and the site of the entry, with special emphasis on resection of all dissected aortic segments if technically possible.", "Between January 1995 and March 2001, 43 consecutive patients underwent operations for acute aortic dissection. In all patients the distal repair was performed under circulatory arrest without the use of an aortic cross-clamp. Fifteen patients underwent aortic arch replacement with additional reconstruction of supra-aortic vessels in 3 patients. Complete replacement of all dissected tissue could be achieved in 21 patients (group 1). Because of the distal extent of the dissection beyond the aortic arch, replacement of all the dissected tissue was not possible in 22 patients (group 2).", "Early mortality was 4.7% (2 patients), and the incidence of perioperative cerebrovascular events was 7.0% (3 patients). All of these events occurred in group 2 (p<0.025). During the follow-up period of 6 years or less, 5 patients died, all from causes not related to the aorta or the aortic valve. A persisting patent false lumen was observed in 14 of the 36 surviving patients (39%)." ], "LABELS": [ "BACKGROUND", "METHODS", "RESULTS" ], "MESHES": [ "Aged", "Anastomosis, Surgical", "Aneurysm, Dissecting", "Aortic Aneurysm", "Blood Vessel Prosthesis Implantation", "Brachiocephalic Trunk", "Cardiac Surgical Procedures", "Female", "Humans", "Male", "Middle Aged" ], "YEAR": "2003", "reasoning_required_pred": "no", "reasoning_free_pred": "yes", "final_decision": "yes", "LONG_ANSWER": "Extended replacement of the dissected ascending aorta and aortic arch can be done with good early and midterm results, even though it requires a complex surgical technique. Therefore we advocate complete replacement of the dissected parts of the aorta in all patients in whom this is technically possible." }, "24074624": { "QUESTION": "Is intensive monitoring during the first transfusion in pediatric patients necessary?", "CONTEXTS": [ "Some pediatric patients, typically those that are very young or felt to be especially sick are temporarily admitted to the intensive care unit (ICU) for observation during their first transfusion. If a significant reaction that requires ICU management does not occur, these patients are then transferred to a regular ward where future blood products are administered. The aim of this project was to determine if heightened observation such as temporary ICU admissions for the first transfusion are warranted.", "From the blood bank records of a tertiary care pediatric hospital, a list of patients on whom a transfusion reaction was reported between 2007 and 2012, the type of reaction and the patient's transfusion history, were extracted. The hospital location where the transfusion occurred, and whether the patient was evaluated by the ICU team or transferred to the ICU for management of the reaction was determined from the patient's electronic medical record.", "There were 174 acute reactions in 150 patients. Of these 150 patients, 13 (8.7%) different patients experienced a reaction during their first transfusion; all 13 patients experienced clinically mild reactions (8 febrile non-hemolytic, 4 mild allergic, and 1 patient who simultaneously had a mild allergic and a febrile non-hemolytic), and none required ICU management. Six severe reactions (6 of 174, 3.4%) involving significant hypotension and/or hypoxia that required acute and intensive management occurred during subsequent (i.e. not the first) transfusion in six patients." ], "LABELS": [ "BACKGROUND", "METHODS", "RESULTS" ], "MESHES": [ "Adolescent", "Adult", "Blood Component Transfusion", "Blood Transfusion", "Child", "Child, Preschool", "Critical Care", "Humans", "Infant", "Infant, Newborn", "Intensive Care Units", "Population Surveillance", "Transfusion Reaction", "Young Adult" ], "YEAR": "2014", "reasoning_required_pred": "yes", "reasoning_free_pred": "no", "final_decision": "no", "LONG_ANSWER": "The practice of intensive observation for the first transfusion in pediatric patients is probably unnecessary." }, "17224424": { "QUESTION": "Effects of exercise training on heart rate and QT interval in healthy young individuals: are there gender differences?", "CONTEXTS": [ "The aim of the present study was to assess the effects of exercise training on heart rate, QT interval, and on the relation between ventricular repolarization and heart rate in men and women.", "A 24 h Holter recording was obtained in 80 healthy subjects (40 males) who differed for the degree of physical activity. Trained individuals showed a lower heart rate and a higher heart rate variability than sedentary subjects, independent of the gender difference in basal heart rate. Mean 24 h QTc was similar in trained and non-trained men, while a significant difference was observed between trained and non-trained women. Exercise training reduced the QT/RR slope in both genders. This effect on the QT/RR relation was more marked in women; in fact, the gender difference in the ventricular repolarization duration at low heart rate observed in sedentary subjects was no longer present among trained individuals." ], "LABELS": [ "AIMS", "METHODS AND RESULTS" ], "MESHES": [ "Electrocardiography", "Exercise", "Female", "Heart Rate", "Humans", "Male", "Rest", "Sex Characteristics", "Ventricular Function" ], "YEAR": "2007", "reasoning_required_pred": "yes", "reasoning_free_pred": "yes", "final_decision": "yes", "LONG_ANSWER": "The results of this study suggest that the cardiovascular response to exercise training may be different in men and women. Women may benefit more from interventions aimed to increase physical activity as a tool for prevention of cardiovascular morbidity and mortality." }, "16097998": { "QUESTION": "Is coeliac disease screening in risk groups justified?", "CONTEXTS": [ "The benefits of serologic screening for coeliac disease in asymptomatic individuals are debatable.AIM: To investigate dietary compliance, quality of life and bone mineral density after long-term treatment in coeliac disease patients found by screening in risk groups.", "The study comprised 53 consecutive screen-detected coeliac patients diagnosed 14 years (median) ago. Dietary compliance was assessed by interview, 4-day food record and serology. Quality of life was evaluated by the Psychological General Well-Being and SF-36 questionnaires, gastrointestinal symptoms by the Gastrointestinal Symptom Rating Scale and bone mineral density by dual-energy x-ray absorptiometry. Comparisons were made to 44 symptom-detected-treated coeliac patients, 110 non-coeliac subjects and the general population.", "A total of 96% of screen-detected and 93% of symptom-detected coeliac patients adhered to a strict or fairly strict gluten-free diet. In screen-detected patients, quality of life and gastrointestinal symptoms were similar to those in symptom-detected patients or non-coeliac controls and bone mineral density was similar to that in the general population." ], "LABELS": [ "BACKGROUND", "METHODS", "RESULTS" ], "MESHES": [ "Adult", "Aged", "Body Mass Index", "Bone Density", "Celiac Disease", "Female", "Fractures, Bone", "Gastrointestinal Diseases", "Humans", "Male", "Mass Screening", "Middle Aged", "Patient Compliance", "Quality of Life", "Risk Factors" ], "YEAR": "2005", "reasoning_required_pred": "yes", "reasoning_free_pred": "yes", "final_decision": "yes", "LONG_ANSWER": "Long-term dietary compliance in screen-detected patients was good. Quality of life and bone mineral density were comparable with those in non-coeliac subjects and the general population. Active screening in coeliac disease risk groups seems to be reasonable rather than harmful." }, "25487603": { "QUESTION": "Analysis of the epidemiological pattern of Shigellosis in Barcelona between 1988 and 2012: Is it an emerging sexually transmitted infection?", "CONTEXTS": [ "The aim of this study was to describe the evolution and epidemiologic characteristics of shigellosis patients over a 25 year period in a large city.", "Shigellosis is a notifiable disease in Spain since 1988. Cases are analyzed in Barcelona residents included in the registry between 1988-2012. A descriptive analysis by sex, age, mode of transmission and Shigella species is presented. Trend analysis and time series were performed.", "Of the 559 cases analyzed, 60.15% were males. A sustained increase was observed in the trend since 2008 in males (p<0,05), especially at the expense of males who had no history of food poisoning or travel to endemic areas. The increasing tendency was greater in males from 21 to 60 years, both for S. flexneri (since 2009), and for S. sonnei (since 2004). In 2012 it was noted that in the men with S. flexneri, the 63% were men who have sex with men." ], "LABELS": [ "INTRODUCTION", "METHODS", "RESULTS" ], "MESHES": [ "Adult", "Disease Notification", "Dysentery, Bacillary", "Emigrants and Immigrants", "Female", "Humans", "Male", "Middle Aged", "Morbidity", "Registries", "Risk Factors", "Seasons", "Sexual Behavior", "Sexually Transmitted Diseases, Bacterial", "Shigella", "Spain", "Species Specificity", "Travel", "Young Adult" ], "YEAR": null, "reasoning_required_pred": "yes", "reasoning_free_pred": "yes", "final_decision": "yes", "LONG_ANSWER": "An increased trend was detected in men who had no history of food poisoning or travel to endemic areas. This increase points to a change in the pattern of shigellosis, becoming predominantly male and its main mechanism probably by sexual transmission." }, "20594006": { "QUESTION": "A patient with myelomeningocele: is untethering necessary prior to scoliosis correction?", "CONTEXTS": [ "Tethering of the spinal cord is thought to increase the chance of neurological injury when scoliosis correction is undertaken. All patients with myelomeningocele (MM) are radiographically tethered, and untethering procedures carry significant morbidity risks including worsening neurological function and wound complications. No guidelines exist as regards untethering in patients with MM prior to scoliosis correction surgery. The authors' aim in this study was to evaluate their experience in patients with MM who were not untethered before scoliosis correction.", "Seventeen patients with MM were retrospectively identified and 1) had no evidence of a clinically symptomatic tethered cord, 2) had undergone spinal fusion for scoliosis correction, and 3) had not been untethered for at least 1 year prior to surgery. The minimum follow-up after fusion was 2 years. Charts and radiographs were reviewed for neurological or shunt complications in the perioperative period.", "The average age of the patients was 12.4 years, and the following neurological levels were affected: T-12 and above, 7 patients; L-1/L-2, 6 patients; L-3, 2 patients; and L-4, 2 patients. All were radiographically tethered as confirmed on MR imaging. Fourteen of the patients (82%) had a ventriculoperitoneal shunt. The mean Cobb angle was corrected from 82 degrees to 35 degrees , for a 57% correction. All patients underwent neuromonitoring of their upper extremities, and some underwent lower extremity monitoring as well. Postoperatively, no patient experienced a new cranial nerve palsy, shunt malfunction, change in urological function, or upper extremity weakness/sensory loss. One patient had transient lower extremity weakness, which returned to baseline within 1 month of surgery." ], "LABELS": [ "OBJECT", "METHODS", "RESULTS" ], "MESHES": [ "Adolescent", "Child", "Female", "Humans", "Lumbar Vertebrae", "Magnetic Resonance Imaging", "Male", "Meningomyelocele", "Neural Tube Defects", "Neurosurgical Procedures", "Radiography", "Risk Factors", "Scoliosis", "Spinal Cord", "Spinal Fusion", "Thoracic Vertebrae", "Treatment Outcome", "Unnecessary Procedures" ], "YEAR": "2010", "reasoning_required_pred": "yes", "reasoning_free_pred": "no", "final_decision": "no", "LONG_ANSWER": "The study results suggested that spinal cord untethering may be unnecessary in patients with MM who are undergoing scoliosis corrective surgery and do not present with clinical symptoms of a tethered cord, even though tethering is radiographically demonstrated." }, "26363639": { "QUESTION": "Is aerobic workload positively related to ambulatory blood pressure?", "CONTEXTS": [ "Cardiovascular disease is prevalent among workers with high levels of occupational physical activity. The increased risk may be due to a high relative aerobic workload, possibly leading to increased blood pressure. However, studies investigating the relation between relative aerobic workload and ambulatory blood pressure (ABP) are lacking. The aim was to explore the relationship between objectively measured relative aerobic workload and ABP.", "A total of 116 cleaners aged 18-65 years were included after informed consent was obtained. A portable device (Spacelabs 90217) was mounted for 24-h measurements of ABP, and an Actiheart was mounted for 24-h heart rate measurements to calculate relative aerobic workload as percentage of relative heart rate reserve. A repeated-measure multi-adjusted mixed model was applied for analysis.", "A fully adjusted mixed model of measurements throughout the day showed significant positive relations (p<0.001): a 1% increase in mean relative aerobic workload was associated with an increase of 0.42 \u00b1 0.05 mmHg (95% CI 0.32-0.52 mmHg) in systolic ABP and 0.30 \u00b1 0.04 mmHg (95% CI 0.22-0.38 mmHg) in diastolic ABP. Correlations between relative aerobic workload and ABP were significant." ], "LABELS": [ "PURPOSE", "METHODS", "RESULTS" ], "MESHES": [ "Adolescent", "Adult", "Aged", "Blood Pressure", "Blood Pressure Monitoring, Ambulatory", "Cross-Sectional Studies", "Exercise", "Female", "Heart Rate", "Humans", "Hypertension", "Male", "Middle Aged", "Workload", "Young Adult" ], "YEAR": "2016", "reasoning_required_pred": "yes", "reasoning_free_pred": "yes", "final_decision": "yes", "LONG_ANSWER": "Because workers may have an elevated relative aerobic workload for several hours each working day, this relationship may elucidate a mechanism behind the increased risk for cardiovascular disease among workers exposed to high levels of occupational physical activity." }, "27405146": { "QUESTION": "PREVALENCE OF THE STREPTOCOCUS AGALACTIAE IN THE PREGNANT WOMAN FROM THE AUTONOMIC CITY OF MELILLA: IS CULTURE A DETERMINANT FACTOR?", "CONTEXTS": [ "The neonatal infection by Streptococcus group B is one of the main causes of neonatal morbi-mortality rate. For this reason a screening is made to each pregnant woman in order to detect its presence, and if it was the case, to apply an antibiotic treatment during labour. The aim of this study was to know the prevalence of this Streptococcus in the pregnant women from Melilla, as well as the differences according to culture and age.", "A descriptive cross-sectional study located in the Hospital Comarcal from Melilla.", "The sample is taken from 280 women: 194 are from Muslim culture (69.3%), 68 are from Christian culture (24.3%) and 18 women from unknown cultures (6.4%). Also it is known that 78 of them are 25 years old or less (27.85%), 158 are between 26 and 34 years old (56.42%) and 44 are 35 years old or more (15.71%)." ], "LABELS": [ "BACKGROUND", "METHOD", "RESULTS" ], "MESHES": [ "Adult", "Christianity", "Cross-Sectional Studies", "Cultural Characteristics", "Female", "Humans", "Islam", "Pregnancy", "Pregnancy Complications, Infectious", "Prevalence", "Rectum", "Retrospective Studies", "Risk Factors", "Streptococcal Infections", "Streptococcus agalactiae", "Vagina", "Young Adult" ], "YEAR": "2016", "reasoning_required_pred": "yes", "reasoning_free_pred": "yes", "final_decision": "yes", "LONG_ANSWER": "The prevalence of vagino-rectal colonization by Streptococcus group B in the pregnant women from Melilla is within the national estimated figures, however it is different if they are from Muslim or Christian culture, being higher in the Muslim population. On one hand both prevalences are within the national statistics, and on the other hand it is observed that there is not any difference according to age." }, "17192736": { "QUESTION": "Is fluoroscopy essential for retrieval of lower ureteric stones?", "CONTEXTS": [ "The aim of this study was to assess the efficacy of ureteroscopy for lower ureteric stones without the use of fluoroscopy.", "Between June 2001 and January 2005, a total of 110 patients with a mean age of 33.5 years (range 12-65) suffering from of lower ureteral calculi (below the upper margin of the sacroiliac joint) prospectively underwent ureteroscopic removal. Retrograde pyelography was avoided, and no safety guidewire was placed. Whenever required, the ureteric meatus was dilated with a ureteric balloon under direct vision. Double-J stent placement was done with the aid of ureteroscopy. A fluoroscope was kept standby. The patients had a postoperative X-ray of the kidney-ureter-bladder region to document the stone clearance.", "The mean stone size was 8.7 mm (range 6-15). Complete clearance without the use of fluoroscopy was achieved in 99 patients (94.2%). Fluoroscopy was required in 6 patients (4%) for calcified stricture (n = 1), duplex system (n = 1), narrow and tortuous meatus causing difficulty in passing the 5-Fr balloon dilator (n = 3), and confirmation of spontaneous passage of the stone (n = 1). Of the 13 patients who required balloon dilatation it was successfully achieved without fluoroscopy. Double-J stenting was done due to mucosal ulceration (n = 3), polypoid reaction (n = 2), and perforation (n = 1). All these patients had correct placement of the stent, as confirmed by X-ray of the kidney-ureter-bladder region postoperatively." ], "LABELS": [ "INTRODUCTION", "PATIENTS AND METHODS", "RESULTS" ], "MESHES": [ "Adolescent", "Adult", "Aged", "Child", "Contraindications", "Fluoroscopy", "Follow-Up Studies", "Humans", "Middle Aged", "Prosthesis Implantation", "Retrospective Studies", "Stents", "Treatment Outcome", "Ureteral Calculi", "Ureteroscopy" ], "YEAR": "2007", "reasoning_required_pred": "no", "reasoning_free_pred": "no", "final_decision": "no", "LONG_ANSWER": "To uphold the notion for radiation exposure to be as low as reasonably achievable, ureteroscopic stone retrieval can safely be done without the use of fluoroscopy in a significant number of patients." }, "14652839": { "QUESTION": "Does the sequence of clamp application during open abdominal aortic aneurysm surgery influence distal embolisation?", "CONTEXTS": [ "Embolisation of atherosclerotic debris during abdominal aortic aneurysm (AAA) repair is responsible for significant peri-operative morbidity. Reports have suggested that preferential clamping of the distal vessel(s) before the proximal aorta may decrease the number of emboli passing distally and hence reduce complications.", "Forty patients undergoing AAA repair were randomised to have either first clamping of the proximal aorta or the iliac vessels. Emboli passing through the Superficial Femoral Arteries were detected with a Transcranial Doppler ultrasound system.", "There was no difference between the two groups in the number of emboli detected (p=0.49) and no significant correlation between number of emboli and dissection time (r=0.0008). However, there was a significantly higher number of emboli in the patient sub-group that were current smokers (p=0.034)." ], "LABELS": [ "BACKGROUND", "METHODS", "RESULTS" ], "MESHES": [ "Aged", "Aortic Aneurysm, Abdominal", "Embolism", "Female", "Humans", "Ligation", "Male", "Middle Aged", "Surgical Instruments", "Vascular Surgical Procedures" ], "YEAR": "2004", "reasoning_required_pred": "no", "reasoning_free_pred": "no", "final_decision": "no", "LONG_ANSWER": "There appears to be no difference in the numbers of emboli detected when the first vascular clamp is applied to the proximal aorta or iliacs." }, "12145243": { "QUESTION": "Are lower fasting plasma glucose levels at diagnosis of type 2 diabetes associated with improved outcomes?", "CONTEXTS": [ "Type 2 diabetes may be present for several years before diagnosis, by which time many patients have already developed diabetic complications. Earlier detection and treatment may reduce this burden, but evidence to support this approach is lacking.", "Glycemic control and clinical and surrogate outcomes were compared for 5,088 of 5,102 U.K. Diabetes Prospective Study participants according to whether they had low (<140 mg/dl [<7.8 mmol/l]), intermediate (140 to<180 mg/dl [7.8 to<10.0 mmol/l]), or high (>or =180 mg/dl [>or =10 mmol/l]) fasting plasma glucose (FPG) levels at diagnosis. Individuals who presented with and without diabetic symptoms were also compared.", "Fewer people with FPG in the lowest category had retinopathy, abnormal biothesiometer measurements, or reported erectile dysfunction. The rate of increase in FPG and HbA(1c) during the study was identical in all three groups, although absolute differences persisted. Individuals in the low FPG group had a significantly reduced risk for each predefined clinical outcome except stroke, whereas those in the intermediate group had significantly reduced risk for each outcome except stroke and myocardial infarction. The low and intermediate FPG groups had a significantly reduced risk for progression of retinopathy, reduction in vibration sensory threshold, or development of microalbuminuria." ], "LABELS": [ "OBJECTIVE", "RESEARCH DESIGN AND METHODS", "RESULTS" ], "MESHES": [ "Aged", "Blood Glucose", "Diabetes Mellitus, Type 2", "Diabetic Retinopathy", "Disease-Free Survival", "Fasting", "Female", "Glycated Hemoglobin A", "Humans", "Hyperglycemia", "Male", "Middle Aged", "Predictive Value of Tests", "Prospective Studies", "Treatment Outcome" ], "YEAR": "2002", "reasoning_required_pred": "yes", "reasoning_free_pred": "yes", "final_decision": "yes", "LONG_ANSWER": "People presenting with type 2 diabetes with lower initial glycemia who may be earlier in the course of their disease had fewer adverse clinical outcomes despite similar glycemic progression. Since most such people are asymptomatic at diagnosis, active case detection programs would be required to identify them." }, "25571931": { "QUESTION": "Do elderly patients call 911 when presented with clinical scenarios suggestive of acute stroke?", "CONTEXTS": [ "Among patients with acute stroke symptoms, delay in hospital admission is the main obstacle for the use of thrombolytic therapy and other interventions associated with decreased mortality and disability. The primary aim of this study was to assess whether an elderly clinical population correctly endorsed the response to call for emergency services when presented with signs and symptoms of stroke using a standardized questionnaire.", "We performed a cross-sectional study among elderly out-patients (\u226560 years) in Buenos Aires, Argentina randomly recruited from a government funded health clinic. The correct endorsement of intention to call 911 was assessed with the Stroke Action Test and the cut-off point was set at \u226575%. Knowledge of stroke and clinical and socio-demographic indicators were also collected and evaluated as predictors of correct endorsement using logistic regression.", "Among 367 elderly adults, 14% correctly endorsed intention to call 911. Presented with the most typical signs and symptoms, only 65% reported that they would call an ambulance. Amaurosis Fugax was the symptom for which was called the least (15%). On average, the correct response was chosen only 37% of the time. Compared to lower levels of education, higher levels were associated to correctly endorsed intention to call 911 (secondary School adjusted OR 3.53, 95% CI 1.59-7.86 and Tertiary/University adjusted OR 3.04, 95% CI 1.12-8.21)." ], "LABELS": [ "BACKGROUND AND PURPOSE", "METHODS", "RESULTS" ], "MESHES": [ "Aged", "Aged, 80 and over", "Amaurosis Fugax", "Ambulances", "Argentina", "Cross-Sectional Studies", "Educational Status", "Emergency Medical Service Communication Systems", "Emergency Medical Services", "Female", "Health Education", "Health Knowledge, Attitudes, Practice", "Humans", "Intention", "Male", "Middle Aged", "Stroke" ], "YEAR": "2015", "reasoning_required_pred": "maybe", "reasoning_free_pred": "no", "final_decision": "maybe", "LONG_ANSWER": "These results suggest the need to provide interventions that are specifically designed to increase awareness of potential stroke signs and symptoms and appropriate subsequent clinical actions." }, "19108857": { "QUESTION": "Cerebromediastinal tuberculosis in a child with a probable Say-Barber-Miller syndrome: a causative link?", "CONTEXTS": [ "Tuberculosis continues to be a public health problem in emerging countries with a recent evidence of increased incidence of extrapulmonary localization in developed countries probably linked to HIV. To our knowledge the occurrence of cerebro-mediastinal tuberculosis in an immuno-competent child has not been previously described; moreover the child we describe has a probable Say-Barber-Miller syndrome. We discuss a putative causative link between this syndrome and the occurrence of tuberculosis.", "A seven-year-old girl presented to our department with a history of infantile encephalopathy since birth characterized by a facial dysmorphy (evocative of a bird face), microcephaly, and mental retardation, and with recurrent infections. The child had complained of back pain for several months; the parents reported anorexia, loss of weight. Spinal and cerebral MRI showed a mediastinal mass involving the spine and cerebral lesions evocative of tuberculomas. The tuberculin interdermal reaction was positive. Culture of a vertebral biopsy was positive for Koch bacillus. Anti-tuberculosis treatment improved general and local status. An extensive immunological work-up was normal." ], "LABELS": [ "INTRODUCTION", "CASE REPORT" ], "MESHES": [ "Anorexia", "Body Dysmorphic Disorders", "Child", "Consanguinity", "Diagnosis, Differential", "Face", "Female", "Humans", "Intellectual Disability", "Male", "Pedigree", "Syndrome", "Tuberculoma" ], "YEAR": "2009", "reasoning_required_pred": "yes", "reasoning_free_pred": "yes", "final_decision": "yes", "LONG_ANSWER": "[corrected] This observation is exceptional in many aspects: very early age of onset of extrapulmonary tuberculosis, no immune deficit, association with a rare congenital neurological syndrome. We discuss the possible link between this entity and the occurrence of tuberculosis." }, "25443385": { "QUESTION": "Are virtual planning and guided surgery for head and neck reconstruction economically viable?", "CONTEXTS": [ "Virtual planning and guided surgery with or without prebent or milled plates are becoming more and more common for mandibular reconstruction with fibular free flaps (FFFs). Although this excellent surgical option is being used more widely, the question of the additional cost of planning and cutting-guide production has to be discussed. In capped payment systems such additional costs have to be offset by other savings if there are no special provisions for extra funding. Our study was designed to determine whether using virtual planning and guided surgery resulted in time saved during surgery and whether this time gain resulted in self-funding of such planning through the time saved.", "All consecutive cases of FFF surgery were evaluated during a 2-year period. Institutional data were used to determine the price of 1 minute of operative time. The time for fibula molding, plate adaptation, and insetting was recorded.", "During the defined period, we performed 20 mandibular reconstructions using FFFs, 9 with virtual planning and guided surgery and 11 freehand cases. One minute of operative time was calculated to cost US $47.50. Multiplying this number by the time saved, we found that the additional cost of virtual planning was reduced from US $5,098 to US $1,231.50 with a prebent plate and from US $6,980 to US $3,113.50 for a milled plate." ], "LABELS": [ "PURPOSE", "MATERIALS AND METHODS", "RESULTS" ], "MESHES": [ "Aged", "Angiography", "Bone Plates", "Bone Transplantation", "Carcinoma, Squamous Cell", "Computer Simulation", "Cost Savings", "Diagnosis-Related Groups", "Female", "Fibula", "Free Tissue Flaps", "Health Care Costs", "Hospital Costs", "Humans", "Imaging, Three-Dimensional", "Male", "Mandibular Neoplasms", "Mandibular Reconstruction", "Middle Aged", "Models, Anatomic", "Operative Time", "Patient Care Planning", "Prospective Studies", "Surgery, Computer-Assisted", "Switzerland", "Tomography, X-Ray Computed", "Transplant Donor Site", "User-Computer Interface" ], "YEAR": "2015", "reasoning_required_pred": "yes", "reasoning_free_pred": "yes", "final_decision": "yes", "LONG_ANSWER": "Even in capped health care systems, virtual planning and guided surgery including prebent or milled plates are financially viable." }, "18719011": { "QUESTION": "Do overweight children necessarily make overweight adults?", "CONTEXTS": [ "To compare growth curves of body mass index from children to adolescents, and then to young adults, in Japanese girls and women in birth cohorts born from 1930 to 1999.", "Retrospective repeated cross sectional annual nationwide surveys (national nutrition survey, Japan) carried out from 1948 to 2005.", "Japan.", "76,635 females from 1 to 25 years of age.", "Body mass index.", "Generally, body mass index decreased in preschool children (2-5 years), increased in children (6-12 years) and adolescents (13-18 years), and slightly decreased in young adults (19-25 years) in these Japanese females. However, the curves differed among birth cohorts. More recent cohorts were more overweight as children but thinner as young women. The increments in body mass index in early childhood were larger in more recent cohorts than in older cohorts. However, the increments in body mass index in adolescents were smaller and the decrease in body mass index in young adults started earlier, with lower peak values in more recent cohorts than in older cohorts. The decrements in body mass index in young adults were similar in all birth cohorts." ], "LABELS": [ "OBJECTIVE", "DESIGN", "SETTING", "PARTICIPANTS", "MAIN OUTCOME MEASURE", "RESULTS" ], "MESHES": [ "Adolescent", "Adult", "Aged", "Body Mass Index", "Child", "Child, Preschool", "Cross-Sectional Studies", "Female", "Growth", "Humans", "Infant", "Japan", "Middle Aged", "Overweight", "Prevalence", "Thinness" ], "YEAR": "2008", "reasoning_required_pred": "no", "reasoning_free_pred": "no", "final_decision": "no", "LONG_ANSWER": "An overweight birth cohort in childhood does not necessarily continue to be overweight in young adulthood. Not only secular trends in body mass index at fixed ages but also growth curves for wide age ranges by birth cohorts should be considered to study obesity and thinness. Growth curves by birth cohorts were produced by a repeated cross sectional annual survey over nearly six decades." }, "9044116": { "QUESTION": "Biliary atresia: should all patients undergo a portoenterostomy?", "CONTEXTS": [ "The management of noncorrectable extra hepatic biliary atresia includes portoenterostomy, although the results of the surgery are variable. This study was done to develop criteria that could successfully predict the outcome of surgery based on preoperative data, including percutaneous liver biopsy, allowing a more selective approach to the care of these babies.", "The charts and biopsy results of 31 patients who underwent a Kasai procedure for biliary atresia between 1984 and 1994 were reviewed. Values for preoperative albumin, bilirubin, age of patient at Kasai, and lowest postoperative bilirubin were recorded. Surgical success was defined as postoperative bilirubin that returned to normal. A pathologist blinded to the child's eventual outcome graded the pre-Kasai needle liver biopsy results according to duct proliferation, ductal plate lesion, bile in ducts, lobular inflammation, giant cells, syncitial giant cells, focal necrosis, bridging necrosis, hepatocyte ballooning, bile in zone 1, 2, and 3, cholangitis, and end-stage cirrhosis. Clinical outcome was then predicted.", "Success after portoenterostomy could not reliably be predicted based on gender, age at Kasai, preoperative bilirubin or albumin levels. Histological criteria, however, predicted outcome in 27 of 31 patients (P<.01). Fifteen of 17 clinical successes were correctly predicted; as were 12 of 14 clinical failures (sensitivity, 86%; specificity, 88%). Individually, the presence of syncitial giant cells, lobular inflammation, focal necrosis, bridging necrosis, and cholangitis, were each associated with failure of the portoenterostomy (P<.05). Bile in zone 1 was associated with clinical success of the procedure (P<.05)." ], "LABELS": [ "PURPOSE", "METHODS", "RESULTS" ], "MESHES": [ "Biliary Atresia", "Biopsy, Needle", "Female", "Humans", "Infant", "Inflammation", "Liver", "Male", "Portoenterostomy, Hepatic", "Predictive Value of Tests", "Prognosis", "Retrospective Studies", "Treatment Outcome" ], "YEAR": "1997", "reasoning_required_pred": "no", "reasoning_free_pred": "no", "final_decision": "no", "LONG_ANSWER": "Based on the predictive information available in a liver biopsy, we conclude that those patients who will not benefit from a Kasai procedure can be identified preoperatively, and channeled immediately to transplantation." }, "23621776": { "QUESTION": "Does a history of unintended pregnancy lessen the likelihood of desire for sterilization reversal?", "CONTEXTS": [ "Unintended pregnancy has been significantly associated with subsequent female sterilization. Whether women who are sterilized after experiencing an unintended pregnancy are less likely to express desire for sterilization reversal is unknown.", "This study used national, cross-sectional data collected by the 2006-2010 National Survey of Family Growth. The study sample included women ages 15-44 who were surgically sterile from a tubal sterilization at the time of interview. Multivariable logistic regression was used to examine the relationship between a history of unintended pregnancy and desire for sterilization reversal while controlling for potential confounders.", "In this nationally representative sample of 1,418 women who were sterile from a tubal sterilization, 78% had a history of at least one unintended pregnancy and 28% expressed a desire to have their sterilization reversed. In unadjusted analysis, having a prior unintended pregnancy was associated with higher odds of expressing desire for sterilization reversal (odds ratio [OR]: 1.80; 95% confidence interval [CI]: 1.15-2.79). In adjusted analysis controlling for sociodemographic factors, unintended pregnancy was no longer significantly associated with desire for reversal (OR: 1.46; 95% CI: 0.91-2.34)." ], "LABELS": [ "BACKGROUND", "METHODS", "RESULTS" ], "MESHES": [ "Adolescent", "Adult", "Cross-Sectional Studies", "Female", "Humans", "Motivation", "Pregnancy", "Pregnancy, Unplanned", "Sterilization Reversal", "Sterilization, Tubal", "United States", "Young Adult" ], "YEAR": "2013", "reasoning_required_pred": "no", "reasoning_free_pred": "no", "final_decision": "no", "LONG_ANSWER": "Among women who had undergone tubal sterilization, a prior history of unintended pregnancy did not decrease desire for sterilization reversal." }, "12970636": { "QUESTION": "Does early discharge with nurse home visits affect adequacy of newborn metabolic screening?", "CONTEXTS": [ "To examine the impact of early discharge on newborn metabolic screening.", "Metabolic screening results were obtained from the Alabama State Lab for all infants born at our hospital between 8/1/97, and 1/31/99, and were matched with an existing database of early discharge infants. An early newborn discharge was defined as a discharge between 24 and 47 hours of age. Metabolic screening tests included phenylketonuria (PKU), hypothyroidism, and congenital adrenal hyperplasia (CAH). Early discharge and traditional stay infants were compared to determine the percentage of newborns screened and the timing of the first adequate specimen.", "The state laboratory received specimens from 3860 infants; 1324 were on early discharge newborns and 2536 infants in the traditional stay group. At least one filter paper test (PKU, hypothyroidism, and CAH) was collected on 99.2% of early discharge infants and 96.0% of traditional stay infants (P<.0001). Early discharge infants had a higher rate of initial filter paper specimens being inadequate (22.9%) compared with traditional stay infants (14.3%, P<.0001) but had a higher rate of repeat specimens when the initial specimen was inadequate (85.0% early discharge vs 75.3% traditional stay, P=.002). The early discharge group was more likely to have an adequate specimen within the first 9 days of life (1001, 98.8% early discharge vs 2016, 96.7% traditional stay, P=.0005)." ], "LABELS": [ "OBJECTIVE", "STUDY DESIGN", "RESULTS" ], "MESHES": [ "Alabama", "Female", "Humans", "Infant, Newborn", "Male", "Medical Indigency", "Metabolic Diseases", "Metabolism, Inborn Errors", "Neonatal Screening", "Nursing Homes", "Patient Discharge", "Prospective Studies" ], "YEAR": "2003", "reasoning_required_pred": "yes", "reasoning_free_pred": "no", "final_decision": "no", "LONG_ANSWER": "In this well established early discharge program with nurse home visits, newborn metabolic screening is not compromised by early discharge." }, "21084567": { "QUESTION": "Are stroke patients' reports of home blood pressure readings reliable?", "CONTEXTS": [ "Home blood pressure (BP) monitoring is gaining increasing popularity among patients and may be useful in hypertension management. Little is known about the reliability of stroke patients' records of home BP monitoring.", "To assess the reliability of home BP recording in hypertensive patients who had suffered a recent stroke or transient ischaemic attack.", "Thirty-nine stroke patients (mean age 73 years) randomized to the intervention arm of a trial of home BP monitoring were included. Following instruction by a research nurse, patients recorded their BPs at home and documented them in a booklet over the next year. The booklet readings over a month were compared with the actual readings downloaded from the BP monitor and were checked for errors or selective bias in recording.", "A total of 1027 monitor and 716 booklet readings were recorded. Ninety per cent of booklet recordings were exactly the same as the BP monitor readings. Average booklet readings were 0.6 mmHg systolic [95% confidence interval (95% CI) -0.6 to 1.8] and 0.3 mmHg diastolic (95% CI -0.3 to 0.8) lower than those on the monitor." ], "LABELS": [ "BACKGROUND", "OBJECTIVE", "METHODS", "RESULTS" ], "MESHES": [ "Adult", "Aged", "Aged, 80 and over", "Blood Pressure Monitoring, Ambulatory", "Cross-Sectional Studies", "Female", "Humans", "Hypertension", "Ischemic Attack, Transient", "Male", "Middle Aged", "Reproducibility of Results", "Self Care", "Self Report", "Stroke Rehabilitation" ], "YEAR": "2011", "reasoning_required_pred": "yes", "reasoning_free_pred": "yes", "final_decision": "yes", "LONG_ANSWER": "This group of elderly stroke patients were able to record their BPs reliably at home. Any bias was small and would be unlikely to affect management. Since BP readings in a GP surgery are often a poor indication of true BP, GPs might consider using hypertensive patients' records of home BP monitoring to help guide treatment decisions." }, "19546588": { "QUESTION": "Does increasing blood pH stimulate protein synthesis in dialysis patients?", "CONTEXTS": [ "Although the mechanism of muscle wasting in end-stage renal disease is not fully understood, there is increasing evidence that acidosis induces muscle protein degradation and could therefore contribute to the loss of muscle protein stores of patients on hemodialysis, a prototypical state of chronic metabolic acidosis (CMA). Because body protein mass is controlled by the balance between synthesis and degradation, protein loss can occur as result of either increased breakdown, impaired synthesis, or both. Correction of acidosis may therefore help to maintain muscle mass and improve the health of patients with CMA. We evaluated whether alkalizing patients on hemodialysis might have a positive effect on protein synthesis and on nutritional parameters.", "Eight chronic hemodialysis patients were treated daily with oral sodium bicarbonate (NaHCO(3)) supplementation for 10-14 days, yielding a pre-dialytic plasma bicarbonate concentration of 28.6 +/-1.6 mmol/l. The fractional synthesis rates (FSR) of muscle protein and albumin were obtained by the L-[(2)H(5)ring]phenylalanine flooding technique.", "Oral NaHCO(3 )supplementation induced a significant increase in serum bicarbonate (21.5 +/- 3.4 vs. 28.6 +/- 1.6 mmol/l; p = 0.018) and blood pH (7.41 vs. 7.46; p = 0.041). The FSR of muscle protein and the FSR of albumin did not change significantly (muscle protein: 2.1 +/- 0.2 vs. 2.0 +/- 0.5% per day, p = 0.39; albumin: 8.3 +/- 2.2 vs. 8.6 +/- 2.5% per day, p = 0.31). Plasma concentrations of insulin-like growth factor 1 decreased significantly (33.4 +/- 21.3 vs. 25.4 +/- 12.3 nmol/l; p = 0.028), whereas thyroid-stimulating hormone, free thyroxin and free triiodothyronine did not change significantly and nutritional parameters showed no improvement." ], "LABELS": [ "BACKGROUND", "METHODS", "RESULTS" ], "MESHES": [ "Administration, Oral", "Adult", "Aged", "Blood Chemical Analysis", "Blood Proteins", "Female", "Humans", "Hydrogen-Ion Concentration", "Kidney Failure, Chronic", "Male", "Middle Aged", "Protein Biosynthesis", "Renal Dialysis", "Sodium Bicarbonate" ], "YEAR": "2009", "reasoning_required_pred": "no", "reasoning_free_pred": "no", "final_decision": "no", "LONG_ANSWER": "In contrast to other findings, raising the blood pH of dialysis patients was not associated with a positive effect on albumin and muscle protein synthesis, or nutritional and endocrinal parameters." }, "19923859": { "QUESTION": "Can T-cell deficiency affect spatial learning ability following toluene exposure?", "CONTEXTS": [ "The present studywas designed to investigate the possible role of T cells in spatial learning ability in mouse after toluene exposure.", "Eight-week-old male wild-type (WT) and nude mice of BALB/c strain were exposed to toluene (0, 9 and 90 ppm) in a nose-only exposure chamber for 30 min per day for 3 consecutive days and then once per week for 4 weeks. Twenty-four hours after the completion of exposure, we examined the spatial learning ability in each mouse using the Morris water maze apparatus.", "In the acquisition phase, a longer escape latency was observed in nude mice exposed to 90 ppm toluene on days 3 and 4 when compared with corresponding WT mice. However, the effect of toluene on the escape latency was not significant in nude mice. In the probe trial, WT mice exposed to 90 ppm toluene showed poor retention memory compared with the control group. In the reversal phase, we did not find any significant difference between groups." ], "LABELS": [ "OBJECTIVE", "METHOD", "RESULTS" ], "MESHES": [ "Animals", "Astrocytes", "Disease Models, Animal", "Immune Tolerance", "Immunity, Cellular", "Immunity, Innate", "Learning Disorders", "Male", "Maze Learning", "Memory Disorders", "Mice", "Mice, Inbred BALB C", "Microglia", "Neuroimmunomodulation", "Neurotoxins", "Solvents", "T-Lymphocytes", "Toluene" ], "YEAR": "2010", "reasoning_required_pred": "yes", "reasoning_free_pred": "yes", "final_decision": "yes", "LONG_ANSWER": "These results indicate that T-cell deficiency may affect spatial learning performance following toluene exposure." }, "18616781": { "QUESTION": "Is there a relationship between homocysteine and vitiligo?", "CONTEXTS": [ "Pigmentary dilution is observed in patients with homocystinuria. Therefore, it is possible that an increase of local homocysteine (Hcy) interferes with normal melanogenesis and plays a role in the pathogenesis of vitiligo. Vitamin B12 and folic acid, levels of which are decreased in vitiligo, are important cofactors in the metabolism of Hcy. Consequently, a nutritional deficiency in either of these two vitamins will result in an increase in homocysteine in the circulation, a finding that we expect to find in vitiligo.", "To determine the level of Hcy in the blood of patients with vitiligo as a first step in revealing if it has any relationship with the pathogenesis of vitiligo and consequently if this will have an impact on the treatment of vitiligo.", "Twenty-six patients of both sexes with vitiligo (age range 20-50 years, mean 31.4 +/- 8.09) and 26 age-matched healthy controls were included in the study. After excluding factors that may affect serum Hcy levels, blood samples from patients and controls were obtained for homocysteine determination by enzyme immunoassay.", "The mean serum level of Hcy was significantly higher in patients with vitiligo than in controls (21.61 +/- 13.28 vs. 13.1 +/- 4.88 micromol L(-1); P<0.001). The Hcy level was significantly higher in male patients than in female patients (28.67 +/- 15.95 vs. 15.56 +/- 6.2 micromol L(-1); P<0.001) and in male controls compared with female controls (15.07 +/- 4.61 vs. 12.05 +/- 4.82 micromol L(-1); P<0.001). The homocysteine level was related to the activity of vitiligo and was significantly higher in patients with progressive disease than in controls (25.4 +/- 14.99 vs. 13.1 +/- 4.88 micromol L(-1); P<0.001). No significant difference in Hcy levels was found between either untreated vitiligo patients (22.77 +/- 13.36 micromol L(-1)) or patients receiving ultraviolet therapy (20.45 +/- 13.73 micromol L(-1)) and the total patient group (21.62 +/- 13.28 micromol L(-1))." ], "LABELS": [ "BACKGROUND", "OBJECTIVE", "METHODS", "RESULTS" ], "MESHES": [ "Adult", "Age Factors", "Case-Control Studies", "Female", "Homocysteine", "Humans", "Male", "Middle Aged", "Sex Factors", "Skin", "Vitiligo" ], "YEAR": "2008", "reasoning_required_pred": "yes", "reasoning_free_pred": "yes", "final_decision": "yes", "LONG_ANSWER": "An elevated homocysteine level may be a precipitating factor for vitiligo in predisposed individuals. In view of the biological role of vitamin B(12) and folic acid in Hcy metabolism, we present our recommendations regarding the investigation and treatment of this common disease." }, "27991408": { "QUESTION": "Do dermatomyositis and polymyositis affect similar thigh muscles?", "CONTEXTS": [ "Dermatomyositis (DM) and polymyositis (PM) commonly cause weakness of the thigh muscles. However, it is debated whether DM and PM affect similar thigh muscles. Muscle oedema on fat-suppressed MRI sequences is thought to represent active inflammation. In this study, we aimed to assess which thigh muscle groups are preferentially inflamed in DM and PM, respectively, using short-tau inversion-recovery MRI sequences.", "We analysed 71 patients from 2 Rheumatology centres, 31 with DM and 40 with PM diagnosed according to the Bohan and Peter criteria. MRI oedema (1=present, 0=absent) was assessed bilaterally on fat-suppressed sequences in 17 pelvic floor and thigh muscles. An MRI oedema score (range 0-17) was calculated by adding the separate scores bilaterally and dividing them by two. Inter-rater variability was assessed by intraclass correlation coefficient. Fisher's exact test was used to compare binomial data.", "Age and gender ratio were similar in patients with DM and PM. Disease duration (months, mean\u00b1SD) was shorter (20\u00b131) in DM than in PM (53\u00b169) (p=0.02). The intraclass correlation coefficient between the radiologists involved was 0.78. Muscle oedema was more common in DM than in PM except in the posterior thigh muscles. In particular, 68% of patients with DM had involvement of at least one anterior thigh muscle versus 38% of patients with PM (p=0.02)." ], "LABELS": [ "OBJECTIVES", "METHODS", "RESULTS" ], "MESHES": [ "Adult", "Aged", "Dermatomyositis", "Female", "Humans", "Magnetic Resonance Imaging", "Male", "Middle Aged", "Muscle, Skeletal", "Polymyositis", "Thigh" ], "YEAR": null, "reasoning_required_pred": "yes", "reasoning_free_pred": "no", "final_decision": "no", "LONG_ANSWER": "Compared with PM, DM affects more thigh muscles, except those of the posterior compartment, which are equally involved in both disorders. These findings may be useful to target physiotherapy at the more frequently affected muscles." }, "14655021": { "QUESTION": "Juvenile ankylosing spondylitis--is it the same disease as adult ankylosing spondylitis?", "CONTEXTS": [ "Juvenile and adult forms of ankylosing spondylitis (AS) have been shown to have different clinical presentation and outcome in Caucasians. We did this retrospective analysis to see if similar differences exist in the Indian population.", "Case records of 210 Indian patients diagnosed with AS according to modified New York criteria were reviewed. Data were collected regarding age of onset, clinical features, drug treatment, and outcome at last follow-up. Patients with onset before 17 years of age were classified as having juvenile AS (JAS) and the rest with adult AS (AAS).", "There were 150 patients with AAS and 60 with JAS. The latter had higher male preponderance, more frequent onset with peripheral arthritis, and greater involvement of hip and knee joints. Valvular dysfunction was seen only in patients with JAS." ], "LABELS": [ "OBJECTIVES", "PATIENTS AND METHODS", "RESULTS" ], "MESHES": [ "Adolescent", "Adult", "Age Factors", "Child", "Female", "Humans", "India", "Male", "Outcome Assessment (Health Care)", "Retrospective Studies", "Spondylitis, Ankylosing" ], "YEAR": "2005", "reasoning_required_pred": "no", "reasoning_free_pred": "no", "final_decision": "no", "LONG_ANSWER": "In this group of subjects, juvenile AS had onset more often with oligoarthritis and enthesitis than with spinal disease. Hip and knee joint involvement was more common in JAS than AAS." }, "26175531": { "QUESTION": "Is Aspiration Thrombectomy Beneficial in Patients Undergoing Primary Percutaneous Coronary Intervention?", "CONTEXTS": [ "It is unclear whether intravenous glycoprotein IIb/IIIa inhibitors or ischemic time might modify any clinical benefits observed with aspiration thrombectomy before primary percutaneous coronary intervention (PCI) in patients with ST-segment-elevation myocardial infarction.", "Electronic databases were searched for trials that randomized ST-segment-elevation myocardial infarction patients to aspiration thrombectomy before PCI versus conventional PCI. Summary estimates were constructed using a DerSimonian-Laird model. Seventeen trials with 20\u2009960 patients were available for analysis. When compared with conventional PCI, aspiration thrombectomy was not associated with a significant reduction in the risk of mortality 2.8% versus 3.2% (risk ratio [RR], 0.89; 95% confidence interval [CI], 0.76-1.04; P=0.13), reinfarction 1.3% versus 1.4% (RR, 0.93; 95% CI, 0.73-1.17; P=0.52), the combined outcome of mortality or reinfarction 4.1% versus 4.6% (RR, 0.90; 95% CI, 0.79-1.02; P=0.11), or stent thrombosis 0.9% versus 1.2% (RR, 0.82; 95% CI, 0.62-1.08; P=0.15). Aspiration thrombectomy was associated with a nonsignificant increase in the risk of stroke 0.6% versus 0.4% (RR, 1.45; 95% CI, 0.96-2.21; P=0.08). Meta-regression analysis did not identify a difference for the log RR of mortality, reinfarction, and the combined outcome of mortality or reinfarction with intravenous glycoprotein IIb/IIIa inhibitors (P=0.17, 0.70, and 0.50, respectively) or with ischemic time (P=0.29, 0.66, and 0.58, respectively)." ], "LABELS": [ "BACKGROUND", "METHODS AND RESULTS" ], "MESHES": [ "Humans", "Mortality", "Myocardial Infarction", "Percutaneous Coronary Intervention", "Platelet Glycoprotein GPIIb-IIIa Complex", "Randomized Controlled Trials as Topic", "Recurrence", "Regression Analysis", "Stroke", "Stroke Volume", "Thrombectomy" ], "YEAR": "2015", "reasoning_required_pred": "no", "reasoning_free_pred": "no", "final_decision": "no", "LONG_ANSWER": "Aspiration thrombectomy before primary PCI is not associated with any benefit on clinical end points and might increase the risk of stroke. Concomitant administration of intravenous glycoprotein IIb/IIIa inhibitors and ischemic time did not seem to influence any potential benefits observed with aspiration thrombectomy." }, "25168472": { "QUESTION": "Can static foot posture measurements predict regional plantar surface area?", "CONTEXTS": [ "The intent of this study was to determine if the use of a single or combination of static foot posture measurements can be used to predict rearfoot, midfoot, and forefoot plantar surface area in individuals with pronated or normal foot types.", "Twelve foot measurements were collected on 52 individuals (mean age 25.8 years) with the change in midfoot width used to place subjects in a pronated or normal foot mobility group. Dynamic plantar contact area was collected during walking with a pressure sensor platform. The 12 measures were entered into a stepwise regression analysis to determine the optimal set of measures associated with regional plantar surface area.", "A two variable model was found to describe the relationship between the foot measurements and forefoot plantar contact area (r(2)=0.79, p<0.0001). A four variable model was found to describe the relationship between the foot measurements and midfoot plantar contact area (r(2)=0.85, p<0.0001) in those individuals with a 1.26cm or greater change in midfoot width." ], "LABELS": [ "BACKGROUND", "METHODS", "RESULTS" ], "MESHES": [ "Adult", "Biomechanical Phenomena", "Female", "Foot", "Humans", "Male", "Posture", "Predictive Value of Tests", "Pressure", "Weight-Bearing" ], "YEAR": "2014", "reasoning_required_pred": "yes", "reasoning_free_pred": "yes", "final_decision": "yes", "LONG_ANSWER": "The results indicate that clinicians can use a combination of simple, reliable and time efficient foot measures to explain 79% and 85% of the plantar surface area in the forefoot and midfoot, respectively." }, "9446993": { "QUESTION": "Can dentists recognize manipulated digital radiographs?", "CONTEXTS": [ "To determine the ability of dentists to recognize digitally manipulated radiographs.", "A poster was presented at the Annual Meeting of the German Society for Periodontology displaying the intra-oral radiographs of 12 different patients. Half of the radiographs were subjected to digital manipulation to add or remove specific features. Dentists were asked to identify these radiographs by means of a questionnaire.", "Thirty-nine dentists submitted usable questionnaires. Statistical evaluation revealed a distribution of hits similar to the random distribution. None of the dentists detected all the six manipulated radiographs; three dentists had five correct, but there were five with only one. An authentic radiograph scored highest as a manipulation." ], "LABELS": [ "OBJECTIVES", "METHODS", "RESULTS" ], "MESHES": [ "Clinical Competence", "Dentists", "Health Knowledge, Attitudes, Practice", "Humans", "Image Processing, Computer-Assisted", "Observer Variation", "Probability", "Radiography, Dental, Digital", "Sensitivity and Specificity", "Surveys and Questionnaires" ], "YEAR": "1997", "reasoning_required_pred": "no", "reasoning_free_pred": "no", "final_decision": "no", "LONG_ANSWER": "Image manipulations which alter the diagnostic content of a radiograph are unlikely to be detected by dentists. Digital radiography requires additional measures for data protection." }, "10473855": { "QUESTION": "Is delayed gastric emptying following pancreaticoduodenectomy related to pylorus preservation?", "CONTEXTS": [ "Delayed gastric emptying (DGE) is the most frequent postoperative complication after pylorus-preserving pancreaticoduodenectomy (PPPD). This prospective, non-randomized study was undertaken to determine whether the incidence of DGE may be reduced by modifying the original reconstructive anatomy with a retrocolic duodenojejunostomy towards an antecolic duodenojejunostomy.", "The study was comprised of 51 patients who underwent PPPD between August 1994 and November 1997. The operation was carried out as originally described but was modified by performing the duodenojejunostomy antecolically. Clinical data were recorded prospectively, with special regard to DGE.", "After PPPD, the nasogastric tube could be removed at a median of 2 days (range 1-22 days) postoperatively; in two patients, the nasogastric tube was reinserted because of vomiting and nausea. A liquid diet was started at a median of 5 days (3-11 days); the patients were able to tolerate a full, regular diet at a median of 10 days (7-28 days). The overall incidence of DGE was 12% (n=6). No postoperative complications other than DGE were exhibited by 36 patients (71%). In this group, DGE was only seen in one patient (3%). In the second group, where postoperative complications other than DGE occurred (n=15), five patients (30%) exhibited DGE (P=0.002)." ], "LABELS": [ "BACKGROUND", "PATIENTS AND METHODS", "RESULTS" ], "MESHES": [ "Female", "Gastric Emptying", "Humans", "Incidence", "Male", "Middle Aged", "Pancreaticoduodenectomy", "Postoperative Complications", "Prospective Studies", "Pylorus" ], "YEAR": "1999", "reasoning_required_pred": "yes", "reasoning_free_pred": "no", "final_decision": "no", "LONG_ANSWER": "DGE after PPPD seems to be of minor clinical importance following uncomplicated surgery. When taking the results into consideration, it can be said that, despite the lack of a control group, antecolic duodenojejunostomy might be the key to a low incidence of DGE after PPPD. In our experience, DGE is linked to the occurrence of other postoperative complications rather than to pylorus preservation." }, "28006766": { "QUESTION": "Is Overexpression of Ki-67 a Prognostic Biomarker of Upper Tract Urinary Carcinoma?", "CONTEXTS": [ "Upper tract urinary carcinoma (UTUC) is a relatively uncommon but aggressive disease. The Ki-67 antigen is a classic marker of cellular proliferation, but there is still controversy regarding the significance and importance of Ki-67 in tumor progression.", "In this study, we first detected Ki-67 expression in UTUC patients by immunohistochemistry (IHC). Subsequently, we quantitatively combined the results with those from the published literature in a meta-analysis after searching several databases.", "IHC results demonstrated that patients with muscle-invasive tumors (T2-T4) had higher Ki-67 expression than those with non-muscle-invasive tumors (Tis-T1), suggesting that high Ki-67 expression may be associated with the aggressive form of UTUC. Kaplan-Meier curves showed that patients with high Ki-67 expression had significantly poorer cancer-specific survival (CSS) and disease-free survival (DFS). Furthermore, multivariate analysis suggested that Ki-67 expression was an independent prognostic factor for CSS (hazard ratio, HR=3.196) and DFS (HR=3.517) in UTUC patients. Then, a meta-analysis of the published literature investigating Ki-67 expression and its effects on UTUC prognosis was conducted. After searching the PubMed, Medline, Embase, Cochrane Library and Scopus databases, 12 articles met the eligibility criteria for this analysis. The eligible studies included a total of 1740 patients with a mean number of 82 patients per study (range, 38-475). The combined results showed that increased Ki-67 levels were associated with poor survival and disease progression, with a pooled HR estimate of 2.081 and 2.791, respectively. In subgroup analysis, the pooled HR was statistically significant for cancer-specific survival (HR=2.276), metastasis-free survival (HR=3.008) and disease-free survival (HR=6.336)." ], "LABELS": [ "BACKGROUND", "METHODS", "RESULTS" ], "MESHES": [ "Aged", "Biomarkers, Tumor", "Disease Progression", "Disease-Free Survival", "Female", "Humans", "Immunohistochemistry", "Kaplan-Meier Estimate", "Ki-67 Antigen", "Male", "Multivariate Analysis", "Prognosis", "Retrospective Studies", "Urologic Neoplasms" ], "YEAR": "2016", "reasoning_required_pred": "yes", "reasoning_free_pred": "yes", "final_decision": "yes", "LONG_ANSWER": "In conclusion, high Ki-67 expression was associated with poor survival in patients with UTUC, as well as a high risk of disease progression, although these findings need to be interpreted with caution. Large-scale, adequately designed, prospective trials are needed to further confirm the value of Ki-67 in prognosis of UTUC patients." }, "11833948": { "QUESTION": "Does a delay in transfer to a rehabilitation unit for older people affect outcome after fracture of the proximal femur?", "CONTEXTS": [ "To detemine the relationship between delay in transfer to rehabilitation wards and outcome for patients aged over 75 years with fracture of the proximal femur.", "An observational study in a district general hospital of all patients admitted to hospital aged over 75 years with fracture of the proximal femur over 3 1/2 years. Outcome data collected included the number of patients discharged back to their usual residence and total hospital length of stay related to age, gender, usual residence and delay in transfer to a rehabilitation ward.", "58% of 455 patients were transferred to a rehabilitation ward. For those patients who were transferred to a rehabilitation ward only age predicted discharge to a more dependent residence. The relative risk for discharge to a more dependent residence for people aged over 85 years compared to younger people was 1.47 (95% CI 1.15-1.88). Delay in transfer to rehabilitation was associated with a longer total hospital length of stay of 0.64 (95% CI 0.23-1.05) days per day of delay in transfer." ], "LABELS": [ "AIMS", "METHODS", "RESULTS" ], "MESHES": [ "Aged", "Aged, 80 and over", "Female", "Femoral Neck Fractures", "Hospital Units", "Humans", "Length of Stay", "Logistic Models", "Male", "Patient Transfer", "Rehabilitation Centers", "Risk Factors", "Time Factors" ], "YEAR": "2001", "reasoning_required_pred": "yes", "reasoning_free_pred": "yes", "final_decision": "yes", "LONG_ANSWER": "Delay in transfer to a rehabilitation ward was associated with a disproportionate increase in total hospital length of stay for patients aged over 75 with fracture of the proximal femur." }, "27757987": { "QUESTION": "Does the treatment of amblyopia normalise subfoveal choroidal thickness in amblyopic children?", "CONTEXTS": [ "Recent studies have found a choroidal thickening in amblyopic eyes and suggested that there might be a relationship between the choroid and amblyopia. The present study aimed to evaluate the effect of a six-month treatment of amblyopia on choroidal thickness in anisometropic hyperopic amblyopic children.", "Thirty-two anisometropic hyperopic children with unilateral amblyopia were included in this prospective study. Subfoveal choroidal thickness was measured as the distance between the retinal pigment epithelium and the chorioscleral edge, by using spectral domain enhanced depth imaging optical coherence tomography. The treatment of amblyopia was performed based on the full correction of the refractive error with eyeglasses, a refractive adaptation phase and occlusion by patching the fellow eye.", "The mean visual acuity of the amblyopic eyes significantly increased from 0.35\u2009\u00b1\u20090.3 to 0.16\u2009\u00b1\u20090.2 logMAR after the treatment (p\u2009<\u20090.001). The mean initial choroidal thickness was significantly higher in the amblyopic eyes than in the fellow eyes (p\u2009=\u20090.019). There were no significant differences between the pre- and post-treatment mean choroidal thickness in the amblyopic eyes (p\u2009=\u20090.428) and in the fellow eyes (p\u2009=\u20090.343). The mean choroidal thickness was still higher in the amblyopic eyes than in the fellow eyes after the treatment (p\u2009=\u20090.006)." ], "LABELS": [ "PURPOSE", "METHODS", "RESULTS" ], "MESHES": [ "Adolescent", "Amblyopia", "Child", "Child, Preschool", "Choroid", "Female", "Humans", "Male", "Visual Acuity" ], "YEAR": "2017", "reasoning_required_pred": "no", "reasoning_free_pred": "no", "final_decision": "no", "LONG_ANSWER": "Although a six-month treatment of amblyopia increased the visual acuity of the anisometropic hyperopic amblyopic eyes, it could not significantly change choroidal thickness. Our results were in accordance with the conventional explanation, which suggests visual cortex and lateral geniculate nucleus abnormalities in the pathophysiology of amblyopia." }, "16772913": { "QUESTION": "A comparison of 500 prefilled textured saline breast implants versus 500 standard textured saline breast implants: is there a difference in deflation rates?", "CONTEXTS": [ "This study provides the first large-volume (1000 implant) comparison of the deflation rates of Poly Implant Prosthesis prefilled textured saline breast implants versus a control group of Mentor Siltex textured saline implants.", "A consecutive series of 500 Poly Implant Prosthesis prefilled textured saline breast implants was compared with a consecutive series of 500 Mentor Siltex breast implants. Each breast implant was evaluated for a 4-year period, and the annual deflation rate (number of deflations during a given year divided by the total number of implants) and cumulative deflation rate (cumulative total of deflations through a given year divided by the total number of implants) were recorded. Statistical significance was calculated using the Fisher's exact test at year 1 and the chi-square analysis at years 2 through 4.", "The cumulative deflation rates of the Poly Implant Prosthesis implants was as follows: year 1, 1.2 percent; year 2, 5.6 percent; year 3, 11.4 percent; and year 4, 15.4 percent. The cumulative deflation rates of the Mentor implants was: year 1, 0.2 percent; year 2, 0.6 percent; year 3, 1.6 percent; and year 4, 4.4 percent. At year 1, the difference between deflation rates was not statistically significant (Fisher's exact test, p>0.05). However, at year 2 (chi-square, 13.29; p<0.001), year 3 (chi-square, 37.91; p<0.001), and year 4 (chi-square, 32.69; p<0.001), the difference was statistically significant." ], "LABELS": [ "BACKGROUND", "METHODS", "RESULTS" ], "MESHES": [ "Breast Implantation", "Breast Implants", "Female", "Follow-Up Studies", "Humans", "Postoperative Complications", "Prosthesis Failure" ], "YEAR": "2006", "reasoning_required_pred": "yes", "reasoning_free_pred": "yes", "final_decision": "yes", "LONG_ANSWER": "There was a statistically significant difference between the overall deflation rates of Poly Implant Prosthesis prefilled textured saline breast implants and Mentor Siltex breast implants at year 2, year 3, and year 4. After 4 years, the 15.56 percent cumulative deflation rate of Poly Implant Prosthesis implants was over 3.5 times higher than the 4.31 percent deflation rate of the Mentor Siltex implants. There may be several factors contributing to the higher deflation rate seen in Poly Implant Prosthesis implants, including possible in vitro deflation before implantation and silicone shell curing technique. Nevertheless, this statistically significant deflation difference must be taken into account when balancing the risks and benefits of Poly Implant Prosthesis breast implants." }, "19575104": { "QUESTION": "Do foreign bodies migrate through the body towards the heart?", "CONTEXTS": [ "Fixation of foreign bodies (FB), in the mucosa, can favor its migration, giving origin to the popular saying: 'FB walk to the heart'.AIM: Describe the mechanisms involved in FB migration and how to diagnose them.", "From a sample of 3,000 foreign bodies, during 40 years, we analyzed four which had extra-lumen migration. We analyzed clinical, radiologic, endoscopic and ultrasound data collected at the medical documentation service.", "Three clinical histories are presented, describing two fish bones and one piece of fish cartilage. FB shifting was analyzed in all of them. Migration started in the esophagus in two, one going to the aorta and the other to the neck area. In the other two, migration started in the pharynx, and the FB moved towards the prevertebral fascia and the other externalized in the submandibular region. The mechanisms and the risks posed to the patient, by FB migration, and the way to diagnose them are hereby discussed." ], "LABELS": [ "UNLABELLED", "METHODOLOGY", "RESULTS" ], "MESHES": [ "Adult", "Esophagus", "Female", "Fiber Optic Technology", "Foreign-Body Migration", "Heart", "Humans", "Male", "Middle Aged", "Pharynx", "Tomography, X-Ray Computed" ], "YEAR": null, "reasoning_required_pred": "no", "reasoning_free_pred": "no", "final_decision": "no", "LONG_ANSWER": "The study allows us to determine that FB can move through the body but not towards the heart. The study also serves as a warning sign: in cases of prolonged histories of FB ingestion, imaging studies are mandatory before endoscopic examination." }, "24449622": { "QUESTION": "Is there a relationship between serum paraoxonase level and epicardial fat tissue thickness?", "CONTEXTS": [ "This study aimed to show the relationship between serum paraoxonase 1 level and the epicardial fat tissue thickness.", "Two hundred and seven patients without any atherosclerotic disease history were included in this cross-sectional observational study. Correlation analysis was performed to determine the correlation between epicardial fat tissue thickness, which was measured by echocardiography and serum paraoxonase 1 level. Also correlation analysis was performed to show correlation between patients' clinical and laboratory findings and the level of serum paraoxonase 1 (PON 1) and the epicardial fat tissue thickness. Pearson and Spearman test were used for correlation analysis.", "No linear correlation between epicardial fat tissue thickness and serum PON 1 found (correlation coefficient: -0.127, p=0.069). When epicardial fat tissue thickness were grouped as 7 mm and over, and below, and 5 mm and over, and below, serum PON 1 level were significantly lower in \u22657 mm group (PON1 : 168.9 U/L) than<7 mm group (PON 1: 253.9 U/L) (p<0.001). Also hypertension prevalence was increased in \u22657 mm group (p=0.001). Serum triglyceride was found to be higher in \u22657 mm group (p=0.014), body mass index was found higher in \u22655 mm group (p=0.006)." ], "LABELS": [ "OBJECTIVE", "METHODS", "RESULTS" ], "MESHES": [ "Adipose Tissue", "Aryldialkylphosphatase", "Coronary Artery Disease", "Cross-Sectional Studies", "Echocardiography", "Female", "Humans", "Male", "Middle Aged", "Pericardium" ], "YEAR": "2014", "reasoning_required_pred": "yes", "reasoning_free_pred": "no", "final_decision": "no", "LONG_ANSWER": "Serum PON 1 level is not correlated with the epicardial fat tissue thickness. But PON 1 level is lower in patients with epicardial fat tissue thickness 7 mm and over. Therefore, increased atherosclerosis progression can be found among patients with 7 mm and higher epicardial fat tissue thickness." }, "12095973": { "QUESTION": "Chemoradiation instead of surgery to treat mid and low rectal tumors: is it safe?", "CONTEXTS": [ "The main treatment for rectal carcinoma is surgery. Preoperative chemoradiation (CRT) is advocated to reduce local recurrence and improve resection of mid and low tethered rectal tumors.", "Fifty-two patients with mid or low rectal tumors underwent CRT (external beam radiation plus 5-fluorouracil plus folinic acid). Patients who had low rectal tumors with complete response (CR) were not submitted to surgical treatment. All other patients were submitted to surgery, independently of the response. Mean follow-up was 32.1 months.", "Five-year overall survival was 60.5%. Clinical evaluation after CRT showed CR in 10 cases (19.2%), all low tumors; incomplete response (>50%) in 21 (40.4%); and no response (<50%) in 19 (36.6%). Among the 10 cases with CR, 8 presented with local recurrence within 3.7 to 8.8 months. Two patients were not submitted to surgery and are still alive without cancer after 37 and 58 months. Thirty-nine patients had radical surgery. Seven had local recurrences after CRT plus surgery (17.9%). Overall survival was negatively affected by lymph node metastases (P =.017) and perineural invasion (P =.026)." ], "LABELS": [ "BACKGROUND", "METHODS", "RESULTS" ], "MESHES": [ "Adenocarcinoma", "Adult", "Aged", "Antimetabolites, Antineoplastic", "Combined Modality Therapy", "Female", "Fluorouracil", "Humans", "Leucovorin", "Male", "Middle Aged", "Neoplasm Recurrence, Local", "Radiotherapy Dosage", "Rectal Neoplasms", "Survival Rate" ], "YEAR": "2002", "reasoning_required_pred": "no", "reasoning_free_pred": "no", "final_decision": "no", "LONG_ANSWER": "Exclusive CRT approach is not safe to treat patients with low infiltrative rectal carcinoma." }, "23761381": { "QUESTION": "Is calibration the cause of variation in liquid chromatography tandem mass spectrometry testosterone measurement?", "CONTEXTS": [ "Testosterone measurement by liquid chromatography tandem mass spectrometry (LC-MS/MS) is well accepted as the preferred technique for the analysis of testosterone. Variation is seen between assays and this may be due to differences in calibration as commercial calibrators for this assay are not readily available. We investigated the effects calibration in routine clinical LC-MS/MS assays.", "All LC-MS/MS users that were registered with the UKNEQAS external quality assurance scheme for testosterone were invited to take part in the study. A set of seven serum samples and serum-based calibrators were sent to all laboratories that expressed an interest. The laboratories were instructed to analyse all samples using there own calibrators and return the results and a method questionnaire for analysis.", "Fifteen laboratories took part in the study. There was no consensus on supplier of testosterone or matrix for the preparation of calibrators and all were prepared in-house. Also, a wide variety of mass spectrometers, internal standards, chromatography conditions and sample extractions were used. The variation in results did not improve when the results were corrected with a common calibrator." ], "LABELS": [ "BACKGROUND", "METHODS", "RESULTS" ], "MESHES": [ "Calibration", "Chromatography, Liquid", "Female", "Humans", "Male", "Tandem Mass Spectrometry", "Testosterone" ], "YEAR": "2013", "reasoning_required_pred": "no", "reasoning_free_pred": "no", "final_decision": "no", "LONG_ANSWER": "The variation in results obtained could not be attributed to variations in calibrators. The differences in methodologies between laboratories must be the reason for this variation." }, "16971978": { "QUESTION": "Are complex coronary lesions more frequent in patients with diabetes mellitus?", "CONTEXTS": [ "Coronary atherosclerotic burden is excessive in diabetic patients. Diabetes mellitus (DM) is an independent predictor for both death and myocardial infarction. It is not known whether the prevalence of complex coronary lesions, such as bifurcation and ostial lesions, is different in diabetics from nondiabetics.", "The aim of present study was to investigate the prevalence of these lesions in patients with DM.", "One thousand fourteen consecutive patients (mean age 61.3+/-10.7 years) were investigated. Coronary angiograms were examined for bifurcation and ostial lesions using a digital quantitative system. Patients were classified as diabetic (n=281) or nondiabetic (n=733).", "Patient mean age, and rates of hypertension and hyperlipidemia were significantly higher in the diabetic group than in the nondiabetic group (P<0.0001), although smoking was significantly lower (P=0.001). Reasons for coronary angiography and treatment were comparable between the two groups. The prevalence of bifurcation lesions and ostial lesions was significantly greater in the diabetic group than in the nondiabetic group (9.8% versus 4.3% [P=0.001] and 38.4% versus 29.2% [P=0.003]in the diabetic group versus the nondiabetic group). The presence of DM and greater age were found to be independent predictors for bifurcation lesions (OR=2.27 [P=0.004] and OR=1.03 [P=0.01], for DM and age, respectively) and ostial lesions (OR=1.40 [P=0.027] and OR=1.02 [P=0.001], for DM and age, respectively) in multivariate analysis." ], "LABELS": [ "BACKGROUND", "OBJECTIVE", "METHODS", "RESULTS" ], "MESHES": [ "Coronary Angiography", "Coronary Artery Disease", "Diabetes Mellitus", "Female", "Humans", "Hyperlipidemias", "Hypertension", "Male", "Middle Aged", "Prevalence", "Turkey" ], "YEAR": "2006", "reasoning_required_pred": "yes", "reasoning_free_pred": "yes", "final_decision": "yes", "LONG_ANSWER": "Complex coronary lesions such as bifurcation and ostial lesions were significantly more common in diabetic patients than in nondiabetic patients. Greater age and the presence of DM were independent predictors for these complex lesions. These results may help to explain the poor prognosis of coronary artery disease among diabetic patients." }, "10375486": { "QUESTION": "Are variations in the use of carotid endarterectomy explained by population Need?", "CONTEXTS": [ "to describe variation in utilisation of carotid endarterectomy (CEA) within two English health regions and explore relationships between use, need and proximity to services.", "consecutive case series of operations. Comparison at a population level with district stroke mortality, hospital admissions and material deprivation.", "standardised utilisation rates for CEA and measures of inter-district variability. Spearman's rank correlation coefficients for associations between variables.", "variation in utilisation rates was considerable (14-fold difference across district populations). More individuals had bilateral surgery in the Yorkshire region than in the Northern (11.7% vs. 5.5%, p=0.002). There was no association between utilisation rates for CEA and district stroke mortality (r=-0.06, 95% CI -0.41 to 0.30) or admission rates for stroke (r=0.17, 95% CI -0.2 to 0.49). There was a strong relationship between residence in districts where services were located and higher utilisation. Rates of CEA were lowest in the regions' most affluent wards." ], "LABELS": [ "OBJECTIVES", "DESIGN", "MAIN OUTCOME MEASURES", "RESULTS" ], "MESHES": [ "Aged", "Cerebrovascular Disorders", "Endarterectomy, Carotid", "England", "Female", "Health Services Accessibility", "Health Services Needs and Demand", "Hospitalization", "Humans", "Male", "Middle Aged" ], "YEAR": "1999", "reasoning_required_pred": "no", "reasoning_free_pred": "no", "final_decision": "no", "LONG_ANSWER": "use of CEA varies widely, depending on area of residence. Variation is not a consequence of differences in need, but reflects clinical practice and supply of services. There is evidence to suggest unmet need for CEA." }, "21558951": { "QUESTION": "Are there effects of intrauterine cocaine exposure on delinquency during early adolescence?", "CONTEXTS": [ "To ascertain whether level of intrauterine cocaine exposure (IUCE) is associated with early adolescent delinquent behavior, after accounting for prenatal exposures to other psychoactive substances and relevant psychosocial factors.", "Ninety-three early adolescents (12.5-14.5 years old) participating since birth in a longitudinal study of IUCE reported delinquent acts via an audio computer-assisted self-interview. Level of IUCE and exposure to cigarettes, alcohol, and marijuana were determined by maternal report, maternal and infant urine assays, and infant meconium assays at birth. Participants reported their exposure to violence on the Violence Exposure Scale for Children-Revised at ages 8.5, 9.5, and 11 years and during early adolescence, and the strictness of supervision by their caregivers during early adolescence.", "Of the 93 participants, 24 (26%) reported \u2265 3 delinquent behaviors during early adolescence. In the final multivariate model (including level of IUCE and cigarette exposure, childhood exposure to violence, and caregiver strictness/supervision) \u2265 3 delinquent behaviors were not significantly associated with level of IUCE but were significantly associated with intrauterine exposure to half a pack or more of cigarettes per day and higher levels of childhood exposure to violence, effects substantially unchanged after control for early adolescent violence exposure." ], "LABELS": [ "OBJECTIVE", "METHODS", "RESULTS" ], "MESHES": [ "Adolescent", "Chi-Square Distribution", "Child", "Cocaine", "Cocaine-Related Disorders", "Female", "Humans", "Juvenile Delinquency", "Longitudinal Studies", "Male", "Multivariate Analysis", "Odds Ratio", "Pregnancy", "Prenatal Exposure Delayed Effects", "Smoking", "Socioeconomic Factors", "Violence" ], "YEAR": "2011", "reasoning_required_pred": "yes", "reasoning_free_pred": "no", "final_decision": "no", "LONG_ANSWER": "In this cohort, prospectively ascertained prenatal exposure to cigarettes and childhood exposure to violence are associated with self-reported delinquent behaviors during early adolescence. Contrary to initial popular predictions, intrauterine cocaine is not a strong predictor of adolescent delinquent behaviors in this cohort." }, "15528969": { "QUESTION": "Is expert breast pathology assessment necessary for the management of ductal carcinoma in situ ?", "CONTEXTS": [ "Current guidelines include a recommendation that a pathologist with expertise in breast disease review all ductal carcinoma in situ (DCIS) specimens due to the presence of significant variability in pathologic reporting of DCIS. The objective of this study was to evaluate the completeness and accuracy of pathologic reporting of DCIS over the past decade and to determine the current impact of expert breast pathology assessment on the management of DCIS.", "All patients with a diagnosis of DCIS referred to a single regional cancer centre between 1982 and 2000 have been reviewed. Inter-observer variability between initial and secondary reports has been evaluated using kappa statistics. For each case, the Van Nuys Prognostic Index (VNPI) using pathologic data obtained from the initial and reviewed pathology reports were compared. The impact of expert breast pathology on risk assessment and treatment was determined.", "481 individuals with DCIS were referred and pathology review was performed on 350 patients (73%). Inter-observer agreement was high for the main pathologic features of DCIS. From 1996 to 2000, secondary pathology assessments lead to a change in the assessment of local recurrence risk in 100 cases (29%) and contributed to a change in treatment recommendation in 93 (43%) cases." ], "LABELS": [ "BACKGROUND", "METHODS", "RESULTS" ], "MESHES": [ "Adult", "Aged", "Aged, 80 and over", "Biopsy", "Breast Neoplasms", "Carcinoma, Intraductal, Noninfiltrating", "Decision Support Techniques", "Female", "Humans", "Medical Records", "Middle Aged", "Observer Variation", "Ontario", "Predictive Value of Tests", "Retrospective Studies" ], "YEAR": "2004", "reasoning_required_pred": "yes", "reasoning_free_pred": "yes", "final_decision": "yes", "LONG_ANSWER": "Expert breast pathology assessments continue to be necessary in the management of DCIS." }, "17062225": { "QUESTION": "Thoracic and thoracoabdominal aneurysm repair: is reimplantation of spinal cord arteries a waste of time?", "CONTEXTS": [ "The impact of different strategies for management of intercostal and lumbar arteries during repair of thoracic and thoracoabdominal aortic aneurysms (TAA/A) on the prevention of paraplegia remains poorly understood.", "One hundred consecutive patients with intraoperative monitoring of motor evoked potentials (MEP) and somatosensory evoked potentials (SSEP) during TAA/A repair involving serial segmental artery sacrifice (October 2002 to December 2004) were reviewed.", "Operative mortality was 6%. The median intensive care unit stay was 2.5 days (IQ range: 1-4 days), and the median hospital stay 10.0 days (IQ range: 8-17 days). Potentials remained unchanged during the course of serial segmental artery sacrifice, or could be returned to baseline levels by anesthetic and blood pressure manipulation, in 99 of 100 cases. An average of 8.0 +/- 2.6 segmental artery pairs were sacrificed overall, with an average of 4.5 +/- 2.1 segmental pairs sacrificed between T7 and L1, where the artery of Adamkiewicz is presumed to arise. Postoperative paraplegia occurred in 2 patients. In 1, immediate paraplegia was precipitated by an intraoperative dissection, resulting in 6 hours of lower body ischemia. A second ambulatory patient had severe paraparesis albeit normal cerebral function after resuscitation from a respiratory arrest." ], "LABELS": [ "BACKGROUND", "METHODS", "RESULTS" ], "MESHES": [ "Adult", "Aged", "Aged, 80 and over", "Aortic Aneurysm, Abdominal", "Aortic Aneurysm, Thoracic", "Arteries", "Blood Vessel Prosthesis Implantation", "Evoked Potentials, Motor", "Evoked Potentials, Somatosensory", "Female", "Humans", "Male", "Middle Aged", "Monitoring, Intraoperative", "Paraplegia", "Replantation", "Retrospective Studies", "Spinal Cord" ], "YEAR": "2006", "reasoning_required_pred": "yes", "reasoning_free_pred": "yes", "final_decision": "yes", "LONG_ANSWER": "With monitoring of MEP and SSEP, sacrifice--without reimplantation--of as many as 15 intercostal and lumbar arteries during TAA/A repair is safe, resulting in acceptably low rates of immediate and delayed paraplegia. This experience suggests that routine surgical implantation of segmental vessels is not indicated, and that, with evolving understanding of spinal cord perfusion, endovascular repair of the entire thoracic aorta should ultimately be possible without spinal cord injury." }, "14713788": { "QUESTION": "Is year of radical prostatectomy a predictor of outcome in prostate cancer?", "CONTEXTS": [ "We examined whether the year in which radical prostatectomy (RP) was performed is a predictor of treatment outcome after controlling for standard prognostic factors.", "We examined the association between RP year and outcome in 6,556 patients from 7 centers using preoperative and pathological features. Patients underwent surgery between 1985 and 2000. The variables analyzed were RP year, clinical stage, pretreatment prostate specific antigen, biopsy Gleason sum, RP Gleason sum, margin status, level of extracapsular extension, seminal vesicle status, lymph node status, neoadjuvant hormones and adjuvant therapy. Median followup was 23 months (maximum 166). Separate Cox multivariate regression analyses were performed to analyze preoperative and postoperative factors.", "RP year was a predictor of outcome on preoperative analysis (p = 0.006) but not on postoperative analysis (p = 0.130). Patient outcome steadily improved with surgery through the mid 1990s and then it appeared to level off." ], "LABELS": [ "PURPOSE", "MATERIALS AND METHODS", "RESULTS" ], "MESHES": [ "Adult", "Aged", "Aged, 80 and over", "Follow-Up Studies", "Humans", "Male", "Middle Aged", "Prostatectomy", "Prostatic Neoplasms", "Time Factors", "Treatment Outcome" ], "YEAR": "2004", "reasoning_required_pred": "no", "reasoning_free_pred": "yes", "final_decision": "yes", "LONG_ANSWER": "When controlling for preoperative features, the year in which RP was performed is a predictor of outcome on multivariate analysis. This effect could not be explained by stage migration." }, "24374414": { "QUESTION": "Does health information exchange reduce redundant imaging?", "CONTEXTS": [ "Broad-based electronic health information exchange (HIE), in which patients' clinical data follow them between care delivery settings, is expected to produce large quality gains and cost savings. Although these benefits are assumed to result from reducing redundant care, there is limited supporting empirical evidence.", "To evaluate whether HIE adoption is associated with decreases in repeat imaging in emergency departments (EDs).DATA SOURCE/", "ED discharge data from the State Emergency Department Databases for California and Florida for 2007-2010 were merged with Health Information Management Systems Society data that report hospital HIE participation.", "Using regression with ED fixed effects and trends, we performed a retrospective analysis of the impact of HIE participation on repeat imaging, comparing 37 EDs that initiated HIE participation during the study period to 410 EDs that did not participate in HIE during the same period. Within 3 common types of imaging tests [computed tomography (CT), ultrasound, and chest x-ray), we defined a repeat image for a given patient as the same study in the same body region performed within 30 days at unaffiliated EDs.", "In our sample there were 20,139 repeat CTs (representing 14.7% of those cases with CT in the index visit), 13,060 repeat ultrasounds (20.7% of ultrasound cases), and 29,703 repeat chest x-rays (19.5% of x-ray cases). HIE was associated with reduced probability of repeat ED imaging in all 3 modalities: -8.7 percentage points for CT [95% confidence interval (CI): -14.7, -2.7], -9.1 percentage points for ultrasound (95% CI: -17.2, -1.1), and -13.0 percentage points for chest x-ray (95% CI: -18.3, -7.7), reflecting reductions of 44%-67% relative to sample means." ], "LABELS": [ "BACKGROUND", "OBJECTIVE", "STUDY SETTING", "METHODS", "RESULTS" ], "MESHES": [ "Adult", "Age Factors", "California", "Continuity of Patient Care", "Diagnostic Imaging", "Electronic Health Records", "Emergency Service, Hospital", "Female", "Florida", "Humans", "Male", "Middle Aged", "Retrospective Studies", "Sex Factors", "Socioeconomic Factors" ], "YEAR": "2014", "reasoning_required_pred": "yes", "reasoning_free_pred": "yes", "final_decision": "yes", "LONG_ANSWER": "HIE was associated with reduced repeat imaging in EDs. This study is among the first to find empirical support for this anticipated benefit of HIE." }, "20736887": { "QUESTION": "Is decompressive surgery effective for spinal cord sarcoidosis accompanied with compressive cervical myelopathy?", "CONTEXTS": [ "A retrospective multicenter study of series of 12 patients with spinal cord sarcoidosis who underwent surgery.", "To evaluate the postoperative outcomes of patients with cervical spinal cord sarcoidosis accompanied with compressive myelopathy and effect of decompressive surgery on the prognosis of sarcoidosis.", "Sarcoidosis is a chronic, multisystem noncaseating granulomatous disease. It is difficult to differentiate spinal cord sarcoidosis from cervical compressive myelopathy. There are no studies regarding the coexistence of compressive cervical myelopathy with cervical spinal cord sarcoidosis and the effect of decompressive surgery.", "Nagoya Spine Group database included 1560 cases with cervical myelopathy treated with cervical laminectomy or laminoplasty from 2001 to 2005. A total of 12 patients (0.08% of cervical myelopathy) were identified spinal cord sarcoidosis treated with decompressive surgery. As a control subject, 8 patients with spinal cord sarcoidosis without compressive lesion who underwent high-dose steroid therapy without surgery were recruited.", "In the surgery group, enhancing lesions on magnetic resonance imaging (MRI) were mostly seen at C5-C6, coincident with the maximum compression level in all cases. Postoperative recovery rates in the surgery group at 1 week and 4 weeks were -7.4% and -1.1%, respectively. Only 5 cases had showed clinical improvement, and the condition of these 5 patients had worsened again at averaged 7.4 weeks after surgery. Postoperative oral steroid therapy was initiated at an average of 6.4 weeks and the average initial dose was 54.0 mg in the surgery group, while 51.3 mg in the nonsurgery group. The recovery rate of the Japanese Orthopedic Association score, which increased after steroid therapy, was better in the nonsurgery group (62.5%) than in the surgery group (18.6%) with significant difference (P<0.01)." ], "LABELS": [ "STUDY DESIGN", "OBJECTIVE", "SUMMARY OF BACKGROUND DATA", "METHODS", "RESULTS" ], "MESHES": [ "Adrenal Cortex Hormones", "Aged", "Cervical Vertebrae", "Databases, Factual", "Decompression, Surgical", "Female", "Humans", "Magnetic Resonance Imaging", "Male", "Middle Aged", "Sarcoidosis", "Spinal Cord Compression", "Spinal Cord Diseases", "Treatment Outcome" ], "YEAR": "2010", "reasoning_required_pred": "no", "reasoning_free_pred": "no", "final_decision": "no", "LONG_ANSWER": "The effect of decompression for spinal cord sarcoidosis with compressive myelopathy was not evident. Early diagnosis for sarcoidosis from other organ and steroid therapy should be needed." }, "22504515": { "QUESTION": "Endovenous laser ablation in the treatment of small saphenous varicose veins: does site of access influence early outcomes?", "CONTEXTS": [ "The study was performed to evaluate the clinical and technical efficacy of endovenous laser ablation (EVLA) of small saphenous varicosities, particularly in relation to the site of endovenous access.", "Totally 59 patients with unilateral saphenopopliteal junction incompetence and small saphenous vein reflux underwent EVLA (810 nm, 14 W diode laser) with ambulatory phlebectomies. Small saphenous vein access was gained at the lowest site of truncal reflux. Patients were divided into 2 groups: access gained above mid-calf (AMC, n = 33) and below mid-calf (BMC, n = 26) levels. Outcomes included Venous Clinical Severity Scores (VCSS), Aberdeen Varicose Vein Questionnaire (AVVQ), patient satisfaction, complications, and recurrence rates.", "Both groups demonstrated significant improvement in VCSS, AVVQ, generic quality of life Short Form 36, and EuroQol scores (P<.05) up to 1 year. No differences were seen between AMC and BMC groups for complications (phlebitis: 2 [6%] and 1 [3.8%], P>.05; paresthesia: 2 [6%] and 5 [19%], P = .223) and recurrence (3 [9%] and 1 [3.8%], P = .623), respectively." ], "LABELS": [ "OBJECTIVE", "METHODS", "RESULTS" ], "MESHES": [ "Adult", "Ambulatory Surgical Procedures", "Chi-Square Distribution", "Endovascular Procedures", "England", "Female", "Humans", "Laser Therapy", "Lasers, Semiconductor", "Male", "Middle Aged", "Paresthesia", "Patient Satisfaction", "Peripheral Nerve Injuries", "Phlebitis", "Prospective Studies", "Quality of Life", "Recurrence", "Risk Assessment", "Risk Factors", "Saphenous Vein", "Severity of Illness Index", "Surveys and Questionnaires", "Time Factors", "Treatment Outcome", "Varicose Veins" ], "YEAR": "2012", "reasoning_required_pred": "no", "reasoning_free_pred": "no", "final_decision": "no", "LONG_ANSWER": "The site of access in our study does not appear to influence complications specifically neural injury or recurrence rates." }, "10577397": { "QUESTION": "Is a pressor necessary during aortic perfusion and oxygenation therapy of cardiac arrest?", "CONTEXTS": [ "Occlusion of the descending aorta and infusion of oxygenated ultrapurified polymerized bovine hemoglobin may improve the efficacy of advanced cardiac life support (ACLS). Because selective aortic perfusion and oxygenation (SAPO) directly increases coronary perfusion pressure, exogenous epinephrine may not be required. The purpose of this study was to determine whether exogenous epinephrine is necessary during SAPO by comparing the rate of return of spontaneous circulation and aortic and coronary perfusion pressures during ACLS-SAPO in animals treated with either intra-aortic epinephrine or saline solution.", "A prospective, randomized, interventional before-after trial with a canine model of ventricular fibrillation cardiac arrest and ACLS based on external chest compression was performed. The ECG, right atrial, aortic arch, and esophageal pulse pressures were measured continuously. A descending aortic occlusion balloon catheter was placed through the femoral artery. Ventricular fibrillation was induced, and no therapy was given during the 10-minute arrest time. Basic life support was then initiated and normalized by standardization of esophageal pulse pressure and central aortic blood gases. After 3 minutes of basic life support, the aortic occlusion balloon was inflated, and 0.01 mg/kg epinephrine or saline solution was administered through the aortic catheter followed by 450 mL of ultrapurified polymerized bovine hemoglobin over 2 minutes. Defibrillation was then attempted. The outcomes and changes in intravascular pressures were compared.", "Aortic pressures were higher during infusions in animals treated with epinephrine. During infusion, the mean aortic relaxation pressure increased by 58+/-5 mm Hg in animals that had received epinephrine versus 20+/-11 mm Hg in those that had received saline placebo. The coronary perfusion pressure during infusion increased by 52+/-8 mm Hg in animals that had received epinephrine versus 26+/-10 mm Hg in those that had received saline. Only 2 of 7 animals in the placebo group had return of spontaneous circulation versus 7 of 8 in the epinephrine group." ], "LABELS": [ "STUDY OBJECTIVE", "METHODS", "RESULTS" ], "MESHES": [ "Adrenergic Agonists", "Animals", "Aorta", "Blood Pressure", "Cardiopulmonary Resuscitation", "Coronary Circulation", "Dogs", "Epinephrine", "Heart Arrest", "Injections, Intra-Arterial", "Prospective Studies", "Random Allocation", "Vasoconstrictor Agents" ], "YEAR": "1999", "reasoning_required_pred": "yes", "reasoning_free_pred": "yes", "final_decision": "yes", "LONG_ANSWER": "The addition of epinephrine to ACLS-SAPO increases vital organ perfusion pressures and improves outcome from cardiac arrest. There appears to be a profound loss of arterial vasomotor tone after prolonged arrest. This loss of vasomotor tone may make exogenous pressors necessary for resuscitation after prolonged cardiac arrest." }, "15369037": { "QUESTION": "Do people who were passive smokers during childhood have increased risk of long-term work disability?", "CONTEXTS": [ "Regular inhalation of tobacco smoke, whether it be voluntary or not, may have profound negative effects on the body. Also intervertebral discs may be affected. The objective of the present study was to test the hypothesis that nurses' aides who were exposed to environmental tobacco smoke (ETS) at home during childhood have an increased risk of long-term sick leave.", "The sample comprised 5563 Norwegian nurses' aides, not on sick leave when they completed a mailed questionnaire in 1999. Of these, 4744 (85.3%) completed a second questionnaire 15 months later. The outcome measure was the incidence proportion of long-term sick leave during the 12 months prior to the follow-up.", "Respondents who reported at baseline that they had been exposed to ETS at home during childhood had increased risk of sick leave exceeding 14 days attributed to neck pain (odds ratio (OR) = 1.34; 95% confidence interval (CI): 1.04-1.73), high back pain (OR=1.49; CI: 1.07-2.06), low back pain (OR=1.21; CI: 0.97-1.50), and any illness (OR=1.23; CI: 1.07-1.42), after adjustments for demographic and familial characteristics, former smoking, current smoking, physical leisure-time activities, work factors, prior neck injury, and affective symptoms. They also had increased risk of sick leave exceeding 8 weeks (OR=1.29; CI: 1.08-1.55)." ], "LABELS": [ "BACKGROUND", "METHODS", "RESULTS" ], "MESHES": [ "Adult", "Affective Symptoms", "Age Factors", "Aged", "Confidence Intervals", "Exercise", "Female", "Follow-Up Studies", "Humans", "Leisure Activities", "Low Back Pain", "Male", "Middle Aged", "Neck Injuries", "Neck Pain", "Norway", "Nurses' Aides", "Odds Ratio", "Prospective Studies", "Risk Factors", "Sick Leave", "Surveys and Questionnaires", "Time Factors", "Tobacco Smoke Pollution" ], "YEAR": "2004", "reasoning_required_pred": "yes", "reasoning_free_pred": "yes", "final_decision": "yes", "LONG_ANSWER": "The study supports the hypothesis that nurses' aides who were exposed to ETS at home during childhood have an increased risk of long-term sickness absence." }, "24073931": { "QUESTION": "Is the covering of the resection margin after distal pancreatectomy advantageous?", "CONTEXTS": [ "In recent years, many advances in pancreatic surgery have been achieved. Nevertheless, the rate of pancreatic fistula following pancreatic tail resection does not differ between various techniques, still reaching up to 30% in prospective multicentric studies. Taking into account contradictory results concerning the usefulness of covering resection margins after distal pancreatectomy, we sought to perform a systematic, retrospective analysis of patients that underwent distal pancreatectomy at our center.", "We retrospectively analysed the data of 74 patients that underwent distal pancreatectomy between 2001 and 2011 at the community hospital in Neuss. Demographic factors, indications, postoperative complications, surgical or interventional revisions, and length of hospital stay were registered to compare the outcome of patients undergoing distal pancreatectomy with coverage of the resection margins vs. patients undergoing distal pancreatectomy without coverage of the resection margins. Differences between groups were calculated using Fisher's exact and Mann-Whitney U test.", "Main indications for pancreatic surgery were insulinoma (n=18, 24%), ductal adenocarcinoma (n=9, 12%), non-single-insulinoma-pancreatogenic-hypoglycemia-syndrome (NSIPHS) (n=8, 11%), and pancreatic cysts with pancreatitis (n=8, 11%). In 39 of 74 (53%) patients no postoperative complications were noted. In detail we found that 23/42 (55%) patients with coverage vs. 16/32 (50%) without coverage of the resection margins had no postoperative complications. The most common complications were pancreatic fistulas in eleven patients (15%), and postoperative bleeding in nine patients (12%). Pancreatic fistulas occurred in patients without coverage of the resection margins in 7/32 (22%) vs. 4/42 (1011%) with coverage are of the resection margins, yet without reaching statistical significance. Postoperative bleeding ensued with equal frequency in both groups (12% with coverage versus 13% without coverage of the resection margins). The reoperation rate was 8%. The hospital stay for patients without coverage was 13 days (5-60) vs. 17 days (8-60) for patients with coverage." ], "LABELS": [ "BACKGROUND", "METHODS", "RESULTS" ], "MESHES": [ "Adolescent", "Adult", "Aged", "Aged, 80 and over", "Child", "Demography", "Female", "Humans", "Male", "Middle Aged", "Pancreas", "Pancreatectomy", "Pancreatic Fistula", "Perioperative Care", "Postoperative Complications", "Young Adult" ], "YEAR": "2013", "reasoning_required_pred": "no", "reasoning_free_pred": "no", "final_decision": "no", "LONG_ANSWER": "The results show no significant difference in the fistula rate after covering of the resection margin after distal pancreatectomy, which contributes to the picture of an unsolved problem." }, "18472368": { "QUESTION": "Does treatment duration affect outcome after radiotherapy for prostate cancer?", "CONTEXTS": [ "The protraction of external beam radiotherapy (RT) time is detrimental in several disease sites. In prostate cancer, the overall treatment time can be considerable, as can the potential for treatment breaks. We evaluated the effect of elapsed treatment time on outcome after RT for prostate cancer.", "Between April 1989 and November 2004, 1,796 men with prostate cancer were treated with RT alone. The nontreatment day ratio (NTDR) was defined as the number of nontreatment days divided by the total elapsed days of RT. This ratio was used to account for the relationship between treatment duration and total RT dose. Men were stratified into low risk (n = 789), intermediate risk (n = 798), and high risk (n = 209) using a single-factor model.", "The 10-year freedom from biochemical failure (FFBF) rate was 68% for a NTDR<33% vs. 58% for NTDR>/=33% (p = 0.02; BF was defined as a prostate-specific antigen nadir + 2 ng/mL). In the low-risk group, the 10-year FFBF rate was 82% for NTDR<33% vs. 57% for NTDR>/=33% (p = 0.0019). The NTDR was independently predictive for FFBF (p = 0.03), in addition to T stage (p = 0.005) and initial prostate-specific antigen level (p<0.0001) on multivariate analysis, including Gleason score and radiation dose. The NTDR was not a significant predictor of FFBF when examined in the intermediate-risk group, high-risk group, or all risk groups combined." ], "LABELS": [ "PURPOSE", "METHODS AND MATERIALS", "RESULTS" ], "MESHES": [ "Humans", "Male", "Neoplasm Staging", "Physical Examination", "Prostatic Neoplasms", "Radiotherapy", "Radiotherapy, Conformal", "Retrospective Studies", "Risk Assessment", "Treatment Outcome" ], "YEAR": "2008", "reasoning_required_pred": "yes", "reasoning_free_pred": "yes", "final_decision": "yes", "LONG_ANSWER": "A proportionally longer treatment duration was identified as an adverse factor in low-risk patients. Treatment breaks resulting in a NTDR of>/=33% (e.g., four or more breaks during a 40-fraction treatment, 5 d/wk) should be avoided." }, "23992109": { "QUESTION": "Is the urinary biomarkers assessment a non-invasive approach to tubular lesions of the solitary kidney?", "CONTEXTS": [ "The solitary kidney (SK) is currently debated in the literature, as living kidney donation is extensively used and the diagnosis of congenital SK is frequent. Tubulointerstitial lesions associated with adaptive phenomena may occur early within the SK.", "Analysis of the significance of urinary biomarkers in the assessment of tubulointerstitial lesions of the SK.", "A cross-sectional study of 37 patients with SK included 18 patients-acquired SK (mean age 56.44\u2009\u00b1\u200912.20 years, interval from nephrectomy 10.94\u2009\u00b1\u20099.37 years), 19 patients-congenital SK (mean age 41.52\u2009\u00b1\u200910.54 years). Urinary NAG, urinary alpha-1-microglobulin, albuminuria, eGFR (CKD-EPI equation) were measured.", "In acquired SK, NAG increased in 60.66%, urinary alpha 1-microglobulin in 16.66%, albuminuria in 55.55% of patients. Inverse correlation with eGFR presented NAG (R(2\u2009)=\u20090.537, p\u2009=\u20090.022), urinary alpha 1-microglobulin (R(2\u2009)=\u20090.702, p\u2009=\u20090.001), albuminuria (R(2\u2009)=\u20090.655, p\u2009=\u20090.003). In congenital SK, NAG increased in 52.63%, urinary alpha 1-microglobulin in 5.26%, albuminuria in 47.36% of patients. In this group, urinary biomarkers correlated inversely with eGFR: NAG (R(2\u2009)=\u20090.743, p\u2009<\u20090.001), urinary alpha 1-microglobulin (R(2\u2009)=\u20090.701, p\u2009=\u20090.001), albuminuria (R(2\u2009)=\u20090.821, p\u2009<\u20090.001). Significant correlations were found between the urinary biomarkers in both groups." ], "LABELS": [ "INTRODUCTION", "AIMS", "METHODS", "RESULTS" ], "MESHES": [ "Acetylglucosaminidase", "Adult", "Aged", "Albuminuria", "Alpha-Globulins", "Biomarkers", "Cross-Sectional Studies", "Female", "Glomerular Filtration Rate", "Humans", "Kidney Diseases", "Male", "Middle Aged" ], "YEAR": "2013", "reasoning_required_pred": "yes", "reasoning_free_pred": "yes", "final_decision": "yes", "LONG_ANSWER": "Urinary biomarkers allow a non-invasive, sensitive, early assessment of the tubular lesions of the SK. Urinary biomarkers of PT injury parallel renal function decline, thus complementing the estimation of GFR. Monitoring of PT dysfunction is mandatory in patients with SK." }, "21164063": { "QUESTION": "Is there a role for fondaparinux in perioperative bridging?", "CONTEXTS": [ "A possible role for fondaparinux as a bridging agent in the perioperative setting is explored.", "Anticoagulation guidelines provide minimal direction on the perioperative use of fondaparinux. Fondaparinux's extended half-life of 17-21 hours complicates its use as a perioperative bridging therapy. The ideal time for discontinuation before surgery is an issue, particularly in surgeries with a high bleeding risk or in which neuraxial anesthesia is used. Guidance for perioperative bridging with fondaparinux must be derived from pharmacokinetic data, surgical prophylaxis trials, case reports, and anesthesia guidelines. Published trials used fondaparinux sodium 2.5 mg daily for venous thromboembolism prophylaxis in surgical patients, and the majority avoided its use before surgery in patients receiving neuraxial anesthesia. Three case reports cited the use of fondaparinux sodium as perioperative bridge therapy; one used a 2.5-mg dose, and the other two used a full treatment dose of 7.5 mg. Furthermore, professional anesthesia guidelines conflict in their recommendations regarding the timing of drug administration with neuraxial catheter use. For these reasons, it may be optimal to avoid fondaparinux use before surgery. In some instances, the use of low-molecular-weight heparin or inpatient use of i.v. unfractionated heparin is not possible, is contraindicated, or has limited efficacy, such as a patient with history of heparin-induced thrombocytopenia or antithrombin III deficiency. Fondaparinux may have a role in bridge therapy for these patients." ], "LABELS": [ "PURPOSE", "SUMMARY" ], "MESHES": [ "Anticoagulants", "Humans", "Perioperative Care", "Polysaccharides", "Practice Guidelines as Topic", "Surgical Procedures, Operative" ], "YEAR": "2011", "reasoning_required_pred": "maybe", "reasoning_free_pred": "no", "final_decision": "no", "LONG_ANSWER": "The role of fondaparinux in perioperative bridge therapy has not been established, and there are some important limitations to its use as a routine bridging agent." }, "21951591": { "QUESTION": "Motor performance in chronic low back pain: is there an influence of pain-related cognitions?", "CONTEXTS": [ "Chronic low back pain (CLBP) is often accompanied by an abnormal motor performance. However, it has not been clarified yet whether these deviations also occur during motor tasks not involving the back and whether the performance is influenced by pain and pain-related cognitions. Therefore, the aim of the present study is to get insight in the contribution of both pain experience and pain-related cognitions to general motor task performance in CLBP.", "13 CLBP patients and 15 healthy subjects performed a hand-function task in three conditions: sitting, lying prone (lying) and lying prone without trunk support (provoking). The last condition was assumed to provoke pain-related cognitions, which was considered successful when a patients' pain expectancy on a numeric rating scale was at least 1 point higher than actual pain experienced. Subjects' performance was expressed in reaction time and movement time. Repeated measures analysis of variance was performed to detect main effect for group and condition. Special interest was given to group*condition interaction, since significant interaction would indicate that patients and healthy subjects performed differently throughout the three conditions.", "Patients were slower throughout all conditions compared to healthy subjects. With respect to the provoking condition, patients showed deteriorated performance compared to lying while healthy subjects' performance remained equal between these two conditions. Further analysis of patients' data showed that provocation was successful in 54% of the patients. Especially this group showed deteriorated performance in the provoking condition." ], "LABELS": [ "BACKGROUND", "METHODS", "RESULTS" ], "MESHES": [ "Adaptation, Psychological", "Chronic Disease", "Cognition", "Female", "Hand", "Humans", "Low Back Pain", "Male", "Middle Aged", "Motor Skills", "Movement", "Pain Measurement", "Pilot Projects", "Reaction Time", "Task Performance and Analysis", "Time Factors" ], "YEAR": "2011", "reasoning_required_pred": "yes", "reasoning_free_pred": "yes", "final_decision": "yes", "LONG_ANSWER": "It can be concluded that CLBP patients in general have worse motor task performance compared to healthy subjects and that provoking pain-related cognitions further worsened performance." }, "18619710": { "QUESTION": "Patient comprehension of emergency department care and instructions: are patients aware of when they do not understand?", "CONTEXTS": [ "To be able to adhere to discharge instructions after a visit to the emergency department (ED), patients should understand both the care that they received and their discharge instructions. The objective of this study is to assess, at discharge, patients' comprehension of their ED care and instructions and their awareness of deficiencies in their comprehension.", "We conducted structured interviews of 140 adult English-speaking patients or their primary caregivers after ED discharge in 2 health systems. Participants rated their subjective understanding of 4 domains: (1) diagnosis and cause; (2) ED care; (3) post-ED care, and (4) return instructions. We assessed patient comprehension as the degree of agreement (concordance) between patients' recall of each of these domains and information obtained from chart review. Two authors scored each case independently and discussed discrepancies before providing a final concordance rating (no concordance, minimal concordance, partial concordance, near concordance, complete concordance).", "Seventy-eight percent of patients demonstrated deficient comprehension (less than complete concordance) in at least 1 domain; 51% of patients, in 2 or more domains. Greater than a third of these deficiencies (34%) involved patients' understanding of post-ED care, whereas only 15% were for diagnosis and cause. The majority of patients with comprehension deficits failed to perceive them. Patients perceived difficulty with comprehension only 20% of the time when they demonstrated deficient comprehension." ], "LABELS": [ "STUDY OBJECTIVE", "METHODS", "RESULTS" ], "MESHES": [ "Adult", "Aged", "Aged, 80 and over", "Comprehension", "Emergency Medical Services", "Female", "Humans", "Interviews as Topic", "Male", "Mental Recall", "Michigan", "Middle Aged", "Patient Compliance", "Patient Education as Topic", "Patients" ], "YEAR": "2009", "reasoning_required_pred": "no", "reasoning_free_pred": "no", "final_decision": "no", "LONG_ANSWER": "Many patients do not understand their ED care or their discharge instructions. Moreover, most patients appear to be unaware of their lack of understanding and report inappropriate confidence in their comprehension and recall." }, "18784527": { "QUESTION": "Can mandibular depiction be improved by changing the thickness of double-oblique computed tomography images?", "CONTEXTS": [ "Multislice helical computed tomography (CT), which can provide detailed 2-D and 3-D reconstructed images, is useful in imaging diagnosis for dental implant treatment. Therefore, in this study, it was performed to clarify the mandibular depiction of double-oblique reconstructed images when changing their thickness.", "A total of 38 sites in the mandibular molar region were examined using multislice helical CT. The thicknesses of the double-oblique images using multislice helical CT scans were reconstructed in 4 conditions: 0.3 mm, 0.9 mm, 1.6 mm, and 4.1 mm. In double-oblique images, mandibular depiction was evaluated by 5 oral radiologists using a subjective rating score.", "In the alveolar crest and the whole of the mandibular canal, the highest value was obtained with 0.9 mm-thick images; however, there was no significant difference between 0.3 mm and 0.9 mm-thick images." ], "LABELS": [ "PURPOSE", "MATERIALS AND METHODS", "RESULTS" ], "MESHES": [ "Alveolar Process", "Anatomy, Cross-Sectional", "Female", "Humans", "Jaw, Edentulous", "Male", "Mandible", "Mandibular Nerve", "Middle Aged", "Tomography, Spiral Computed" ], "YEAR": "2008", "reasoning_required_pred": "no", "reasoning_free_pred": "no", "final_decision": "no", "LONG_ANSWER": "We concluded that depiction of the superior wall of the mandibular canal cannot be improved by changing the thickness of images." }, "15919266": { "QUESTION": "Adjuvant radiation of stage III thymoma: is it necessary?", "CONTEXTS": [ "The criteria for administration of adjuvant radiation therapy after thymoma resection remains controversial, and it is unclear whether patients with Masaoka stage III thymoma benefit from adjuvant radiation. The goal of this report was to determine whether or not this group benefits from radiation therapy in disease-specific survival and disease-free survival.", "Case records of the Massachusetts General Hospital were retrospectively reviewed from 1972 to 2004. One hundred and seventy-nine patients underwent resection for thymoma, of which 45 had stage III disease.", "Forty-five stage III patients underwent resection and in 36 it was complete. Thirty-eight stage III patients received radiation therapy. Baseline prognostic factors between radiated and nonradiated groups were similar. The addition of adjuvant radiotherapy did not alter local or distant recurrence rates in patients with stage III thymoma. Disease-specific survival at 10 years in stage III patients who did not receive radiation was 75% (95% confidence interval, 32% to 100%) and in patients who did receive radiation therapy it was 79% (95% confidence interval, 64% to 94%) (p = 0.21). The most common site of relapse was the pleura." ], "LABELS": [ "BACKGROUND", "METHODS", "RESULTS" ], "MESHES": [ "Adolescent", "Adult", "Female", "Humans", "Male", "Middle Aged", "Neoplasm Recurrence, Local", "Neoplasm Staging", "Radiotherapy, Adjuvant", "Retrospective Studies", "Survival Analysis", "Thymoma", "Thymus Neoplasms" ], "YEAR": "2005", "reasoning_required_pred": "no", "reasoning_free_pred": "no", "final_decision": "no", "LONG_ANSWER": "Most patients who have stage III thymoma undergo complete resection. Some patients enjoy prolonged disease-free survival without adjuvant radiation after resection of stage III thymoma. Radiation does not seem to prevent pleural recurrences when given after resection of stage III thymomas. The use of routine adjuvant radiation after a complete resection of stage III thymoma needs to be re-addressed. There may be a role for the use of chemotherapy to reduce pleural recurrences." }, "2503176": { "QUESTION": "Inhibin: a new circulating marker of hydatidiform mole?", "CONTEXTS": [ "To define the concentrations of inhibin in serum and tissue of patients with hydatidiform mole and assess their value as a clinical marker of the condition.", "Prospective study of new patients with hydatidiform mole, comparison of paired observations, and case-control analysis.", "A university hospital, two large public hospitals, and a private women's clinic in Japan.", "Seven consecutive referred patients seen over four months with newly diagnosed complete hydatidiform mole, including one in whom the mole was accompanied by viable twin fetuses (case excluded from statistical analysis because of unique clinical features). All patients followed up for six months after evacuation of molar tissue.", "Correlation of serum inhibin concentrations with trophoblastic disease.", "Serum concentrations of inhibin, human chorionic gonadotrophin, and follicle stimulating hormone were compared before and seven to 10 days after evacuation of the mole. Before evacuation the serum inhibin concentrations (median 8.3 U/ml; 95% confidence interval 2.4 to 34.5) were significantly greater than in 21 normal women at the same stage of pregnancy (2.8 U/ml; 2.1 to 3.6), and inhibin in molar tissue was also present in high concentrations (578 U/ml cytosol; 158 to 1162). Seven to 10 days after evacuation inhibin concentrations in serum samples from the same patients declined significantly to values (0.4 U/ml; 0.1 to 1.4) similar to those seen in the follicular phase of normal menstrual cycles. None of the four patients whose serum inhibin concentrations were 0.4 U/ml or less after evacuation developed persistent trophoblastic disease. Though serum human chorionic gonadotrophin concentrations declined after evacuation (6.6 x 10(3) IU/l; 0.8 x 10(3) to 32.6 x 10(3], they remained far higher than in non-pregnant women. Serum follicle stimulating hormone concentrations remained suppressed." ], "LABELS": [ "OBJECTIVE", "DESIGN", "SETTING", "PATIENTS", "END POINT", "MEASUREMENTS AND MAIN RESULTS" ], "MESHES": [ "Adolescent", "Adult", "Biomarkers, Tumor", "Chorionic Gonadotropin", "Female", "Humans", "Hydatidiform Mole", "Inhibins", "Middle Aged", "Pregnancy", "Radioimmunoassay", "Uterine Neoplasms" ], "YEAR": "1989", "reasoning_required_pred": "yes", "reasoning_free_pred": "yes", "final_decision": "yes", "LONG_ANSWER": "In this small study serum inhibin concentrations higher than those found in the early follicular phase one to two weeks after evacuation of a hydatidiform mole seemed to be specific for persistent trophoblastic disease. Further data are needed to confirm these promising results." }, "22108230": { "QUESTION": "Is the fibronectin-aggrecan complex present in cervical disk disease?", "CONTEXTS": [ "To investigate the presence of inflammatory cytokines and the fibronectin-aggrecan complex (FAC) in persons undergoing surgical treatment for cervical radiculopathy caused by disk herniation.", "Single-center, prospective, consecutive case series.", "A single large academic institution.", "A total of 11 patients with radiculopathic pain and magnetic resonance imaging findings positive for disk herniation elected to undergo single-level cervical diskectomy.", "Lavage was performed by needle injection and aspiration upon entering the disk space for fluoroscopic localization before diskectomy.", "The lavage fluid was assayed for pH and the FAC, as well as for the cytokines interleukin-6 (IL-6), interferon-\u03b3, monocyte chemotactic protein (MCP), and macrophage inhibitory protein-1\u03b2.", "The subjects were 7 women and 4 men with a mean age of 50.6 years (SE 9.7; range, 36-70 years). The mean concentrations (SE; range) in picograms per milliliter were 7.9 (4.4; 0-44) for IL-6, 25.3 (15.5; 0-159) for interferon-\u03b3, 16.1 (11.9; 0-121) for MCP, and 6.1 (2.8; 0-29) for macrophage inhibitory protein-1\u03b2. The optical density of the FAC at 450 nm was 0.151 (0.036; 0.1-0.32), and the pH was 6.68 (0.1; 6.10-7.15). Statistically significant correlations were found between MCP and FAC (P = .036) and between FAC and pH (P = .008)." ], "LABELS": [ "OBJECTIVE", "DESIGN", "SETTING", "PATIENTS", "METHODS OR INTERVENTIONS", "MAIN OUTCOME MEASUREMENTS", "RESULTS" ], "MESHES": [ "Adult", "Aged", "Aggrecans", "Biomarkers", "Cervical Vertebrae", "Female", "Fibronectins", "Follow-Up Studies", "Humans", "Intervertebral Disc Degeneration", "Intervertebral Disc Displacement", "Magnetic Resonance Imaging", "Male", "Middle Aged", "Prospective Studies", "Severity of Illness Index" ], "YEAR": "2011", "reasoning_required_pred": "yes", "reasoning_free_pred": "yes", "final_decision": "yes", "LONG_ANSWER": "Biochemical analysis of injured cervical intervertebral disks reveals the presence of inflammatory markers such as MCP, fragments of structural matrix proteins such as FAC, and a correlation with pH. Further evaluation of the FAC as a potential diagnostic biomarker or therapeutic target is warranted in the cervical spine." }, "20338971": { "QUESTION": "Does peritoneal dialysis affect halitosis in patients with end-stage renal disease?", "CONTEXTS": [ "There are various causes of halitosis, one of which is chronic renal failure. The objective of this study was to investigate halitosis levels in end-stage renal disease (ESRD) patients before and after peritoneal dialysis (PD) therapy.", "42 subjects with ESRD were included in this study. The presence of halitosis was assessed using an organoleptic measurement and compared with blood urea nitrogen (BUN) levels and salivary flow rates. Decayed, missing, and filled teeth (DMFT) index and Community Periodontal Index (CPI) were calculated. All measurements were done before and after patients had received 3 months of PD therapy.", "Mean serum BUN level was found to be lower (46.05 \u00b1 13.30 vs 91.24 \u00b1 31.28 mg/dL), salivary flow rate higher (0.34 \u00b1 0.07 vs 0.26 \u00b1 0.04 mL/minute), and halitosis level lower (2.39 \u00b1 0.60 vs 3.90 \u00b1 0.37) at the end of 3 months of PD therapy than at the beginning of PD therapy. There was no significant difference in CPI or DMFT index before and after PD therapy (p>0.05). There was statistically significant positive correlation between the presence of halitosis and BUN levels (r = 0.702, p = 0.001 before PD; r = 0.45, p = 0.002 after PD) and a negative correlation between the presence of halitosis and salivary flow rates (r = -0.69, p = 0.000 before PD; r = -0.37, p = 0.01 after PD)." ], "LABELS": [ "OBJECTIVE", "METHODS", "RESULTS" ], "MESHES": [ "Adult", "Blood Urea Nitrogen", "Disease Progression", "Female", "Follow-Up Studies", "Halitosis", "Humans", "Kidney Failure, Chronic", "Male", "Middle Aged", "Peritoneal Dialysis", "Risk Factors", "Saliva", "Young Adult" ], "YEAR": null, "reasoning_required_pred": "yes", "reasoning_free_pred": "yes", "final_decision": "yes", "LONG_ANSWER": "High BUN levels and low salivary flow rates were found to be associated with halitosis. PD may play an important role in decreasing the level of halitosis in ESRD patients." }, "17276801": { "QUESTION": "Can elevated troponin I levels predict complicated clinical course and inhospital mortality in patients with acute pulmonary embolism?", "CONTEXTS": [ "The purpose of this study was to evaluate the value of elevated cardiac troponin I (cTnI) for prediction of complicated clinical course and in-hospital mortality in patients with confirmed acute pulmonary embolism (PE).", "This study was a retrospective chart review of patients diagnosed as having PE, in whom cTnI testing was obtained at emergency department (ED) presentation between January 2002 and April 2006. Clinical characteristics; echocardiographic right ventricular dysfunction; inhospital mortality; and adverse clinical events including need for inotropic support, mechanical ventilation, and thrombolysis were compared in patients with elevated cTnI levels vs patients with normal cTnI levels. One hundred sixteen patients with PE were identified, and 77 of them (66%) were included in the study. Thirty-three patients (42%) had elevated cTnI levels. Elevated cTnI levels were associated with inhospital mortality (P = .02), complicated clinical course (P<.001), and right ventricular dysfunction (P<.001). In patients with elevated cTnI levels, inhospital mortality (odds ratio [OR], 3.31; 95% confidence interval [CI], 1.82-9.29), hypotension (OR, 7.37; 95% CI, 2.31-23.28), thrombolysis (OR, 5.71; 95% CI, 1.63-19.92), need for mechanical ventilation (OR, 5.00; 95% CI, 1.42-17.57), and need for inotropic support (OR, 3.02; 95% CI, 1.03-8.85) were more prevalent. The patients with elevated cTnI levels had more serious vital parameters (systolic blood pressure, pulse, and oxygen saturation) at ED presentation." ], "LABELS": [ "OBJECTIVE", "METHODS AND RESULTS" ], "MESHES": [ "Acute Disease", "Adult", "Aged", "Aged, 80 and over", "Emergency Service, Hospital", "Female", "Hospital Mortality", "Humans", "Male", "Middle Aged", "Predictive Value of Tests", "Pulmonary Embolism", "Retrospective Studies", "Risk Assessment", "Treatment Outcome", "Troponin I" ], "YEAR": "2007", "reasoning_required_pred": "yes", "reasoning_free_pred": "yes", "final_decision": "yes", "LONG_ANSWER": "Our results indicate that elevated cTnI levels are associated with higher risk for inhospital mortality and complicated clinical course. Troponin I may play an important role for the risk assessment of patients with PE. The idea that an elevation in cTnI levels is a valuable parameter for the risk stratification of patients with PE needs to be examined in larger prospective studies." }, "20497880": { "QUESTION": "Is transurethral catheterisation the ideal method of bladder drainage?", "CONTEXTS": [ "Bladder catheterisation is a routine part of major abdominal surgery. Transurethral catheterisation is the most common method of bladder drainage but is also notorious for its discomfort and increased risk of urinary tract infection. The present study aimed to establish patient satisfaction with transurethral catheterisation and to assess the incidence of clinically significant urinary tract infections after transurethral catheterisation through survey.", "All patients who underwent major open abdominal surgery between October 2006 and December 2008 and required standard transurethral bladder catheterisation, were asked to participate in the study. Fifty patients were recruited.", "Male patients were more dissatisfied than their female counterparts with transurethral catheterisation (satisfaction score: 4.18/10 vs. 2.75/10; p = 0.05). Male patients had more than double the score for pain at the urinary meatus with the catheter in situ (p =0.012) and during urine catheter removal (p = 0.013). Half the patients in the study also had symptoms of urinary tract infection after catheter removal." ], "LABELS": [ "OBJECTIVE", "METHODS", "RESULTS" ], "MESHES": [ "Adult", "Aged", "Catheters, Indwelling", "Female", "Health Care Surveys", "Humans", "Incidence", "Male", "Middle Aged", "Pain", "Patient Satisfaction", "Urinary Catheterization", "Urinary Tract Infections" ], "YEAR": "2010", "reasoning_required_pred": "no", "reasoning_free_pred": "no", "final_decision": "no", "LONG_ANSWER": "Our study emphasised the discomfort of transurethral urinary catheters, especially in male patients, and the high incidence of urinary tract infections in both sexes. Consideration should be given to the utilisation of alternative methods of bladder drainage, such as suprapubic catheterisation, which can be performed with ease during laparotomy." }, "23495128": { "QUESTION": "The colour of pain: can patients use colour to describe osteoarthritis pain?", "CONTEXTS": [ "The aim of the present study was to explore patients' views on the acceptability and feasibility of using colour to describe osteoarthritis (OA) pain, and whether colour could be used to communicate pain to healthcare professionals.", "Six group interviews were conducted with 17 patients with knee OA. Discussion topics included first impressions about using colour to describe pain, whether participants could associate their pain with colour, how colours related to changes to intensity and different pain qualities, and whether they could envisage using colour to describe pain to healthcare professionals.", "The group interviews indicated that, although the idea of using colour was generally acceptable, it did not suit all participants as a way of describing their pain. The majority of participants chose red to describe high-intensity pain; the reasons given were because red symbolized inflammation, fire, anger and the stop signal in a traffic light system. Colours used to describe the absence of pain were chosen because of their association with positive emotional feelings, such as purity, calmness and happiness. A range of colours was chosen to represent changes in pain intensity. Aching pain was consistently identified as being associated with colours such as grey or black, whereas sharp pain was described using a wider selection of colours. The majority of participants thought that they would be able to use colour to describe their pain to healthcare professionals, although issues around the interpretability and standardization of colour were raised." ], "LABELS": [ "OBJECTIVE", "METHODS", "RESULTS" ], "MESHES": [ "Aged", "Aged, 80 and over", "Color", "Communication", "Feasibility Studies", "Female", "Humans", "Interviews as Topic", "Male", "Middle Aged", "Nurse-Patient Relations", "Osteoarthritis, Knee", "Pain Measurement", "Physician-Patient Relations" ], "YEAR": "2014", "reasoning_required_pred": "yes", "reasoning_free_pred": "yes", "final_decision": "yes", "LONG_ANSWER": "For some patients, using colour to describe their pain experience may be a useful tool to improve doctor-patient communication." }, "26701174": { "QUESTION": "Inpatient Pediatric Tonsillectomy: Does Hospital Type Affect Cost and Outcomes of Care?", "CONTEXTS": [ "To ascertain whether hospital type is associated with differences in total cost and outcomes for inpatient tonsillectomy.", "Cross-sectional analysis of the 2006, 2009, and 2012 Kids' Inpatient Database (KID).", "Children \u226418 years of age undergoing tonsillectomy with/without adenoidectomy were included. Risk-adjusted generalized linear models assessed for differences in hospital cost and length of stay (LOS) among children managed by (1) non-children's teaching hospitals (NCTHs), (2) children's teaching hospitals (CTHs), and (3) nonteaching hospitals (NTHs). Risk-adjusted logistic regression compared the odds of major perioperative complications (hemorrhage, respiratory failure, death). Models accounted for clustering of patients within hospitals, were weighted to provide national estimates, and controlled for comorbidities.", "The 25,685 tonsillectomies recorded in the KID yielded a national estimate of 40,591 inpatient tonsillectomies performed in 2006, 2009, and 2012. The CTHs had significantly higher risk-adjusted total cost and LOS per tonsillectomy compared with NCTHs and NTHs ($9423.34/2.8 days, $6250.78/2.11 days, and $5905.10/2.08 days, respectively; P<.001). The CTHs had higher odds of complications compared with NCTHs (odds ratio [OR], 1.48; 95% CI, 1.15-1.91; P = .002) but not when compared with NTHs (OR, 1.19; 95% CI, 0.89-1.59; P = .23). The CTHs were significantly more likely to care for patients with comorbidities (P<.001)." ], "LABELS": [ "OBJECTIVE", "STUDY DESIGN", "SUBJECTS AND METHODS", "RESULTS" ], "MESHES": [ "Adenoidectomy", "Adolescent", "Child", "Child, Preschool", "Cross-Sectional Studies", "Female", "Hospital Charges", "Hospital Costs", "Humans", "Infant", "Inpatients", "Length of Stay", "Male", "Outcome and Process Assessment (Health Care)", "Postoperative Complications", "Tonsillectomy" ], "YEAR": "2016", "reasoning_required_pred": "yes", "reasoning_free_pred": "yes", "final_decision": "yes", "LONG_ANSWER": "Significant differences in costs, outcomes, and patient factors exist for inpatient tonsillectomy based on hospital type. Although reasons for these differences are not discernable using isolated claims data, findings provide a foundation to further evaluate patient, institutional, and system-level factors that may reduce cost of care and improve value for inpatient tonsillectomy." }, "24695920": { "QUESTION": "Does anterior laxity of the uninjured knee influence clinical outcomes of ACL reconstruction?", "CONTEXTS": [ "The purpose of this study was to evaluate the association between the postoperative outcomes of anterior cruciate ligament (ACL) reconstruction and the anterior laxity of the uninjured knee.", "We retrospectively reviewed 163 patients who had undergone unilateral ACL reconstruction from January 2002 to August 2009. Patients were divided into three groups according to the anterior laxity of the contralateral, normal knee in 30\u00b0 of knee flexion as measured with a KT2000 arthrometer exerting a force of 134 N:<5 mm for Group 1, 5 to 7.5 mm for Group 2, and>7.5 mm for Group 3. Anterior laxity of the uninjured knee was assessed preoperatively, and anterior laxity of the reconstructed knee was assessed at twenty-four months postoperatively. Anterior stability of the knee was also assessed with use of the Lachman and pivot-shift tests. Functional outcomes were assessed with the Lysholm score and the International Knee Documentation Committee (IKDC) score.", "The three groups differed significantly with respect to the postoperative side-to-side difference in anterior laxity (p = 0.015), Lysholm score (p<0.001), and IKDC subjective score (p<0.001). The mean side-to-side difference in anterior laxity of the reconstructed knee was 2.1 \u00b1 1.3 mm in Group 1, 2.2 \u00b1 1.3 mm in Group 2, and 2.9 \u00b1 1.4 mm in Group 3. The postoperative Lysholm score was 91.8 \u00b1 4.5 in Group 1, 90.3 \u00b1 5.5 in Group 2, and 85.4 \u00b1 6.6 in Group 3. The postoperative IKDC subjective score was 89.3 \u00b1 6.4 in Group 1, 87.9 \u00b1 6.0 in Group 2, and 82.6 \u00b1 8.2 in Group 3. Post hoc testing showed that Group 3 had significantly greater anterior laxity (p \u2264 0.039) and lower functional scores (p \u2264 0.001) compared with Groups 1 and 2." ], "LABELS": [ "BACKGROUND", "METHODS", "RESULTS" ], "MESHES": [ "Adolescent", "Adult", "Aged", "Anterior Cruciate Ligament Reconstruction", "Arthrometry, Articular", "Female", "Health Status Indicators", "Humans", "Joint Instability", "Knee Injuries", "Knee Joint", "Male", "Middle Aged", "Observer Variation", "Postoperative Period", "Range of Motion, Articular", "Recovery of Function", "Retrospective Studies", "Treatment Outcome", "Young Adult" ], "YEAR": "2014", "reasoning_required_pred": "yes", "reasoning_free_pred": "yes", "final_decision": "yes", "LONG_ANSWER": "Greater anterior laxity of the uninjured knee was associated with poorer stability and functional outcomes after ACL reconstruction. Excessive anterior laxity of the uninjured knee thus appears to represent a risk factor for inferior outcomes." }, "15488260": { "QUESTION": "Is the type of remission after a major depressive episode an important risk factor to relapses in a 4-year follow up?", "CONTEXTS": [ "Rates of relapse and predictive relapse factors were studied over more than 4 years in a sample of Spanish outpatients with DSM-III-R criteria for unipolar major depressive episode.", "A final sample of 139 outpatient was followed monthly in a naturalistic study. The Structured Clinical Interview for DSM-III-R was used. Phases of evolution were recorded using the Hamilton Depression Rating Scale, applying the Frank criteria. Survival analysis, Kaplan-Meier product limit and proportional hazards models were used.", "A higher rate of relapses was observed in the partial remission group (91.4%) compared to the complete remission one (51.3%). The four factors with predictive relapse value were: \"partial remission versus complete remission\", \"the intensity of clinical symptoms\", \"the age\" and \"the number of previous depressive episodes\". The existence of partial remission was the most powerful predictive factor.", "The decreasing sample size during the follow-up and the difficulty in warranting the treatment compliance." ], "LABELS": [ "BACKGROUND", "METHODS", "RESULTS", "LIMITATIONS" ], "MESHES": [ "Adult", "Aged", "Antidepressive Agents", "Depressive Disorder, Major", "Diagnostic and Statistical Manual of Mental Disorders", "Drug Therapy, Combination", "Female", "Follow-Up Studies", "Humans", "Male", "Middle Aged", "Personality Inventory", "Prospective Studies", "Recurrence", "Risk Factors", "Serotonin Uptake Inhibitors", "Spain", "Survival Analysis" ], "YEAR": "2004", "reasoning_required_pred": "yes", "reasoning_free_pred": "yes", "final_decision": "yes", "LONG_ANSWER": "At medium term, relapse rates for a major depressive episode are high. Partial remission after a depressive episode seems to be an important predictive factor for relapses in a 4-year follow-up." }, "15466981": { "QUESTION": "Prostate-specific antigen and free prostate-specific antigen in the early detection of prostate cancer: do combination tests improve detection?", "CONTEXTS": [ "The combined use of free and total prostate-specific antigen (PSA) in early detection of prostate cancer has been controversial. This article systematically evaluates the discriminating capacity of a large number of combination tests.", "Free and total PSA were analyzed in stored serum samples taken prior to diagnosis in 429 cases and 1,640 controls from the Physicians' Health Study. We used a classification algorithm called logic regression to search for clinically useful tests combining total and percent free PSA and receiver operating characteristic analysis and compared these tests with those based on total and complexed PSA. Data were divided into training and test subsets. For robustness, we considered 35 test-train splits of the original data and computed receiver operating characteristic curves for each test data set.", "The average area under the receiver operating characteristic curve across test data sets was 0.74 for total PSA and 0.76 for the combination tests. Combination tests with higher sensitivity and specificity than PSA>4.0 ng/mL were identified 29 out of 35 times. All these tests extended the PSA reflex range to below 4.0 ng/mL. Receiver operating characteristic curve analysis indicated that the overall diagnostic performance as expressed by the area under the curve did not differ significantly for the different tests." ], "LABELS": [ "BACKGROUND", "METHODS", "RESULTS" ], "MESHES": [ "Aged", "Case-Control Studies", "Diagnosis, Differential", "Humans", "Male", "Mass Screening", "Middle Aged", "Prostate-Specific Antigen", "Prostatic Neoplasms", "Reference Values", "Sensitivity and Specificity" ], "YEAR": "2004", "reasoning_required_pred": "yes", "reasoning_free_pred": "yes", "final_decision": "yes", "LONG_ANSWER": "Tests combining total and percent free PSA show modest overall improvements over total PSA. However, utilization of percent free PSA below a PSA threshold of 4 ng/mL could translate into a practically important reduction in unnecessary biopsies without sacrificing cancers detected." }, "18603989": { "QUESTION": "Can homemade alcohol (Raksi) be useful for preserving dead bodies?", "CONTEXTS": [ "Embalming is the through disinfection and art of preserving bodies after death using chemical substances. It keeps a body life like in appearance during the time it lies in a state prior to funeral.", "This study was undertaken to investigate the effectiveness of Raksi in sacrificed rats in arresting postmortem changes and establishing scientific fact whether Raksi can be an alternative to standard embalming constituent if it is not available.", "50 albino rats were systematically randomized into control and experiment groups. Raksi and distilled water were injected for embalming purpose intraventricularly in experiment and control groups of rats respectively and kept for 48 to 96 hours for observation for postmortem changes.", "Observations made at 48 and 72 hours of embalming revealed that Raksi can arrest postmortem changes in the rats up to 72 hours (3rd day) successfully in the experimental group whereas moderate to severe postmortem changes were seen in the control group. The experimental group showed mild degree of putrefactive changes, liberation of gases and liquefaction of tissues only at 96 hours (4th day) of embalming.", "The Raksi used in this experiment contained 34% of alcohol, which was determined by an alcohol hydrometer. Experiment clearly demonstrated from its result that raksi can be utilised temporarily for embalming since it contains alcohol and has preservative, bactericidal and disinfectant properties." ], "LABELS": [ "INTRODUCTION", "OBJECTIVE", "MATERIAL AND METHODS", "RESULT", "DISCUSSION" ], "MESHES": [ "Alcohols", "Animals", "Embalming", "Nepal", "Postmortem Changes", "Random Allocation", "Rats", "Rats, Wistar" ], "YEAR": null, "reasoning_required_pred": "yes", "reasoning_free_pred": "yes", "final_decision": "yes", "LONG_ANSWER": "It is concluded from the study that this knowledge if applied to dead human subjects, may preserve dead bodies temporarily allowing delayed funeral." }, "21789019": { "QUESTION": "Do elderly cancer patients have different care needs compared with younger ones?", "CONTEXTS": [ "The increasingly older population confronts oncologists with an imposing challenge: older cancer patients have specific healthcare needs both independent of and associated with the diagnosis of cancer. The aim of the present study is to examine whether elderly versus younger cancer patients have different needs with respect to attendance, treatment and information.", "This is an observational and cross-sectional study. Cancer patients aged 35 to 82 years were consecutively interviewed. The group was divided into two subgroups aged \u226465 and \u226566 years old. The Needs Evaluation Questionnaire (NEQ) was used to assess patients' needs and demographic variables were collected. Data analysis was carried out by means of cross-tabulation analyses and the chi-square test.", "The requests most frequently expressed by the older group concerned financial-insurance information (73.9%), the need to talk to people with the same illness (71.7%), the need to receive more comprehensible information from doctors and nurses (71.7%), and the need for a better dialogue with clinicians (69.6%). Few significant differences between the two age subgroups were found, with the exception of issues such as the need for intimacy and support." ], "LABELS": [ "AIMS AND BACKGROUND", "METHODS AND STUDY DESIGN", "RESULTS" ], "MESHES": [ "Adult", "Age Factors", "Aged", "Aged, 80 and over", "Chi-Square Distribution", "Cross-Sectional Studies", "Female", "Health Services Needs and Demand", "Humans", "Insurance Coverage", "Insurance, Health", "Male", "Middle Aged", "Neoplasms", "Patient Satisfaction", "Professional-Patient Relations", "Surveys and Questionnaires" ], "YEAR": null, "reasoning_required_pred": "yes", "reasoning_free_pred": "no", "final_decision": "no", "LONG_ANSWER": "Elderly patients have informational and relational needs similar to those of younger patients. In fact, most of the whole sample flagged up the need for higher levels of satisfaction especially concerning a better dialogue with health staff, along with a new factor: the need to talk to people with the same illness." }, "14692023": { "QUESTION": "Is breast cancer survival improving?", "CONTEXTS": [ "Despite advances in therapies for breast cancer, improvement in survival for patients with recurrent or metastatic breast cancer has been difficult to establish. The objective of the current study was to determine whether the survival of women with recurrent breast cancer has improved from 1974 to 2000.", "The authors analyzed the survival experience of 834 women who developed recurrent breast cancer between November 1974 and December 2000. All patients had been treated previously with adjuvant anthracycline-based protocols. Patients were divided into five consecutive groups based on year of breast cancer recurrence, and survival was compared across the five groups. Because some prognostic variables were divided unevenly divided among the cohorts, a multivariate model was created to determine the association of year of recurrence and survival after accounting for other prognostic factors.", "In the unadjusted analysis, there was a statistically significant improvement in survival across the five groups, and the more recent cohorts had longer survival (P<0.001). Other variables that predicted longer survival after breast cancer recurrence included smaller initial tumor size, lower stage of disease, fewer lymph nodes involved, longer disease-free interval, estrogen receptor-positive tumors, and nonvisceral dominant site of disease recurrence. In the multivariate analysis, which adjusted for these prognostic factors, year of recurrence was associated with a trend toward improved survival, with a 1% reduction in risk for each increasing year." ], "LABELS": [ "BACKGROUND", "METHODS", "RESULTS" ], "MESHES": [ "Adult", "Aged", "Breast Neoplasms", "Cohort Studies", "Female", "Humans", "Middle Aged", "Multivariate Analysis", "Neoplasm Recurrence, Local", "Prognosis", "Retrospective Studies", "Survival Analysis" ], "YEAR": "2004", "reasoning_required_pred": "yes", "reasoning_free_pred": "yes", "final_decision": "yes", "LONG_ANSWER": "For these cohorts of patients, the authors present data suggesting that the prognosis for patients with recurrent breast cancer improved between 1974 and 2000." }, "25150098": { "QUESTION": "Can common carotid intima media thickness serve as an indicator of both cardiovascular phenotype and risk among black Africans?", "CONTEXTS": [ "It is not known whether common carotid intima media thickness (CIMT) can serve as a surrogate marker of cardiovascular risk among black Africans. Therefore, we examined whether CIMT differed significantly among individuals with distinct cardiovascular phenotype and correlated significantly with traditional cardiovascular risk factors in a black African population.", "CIMT was measured in 456 subjects with three distinct cardiovascular phenotypes - 175 consecutive Nigerian African stroke patients, 161 hypertensive patients without stroke and 120 normotensive non-smoking adults. For each pair of cardiovascular phenotypes, c-statistics were obtained for CIMT and traditional vascular risk factors (including age, gender, weight, waist circumference, smoking, alcohol, systolic and diastolic blood pressures, fasting plasma glucose, fasting total cholesterol). Pearson's correlation coefficients were calculated to quantify bivariate relationships.", "Bilaterally, CIMT was significantly different among the three cardiovascular phenotypes (right: p\u2009<\u20090.001, F\u2009=\u200933.8; left: p\u2009<\u20090.001, F\u2009=\u200948.6). CIMT had a higher c-statistic for differentiating stroke versus normotension (c\u2009=\u20090.78 right; 0.82 left, p\u2009<\u20090.001) and hypertension versus normotension (c\u2009=\u20090.65 right; 0.71 left, p\u2009<\u20090.001) than several traditional vascular risk factors. Bilaterally, combining all subjects, CIMT was the only factor that correlated significantly (right: 0.12\u2009\u2264\u2009r\u2009\u2264\u20090.41, 0.018\u2009\u2264\u2009p\u2009<\u20090.0001; left: 0.18\u2009\u2264\u2009r\u2009\u2264\u20090.41, 0.005\u2009\u2264\u2009p\u2009<\u20090.0001) to all the traditional cardiovascular risk factors assessed." ], "LABELS": [ "BACKGROUND", "METHODS", "FINDINGS" ], "MESHES": [ "Adult", "African Continental Ancestry Group", "Aged", "Carotid Artery Diseases", "Carotid Artery, Common", "Carotid Intima-Media Thickness", "Case-Control Studies", "Cross-Sectional Studies", "Female", "Humans", "Hypertension", "Male", "Middle Aged", "Nigeria", "Phenotype", "Predictive Value of Tests", "Risk Assessment", "Risk Factors", "Stroke" ], "YEAR": "2015", "reasoning_required_pred": "yes", "reasoning_free_pred": "yes", "final_decision": "yes", "LONG_ANSWER": "Our findings support CIMT as a significant indicator of both cardiovascular risk and phenotype among adult black Africans. However, specific thresholds need to be defined based on prospective studies." }, "20337874": { "QUESTION": "Delayed diagnosis of anorectal malformations: are current guidelines sufficient?", "CONTEXTS": [ "Infants referred to our institution with a final diagnosis of ARM were retrospectively reviewed between 2001 and 2009. The first cohort consisted of patients that were referred between November 2001 and November 2006 with the diagnosis of an ARM that had been delayed for more than 48 h. The second cohort was those referred between December 2006 and May 2009 with whom the diagnosis of ARM had not been made within 24 h of birth.", "Nineteen infants were referred with delayed diagnosis of an ARM over the 7.5 years of the study. Of 44 patients referred to our institution between December 2006 and May 2009, diagnosis of an ARM was delayed more than 24 h in 14 (32%). There was no difference in gender, birth weight, prematurity, type of malformation or presence of associated anomalies between those with timely and delayed diagnosis of their ARM. A significantly greater proportion of those with a delayed diagnosis presented with obstructive symptoms (86% vs. 27%, P<0.001), including abdominal distension (57%) and delayed passage of meconium or stool (29%). Despite undergoing neonatal examination, the diagnosis of ARM was missed in 12 patients overall." ], "LABELS": [ "METHODS", "RESULTS" ], "MESHES": [ "Cohort Studies", "Delayed Diagnosis", "Digestive System Abnormalities", "Female", "Guidelines as Topic", "Humans", "Infant, Newborn", "Male", "New South Wales", "Rectal Fistula", "Rectum", "Retrospective Studies" ], "YEAR": "2010", "reasoning_required_pred": "no", "reasoning_free_pred": "no", "final_decision": "no", "LONG_ANSWER": "Delayed diagnosis of an ARM appears to be common, occurring in approximately 32% of patients referred to our institution over the last 2.5 years. Current guidelines appear insufficient to ensure prompt diagnosis of ARM." }, "19198736": { "QUESTION": "Is being small for gestational age a risk factor for retinopathy of prematurity?", "CONTEXTS": [ "To analyze prevalence and risk factors for retinopathy of prematurity (ROP) among preterm infants born small for gestational age (SGA) and appropriate for gestational age (AGA).", "A prospective cohort study included preterm infants with birth weight (BW)70 years; however this was not statistically significant (ANOVA, p=0.06). In the same age groups, SHBG significantly increased (31.0+/-15.0 to 47.5+/-15.0 nmol/L, p<0.001) whereas free and available testosterone significantly decreased (10.6+/-3.2 to 6.4+/-3.6 ng/dL and 266.6+/-81.2 to 152.2+/-97.6 ng/dL, respectively, p<0.0001). Overall (n=96), available testosterone confirmed PADAM diagnosis in 27 cases (28.1%). The ADAM tool rendered a 83.3% sensitivity and 19.7% specificity in the detection of PADAM. Item 1 (decreased sexual desire) was a better predictor of hypogonadism than the complete questionnaire (63.3% sensitivity and 66.7% specificity)." ], "LABELS": [ "BACKGROUND", "OBJECTIVE", "METHODS", "RESULTS" ], "MESHES": [ "Adult", "Age Factors", "Aged", "Aged, 80 and over", "Androgens", "Andropause", "Cross-Sectional Studies", "Humans", "Male", "Middle Aged", "Serum Albumin", "Sex Hormone-Binding Globulin", "Statistics, Nonparametric", "Surveys and Questionnaires", "Testosterone" ], "YEAR": "2009", "reasoning_required_pred": "no", "reasoning_free_pred": "no", "final_decision": "no", "LONG_ANSWER": "In this series, in accordance to available testosterone, the prevalence of PADAM was determined to be high, in which the ADAM questionnaire rendered a low diagnostic efficiency. PADAM diagnosis could be clinically suspected when symptoms of sexual dysfunction are present." }, "25699562": { "QUESTION": "Does the Transmissible Liability Index (TLI) assessed in late childhood predict suicidal symptoms at young adulthood?", "CONTEXTS": [ "Our previous work demonstrated that the Transmissible Liability Index (TLI), an instrument designed as an index of liability for substance use disorder (SUD), is associated with risk of substance use disorder. This longitudinal study assessed whether TLI measured in 10-12-year-olds (late childhood) predicts suicidal behavior from age 12-14 (preadolescence) to age 25 (young adulthood). We hypothesized that TLI would predict number and severity of suicide attempts.", "Subjects were sons of men who had lifetime history of SUD (n\u2009=\u2009250), called the High Average Risk (HAR) group, and sons of men with no lifetime history of a SUD (n\u2009=\u2009250), called the Low Average Risk (LAR) group. The TLI was delineated at baseline (age 10-12), and age-specific versions were administered at 12-14, 16, 19, 22, and 25 years of age.", "TLI was significantly associated with number and severity of lifetime suicide attempts." ], "LABELS": [ "OBJECTIVE", "METHODS", "RESULTS" ], "MESHES": [ "Adolescent", "Adult", "Child", "Humans", "Longitudinal Studies", "Male", "Risk Factors", "Suicidal Ideation", "Suicide, Attempted", "Young Adult" ], "YEAR": "2015", "reasoning_required_pred": "yes", "reasoning_free_pred": "yes", "final_decision": "yes", "LONG_ANSWER": "These findings confirm the hypothesis that TLI assessed at late childhood is a predictor of frequency and severity of suicidal behavior from preadolescence to young adulthood." }, "17682349": { "QUESTION": "Are there gender differences in the reasons why African Americans delay in seeking medical help for symptoms of an acute myocardial infarction?", "CONTEXTS": [ "To identify gender differences in delay time and the reasons why African Americans delay in seeking medical care for symptoms of acute myocardial infarction (AMI).", "Cross-sectional.", "Five hospitals in the San Francisco and East Bay areas.", "Sixty-one African American men and women diagnosed with an AMI.", "Prehospital delay time.", "Median delay time was longer for women compared to men (4.4 hours vs 3.5 hours), although the difference was not significant. Single women delayed longer than single men (P = .03), and women who were alone when symptoms began delayed longer than women with someone (P = .03). Women who received advice to seek help or call 911 upon symptom onset had shorter delays compared to women who were not advised to call 911 (P = .01). Men at home delayed longer than men who experienced their symptoms outside the home (P = .01). Men with emergency room insurance delayed longer than men without emergency room insurance (P = .03), and men who took an ambulance to the hospital had shorter delay times than men who took other means of transportation (P = .04)." ], "LABELS": [ "OBJECTIVES", "DESIGN", "SETTING", "PATIENTS", "MAIN OUTCOME MEASURES", "RESULTS" ], "MESHES": [ "Acute Disease", "Adult", "African Americans", "Aged", "Aged, 80 and over", "Female", "Health Services Needs and Demand", "Humans", "Male", "Middle Aged", "Myocardial Infarction", "Patient Acceptance of Health Care", "San Francisco", "Surveys and Questionnaires", "Time Factors" ], "YEAR": "2007", "reasoning_required_pred": "yes", "reasoning_free_pred": "yes", "final_decision": "yes", "LONG_ANSWER": "Women compared to men often delay seeking treatment for an AMI, which further increases their risks. Our findings suggest specific characteristics that can serve as a profile to those African Americans most likely to delay seeking treatment for AMI." }, "18251357": { "QUESTION": "Does histologic chorioamnionitis correspond to clinical chorioamnionitis?", "CONTEXTS": [ "To evaluate the degree to which histologic chorioamnionitis, a frequent finding in placentas submitted for histopathologic evaluation, correlates with clinical indicators of infection in the mother.", "A retrospective review was performed on 52 cases with a histologic diagnosis of acute chorioamnionitis from 2,051 deliveries at University Hospital, Newark, from January 2003 to July 2003. Third-trimester placentas without histologic chorioamnionitis (n = 52) served as controls. Cases and controls were selected sequentially. Maternal medical records were reviewed for indicators of maternal infection.", "Histologic chorioamnionitis was significantly associated with the usage of antibiotics (p = 0.0095) and a higher mean white blood cell count (p = 0.018). The presence of 1 or more clinical indicators was significantly associated with the presence of histologic chorioamnionitis (p = 0.019)." ], "LABELS": [ "OBJECTIVE", "STUDY DESIGN", "RESULTS" ], "MESHES": [ "Adult", "Anti-Bacterial Agents", "Case-Control Studies", "Chorioamnionitis", "Diagnosis, Differential", "Female", "Gestational Age", "Humans", "Infant, Newborn", "Lymphocyte Count", "Placenta", "Pregnancy", "Pregnancy Complications, Infectious", "Pregnancy Trimester, Third", "Puerperal Infection", "Retrospective Studies" ], "YEAR": "2008", "reasoning_required_pred": "yes", "reasoning_free_pred": "yes", "final_decision": "yes", "LONG_ANSWER": "Histologic chorioamnionitis is a reliable indicator of infection whether or not it is clinically apparent." }, "20971618": { "QUESTION": "Are lifetime prevalence of impetigo, molluscum and herpes infection really increased in children having atopic dermatitis?", "CONTEXTS": [ "Cutaneous infections such as impetigo contagiosum (IC), molluscum contagiosum (MC) and herpes virus infection (HI) appear to be associated with atopic dermatitis (AD), but there are no reports of concrete epidemiological evidence.", "We evaluated the association of childhood AD with these infections by conducting a population-based cross-sectional study.", "Enrolled in this study were 1117 children aged 0-6 years old attending nursery schools in Ishigaki City, Okinawa Prefecture, Japan. Physical examination was performed by dermatologists, and a questionnaire was completed on each child's history of allergic diseases including AD, asthma, allergic rhinitis and egg allergy, and that of skin infections including IC, MC and HI, as well as familial history of AD.", "In 913 children (AD; 132), a history of IC, MC or HI was observed in 45.1%, 19.7%, and 2.5%, respectively. Multiple logistic regression analysis revealed that the odds of having a history of IC were 1.8 times higher in AD children than in non-AD children. Meanwhile, a history of MC was significantly correlated to the male gender, but not to a personal history of AD. As for HI, we found no correlated factors in this study." ], "LABELS": [ "BACKGROUND", "OBJECTIVE", "METHODS", "RESULTS" ], "MESHES": [ "Child", "Child, Preschool", "Cross-Sectional Studies", "Dermatitis, Atopic", "Female", "Herpesviridae Infections", "Humans", "Impetigo", "Infant", "Japan", "Male", "Molluscum Contagiosum", "Prevalence", "Risk Factors" ], "YEAR": "2010", "reasoning_required_pred": "maybe", "reasoning_free_pred": "maybe", "final_decision": "maybe", "LONG_ANSWER": "The lifetime prevalence of IC was indeed higher in young children with a history of AD." }, "19648304": { "QUESTION": "Does sonographic needle guidance affect the clinical outcome of intraarticular injections?", "CONTEXTS": [ "This randomized controlled study addressed whether sonographic needle guidance affected clinical outcomes of intraarticular (IA) joint injections.", "In total, 148 painful joints were randomized to IA triamcinolone acetonide injection by conventional palpation-guided anatomic injection or sonographic image-guided injection enhanced with a one-handed control syringe (the reciprocating device). A one-needle, 2-syringe technique was used, where the first syringe was used to introduce the needle, aspirate any effusion, and anesthetize and dilate the IA space with lidocaine. After IA placement and synovial space dilation were confirmed, a syringe exchange was performed, and corticosteroid was injected with the second syringe through the indwelling IA needle. Baseline pain, procedural pain, pain at outcome (2 weeks), and changes in pain scores were measured with a 0-10 cm visual analog pain scale (VAS).", "Relative to conventional palpation-guided methods, sonographic guidance resulted in 43.0% reduction in procedural pain (p<0.001), 58.5% reduction in absolute pain scores at the 2 week outcome (p<0.001), 75% reduction in significant pain (VAS pain score>or = 5 cm; p<0.001), 25.6% increase in the responder rate (reduction in VAS score>or = 50% from baseline; p<0.01), and 62.0% reduction in the nonresponder rate (reduction in VAS score<50% from baseline; p<0.01). Sonography also increased detection of effusion by 200% and volume of aspirated fluid by 337%." ], "LABELS": [ "OBJECTIVE", "METHODS", "RESULTS" ], "MESHES": [ "Adrenal Cortex Hormones", "Adult", "Aged", "Arthritis, Rheumatoid", "Female", "Humans", "Injections, Intra-Articular", "Joints", "Male", "Middle Aged", "Osteoarthritis", "Outcome Assessment (Health Care)", "Pain Measurement", "Treatment Outcome", "Triamcinolone Acetonide", "Ultrasonography" ], "YEAR": "2009", "reasoning_required_pred": "yes", "reasoning_free_pred": "yes", "final_decision": "yes", "LONG_ANSWER": "Sonographic needle guidance significantly improves the performance and outcomes of outpatient IA injections in a clinically significant manner." }, "23222920": { "QUESTION": "Pancreas retransplantation: a second chance for diabetic patients?", "CONTEXTS": [ "If pancreas transplantation is a validated alternative for type 1 diabetic patients with end-stage renal disease, the management of patients who have lost their primary graft is poorly defined. This study aims at evaluating pancreas retransplantation outcome.", "Between 1976 and 2008, 569 pancreas transplantations were performed in Lyon and Geneva, including 37 second transplantations. Second graft survival was compared with primary graft survival of the same patients and the whole population. Predictive factors of second graft survival were sought. Patient survival and impact on kidney graft function and survival were evaluated.", "Second pancreas survival of the 17 patients transplanted from 1995 was close to primary graft survival of the whole population (71% vs. 79% at 1 year and 59% vs. 69% at 5 years; P=0.5075) and significantly better than their first pancreas survival (71% vs. 29% at 1 year and 59% vs. 7% at 5 years; P=0.0008) regardless of the cause of first pancreas loss. The same results were observed with all 37 retransplantations. Survival of second simultaneous pancreas and kidney transplantations was better than survival of second pancreas after kidney. Patient survival was excellent (89% at 5 years). Pancreas retransplantation had no impact on kidney graft function and survival (100% at 5 years)." ], "LABELS": [ "BACKGROUND", "METHODS", "RESULTS" ], "MESHES": [ "Adult", "Diabetes Mellitus, Type 1", "Female", "France", "Graft Rejection", "Graft Survival", "Humans", "Kaplan-Meier Estimate", "Kidney Failure, Chronic", "Kidney Transplantation", "Male", "Middle Aged", "Pancreas Transplantation", "Postoperative Complications", "Reoperation", "Retrospective Studies", "Switzerland", "Time Factors", "Treatment Outcome" ], "YEAR": "2013", "reasoning_required_pred": "yes", "reasoning_free_pred": "yes", "final_decision": "yes", "LONG_ANSWER": "Pancreas retransplantation is a safe procedure with acceptable graft survival that should be proposed to diabetic patients who have lost their primary graft." }, "21864397": { "QUESTION": "Factors determining the survival of nasopharyngeal carcinoma with lung metastasis alone: does combined modality treatment benefit?", "CONTEXTS": [ "Nasopharyngeal carcinoma (NPC) with lung metastasis alone has been reported as a relatively favorable prognostic group, and combined modality treatment might be indicated for selected cases. However, the prognostic factors determining survival of this group and the indication of combined therapy have not been thoroughly studied.", "We retrospectively reviewed 246 patients of NPC with lung metastasis(es) alone presented at diagnosis or as the first failure after primary treatment from 1993 to 2008 in an academic tertiary hospital. Univariate and multivariate survival analyses of post-metastasis survival (PMS) and overall survival (OS) were carried out to determine the prognostic factors.", "The 3-year, 5-year, and 10-year of PMS and OS for the whole cohort were 34.3%, 17.0%, 8.6% and 67.8%, 45.4%, 18.5%, respectively. The median PMS (45.6 months vs. 23.7 months) and OS (73.7 months vs. 46.2 months) of patients treated with combined therapy was significantly longer than that of those treated with chemotherapy alone (P<0.001). Age, disease-free interval (DFI) and treatment modality were evaluated as independent prognostic factors of OS, while only age and treatment modality retain their independent significance in PMS analysis. In stratified survival analysis, compared to chemotherapy alone, combined therapy could benefit the patients with DFI>1 year, but not those with DFI \u2264 1 year." ], "LABELS": [ "BACKGROUND", "METHODS", "RESULTS" ], "MESHES": [ "Adult", "Carcinoma", "Chemotherapy, Adjuvant", "Disease-Free Survival", "Female", "Humans", "Lung Neoplasms", "Male", "Middle Aged", "Nasopharyngeal Neoplasms", "Neoadjuvant Therapy", "Neoplasm Recurrence, Local", "Prognosis", "Retrospective Studies", "Survival Rate" ], "YEAR": "2011", "reasoning_required_pred": "yes", "reasoning_free_pred": "maybe", "final_decision": "yes", "LONG_ANSWER": "Age \u2264 45 years, DFI>1 year, and the combined therapy were good prognostic factors for NPC patients with lung metastasis(es) alone. The combination of local therapy and the basic chemotherapy should be considered for these patients with DFI>1 year." }, "26601554": { "QUESTION": "Do viral infections have a role in benign paroxysmal positional vertigo?", "CONTEXTS": [ "To investigate the role of viral infection in benign paroxysmal positional vertigo (BPPV).", "In this retrospective study, 483 patients with BPPV were included in the study group. The control group consisted of 461 healthy subjects. In both groups, serologic analysis of viral agents (HSV1, HSV2, Herpes zoster, EBV, CMV, adenovirus, influenza, and parainfluenza virus) was performed.", "With the exception of influenza and parainfluenza, all viral serology values were higher in the BBPV group than the control group. We also observed seasonal variation. The BPPV group exhibited elevated values for HSV1 and adenovirus in March and May, for Herpes zoster, adenovirus, and influenza in April, for HSV1 in June, and for HSV1 and CMV in September, compared to the control group. In October, the BPPV group showed increased values for all of the viruses studied, compared to the control group." ], "LABELS": [ "OBJECTIVES", "METHODS", "RESULTS" ], "MESHES": [ "Adolescent", "Adult", "Aged", "Antibodies, Viral", "Benign Paroxysmal Positional Vertigo", "Child", "Female", "Follow-Up Studies", "Humans", "Incidence", "Male", "Middle Aged", "Retrospective Studies", "Turkey", "Virus Diseases", "Viruses", "Young Adult" ], "YEAR": "2015", "reasoning_required_pred": "yes", "reasoning_free_pred": "yes", "final_decision": "yes", "LONG_ANSWER": "BPPV is associated with positive viral serology, particularly during certain months of the year, mainly in spring and autumn. Viral infection might promote BPPV attacks due to the development of vestibulopathy or induce secondary BPPV via viral infection-related neurolabyrinthitis." }, "19322056": { "QUESTION": "Does the enteral feeding advancement affect short-term outcomes in very low birth weight infants?", "CONTEXTS": [ "Controversy exists regarding the optimal enteral feeding regimen of very low birth weight infants (VLBW). Rapid advancement of enteral feeding has been associated with an increased rate of necrotizing enterocolitis. In contrast, delaying enteral feeding may have unfavorable effects on nutrition, growth, and neurodevelopment. The aim is to compare the short-term outcomes of VLBW infants in tertiary care centers according to their enteral feeding advancement.", "We prospectively studied the influence of center-specific enteral feeding advancement in 1430 VLBW infants recruited from 13 tertiary neonatal intensive care units in Germany on short-term outcome parameters. The centers were post hoc stratified to \"rapid advancement to full enteral feeds\" (median duration of advancement to full enteral feeds12.5 days; 7 centers), that is, slow advancement (SA).", "VLBW infants born in centers with SA (n = 713) had a significantly higher rate of sepsis compared with VLBW infants born in centers with RA (n = 717), which was particularly evident for late-onset sepsis (14.0% vs 20.4%; P = 0.002). Furthermore, more central venous lines (48.6% vs 31.1%, P<0.001) and antibiotics (92.4% vs 77.7%, P<0.001) were used in centers with SA." ], "LABELS": [ "BACKGROUND AND OBJECTIVES", "PATIENTS AND METHODS", "RESULTS" ], "MESHES": [ "Anti-Bacterial Agents", "Enteral Nutrition", "Female", "Humans", "Infant, Newborn", "Infant, Premature", "Infant, Very Low Birth Weight", "Intensive Care Units, Neonatal", "Pregnancy", "Prospective Studies", "Sepsis", "Time Factors", "Treatment Outcome" ], "YEAR": "2009", "reasoning_required_pred": "yes", "reasoning_free_pred": "yes", "final_decision": "yes", "LONG_ANSWER": "Center differences in enteral feeding advancement occur and may have a significant impact on short-term outcomes such as nosocomial sepsis. Large, multicenter, prospective trials are required to further elucidate the optimal feeding strategy for VLBW infants." }, "22522271": { "QUESTION": "Is late-night salivary cortisol a better screening test for possible cortisol excess than standard screening tests in obese patients with Type 2 diabetes?", "CONTEXTS": [ "Forty obese patients with T2DM without clinical features of Cushing's syndrome were recruited. Plasma, urinary and salivary cortisol were measured directly by an enzyme-linked immunosorbent assay using monoclonal antibodies. The specificities of the three tests using various cutoffs were calculated and compared, employing the assumption that none of the patients had hypercortisolism.", "The patients had a mean age and BMI of 56 years (range 31-75) and 37 kg/m\u00b2 (31-56) respectively. All 40 provided late-night salivary cortisol samples. Thirty-eight patients completed all three tests. Two patients only completed two screening tests. The specificities of late-night salivary cortisol (cutoff 10 nmol/L), 24hr UFC (400 nmol) and 1mg DST (50 nmol/L) were 70% (95% CI 53-83%), 90% (76-97%) and 72% (55-85%) respectively. The specificity of late-night salivary cortisol was significantly less than 24 hr UFC (P=0.039) but not 1mg DST (P>0.99)." ], "LABELS": [ "METHODS", "RESULTS" ], "MESHES": [ "Adult", "Aged", "Cushing Syndrome", "Diabetes Mellitus, Type 2", "Female", "Humans", "Hydrocortisone", "Male", "Middle Aged", "Obesity", "Saliva", "Time Factors", "Urinalysis" ], "YEAR": "2012", "reasoning_required_pred": "no", "reasoning_free_pred": "yes", "final_decision": "no", "LONG_ANSWER": "Late-night salivary cortisol has a poor specificity for cortisol excess in obese patients with T2DM with 24 hr UFC showing significantly better specificity in our population." }, "9363529": { "QUESTION": "Does psychological distress predict disability?", "CONTEXTS": [ "To evaluate psychological distress as a predictor of disability due to common chronic disorders.", "A 10-year follow-up study was carried out among a representative cohort (N = 8655) of 18-64 year old Finnish farmers, who had participated in a health survey in 1979 and were able to work at baseline. A record linkage with the nationwide register of the Social Insurance Institution was made to identify disability pensions granted between 1980 and 1990 in the cohort. The medical certificates of 1004 (11.6%) prematurely retired farmers were reviewed to confirm and classify disabling conditions. A sum score based on self-reports of 11 symptoms at the baseline was used as a measure of psychological distress.", "After adjustment for age, sex, smoking and body mass index, the cause-specific relative risks (RR) (95% confidence intervals [CI]) of disability in the highest quartile of the psychological distress score as compared with the lowest quartile were for myocardial infarction 2.34 (95% CI: 1.17-4.69), for depression 2.50 (95% CI: 1.09-5.72), for neck-shoulder disorders 1.98 (95% CI: 1.26-3.11), for unspecified low-back disorders 1.76 (95% CI: 1.24-2.49), for knee osteoarthritis 1.55 (95% CI: 0.91-2.63) and for trip osteoarthritis 0.89 (95% CI: 0.42-1.85). The corresponding RR for overall disability was 1.76 (95% CI: 1.44-2.14) in the highest quartile of psychological distress score as compared with the lowest quartile." ], "LABELS": [ "STUDY OBJECTIVE", "STUDY POPULATION AND METHODS", "RESULTS" ], "MESHES": [ "Adolescent", "Adult", "Age Distribution", "Agriculture", "Cardiovascular Diseases", "Cohort Studies", "Disabled Persons", "Female", "Finland", "Health Surveys", "Humans", "Incidence", "Male", "Middle Aged", "Musculoskeletal Diseases", "Predictive Value of Tests", "Proportional Hazards Models", "Registries", "Risk Factors", "Sex Distribution", "Stress, Psychological" ], "YEAR": "1997", "reasoning_required_pred": "yes", "reasoning_free_pred": "yes", "final_decision": "yes", "LONG_ANSWER": "Psychological distress is an independent risk factor for disability. Its predictive significance varies between disorders leading to functional deterioration. The association mechanisms are likely to vary from one disorder to another." }, "15962678": { "QUESTION": "Does preloading with colloids prevent hemodynamic changes when neurosurgical patients are subsequently changed to the seated position?", "CONTEXTS": [ "This prospective, randomized, double-blind study was designed to determine and compare the usefulness of preloading colloids (Haemaccel) 10 ml/Kg before positioning whether it can prevent hemodynamic changes during seated positioning or not.", "The authors studied 20 patients by randomly dividing them into 2 groups. The control group was given crystalloid as maintenance and deposit replacement but the study group was given extra colloids 10 ml/Kg 30 minutes before starting general anesthesia. Both groups were monitored and given anesthesia, balanced technique. Systolic and diastolic blood pressures, heart rate, central venous pressure (CVP) at different time intervals in the sitting position for 30 minutes were recorded. Statistical analysis was done by Student t-test, Chi-square test and ANOVA (p-value<0. 05 considered significant).", "The results showed that systolic blood pressure at 15, 20, 30 minutes and CVP at 15, 25, 30 minutes after positioning in the study group was maintained significantly compared to the control group and there were no significant changes in diastolic blood pressure and heart rate. There were no other complications during the sitting period." ], "LABELS": [ "OBJECTIVE", "MATERIAL AND METHOD", "RESULTS" ], "MESHES": [ "Adult", "Blood Pressure", "Double-Blind Method", "Female", "Fluid Therapy", "Hemodynamics", "Humans", "Male", "Middle Aged", "Neurosurgical Procedures", "Plasma Substitutes", "Polygeline", "Posture", "Preoperative Care", "Prospective Studies" ], "YEAR": "2005", "reasoning_required_pred": "yes", "reasoning_free_pred": "yes", "final_decision": "yes", "LONG_ANSWER": "It is concluded that preloading colloid fluid prior to repositioning could prevent the decrease of systolic blood pressure and central venous pressure during sitting positioning without other complications." }, "7547656": { "QUESTION": "Does continuous intravenous infusion of low-concentration epinephrine impair uterine blood flow in pregnant ewes?", "CONTEXTS": [ "Bolus intravenous injection of epinephrine can decrease uterine blood flow. This study examined the effects of intravenous infusion of epinephrine on uterine blood flow in the gravid ewe.", "Maternal and fetal vascular catheters and a maternal electromagnetic uterine artery flow probe were implanted in 10 near-term gravid ewes. After recovery, saline, 0.125% bupivacaine, 0.125% bupivacaine with 1:200,000 epinephrine, 0.125% bupivacaine with 1:400,000 epinephrine, and 0.125% bupivacaine with 1:800,000 epinephrine were infused into the maternal superior vena cava. Drugs were infused at 10 mL/h for 30 minutes and then at 20 mL/h for an additional 30 minutes. Animals also received an intravenous bolus of epinephrine 15 micrograms. Throughout all infusions, maternal heart rate, systemic and pulmonary blood pressures, uterine blood flow, cardiac output, and acid-base balance were measured, as well as fetal heart rate, blood pressure, and acid-base balance.", "Epinephrine 15 micrograms decreased uterine blood flow to 68 +/- 14% of baseline (mean +/- SD). Infusion of all solutions had no effect on any measured hemodynamic variable." ], "LABELS": [ "BACKGROUND AND OBJECTIVES", "METHODS", "RESULTS" ], "MESHES": [ "Anesthesia, Obstetrical", "Animals", "Dose-Response Relationship, Drug", "Epinephrine", "Female", "Fetus", "Hemodynamics", "Infusions, Intravenous", "Pregnancy", "Pregnancy, Animal", "Sheep", "Uterus" ], "YEAR": null, "reasoning_required_pred": "yes", "reasoning_free_pred": "no", "final_decision": "no", "LONG_ANSWER": "In gravid ewes, intravenous infusion of0.05)." ], "LABELS": [ "BACKGROUND", "PATIENTS AND METHODS", "RESULTS" ], "MESHES": [ "Adult", "Aged", "Aged, 80 and over", "Biomarkers, Tumor", "Carcinoma, Non-Small-Cell Lung", "DNA Mutational Analysis", "Female", "Genes, erbB-1", "Humans", "Lung Neoplasms", "Male", "Middle Aged", "Mutation", "Neoplasm Staging", "Polymerase Chain Reaction", "Sensitivity and Specificity" ], "YEAR": "2013", "reasoning_required_pred": "yes", "reasoning_free_pred": "yes", "final_decision": "yes", "LONG_ANSWER": "Serum sample might be alternatively used in the difficult time of getting tumor tissue for analyzing the status of EGFR mutation status in patients with advanced NSCLC." }, "25793749": { "QUESTION": "Do Web-based and clinic samples of gay men living with HIV differ on self-reported physical and psychological symptoms?", "CONTEXTS": [ "Although the Internet is commonly used to recruit samples in studies of human immunodeficiency virus (HIV)-related risk behaviors, it has not been used to measure patient-reported well-being. As the burden of long-term chronic HIV infection rises, the Internet may offer enormous potential for recruitment to research and interventions.", "This study aimed to compare two samples of gay men living with HIV, one recruited via the Web and the other recruited in outpatient settings, in terms of self-reported physical and psychological symptom burden.", "The Internet sample was recruited from a UK-wide Web-based survey of gay men with diagnosed HIV. Of these, 154 respondents identified themselves as resident in London and were included in this analysis. The HIV clinic sample was recruited from five HIV outpatient clinics. Of these participants, 400 gay men recruited in London clinics were included in this analysis.", "The Web-based sample was younger than the clinic sample (37.3 years, SD 7.0 vs 40.9 years, SD 8.3), more likely to be in paid employment (72.8%, 99/136 vs 60.1%, 227/378), less likely to be on antiretroviral therapy (ART) (58.4%, 90/154 vs 68.0%, 266/391), and had worse mean psychological symptom burden compared to the clinic sample (mean scores: 1.61, SD 1.09 vs 1.36, SD 0.96) but similar physical symptom burden (mean scores: 0.78, SD 0.65 vs 0.70, SD 0.74). In multivariable logistic regression, for the physical symptom burden model, adjusted for age, ethnicity, employment status, and ART use, the recruitment setting (ie, Web-based vs clinic) was not significantly associated with high physical symptom score. The only variable that remained significantly associated with high physical symptom score was employment status, with those in employment being less likely to report being in the upper (worst) physical symptom tertile versus the other two tertiles (adjusted OR 0.41, 95% CI 0.28-0.62, P<.001). For the psychological symptom burden model, those recruited via the Web were significantly more likely to report being in the upper (worst) tertile (adjusted OR 2.20, 95% CI 1.41-3.44, P=.001). In addition, those in employment were less likely to report being in the upper (worst) psychological symptom tertile compared to those not in employment (adjusted OR 0.32, 95% CI 0.21-0.49, P<.001)." ], "LABELS": [ "BACKGROUND", "OBJECTIVE", "METHODS", "RESULTS" ], "MESHES": [ "Adolescent", "Adult", "Ambulatory Care Facilities", "Cost of Illness", "Data Collection", "Employment", "Ethnic Groups", "HIV Infections", "Homosexuality, Male", "Humans", "Internet", "Male", "Middle Aged", "Patient Selection", "Risk-Taking", "Self Report", "United Kingdom" ], "YEAR": "2015", "reasoning_required_pred": "maybe", "reasoning_free_pred": "yes", "final_decision": "maybe", "LONG_ANSWER": "Our data have revealed a number of differences. Compared to the clinic sample, the Web-based sample had worse psychological symptom burden, younger average age, higher prevalence of employment, and a lower proportion on ART. For future research, we recommend that Web-based data collection should include the demographic variables that we note differed between samples. In addition, we recognize that each recruitment method may bring inherent sampling bias, with clinic populations differing by geographical location and reflecting those accessing regular medical care, and Web-based sampling recruiting those with greater Internet access and identifying survey materials through specific searches and contact with specific websites." }, "15095519": { "QUESTION": "Are patients with diabetes receiving the same message from dietitians and nurses?", "CONTEXTS": [ "The purpose of this study was to determine if registered dietitian (RD) and registered nurse (RN) certified diabetes educators (CDEs) provide similar recommendations regarding carbohydrates and dietary supplements to individuals with diabetes.", "A survey was mailed to CDEs in the southern United States. Participants were asked to indicate their recommendations for use of carbohydrates, fiber, artificial sweeteners, and 12 selected dietary and herbal supplements when counseling individuals with diabetes.", "The survey sample consisted of 366 CDEs: 207 were RNs and 159 were RDs. No statistically significant differences were found between RNs and RDs in typical carbohydrate recommendations for treatment of diabetes. However, RDs were more likely than RNs to make recommendations for fiber intake or use of the glycemic index. A significant difference also was found in the treatment of hypoglycemia: RNs were more likely than RDs to recommend consuming a carbohydrate source with protein to treat hypoglycemia." ], "LABELS": [ "PURPOSE", "METHODS", "RESULTS" ], "MESHES": [ "Certification", "Diabetes Mellitus", "Diet, Diabetic", "Dietary Carbohydrates", "Dietary Fiber", "Dietary Proteins", "Dietetics", "Female", "Guideline Adherence", "Humans", "Male", "Nurse Clinicians", "Nutritional Sciences", "Patient Education as Topic", "Phytotherapy", "Practice Guidelines as Topic", "Self Care", "Surveys and Questionnaires", "Sweetening Agents", "United States" ], "YEAR": null, "reasoning_required_pred": "no", "reasoning_free_pred": "yes", "final_decision": "no", "LONG_ANSWER": "Although some differences existed, RD and RN CDEs are making similar overall recommendations in the treatment of individuals with diabetes." }, "25981682": { "QUESTION": "Preparing patients at high risk of falls for discharge home after rehabilitation: Do we meet the guidelines?", "CONTEXTS": [ "Medical records of 121 people who received inpatient rehabilitation were retrospectively screened; records of 50 people discharged home and at high falls risk (fall in last 12 months, fall preceding/during admission) were audited. Data extracted included falls risk identification during rehabilitation and in discharge documentation; falls risk factors assessed; and fall prevention strategies implemented.", "Discharge documentation correctly identified falls risk for just nine of the 50 people. Patients at high falls risk had a median of 8.0 (interquartile range 6-10) of 17 risk factors. There was limited evidence of assessment for osteoporosis (n\u2009=\u20098), footwear (n\u2009=\u20094) and visual assessment in the previous 2 years (n\u2009=\u20091). Patients received a median of 6.5 (interquartile range 5-9) out of 16 possible strategies. Common strategies were mobility (n\u2009=\u200948), strength (n\u2009=\u200944) and Personal Activity of Daily Living training (n\u2009=\u200943). For 12 risk factors, if the factor was present, there was evidence of a strategy in more than 80% of records." ], "LABELS": [ "METHODS", "RESULTS" ], "MESHES": [ "Accidental Falls", "Aged", "Aged, 80 and over", "Australia", "Female", "Guideline Adherence", "Humans", "Male", "Middle Aged", "Patient Discharge", "Patient Handoff", "Practice Guidelines as Topic", "Rehabilitation", "Retrospective Studies", "Risk Assessment" ], "YEAR": "2016", "reasoning_required_pred": "yes", "reasoning_free_pred": "no", "final_decision": "no", "LONG_ANSWER": "There was little evidence that people at high risk of falls received systematic falls risk assessment during rehabilitation. When a risk was identified, generally a strategy was implemented. However, failure to assess some risk factors might have limited fall prevention strategies offered. Failure to adequately address risks during hospitalization could contribute to falls post-discharge. Geriatr Gerontol Int 2016; 16: 570-576." }, "18284441": { "QUESTION": "Expression of c-kit protooncogen in hepatitis B virus-induced chronic hepatitis, cirrhosis and hepatocellular carcinoma: has it a diagnostic role?", "CONTEXTS": [ "Paraffin-embedded tissues in Cukurova University Faculty of Medicine Department of Pathology between January 2002 and February 2006 were searched restrospectively to investigate this issue. We performed immunohistochemistry on biopsies of 125 patients with HBV infection, grouped as: mild, moderate and severe hepatitis, cirrhosis and HCC, 25 patients in each of them, using anti c-kit monoclonal antibody. The severity of parenchymal inflammation and of interface hepatitis was semiquantitatively graded on a haematoxylin and eosin stained paraffin sections. Additionally, 50 more HCC, formed on HBV basis, were studied to determine the prevalence of c-kit overexpression.", "In cirrhotic liver, lower intensity of staining and rarely c-kit positivity were present. The greatest number of the c-kit positivity and higher intensity of staining was found in the livers of patients with severe hepatitis and HCC. In chronic hepatitis B infection, the staining intensity was parallel with the grade and stage of the disease. In the areas where fibrosis was seen, c-kit positivity was rare or absent. In the HCC specimens, c-kit positivity appeared both inside and around the cancerous nodes. C-kit expression was observed in 62 of 75 HCC tissue specimens (82%) (p<0.001)." ], "LABELS": [ "MATERIALS AND METHODS", "RESULTS" ], "MESHES": [ "Adult", "Aged", "Biomarkers, Tumor", "Biopsy", "Carcinoma, Hepatocellular", "Early Diagnosis", "Female", "Hepatitis B, Chronic", "Hepatocytes", "Humans", "Immunohistochemistry", "Liver Cirrhosis", "Liver Neoplasms", "Male", "Middle Aged", "Multivariate Analysis", "Proto-Oncogene Proteins c-kit", "Severity of Illness Index" ], "YEAR": "2008", "reasoning_required_pred": "maybe", "reasoning_free_pred": "yes", "final_decision": "maybe", "LONG_ANSWER": "C-kit positivity was observed in the mitotic, proliferating and also dysplastic hepatic cells. These results suggest that c-kit expression may be used as an early diagnostic indicator for HBV induced HCC." }, "10173769": { "QUESTION": "Longer term quality of life and outcome in stroke patients: is the Barthel index alone an adequate measure of outcome?", "CONTEXTS": [ "To consider whether the Barthel Index alone provides sufficient information about the long term outcome of stroke.", "Cross sectional follow up study with a structured interview questionnaire and measures of impairment, disability, handicap, and general health. The scales used were the hospital anxiety and depression scale, mini mental state examination, Barthel index, modified Rankin scale, London handicap scale, Frenchay activities index, SF36, Nottingham health profile, life satisfaction index, and the caregiver strain index.", "South east London.", "People, and their identified carers, resident in south east London in 1989-90 when they had their first in a life-time stroke aged under 75 years.", "Observational study.", "Comparison and correlation of the individual Barthel index scores with the scores on other outcome measures.", "One hundred and twenty three (42%) people were known to be alive, of whom 106 (86%) were interviewed. The median age was 71 years (range 34-79). The mean interval between the stroke and follow up was 4.9 years. The rank correlation coefficients between the Barthel and the different dimensions of the SF36 ranged from r = 0.217 (with the role emotional dimension) to r = 0.810 (with the physical functioning dimension); with the Nottingham health profile the range was r = -0.189 (with the sleep dimension, NS) to r = -0.840 (with the physical mobility dimension); with the hospital and anxiety scale depression component the coefficient was r = -0.563, with the life satisfaction index r = 0.361, with the London handicap scale r = 0.726 and with the Frenchay activities index r = 0.826." ], "LABELS": [ "OBJECTIVES", "DESIGN", "SETTING", "SUBJECTS", "INTERVENTIONS", "MAIN OUTCOME MEASURES", "RESULTS" ], "MESHES": [ "Activities of Daily Living", "Adult", "Aged", "Caregivers", "Cerebrovascular Disorders", "Cognition Disorders", "Cohort Studies", "Cross-Sectional Studies", "Disabled Persons", "Humans", "London", "Middle Aged", "Outcome Assessment (Health Care)", "Patient Satisfaction", "Quality of Life", "Registries", "State Medicine" ], "YEAR": "1997", "reasoning_required_pred": "no", "reasoning_free_pred": "yes", "final_decision": "yes", "LONG_ANSWER": "The place of the Barthel index as the standard outcome measure for populations of stroke patients is still justified for long term follow up, and may be a proxy for different outcome measures intended for the assessment of other domains." }, "20354380": { "QUESTION": "Do women residents delay childbearing due to perceived career threats?", "CONTEXTS": [ "To assess gender differences among residents regarding their plans to have children during residency and determine the most influential reasons for these differences.", "Using the Health Belief Model as a framework, the authors created an instrument to survey 424 residents from 11 residency programs at three academic medical institutions about their intentions to have children during residency. The authors developed a scale to assess the perceived career threats of having children during residency, evaluated its psychometric properties, and calculated the effect of the mediators.", "The response rate was 77% (328/424). Forty-one percent of men versus 27% of women planned to have children during residency (P = .01). The instrument measured four career threats-extended training, loss of fellowship positions, pregnancy complications, and interference with career plans-on a five-point Likert scale. The scale had a Cronbach alpha of 0.84 and an eigenvalue of 2.2. Compared with men, women had higher scores for each item and a higher mean score (2.9 versus 2.1, P = .001), signifying greater belief in the potential of pregnancy to threaten careers. After adjusting for age, institution, postgraduate year, and knowledge of parental leave policies, women were less likely to plan to have children during residency (odds ratio 0.46 [95% confidence interval 0.25-0.84]). In mediation analysis, threats to career explained 67% of the gender variance." ], "LABELS": [ "PURPOSE", "METHOD", "RESULTS" ], "MESHES": [ "Adult", "Career Mobility", "Cross-Sectional Studies", "Female", "Humans", "Internship and Residency", "Male", "Physicians, Women", "Pregnancy", "Psychometrics", "Reproductive Behavior", "Sex Factors", "Surveys and Questionnaires", "United States" ], "YEAR": "2010", "reasoning_required_pred": "yes", "reasoning_free_pred": "yes", "final_decision": "yes", "LONG_ANSWER": "Women residents intentionally postpone pregnancy because of perceived threats to their careers. Medical educators should be aware of these findings when counseling female trainees." }, "9616411": { "QUESTION": "Do general practitioner hospitals reduce the utilisation of general hospital beds?", "CONTEXTS": [ "To assess whether populations with access to general practitioner hospitals (GP hospitals) utilise general hospitals less than populations without such access.", "Observational study comparing the total rates of admissions and of occupied bed days in general hospitals between populations with and without access to GP hospitals. Comparisons were also made separately for diagnoses commonly encountered in GP hospitals.", "Two general hospitals serving the population of Finnmark county in north Norway.", "35,435 admissions based on five years' routine recordings from the two hospitals.", "The total rate of admission to general hospitals was lower in peripheral municipalities with a GP hospital than in central municipalities without this kind of institution, 26% and 28% lower for men and women respectively. The corresponding differences were 38% and 52%, when analysed for occupied bed days. The differences were most pronounced for patients with respiratory diseases, cardiac failure, and cancer who are primarily or intermediately treated or cared for in GP hospitals, and for patients with stroke and fractures, who are regularly transferred from general hospitals to GP hospitals for longer term follow up care." ], "LABELS": [ "STUDY OBJECTIVE", "DESIGN", "SETTING", "PATIENTS", "MAIN RESULTS" ], "MESHES": [ "Adolescent", "Adult", "Aged", "Bed Occupancy", "Child", "Child, Preschool", "Female", "Health Services Accessibility", "Hospitals, County", "Hospitals, General", "Hospitals, Group Practice", "Humans", "Infant", "Infant, Newborn", "Length of Stay", "Male", "Middle Aged", "Norway", "Patient Admission", "Retrospective Studies" ], "YEAR": "1998", "reasoning_required_pred": "yes", "reasoning_free_pred": "yes", "final_decision": "yes", "LONG_ANSWER": "GP hospitals seem to reduce the utilisation of general hospitals with respect to admissions as well as occupied bed days." }, "22205377": { "QUESTION": "Some aspects of social exclusion: do they influence suicide mortality?", "CONTEXTS": [ "The current study is aimed to assess the relationship between the 'economic/employment' and 'social/welfare' dimensions of social exclusion and suicide mortality in Europe.", "Suicide rates for 26 countries were obtained from the WHO. Data on social expenditure were obtained from the OECD database. Employment rates and GDP were obtained from the Total Economy Database. Questions about citizens' attitudes towards different aspects of social exclusion were taken from the European Social Survey. Structural equation modelling was applied to research the theoretical structure of the variables.", "All variables are statistically significant in male and female models except of the relationships between 'economic/employment' and 'social/welfare' dimensions and female suicides; and the relationship between 'employment rates' and 'economic/employment' dimension. Suicide mortality rates among both males and females are influenced negatively by 'economic/employment' and 'social/welfare' dimensions. Among females, the influence of 'social/welfare' dimension is stronger compared to the 'economic/employment' dimension. The remaining influence of GDP is positive in both models." ], "LABELS": [ "BACKGROUND", "METHODS", "RESULTS" ], "MESHES": [ "Employment", "Europe", "Female", "Humans", "Male", "Risk Factors", "Sex Distribution", "Social Distance", "Social Welfare", "Socioeconomic Factors", "Suicide" ], "YEAR": "2013", "reasoning_required_pred": "yes", "reasoning_free_pred": "yes", "final_decision": "yes", "LONG_ANSWER": "Both 'economic/employment' and 'social/welfare' dimensions of social exclusion significantly influence suicide mortality among males. The influence of 'economic/employment' and 'social/welfare' dimensions of social exclusion on female suicide mortality is controversial. Social exclusion might be considered as a risk factor for suicide mortality in Europe." }, "16241924": { "QUESTION": "Does parity increase insulin resistance during pregnancy?", "CONTEXTS": [ "To study the effect of parity on impairment of insulin sensitivity during pregnancy and on the risk of gestational diabetes (GDM).", "We studied the relationship between parity and peripheral insulin sensitivity index (ISI(OGTT)) or GDM in 1880 caucasian women, who underwent a 100-g, 3-h oral glucose tolerance test (OGTT) between the 24th and 28th gestational week and in 75 women who underwent an OGTT in two consecutive pregnancies. A proxy for beta-cell function (basal plasma C peptide/fasting plasma glucose; CP/FPG) was also measured.", "By univariate analysis parity was related to decreased ISI(OGTT) and to increased CP/FPG in those with parity>3 and likewise GDM, diagnosed in 124 women (6.58%), was linearly related to parity (P = 0.0034) and strongly age dependent. The relationships between parity and ISI(OGTT), CP/FPG and GDM were no longer significant after adjustment for age, pregestational body mass index (BMI), and weight gain. GDM was significantly related to age and pregestational weight, while ISI(OGTT) and CP/FPG were inversely related to prepregnancy BMI or weight gain. In comparison with the index pregnancy, the subsequent pregnancy was characterized by an increase in actual and prepregnancy BMI, in 2 h area under curve (AUC) glucose and by a decrease in ISI(OGTT) (P = 0.0001). The longer the time interval between pregnancies and the higher the increment in pregestational BMI or in weight gain during the pregnancy, the greater were the ISI(OGTT) decrease and 2-h AUC glucose increase." ], "LABELS": [ "AIMS", "METHODS", "RESULTS" ], "MESHES": [ "Adult", "Blood Glucose", "Body Mass Index", "Cross-Sectional Studies", "Diabetes, Gestational", "Female", "Glucose Tolerance Test", "Humans", "Insulin Resistance", "Parity", "Pregnancy", "Pregnancy Trimester, Third", "Retrospective Studies", "Risk Factors", "Weight Gain" ], "YEAR": "2005", "reasoning_required_pred": "no", "reasoning_free_pred": "no", "final_decision": "no", "LONG_ANSWER": "Parity is not directly linked to insulin sensitivity deterioration, to CP/FPG increase during pregnancy, or to GDM appearance, although it is linked through the mediation of progressive ageing and weight gain either before or during pregnancy, when there is a sufficiently long time interval between pregnancies." }, "12484580": { "QUESTION": "Informed consent for total hip arthroplasty: does a written information sheet improve recall by patients?", "CONTEXTS": [ "To ascertain whether a written information sheet is acceptable to patients and improves recall of the consent interview.", "Prospective randomised controlled study using questionnaires, comparing a group of patients given information in a written sheet with appropriate explanation to a group given verbal information alone.", "A specialist orthopaedic surgery unit.", "The test group was 126 patients undergoing revision or primary total hip arthroplasty; 65 patients were given information verbally, 61 patients were given written information.", "Patients' recall of information given, tested with a questionnaire completed on admission (mean of 18 days later).", "The patients receiving written information scored significantly higher (48% correct answers) than the patients receiving verbal information (38% correct answers)." ], "LABELS": [ "OBJECTIVE", "DESIGN", "SETTING", "PATIENTS", "OUTCOME MEASURE", "RESULTS" ], "MESHES": [ "Arthroplasty, Replacement, Hip", "Female", "Humans", "Informed Consent", "Male", "Mental Recall", "Patient Education as Topic", "Professional-Patient Relations", "Prospective Studies", "Surveys and Questionnaires" ], "YEAR": "2002", "reasoning_required_pred": "yes", "reasoning_free_pred": "yes", "final_decision": "yes", "LONG_ANSWER": "Written information sheets contribute to the process of informed consent. As patients' recall of information is generally poor, the sheets may also be useful medicolegally, as a permanent record of what was discussed." }, "24599411": { "QUESTION": "Is gastric cancer different in Korea and the United States?", "CONTEXTS": [ "To compare the characteristics and prognoses of gastric cancers by tumor location in Korean and U.S. subjects after curative-intent (R0) resection for gastric cancer (GC).", "Data were collected for all patients who had undergone R0 resection at one U.S. institution (n = 567) and one South Korean institution (n = 1,620). Patients with gastroesophageal junction tumors or neoadjuvant therapy were excluded. Patient, surgical, and pathologic variables were compared by tumor location. Factors associated with disease-specific survival (DSS) were determined via multivariate analysis.", "In the Korean cohort, significantly more upper third GC (UTG) patients had undifferentiated, diffuse type, and advanced stage cancers compared to lower third GC (LTG) and middle third GC (MTG) patients. In the U.S. cohort, however, T stage was relatively evenly distributed among UTG, MTG, and LTG patients. The independent predictors of DSS in the Korean cohort were T stage, tumor size, retrieved and positive lymph node counts, and age, but in the U.S. cohort, the only independent predictors were T stage and positive lymph node count. Tumor size significantly affected DSS of Korean UTG patients but not U.S. UTG patients." ], "LABELS": [ "PURPOSE", "METHODS", "RESULTS" ], "MESHES": [ "Adenocarcinoma", "Adolescent", "Adult", "Aged", "Aged, 80 and over", "Female", "Follow-Up Studies", "Humans", "Male", "Middle Aged", "Neoplasm Staging", "Nomograms", "Prognosis", "Republic of Korea", "Retrospective Studies", "Stomach Neoplasms", "Survival Rate", "Tomography, X-Ray Computed", "United States", "Young Adult" ], "YEAR": "2014", "reasoning_required_pred": "yes", "reasoning_free_pred": "yes", "final_decision": "yes", "LONG_ANSWER": "There were significant differences in tumor characteristics by tumor location within and between both national cohorts. On the basis of these findings, further study to investigate the biological difference between the two countries is needed." }, "23860049": { "QUESTION": "Do we need imaging to diagnose appendicitis in children?", "CONTEXTS": [ "To evaluate the role of clinical assessment with selective use of imaging studies in the management of suspected acute appendicitis in children.", "Medical records of children referred to Emergency Room in 2010 for suspected appendicitis were retrospectively reviewed. Diagnostic investigations divided by age and sex were related to pathological findings. Negative appendectomy and complication rates were calculated.", "923 children needed surgical assessment : In 75.7% of them surgical indication was excluded and 24.3% were admitted to surgical ward for observation. Appendectomy was eventually performed in 137 patients (61.9%), 82.4% of them without any preoperative imaging while 17.6% underwent selective studies, mainly abdominal ultrasonography (14.6%). Imaging was requested twice as frequently in not operated admitted children (39.3%) than in the operated ones (17.5%, P<0.001). Overall complicated appendicitis rate (peritonitis and abscess) resulted 26.4% and negative appendectomy rate 8.8%. Females older than 10 years presented histologically not-confirmed appendicitis in 22.2% of cases, while the younger ones presented more frequently complicated appendicitis (29.3%)." ], "LABELS": [ "BACKGROUND", "PATIENTS AND METHODS", "RESULTS" ], "MESHES": [ "Acute Disease", "Adolescent", "Appendectomy", "Appendicitis", "Child", "Child, Preschool", "Diagnosis, Differential", "Diagnostic Errors", "Diagnostic Imaging", "Early Diagnosis", "Female", "Follow-Up Studies", "Humans", "Infant", "Infant, Newborn", "Male", "Retrospective Studies" ], "YEAR": null, "reasoning_required_pred": "no", "reasoning_free_pred": "maybe", "final_decision": "maybe", "LONG_ANSWER": "Clinical assessment is the key to diagnose appendicitis. Nevertheless, in girls older than 10 years, selected use of imaging should be implemented to avoid unnecessary appendectomies. Imaging of choice in equivocal cases should be ultrasonography." }, "7664228": { "QUESTION": "Discharging patients earlier from Winnipeg hospitals: does it adversely affect quality of care?", "CONTEXTS": [ "To determine whether decreasing lengths of stay over time for selected diagnostic categories were associated with increased hospital readmission rates and mean number of physician visits after discharge.", "Retrospective descriptive study.", "The seven large (125 beds or more) acute care hospitals in Winnipeg.", "Manitoba residents admitted to any one of the seven hospitals because acute myocardial infarction (AMI), bronchitis or asthma, transurethral prostatectomy (TURP) and uterine or adnexal procedures for nonmalignant disease during the fiscal years 1989-90 to 1992-93. Patients from out of province, those who died in hospital, those with excessively long stays (more than 60 days) and those who were transferred to or from another institution were excluded.", "Length of hospital stay, and rate of readmission within 30 days after discharge for all four categories and mean number of physician visits within 30 days after discharge for two categories (AMI and bronchitis or asthma.", "The length of stay decreased significantly over the 4 years for all of the four categories, the smallest change being observed for patients with AMI (11.1%) and the largest for those with bronchitis or asthma (22.0%). The readmission rates for AMI, bronchitis or asthma, and TURP showed no consistent change over the 4 years. The readmission rate for uterine or adnexal procedures increased significantly between the first and second year (chi 2 = 4.28, p = 0.04) but then remained constant over the next 3 years. The mean number of physician visits increased slightly for AMI in the first year (1.92 to 2.01) and then remained virtually the same. It decreased slightly for bronchitis or asthma over the 4 years. There was no significant correlation between length of stay and readmission rates for individual hospitals in 1992-93 in any of the four categories. Also, no correlation was observed between length of stay and mean number of physician visits for individual hospitals in 1992-93 in the categories AMI and bronchitis or asthma." ], "LABELS": [ "OBJECTIVE", "DESIGN", "SETTING", "PATIENTS", "OUTCOME MEASURES", "RESULTS" ], "MESHES": [ "Diagnosis-Related Groups", "Humans", "Length of Stay", "Manitoba", "Office Visits", "Patient Readmission", "Quality of Health Care", "Retrospective Studies" ], "YEAR": "1995", "reasoning_required_pred": "no", "reasoning_free_pred": "no", "final_decision": "no", "LONG_ANSWER": "Improving hospital efficiency by shortening length of stay does not appear to result in increased rates of readmission or numbers of physician visits within 30 days after discharge from hospital. Research is needed to identify optimal lengths of stay and expected readmission rates." }, "22233470": { "QUESTION": "Does the distribution of health care benefits in Kenya meet the principles of universal coverage?", "CONTEXTS": [ "The 58th World Health Assembly called for all health systems to move towards universal coverage where everyone has access to key promotive, preventive, curative and rehabilitative health interventions at an affordable cost. Universal coverage involves ensuring that health care benefits are distributed on the basis of need for care and not on ability to pay. The distribution of health care benefits is therefore an important policy question, which health systems should address. The aim of this study is to assess the distribution of health care benefits in the Kenyan health system, compare changes over two time periods and demonstrate the extent to which the distribution meets the principles of universal coverage.", "Two nationally representative cross-sectional households surveys conducted in 2003 and 2007 were the main sources of data. A comprehensive analysis of the entire health system is conducted including the public sector, private-not-for-profit and private-for-profit sectors. Standard benefit incidence analysis techniques were applied and adopted to allow application to private sector services.", "The three sectors recorded similar levels of pro-rich distribution in 2003, but in 2007, the private-not-for-profit sector was pro-poor, public sector benefits showed an equal distribution, while the private-for-profit sector remained pro-rich. Larger pro-rich disparities were recorded for inpatient compared to outpatient benefits at the hospital level, but primary health care services were pro-poor. Benefits were distributed on the basis of ability to pay and not on need for care." ], "LABELS": [ "BACKGROUND", "METHODS", "RESULTS" ], "MESHES": [ "Cross-Sectional Studies", "Female", "Health Policy", "Health Services Needs and Demand", "Healthcare Disparities", "Hospitals, Proprietary", "Hospitals, Public", "Hospitals, Voluntary", "Humans", "Insurance Benefits", "Insurance, Health", "Kenya", "Male", "Socioeconomic Factors", "Universal Coverage" ], "YEAR": "2012", "reasoning_required_pred": "no", "reasoning_free_pred": "no", "final_decision": "no", "LONG_ANSWER": "The principles of universal coverage require that all should benefit from health care according to need. The Kenyan health sector is clearly inequitable and benefits are not distributed on the basis of need. Deliberate efforts should be directed to restructuring the Kenyan health system to address access barriers and ensure that all Kenyans benefit from health care when they need it." }, "17971187": { "QUESTION": "Cholesterol screening in school children: is family history reliable to choose the ones to screen?", "CONTEXTS": [ "The study was carried on 2096 school children (1043 male, 1053 female) in Ankara. Their mean age was 9.03 years. Demographic properties of the study group and their families were determined and the serum lipid levels of the subjects were obtained. The relation between these demographic properties and lipid levels were investigated.", "In 135 of the subjects' serum cholesterol level was>or=200 mg/dL and in 83 subjects serum LDL-cholesterol level was>or=130 mg/dL. Despite 64.4% of the subjects reported a family history of hyperlipidemia, no relations between family history and serum lipid levels were found." ], "LABELS": [ "METHODS", "RESULTS" ], "MESHES": [ "Adolescent", "Atherosclerosis", "Body Mass Index", "Child", "Child, Preschool", "Family", "Female", "Humans", "Hyperlipidemias", "Lipoproteins", "Male", "Mass Screening", "Medical History Taking", "Risk Factors", "Socioeconomic Factors", "Surveys and Questionnaires", "Turkey" ], "YEAR": "2007", "reasoning_required_pred": "no", "reasoning_free_pred": "no", "final_decision": "no", "LONG_ANSWER": "We suggest that regardless of family history, all children over 5 years should be screened for hyperlipidemia. Education about hyperlipidemia and precautions for its complications should be given to both children and families. The best and easiest way to reach children is to screen them at schools. School is also a good place for education of children about hyperlipidemia and risk factors." }, "27394685": { "QUESTION": "Bony defects in chronic anterior posttraumatic dislocation of the shoulder: Is there a correlation between humeral and glenoidal lesions?", "CONTEXTS": [ "The prevalence of combined humeral and glenoid defects varies between 79 and 84\u00a0% in case of chronic posttraumatic anterior shoulder instability. The main goal of this study was to evaluate the relationship between humeral and glenoid defects based on quantitative radiological criteria.", "A retrospective study was performed between 2000 and 2011 including patients who underwent primary surgical shoulder stabilization for chronic posttraumatic anterior shoulder instability, with bone defects in both the glenoid and humerus and a healthy contralateral shoulder. The following measurements were taken: D/R ratio (Hill-Sachs lesion depth/humeral head radius) on an AP X-ray in internal rotation and the D1/D2 ratio [diameter of the involved glenoid articular surfaces (D1)/the healthy one (D2)] on a comparative Bernageau glenoid profile view. Measurements were taken by two observers. Correlations were determined by the Spearman correlation coefficients (r), Bland and Altman diagrams, and intra-class correlation coefficients (ICC). A sample size calculation was done.", "Thirty patients were included, 25 men/5 women, mean age 29.8\u00a0\u00b1\u00a011.2\u00a0years. The mean D/R was 23\u00a0\u00b1\u00a012\u00a0% for observer 1 and 23\u00a0\u00b1\u00a010\u00a0% for observer 2. The mean D1/D2 was 95\u00a0\u00b1\u00a04\u00a0% for observer 1 and 94\u00a0\u00b1\u00a06\u00a0% for observer 2. No significant correlation was found between humeral and glenoid bone defects by observer 1 (r\u00a0=\u00a00.23, p\u00a0=\u00a00.22) or observer 2 (r\u00a0=\u00a00.05, p\u00a0=\u00a00.78). Agreement of the observers for the D/R ratio was excellent (ICC\u00a0=\u00a00.89\u00a0\u00b1\u00a00.04, p\u00a0<\u00a00.00001) and good for the D1/D2 ratio (ICC\u00a0=\u00a00.54\u00a0\u00b1\u00a00.14, p\u00a0=\u00a00.006)." ], "LABELS": [ "BACKGROUND", "METHODS", "RESULTS" ], "MESHES": [ "Adult", "Arthrodesis", "Female", "France", "Humans", "Humerus", "Joint Instability", "Male", "Outcome and Process Assessment (Health Care)", "Prevalence", "Radiography", "Retrospective Studies", "Scapula", "Shoulder Dislocation", "Shoulder Joint", "Statistics as Topic" ], "YEAR": "2016", "reasoning_required_pred": "no", "reasoning_free_pred": "no", "final_decision": "no", "LONG_ANSWER": "Humeral and glenoid bone defects were not correlated. Inter-observer reliability was excellent for the D/R ratio and good for the D1/D2 ratio." }, "25079920": { "QUESTION": "Do parents recall and understand children's weight status information after BMI screening?", "CONTEXTS": [ "As parents of young children are often unaware their child is overweight, screening provides the opportunity to inform parents and provide the impetus for behaviour change. We aimed to determine if parents could recall and understand the information they received about their overweight child after weight screening.", "Randomised controlled trial of different methods of feedback.", "Participants were recruited through primary and secondary care but appointments took place at a University research clinic.", "1093 children aged 4-8\u2005years were screened. Only overweight children (n=271, 24.7%) are included in this study. Parents of overweight children were randomised to receive feedback regarding their child's weight using best practice care (BPC) or motivational interviewing (MI) at face-to-face interviews typically lasting 20-40\u2005min. 244 (90%) parents participated in a follow-up interview 2\u2005weeks later to assess recall and understanding of information from the feedback session.", "Interviews were audio-taped and transcribed verbatim before coding for amount and accuracy of recall. Scores were calculated for total recall and sub-categories of interest.", "Overall, 39% of the information was recalled (mean score 6.3 from possible score of 16). Parents given feedback via BPC recalled more than those in the MI group (difference in total score 0.48; 95% CI 0.05 to 0.92). Although 94% of parents were able to correctly recall their child's weight status, fewer than 10 parents could accurately describe what the measurements meant. Maternal education (0.81; 0.25 to 1.37) and parental ratings of how useful they found the information (0.19; 0.04 to 0.35) were significant predictors of recall score in multivariate analyses." ], "LABELS": [ "OBJECTIVES", "DESIGN", "SETTING", "PARTICIPANTS AND INTERVENTION", "PRIMARY AND SECONDARY OUTCOME MEASURES", "RESULTS" ], "MESHES": [ "Adult", "Body Mass Index", "Body Weight", "Child", "Child, Preschool", "Feedback", "Female", "Humans", "Male", "Mass Screening", "Mental Recall", "Middle Aged", "Motivational Interviewing", "Parents", "Pediatric Obesity", "Research Design" ], "YEAR": "2014", "reasoning_required_pred": "no", "reasoning_free_pred": "maybe", "final_decision": "maybe", "LONG_ANSWER": "While parents remember that their child's body mass index is higher than recommended, they are unable to remember much of the information and advice provided about the result." }, "17355582": { "QUESTION": "Does ambulatory process of care predict health-related quality of life outcomes for patients with chronic disease?", "CONTEXTS": [ "The validity of quality of care measurement has important implications for practicing clinicians, their patients, and all involved with health care delivery. We used empirical data from managed care patients enrolled in west coast physician organizations to test the hypothesis that observed changes in health-related quality of life across a 2.5-year window reflecting process of care.DATA SOURCES/", "Patient self-report data as well as clinically detailed medical record review regarding 963 patients with chronic disease associated with managed care from three west coast states.", "Prospective cohort study of change in health-related quality of life scores across 30 months as measured by change in SF-12 physical component scores.DATA COLLECTION/", "Patient self-report and medical record abstraction.", "We found a positive relationship between better process scores and higher burden of illness (p<.05). After adjustment for burden of illness, using an instrumental variables approach revealed better process is associated with smaller declines in SF-12 scores across a 30-month observation window (p=.014). The application of the best quartile of process of care to patients currently receiving poor process is associated with a 4.24 increment in delta SF-12-physical component summary scores." ], "LABELS": [ "OBJECTIVE", "STUDY SETTING", "STUDY DESIGN", "EXTRACTION METHODS", "PRINCIPAL FINDINGS" ], "MESHES": [ "Aged", "Chronic Disease", "Female", "Health Status", "Humans", "Male", "Managed Care Programs", "Middle Aged", "Outcome and Process Assessment (Health Care)", "Prospective Studies", "Quality of Health Care", "Quality of Life" ], "YEAR": "2007", "reasoning_required_pred": "yes", "reasoning_free_pred": "yes", "final_decision": "yes", "LONG_ANSWER": "The use of instrumental variables allowed us to demonstrate a significant relationship between better ambulatory process of care and better health-related quality of life. This finding underscores the importance of efforts to improve the process of care." }, "20736672": { "QUESTION": "Does perspective-taking increase patient satisfaction in medical encounters?", "CONTEXTS": [ "To assess whether perspective-taking, which researchers in other fields have shown to induce empathy, improves patient satisfaction in encounters between student-clinicians and standardized patients (SPs).", "In three studies, randomly assigned students (N = 608) received either a perspective-taking instruction or a neutral instruction prior to a clinical skills examination in 2006-2007. SP satisfaction was the main outcome in all three studies. Study 1 involved 245 third-year medical students from two universities. Studies 2 and 3 extended Study 1 to examine generalizability across student and SP subpopulations. Study 2 (105 physician assistant students, one university) explored the effect of perspective-taking on African American SPs' satisfaction. Study 3 (258 third-year medical students, two universities) examined the intervention's effect on students with high and low baseline perspective-taking tendencies.", "Intervention students outscored controls in patient satisfaction in all studies: Study 1: P = .01, standardized effect size = 0.16; Study 2: P = .001, standardized effect size = 0.31; Study 3: P = .009, standardized effect size = 0.13. In Study 2, perspective-taking improved African American SPs' satisfaction. In Study 3, intervention students with high baseline perspective-taking tendencies outscored controls (P = .0004, standardized effect size = 0.25), whereas those with low perspective-taking tendencies did not (P = .72, standardized effect size = 0.00)." ], "LABELS": [ "PURPOSE", "METHOD", "RESULTS" ], "MESHES": [ "Analysis of Variance", "Clinical Competence", "Communication", "Curriculum", "Education, Medical, Undergraduate", "Educational Measurement", "Empathy", "Humans", "Patient Satisfaction", "Physician-Patient Relations", "Students, Medical" ], "YEAR": "2010", "reasoning_required_pred": "maybe", "reasoning_free_pred": "yes", "final_decision": "maybe", "LONG_ANSWER": "Perspective-taking increased patient satisfaction in all three studies, across medical schools, clinical disciplines, and racially diverse students and SPs. Perspective-taking as a means for improving patient satisfaction deserves further exploration in clinical training and practice." }, "12419743": { "QUESTION": "Is first-line single-agent mitoxantrone in the treatment of high-risk metastatic breast cancer patients as effective as combination chemotherapy?", "CONTEXTS": [ "To determine whether patients with high-risk metastatic breast cancer draw benefit from combination chemotherapy as first-line treatment.", "A total of 260 women with measurable metastatic breast cancer fulfilling high-risk criteria, previously untreated with chemotherapy for their metastatic disease, were randomized to receive either mitoxantrone 12 mg/m(2) or the combination of fluorouracil 500 mg/m(2), epirubicin 50 mg/m(2) and cyclophosphamide 500 mg/m(2) (FEC) every 3 weeks. Treatment was continued until complete remission plus two cycles, or until disease progression. In the case of partial remission or stable disease, treatment was stopped after 12 cycles. Second-line treatment was vindesine, mitomycin and prednisolone. Gain from treatment was estimated using a modified Brunner's score composed of time to progression, patients' rating of the treatment benefit, alopecia, vomiting and performance status.", "After recruitment from 1992 to 1997 and observation from 1997 to 1999, the final evaluation showed that single-agent treatment with mitoxantrone does not differ significantly from combination treatment with FEC in terms of response, objective remission rate, remission duration, time to response, time to best response, time to progression or overall survival. There was, however, a significant difference in gain from treatment using a modified Brunner's score favoring the single-agent treatment arm. There was no evidence that any subgroup would fare better with combination treatment." ], "LABELS": [ "BACKGROUND", "PATIENTS AND METHODS", "RESULTS" ], "MESHES": [ "Adult", "Aged", "Antineoplastic Combined Chemotherapy Protocols", "Biopsy, Needle", "Bone Neoplasms", "Breast Neoplasms", "Cyclophosphamide", "Disease-Free Survival", "Epirubicin", "Female", "Fluorouracil", "Germany", "Humans", "Liver Neoplasms", "Logistic Models", "Lung Neoplasms", "Middle Aged", "Mitoxantrone", "Neoplasm Staging", "Probability", "Proportional Hazards Models", "Quality of Life", "Risk Assessment", "Sensitivity and Specificity", "Statistics, Nonparametric", "Survival Analysis", "Treatment Outcome" ], "YEAR": "2002", "reasoning_required_pred": "no", "reasoning_free_pred": "yes", "final_decision": "yes", "LONG_ANSWER": "No significant difference was detected between the treatment with mitoxantrone as a single agent and the combination of low-dose FEC in terms of response or survival; therefore, the imperative of the necessity of first-line combination chemotherapy for patients with high-risk metastatic breast cancer may be questioned. Since toxicity and quality of life score favored the single-agent mitoxantrone treatment arm, this treatment may be offered to patients preferring quality of life to a potential small prolongation of survival." }, "10973547": { "QUESTION": "Are patients with Werlhof's disease at increased risk for bleeding complications when undergoing cardiac surgery?", "CONTEXTS": [ "It is generally assumed, that patients with Werlhof's disease (WD) are at increased risk for bleeding complications when undergoing cardiac surgery with extracorporeal circulation. Therefore we performed this case control study to estimate the real risk for bleeding complications of these patients.", "Between 05/95 and 07/98, ten patients with WD (eight males, two females) underwent cardiac surgery employing extracorporeal circulation (WD-group). Five of these patients with platelet counts below 80/nl were treated by immunoglobulins preoperatively. Each patient with WD was matched to five patients without WD (no-WD-group) using diagnosis, age, gender, ejection fraction, number of distal anastomosis and body-mass-index as matching criteria.", "Mean number of platelet counts were significant lower in the WD-group than in the no-WD-group despite a significant increase of platelet counts after immunoglobulin treatment (54/nl-->112/nl, P=0.018). On the day before, directly after and on the first day after surgery they were 141/nl vs. 215/nl (P=0.012), 75/nl vs. 147/nl (P=0.001) and 93/nl vs. 136/nl (P=0.009). Accordingly, patients of the WD-group received significantly more platelet concentrates than patients of the no-WD-group (mean number of platelet concentrates: 2.3 versus 0.7, P=0.007). Total drainage loss via the mediastinal chest tubes was almost identical (1197 ml in the no-WD-group and 1140 ml in the WD-group). One patient of each group suffered from a bleeding complication requiring reexploration. Three patients of the no-WD-group (6%) and one patient of the WD-group (10%) expired postoperatively unrelated to WD." ], "LABELS": [ "BACKGROUND", "METHODS", "RESULTS" ], "MESHES": [ "Adult", "Aged", "Blood Loss, Surgical", "Blood Transfusion", "Cardiopulmonary Bypass", "Coronary Artery Bypass", "Coronary Disease", "Female", "Humans", "Immunoglobulins", "Male", "Middle Aged", "Platelet Count", "Postoperative Hemorrhage", "Prognosis", "Purpura, Thrombocytopenic, Idiopathic", "Retrospective Studies", "Risk Factors", "Splenectomy" ], "YEAR": "2000", "reasoning_required_pred": "yes", "reasoning_free_pred": "no", "final_decision": "no", "LONG_ANSWER": "Patients with WD may possibly undergo cardiac surgery without a markedly enhanced risk for bleeding complications despite a more than usual transfusion requirement and significantly lower platelet counts perioperatively." }, "25499207": { "QUESTION": "Is neck pain associated with worse health-related quality of life 6 months later?", "CONTEXTS": [ "Current evidence suggests that neck pain is negatively associated with health-related quality of life (HRQoL). However, these studies are cross-sectional and do not inform the association between neck pain and future HRQoL.", "The purpose of this study was to investigate the association between increasing grades of neck pain severity and HRQoL 6 months later. In addition, this longitudinal study examines the crude association between the course of neck pain and HRQoL.", "This is a population-based cohort study.", "Eleven hundred randomly sampled Saskatchewan adults were included.", "Outcome measures were the mental component summary (MCS) and physical component summary (PCS) of the Short-Form-36 (SF-36) questionnaire.", "We formed a cohort of 1,100 randomly sampled Saskatchewan adults in September 1995. We used the Chronic Pain Questionnaire to measure neck pain and its related disability. The SF-36 questionnaire was used to measure physical and mental HRQoL 6 months later. Multivariable linear regression was used to measure the association between graded neck pain and HRQoL while controlling for confounding. Analysis of variance and t tests were used to measure the crude association among four possible courses of neck pain and HRQoL at 6 months. The neck pain trajectories over 6 months were no or mild neck pain, improving neck pain, worsening neck pain, and persistent neck pain. Finally, analysis of variance was used to examine changes in baseline to 6-month PCS and MCS scores among the four neck pain trajectory groups.", "The 6-month follow-up rate was 74.9%. We found an exposure-response relationship between neck pain and physical HRQoL after adjusting for age, education, arthritis, low back pain, and depressive symptomatology. Compared with participants without neck pain at baseline, those with mild (\u03b2=-1.53, 95% confidence interval [CI]=-2.83, -0.24), intense (\u03b2=-3.60, 95% CI=-5.76, -1.44), or disabling (\u03b2=-8.55, 95% CI=-11.68, -5.42) neck pain had worse physical HRQoL 6 months later. We did not find an association between neck pain and mental HRQoL. A worsening course of neck pain and persistent neck pain were associated with worse physical HRQoL." ], "LABELS": [ "BACKGROUND CONTEXT", "PURPOSE", "STUDY DESIGN", "PATIENT SAMPLE", "OUTCOME MEASURES", "METHODS", "RESULTS" ], "MESHES": [ "Adult", "Aged", "Canada", "Cohort Studies", "Cross-Sectional Studies", "Female", "Humans", "Longitudinal Studies", "Male", "Middle Aged", "Neck Pain", "Quality of Life", "Surveys and Questionnaires" ], "YEAR": "2015", "reasoning_required_pred": "yes", "reasoning_free_pred": "maybe", "final_decision": "yes", "LONG_ANSWER": "We found that neck pain was negatively associated with physical but not mental HRQoL. Our analysis suggests that neck pain may be a contributor of future poor physical HRQoL in the population. Raising awareness of the possible future impact of neck pain on physical HRQoL is important for health-care providers and policy makers with respect to the management of neck pain in populations." }, "26133538": { "QUESTION": "Does oral \u03b1-galactosidase relieve irritable bowel symptoms?", "CONTEXTS": [ "Abdominal bloating is reported by a majority of irritable bowel syndrome (IBS) patients. Excess colonic fermentation may cause gaseous symptoms. Several foodstuffs contain oligosaccharides with an \u03b1-galactosidic linkage that is resistant to mammalian hydrolases. Assisted hydrolysis by exogenous \u03b1-galactosidase enzyme (AG) could offer a way of controlling IBS symptoms by reducing colonic fermentation and gas production. The aim of this study was to assess the effect of AG on symptom severity and quality of life in IBS patients with abdominal bloating or flatulence.", "A total of 125 subjects with IBS received AG or placebo at meals for 12 weeks. IBS-Symptom Severity Score (IBS-SSS) and quality of life (QoL) were assessed at baseline, during the treatment and at 4-week follow-up.", "AG showed a trend toward a more prominent decrease in IBS-SSS. The responder rate at week 16 was higher for the AG group. No difference was detected in QoL between AG and placebo groups. A total of 25 patients (18 in AG group and 7 in placebo group, p = 0.016) withdrew from the study. Abdominal pain and diarrhea were more often reported as reason for withdrawal in AG group." ], "LABELS": [ "OBJECTIVE", "METHODS", "RESULTS" ], "MESHES": [ "Abdominal Pain", "Administration, Oral", "Adult", "Diarrhea", "Female", "Finland", "Flatulence", "Gastrointestinal Agents", "Humans", "Irritable Bowel Syndrome", "Male", "Middle Aged", "Quality of Life", "Severity of Illness Index", "Treatment Outcome", "alpha-Galactosidase" ], "YEAR": "2016", "reasoning_required_pred": "yes", "reasoning_free_pred": "no", "final_decision": "no", "LONG_ANSWER": "We found no evidence to support the use of AG routinely in IBS patients. Improvement of clinical response at 4-week follow-up may suggest a long-term effect of unknown mechanism, but could also be attributed to non-responder drop out. Gastrointestinal (GI) side effects may be a coincidence in this study, but irritation of GI tract by AG administration cannot be excluded." }, "26163474": { "QUESTION": "Is there a connection between sublingual varices and hypertension?", "CONTEXTS": [ "Sublingual varices have earlier been related to ageing, smoking and cardiovascular disease. The aim of this study was to investigate whether sublingual varices are related to presence of hypertension.", "In an observational clinical study among 431 dental patients tongue status and blood pressure were documented. Digital photographs of the lateral borders of the tongue for grading of sublingual varices were taken, and blood pressure was measured. Those patients without previous diagnosis of hypertension and with a noted blood pressure \u2265 140 mmHg and/or \u2265 90 mmHg at the dental clinic performed complementary home blood pressure during one week. Those with an average home blood pressure \u2265 135 mmHg and/or \u2265 85 mmHg were referred to the primary health care centre, where three office blood pressure measurements were taken with one week intervals. Two independent blinded observers studied the photographs of the tongues. Each photograph was graded as none/few (grade 0) or medium/severe (grade 1) presence of sublingual varices. Pearson's Chi-square test, Student's t-test, and multiple regression analysis were applied. Power calculation stipulated a study population of 323 patients.", "An association between sublingual varices and hypertension was found (OR = 2.25, p<0.002). Mean systolic blood pressure was 123 and 132 mmHg in patients with grade 0 and grade 1 sublingual varices, respectively (p<0.0001, CI 95 %). Mean diastolic blood pressure was 80 and 83 mmHg in patients with grade 0 and grade 1 sublingual varices, respectively (p<0.005, CI 95 %). Sublingual varices indicate hypertension with a positive predictive value of 0.5 and a negative predictive value of 0.80." ], "LABELS": [ "BACKGROUND", "METHODS", "RESULTS" ], "MESHES": [ "Adult", "Aged", "Angina Pectoris", "Atrial Fibrillation", "Blood Pressure", "Female", "Humans", "Hypertension", "Image Processing, Computer-Assisted", "Male", "Middle Aged", "Myocardial Infarction", "Myocardial Ischemia", "Photography", "Predictive Value of Tests", "Sensitivity and Specificity", "Smoking", "Stroke", "Tongue", "Varicose Veins" ], "YEAR": "2015", "reasoning_required_pred": "yes", "reasoning_free_pred": "yes", "final_decision": "yes", "LONG_ANSWER": "An association was found between sublingual varices and hypertension. Examining the lateral borders of the tongue is easily done, causes no harm and could be a valuable method for the dental profession to take active part in preventive healthcare." }, "14968373": { "QUESTION": "Can CT predict the level of CSF block in tuberculous hydrocephalus?", "CONTEXTS": [ "Treatment of obstructive hydrocephalus in children with tuberculous meningitis (TBM) depends on the level of the cerebrospinal fluid (CSF) block. Air-encephalography is regarded as the gold standard for differentiating communicating and non-communicating hydrocephalus. Since air-encephalography involves a lumbar puncture, it carries the risk of cerebral herniation. AIM. The aim of this study was to determine whether communicating and non-communicating hydrocephalus in TBM can be differentiated by means of cranial computerised tomography (CT).", "A number of CT indices were measured in 50 children with communicating and 34 children with non-communicating hydrocephalus according to air-encephalographic findings.", "The only CT finding that correlated with the type of hydrocephalus was the shape of the third ventricle. Significantly more children with non-communicating hydrocephalus had a rounded third ventricle than those with communicating hydrocephalus." ], "LABELS": [ "INTRODUCTION", "METHODS", "RESULTS" ], "MESHES": [ "Cerebrospinal Fluid", "Child, Preschool", "Diagnosis, Differential", "Female", "Humans", "Hydrocephalus", "Infant", "Male", "Pneumoencephalography", "Radiographic Image Interpretation, Computer-Assisted", "Retrospective Studies", "Sensitivity and Specificity", "Third Ventricle", "Tomography, X-Ray Computed", "Tuberculosis, Meningeal" ], "YEAR": "2004", "reasoning_required_pred": "yes", "reasoning_free_pred": "no", "final_decision": "yes", "LONG_ANSWER": "CT is therefore not useful in determining the level of CSF block in TBM. Air-encephalography remains the most reliable way of determining the level of CSF obstruction." }, "24698298": { "QUESTION": "MR arthrography of the shoulder: do we need local anesthesia?", "CONTEXTS": [ "To assess pain intensity with and without subcutaneous local anesthesia prior to intraarticular administration of contrast medium for magnetic resonance arthrography (MRa) of the shoulder.", "This single-center study was conducted after an IRB waiver of authorization, between January 2010 and December 2012. All patients provided written, informed consent for the procedure. Our prospectively populated institutional database was searched, based on our inclusion criteria. There were 249 outpatients (178 men and 71 women; mean age, 44.4 years \u00b1 14.6; range, 15-79) who underwent MRa and were enrolled in this study. Patients were excluded if they had received surgery of the shoulder before MRa, had undergone repeated MRa of the same shoulder, and/or had undergone MRa of both shoulders on the same day. Patients were randomly assigned into one of three groups. Patients in group A (n=61) received skin infiltration with local anesthesia. Patients in control group B (n=92) and group C (n=96) did not receive local anesthesia. Pain levels were immediately assessed after the injection for MRa using a horizontal visual analog scale (VAS) that ranged from 0 to 10. To compare the pain scores of the three groups for male and female patients, a two-way analysis of variance was used. A p-value equal to or less than 0.05 was considered to indicate a significant result.", "Patients who received local anesthesia (group A) showed a mean pain level on the VAS of 2.6 \u00b1 2.3. In patients who did not receive local anesthetics (groups B and C), a mean pain level on the VAS of 2.6 \u00b1 2.2 and 2.7 \u00b1 2.4 were detected, respectively. Between the three groups, no statistically significant difference in pain intensity was detected (p=.960). There were significant differences in subjective pain perception between men and women (p=.009). Moreover, the sex difference in all three groups was equal (p=.934)." ], "LABELS": [ "PURPOSE", "MATERIALS AND METHODS", "RESULTS" ], "MESHES": [ "Adolescent", "Adult", "Aged", "Anesthetics, Local", "Contrast Media", "Female", "Humans", "Injections, Intra-Articular", "Lidocaine", "Magnetic Resonance Imaging", "Male", "Meglumine", "Middle Aged", "Organometallic Compounds", "Pain Measurement", "Shoulder Pain", "Treatment Outcome", "Young Adult" ], "YEAR": "2014", "reasoning_required_pred": "no", "reasoning_free_pred": "no", "final_decision": "no", "LONG_ANSWER": "Local anesthesia is not required to lower a patient's pain intensity when applying intra-articular contrast media for MR arthrography of the shoulder. This could result in reduced costs and a reduced risk of adverse reactions, without an impact on patient comfort." }, "23088164": { "QUESTION": "Does cognitive function predict frequency compressed speech recognition in listeners with normal hearing and normal cognition?", "CONTEXTS": [ "The aim was to investigate the relationship between cognitive ability and frequency compressed speech recognition in listeners with normal hearing and normal cognition.", "Speech-in-noise recognition was measured using Institute of Electrical and Electronic Engineers sentences presented over earphones at 65 dB SPL and a range of signal-to-noise ratios. There were three conditions: unprocessed, and at frequency compression ratios of 2:1 and 3:1 (cut-off frequency, 1.6 kHz). Working memory and cognitive ability were measured using the reading span test and the trail making test, respectively.", "Participants were 15 young normally-hearing adults with normal cognition.", "There was a statistically significant reduction in mean speech recognition from around 80% when unprocessed to 40% for 2:1 compression and 30% for 3:1 compression. There was a statistically significant relationship between speech recognition and cognition for the unprocessed condition but not for the frequency-compressed conditions." ], "LABELS": [ "OBJECTIVE", "DESIGN", "STUDY SAMPLE", "RESULTS" ], "MESHES": [ "Acoustic Stimulation", "Adult", "Analysis of Variance", "Audiometry, Speech", "Cognition", "Female", "Humans", "Male", "Memory", "Noise", "Perceptual Masking", "Recognition (Psychology)", "Signal Detection, Psychological", "Speech Perception", "Trail Making Test" ], "YEAR": "2013", "reasoning_required_pred": "no", "reasoning_free_pred": "no", "final_decision": "no", "LONG_ANSWER": "The relationship between cognitive functioning and recognition of frequency compressed speech-in-noise was not statistically significant. The findings may have been different if the participants had been provided with training and/or time to 'acclimatize' to the frequency-compressed conditions." }, "24481006": { "QUESTION": "Should cavitation in proximal surfaces be reported in cone beam computed tomography examination?", "CONTEXTS": [ "79 adjacent proximal surfaces without restorations in permanent teeth were examined. Patients suspected to have carious lesions after a visual clinical and a bitewing examination participated in a CBCT examination (Kodak 9000 3D, 5 \u00d7 3.7 cm field of view, voxel size 0.07 mm). Ethical approval and informed consent were obtained according to the Helsinki Declaration. Radiographic assessment recording lesions with or without cavitation was performed by two observers in bitewings and CBCT sections. Orthodontic separators were placed interdentally between two lesion-suspected surfaces. The separator was removed after 3 days and the surfaces recorded as cavitated (yes/no), i.e. validated clinically. Differences between the two radiographic modalities (sensitivity, specificity and overall accuracy) were estimated by analyzing the binary data in a generalized linear model.", "For both observers, sensitivity was significantly higher for CBCT than for bitewings (average difference 33%, p<0.001) while specificity was not significantly different between the methods (p = 0.19). The overall accuracy was also significantly higher for CBCT (p<0.001)." ], "LABELS": [ "MATERIALS AND METHODS", "RESULTS" ], "MESHES": [ "Adolescent", "Adult", "Bicuspid", "Cone-Beam Computed Tomography", "Dental Caries", "Female", "Humans", "Image Processing, Computer-Assisted", "Linear Models", "Male", "Middle Aged", "Molar", "Observer Variation", "Physical Examination", "Radiography, Bitewing", "Reproducibility of Results", "Sensitivity and Specificity", "Tooth Crown", "Tooth Discoloration", "Young Adult" ], "YEAR": "2014", "reasoning_required_pred": "yes", "reasoning_free_pred": "yes", "final_decision": "yes", "LONG_ANSWER": "CBCT was more accurate in detecting cavitation in proximal surfaces than bitewing radiographs; therefore a CBCT examination performed for other clinical applications should also be assessed for proximal surface cavities in teeth without restorations, and when detected, this pathology must be part of the dentist's report." }, "17032327": { "QUESTION": "Do supervised colorectal trainees differ from consultants in terms of quality of TME surgery?", "CONTEXTS": [ "The quality of surgical excision is held to be a major determinant of outcome following surgery for rectal cancer. Macroscopic examination of the excised mesorectum allows for reproducible assessment of the quality of surgery. We aimed to determine whether quality of excision undertaken by colorectal trainees under supervision was comparable with that performed by consultants, as measured using mesorectal grades.", "A total of 130 consecutive patients undergoing potentially curative resection for primary adenocarcinoma of the rectum in our centre from 2001 to 2003 were included in the study. The pathologists graded the excised mesorectum according to staged classification proposed by Quirke. The outcome (quality of mesorectal excision and secondary outcomes including local recurrence and overall recurrence) of operations performed by consultants was compared with that of trainees. Statistical significance was tested using Pearson chi(2) test.", "Eighty-nine operations were performed by consultants and 41 by senior colorectal trainees with consultant supervision. Forty-four patients (49%) had good mesorectum when operated by consultants in comparison with 17 (41.5%) by the trainees. There was no statistically significant difference (P = 0.717) between the two groups in terms of quality of mesorectum excised after potentially curative resection. Furthermore, there were seven local recurrences in patients operated by consultants (7.8%) when compared with four in the trainee group (9.5%) and once again there was no statistical significance between the two groups (P = 0.719)." ], "LABELS": [ "OBJECTIVE", "METHOD", "RESULTS" ], "MESHES": [ "Aged", "Clinical Competence", "Digestive System Surgical Procedures", "Female", "Humans", "Internship and Residency", "Male", "Neoplasm Recurrence, Local", "Quality of Health Care", "Rectal Neoplasms", "Rectum", "Retrospective Studies", "Treatment Outcome", "United Kingdom" ], "YEAR": "2006", "reasoning_required_pred": "no", "reasoning_free_pred": "no", "final_decision": "no", "LONG_ANSWER": "We conclude that the quality of rectal cancer excision, as defined by mesorectal grades, achieved by supervised colorectal trainees is comparable with that achieved by consultants." }, "22658587": { "QUESTION": "Affect-regulated exercise intensity: does training at an intensity that feels 'good' improve physical health?", "CONTEXTS": [ "Affect-regulated exercise to feel 'good' can be used to control exercise intensity amongst both active and sedentary individuals and should support exercise adherence. It is not known, however, whether affect-regulated exercise training can lead to physical health gains. The aim of this study was to examine if affect-regulated exercise to feel 'good' leads to improved fitness over the course of an 8-week training programme.", "A repeated measures design (pretest-posttest) with independent groups (training and control).", "20 sedentary females completed a submaximal graded exercise test and were then allocated to either a training group or control group. The training group completed two supervised sessions and one unsupervised session per week for 8 weeks. Exercise intensity was affect-regulated to feel 'good'. Following the 8 weeks of training, both groups completed a second submaximal graded exercise test.", "Repeated measures analyses of variance indicated a significant increase in the time to reach ventilatory threshold in the training group (318 \u00b1 23.7s) compared to control (248 \u00b1 16.9s). Overall compliance to training was high (>92%). Participants in the training group exercised at intensities that would be classified as being in the lower range of the recommended guidelines (\u2248 50% V\u02d9O(2) max) for cardiovascular health." ], "LABELS": [ "OBJECTIVES", "DESIGN", "METHODS", "RESULTS" ], "MESHES": [ "Adolescent", "Adult", "Affect", "Exercise", "Female", "Heart Rate", "Humans", "Middle Aged", "Oxygen Consumption", "Physical Fitness", "Sedentary Lifestyle", "Young Adult" ], "YEAR": "2012", "reasoning_required_pred": "yes", "reasoning_free_pred": "yes", "final_decision": "yes", "LONG_ANSWER": "Affect-regulated exercise to feel 'good' can be used in a training programme to regulate exercise intensity. This approach led to a 19% increase in time to reach ventilatory threshold, which is indicative of improved fitness." }, "8916748": { "QUESTION": "Do socioeconomic differences in mortality persist after retirement?", "CONTEXTS": [ "To assess the risk of death associated with work based and non-work based measures of socioeconomic status before and after retirement age.", "Follow up study of mortality in relation to employment grade and car ownership over 25 years.", "The first Whitehall study.", "18,133 male civil servants aged 40-69 years who attended a screening examination between 1967 and 1970.", "Death.", "Grade of employment was a strong predictor of mortality before retirement. For men dying at ages 40-64 the lowest employment grade had 3.12 times the mortality of the highest grade (95% confidence interval 2.4 to 4.1). After retirement the ability of grade to predict mortality declined (rate ratio 1.86; 1.6 to 2.2). A non-work based measure of socioeconomic status (car ownership) predicted mortality less well than employment grade before retirement but its ability to predict mortality declined less after retirement. Using a relative index of inequality that was sensitive to the distribution among socioeconomic groups showed employment grade and car ownership to have independent associations with mortality that were of equal magnitude after retirement. The absolute difference in death rates between the lowest and highest employment grades increased with age from 12.9 per 1000 person years at ages 40-64 to 38.3 per 1000 at ages 70-89." ], "LABELS": [ "OBJECTIVE", "DESIGN", "SETTING", "SUBJECTS", "MAIN OUTCOME MEASURE", "RESULTS" ], "MESHES": [ "Adult", "Age Factors", "Aged", "Cause of Death", "England", "Follow-Up Studies", "Government", "Humans", "Male", "Middle Aged", "Mortality", "Retirement", "Social Class", "Socioeconomic Factors", "Survival Rate" ], "YEAR": "1996", "reasoning_required_pred": "yes", "reasoning_free_pred": "yes", "final_decision": "yes", "LONG_ANSWER": "Socioeconomic differences in mortality persist beyond retirement age and in magnitude increase with age. Social differentials in mortality based on an occupational status measure seem to decrease to a greater degree after retirement than those based on a non-work measure. This suggests that alongside other socioeconomic factors work itself may play an important part in generating social inequalities in health in men of working age." }, "16361634": { "QUESTION": "Women with synchronous primary cancers of the endometrium and ovary: do they have Lynch syndrome?", "CONTEXTS": [ "Lynch syndrome (hereditary nonpolyposis colorectal cancer; HNPCC) is an autosomal-dominant cancer predisposition syndrome that increases risk for multiple cancers, including colon, endometrial, and ovarian cancer. Revised Bethesda Criteria recommend that patients with two HNPCC-associated cancers undergo molecular evaluation to determine whether they have a mismatch repair (MMR) defect associated with HNPCC. The purpose of our study was to determine the likelihood of MMR defects (MSH2, MSH6, MLH1) in women with synchronous endometrial and ovarian cancer.", "Between 1989 and 2004, 102 women with synchronous endometrial and ovarian cancers were identified; 59 patients had tumor blocks available for analysis. Patients were divided into risk groups based on family history: high (met Amsterdam criteria), medium (personal history or first-degree relative with an HNPCC-associated cancer), and low (all others). Protein expression for MSH2, MSH6, and MLH1 was evaluated by immunohistochemistry. Microsatellite instability and MLH1 promoter methylation analyses were performed on a subset of cases.", "Median age was 50 years. Two patients met Amsterdam criteria for HNPCC. Five additional patients, all medium-risk, had molecular findings consistent with a germline mutation of either MSH2 or MLH1. None of the low-risk patients had molecular results consistent with a germline mutation." ], "LABELS": [ "PURPOSE", "PATIENTS AND METHODS", "RESULTS" ], "MESHES": [ "Adult", "Aged", "Aged, 80 and over", "Base Pair Mismatch", "Cohort Studies", "Colorectal Neoplasms, Hereditary Nonpolyposis", "DNA Methylation", "DNA Repair", "Endometrial Neoplasms", "Female", "Gene Expression Profiling", "Humans", "Immunohistochemistry", "Microsatellite Repeats", "Middle Aged", "Neoplasms, Multiple Primary", "Ovarian Neoplasms", "Pedigree", "Risk Factors" ], "YEAR": "2005", "reasoning_required_pred": "yes", "reasoning_free_pred": "maybe", "final_decision": "maybe", "LONG_ANSWER": "Overall, 7% of women in our cohort met either clinical or molecular criteria for Lynch syndrome. All of these women had a prior history or a first-degree relative with an HNPCC-associated cancer. Limiting genetic evaluation to women with synchronous endometrial and ovarian cancer who have a family history suggestive of HNPCC may appropriately identify women with Lynch syndrome." }, "25756710": { "QUESTION": "Can emergency physicians accurately and reliably assess acute vertigo in the emergency department?", "CONTEXTS": [ "To validate a clinical diagnostic tool, used by emergency physicians (EPs), to diagnose the central cause of patients presenting with vertigo, and to determine interrater reliability of this tool.", "A convenience sample of adult patients presenting to a single academic ED with isolated vertigo (i.e. vertigo without other neurological deficits) was prospectively evaluated with STANDING (SponTAneousNystagmus, Direction, head Impulse test, standiNG) by five trained EPs. The first step focused on the presence of spontaneous nystagmus, the second on the direction of nystagmus, the third on head impulse test and the fourth on gait. The local standard practice, senior audiologist evaluation corroborated by neuroimaging when deemed appropriate, was considered the reference standard. Sensitivity and specificity of STANDING were calculated. On the first 30 patients, inter-observer agreement among EPs was also assessed.", "Five EPs with limited experience in nystagmus assessment volunteered to participate in the present study enrolling 98 patients. Their average evaluation time was 9.9 \u00b1 2.8\u2009min (range 6-17). Central acute vertigo was suspected in 16 (16.3%) patients. There were 13 true positives, three false positives, 81 true negatives and one false negative, with a high sensitivity (92.9%, 95% CI 70-100%) and specificity (96.4%, 95% CI 93-38%) for central acute vertigo according to senior audiologist evaluation. The Cohen's kappas of the first, second, third and fourth steps of the STANDING were 0.86, 0.93, 0.73 and 0.78, respectively. The whole test showed a good inter-observer agreement (k = 0.76, 95% CI 0.45-1)." ], "LABELS": [ "OBJECTIVE", "METHODS", "RESULTS" ], "MESHES": [ "Acute Disease", "Algorithms", "Clinical Competence", "Decision Support Systems, Clinical", "Emergency Medicine", "Emergency Service, Hospital", "Humans", "Prospective Studies", "Vertigo" ], "YEAR": "2015", "reasoning_required_pred": "yes", "reasoning_free_pred": "yes", "final_decision": "yes", "LONG_ANSWER": "In the hands of EPs, STANDING showed a good inter-observer agreement and accuracy validated against the local standard of care." }, "23916653": { "QUESTION": "Orthostatic myoclonus: an underrecognized cause of unsteadiness?", "CONTEXTS": [ "Recently, orthostatic myoclonus (OM) has been suggested as a cause of gait impairment and unsteadiness in neurodegenerative diseases. The aim of this study was to investigate the frequency of orthostatic myoclonus, its clinical characteristics and the underlying associated neurological disorders.", "A retrospective analysis of clinical data and electromyogram surface recordings from subjects with unexplained unsteadiness/gait impairment was performed. Diagnosis of OM was made when a pattern of non-rhythmic bursts was observed (duration range 20-100 ms; bursts per second \u226416).", "Among 93 subjects studied, OM was the most frequent disorder (n = 16; 17.2%), followed by orthostatic tremor (13.9%) and low frequency tremors during orthostatism (12.9%). All patients with OM complained about unsteadiness during orthostatism and/or during gait. Leg jerking was only observed by visual inspection during orthostatism in four subjects and two also presented falls. Eleven out of 16 patients (68.7%) with OM had an associated neurodegenerative disease, such as multiple system atrophy (n = 3) Parkinson's disease (n = 2), Alzheimer's disease (n = 2), mild cognitive impairment (n = 2) and normal pressure hydrocephalus (n = 2). Although four subjects showed improvement of orthostatic myoclonus with antimyoclonic treatment, the follow-up was not systematic enough to evaluate their therapeutic effect on OM." ], "LABELS": [ "BACKGROUND AND PURPOSE", "METHODS", "RESULTS" ], "MESHES": [ "Aged", "Aged, 80 and over", "Dizziness", "Electromyography", "Female", "Follow-Up Studies", "Gait Disorders, Neurologic", "Humans", "Male", "Middle Aged", "Myoclonus", "Neurodegenerative Diseases", "Retrospective Studies" ], "YEAR": "2013", "reasoning_required_pred": "yes", "reasoning_free_pred": "yes", "final_decision": "yes", "LONG_ANSWER": "Orthostatic myoclonus is often underdiagnosed and can be considered a possible cause of unsteadiness in subjects with neurodegenerative diseases. Electromyography surface recording is thereby an aid for investigating unsteadiness of unknown origin." }, "12790890": { "QUESTION": "Is the cell death in mesial temporal sclerosis apoptotic?", "CONTEXTS": [ "Mesial temporal sclerosis (MTS) is characterized by neuronal loss in the hippocampus. Studies on experimental models and patients with intractable epilepsy suggest that apoptosis may be involved in neuronal death induced by recurrent seizures.", "We searched evidence for apoptotic cell death in temporal lobes resected from drug-resistant epilepsy patients with MTS by using the terminal deoxynucleotidyl transferase (TdT) and digoxigenin-11-dUTP (TUNEL) method and immunohistochemistry for Bcl-2, Bax, and caspase-cleaved actin fragment, fractin. The temporal lobe specimens were obtained from 15 patients (six women and nine men; mean age, 29 +/- 8 years).", "Unlike that in normal adult brain, we observed Bcl-2 immunoreactivity in some of the remaining neurons dispersed throughout the hippocampus proper as well as in most of the reactive astroglia. Bax immunopositivity was increased in almost all neurons. Fractin immunostaining, an indicator of caspase activity, was detected in approximately 10% of these neurons. Despite increased Bax expression and activation of caspases, we could not find evidence for DNA fragmentation by TUNEL staining. We also could not detect typical apoptotic changes in nuclear morphology by Hoechst-33258 or hematoxylin counterstaining." ], "LABELS": [ "PURPOSE", "METHODS", "RESULTS" ], "MESHES": [ "Actin Cytoskeleton", "Adult", "Age of Onset", "Apoptosis", "Astrocytes", "Cell Count", "Cell Death", "DNA Fragmentation", "Epilepsy, Temporal Lobe", "Female", "Hippocampus", "Humans", "Immunohistochemistry", "In Situ Nick-End Labeling", "Male", "Neurons", "Sclerosis", "Temporal Lobe" ], "YEAR": "2003", "reasoning_required_pred": "maybe", "reasoning_free_pred": "no", "final_decision": "maybe", "LONG_ANSWER": "These data suggest that either apoptosis is not involved in cell loss in MTS, or a very slow rate of cell demise may have precluded detecting TUNEL-positive neurons dying through apoptosis. Increased Bax expression and activation of caspases support the latter possibility." }, "23449952": { "QUESTION": "Contrast-enhanced MR imaging of hand and finger joints in patients with early rheumatoid arthritis: do we really need a full dose of gadobenate dimeglumine for assessing synovial enhancement at 3 T?", "CONTEXTS": [ "To investigate the diagnostic value of a half dose compared with a full dose of gadobenate dimeglumine in the assessment of synovitis or tenosynovitis in the wrist and finger joints in patients with early rheumatoid arthritis (RA) and a disease activity score greater than 3.2.", "With institutional review board approval and informed consent, 57 patients with early RA underwent 3-T magnetic resonance (MR) imaging with two different doses of contrast media. The contrast enhancement was measured in inflamed synovial tissue at half dose (0.05 mmol per kilogram of body weight) and at full dose (0.1 mmol/kg) by using T1-weighted sequences with fat saturation. The differences and the correlation of signal intensities (SIs) at half- and full-dose sequences were compared by using the paired t test and Pearson correlations. Image quality, Rheumatoid Arthritis MRI Score (RAMRIS), and tenosynovitis score on half- and full-dose images were compared by two observers using the Wilcoxon test. Interrater agreement was assessed by using \u03ba statistics.", "A significant difference in SI was found between half-dose and full-dose gadobenate dimeglumine-enhanced synovial tissue (mean: 914.35 \u00b1 251.1 vs 1022 \u00b1 244.5, P<.001). Because the SI showed high correlation between the ratio at half dose and full dose (r = 0.875), the formula, ratio of synovial enhancement to saline syringe at full dose = 0.337 + 1.070 \u00d7 ratio of synovial enhancement to saline syringe at half dose, can be used to convert the normalized value of half dose to full dose. However, no difference in RAMRIS (score 0 in 490 of 1026 joints; score 1 in 344; score 2 in 158; and score 3 in 34) or tenosynovitis scores in grading synovitis or tenosynovitis in image quality and in assessment of synovial enhancement was detected between half-dose and full-dose images (P = 1)." ], "LABELS": [ "PURPOSE", "MATERIALS AND METHODS", "RESULTS" ], "MESHES": [ "Adult", "Aged", "Aged, 80 and over", "Arthritis, Rheumatoid", "Contrast Media", "Female", "Finger Joint", "Hand", "Humans", "Image Interpretation, Computer-Assisted", "Linear Models", "Magnetic Resonance Imaging", "Male", "Meglumine", "Middle Aged", "Organometallic Compounds", "Prospective Studies", "Synovitis", "Tenosynovitis", "Wrist Joint" ], "YEAR": "2013", "reasoning_required_pred": "no", "reasoning_free_pred": "no", "final_decision": "no", "LONG_ANSWER": "Postcontrast synovial SIs showed high correlation between half dose and full dose, and image quality was rated identically. Therefore, half-dose gadobenate dimeglumine at 3-T MR imaging may be sufficient for assessing synovitis or tenosynovitis in early RA." }, "10430303": { "QUESTION": "Does laparoscopic cholecystectomy influence peri-sinusoidal cell activity?", "CONTEXTS": [ "To investigate the influence of laparoscopic procedures on perisinusoidal cell function.", "In 31 patients who underwent laparoscopic cholecystectomy for symptomatic cholelithiasis, the serum levels of beta-N-acetyl hexosaminidase (beta-NAH) and hyaluronic acid (HA) were measured. Six female patients, who underwent other laparoscopic procedures, not involving the hepatobiliary system, served as controls.", "HA serum levels increased significantly in both study and control groups, while beta-NAH serum levels remained within normal values. Post-operative AST and ALT serum levels increased significantly only in the study group. No elevation of serum ALP was noted in any of our patients, and post-operative bilirubin levels did not increase in patients with normal pre-operative levels." ], "LABELS": [ "AIMS", "METHODOLOGY", "RESULTS" ], "MESHES": [ "Adult", "Aged", "Cholecystectomy, Laparoscopic", "Cholelithiasis", "Endothelium, Vascular", "Female", "Humans", "Hyaluronic Acid", "Kupffer Cells", "Liver Function Tests", "Middle Aged", "Postoperative Complications", "beta-N-Acetylhexosaminidases" ], "YEAR": null, "reasoning_required_pred": "no", "reasoning_free_pred": "yes", "final_decision": "yes", "LONG_ANSWER": "Laparoscopic procedures caused detectable damage to Kupffer and endothelial cells as reflected by elevation of post-operative HA serum levels. The damage to the liver hepatocytes and perisinusoidal cells has no clinical significance and the laparoscopic procedure itself is the probable cause of the Kupffer and endothelial cells damage, while other mechanisms caused damage to the hepatocytes." }, "20605051": { "QUESTION": "Does case-mix based reimbursement stimulate the development of process-oriented care delivery?", "CONTEXTS": [ "Reimbursement based on the total care of a patient during an acute episode of illness is believed to stimulate management and clinicians to reduce quality problems like waiting times and poor coordination of care delivery. Although many studies already show that this kind of case-mix based reimbursement leads to more efficiency, it remains unclear whether care coordination improved as well. This study aims to explore whether case-mix based reimbursement stimulates development of care coordination by the use of care programmes, and a process-oriented way of working.", "Data for this study were gathered during the winter of 2007/2008 in a survey involving all Dutch hospitals. Descriptive and structural equation modelling (SEM) analyses were conducted.", "SEM reveals that adoption of the case-mix reimbursement within hospitals' budgeting processes stimulates hospitals to establish care programmes by the use of process-oriented performance measures. However, the implementation of care programmes is not (yet) accompanied by a change in focus from function (the delivery of independent care activities) to process (the delivery of care activities as being connected to a chain of interdependent care activities)." ], "LABELS": [ "OBJECTIVES", "METHODS", "RESULTS" ], "MESHES": [ "Budgets", "Delivery of Health Care", "Diagnosis-Related Groups", "Economics, Hospital", "Health Care Surveys", "Humans", "Models, Theoretical", "Netherlands", "Patient Care Management", "Quality of Health Care", "Reimbursement Mechanisms" ], "YEAR": "2010", "reasoning_required_pred": "yes", "reasoning_free_pred": "yes", "final_decision": "yes", "LONG_ANSWER": "This study demonstrates that hospital management can stimulate the development of care programmes by the adoption of case-mix reimbursement within hospitals' budgeting processes. Future research is recommended to confirm this finding and to determine whether the establishment of care programmes will in time indeed lead to a more process-oriented view of professionals." }, "23048048": { "QUESTION": "Risky sexual behavior among college students With ADHD: is the mother-child relationship protective?", "CONTEXTS": [ "This study examined the extent to which ADHD was associated with risky sexual behaviors (RSBs) in a sample of 92 undergraduates with (n = 44) and without (n = 48) ADHD. Mother-child relationship quality was examined as a potential moderator.", "We conducted comprehensive assessments for ADHD and comorbid conditions and collected measures of RSB and mother-child relationship quality.", "Female students with ADHD were least likely to use condoms than males overall and females without ADHD. An interaction between ADHD and mother-child relationship quality accounted for significant variance in the number of past-year sexual partners, such that a high-quality relationship was protective only for students with ADHD. No other significant associations were found between ADHD and RSB." ], "LABELS": [ "OBJECTIVE", "METHOD", "RESULTS" ], "MESHES": [ "Adolescent", "Attention Deficit Disorder with Hyperactivity", "Condoms", "Female", "Humans", "Interviews as Topic", "Male", "Mother-Child Relations", "Risk", "Risk-Taking", "Sex Factors", "Sexual Behavior", "Students", "Surveys and Questionnaires", "United States", "Young Adult" ], "YEAR": "2015", "reasoning_required_pred": "yes", "reasoning_free_pred": "yes", "final_decision": "yes", "LONG_ANSWER": "Results suggest that female college students with ADHD are at risk for unprotected sex. Moreover, a positive mother-child relationship may be protective for college students with ADHD in relation to RSB." }, "16991071": { "QUESTION": "Rotator cuff tear--an occupational disease?", "CONTEXTS": [ "In literature there are only few data which describe the influence of occupation on the development of rotator cuff disease.", "In a retrospective study, 760 open rotator cuff repairs were analysed and related to the profession and occupational load. Exclusion criteria were traumatic tears and sports injuries. All male persons were statistically analysed and the data compared with occupational patterns of the region, obtained from the Federal Statistical State Office.", "Rotator cuff repairs were performed in 472 males who had no evidence for a traumatic origin. After statistical analysis (p<0.001) we found significantly more patients working in agriculture and forestry (6.38% versus 1.07% in Bavaria) and in the building industry (35.11% versus 13.40% in Bavaria)." ], "LABELS": [ "BACKGROUND", "METHODS", "RESULTS" ], "MESHES": [ "Adult", "Aged", "Comorbidity", "Employment", "Female", "Germany", "Humans", "Male", "Middle Aged", "Occupational Diseases", "Prevalence", "Risk Assessment", "Risk Factors", "Rotator Cuff Injuries", "Rupture", "Shoulder Impingement Syndrome", "Workload" ], "YEAR": null, "reasoning_required_pred": "yes", "reasoning_free_pred": "yes", "final_decision": "yes", "LONG_ANSWER": "Our data suggest that working exposure increases the risk or leads to the clinical manifestation of rotator cuff tears. Although a detailed analysis of individual physical exposure is not available yet, the statistical results indicate that rotator cuff tears must be taken into consideration as a result of ergonomic exposure." }, "12090319": { "QUESTION": "Is there a need for pelvic CT scan in cases of renal cell carcinoma?", "CONTEXTS": [ "To determine the necessity of pelvic computed tomography (CT) in patients of renal cell carcinoma (RCC).", "We reviewed the records of 400 patients of RCC, who underwent treatment at our institution between January 1988 and February 2001. These patients were evaluated pre-operatively with ultrasonograms (USG) and contrast enhanced CT scan of the abdomen and pelvis. USG or CT scans of these cases were reviewed for presence of pathology in the pelvis, which were classified into 3 categories viz; benign and likely to be insignificant, benign and likely to be significant; and malignant.", "Of the 400 cases, 114 were stage I, 68 were stage II, 99 were stage III and 119 were stage IV. In all patients, tumour was identified in the kidney on preoperative CT scan. Fourteen patients (3.5%) had an abnormality on pelvic CT. Five (1.25%) had category 1, three (0.75%) had category 2 and six (1.5%) had category 3 abnormality on pelvic CT. However, all these abnormalities in pelvis were detected prior to CT by other investigations (USG or plain x-ray). Of the six cases with malignant findings, two had superficial bladder cancer, one had RCC in a pelvic kidney and three had bone metastases in the pelvis." ], "LABELS": [ "OBJECTIVES", "MATERIALS AND METHODS", "RESULTS" ], "MESHES": [ "Adult", "Aged", "Biopsy, Needle", "Carcinoma, Renal Cell", "Female", "Follow-Up Studies", "Humans", "India", "Kidney Neoplasms", "Male", "Middle Aged", "Needs Assessment", "Neoplasm Staging", "Nephrectomy", "Pelvis", "Preoperative Care", "Registries", "Retrospective Studies", "Tomography, X-Ray Computed", "Unnecessary Procedures" ], "YEAR": "2002", "reasoning_required_pred": "no", "reasoning_free_pred": "no", "final_decision": "no", "LONG_ANSWER": "Pelvic CT does not offer additional information in the vast majority of cases with RCC and should be performed selectively. Thus the cost of diagnostic imaging in RCC can be reduced." }, "24614851": { "QUESTION": "Prognostic factors for cervical spondylotic amyotrophy: are signs of spinal cord involvement associated with the neurological prognosis?", "CONTEXTS": [ "The purpose of this study was to clarify the prognostic factors for cervical spondylotic amyotrophy (CSA).", "The authors retrospectively reviewed the medical records of 47 consecutive patients with CSA in whom the presence/absence of the pyramidal tract sign was noted. We analyzed whether the age, sex, presence of diabetes mellitus, medication (vitamin B12), type of the most atrophic and impaired muscle, the muscle strength at the presentation, the presence of the pyramidal tract sign, magnetic resonance imaging (MRI) findings, including the presence and number of T2 high signal intensity areas (T2 HIA) in the spinal cord and the conversion to surgery were associated with the recovery of muscle strength in the patients. In addition, we also investigated whether the duration of symptoms before surgery and the type of surgery were associated with the recovery of muscle strength in patients who required conversion to surgical treatment.", "The presence of T2 HIA on MRI (P=0.002), the number of T2 HIA on MRI (P=0.002) and conversion to surgery (P=0.015) were found to be significantly associated with a poorer recovery at the observational final follow-up. Further, the presence of the pyramidal tract sign (P=0.043) was significantly associated with a poor recovery at the final follow-up after surgery." ], "LABELS": [ "OBJECTIVES", "METHODS", "RESULTS" ], "MESHES": [ "Adult", "Aged", "Aged, 80 and over", "Diabetes Mellitus", "Female", "Follow-Up Studies", "Humans", "Magnetic Resonance Imaging", "Male", "Middle Aged", "Muscle Strength", "Muscle, Skeletal", "Nervous System Diseases", "Pyramidal Tracts", "Retrospective Studies", "Spinal Cord Injuries", "Spondylosis", "Statistics, Nonparametric" ], "YEAR": "2014", "reasoning_required_pred": "yes", "reasoning_free_pred": "yes", "final_decision": "yes", "LONG_ANSWER": "The presence of a high signal intensity change on T2-weighted MRI and the pyramidal tract sign can be used as prognostic factors for patients with CSA." }, "17578985": { "QUESTION": "Parasacral sciatic nerve block: does the elicited motor response predict the success rate?", "CONTEXTS": [ "In this prospective, randomized, double-blind study, we compared the tibial and the peroneal evoked motor response with regard to efficacy of sciatic nerve block using the parasacral approach.", "Twenty-six ASA I-III patients scheduled for elective lower limb surgery were randomized to receive a parasacral sciatic block, using a nerve stimulator technique seeking either a tibial (n = 14) or peroneal (n = 12) motor response. After the evoked motor response was obtained, a solution of 10 mL 2% lidocaine with epinephrine and 10 mL 0.75% ropivacaine (actual final concentration of epinephrine, 1/160,000) was slowly injected through the needle. Sensory and motor blocks were assessed every 5 min for 30 min by an anesthesiologist blinded to the elicited motor response. If the block was not complete 30 min after injection of the local anesthetics, it was considered as failed, and general anesthesia was supplemented.", "Time to perform the block and level of minimal and maximal stimulation were not different between groups. The success rate of complete block was significantly higher in the tibial compared to the peroneal group (11 of 14 vs 2 of 12; P = 0.002)." ], "LABELS": [ "BACKGROUND", "METHODS", "RESULTS" ], "MESHES": [ "Adult", "Aged", "Anesthetics, Local", "Double-Blind Method", "Evoked Potentials, Motor", "Female", "Humans", "Male", "Middle Aged", "Nerve Block", "Predictive Value of Tests", "Prospective Studies", "Sacrum", "Sciatic Nerve" ], "YEAR": "2007", "reasoning_required_pred": "maybe", "reasoning_free_pred": "yes", "final_decision": "maybe", "LONG_ANSWER": "Eliciting a tibial motor response predicts a higher success rate than eliciting a peroneal motor response with parasacral sciatic nerve block." }, "26965932": { "QUESTION": "Is Bare-Metal Stent Implantation Still Justifiable in High Bleeding Risk Patients Undergoing Percutaneous Coronary Intervention?", "CONTEXTS": [ "This study sought to investigate the ischemic and bleeding outcomes of patients fulfilling high bleeding risk (HBR) criteria who were randomized to zotarolimus-eluting Endeavor Sprint stent (E-ZES) or bare-metal stent (BMS) implantation followed by an abbreviated dual antiplatelet therapy (DAPT) duration for stable or unstable coronary artery disease.", "DES instead of BMS use remains controversial in HBR patients, in whom long-term DAPT poses safety concerns.", "The ZEUS (Zotarolimus-Eluting Endeavor Sprint Stent in Uncertain DES Candidates) is a multinational, randomized single-blinded trial that randomized among others, in a stratified manner, 828 patients fulfilling pre-defined clinical or biochemical HBR criteria-including advanced age, indication to oral anticoagulants or other pro-hemorrhagic medications, history of bleeding and known anemia-to receive E-ZES or BMS followed by a protocol-mandated 30-day DAPT regimen. The primary endpoint of the study was the 12-month major adverse cardiovascular event rate, consisting of death, myocardial infarction, or target vessel revascularization.", "Compared with patients without, those with 1 or more HBR criteria had worse outcomes, owing to higher ischemic and bleeding risks. Among HBR patients, major adverse cardiovascular events occurred in 22.6% of the E-ZES and 29% of the BMS patients (hazard ratio: 0.75; 95% confidence interval: 0.57 to 0.98; p = 0.033), driven by lower myocardial infarction (3.5% vs. 10.4%; p<0.001) and target vessel revascularization (5.9% vs. 11.4%; p = 0.005) rates in the E-ZES arm. The composite of definite or probable stent thrombosis was significantly reduced in E-ZES recipients, whereas bleeding events did not differ between stent groups." ], "LABELS": [ "OBJECTIVES", "BACKGROUND", "METHODS", "RESULTS" ], "MESHES": [ "Aged", "Aged, 80 and over", "Cardiovascular Agents", "Coronary Artery Disease", "Drug Therapy, Combination", "Drug-Eluting Stents", "Female", "Hemorrhage", "Humans", "Male", "Metals", "Myocardial Infarction", "Patient Selection", "Percutaneous Coronary Intervention", "Platelet Aggregation Inhibitors", "Prosthesis Design", "Risk Assessment", "Risk Factors", "Single-Blind Method", "Sirolimus", "Stents", "Time Factors", "Treatment Outcome" ], "YEAR": "2016", "reasoning_required_pred": "yes", "reasoning_free_pred": "no", "final_decision": "yes", "LONG_ANSWER": "Among HBR patients with stable or unstable coronary artery disease, E-ZES implantation provides superior efficacy and safety as compared with conventional BMS. (Zotarolimus-Eluting Endeavor Sprint Stent in Uncertain DES Candidates [ZEUS]; NCT01385319)." }, "15137012": { "QUESTION": "Ovarian torsion in children: is oophorectomy necessary?", "CONTEXTS": [ "Most pediatric surgeons perform oophorectomy in girls presenting with ovarian torsion in which the ovary appears necrotic. However, the adult gynecology literature suggests that many ovaries can be treated by detorsion alone.", "All children with ovarian torsion on the gynecology and general surgery services between 1988 and 2002 were reviewed.", "There were 36 torsions in 34 children. Seventeen underwent detorsion with or without ovarian cystectomy, and 19 had oophorectomy (mean age 10 years in both groups). Torsion was suspected preoperatively in 94% of the detorsion cases and in 47% of the oophorectomy patients. Median time from presentation to surgery was significantly lower in the detorsion than the oophorectomy group (median 14 v 27 hours; P =.04). Postoperative complications and length of stay were similar between the 2 groups. Despite the ovary being judged intraoperatively as moderately to severely ischemic in 53% of the detorsion cases, follow-up sonogram or ovarian biopsy available in 14 of the 17 cases showed normal ovary with follicular development in each case." ], "LABELS": [ "BACKGROUND", "METHODS", "RESULTS" ], "MESHES": [ "Adolescent", "Child", "Child, Preschool", "Fallopian Tubes", "Female", "Follow-Up Studies", "Humans", "Ovarian Diseases", "Ovariectomy", "Torsion Abnormality", "Treatment Outcome" ], "YEAR": "2004", "reasoning_required_pred": "no", "reasoning_free_pred": "no", "final_decision": "no", "LONG_ANSWER": "Simple detorsion was not accompanied by an increase in morbidity, and all patients studied had functioning ovarian tissue on follow-up despite the surgeon's assessment of the degree of ovarian ischemia. Detorsion is the procedure of choice for most cases of ovarian torsion in children." }, "21194998": { "QUESTION": "Does minimal access major surgery in the newborn hurt less?", "CONTEXTS": [ "Minimal access surgery (MAS) in adults is associated with less postoperative pain in comparison to conventional 'open' surgery. It is not known whether this holds true for neonates as well. Less pain would imply that opioid consumption can be reduced, which has a beneficial effect on morbidity.AIM: To evaluate potential differences in' opioid consumption between neonates undergoing thoracoscopic minimal access surgery or conventional surgery of esophageal atresia (EA) and congenital diaphragmatic hernia (CDH).", "In this retrospective cohort study we included two controls for each MAS patient, matched on diagnosis, sex and age at surgery. Opioid dose titration was based on validated pain scores (VAS and COMFORT behaviour), applied by protocol. Cumulative opioid doses at 12, 24, 48 h and 7 days postoperatively were compared between groups with the Mann-Whitney test.", "The study group consisted of 24 MAS patients (14 EA; 10 CDH). These were matched to 48 control patients (28 EA; 20 CDH). At none of the time points cumulative opioid (median in mg/kg (IQR)) doses significantly differed between MAS patients and controls, both with CDH and EA. For example at 24 h postoperative for CDH patients cumulative opioid doses were [0.84(0.61-1.83) MAS vs. 1.06(0.60-1.36) p=1.0] controls, For EApatients at 24 h the cumulative opioid doses were [0.48(0.30-0.75) MAS vs. 0.49(0.35-0.79) p=0.83] controls. This held true for the postoperative pain scores as well." ], "LABELS": [ "BACKGROUND", "METHODS", "RESULTS" ], "MESHES": [ "Analgesics, Opioid", "Esophageal Atresia", "Female", "Fentanyl", "Hernia, Diaphragmatic", "Hernias, Diaphragmatic, Congenital", "Humans", "Infant", "Infant, Newborn", "Male", "Minimally Invasive Surgical Procedures", "Morphine", "Pain Measurement", "Pain, Postoperative", "Retrospective Studies" ], "YEAR": "2011", "reasoning_required_pred": "yes", "reasoning_free_pred": "no", "final_decision": "no", "LONG_ANSWER": "Minimal access surgery for the repair of esophageal atresia or congenital diaphragmatic hernia is not associated with less cumulative opioid doses." }, "15787677": { "QUESTION": "Does aerobic fitness influence microvascular function in healthy adults at risk of developing Type 2 diabetes?", "CONTEXTS": [ "Twenty-seven healthy normal glucose-tolerant humans with either a previous diagnosis of gestational diabetes or having two parents with Type 2 diabetes and 27 healthy adults who had no history of diabetes were recruited. Maximal oxygen uptake was assessed using an incremental exercise test to exhaustion. Skin microvascular function was assessed using laser Doppler techniques as the maximum skin hyperaemic response to a thermal stimulus (maximum hyperaemia) and the forearm skin blood flow response to the iontophoretic application of acetylcholine (ACh) and sodium nitroprusside.", "Maximal oxygen uptake was not significantly different in the 'at-risk' group compared with healthy controls. Maximum hyperaemia was reduced in those 'at risk' (1.29 +/- 0.30 vs. 1.46 +/- 0.33 V, P = 0.047); however, the peak response to acetylcholine or sodium nitroprusside did not differ in the two groups. A significant positive correlation was demonstrated between maximal oxygen uptake and maximum hyperaemia (r = 0.52, P = 0.006 l/min and r = 0.60, P = 0.001 ml/kg/min) and peak ACh response (r = 0.40, P = 0.04 l/min and r = 0.47, P = 0.013 ml/kg/min) in the 'at-risk' group when expressed in absolute (l/min) or body mass-related (ml/kg/min) terms. No significant correlations were found in the control group." ], "LABELS": [ "METHODS", "RESULTS" ], "MESHES": [ "Adult", "Anthropometry", "Blood Glucose", "Blood Pressure", "Diabetes Mellitus, Type 2", "Diabetes, Gestational", "Disease Susceptibility", "Female", "Glucose Tolerance Test", "Humans", "Insulin", "Lipids", "Male", "Microcirculation", "Middle Aged", "Motor Activity", "Oxygen Consumption", "Physical Fitness", "Pregnancy", "Skin" ], "YEAR": "2005", "reasoning_required_pred": "maybe", "reasoning_free_pred": "maybe", "final_decision": "maybe", "LONG_ANSWER": "In this 'at-risk' group with skin microvascular dysfunction maximal oxygen uptake was not reduced compared with healthy controls. However, in the 'at-risk' group alone, individuals with higher levels of aerobic fitness also had better microvascular and endothelial responsiveness." }, "24625433": { "QUESTION": "Are high flow nasal cannulae noisier than bubble CPAP for preterm infants?", "CONTEXTS": [ "Noise exposure in the neonatal intensive care unit is believed to be a risk factor for hearing loss in preterm neonates. Continuous positive airway pressure (CPAP) devices exceed recommended noise levels. High flow nasal cannulae (HFNC) are an increasingly popular alternative to CPAP for treating preterm infants, but there are no in vivo studies assessing noise production by HFNC.", "To study whether HFNC are noisier than bubble CPAP (BCPAP) for preterm infants.", "An observational study of preterm infants receiving HFNC or BCPAP. Noise levels within the external auditory meatus (EAM) were measured using a microphone probe tube connected to a calibrated digital dosimeter. Noise was measured across a range of frequencies and reported as decibels A-weighted (dBA).", "A total of 21 HFNC and 13 BCPAP noise measurements were performed in 21 infants. HFNC gas flows were 2-5 L/min, and BCPAP gas flows were 6-10 L/min with set pressures of 5-7 cm of water. There was no evidence of a difference in average noise levels measured at the EAM: mean difference (95% CI) of -1.6 (-4.0 to 0.9) dBA for HFNC compared to BCPAP. At low frequency (500 Hz), HFNC was mean (95% CI) 3.0 (0.3 to 5.7) dBA quieter than BCPAP. Noise increased with increasing BCPAP gas flow (p=0.007), but not with increasing set pressure. There was a trend to noise increasing with increasing HFNC gas flows." ], "LABELS": [ "BACKGROUND", "OBJECTIVE", "METHODS", "RESULTS" ], "MESHES": [ "Catheters", "Continuous Positive Airway Pressure", "Environmental Monitoring", "Humans", "Infant, Newborn", "Infant, Premature", "Infant, Premature, Diseases", "Intensive Care Units, Neonatal", "Nasal Cavity", "Noise", "Noninvasive Ventilation", "Terminology as Topic" ], "YEAR": "2014", "reasoning_required_pred": "no", "reasoning_free_pred": "no", "final_decision": "no", "LONG_ANSWER": "At the gas flows studied, HFNC are not noisier than BCPAP for preterm infants." }, "25304452": { "QUESTION": "Nutritional assessment of gluten-free diet. Is gluten-free diet deficient in some nutrient?", "CONTEXTS": [ "The gluten-free diet has traditionally been accepted as a healthy diet, but there are articles advocating that it may have some nutritional deficiencies. The current study assesses whether there was any change in the contributions of calories, essential elements, proportion of fatty acids, vitamins, minerals and fiber in children who were diagnosed with celiac diseases, comparing the diet with gluten prior one year after diagnosis with the diet without gluten to the year of diagnosis. The level of clinical or analytical impact that nutritional deficits could have was also assessed.", "A prospective,descriptive, observational study in which information was collected from a dietary survey, anthropometric and analytical data at pre-diagnosis of celiac disease and following a gluten diet and one year after celiac disease diagnosis, under gluten-free diet.", "A total of 37 patients meet the study criteria. A decrease in the intake of saturated fatty acids was found, with an increase of monounsaturated fatty acids and an increase in the intake of phosphorus in the diet without gluten. A deficient intake of vitamin D was found in both diets. Clinically, at year of gluten-free diet there was an improvement in weight and size. Analytically, there was an improvement in hemoglobin, ferritin, vitamin D, and parathyroid hormone in plasma." ], "LABELS": [ "INTRODUCTION", "MATERIAL AND METHODS", "RESULTS" ], "MESHES": [ "Adolescent", "Celiac Disease", "Child", "Child, Preschool", "Diet, Gluten-Free", "Female", "Humans", "Infant", "Male", "Nutrition Assessment", "Nutritive Value", "Prospective Studies" ], "YEAR": "2015", "reasoning_required_pred": "yes", "reasoning_free_pred": "no", "final_decision": "yes", "LONG_ANSWER": "The gluten-free diet has minimal deficiencies, similar to those present in the diet with gluten, with an improvement in the lipid profile by increasing the proportion of monounsaturated fatty acids to the detriment of saturated fatty acids." }, "19711462": { "QUESTION": "Standardizing care in medical oncology: are Web-based systems the answer?", "CONTEXTS": [ "Medical oncology is embracing information technology to standardize care and improve patient outcomes, with a range of Web-based systems used internationally. The authors' aim was to determine the factors affecting the uptake and use of a Web-based protocol system for medical oncology in the Australian setting.", "The authors conducted 50 interviews and observed medical oncology physicians, nurses, and pharmacists in their treatment setting at 6 hospitals in different geographic locations.", "The Web-based system plays a major role in guiding oncology treatment across participating sites. However, its use varies according to hospital location, clinician roles, and experience. A range of issues impact on clinicians' attitudes toward and use of the Web-based system. Important factors are clinician-specific (eg, their need for autonomy and perceptions of lack of time) or environmental (eg, hospital policy on protocol use, endorsement of the system, and the availability of appropriate infrastructure, such as sufficient computers). The level of education received regarding the system was also found to be integral to its ongoing use." ], "LABELS": [ "BACKGROUND", "METHODS", "RESULTS" ], "MESHES": [ "Attitude of Health Personnel", "Australia", "Delivery of Health Care", "Evidence-Based Medicine", "Internet", "Medical Oncology", "Practice Patterns, Physicians'" ], "YEAR": "2009", "reasoning_required_pred": "maybe", "reasoning_free_pred": "maybe", "final_decision": "maybe", "LONG_ANSWER": "Although the provision of high-quality evidence-based resources, electronic or otherwise, is essential for standardizing care and improving patient outcomes, the authors' findings demonstrate that this alone does not ensure uptake. It is important to understand end-users, the environment in which they operate, and the basic infrastructure required to implement such a system. Implementation must also be accompanied by continuing education and endorsement to ensure both long-term sustainability and use of the system to its full potential." }, "28196511": { "QUESTION": "Antiretroviral therapy related adverse effects: Can sub-Saharan Africa cope with the new \"test and treat\" policy of the World Health Organization?", "CONTEXTS": [ "Recent studies have shown that early antiretroviral therapy (ART) initiation results in significant HIV transmission reduction. This is the rationale behind the \"test and treat\" policy of the World Health Organization (WHO). Implementation of this policy will lead to an increased incidence of ART-related adverse effects, especially in sub-Saharan Africa (SSA). Is the region yet ready to cope with such a challenging issue?", "The introduction and widespread use of ART have drastically changed the natural history of HIV/AIDS, but exposure to ART leads to serious medication-related adverse effects mainly explained by mitochondrial toxicities, and the situation will get worse in the near future. Indeed, ART is associated with an increased risk of developing cardiovascular disease, lipodystrophy, prediabetes and overt diabetes, insulin resistance and hyperlactatemia/lactic acidosis. The prevalence of these disorders is already high in SSA, and the situation will be exacerbated by the implementation of the new WHO recommendations. Most SSA countries are characterized by (extreme) poverty, very weak health systems, inadequate and low quality of health services, inaccessibility to existing health facilities, lack of (qualified) health personnel, lack of adequate equipment, inaccessibility and unaffordability of medicines, and heavy workload in a context of a double burden of disease. Additionally, there is dearth of data on the incidence and predictive factors of ART-related adverse effects in SSA, to anticipate on strategies that should be put in place to prevent the occurrence of these conditions or properly estimate the upcoming burden and prepare an adequate response plan. These are required if we are to anticipate and effectively prevent this upcoming burden." ], "LABELS": [ "BACKGROUND", "MAIN BODY" ], "MESHES": [ "Acquired Immunodeficiency Syndrome", "Africa South of the Sahara", "Anti-HIV Agents", "Antiretroviral Therapy, Highly Active", "HIV Infections", "Health Policy", "Humans", "Poverty", "Prevalence", "World Health Organization" ], "YEAR": "2017", "reasoning_required_pred": "no", "reasoning_free_pred": "no", "final_decision": "no", "LONG_ANSWER": "While SSA would be the first region to experience the huge benefits of implementing the \"test and treat\" policy of the WHO, the region is not yet prepared to manage the consequential increased burden of ART-related toxic and metabolic complications. Urgent measures should be taken to fill the lacunae if SSA is not to become over-burdened by the consequences of the \"test and treat\" policy." }, "24591144": { "QUESTION": "Are the elderly with oropharyngeal carcinoma undertreated?", "CONTEXTS": [ "To determine if elderly patients with oropharyngeal squamous cell carcinoma (OPSCC) are receiving less treatment and to evaluate the benefit of aggressive therapy in this population.", "Retrospective analysis of a large population database.", "Patients in the Surveillance, Epidemiology, and End Results database with OPSCC diagnosed from 2004 to 2009 were included. The patients were categorized into age groups 45 to 54, 55 to 64, 65 to 74, 75 to 84, and 85 years and older, then further categorized by treatment status. Kaplan-Meier analysis of disease-specific survival (DSS) for late-stage (III and IV) OPSCC was performed for all age and treatment categories, followed by a multivariate cox regression of treatment status, tumor site, race, stage, and sex per age group.", "A total of 14,909 patients with OPSCC were identified. In our demographic data, we observed a significant increase in the number of patients who did not receive treatment (surgery, radiation, or combined therapy) after age 55. Kaplan-Meier analysis showed that age groups 65 to 74 and 75 to 84 had substantial benefits in DSS with surgery, radiation, or combined therapy. Multivariable analysis did not demonstrate any statistically significant difference in the hazard ratios for combined treatment among age groups 45 to 54, 55 to 64, 65 to 74, and 75 to 84." ], "LABELS": [ "HYPOTHESIS", "STUDY DESIGN", "METHODS", "RESULTS" ], "MESHES": [ "Age Factors", "Aged", "Aged, 80 and over", "Carcinoma, Squamous Cell", "Female", "Humans", "Male", "Middle Aged", "Oropharyngeal Neoplasms", "Retrospective Studies", "Vulnerable Populations" ], "YEAR": "2014", "reasoning_required_pred": "maybe", "reasoning_free_pred": "yes", "final_decision": "maybe", "LONG_ANSWER": "Proportionally fewer elderly patients with OPSCC are being treated than younger individuals. These patients can have significant benefits from aggressive treatments despite their older age as shown by our survival analysis. We recommend the use of objective measures to assess patient fitness to reduce the potential for undertreatment in the elderly population." }, "21431987": { "QUESTION": "Preoperative staging of patients with liver metastases of colorectal carcinoma. Does PET/CT really add something to multidetector CT?", "CONTEXTS": [ "This study was designed to determine prospectively whether the systematic use of PET/CT associated with conventional techniques could improve the accuracy of staging in patients with liver metastases of colorectal carcinoma. We also assessed the impact on the therapeutic strategy.", "Between 2006 and 2008, 97 patients who were evaluated for resection of LMCRC were prospectively enrolled. Preoperative workup included multidetector-CT (MDCT) and PET/CT. In 11 patients with liver steatosis or iodinated contrast allergy, MR also was performed. Sixty-eight patients underwent laparotomy. Sensitivity, specificity, positive predictive value (PPV), and negative predictive values for hepatic and extrahepatic staging of MDCT and PET-CT were calculated.", "In a lesion-by-lesion analysis of the hepatic staging, the sensitivity of MDCT/RM was superior to PET/CT (89.2 vs. 55%, p\u00a0<\u00a00.001). On the extrahepatic staging, PET/CT was superior to MDCT/MR only for the detection of locoregional recurrence (p\u00a0=\u00a00.03) and recurrence in uncommon sites (p\u00a0=\u00a00.016). New findings in PET/CT resulted in a change in therapeutic strategy in 17 patients. However, additional information was correct only in eight cases and wrong in nine patients." ], "LABELS": [ "PURPOSE", "METHODS", "RESULTS" ], "MESHES": [ "Colorectal Neoplasms", "Female", "Fluorodeoxyglucose F18", "Follow-Up Studies", "Humans", "Liver Neoplasms", "Male", "Middle Aged", "Neoplasm Recurrence, Local", "Neoplasm Staging", "Positron-Emission Tomography", "Preoperative Care", "Prognosis", "Prospective Studies", "Radiopharmaceuticals", "Sensitivity and Specificity", "Survival Rate", "Tomography, X-Ray Computed" ], "YEAR": "2011", "reasoning_required_pred": "no", "reasoning_free_pred": "no", "final_decision": "no", "LONG_ANSWER": "PET/CT has a limited role in hepatic staging of LMCRC. Although PET-CT has higher sensitivity for the detection of extrahepatic disease in some anatomic locations, its results are hampered by its low PPV. PET/CT provided additional useful information in 8% of the cases but also incorrect and potentially harmful data in 9% of the staging. Our findings support a more selective use of PET/CT, basically in patients with high risk of local recurrence." }, "18928979": { "QUESTION": "Can myometrial electrical activity identify patients in preterm labor?", "CONTEXTS": [ "The objective of the study was to determine whether myometrial electrical activity can differentiate false from true preterm labor.", "Electrical uterine myography (EUM) was measured prospectively on 87 women, gestational age less than 35 weeks. The period between contractions, power of contraction peaks and movement of center of electrical activity (RMS), was used to develop an index score (1-5) for prediction of preterm delivery (PTD) within 14 days of the test. The score was compared with fetal fibronectin (fFN) and cervical length (CL).", "Patients delivering within 14 days from testing showed a higher index and mean RMS (P = .000). No patients with EUM index scores of 1-2 delivered in this time frame. Combining EUM with CL or fFN increased predictability. Logistic regression revealed that history of PTD and EUM index had 4- to 5-fold increased risk for PTD. Gestational age at testing, body mass index, fFN, and CL were nonsignificant contributors to PTD risk." ], "LABELS": [ "OBJECTIVE", "STUDY DESIGN", "RESULTS" ], "MESHES": [ "Adult", "Body Mass Index", "Electromyography", "Female", "Fetus", "Fibronectins", "Humans", "Logistic Models", "Myometrium", "Obstetric Labor, Premature", "Pregnancy", "Prospective Studies", "Sensitivity and Specificity", "Uterine Contraction" ], "YEAR": "2008", "reasoning_required_pred": "yes", "reasoning_free_pred": "yes", "final_decision": "yes", "LONG_ANSWER": "Measuring myometrial electrical activity may enhance identification of patients in true premature labor." }, "25752725": { "QUESTION": "Schizophrenia patients with high intelligence: A clinically distinct sub-type of schizophrenia?", "CONTEXTS": [ "Schizophrenia patients are typically found to have low IQ both pre- and post-onset, in comparison to the general population. However, a subgroup of patients displays above average IQ pre-onset. The nature of these patients' illness and its relationship to typical schizophrenia is not well understood. The current study sought to investigate the symptom profile of high-IQ schizophrenia patients.", "We identified 29 schizophrenia patients of exceptionally high pre-morbid intelligence (mean estimated pre-morbid intelligence quotient (IQ) of 120), of whom around half also showed minimal decline (less than 10 IQ points) from their estimated pre-morbid IQ. We compared their symptom scores (SAPS, SANS, OPCRIT, MADRS, GAF, SAI-E) with a comparison group of schizophrenia patients of typical IQ using multinomial logistic regression.", "The patients with very high pre-morbid IQ had significantly lower scores on negative and disorganised symptoms than typical patients (RRR=0.019; 95% CI=0.001, 0.675, P=0.030), and showed better global functioning and insight (RRR=1.082; 95% CI=1.020, 1.148; P=0.009). Those with a minimal post-onset IQ decline also showed higher levels of manic symptoms (RRR=8.213; 95% CI=1.042, 64.750, P=0.046)." ], "LABELS": [ "BACKGROUND", "METHODS", "RESULTS" ], "MESHES": [ "Adult", "Cognition", "Female", "Humans", "Intelligence", "Intelligence Tests", "Male", "Middle Aged", "Neuropsychological Tests", "Schizophrenia", "Schizophrenic Psychology" ], "YEAR": "2015", "reasoning_required_pred": "yes", "reasoning_free_pred": "yes", "final_decision": "yes", "LONG_ANSWER": "These findings provide evidence for the existence of a high-IQ variant of schizophrenia that is associated with markedly fewer negative symptoms than typical schizophrenia, and lends support to the idea of a psychosis spectrum or continuum over boundaried diagnostic categories." }, "12855939": { "QUESTION": "Is ankle/arm pressure predictive for cardiovascular mortality in older patients living in nursing homes?", "CONTEXTS": [ "longitudinal descriptive study.", "2 large nursing homes in Turin, Italy.", "418 dependent elderly (83 males, 335 females, mean age 83.7+/-8.5 y, range 55-102) living in the nursing homes.", "the prevalence of peripheral arterial disease (PAD) was evaluated using a Doppler Ultrasound measurement of AAI (Ankle/Arm blood pressure Index). Death causes according to ICD-9-CM were ascertained on patient's clinical records.", "Diagnosis of PAD was made in 122 subjects (29.2%) with AAI<0.90. After a 3 year follow-up 203 patients (48.6%) died. The presence of PAD was not related to total mortality or to mortality for ischemic heart disease (IHD), cerebrovascular disease or other causes. IHD mortality was significantly and independently related to low haemoglobin values, previous cerebrovascular disease, polypharmacy and poor mobility conditions." ], "LABELS": [ "DESIGN", "SETTING", "SUBJECTS", "MEASUREMENTS", "RESULTS" ], "MESHES": [ "Aged", "Aged, 80 and over", "Ankle", "Arm", "Blood Pressure", "Cardiovascular Diseases", "Female", "Humans", "Longitudinal Studies", "Male", "Nursing Homes", "Prognosis" ], "YEAR": "2003", "reasoning_required_pred": "no", "reasoning_free_pred": "no", "final_decision": "no", "LONG_ANSWER": "The prevalence of PAD is high in nursing home residents. AAI is not predictive for IHD mortality in this population. In very frail elderly traditional risk factors and PAD are less important predictors of death compared to poor functional status, nutritional factors and previous cardiovascular disease." }, "24476003": { "QUESTION": "Is nasogastric decompression useful in prevention of leaks after laparoscopic sleeve gastrectomy?", "CONTEXTS": [ "Although its excellent results, laparoscopic sleeve gastrectomy (LSG) presents major complications ranging from 0% to 29%. Among them, the staple line leak presents an incidence varying from 0% to 7%. Many trials debated about different solutions in order to reduce leaks' incidence. No author has investigated the role of gastric decompression in the prevention of this complication. Aim of our work is to evaluate if this procedure can play a role in avoiding the occurrence of staple line leaks after LSG.", "Between January 2008 and November 2012, 145 patients were prospectively and randomly included in the study. Seventy patients composed the group A, whose operations were completed with placement of nasogastric tube; the other 75 patients were included in the group B, in which no nasogastric tube was placed.", "No statistical differences were observed between group A and group B regarding gender distribution, age, weight, and BMI. No intraoperative complications and no conversion occurred in both groups. Intraoperative blood loss (50.1 \u00b1 42.3 vs. 52.5 \u00b1 37.6 ml, respectively) and operative time (65.4 \u00b1 25.5 vs. 62.6 \u00b1 27.8 min, respectively) were comparable between the two groups (p: NS). One staple line leak (1.4%) occurred on 6th postoperative day in group A patients. No leak was observed in group B patients. Postoperative hospital stay was significantly longer in group A vs. group B patients (7.6 \u00b1 3.4 vs. 6.2 \u00b1 3.1 days, respectively, p: 0.04)." ], "LABELS": [ "INTRODUCTION", "MATERIALS AND METHODS", "RESULTS" ], "MESHES": [ "Adult", "Bariatric Surgery", "Female", "Gastrectomy", "Humans", "Intubation, Gastrointestinal", "Male", "Middle Aged", "Obesity, Morbid", "Postoperative Complications", "Prospective Studies", "Young Adult" ], "YEAR": "2014", "reasoning_required_pred": "no", "reasoning_free_pred": "no", "final_decision": "no", "LONG_ANSWER": "Routine placement of nasogastric tube in patients operated of LSG seems not useful in reducing leaks' incidence." }, "24606627": { "QUESTION": "The objective structured clinical examination: can physician-examiners participate from a distance?", "CONTEXTS": [ "Currently, a 'pedagogical gap' exists in distributed medical education in that distance educators teach medical students but typically do not have the opportunity to assess them in large-scale examinations such as the objective structured clinical examination (OSCE). We developed a remote examiner OSCE (reOSCE) that was integrated into a traditional OSCE to establish whether remote examination technology may be used to bridge this gap. The purpose of this study was to explore whether remote physician-examiners can replace on-site physician-examiners in an OSCE, and to determine the feasibility of this new examination method.", "Forty Year 3 medical students were randomised into six reOSCE stations that were incorporated into two tracks of a 10-station traditional OSCE. For the reOSCE stations, student performance was assessed by both a local examiner (LE) in the room and a remote examiner (RE) who viewed the OSCE encounters from a distance. The primary endpoint was the correlation of scores between LEs and REs across all reOSCE stations. The secondary endpoint was a post-OSCE survey of both REs and students.", "Statistically significant correlations were found between LE and RE checklist scores for history taking (r = 0.64-r = 0.80), physical examination (r = 0.41-r = 0.54), and management stations (r = 0.78). Correlations between LE and RE global ratings were more varied (r = 0.21-r = 0.77). Correlations on three of the six stations reached significance. Qualitative analysis of feedback from REs and students showed high acceptance of the reOSCE despite technological issues." ], "LABELS": [ "OBJECTIVES", "METHODS", "RESULTS" ], "MESHES": [ "Attitude of Health Personnel", "Checklist", "Clinical Competence", "Education, Medical", "Educational Measurement", "Educational Technology", "Faculty, Medical", "Feasibility Studies", "Female", "Humans", "Male", "Medical History Taking", "Physical Examination", "Remote Consultation", "Reproducibility of Results", "Students, Medical" ], "YEAR": "2014", "reasoning_required_pred": "no", "reasoning_free_pred": "yes", "final_decision": "yes", "LONG_ANSWER": "This preliminary study demonstrated that OSCE ratings by LEs and REs were reasonably comparable when using checklists. Remote examination may be a feasible and acceptable way of assessing students' clinical skills, but further validity evidence will be required before it can be recommended for use in high-stakes examinations." }, "23386371": { "QUESTION": "CPAP therapy in patients with idiopathic pulmonary fibrosis and obstructive sleep apnea: does it offer a better quality of life and sleep?", "CONTEXTS": [ "The recent literature shows an increased incidence of obstructive sleep apnea (OSA) in patients with idiopathic pulmonary fibrosis (IPF). On the other hand, there are no published studies related to continuous positive airway pressure (CPAP) treatment in this patient group. Our aim was to assess the effect of CPAP on sleep and overall life quality parameters in IPF patients with OSA and to recognize and overcome possible difficulties in CPAP initiation and acceptance by these patients.", "Twelve patients (ten males and two females, age 67.1\u2009\u00b1\u20097.2\u00a0years) with newly diagnosed IPF and moderate to severe OSA, confirmed by overnight attended polysomnography, were included. Therapy with CPAP was initiated after a formal in-lab CPAP titration study. The patients completed the Epworth Sleepiness Scale (ESS), the Pittsburgh Sleep Quality Index (PSQI), the Functional Outcomes in Sleep Questionnaire (FOSQ), the Fatigue Severity Scale (FSS), the SF-36 quality of life questionnaire, and the Beck Depression Inventory (BDI) at CPAP initiation and after 1, 3, and 6\u00a0months of effective CPAP therapy.", "A statistically significant improvement was observed in the FOSQ at 1, 3, and 6\u00a0months after CPAP initiation (baseline 12.9\u2009\u00b1\u20092.9 vs. 14.7\u2009\u00b1\u20092.6 vs. 15.8\u2009\u00b1\u20092.1 vs. 16.9\u2009\u00b1\u20091.9, respectively, p\u2009=\u20090.02). Improvement, although not statistically significant, was noted in ESS score (9.2\u2009\u00b1\u20095.6 vs. 7.6\u2009\u00b1\u20094.9 vs. 7.5\u2009\u00b1\u20095.3 vs. 7.7\u2009\u00b1\u20095.2, p\u2009=\u20090.84), PSQI (10.7\u2009\u00b1\u20094.4 vs. 10.1\u2009\u00b1\u20094.3 vs. 9.4\u2009\u00b1\u20094.7 vs. 8.6\u2009\u00b1\u20095.2, p\u2009=\u20090.66), FSS (39.5\u2009\u00b1\u200910.2 vs. 34.8\u2009\u00b1\u20098.5 vs. 33.6\u2009\u00b1\u200910.7 vs. 33.4\u2009\u00b1\u200910.9, p\u2009=\u20090.44), SF-36 (63.2\u2009\u00b1\u200913.9 vs. 68.9\u2009\u00b1\u200913.5 vs. 72.1\u2009\u00b1\u200912.9 vs. 74.4\u2009\u00b1\u200911.3, p\u2009=\u20090.27), and BDI (12.9\u2009\u00b1\u20095.5 vs. 10.7\u2009\u00b1\u20094.3 vs. 9.4\u2009\u00b1\u20094.8 vs. 9.6\u2009\u00b1\u20094.5, p\u2009=\u20090.40). Two patients had difficulty complying with CPAP for a variety of reasons (nocturnal cough, claustrophobia, insomnia) and stopped CPAP use after the first month, despite intense follow-up by the CPAP clinic staff. Heated humidification was added for all patients in order to improve the common complaint of disabling nocturnal cough." ], "LABELS": [ "BACKGROUND", "METHODS", "RESULTS" ], "MESHES": [ "Activities of Daily Living", "Aged", "Continuous Positive Airway Pressure", "Disorders of Excessive Somnolence", "Female", "Greece", "Humans", "Male", "Middle Aged", "Polysomnography", "Pulmonary Fibrosis", "Quality of Life", "Sleep Apnea, Obstructive", "Surveys and Questionnaires", "Treatment Outcome" ], "YEAR": "2013", "reasoning_required_pred": "maybe", "reasoning_free_pred": "yes", "final_decision": "maybe", "LONG_ANSWER": "Effective CPAP treatment in IPF patients with OSA results in a significant improvement in daily living activities based on the FOSQ, namely an OSA-specific follow-up instrument. Improvement was also noted in other questionnaires assessing quality of life, though not to a statistically significant degree, probably because of the multifactorial influences of IPF on physical and mental health. The probability of poor CPAP compliance was high and could only be eliminated with intense follow-up by the CPAP clinic staff." }, "10922093": { "QUESTION": "Does open access endoscopy close the door to an adequately informed patient?", "CONTEXTS": [ "The use of open access endoscopy is increasing. Its effect on the adequacy of patient informed consent, procedure acceptance and the impact on subsequent communication/transfer of procedure results to the patient have not been evaluated. The aim of our study was to compare the extent of preknowledge of procedures and test explanation, patient medical complexity, information transfer and overall patient satisfaction between a patient group referred for outpatient open access endoscopy versus a patient group from a gastrointestinal (GI) subspecialty clinic.", "Information was obtained from all patients presenting for outpatient upper and lower endoscopy by using a 1-page questionnaire. Patients from the two groups who had an outpatient upper/lower endoscopic procedure were contacted by phone after the procedure to obtain information with a standardized questionnaire.", "The open access patients reported receiving significantly less information to help them identify the procedure (p<0.01) and less explanation concerning the nature of the procedure than the group of patients referred from the subspecialty clinic (p<0.005). There was no difference between the two groups in satisfaction scores for examinations performed under conscious sedation. For flexible sigmoidoscopy without sedation, however, the GI clinic patient group were more satisfied with their procedure. The majority of patients, regardless of access, were more likely to receive endoscopic results from a gastroenterologist than the referring physician. Furthermore, the patients in the GI clinic group who underwent colonoscopy felt significantly better at follow-up." ], "LABELS": [ "BACKGROUND", "METHODS", "RESULTS" ], "MESHES": [ "Adult", "Aged", "Ambulatory Care", "Endoscopy, Gastrointestinal", "Female", "Gastrointestinal Diseases", "Health Care Surveys", "Health Knowledge, Attitudes, Practice", "Health Services Accessibility", "Humans", "Informed Consent", "Male", "Middle Aged", "Patient Satisfaction", "Probability", "Referral and Consultation", "Surveys and Questionnaires", "Truth Disclosure", "United States" ], "YEAR": "2000", "reasoning_required_pred": "yes", "reasoning_free_pred": "yes", "final_decision": "yes", "LONG_ANSWER": "Patients undergoing open access procedures are less likely to be properly informed about their endoscopic procedure. Our results indicate that with open access endoscopy, a defined mechanism needs to be in place for communication of endoscopic results to the patient." }, "20602101": { "QUESTION": "Is hypoalbuminemia an independent prognostic factor in patients with gastric cancer?", "CONTEXTS": [ "Studies have indicated that hypoalbuminemia is associated with decreased survival of patients with gastric cancer. However, the prognostic value of albumin may be secondary to an ongoing systemic inflammatory response. The aim of the study was to assess the relation between hypoalbuminemia, the systemic inflammatory response, and survival in patients with gastric cancer.", "Patients diagnosed with gastric carcinoma attending the upper gastrointestinal surgical unit in the Royal Infirmary, Glasgow between April 1997 and December 2005 and who had a pretreatment measurement of albumin and C-reactive protein (CRP) were studied.", "Most of the patients had stage III/IV disease and received palliative treatment. The minimum follow-up was 15 months. During follow-up, 157 (72%) patients died of their cancer. On univariate analysis, stage (p<0.001), treatment (p<0.001), albumin level (p<0.001), and CRP level (p<0.001) were significant predictors of survival. On multivariate analysis, stage (p<0.001), treatment (p<0.001), and CRP level (p<0.001) remained significant predictors of survival. Albumin was no longer an independent predictor of survival." ], "LABELS": [ "BACKGROUND", "METHODS", "RESULTS" ], "MESHES": [ "Aged", "Albumins", "C-Reactive Protein", "Female", "Humans", "Hypoalbuminemia", "Male", "Middle Aged", "Predictive Value of Tests", "Prognosis", "Stomach Neoplasms", "Survival Analysis", "Systemic Inflammatory Response Syndrome" ], "YEAR": "2010", "reasoning_required_pred": "no", "reasoning_free_pred": "no", "final_decision": "no", "LONG_ANSWER": "Low albumin concentrations are associated with poorer survival in patients with gastric cancer. However, the strength of this relation with survival is dependent on the presence of a systemic inflammatory response, as evidenced by an elevated CRP level. Therefore, it appears that the relation between hypoalbuminemia and poor survival is secondary to that of the systemic inflammatory response." }, "18065862": { "QUESTION": "Can the postoperative pain level be predicted preoperatively?", "CONTEXTS": [ "We searched for factors present preoperatively which could be used to predict the intensity of postoperative pain.", "We undertook a prospective study among all patients aged over 18 years who underwent shoulder surgery from January to September 2004 in our unit. The study included 86 patients for which the following data were noted preoperatively: past history of pain, intensity and duration of prior pain, triggering factors, localization, psychological context. The intensity of the postoperative pain was measured on a visual analog scale (VAS); measurements were noted up to 24 hours postop then at one month. Data processing was performed with Statview5.5.", "Significant postoperative pain was correlated with a prior history of surgical pain, with duration of prior pain, with intensity of preoperative pain, and with depression.", "Significant sustained preoperative pain can favor memory of pain leading to postoperative sensitivization of nociception neurons. Intense postoperative pain can favor the development of refractory chronic pain." ], "LABELS": [ "PURPOSE OF THE STUDY", "MATERIAL AND METHODS", "RESULTS", "DISCUSSION" ], "MESHES": [ "Acetaminophen", "Adolescent", "Adult", "Aged", "Analgesia, Patient-Controlled", "Analgesics, Non-Narcotic", "Anxiety", "Depression", "Female", "Follow-Up Studies", "Forecasting", "Humans", "Life Change Events", "Male", "Medical History Taking", "Middle Aged", "Morphine", "Narcotics", "Pain Measurement", "Pain, Postoperative", "Prospective Studies", "Shoulder Joint" ], "YEAR": "2007", "reasoning_required_pred": "yes", "reasoning_free_pred": "yes", "final_decision": "yes", "LONG_ANSWER": "Significant postoperative pain can be expected in the following situations: pain after prior surgery, presence of chronic pain sustained for more than six months, intense preoperative pain, state of depression." }, "9569972": { "QUESTION": "Proliferative index obtained by DNA image cytometry. Does it add prognostic information in Auer IV breast cancer?", "CONTEXTS": [ "To investigate whether the S + G2/M fraction (proliferative index) is a prognostic determinant in breast cancers classified as Auer IV.", "Prognostic evaluation of Auer IV DNA histograms with respect to the high versus low S + G2/M fraction, obtained by image cytometry on consecutive breast cancer imprint preparations.", "When studying recurrence-free survival (n = 136), the prognostic value of S + G2/M was found to vary with time: it was negligible before the median time to relapse (1.5 years) but thereafter statistically significant, in both univariate and multivariate analysis. The same pattern was found when overall survival was used as the end point; the effect was delayed to about the median time until death (three years). Tumors with a low S + G2/M fraction were smaller and more often estrogen receptor- and progesterone receptor-positive than those with a high S + G2/M fraction." ], "LABELS": [ "OBJECTIVE", "STUDY DESIGN", "RESULTS" ], "MESHES": [ "Breast Neoplasms", "DNA, Neoplasm", "Disease-Free Survival", "Female", "G2 Phase", "Humans", "Image Cytometry", "Mitosis", "Prognosis", "S Phase", "Survivors" ], "YEAR": "1998", "reasoning_required_pred": "yes", "reasoning_free_pred": "yes", "final_decision": "yes", "LONG_ANSWER": "According to ICM-DNA values corresponding to the S + G2/M region, patients with breast cancers classified as Auer IV can be divided into subgroups with different tumor characteristics and prognoses." }, "25406780": { "QUESTION": "Does skin care frequency affect the severity of incontinence-associated dermatitis in critically ill patients?", "CONTEXTS": [ "Incontinence-associated dermatitis (IAD) is a potentially serious skin injury that can lead to pressure ulcers (PUs). Multiple studies have indicated the need for evidence to find the most effective skin care protocol to reduce the incidence and severity of IAD in critically ill patients.", "To compare the incidence and severity of IAD in two groups on a progressive care unit (PCU) using a defined skin care protocol: cleaning with a gentle cleanser and moisturizer, then applying a skin protectant/barrier. The control group received the skin care protocol every 12 hours and the interventional group received the protocol every 6 hours; both groups also received it as needed.", "A 9-month randomized prospective study was conducted on 99 patients (N = 55 in the intervention group and N = 44 in the control group) who were incontinent of urine, stool, or both, or had a fecal diversion device or urinary catheter for more than 2 days.", "The dermatitis score in the intervention group on discharge was significantly less (7.1%; P \u2264 0.001) in the moderate IAD group than in the control group (10.9%). The dermatitis score means and P values of each group were compared using a paired t test." ], "LABELS": [ "BACKGROUND", "OBJECTIVE", "METHODS", "RESULTS" ], "MESHES": [ "Aged", "Critical Illness", "Dermatitis", "Fecal Incontinence", "Female", "Humans", "Male", "Nursing Evaluation Research", "Prospective Studies", "Severity of Illness Index", "Skin Care", "Urinary Incontinence" ], "YEAR": "2014", "reasoning_required_pred": "yes", "reasoning_free_pred": "yes", "final_decision": "yes", "LONG_ANSWER": "The researchers studied a defined skin care protocol using a cleanser with aloe vera and a cleansing lotion, followed by application of either a moisture barrier with silicone or skin protectant with zinc oxide and menthol, undertaken at two different frequencies. Data revealed the incidence of moderate IAD was decreased in the experimental group (receiving the skin protocol every 6 hours and p.r.n.)." }, "20629769": { "QUESTION": "Is primary angioplasty an acceptable alternative to thrombolysis?", "CONTEXTS": [ "The National Infarct Angioplasty Project assessed the feasibility of establishing a comprehensive primary angioplasty service. We aimed to compare satisfaction at intervention hospitals offering angioplasty-based care and control hospitals offering thrombolysis-based care.", "Mixed methods, with postal survey of patients and their carers, supported by semi-structured interviews.", "Survey of 682 patients and 486 carers, and interviews with 33 patients and carers, in eight English hospitals.", "Primary angioplasty or thrombolysis.", "Satisfaction with treatment.", "Responses were received from 595/682 patients (87%) and 418/486 carers (86%). Satisfaction with overall care was high at both intervention and control sites (78% vs. 71% patients rated their care as 'excellent', P = 0.074). Patient satisfaction was higher at intervention sites for some aspects of care such as speed of treatment (80% vs. 67%'excellent', P = 0.001). Convenience of visiting was rated lower at intervention sites by carers (12% vs. 1%'poor', P = 0.001). During interviews, carers reported that they accepted the added inconvenience of visiting primary angioplasty sites in the context of this life-saving treatment. Patient satisfaction with discharge and aftercare was lower in both treatment groups than for other aspects of care." ], "LABELS": [ "OBJECTIVE", "DESIGN", "SETTING AND PARTICIPANTS", "INTERVENTIONS", "MAIN OUTCOME MEASURES", "RESULTS" ], "MESHES": [ "Adolescent", "Adult", "Aged", "Aged, 80 and over", "Angioplasty", "Caregivers", "England", "Health Care Surveys", "Humans", "Interviews as Topic", "Male", "Middle Aged", "Myocardial Infarction", "Patient Satisfaction", "Thrombolytic Therapy", "Young Adult" ], "YEAR": "2010", "reasoning_required_pred": "yes", "reasoning_free_pred": "yes", "final_decision": "yes", "LONG_ANSWER": "Reorganization of care to offer a primary angioplasty service was acceptable to patients and their carers. Satisfaction levels were high regardless of the type of care received, with the exception of discharge and aftercare." }, "23149821": { "QUESTION": "Should HIV-infected patients be screened for silent myocardial ischaemia using gated myocardial perfusion SPECT?", "CONTEXTS": [ "A higher prevalence of cardiovascular risk factors (CRFs) in HIV-infected patients, together with chronic infection and treatments, has resulted in an increased risk of silent myocardial ischaemia (SMI). The objective of this study was to evaluate whether myocardial SPECT should be used for screening HIV-infected patients with no clinical symptoms of coronary artery disease.", "The prevalence of SMI detected by myocardial SPECT was determined in 94 HIV-infected patients with a normal clinical cardiovascular examination in relation to anthropomorphic parameters, CRFs, inflammatory and HIV infection status, and treatment.", "Coronary artery disease was detected in nine patients (eight with ischaemia, one with myocardial infarction), corresponding to 9.6 % positivity. All but two of the scintigraphic diagnoses of ischaemia were confirmed by coronarography. Univariate analysis revealed that the overall number of CRFs and the combination of gender and age were associated with a diagnosis of SMI (p<0.05). According to multivariate analysis, the only independent parameter significantly associated with the scintigraphic diagnosis of SMI was the combination of gender and age (p = 0.01). All the positive myocardial SPECT scans were in men older than 52 years with at least two other CRFs. In this subpopulation of 47 patients, the prevalence of SMI detected by myocardial SPECT reached 19.2 %." ], "LABELS": [ "PURPOSE", "METHODS", "RESULTS" ], "MESHES": [ "Aged", "Algorithms", "Anthropometry", "Anti-Retroviral Agents", "Female", "HIV Infections", "Humans", "Inflammation", "Male", "Middle Aged", "Myocardial Ischemia", "Odds Ratio", "Perfusion", "Prevalence", "Radionuclide Imaging", "Risk Factors", "Tomography, Emission-Computed, Single-Photon" ], "YEAR": "2013", "reasoning_required_pred": "maybe", "reasoning_free_pred": "yes", "final_decision": "maybe", "LONG_ANSWER": "In male HIV-infected patients older than 52 years and with at least two other CRFs, screening for SMI using myocardial SPECT was about four times more likely to be positive than in the general population. This may motivate physicians to advise these patients to undergo more systematic screening for SMI using this technique." }, "11483547": { "QUESTION": "Does the aggressive use of polyvalent antivenin for rattlesnake bites result in serious acute side effects?", "CONTEXTS": [ "To determine the incidence and severity of acute side effects from the use of polyvalent antivenin in victims of rattlesnake bites.", "We retrospectively reviewed the records of all patients who presented with rattlesnake bites to a university teaching hospital during an 11-year period. From patient medical records, we extracted demographic data, clinical measurements, and outcomes during emergency department evaluation and subsequent hospitalization. Data regarding serum sickness were not collected.", "Primary outcome variables were the occurrence of immediate hypersensitivity reaction to antivenin, the type of reaction, permanent disability at hospital discharge, and mortality.", "We identified a total of 73 patients with rattlesnake bites during the study period. Bite envenomation was graded as nonenvenomated, 7 patients (10%); mild, 23 patients (32%); moderate, 32 patients (44%); and severe, 11 patients (15%). We identified 65 patients who received antivenin. Antivenin doses ranged from 1 to 30 vials per patient (mean, 12.0 +/- 6.0), for a total of 777 vials. In 43 patients (66%), 10 or more vials of antivenin were given. The mean number of vials of antivenin given to each snakebite grade were as follows: mild, 8.4 (+/-4.0); moderate, 11.8 (+/-5.7); and severe, 18.7 (+/-6.3). No deaths, amputations, or permanent disability from snakebite occurred in the patients receiving antivenin. Acute side effects of antivenin-occurring within the first 6 hours after administration-were seen in 12 patients (18%; 95% confidence interval, 10%-30%). Acute side effects consisted solely of urticaria in all but 1 patient (2%; 95% confidence interval, 0%-8%). This patient had a history of previous antivenin reaction and required a short course of intravenous epinephrine for blood pressure support. No other complications occurred." ], "LABELS": [ "OBJECTIVE", "DESIGN", "OUTCOME MEASURES", "RESULTS" ], "MESHES": [ "Adolescent", "Adult", "Aged", "Animals", "Antivenins", "Crotalus", "Female", "Humans", "Infant", "Male", "Retrospective Studies", "Snake Bites", "Urticaria" ], "YEAR": "2001", "reasoning_required_pred": "no", "reasoning_free_pred": "no", "final_decision": "no", "LONG_ANSWER": "The administration of polyvalent Crotalidae antivenin is safe. Acute hypersensitivity, when it occurs, consists solely in most cases of urticaria. Serious side effects are uncommon." }, "27884344": { "QUESTION": "Do resident's leadership skills relate to ratings of technical skill?", "CONTEXTS": [ "This study sought to compare general surgery research residents' survey information regarding self-efficacy ratings to their observed performance during a simulated small bowel repair. Their observed performance ratings were based on their leadership skills in directing their assistant.", "Participants were given 15\u00a0min to perform a bowel repair using bovine intestines with standardized injuries. Operative assistants were assigned to help assist with the repair. Before the procedure, participants were asked to rate their expected skills decay, task difficulty, and confidence in addressing the small bowel injury. Interactions were coded to identify the number of instructions given by the participants to the assistant during the repair. Statistical analyses assessed the relationship between the number of directional instructions and participants' perceptions self-efficacy measures. Directional instructions were defined as any dialog by the participant who guided the assistant to perform an action.", "Thirty-six residents (58.3% female) participated in the study. Participants who rated lower levels of decay in their intraoperative decision-making and small bowel repair skills were noted to use their assistant more by giving more instructions. Similarly, a higher number of instructions correlated with lower perceived difficulty in selecting the correct suture, suture pattern, and completing the entire surgical task." ], "LABELS": [ "BACKGROUND", "METHODS", "RESULTS" ], "MESHES": [ "Animals", "Cattle", "Clinical Competence", "Clinical Decision-Making", "Female", "General Surgery", "Humans", "Internship and Residency", "Interprofessional Relations", "Intestines", "Leadership", "Male", "Midwestern United States", "Self Efficacy" ], "YEAR": "2016", "reasoning_required_pred": "yes", "reasoning_free_pred": "yes", "final_decision": "yes", "LONG_ANSWER": "General surgery research residents' intraoperative leadership skills showed significant correlations to their perceptions of skill decay and task difficulty during a bowel repair. Evaluating resident's directional instructions may provide an additional individualized intraoperative assessment metric. Further evaluation relating to operative performance outcomes is warranted." }, "18800356": { "QUESTION": "The FOOTSTEP self-management foot care programme: are rheumatoid arthritis patients physically able to participate?", "CONTEXTS": [ "The FOOTSTEP self-management foot care programme is a clinical and cost-effective programme for basic foot care in the elderly. The aim of this study was to determine if patients with rheumatoid arthritis (RA) would be physically able to participate.", "A consecutive cohort of RA patients undergoing podiatry care underwent tests for sight, reach and grip strength to determine their physical ability to undertake self-managed foot care.", "Thirty RA patients (10 male, 20 female), with a median age of 61 years (range 42 to 84) and disease duration of 10 years (range one to 40), were recruited. All patients passed the sight test, whereas the reach and grip tests were passed by 77% and 67% of patients, respectively. Only 57% of patients passed all the physical tests. Patients who failed the physical tests were older, and had longer disease duration and higher physical disability, pain and general health scores but these were not statistically different." ], "LABELS": [ "BACKGROUND", "METHODS", "RESULTS" ], "MESHES": [ "Activities of Daily Living", "Adult", "Aged", "Aged, 80 and over", "Arthritis, Rheumatoid", "Disability Evaluation", "Female", "Foot Diseases", "Humans", "Hygiene", "Male", "Middle Aged", "Patient Participation", "Podiatry", "Program Evaluation", "Self Care" ], "YEAR": "2009", "reasoning_required_pred": "yes", "reasoning_free_pred": "maybe", "final_decision": "maybe", "LONG_ANSWER": "Just over half the patients in this present cohort may be physically able to undertake some aspects of self-managed foot care, including nail clipping and filing, callus filing and daily hygiene and inspection." }, "22301406": { "QUESTION": "CYP2D6*4 allele and breast cancer risk: is there any association?", "CONTEXTS": [ "CYP2D6 is an important cytochrome P450 enzyme. These enzymes catalyse the oxidative biotransformation of about 25% of clinically important drugs as well as the metabolism of numerous environmental chemical carcinogens. The most frequent null allele of CYP2D6 in European populations, CYP2D6*4, has been studied here in order to elucidate whether a relationship exists between this allele and the risk of developing breast cancer in a Spanish population.", "Ninety-six breast cancer Spanish patients and one hundred healthy female volunteers were genotyped for the CYP2D6*4 allele using AmpliChip CYP450 Test technology.", "Homozygous CYP2D6*4 frequency was significant lower in breast cancer patients than in the control group (OR=0.22, p=0.04). The heterozygous CYP2D6*4 group also displayed lower values in patients than in controls but the difference was not significant (OR=0.698, p=0.28). Therefore, the presence of the CYP2D6*4 allele seems to decrease susceptibility to breast carcinoma in the selected population." ], "LABELS": [ "BACKGROUND", "MATERIALS AND METHODS", "RESULTS" ], "MESHES": [ "Adult", "Aged", "Alleles", "Breast Neoplasms", "Carcinoma, Ductal, Breast", "Carcinoma, Intraductal, Noninfiltrating", "Carcinoma, Lobular", "Case-Control Studies", "Cytochrome P-450 CYP2D6", "DNA", "Female", "Heterozygote", "Homozygote", "Humans", "Middle Aged", "Phenotype", "Polymerase Chain Reaction", "Polymorphism, Genetic", "Risk Factors", "Spain" ], "YEAR": "2012", "reasoning_required_pred": "yes", "reasoning_free_pred": "yes", "final_decision": "yes", "LONG_ANSWER": "A possible decreased transformation of procarcinogens by CYP2D6*4 poor metabolisers could result in a protective effect against carcinogens." }, "26079501": { "QUESTION": "Does base deficit predict mortality in patients with severe traumatic brain injury?", "CONTEXTS": [ "Base Deficit (BD) is a marker of tissue hypoxia in polytrauma patients. It guides resuscitative measures, and predicts outcomes, complications and mortality. The aim of this study was to examine the presence of BD in patients with isolated severe traumatic brain injury (TBI), and to assess if it correlates with the outcomes in these patients.", "This was a retrospective observational study. All patients over the age of 16 years presenting to Aga Khan University Hospital from 2009 to 2013 with isolated TBI, were included. Data was extracted from 2009 to 2013. Glasgow Outcome Scale (GOS) of 4 and 5 at last follow up was categorized as favorable outcome. Data was analyzed using SPSS version 19 and receiver operative curve (ROC) was generated for BD as a predictor of mortality and unfavorable outcome.", "One hundred and eight patients were analyzed. Ninety-eight (90.7%) were males. Mean age was 36.69 \u00b1 17.65. Eighty-eight (81.5%) patients had BD, while 20 (18.5%) patients had base excess. 62 (58.5%) of the patients had unfavorable outcomes. BD on admission had a statistically significant negative correlation with Glasgow Coma Scale (GCS) on presentation (r = -0.239, p = 0.025) and Revised Trauma Score (RTS) (r = -0.214, p = 0.046). However, there was no statistically significant difference in means of BD between survivors and non survivors. Area under receiver operator curve (ROC) for BD as a predictor of mortality statistically non-significant." ], "LABELS": [ "OBJECTIVE", "METHOD", "RESULTS" ], "MESHES": [ "Acid-Base Imbalance", "Adult", "Biomarkers", "Brain Injuries", "Female", "Glasgow Coma Scale", "Humans", "Hypoxia", "Male", "Middle Aged", "Prognosis", "Retrospective Studies", "Young Adult" ], "YEAR": "2015", "reasoning_required_pred": "no", "reasoning_free_pred": "no", "final_decision": "no", "LONG_ANSWER": "Although BD is correlated with GCS at presentation and RTS, it is not a reliable prognostic marker for outcome and mortality in patients with isolated TBI." }, "15774570": { "QUESTION": "Does increased use of private health care reduce the demand for NHS care?", "CONTEXTS": [ "The use of the private sector for health care is increasing, but it is unclear whether this will reduce demand on the NHS. The aim of this study was to examine the relationship between private and NHS outpatient referral rates accounting for their association with deprivation.", "This is a prospective survey of general practitioner referrals to private and NHS consultant-led services between 1 January and 31 December 2001 from 10 general practices in the Trent Focus Collaborative Research Network, United Kingdom. Patient referrals were aggregated to give private and NHS referral rates for each electoral ward in each practice.", "Of 17,137 referrals, 90.4 percent (15,495) were to the NHS and 9.6 percent (1642) to the private sector. Private referral rates were lower in patients from the most deprived fifth of wards compared with the least deprived fifth (rate ratio 0.25, 95 percent CI 0.15 to 0.41, p<0.001), whereas NHS referral rates were slightly higher in patients in the most deprived fifth of wards (rate ratio 1.18, 95 percent CI 0.98 to 1.42, p = 0.08) both after age standardisation and adjustment for practice. The NHS referral rate was significantly higher (rate ratio 1.40, 95 percent CI 1.15 to 1.71, p = 0.001) in wards with private referral rates in the top fifth compared with the bottom fifth after adjustment for deprivation and practice." ], "LABELS": [ "BACKGROUND", "METHODS", "RESULTS" ], "MESHES": [ "Adolescent", "Adult", "Aged", "Catchment Area (Health)", "Child", "Child, Preschool", "Computer Systems", "Confidence Intervals", "England", "Family Practice", "Female", "Health Care Surveys", "Health Services Needs and Demand", "Hospitals, Private", "Hospitals, Public", "Humans", "Infant", "Infant, Newborn", "Male", "Medicine", "Middle Aged", "Odds Ratio", "Practice Patterns, Physicians'", "Private Practice", "Referral and Consultation", "Specialization", "State Medicine", "Vulnerable Populations" ], "YEAR": "2005", "reasoning_required_pred": "yes", "reasoning_free_pred": "no", "final_decision": "no", "LONG_ANSWER": "Increased private health care activity does not reduce the demand for NHS care: NHS and private referral rates were positively associated with each other after adjusting for age, deprivation and practice." }, "18319270": { "QUESTION": "Does confined placental mosaicism account for adverse perinatal outcomes in IVF pregnancies?", "CONTEXTS": [ "IVF singletons have poorer perinatal outcomes than singletons from spontaneous conceptions. This may be due to the influence of ovarian stimulation on the chromosomal constitution of the embryos which could be translated into localized chromosomal anomalies in the placenta. The aim of this study was to compare the incidence of confined placental mosaicism (CPM) in IVF/ICSI pregnancies and spontaneous conceptions.", "We conducted a multi-centre retrospective analysis of karyotype results obtained by chorionic villus sampling (CVS), performed due to advanced maternal age (>or=36 years at 18 weeks of gestation), in the Netherlands between 1995 and 2005.", "From a total of 322 246 pregnancies, 20 885 CVS results were analysed: 235 in the IVF/ICSI group and 20 650 in the control group. The mean age of women in both groups was 38.4 years (mean difference -0.08, 95% CI -0.35 to 0.18). Data relating to the fetal karyotype were missing in 143 cases in the control group. When taking into account missing data, the incidence of CPM was lower in the IVF-ICSI group than in the control group, 1.3% versus 2.2% (odds ratio 0.59, 95% CI 0.19-1.85), whereas the incidence of fetal chromosomal anomalies was increased 4.3% versus 2.4% (odds ratio 1.81, 95% CI 0.95-3.42). Neither differences were statistically significant." ], "LABELS": [ "BACKGROUND", "METHODS", "RESULTS" ], "MESHES": [ "Adult", "Chorionic Villi Sampling", "Female", "Fertilization in Vitro", "Fetus", "Humans", "Karyotyping", "Mosaicism", "Ovulation Induction", "Placenta", "Pregnancy", "Pregnancy Outcome", "Retrospective Studies", "Sperm Injections, Intracytoplasmic" ], "YEAR": "2008", "reasoning_required_pred": "no", "reasoning_free_pred": "no", "final_decision": "no", "LONG_ANSWER": "The incidence of CPM is not increased in IVF/ICSI pregnancies compared with spontaneous conceptions. CPM probably does not account for the adverse perinatal outcomes following IVF/ICSI." }, "26209118": { "QUESTION": "Utility of unenhanced fat-suppressed T1-weighted MRI in children with sickle cell disease -- can it differentiate bone infarcts from acute osteomyelitis?", "CONTEXTS": [ "Children with sickle cell disease (SCD) are at risk of bone infarcts and acute osteomyelitis. The clinical differentiation between a bone infarct and acute osteomyelitis is a diagnostic challenge. Unenhanced T1-W fat-saturated MR images have been proposed as a potential tool to differentiate bone infarcts from osteomyelitis.", "To evaluate the reliability of unenhanced T1-W fat-saturated MRI for differentiation between bone infarcts and acute osteomyelitis in children with SCD.", "We retrospectively reviewed the records of 31 children (20 boys, 11 girls; mean age 10.6 years, range 1.1-17.9 years) with SCD and acute bone pain who underwent MR imaging including unenhanced T1-W fat-saturated images from 2005 to 2010. Complete clinical charts were reviewed by a pediatric hematologist with training in infectious diseases to determine a clinical standard to define the presence or absence of osteomyelitis. A pediatric radiologist reviewed all MR imaging and was blinded to clinical information. Based on the signal intensity in T1-W fat-saturated images, the children were further classified as positive for osteomyelitis (low bone marrow signal intensity) or positive for bone infarct (high bone marrow signal intensity).", "Based on the clinical standard, 5 children were classified as positive for osteomyelitis and 26 children as positive for bone infarct (negative for osteomyelitis). The bone marrow signal intensity on T1-W fat-saturated imaging was not significant for the differentiation between bone infarct and osteomyelitis (P\u2009=\u20090.56). None of the additional evaluated imaging parameters on unenhanced MRI proved reliable in differentiating these diagnoses." ], "LABELS": [ "BACKGROUND", "OBJECTIVE", "MATERIALS AND METHODS", "RESULTS" ], "MESHES": [ "Acute Disease", "Adolescent", "Anemia, Sickle Cell", "Bone and Bones", "Child", "Child, Preschool", "Diagnosis, Differential", "Female", "Humans", "Infant", "Magnetic Resonance Imaging", "Male", "Osteomyelitis", "Retrospective Studies" ], "YEAR": "2015", "reasoning_required_pred": "no", "reasoning_free_pred": "no", "final_decision": "no", "LONG_ANSWER": "The bone marrow signal intensity on unenhanced T1-W fat-saturated MR images is not a reliable criterion to differentiate bone infarcts from osteomyelitis in children." }, "11713724": { "QUESTION": "Cancer of the buccal mucosa: are margins and T-stage accurate predictors of local control?", "CONTEXTS": [ "Cancer of the buccal mucosa is an uncommon and aggressive neoplasm of the oral cavity. Less than 2% of patients treated for cancer of the oral cavity at Roswell Park Cancer Institute (RPCI) from 1971 to 1997 had primary buccal cancers. Because the majority of these patients did not undergo any adjuvant treatment, this group provided us with the opportunity to assess the relationship between margin status and local recurrence for both small (T1-T2) and large (T3-T4) tumors treated with surgery alone.", "The RPCI tumor registry database reported 104 patients who were treated for buccal carcinoma. A retrospective chart review identified 27 patients who met our criteria for a buccal mucosal primary tumor (epicenter of the mass in the buccal mucosa). There were 13 men and 14 women, ranging in age from 34 to 94 years (mean, 75). Data were collected regarding patient demographics, presenting symptoms, stage, treatment received, and outcome.", "All patients underwent surgical resection of their primary lesion; 21 (75%) had T1 or T2 tumors. The rate of local recurrence was 56% for the group as a whole. Patients with close or positive margins had a 66% local failure rate as compared with 52% when surgical margins were negative (greater than or equal to 5 mm from the resection margin after tissue fixation; P = ns). Among those in whom negative margins were achieved, patients with T1-T2 disease had a 40% local failure rate with surgical resection alone." ], "LABELS": [ "UNLABELLED", "MATERIALS AND METHODS", "RESULTS" ], "MESHES": [ "Adult", "Aged", "Aged, 80 and over", "Biopsy, Needle", "Carcinoma, Squamous Cell", "Cheek", "Disease-Free Survival", "Female", "Humans", "Male", "Middle Aged", "Mouth Mucosa", "Mouth Neoplasms", "Neoplasm Staging", "Predictive Value of Tests", "Probability", "Prognosis", "Registries", "Retrospective Studies", "Sensitivity and Specificity", "Survival Rate" ], "YEAR": null, "reasoning_required_pred": "no", "reasoning_free_pred": "no", "final_decision": "no", "LONG_ANSWER": "Local excision of T1 and T2 buccal mucosa cancers with pathologically negative margins had a high rate of local recurrence in our series. Low T-stage and negative margins are not adequate predictors of local control. Even early buccal tumors may benefit from adjuvant therapy to enhance local control." }, "25070942": { "QUESTION": "Can mass drug administration lead to the sustainable control of schistosomiasis?", "CONTEXTS": [ "In the Philippines, the current national control strategy for schistosomiasis is annual mass drug administration (MDA) with 40 mg/kg of praziquantel in all schistosomiasis-endemic villages with a prevalence \u226510%.", "A cross-sectional survey of schistosomiasis was conducted in 2012 on 18 221 individuals residing in 22 schistosomiasis-endemic villages in the province of Northern Samar. The prevalence of schistosomiasis, intensity of Schistosoma infection, and morbidity of disease were assessed.", "Despite an active schistosomiasis-control program in Northern Samar for>30 years, which included a MDA campaign in the last 5 years, the mean prevalence of schistosomiasis among 10 435 evaluated subjects was 27.1% (95% confidence interval [CI], 26.3%-28.0%), and the geometric mean intensity of infection among 2832 evaluated subjects was 17.2 eggs per gram of feces (95% CI, 16.4-18.1). Ultrasonography revealed high levels of schistosomiasis-induced morbidity in the schistosomiasis-endemic communities. Left lobe liver enlargement (\u226570 mm) was evident in 89.3% of subjects. Twenty-five percent of the study population had grade II/III liver parenchyma fibrosis, and 13.3% had splenomegaly (\u2265100 mm)." ], "LABELS": [ "BACKGROUND", "METHODS", "RESULTS" ], "MESHES": [ "Adolescent", "Adult", "Aged", "Animals", "Anthelmintics", "Child", "Child, Preschool", "Cross-Sectional Studies", "Drug Therapy", "Female", "Health Services Research", "Humans", "Male", "Middle Aged", "Philippines", "Praziquantel", "Prevalence", "Rural Population", "Schistosomiasis", "Young Adult" ], "YEAR": "2015", "reasoning_required_pred": "no", "reasoning_free_pred": "no", "final_decision": "no", "LONG_ANSWER": "MDA on its own was insufficient to control the prevalence of schistosomiasis, intensity of Schistosoma infection, or morbidity of the disease. Alternative control measures will be needed to complement the existing national MDA program." }, "19683101": { "QUESTION": "Can D-dimer become a new diagnostic parameter for acute appendicitis?", "CONTEXTS": [ "In this study, we investigated D-dimer serum level as a diagnostic parameter for acute appendicitis.", "Forty-nine patients were enrolled in the study. Patients were classified according to age; sex; duration between the beginning of pain and referral to a hospital or clinic; Alvarado scores; and in physical examination, presence of muscular defense, the number of leukocytes, preoperative ultrasonography, and D-dimer levels of histopathologic study groups were analyzed.", "Of the patients enrolled in the study, 26.5% were females and 73.5% males. The average age was 21 years (range, 16-38 years) and 81.7% acute appendicitis (AA). According the duration of pain, 63.2% of the patients were referred to the hospital within the first 24 hours, 26.5% of the patients were referred to the hospital within 24 to 48 hours, and 10.3% were referred to the hospital within a period of more than 48 hours. No statistically significant difference was determined regarding D-dimer levels between the histopathologic study groups (P>.05). Alvarado scores lower than 7 were found in 36.7% and 7 or higher in 63.3% of the patients. There was no statistically significant difference related with D-dimer levels between histopathologic study groups (P>.05). The ratio of cases with a number of leukocytes below the upper limit were determined respectively as 32.7% and 67.3%, and no statistically significant difference was found regarding d-dimer levels between histopathologic study groups (P>.05)." ], "LABELS": [ "INTRODUCTION", "MATERIALS AND METHODS", "RESULTS" ], "MESHES": [ "Acute Disease", "Adolescent", "Adult", "Appendicitis", "Female", "Fibrin Fibrinogen Degradation Products", "Humans", "Male", "Retrospective Studies", "Sensitivity and Specificity", "Young Adult" ], "YEAR": "2009", "reasoning_required_pred": "no", "reasoning_free_pred": "no", "final_decision": "no", "LONG_ANSWER": "Increased D-dimer levels should not be considered as a diagnostic parameter in diagnosis of acute appendicitis." }, "10783841": { "QUESTION": "Is there a relationship between rheumatoid arthritis and periodontal disease?", "CONTEXTS": [ "1,412 individuals attending the University of Queensland's School of Dentistry were assessed for the prevalence of periodontal disease and rheumatoid arthritis. Analysis of data obtained from a self-reported health questionnaire and dental records was carried out and included: number of individuals referred for advanced periodontal care (test group); number of individuals attending for routine dentistry; determination of rheumatoid arthritis, cardiovascular disease and diabetes mellitus through self-reporting and assessment of prescription medications; assessment of periodontal disease through assessment of existing oral radiographs.", "In patients referred for periodontal treatment, the prevalence of self-reported rheumatoid arthritis was 3.95% which is significantly higher than that seen in patients not referred for periodontal treatment (0.66%) and also that reported in the general population (1%). Of those referred patients with rheumatoid arthritis, 62.5% had advanced forms of periodontal disease. These results were mirrored in the results of the self-reported prevalence of cardiovascular disease and diabetes mellitus which was consistent with the published higher prevalence in periodontal patients." ], "LABELS": [ "METHODS", "RESULTS" ], "MESHES": [ "Adult", "Aged", "Alveolar Bone Loss", "Arthritis, Rheumatoid", "Cardiovascular Diseases", "Dental Records", "Diabetes Mellitus", "Drug Prescriptions", "Humans", "Middle Aged", "Periodontal Diseases", "Periodontitis", "Prevalence", "Queensland", "Radiography, Dental", "Reproducibility of Results", "Risk Factors", "Surveys and Questionnaires" ], "YEAR": "2000", "reasoning_required_pred": "yes", "reasoning_free_pred": "yes", "final_decision": "yes", "LONG_ANSWER": "Based on data derived from self-reported health conditions, and not withstanding the limitations of such a study, we conclude that there is good evidence to suggest that individuals with moderate to severe periodontal disease are at higher risk of suffering from rheumatoid arthritis and vice versa." }, "11862129": { "QUESTION": "Do clinical variables predict pathologic radiographs in the first episode of wheezing?", "CONTEXTS": [ "To determine if clinical variables assessed in relation to Albuterol aerosol treatments accurately identify children with pathologic radiographs during their initial episode of bronchospasm.", "A prospective convenience sample of children with a first episode of wheezing. Data collected included demographics, baseline and post-treatment clinical score and physical examination, number of aerosols, requirement for supplemental oxygen, and disposition. Chest radiographs were obtained and interpreted, and patients were divided into 2 groups based on a pathologic versus nonpathologic radiograph interpretation. Chi2 testing was performed for categoric variables, and the student t test was performed for continuous variables. A discriminant analysis was used to develop a model.", "Pathologic radiographs were identified in 61 patients (9%). Between groups, a significant difference was noted for pretreatment oxygen saturation only. Clinical score, respiratory rate, and presence of rales both pretreatment and posttreatment were not significantly different between groups. The discriminant analysis correctly predicted 90% of nonpathologic radiographs but only 15% of pathologic radiographs." ], "LABELS": [ "OBJECTIVE", "METHODS", "RESULTS" ], "MESHES": [ "Adolescent", "Albuterol", "Bronchodilator Agents", "Child", "Child, Preschool", "Discriminant Analysis", "Female", "Humans", "Infant", "Male", "Multivariate Analysis", "Physical Examination", "Prospective Studies", "Radiography", "Respiratory Sounds", "Respiratory Tract Diseases", "Sensitivity and Specificity", "Statistics, Nonparametric" ], "YEAR": "2002", "reasoning_required_pred": "no", "reasoning_free_pred": "no", "final_decision": "no", "LONG_ANSWER": "Clinical variables, either isolated or as components of a model, could not identify all children with pathologic radiographs." }, "23455575": { "QUESTION": "Globulomaxillary cysts--do they really exist?", "CONTEXTS": [ "The so-called \"globulomaxillary cyst\", described as a fissural cyst, caused by entrapped epithelium between the nasal and maxillary process, is no longer considered for its own entity. Nevertheless, cystic lesions, which correspond to the previous image of globulomaxillary cysts, do still occur in daily practice. This raises the question to which entities pathological processes in this particular region actually belong to.", "In a retrospective study, 17 cases (12 men and 5 women, 12-59\u00a0years old) of primarily diagnosed globulomaxillary cysts are analysed according to clinical, radiological and histological aspects, catamnestic processed and assigned to a new entity. The results are compared with the international literature and draws conclusions on the diagnostic and therapeutic procedure.", "Seven lateral periodontal cysts, four radicular cysts, two keratocystic odontogenic tumours, one adenomatoid odontogenic tumour, one periapical granuloma, one residual cyst and one undefined jaw cyst were determined." ], "LABELS": [ "OBJECTIVES", "MATERIALS AND METHODS", "RESULTS" ], "MESHES": [ "Adolescent", "Adult", "Child", "Cysts", "Female", "Humans", "Male", "Maxillary Diseases", "Middle Aged", "Retrospective Studies", "Young Adult" ], "YEAR": "2014", "reasoning_required_pred": "yes", "reasoning_free_pred": "no", "final_decision": "no", "LONG_ANSWER": "According to the results of our study and the data from the international literature, the entity globulomaxillary cyst is no longer justified." }, "21848798": { "QUESTION": "MiraLAX vs. Golytely: is there a significant difference in the adenoma detection rate?", "CONTEXTS": [ "In recent clinical trials (RCT) of bowel preparation, Golytely was more efficacious than MiraLAX. We hypothesised that there is a difference in adenoma detection between Golytely and MiraLAX.", "To compare the adenoma detection rate (ADR) between these bowel preparations, and to identify independent predictors of bowel preparation quality and adenoma detection.", "This was a post hoc analysis of an RCT that assessed efficacy and patient tolerability of Golytely vs. MiraLAX/Gatorade in average risk screening colonoscopy patients. Bowel preparation quality was measured with the Boston Bowel Preparation Scale (BBPS). An excellent/good equivalent BBPS score was defined as \u2265 7. Polyp pathology review was performed. ADR was defined as the proportion of colonoscopies with an adenoma. Univariate and multivariate analyses were conducted.", "One hundred and ninety patients were prospectively enrolled (87 MiraLAX, 103 Golytely). Golytely had a higher rate of a BBPS score \u2265 7 (82.5% vs. MiraLAX 67.8%, P=0.02). The ADR in the Golytely cohort was 26.2% (27/103), and was 16.1% (14/87) for MiraLAX (P = 0.091). On multivariate analyses, Golytely was 2.13 \u00d7 more likely to be associated with a BBPS \u2265 7 (95% CI 1.05-4.32, P = 0.04) and 2.28 \u00d7 more likely to be associated with adenoma detection (95% CI 1.05-4.98, P = 0.04) than MiraLAX." ], "LABELS": [ "BACKGROUND", "AIMS", "METHODS", "RESULTS" ], "MESHES": [ "Adenoma", "Age Factors", "Aged", "Colonoscopy", "Colorectal Neoplasms", "Electrolytes", "Female", "Humans", "Male", "Mass Screening", "Middle Aged", "Multivariate Analysis", "Polyethylene Glycols", "Preoperative Care", "Randomized Controlled Trials as Topic", "Retrospective Studies", "Solvents" ], "YEAR": "2011", "reasoning_required_pred": "yes", "reasoning_free_pred": "yes", "final_decision": "yes", "LONG_ANSWER": "Golytely was more efficacious than MiraLAX in bowel cleansing, and was independently associated with both bowel prep quality (BBPS \u2265 7) and higher adenoma detection. Golytely should be used as first line for bowel prep for colonoscopy. Studies with larger populations are needed to confirm these results." }, "25103647": { "QUESTION": "Does government assistance improve utilization of eye care services by low-income individuals?", "CONTEXTS": [ "To examine whether government-funded, low-income vision care programs improve use of eye care services by low-income individuals in Canada.", "Cross-sectional survey.", "27,375 white respondents to the Canadian Community Health Survey (CCHS) Healthy Aging 2008/2009.", "Government-funded, low-income vision care programs were reviewed. The amount of assistance provided was compared with professional fee schedules for general/routine eye examinations and market prices for eyeglasses. The utilization of eye care providers was derived from the CCHS.", "To receive low-income vision care assistance, individuals must be in receipt of social assistance. Criteria for receiving social assistance are stringent. The Canadian Financial Capability Survey revealed that 7.9% of Canadians aged 45 to 64 years and 5.5% aged \u226565 years received social assistance in 2009. The CCHS found in 2008/2009 that 12.5% of citizens aged 45 to 64 years and 13.2% of those aged \u226565 years had difficulty paying for basic expenses such as food. In 5 provinces, low-income vision care assistance fully covers a general/routine eye examination. In the remainder, the assistance provided is insufficient for a general/routine eye examination. The assistance for eyeglasses is inadequate in 5 provinces, requiring out-of-pocket copayments. Among middle-aged whites who self-reported not having glaucoma, cataracts, diabetes, or vision problems not corrected by lenses, utilization of eye care providers was 28.1% among those with financial difficulty versus 41.9% among those without (p<0.05), giving a prevalence ratio 0.68 (95% CI 0.57-0.80) adjusted for age, sex and education." ], "LABELS": [ "OBJECTIVE", "DESIGN", "PARTICIPANTS", "METHODS", "RESULTS" ], "MESHES": [ "Adult", "Aged", "Canada", "Cross-Sectional Studies", "Female", "Government Programs", "Health Care Surveys", "Health Services", "Health Services Accessibility", "Health Services Research", "Humans", "Male", "Medical Assistance", "Middle Aged", "National Health Programs", "Ophthalmology", "Optometry", "Poverty", "Young Adult" ], "YEAR": "2014", "reasoning_required_pred": "maybe", "reasoning_free_pred": "no", "final_decision": "maybe", "LONG_ANSWER": "Despite government assistance, low-income individuals use vision care services less often than wealthy individuals." }, "19406119": { "QUESTION": "Does telmisartan prevent hepatic fibrosis in rats with alloxan-induced diabetes?", "CONTEXTS": [ "This study evaluated the effect of telmisartan on the livers of diabetic rats and also aimed to determine the hepatic distribution and role of transforming growth factor beta (TGF-beta) in diabetes-related hepatic degeneration while taking into account the possible protective effects of telmisartan.", "Fifteen adult male rats were used and divided into three groups: the non-diabetic healthy group, alloxan-induced diabetic control group, and the alloxan-induced diabetic telmisartan group. The non-diabetic healthy group and the diabetic control group were exposed to saline for 30 days, while the group treated with diabetic drugs was orally administered telmisartan for 30 days (10 mg/kg/day). At the end of the experiment, the rats were sacrificed and the livers were dissected and transferred into the fixation solution. The livers were then evaluated using stereological and histopathological methods.", "Our study of the numerical density of hepatocytes shows a significant difference between the diabetic control group and diabetic rats treated with telmisartan. Immunohistochemical staining for TGF-beta in liver sections of the diabetic rats treated with telmisartan showed no immunoreactivity. The diabetic control group was determined to be strongly immunoreactive to TGF-beta." ], "LABELS": [ "AIMS", "METHODS", "RESULTS" ], "MESHES": [ "Alloxan", "Angiotensin II Type 1 Receptor Blockers", "Animals", "Benzimidazoles", "Benzoates", "Diabetes Mellitus, Experimental", "Fibrosis", "Immunohistochemistry", "Liver", "Male", "Microscopy, Electron", "Rats", "Rats, Wistar", "Transforming Growth Factor beta" ], "YEAR": "2009", "reasoning_required_pred": "yes", "reasoning_free_pred": "yes", "final_decision": "yes", "LONG_ANSWER": "Results suggest that telmisartan may reduce type-I diabetes mellitus-induced hepatic injury by suppressing activated hepatic stellate cells through concomitant TGF-beta1 down-regulation." }, "24359102": { "QUESTION": "Two-year follow-up survey of patients with allergic contact dermatitis from an occupational cohort: is the prognosis dependent on the omnipresence of the allergen?", "CONTEXTS": [ "Skin diseases are the most frequently recognized occupational diseases in Denmark. The prognosis for occupational contact dermatitis is often poor.", "To investigate the prognosis, assessed by eczema, job status and skin-related quality of life, among patients allergic to rubber chemicals and latex (ubiquitous allergens) and epoxy (nonubiquitous allergen), 2\u00a0years after recognition of occupational allergic contact dermatitis.", "From a cohort of all patients recognized as having occupational dermatitis by the Danish National Board of Industrial Injuries in 2010, 199 patients with relevant rubber allergy (contact allergy to rubber chemicals or contact urticaria from latex) or epoxy allergy were identified. Follow-up consisted of a questionnaire covering current severity of eczema, employment, exposure and quality of life.", "The response rate was 75%. Clearance of eczema was reported by 11% of patients and 67% reported improvement. Overall 22% of patients with allergy to a nonubiquitous allergen had total clearance of eczema compared with 10% of cases allergic to ubiquitous allergens and 0% of those with contact urticaria (P\u00a0=\u00a00\u00b7116). Improvement was significantly more frequent in those who had changed jobs compared with those who had not (P\u00a0=\u00a00\u00b701)." ], "LABELS": [ "BACKGROUND", "OBJECTIVES", "METHODS", "RESULTS" ], "MESHES": [ "Adolescent", "Adult", "Aged", "Allergens", "Denmark", "Dermatitis, Allergic Contact", "Dermatitis, Occupational", "Eczema", "Employment", "Female", "Follow-Up Studies", "Humans", "Latex Hypersensitivity", "Male", "Middle Aged", "Prognosis", "Quality of Life", "Urticaria", "Young Adult" ], "YEAR": "2014", "reasoning_required_pred": "yes", "reasoning_free_pred": "yes", "final_decision": "yes", "LONG_ANSWER": "At follow-up, patients with contact urticaria had significantly poorer prognosis than those with contact allergy, and there was a trend towards a poorer prognosis for those with contact allergy to rubber chemicals than those with allergy to epoxy. A significant positive association between job change and improvement was found." }, "18802997": { "QUESTION": "Can calprotectin predict relapse risk in inflammatory bowel disease?", "CONTEXTS": [ "Assessing the clinical course of inflammatory bowel disease (IBD) patients consists of periodical clinical evaluations and laboratory tests. We aimed to assess the role of calprotectin tests in predicting clinical relapse in IBD patients.", "Ninety-seven patients with ulcerative colitis (UC) and 65 with Crohn's disease (CD) in clinical remission were prospectively included in the study. A 10-g stool sample was collected for calprotectin assay. The cutoff level was set at 130 mg/kg of feces. Patients were followed up for 1 yr after the test or until relapse. The cumulative proportion of relapses was estimated by the Kaplan-Meier analysis. Statistics for equality of survival distribution were tested using the log-rank test.", "The calprotectin test was positive in 44 UC patients and 26 of them relapsed within a year, while 11 of 53 UC patients with a negative calprotectin test relapsed within the same time frame. Thirty CD patients had a positive calprotectin test and 13 of them relapsed within a year, as did 7 of the 35 with a negative test result. A significant correlation emerged between a positive calprotectin test and the probability of relapse in UC patients (P= 0.000). In CD patients, only cases of colonic CD showed a significant correlation between a positive calprotectin test and the probability of relapse, i.e., 6 colonic CD patients were positive for the calprotectin test and 4 relapsed (P= 0.02)." ], "LABELS": [ "OBJECTIVE", "METHODS", "RESULTS" ], "MESHES": [ "Adolescent", "Adult", "Aged", "Aged, 80 and over", "Colitis, Ulcerative", "Crohn Disease", "Disease-Free Survival", "Feces", "Female", "Follow-Up Studies", "Humans", "Leukocyte L1 Antigen Complex", "Male", "Middle Aged", "Predictive Value of Tests", "Prospective Studies", "ROC Curve", "Recurrence" ], "YEAR": "2008", "reasoning_required_pred": "maybe", "reasoning_free_pred": "yes", "final_decision": "maybe", "LONG_ANSWER": "Measuring calprotectin may help to identify UC and colonic CD patients at higher risk of clinical relapse." }, "26304701": { "QUESTION": "Can nurse-led preoperative education reduce anxiety and postoperative complications of patients undergoing cardiac surgery?", "CONTEXTS": [ "The effect of preoperative education on anxiety and postoperative outcomes of cardiac surgery patients remains unclear.AIM: The aim of the study was to estimate the effectiveness of a nurse-led preoperative education on anxiety and postoperative outcomes.", "A randomised controlled study was designed. All the patients who were admitted for elective cardiac surgery in a general hospital in Athens with knowledge of the Greek language were eligible to take part in the study. Patients in the intervention group received preoperative education by specially trained nurses. The control group received the standard information by the ward personnel. Measurements of anxiety were conducted on admission-A, before surgery-B and before discharge-C by the state-trait anxiety inventory.", "The sample consisted of 395 patients (intervention group: 205, control group: 190). The state anxiety on the day before surgery decreased only in the intervention group (34.0 (8.4) versus 36.9 (10.7); P=0.001). The mean decrease in state score during the follow-up period was greater in the intervention group (P=0.001). No significant difference was found in the length of stay or readmission. Lower proportions of chest infection were found in the intervention group (10 (5.3) versus 1 (0.5); P=0.004). Multivariate linear regression revealed that education and score in trait anxiety scale on admission are independent predictors of a reduction in state anxiety." ], "LABELS": [ "BACKGROUND", "METHODS", "RESULTS" ], "MESHES": [ "Adult", "Aged", "Aged, 80 and over", "Anxiety Disorders", "Cardiac Surgical Procedures", "Female", "Humans", "Male", "Middle Aged", "Nurse-Patient Relations", "Patient Education as Topic", "Postoperative Complications", "Preoperative Care", "Preoperative Period" ], "YEAR": "2016", "reasoning_required_pred": "yes", "reasoning_free_pred": "yes", "final_decision": "yes", "LONG_ANSWER": "Preoperative education delivered by nurses reduced anxiety and postoperative complications of patients undergoing cardiac surgery, but it was not effective in reducing readmissions or length of stay." }, "19058191": { "QUESTION": "Is there a discrepancy between patient and physician quality of life assessment?", "CONTEXTS": [ "Quality of Life (QoL) assessment remains integral in the investigation of women with lower urinary tract dysfunction. Previous work suggests that physicians tend to underestimate patients' symptoms and the bother that they cause. The aim of this study was to assess the relationship between physician and patient assessed QoL using the Kings Health Questionnaire (KHQ).", "Patients complaining of troublesome lower urinary tract symptoms (LUTS) were recruited from a tertiary referral urodynamic clinic. Prior to their clinic appointment they were sent a KHQ, which was completed before attending. After taking a detailed urogynecological history, a second KHQ was filled in by the physician, blinded to the patient responses, on the basis of their impression of the symptoms elicited during the interview. These data were analyzed by an independent statistician. Concordance between patient and physician assessment for individual questions was assessed using weighted kappa analysis. QoL scores were compared using Wilcoxons signed rank test.", "Seventy-five patients were recruited over a period of 5 months. Overall, the weighted kappa showed relatively poor concordance between the patient and physician responses; mean kappa: 0.33 (range 0.18-0.57). The physician underestimated QoL score in 4/9 domains by a mean of 5.5% and overestimated QoL score in 5/9 domains by a mean of 6.9%. In particular, physicians underestimated the impact of LUTS on social limitations and emotions (P<0.05)." ], "LABELS": [ "AIMS", "METHODS", "RESULTS" ], "MESHES": [ "Adult", "Aged", "Data Interpretation, Statistical", "Female", "Humans", "Middle Aged", "Patients", "Physicians", "Prospective Studies", "Quality of Life", "Sample Size", "Surveys and Questionnaires", "Urologic Diseases", "Young Adult" ], "YEAR": "2009", "reasoning_required_pred": "yes", "reasoning_free_pred": "yes", "final_decision": "yes", "LONG_ANSWER": "This study confirms that physicians often differ from patients in the assessment of QoL. This is most likely due to a difference in patient-physician perception of \"significant\" LUTS and clearly demonstrates the importance of patient evaluated QoL in routine clinical assessment." }, "27549226": { "QUESTION": "Impact of MPH programs: contributing to health system strengthening in low- and middle-income countries?", "CONTEXTS": [ "The \"health workforce\" crisis has led to an increased interest in health professional education, including MPH programs. Recently, it was questioned whether training of mid- to higher level cadres in public health prepared graduates with competencies to strengthen health systems in low- and middle-income countries. Measuring educational impact has been notoriously difficult; therefore, innovative methods for measuring the outcome and impact of MPH programs were sought. Impact was conceptualized as \"impact on workplace\" and \"impact on society,\" which entailed studying how these competencies were enacted and to what effect within the context of the graduates' workplaces, as well as on societal health.", "This is part of a larger six-country mixed method study; in this paper, the focus is on the qualitative findings of two English language programs, one a distance MPH program offered from South Africa, the other a residential program in the Netherlands. Both offer MPH training to students from a diversity of countries. In-depth interviews were conducted with 10 graduates (per program), working in low- and middle-income health systems, their peers, and their supervisors.", "Impact on the workplace was reported as considerable by graduates and peers as well as supervisors and included changes in management and leadership: promotion to a leadership position as well as expanded or revitalized management roles were reported by many participants. The development of leadership capacity was highly valued amongst many graduates, and this capacity was cited by a number of supervisors and peers. Wider impact in the workplace took the form of introducing workplace innovations such as setting up an AIDS and addiction research center and research involvement; teaching and training, advocacy, and community engagement were other ways in which graduates' influence reached a wider target grouping. Beyond the workplace, an intersectoral approach, national reach through policy advisory roles to Ministries of Health, policy development, and capacity building, was reported. Work conditions and context influenced conduciveness for innovation and the extent to which graduates were able to have effect. Self-selection of graduates and their role in selecting peers and supervisors may have resulted in some bias, some graduates could not be traced, and social acceptability bias may have influenced findings." ], "LABELS": [ "BACKGROUND", "METHODS", "RESULTS" ], "MESHES": [ "Attitude of Health Personnel", "Delivery of Health Care", "Developing Countries", "Education, Distance", "Education, Graduate", "Education, Public Health Professional", "Humans", "Income", "Internet", "Leadership", "Netherlands", "Professional Role", "Public Health", "Public Health Practice", "Qualitative Research", "South Africa", "Workplace" ], "YEAR": "2016", "reasoning_required_pred": "yes", "reasoning_free_pred": "yes", "final_decision": "yes", "LONG_ANSWER": "There was considerable impact at many levels; graduates were perceived to be able to contribute significantly to their workplaces and often had influence at the national level. Much of the impact described was in line with public health educational aims. The qualitative method study revealed more in-depth understanding of graduates' impact as well as their career pathways." }, "24799031": { "QUESTION": "Is diffusion-weighted imaging a significant indicator of the development of vascularization in hypovascular hepatocellular lesions?", "CONTEXTS": [ "The objective was to evaluate the efficacy of diffusion-weighted imaging (DWI) in predicting the development of vascularization in hypovascular hepatocellular lesions (HHLs).", "Forty-two HHLs that were diagnosed by computed tomographic (CT) arteriography were evaluated retrospectively. The lesion on DWI was classified as isointense, hypointense, or hyperintense. Follow-up studies that included intravenous dynamic CT or magnetic resonance imaging were performed.", "The 730-day cumulative developments of vascularization in hypointense, isointense, and hyperintense lesions were 17%, 30%, and 40%, respectively. The differences among these developments were not statistically significant." ], "LABELS": [ "OBJECTIVE", "MATERIALS AND METHODS", "RESULTS" ], "MESHES": [ "Aged", "Aged, 80 and over", "Carcinoma, Hepatocellular", "Contrast Media", "Diffusion Magnetic Resonance Imaging", "Female", "Gadolinium DTPA", "Hepatitis B, Chronic", "Hepatitis C, Chronic", "Humans", "Liver Diseases, Alcoholic", "Liver Neoplasms", "Male", "Middle Aged", "Neovascularization, Pathologic", "Non-alcoholic Fatty Liver Disease", "Reproducibility of Results", "Retrospective Studies", "Tomography, X-Ray Computed" ], "YEAR": null, "reasoning_required_pred": "no", "reasoning_free_pred": "no", "final_decision": "no", "LONG_ANSWER": "The signal intensity on DWI showed no significant difference in the development of vascularization." }, "25043083": { "QUESTION": "Are failures of anterior cruciate ligament reconstruction associated with steep posterior tibial slopes?", "CONTEXTS": [ "Recently, increasing number of literature has identified the posterior tibial slope (PTS) as one of the risk factors of primary anterior cruciate ligament (ACL) injury. However, few studies concerning the association between failure of ACL reconstruction (ACLR) and PTS have been published. The objective of this study was to explore the association between the failure of ACLR and PTS at a minimum of two years follow-up.", "Two hundred and thirty eight eligible patients from June 2009 to October 2010 were identified from our database. A total of 20 failure cases of ACLR and 20 randomly selected controls were included in this retrospective study. The demographic data and the results of manual maximum side-to-side difference with KT-1000 arthrometer at 30\u00b0 of knee flexion and pivot-shift test before the ACLR and at the final follow-up were collected. The medial and lateral PTSs were measured using the magnetic resonance imaging (MRI) scan, based on Hudek's measurement. A comparison of PTS between the two groups was performed.", "The overall failure rate of the present study was 8.4%. Of the 40 participants, the mean medial PTS was 4.1\u00b0 \u00b1 3.2\u00b0 and the mean lateral PTS was 4.6\u00b0 \u00b1 2.6\u00b0. The medial PTS of the ACLR failure group was significantly steeper than the control group (3.5\u00b0 \u00b1 2.5\u00b0 vs. 6.1\u00b0 \u00b1 2.1\u00b0, P = 0.000). Similarly, the lateral PTS of the ACLR failure group was significantly steeper than the control group (2.9\u00b0 \u00b1 2.1\u00b0 vs. 5.5\u00b0 \u00b1 3.0\u00b0, P = 0.006). For medial PTS \u2265 5\u00b0, the odds ratio of ACLR failure was 6.8 (P = 0.007); for lateral PTS \u22655\u00b0, the odds ratio of ACLR failure was 10.8 (P = 0.000)." ], "LABELS": [ "BACKGROUND", "METHODS", "RESULTS" ], "MESHES": [ "Adolescent", "Adult", "Anterior Cruciate Ligament", "Anterior Cruciate Ligament Injuries", "Anterior Cruciate Ligament Reconstruction", "Case-Control Studies", "Female", "Humans", "Male", "Retrospective Studies", "Tibia", "Young Adult" ], "YEAR": "2014", "reasoning_required_pred": "yes", "reasoning_free_pred": "yes", "final_decision": "yes", "LONG_ANSWER": "Both medial and lateral PTS were significantly steeper in failures of ACLR than the control group. Medial or lateral PTS \u22655\u00b0 was a new risk factor of ACLR failure." }, "26516021": { "QUESTION": "Does evidence-based practice improve patient outcomes?", "CONTEXTS": [ "Evidence-based practice (EBP) is widely promoted, but does EBP produce better patient outcomes? We report a natural experiment when part of the internal medicine service in a hospital was reorganized in 2003 to form an EBP unit, the rest of the service remaining unchanged. The units attended similar patients until 2012 permitting comparisons of outcomes and activity.", "We used routinely collected statistics (2004-11) to compare the two different methods of practice and test whether patients being seen by the EBP unit differed from standard practice (SP) patients. Data were available by doctor and year. To check for differences between the EBP and SP doctors prior to reorganization, we used statistics from 2000 to 2003. We looked for changes in patient outcomes or activity following reorganization and whether the EBP unit was achieving significantly different results from SP. Data across the periods were combined and tested using Mann-Whitney test.", "No statistically significant differences in outcomes were detected between the EBP and the SP doctors prior to reorganization. Following the unit's establishment, the mortality of patients being treated by EBP doctors compared with their previous performance dropped from 7.4% to 6.3% (P\u2009<\u20090.02) and length of stay from 9.15 to 6.01 days (P\u2009=\u20090.002). No statistically significant improvements were seen in SP physicians' performance. No differences in the proportion of patients admitted or their complexity between the services were detected. Despite this, EBP patients had a clinically significantly lower risk of death 6.27% versus 7.75% (P\u2009<\u20090.001) and a shorter length of stay 6.01 versus 8.46 days (P\u2009<\u20090.001) than SP patients. Readmission rates were similar: 14.4% (EBP); 14.5% (SP). EBP doctors attended twice as many patients/doctor as SP doctors." ], "LABELS": [ "RATIONALE, AIMS AND OBJECTIVES", "METHODS", "RESULTS" ], "MESHES": [ "Evidence-Based Medicine", "Hospital Administration", "Hospital Bed Capacity", "Humans", "Internal Medicine", "Length of Stay", "Patient Readmission", "Practice Patterns, Physicians'", "Spain", "Treatment Outcome" ], "YEAR": "2015", "reasoning_required_pred": "yes", "reasoning_free_pred": "yes", "final_decision": "yes", "LONG_ANSWER": "The EBP unit was associated with better patient outcomes and more efficient performance than achieved by the same physicians previously or by SP concurrently." }, "23736032": { "QUESTION": "Multidisciplinary decisions in breast cancer: does the patient receive what the team has recommended?", "CONTEXTS": [ "A multidisciplinary team (MDT) approach to breast cancer management is the gold standard. The aim is to evaluate MDT decision making in a modern breast unit.", "All referrals to the breast MDT where breast cancer was diagnosed from 1 July 2009 to 30 June 2011 were included. Multidisciplinary team decisions were compared with subsequent patient management and classified as concordant or discordant.", "Over the study period, there were 3230 MDT decisions relating to 705 patients. Overall, 91.5% (2956 out of 3230) of decisions were concordant, 4.5% (146 out of 3230), were discordant and 4% (128 out of 3230) had no MDT decision. Of 146 discordant decisions, 26 (17.8%) were considered 'unjustifiable' as there was no additional information available after the MDT to account for the change in management. The remaining 120 discordant MDT decisions were considered 'justifiable', as management was altered due to patient choice (n=61), additional information available after MDT (n=54) or MDT error (n=5)." ], "LABELS": [ "BACKGROUND", "METHODS", "RESULTS" ], "MESHES": [ "Breast Neoplasms", "Carcinoma", "Choice Behavior", "Decision Making", "Female", "Guideline Adherence", "Humans", "Interdisciplinary Communication", "Medical Errors", "Patient Access to Records", "Patient Care Team", "Patient Compliance", "Patient Education as Topic", "Physician-Patient Relations", "Referral and Consultation", "Retrospective Studies" ], "YEAR": "2013", "reasoning_required_pred": "yes", "reasoning_free_pred": "yes", "final_decision": "yes", "LONG_ANSWER": "The vast majority of MDT decisions are implemented. Management alteration was most often due to patient choice or additional information available after the MDT. A minority of management alterations were 'unjustifiable' and the authors recommend that any patient whose treatment is subsequently changed should have MDT rediscussion prior to treatment." }, "17312514": { "QUESTION": "Does quilting suture prevent seroma in abdominoplasty?", "CONTEXTS": [ "Seroma is the most frequent complication in abdominoplasty. Some patients are more prone to develop this complication. Ultrasound is a well-known method with which to diagnose seroma in the abdominal wall. The purpose of this study was to verify the efficacy of the use of quilting suture to prevent seroma.", "Twenty-one female patients who presented with abdominal deformity type III/A according to the authors' classification of abdominal skin and myoaponeurotic deformity had undergone abdominoplasty. The selected patients should have had at least one of the following characteristics: body mass index greater than 25 kg/m; weight loss greater than 10 kg; previous incision in the supraumbilical region; or present thinning of the subcutaneous in the area above the umbilicus. Ultrasound was performed for every patient from 15 to 18 days after the operation to search for fluid collection in the abdominal wall.", "The average fluid collection found was 8.2 cc per patient. Only two patients underwent aspiration because ultrasound showed greater than 20 cc collected above the fascial layer. These patients did not present with recurrence of seroma after aspiration." ], "LABELS": [ "BACKGROUND", "METHODS", "RESULTS" ], "MESHES": [ "Abdominal Wall", "Cosmetic Techniques", "Female", "Humans", "Postoperative Complications", "Seroma", "Suture Techniques", "Ultrasonography" ], "YEAR": "2007", "reasoning_required_pred": "yes", "reasoning_free_pred": "yes", "final_decision": "yes", "LONG_ANSWER": "The quilting suture seems to be an efficient technique with which to prevent seroma formation." }, "23002947": { "QUESTION": "Does feeding tube insertion and its timing improve survival?", "CONTEXTS": [ "To examine survival with and without a percutaneous endoscopic gastrostomy (PEG) feeding tube using rigorous methods to account for selection bias and to examine whether the timing of feeding tube insertion affected survival.", "Prospective cohort study.", "All U.S. nursing homes (NHs).", "Thirty-six thousand four hundred ninety-two NH residents with advanced cognitive impairment from dementia and new problems eating studied between 1999 and 2007.", "Survival after development of the need for eating assistance and feeding tube insertion.", "Of the 36,492 NH residents (88.4% white, mean age 84.9, 87.4% with one feeding tube risk factor), 1,957 (5.4%) had a feeding tube inserted within 1\u00a0year of developing eating problems. After multivariate analysis correcting for selection bias with propensity score weights, no difference was found in survival between the two groups (adjusted hazard ratio (AHR)\u00a0=\u00a01.03, 95% confidence interval (CI)\u00a0=\u00a00.94-1.13). In residents who were tube-fed, the timing of PEG tube insertion relative to the onset of eating problems was not associated with survival after feeding tube insertion (AHR\u00a0=\u00a01.01, 95% CI\u00a0=\u00a00.86-1.20, persons with a PEG tube inserted within 1\u00a0month of developing an eating problem versus later (4\u00a0months) insertion)." ], "LABELS": [ "OBJECTIVES", "DESIGN", "SETTING", "PARTICIPANTS", "MEASUREMENTS", "RESULTS" ], "MESHES": [ "Aged, 80 and over", "Female", "Gastrostomy", "Humans", "Intubation, Gastrointestinal", "Male", "Prospective Studies", "Survival Rate", "Time Factors" ], "YEAR": "2012", "reasoning_required_pred": "no", "reasoning_free_pred": "no", "final_decision": "no", "LONG_ANSWER": "Neither insertion of PEG tubes nor timing of insertion affect survival." }, "23453079": { "QUESTION": "Does prostate morphology affect outcomes after holmium laser enucleation?", "CONTEXTS": [ "To determine whether prostate morphology or technique used has any effect on postoperative outcomes after holmium laser enucleation of the prostate.", "A retrospective review of prospectively collected data was completed for all patients undergoing a holmium laser enucleation of the prostate at our institution. Prostate morphology was classified as either \"bilobar\" or \"trilobar\" according to the cystoscopic appearance. The baseline characteristics, complications, and postoperative outcomes were collected.", "A total of 304 patients with either \"bilobar\" (n\u00a0= 142) or \"trilobar\" (n\u00a0= 162) prostate morphology were included. The trilobar group was more likely to have longer operative times (112 vs 100 minutes, P\u00a0= .04), although this difference was not significant on multivariate analysis. The postoperative outcomes were similar between the 2 groups for American Urological Association symptom score, change in American Urological Association symptom score, bother score, maximal flow rate, change in maximal flow rate, postvoid residual urine volume, and complication rate. However, the trilobar group had a significantly greater decrease in their PVR urine volume (296 vs 176 mL, P\u00a0= .01), a difference that persisted on multivariate analysis. A subset analysis of the trilobar prostates revealed that performing a 2-lobe technique achieved shorter operative and enucleation times, although the difference was not significant." ], "LABELS": [ "OBJECTIVE", "MATERIALS AND METHODS", "RESULTS" ], "MESHES": [ "Aged", "Humans", "Laser Therapy", "Lasers, Solid-State", "Male", "Middle Aged", "Prostate", "Prostatectomy", "Prostatic Hyperplasia", "Retrospective Studies", "Treatment Outcome" ], "YEAR": "2013", "reasoning_required_pred": "yes", "reasoning_free_pred": "yes", "final_decision": "yes", "LONG_ANSWER": "Those patients with trilobar prostate morphology are more likely to achieve a greater decrease in the PVR urine volume after holmium laser enucleation of the prostate. All other outcomes appeared to be similar between the 2 groups. In addition, a 2-lobe technique can be safely used for trilobar prostates when deemed feasible by the surgeon." }, "15053041": { "QUESTION": "Do acute changes in heart rate by isoproterenol affect aortic stiffness in patients with hypertension?", "CONTEXTS": [ "Increased aortic stiffness is a independent risk factor of cardiovascular disease in patients with hypertension. Acute changes of the heart rate (HR) have been reported not to affect the aortic stiffness in pacing. However, it is unknown whether acute changes in HR caused by sympathomimetics can affect the aortic stiffness in patients with hypertension. We investigated the effect of acute changes in HR produced by isoproterenol on the aortic stiffness in 17 hypertensive patientss (mean age: 59 +/- 9 years).", "All vasoactive drugs were discontinued at least 3 days before the study. The carotid-to-femoral pulse wave velocity (PWV) was measured by the foot-to-foot method. The pulse waves were recorded at the baseline and at every increase of HR by 5 to 10 bpm with a gradual increase of the dose of isoproterenol. The blood pressures and HR were measured simultaneously. For the analysis, HR, PWV, compliance (C), and compliance index (Ci) were converted as percent changes (delta) from the baseline values. Percent changes of the parameters of the aortic stiffness, i.e., deltaPWV, deltaC, and deltaCi, were grouped by every 10% increase in deltaHR.", "There was no significant difference among groups in deltaPWV, deltaC and deltaCi (p>0.05 for each of the group). The regression analysis showed no significant correlation of deltaHR with deltaPWV and deltaC (r = 0.18, 0.13 respectively, p>0.05 for each). deltaCi had a poor correlation with deltaHR (r = 0.22, p<0.05). However, only 4.6% of deltaCi could be referred to deltaHR (r2 = 0.046)." ], "LABELS": [ "BACKGROUND", "METHODS", "RESULTS" ], "MESHES": [ "Aged", "Aorta", "Chest Pain", "Coronary Disease", "Elasticity", "Female", "Heart Rate", "Humans", "Hypertension", "Isoproterenol", "Linear Models", "Male", "Middle Aged", "Pulsatile Flow", "Sympathomimetics" ], "YEAR": "2004", "reasoning_required_pred": "no", "reasoning_free_pred": "no", "final_decision": "no", "LONG_ANSWER": "Aortic stiffness was not affected by acute changes in HR produced by isoproterenol which suggests that it is not necessary to consider acute changes in HR when measuring aortic PWV." }, "9465206": { "QUESTION": "\"Occult\" posttraumatic lesions of the knee: can magnetic resonance substitute for diagnostic arthroscopy?", "CONTEXTS": [ "We investigated the actual role of MRI versus arthroscopy in the detection and characterization of occult bone and/or cartilage injuries in patients with previous musculoskeletal trauma of the knee, pain and severe functional impairment. Occult post-traumatic osteochondral injuries of the knee are trauma-related bone and/or cartilage damage missed at plain radiography.", "We retrospectively selected 70 patients (men:women = 7:3; age range: 35 +/- 7 years) with a history of acute musculoskeletal trauma, negative conventional radiographs, pain and limited joint movements. All patients were submitted to conventional radiography, arthroscopy and MRI, the latter with 0.5 T units and T1-weighted SE. T2-weighted GE and FIR sequences with fat suppression.", "We identified three types of occult post-traumatic injuries by morpho-topographic and signal intensity patterns: bone bruises (no. 25), subchondral (no. 33) and osteochondral (no. 35) injuries. Arthroscopy depicted 45 osteochondral and 19 chondral injuries. A bone bruise was defined as a typical subcortical area of signal loss, with various shapes, on T1-weighted images and of increased signal intensity on T2-weighted and FIR images. The cortical bone and articular cartilage were normal in all cases, while osteochondral injuries exhibited associated bone and cartilage damage with the same abnormal MR signal intensity. Sprain was the mechanism of injury in 52 cases, bruise in 12 and stress in 6. In 52 sprains (30 in valgus), the injury site was the lateral compartment in 92.3% of cases (100% in valgus), associated with meniscal damage in 73% of cases (90% in valgus) and with ligament injury in 90.4% (100% in valgus). In 12 bruises, the injury site was the lateral compartment in 58.3% of cases, the knee cap in 25% and the medial compartment in 16.7%; meniscal damage was associated in 25% of cases and ligament damage in 8.3%. In 6 stress injuries, the injury site was localized in the medial tibial condyle in 80% of cases, while meniscal and ligament tears were absent." ], "LABELS": [ "PURPOSE", "MATERIAL AND METHODS", "RESULTS AND DISCUSSION" ], "MESHES": [ "Adult", "Arthroscopy", "Cartilage, Articular", "Contusions", "Female", "Fractures, Bone", "Humans", "Knee Injuries", "Knee Joint", "Ligaments, Articular", "Magnetic Resonance Imaging", "Male", "Menisci, Tibial", "Radiography", "Retrospective Studies", "Sensitivity and Specificity", "Tibial Meniscus Injuries" ], "YEAR": "1997", "reasoning_required_pred": "yes", "reasoning_free_pred": "maybe", "final_decision": "yes", "LONG_ANSWER": "After comparing MR with arthroscopic findings and reviewing the available literature, we conclude that arthroscopy permits the direct visualization of even fine articular surface changes but does not depict the subchondral bone, the most frequent site of injury detected with MRI. MRI was a very useful tool in the detection and characterization of the different types of occult bone and/or cartilage injuries and showed a statistically significant correlation between site and distribution of bone and cartilage injuries and between internal damage and trauma mechanisms. Therefore, we believe that MRI can help avoid diagnostic arthroscopy in the patients with a history of post-traumatic pain, acute articular blocks and negative radiographic findings." }, "19593710": { "QUESTION": "Could ESC (Electronic Stability Control) change the way we drive?", "CONTEXTS": [ "ESC (Electronic Stability Control) is a crash avoidance technology that reduces the likelihood of collisions involving loss of control. Although past and emerging research indicates that ESC is effective in reducing collision rates and saving lives, and its inclusion in all vehicle platforms is encouraged, drivers may demonstrate behavioral adaptation or an overreliance on ESC that could offset or reduce its overall effectiveness. The main objective of the present study was to determine whether behavioral adaptation to ESC is likely to occur upon the widespread introduction of ESC into the Canadian vehicle fleet. Secondary objectives were to confirm the results of a previous ESC public survey and to generate a baseline measure for the future assessment of planned and ongoing ESC promotional activities in Canada.", "Two separate telephone surveys evaluated drivers' perceptions and awareness of ESC. The first surveyed 500 randomly selected owners/drivers of passenger vehicles. The second surveyed 1017 owners/drivers of 2006-2008 ESC-equipped passenger vehicles from the provinces of Quebec and British Columbia, Canada.", "Though ESC drivers were much more likely than drivers of other vehicles to be aware of ESC (77% vs. 39%) and that their own vehicle was equipped with it (63% vs. 8%), 23 percent had never heard of it. Ninety percent of drivers who knew that their vehicle was equipped with ESC believed that ESC had made it safer to drive and reported being confident that ESC would work in an emergency. Twenty-three percent of ESC owners who knew their vehicle had ESC reported noticing long-lasting changes in their driving behavior since they began driving the vehicle." ], "LABELS": [ "OBJECTIVE", "METHODS", "RESULTS" ], "MESHES": [ "Accidents, Traffic", "Adolescent", "Age Factors", "Automobile Driving", "Automobiles", "Awareness", "Behavior", "Canada", "Data Collection", "Educational Status", "Female", "Humans", "Interviews as Topic", "Logistic Models", "Male", "Protective Devices", "Public Opinion", "Risk-Taking", "Sex Factors" ], "YEAR": "2009", "reasoning_required_pred": "yes", "reasoning_free_pred": "maybe", "final_decision": "yes", "LONG_ANSWER": "Collectively, results suggest that behavioral adaptation to ESC is likely in certain drivers; however, its proven effectiveness in reducing the likelihood of being involved in a serious crash probably outweighs any potential increases in unsafe driving. To fully benefit from ESC, vehicle manufacturers are encouraged to market ESC-equipped vehicles in a realistic, safe manner. Driver training and safety organizations are also encouraged to provide balanced educational information about ESC to their members." }, "8521557": { "QUESTION": "The insertion allele of the ACE gene I/D polymorphism. A candidate gene for insulin resistance?", "CONTEXTS": [ "The insertion/deletion (ID) polymorphism of the angiotensin-converting enzyme (ACE) gene has been associated with increased coronary heart disease (CHD), although the mechanism of this association is not apparent. We tested the hypothesis that the deletion allele of the ACE gene is associated with insulin resistance.", "We related ACE genotype to components of the insulin-resistance syndrome in 103 non-insulin-dependent diabetic (NIDDM) and 533 nondiabetic white subjects. NIDDM subjects with the DD genotype had significantly lower levels of specific insulin (DD 38.6, ID 57.1, and II 87.4 pmol.L-1 by ANOVA, P = .011). Non-insulin-treated subjects with the DD genotype had increased insulin sensitivity by HOMA % (DD 56.4%, II 29.4%, P = .027) and lower levels of des 31,32 proinsulin (DD 3.3, II 7.6 pmol.L-1, P = .012) compared with II subjects. There were no differences in prevalence of CHD or levels of blood pressure, serum lipids, or plasminogen activator inhibitor-1 (PAI-1) activity between the three ACE genotypes. In nondiabetic subjects there were no differences in insulin sensitivity, levels of insulin-like molecules, blood pressure, PAI-1, serum lipids, or CHD prevalence between the three ACE genotypes." ], "LABELS": [ "BACKGROUND", "METHODS AND RESULTS" ], "MESHES": [ "Alleles", "Case-Control Studies", "Coronary Disease", "DNA Transposable Elements", "Diabetes Mellitus, Type 2", "Diabetic Angiopathies", "Female", "Genotype", "Humans", "Insulin Resistance", "Male", "Middle Aged", "Peptidyl-Dipeptidase A", "Plasminogen Activator Inhibitor 1", "Polymerase Chain Reaction", "Polymorphism, Genetic" ], "YEAR": "1995", "reasoning_required_pred": "yes", "reasoning_free_pred": "no", "final_decision": "no", "LONG_ANSWER": "We conclude that increased cardiovascular risk of the DD genotype is not mediated through insulin resistance or abnormalities in fibrinolysis. Conversely, we report an increased sensitivity in NIDDM subjects with the ACE DD genotype." }, "24340838": { "QUESTION": "Do ventricular arrhythmias in athletes subside over time?", "CONTEXTS": [ "Sudden death in athletes can occur during sport activities and is presumably related to ventricular arrhythmias.", "To investigate the long-term follow-up ofathletes with ventricular arrhythmias during an exercise test.", "From a database of 56,462 athletes we identified 192 athletes (35 years old who had ventricular arrhythmias during an exercise test. Ninety athletes had>or =3 ventricular premature beats (VPB) (group A) and 102 athletes had ventricular couplets or non-sustained ventricular tachycardia during an exercise test (group B). A control group of 92 athletesfrom without ventricular arrhythmias was randomly seleclted from the database (group C). Of the 192 athletes 39 returnied for a repeat exercise test after a mean follow-up period of 70 +/- 25 months and they constitute the study population.", "Twelve athletes from group A, 21 fromgroup B and 6 from group C returned for a repeat exercise test. The athletes reached a significantly lower peak heart rate during their follow-up exercise test (P = 0.001). More athletes were engaged in competitive sports during their initialexercise test than in the follow-up test (P = 0.021). Most of theathletes who had VPB and/orventricular couplets and/or NSVT during their initial exercise test had far fewer ventricular arrhythmias in the follow-up exercise test (P = 0.001)." ], "LABELS": [ "BACKGROUND", "OBJECTIVES", "METHODS", "RESULTS" ], "MESHES": [ "Adult", "Arrhythmias, Cardiac", "Athletes", "Case-Control Studies", "Databases, Factual", "Exercise Test", "Follow-Up Studies", "Heart Rate", "Humans", "Sports", "Tachycardia, Ventricular", "Time Factors", "Ventricular Premature Complexes", "Young Adult" ], "YEAR": "2013", "reasoning_required_pred": "yes", "reasoning_free_pred": "yes", "final_decision": "yes", "LONG_ANSWER": "Athletes engaged in competitive sports are more likely to develop ventricular arrhythmias during exercise. These arrhythmias subside over time when athletes are engaged in non-competitive sports." }, "24751724": { "QUESTION": "Does ethnicity affect where people with cancer die?", "CONTEXTS": [ "Ageing is a growing issue for people from UK black, Asian and minority ethnic (BAME) groups. The health experiences of these groups are recognised as a 'tracer' to measure success in end of life patient-preferred outcomes that includes place of death (PoD).AIM: To examine patterns in PoD among BAME groups who died of cancer.", "Mortality data for 93,375 cancer deaths of those aged \u226565 years in London from 2001-2010 were obtained from the UK Office for National Statistics (ONS). Decedent's country of birth was used as a proxy for ethnicity. Linear regression examined trends in place of death across the eight ethnic groups and Poisson regression examined the association between country of birth and place of death.", "76% decedents were born in the UK, followed by Ireland (5.9%), Europe(5.4%) and Caribbean(4.3%). Most deaths(52.5%) occurred in hospital, followed by home(18.7%). During the study period, deaths in hospital declined with an increase in home deaths; trend for time analysis for those born in UK(0.50%/yr[0.36-0.64%]p<0.001), Europe (1.00%/yr[0.64-1.30%]p<0.001), Asia(1.09%/yr[0.94-1.20%]p<0.001) and Caribbean(1.03%/yr[0.72-1.30%]p<0.001). However, time consistent gaps across the geographical groups remained. Following adjustment hospital deaths were more likely for those born in Asia(Proportion ratio(PR)1.12[95%CI1.08-1.15]p<0.001) and Africa(PR 1.11[95%CI1.07-1.16]p<0.001). Hospice deaths were less likely for those born in Asia(PR 0.73 [0.68-0.80] p<0.001), Africa (PR 0.83[95%CI0.74-0.93]p<0.001), and 'other' geographical regions (PR0.90[95% 0.82-0.98]p<0.001). Home deaths were less likely for those born in the Caribbean(PR0.91[95%CI 0.85-0.98]p<0.001)." ], "LABELS": [ "BACKGROUND", "MATERIAL AND METHODS", "RESULTS" ], "MESHES": [ "Demography", "Ethnic Groups", "Female", "Hospice Care", "Humans", "London", "Male", "Neoplasms", "Parturition", "Regression Analysis", "Terminal Care" ], "YEAR": "2014", "reasoning_required_pred": "yes", "reasoning_free_pred": "yes", "final_decision": "yes", "LONG_ANSWER": "Location of death varies by country of birth. BAME groups are more likely to die in a hospital and less likely to die at home or in a hospice. Further investigation is needed to determine whether these differences result from patient-centred preferences, or other environment or service-related factors. This knowledge will enable strategies to be developed to improve access to relevant palliative care and related services, where necessary." }, "21368683": { "QUESTION": "Are patients with serious mental illness more likely to be admitted to nursing homes with more deficiencies in care?", "CONTEXTS": [ "Patients diagnosed with serious mental illness (SMI) who qualify for nursing home placement tend to require high levels of both psychiatric and nursing care. However, it is unknown whether they are equally likely to be admitted to nursing homes with adequate quality of care compared with other patients.", "We analyzed a national cohort of more than 1.3 million new nursing home admissions in 2007 using the minimum data set. The total and healthcare-related deficiency citations for each facility were obtained from the Online Survey, Certification, and Reporting file. Bivariate and multivariate regression analyses determined the association of schizophrenia or bipolar disorder with admissions to facilities with higher deficiencies.", "Compared with other patients, patients with schizophrenia (n=23,767) tended to enter nursing homes with more total (13.3 vs. 11.2, P<0.001) and healthcare-related deficiencies (8.6 vs. 7.2, P<0.001); and patients with bipolar disorder (n=19,741) were more likely to enter facilities with more problematic care too (12.5 vs. 11.2, P<0.001 for total deficiencies; and 8.2 vs. 7.2, P<0.001 for healthcare-related deficiencies). After sequentially controlling for the within-county choice of facilities, patient characteristics, and facility covariates, the association of SMI with admitting to higher-deficiency nursing homes persisted." ], "LABELS": [ "BACKGROUND", "METHODS", "RESULTS" ], "MESHES": [ "Aged", "Aged, 80 and over", "Bipolar Disorder", "Cohort Studies", "Confidence Intervals", "Female", "Humans", "Male", "Middle Aged", "Nursing Homes", "Odds Ratio", "Patient Admission", "Patients", "Quality of Health Care", "Regression Analysis", "Schizophrenia", "Severity of Illness Index", "United States" ], "YEAR": "2011", "reasoning_required_pred": "yes", "reasoning_free_pred": "yes", "final_decision": "yes", "LONG_ANSWER": "Patients diagnosed with schizophrenia or bipolar disorder (ie, SMI) were more likely than other patients to be admitted to nursing homes with higher deficiency citations for both overall quality and clinical care quality. Further research is necessary to understand the reasons behind the disparity in quality of nursing home care associated with SMI." }, "27832831": { "QUESTION": "Does an additional antirotation U-Blade (RC) lag screw improve treatment of AO/OTA 31 A1-3 fractures with gamma 3 nail?", "CONTEXTS": [ "Poor bone quality and unstable fractures increase the cut-out rate in implants with gliding lag screws. The U-Blade (RC) lag screw for the Gamma3\u00aenail was introduced to provide monoaxial rotational stability of the femoral head and neck fragment. The purpose of this study was to evaluate whether the use of the U-Blade (RC) lag screw is associated with reduced cut-out in patients with OTA/AO 31A1-3 fractures.MATERIAL &", "Between 2009 and 2014, 751 patients with OTA/AO 31A1-3 fractures were treated with a Gamma3\u00aenail at our institution. Out of this sample 199 patients were treated with U-blade (RC) lag screws. A total of 135 patients (117 female, 18 male) with standard lag screw (treatment group A) were matched equally regarding age (\u00b14 years) sex, fracture type and location to 135 patients with U-blade (RC) lag screw (treatment group B). Within a mean follow up of 9.2 months (range 6-18 months) we assessed the cut-out rate, the calTAD, lag screw migration, the Parker's mobility score and the Parker's ratio at postoperatively, six and 12 months following surgery. Furthermore we recorded all complications, ASA-Score, hospital stay and duration of surgery retrospectively.", "The most common fracture among group B with a cut-out of the lag screw were AO/OTA 2.3 and 3.2 fractures whereas in group A cut-out was most commonly seen in AO/OTA 2.1, 2.2 and 2.3 fractures, there was no significant reduction of the cut-out rate in group B 2.2% (n=3) compared to group A 3.7% (n=5). The duration of surgery was significantly shorter in group A (p<0.05). There was no significant difference in lag screw placement, the Parker's ratio and mobilization." ], "LABELS": [ "INTRODUCTION", "METHODS", "RESULTS" ], "MESHES": [ "Adult", "Aged", "Aged, 80 and over", "Bone Nails", "Bone Screws", "Female", "Femoral Fractures", "Femur Neck", "Fracture Fixation, Intramedullary", "Hip Fractures", "Humans", "Male", "Middle Aged", "Prosthesis Failure", "Retrospective Studies", "Risk Factors", "Treatment Outcome" ], "YEAR": "2016", "reasoning_required_pred": "no", "reasoning_free_pred": "no", "final_decision": "no", "LONG_ANSWER": "In our study the U-Blade (RC) lag screw did not reduce the cut-out in treatment of OTA/AO 31A1-3 fractures at all. Considering the longer duration of surgery and the higher costs of the U-Blade (RC) lag screw, our results do not justify its use. However, further prospective randomized studies will be necessary." }, "15208005": { "QUESTION": "The Omega-3 Index: a new risk factor for death from coronary heart disease?", "CONTEXTS": [ "Low intakes or blood levels of eicosapentaenoic and docosahexaenoic acids (EPA + DHA) are independently associated with increased risk of death from coronary heart disease (CHD). In randomized secondary prevention trials, fish or fish oil have been demonstrated to reduce total and CHD mortality at intakes of about 1 g/day. Red blood cell (RBC) fatty acid (FA) composition reflects long-term intake of EPA + DHA. We propose that the RBC EPA + DHA (hereafter called the Omega-3 Index) be considered a new risk factor for death from CHD.", "We conducted clinical and laboratory experiments to generate data necessary for the validation of the Omega-3 Index as a CHD risk predictor. The relationship between this putative marker and risk for CHD death, especially sudden cardiac death (SCD), was then evaluated in several published primary and secondary prevention studies.", "The Omega-3 Index was inversely associated with risk for CHD mortality. An Omega-3 Index of>or = 8% was associated with the greatest cardioprotection, whereas an index of2) of men and women were compared using the chi-square test. Multiple regression analysis was used to assess sex as a predictor of recanalization and poor 3-month outcome after controlling for age, baseline NIH Stroke Scale (NIHSS), time to treatment, hypertension, and blood glucose.", "369 patients had TCD examinations before or during IV rt-PA treatment. The 199 (53.9%) men and 170 (46.1%) women had mean ages of 67\u2009\u00b1\u200913 and 70\u2009\u00b1\u200914 years, respectively. The sexes did not differ significantly in baseline stroke severity, time to TCD examination, or time to thrombolysis. Of the men, 68 (34.2%) had complete recanalization, 58 (29.1%) had partial recanalization, and 73 (36.6%) had no recanalization. Of the women, 53 (31.2%) had complete recanalization, 46 (27%) had partial recanalization, and 71 (41.8%) had no recanalization (p\u2009=\u20090.6). Multiple regression analyses showed no difference between the sexes in recanalization rate, time to recanalization, or clinical outcome at 3 months." ], "LABELS": [ "BACKGROUND", "METHODS", "RESULTS" ], "MESHES": [ "Aged", "Female", "Fibrinolytic Agents", "Humans", "Male", "Middle Aged", "Recombinant Proteins", "Sex Factors", "Stroke", "Tissue Plasminogen Activator", "Ultrasonography, Doppler, Transcranial" ], "YEAR": "2014", "reasoning_required_pred": "no", "reasoning_free_pred": "no", "final_decision": "no", "LONG_ANSWER": "In our study; sex is not a significant predictor of recanalization rate, time to recanalization or 3-month outcome in stroke patients following IV rt-PA." }, "9278754": { "QUESTION": "Are head and neck specific quality of life measures necessary?", "CONTEXTS": [ "The purpose of this study was to determine whether head and neck-specific health status domains are distinct from those assessed by general measures of quality-of-life (QOL).", "Cross-sectional study of 55 head and neck cancer patients in tertiary academic center was made. Three head and neck-specific measures,-including the Head&Neck Survey (H&NS); a brief, multi-item test which generates domain scores; and a general health measure,-were administered.", "The H&NS was highly reliable and more strongly correlated to the specific measures than to the general measure. Eating/swallowing (ES) and speech/communication (SC) were not well correlated with general health domains. Head and neck pain was highly correlated to general bodily pain (0.88, p<.0001). Despite correlations to some general health domains, appearance (AP) was not fully reflected by any other domain." ], "LABELS": [ "BACKGROUND", "METHODS", "RESULTS" ], "MESHES": [ "Activities of Daily Living", "Attitude to Health", "Carcinoma, Squamous Cell", "Communication", "Cross-Sectional Studies", "Deglutition", "Eating", "Emotions", "Face", "Female", "Follow-Up Studies", "Head and Neck Neoplasms", "Health Status", "Humans", "Life Style", "Male", "Mental Health", "Middle Aged", "Pain", "Pain Management", "Quality of Life", "Reproducibility of Results", "Role", "Social Adjustment", "Speech" ], "YEAR": "1997", "reasoning_required_pred": "yes", "reasoning_free_pred": "yes", "final_decision": "yes", "LONG_ANSWER": "Head and neck-specific QOL measures are necessary and should include domains that reflect ES, SC, and AP." }, "8738894": { "QUESTION": "Diabetes mellitus among Swedish art glass workers--an effect of arsenic exposure?", "CONTEXTS": [ "The purpose of this study was to search for evidence of an association between occupational arsenic exposure and diabetes mellitus, as implied by the relation of this disease to arsenic in drinking water in a recent study from Taiwan.", "A case-referent analysis on death records of 5498 individuals in the art glass producing part of southeastern Sweden was performed. Out of all the enrolled subjects, 888 were glass workers. According to occupational title, glassblowers, foundry workers, and unspecified workers were regarded as potentially exposed to arsenic. Persons with a diagnosis of diabetes mellitus either as an underlying or contributing cause of death were considered cases. Referents were decedents without any indication of cancer, cardiovascular disease, or diabetes.", "A slightly elevated risk [Mantel-Haenszel odds ratio (MH-OR) 1.2, 95% confidence interval (95% CI) 0.82-1.8] was found for diabetes mellitus among the glassworks employees, especially in combination with cardiovascular disease (MH-OR 1.4, 95% CI 0.81-2.3). For the glassblowers, other foundry workers and unspecified glassworkers probably exposed to arsenic, the M-H odds ratio was 1.4 (95% CI 0.92-2.2). Unspecified glass workers, who probably included persons with high exposure, carried the higher risk (MH-OR 1.8, 95% CI 1.1-2.8)." ], "LABELS": [ "OBJECTIVES", "METHODS", "RESULTS" ], "MESHES": [ "Age Distribution", "Aged", "Arsenic", "Cause of Death", "Diabetes Complications", "Diabetes Mellitus", "Glass", "Humans", "Male", "Middle Aged", "Occupational Exposure", "Occupations", "Poisons", "Retrospective Studies", "Risk Assessment", "Sweden" ], "YEAR": "1996", "reasoning_required_pred": "yes", "reasoning_free_pred": "no", "final_decision": "no", "LONG_ANSWER": "The observations from this study provide limited support for the possibility that occupational arsenic exposure could play a role in the development of diabetes mellitus. Many other metallic compounds are also used in art glass production, however, and there is a possibility of confounding." }, "24671913": { "QUESTION": "Does SYNTAX score predict in-hospital outcomes in patients with ST elevation myocardial infarction undergoing primary percutaneous coronary intervention?", "CONTEXTS": [ "SYNTAX score (SxS) has been demonstrated to predict long-term outcomes in stable patients with coronary artery disease. But its prognostic value for patients with acute coronary syndrome remains unknown.AIM: To evaluate whether SxS could predict in-hospital outcomes for patients admitted with ST elevation myocardial infarction (STEMI) who undergo primary percutaneous coronary intervention (pPCI).", "The study included 538 patients with STEMI who underwent pPCI between January 2010 and December 2012. The patients were divided into two groups: low SxS (<22) and high SxS (>22). The SxS of all patients was calculated from aninitial angiogram and TIMI flow grade of infarct related artery was calculated after pPCI. Left ventricular systolic functions of the patients were evaluated with an echocardiogram in the following week. The rates of reinfarction and mortality during hospitalisation were obtained from the medical records of our hospital.", "The high SxS group had more no-reflow (41% and 25.1%, p<0.001, respectively), lower ejection fraction (38.2 \u00b1 7.5% and 44.6 \u00b1 8.8%, p<0.001, respectively), and greater rates of re-infarction (9.5% and 7.3%, p = 0.037, respectively) and mortality (0.9% and 0.2%, p = 0.021, respectively) during hospitalisation compared to the low SxS group. On multivariate logistic regression analysis including clinical variables, SxS was an independent predictor of no-reflow (OR 1.081, 95% CI 1.032-1.133, p = 0.001)." ], "LABELS": [ "BACKGROUND", "METHODS", "RESULTS" ], "MESHES": [ "Aged", "Female", "Hospital Mortality", "Humans", "Male", "Middle Aged", "Myocardial Infarction", "Percutaneous Coronary Intervention", "Poland", "Prognosis", "Risk Assessment", "Risk Factors", "Severity of Illness Index", "Treatment Outcome" ], "YEAR": "2014", "reasoning_required_pred": "yes", "reasoning_free_pred": "yes", "final_decision": "yes", "LONG_ANSWER": "SxS is a useful tool that can predict in-hospital outcomes of patients with STEMI undergoing pPCI." }, "22540518": { "QUESTION": "Is micro-computed tomography reliable to determine the microstructure of the maxillary alveolar bone?", "CONTEXTS": [ "To analyze the reliability of micro-computed tomography (micro-CT) to assess bone density and the microstructure of the maxillary bones at the alveolar process in human clinics by direct comparison with conventional stereologic-based histomorphometry.", "Analysis of osseous microstructural variables including bone volumetric density (BV/TV) of 39 biopsies from the maxillary alveolar bone was performed by micro-CT. Conventional stereologic-based histomorphometry of 10 bone biopsies was performed by optic microscopy (OM) and low-vacuum surface electronic microscopy (SEM). Percentages of bone between micro-CT and conventional stereologic-based histomorphometry were compared.", "Significant positive correlations were observed between BV/TV and the percentage of bone (%Bone) analyzed by SEM (r\u00a0=\u00a00.933, P\u00a0<\u00a00.001), by toluidine blue staining OM (r\u00a0=\u00a00.950, P\u00a0<\u00a00.001) and by dark field OM (r\u00a0=\u00a00.667, P\u00a0=\u00a00.05). The high positive correlation coefficient between BV/TV and trabecular thickness illustrates that a value of BV/TV upper than 50% squares with a bone presenting most of their trabecules thicker than 0.2\u00a0mm. The high negative correlation between BV/TV and trabecular separation shows that values of BV/TV upper than 50% squares with a bone presenting most of their trabecules separated less than 0.3\u00a0mm each other." ], "LABELS": [ "OBJECTIVES", "MATERIALS AND METHODS", "RESULTS" ], "MESHES": [ "Adult", "Aged", "Alveolar Process", "Biopsy", "Bone Density", "Coloring Agents", "Dental Implantation, Endosseous", "Female", "Humans", "Image Processing, Computer-Assisted", "Male", "Maxilla", "Microscopy", "Microscopy, Electron", "Middle Aged", "Reproducibility of Results", "Spectrometry, X-Ray Emission", "Tolonium Chloride", "X-Ray Microtomography", "Young Adult" ], "YEAR": "2013", "reasoning_required_pred": "yes", "reasoning_free_pred": "yes", "final_decision": "yes", "LONG_ANSWER": "BV/TV assessed by micro-CT correlates with the percentage of bone assessed by conventional stereologic-based histomorphometry. Micro-CT is a reliable technique to determine the bone density and the microstructure of the maxillary alveolar bone at the site of dental implant insertion." }, "16538201": { "QUESTION": "Does use of hydrophilic guidewires significantly improve technical success rates of peripheral PTA?", "CONTEXTS": [ "To determine whether the use of hydrophilic guidewires has increased the technical success rate of peripheral percutaneous transluminal angioplasty (PTA).MATERIAL/", "We performed 125 procedures and analyzed the technical success rates of PTA using the conventional guidewire first and then if needed, the hydrophilic guidewire for iliac and SFA stenoses or occlusions. Angioplasty was performed on 25 stenosed, 25 occluded iliac arteries and 25 stenosed, 50 occluded femoral arteries. The result was defined as technical success when the lesion was crossed by a guidewire and balloon, then it was dilated with restoration of vessel lumen and less than 30% residual stenosis and the rise in ABI values was at least 0.15 after 24 hours.", "The technical success rate after PTA of stenosed iliac arteries was achieved in 96% (24/25) using conventional wires and 100% using hydrophilic guidewire; in iliac occlusions, the rates were 60% (15/25) and 96%, respectively; in femoral stenosis - 84% (21/25) and 100%; in occlusions in the first group: lesions<10 cm -64% (16/25) and 96%, in the second group: lesions>10 cm -48% (12/25) and 88%. In the iliac group, there was no significant difference in the success of iliac stenosis PTA. However, there were significant changes in the success rates of PTA performed for SFA stenosis and iliac and SFA occlusions." ], "LABELS": [ "BACKGROUND", "METHODS", "RESULTS" ], "MESHES": [ "Angioplasty, Balloon", "Arterial Occlusive Diseases", "Constriction, Pathologic", "Femoral Artery", "Humans", "Iliac Artery", "Treatment Outcome" ], "YEAR": "2004", "reasoning_required_pred": "maybe", "reasoning_free_pred": "yes", "final_decision": "maybe", "LONG_ANSWER": "In summary, we report an overall improvement and high technical success rate for peripherial PTA. The use of hydrophilic guidewires made significant difference to the technical success rate of PTA, especially in occlusion and more complicated lesions." }, "17054994": { "QUESTION": "Does frozen section alter surgical management of multinodular thyroid disease?", "CONTEXTS": [ "Frozen section (FS) evaluation during thyroid surgery is often used to guide intraoperative management. We sought to determine the utility of FS in patients undergoing thyroidectomy for multinodular thyroid disease.", "From May 1994 through November 2004, 236 patients with multinodular goiter underwent thyroidectomy at our institution. Patient data were retrospectively analyzed to see if a frozen section was performed during the procedure and whether it changed the patient's outcome.", "Of the 236 patients, 135 (57%) had intra-operative FS. There were no differences between patients who had FS analysis and those who did not with regard to age, gender, and the incidence of malignancy. Of the patients who had FS, 4/135 (3%) were subsequently diagnosed with thyroid cancer on permanent histology. Three of these FS were misread as benign. Therefore, the sensitivity of FS for the diagnosis of thyroid cancer was only 25%. Importantly, in none of the 135 patients did FS alter the intraoperative management." ], "LABELS": [ "BACKGROUND", "METHODS", "RESULTS" ], "MESHES": [ "Adult", "Aged", "Carcinoma, Papillary", "Female", "Frozen Sections", "Goiter, Nodular", "Humans", "Incidence", "Intraoperative Care", "Lymphoma", "Male", "Middle Aged", "Retrospective Studies", "Sensitivity and Specificity", "Thyroid Neoplasms", "Thyroidectomy", "Treatment Outcome" ], "YEAR": "2006", "reasoning_required_pred": "no", "reasoning_free_pred": "no", "final_decision": "no", "LONG_ANSWER": "While FS was commonly used in patients undergoing thyroidectomy at our institution, in no patient over the last decade did FS correctly alter the intraoperative management. Given the time required to perform FS and the cost associated with it, we believe that routine FS should not be performed in these patients." }, "26999038": { "QUESTION": "Could chest wall rigidity be a factor in rapid death from illicit fentanyl abuse?", "CONTEXTS": [ "There has been a significant spike in fentanyl-related deaths from illicit fentanyl supplied via the heroin trade. Past fentanyl access was primarily oral or dermal via prescription fentanyl patch diversion. One factor potentially driving this increase in fatalities is the change in route of administration. Rapid intravenous (IV) fentanyl can produce chest wall rigidity. We evaluated post-mortem fentanyl and norfentanyl concentrations in a recent surge of lethal fentanyl intoxications.", "Fentanyl related deaths from the Franklin County coroner's office from January to September 2015 were identified. Presumptive positive fentanyl results were confirmed by quantitative analysis using liquid chromatography tandem mass spectrometry (LC/MS/MS) and were able to quantify fentanyl, norfentanyl, alfentanyl, and sufentanyl.", "48 fentanyl deaths were identified. Mean fentanyl concentrations were 12.5\u2009ng/ml, (range 0.5\u2009ng/ml to\u2009>40\u2009ng/ml). Mean norfentanyl concentrations were 1.9\u2009ng/ml (range none detected to 8.3\u2009ng/ml). No appreciable concentrations of norfentanyl could be detected in 20 of 48 cases (42%) and were less than 1\u2009ng/ml in 25 cases (52%). Elevated fentanyl concentrations did not correlate with rises in norfentanyl levels. In several cases fentanyl concentrations were strikingly high (22\u2009ng/ml and 20\u2009ng/ml) with no norfentanyl detected.", "The lack of any measurable norfentanyl in half of our cases suggests a very rapid death, consistent with acute chest rigidity. An alternate explanation could be a dose-related rapid onset of respiratory arrest. Deaths occurred with low levels of fentanyl in the therapeutic range (1-2\u2009ng/ml) in apparent non-na\u00efve opiate abusers. Acute chest wall rigidity is a well-recognized complication in the medical community but unknown within the drug abuse community. The average abuser of illicit opioids may be unaware of the increasing fentanyl content of their illicit opioid purchase." ], "LABELS": [ "BACKGROUND", "METHODS", "RESULTS", "DISCUSSION" ], "MESHES": [ "Administration, Cutaneous", "Administration, Oral", "Adult", "Analgesics, Opioid", "Autopsy", "Cause of Death", "Drug Overdose", "Female", "Fentanyl", "Forensic Toxicology", "Humans", "Male", "Middle Aged", "Narcotics", "Tandem Mass Spectrometry", "Thoracic Wall", "Young Adult" ], "YEAR": "2016", "reasoning_required_pred": "yes", "reasoning_free_pred": "yes", "final_decision": "yes", "LONG_ANSWER": "In summary we believe sudden onset chest wall rigidity may be a significant and previously unreported factor leading to an increased mortality, from illicit IV fentanyl use. Fentanyl and norfentanyl ratios and concentrations suggest a more rapid onset of death given the finding of fentanyl without norfentanyl in many of the fatalities. Chest wall rigidity may help explain the cause of death in these instances, in contrast to the typical opioid-related overdose deaths. Intravenous heroin users should be educated regarding this potentially fatal complication given the increasingly common substitution and combination with heroin of fentanyl." }, "23598835": { "QUESTION": "Is severe pain immediately after spinal augmentation a predictor of long-term outcomes?", "CONTEXTS": [ "Severe, immediate postprocedural pain and the need for analgesics after vertebroplasty can be a discouraging experience for patients and caregivers. The goal of this study was to investigate whether the presence of severe pain immediately after vertebroplasty predicts short- and long-term pain relief.", "A chart review was performed to categorize patients regarding pain severity and analgesic usage immediately after vertebroplasty (<4 h). \"Severe\" pain was defined as at least 8 of 10 with the 10-point VAS. Outcomes were pain severity and pain medication score and usage at 1 month and 1 year after vertebroplasty. Outcomes and clinical characteristics were compared between groups by using the Wilcoxon signed-rank test and the Fisher exact test.", "Of the 429 vertebroplasty procedures identified, 69 (16%) were associated with severe pain, and 133 (31%) were associated with analgesic administration immediately after the procedure. The group experiencing severe pain had higher preprocedure median VAS rest pain scores (5 [IQR, 2-7]) and activitypain scores (10 [IQR, 8-10]) compared with patients who did not experience severe pain (3 [IQR, 1-6]; P = .0208, and 8 [IQR, 7-10]; P = .0263, respectively). At 1 month postprocedure, VAS rest and activity pain scores were similar between the severe pain group and the nonsevere pain group (P = .16 and P = .25, respectively) and between the group receiving pain medication and the group not receiving pain medication (P = .25 and P = .67, respectively). This similarity continued for 1 year after the procedure. Analgesic usage was similar among all groups at 1 year postprocedure." ], "LABELS": [ "BACKGROUND AND PURPOSE", "MATERIALS AND METHODS", "RESULTS" ], "MESHES": [ "Aged", "Aged, 80 and over", "Causality", "Chronic Pain", "Comorbidity", "Female", "Humans", "Male", "Minnesota", "Pain Measurement", "Pain, Postoperative", "Prevalence", "Prognosis", "Retrospective Studies", "Risk Factors", "Spinal Fractures", "Treatment Outcome", "Vertebroplasty" ], "YEAR": "2013", "reasoning_required_pred": "no", "reasoning_free_pred": "no", "final_decision": "no", "LONG_ANSWER": "Patients with severe pain immediately after vertebroplasty have similar long-term outcomes compared with patients without severe pain." }, "17890090": { "QUESTION": "Can computerised tomography replace bone scintigraphy in detecting bone metastases from breast cancer?", "CONTEXTS": [ "The aim of this study was to determine whether bone scans (BS) can be avoided if pelvis was included in CT thorax and abdomen to detect bony metastases from breast cancer.", "Results of 77 pairs of CT (thorax, abdomen, and pelvis) and BS in newly diagnosed patients with metastatic breast cancer (MBC) were compared prospectively for 12 months. Both scans were blindly assessed by experienced radiologists and discussed at multidisciplinary team meetings regarding the diagnosis of bone metastases.", "CT detected metastatic bone lesions in 43 (98%) of 44 patients with bone metastases. The remaining patient had a solitary, asymptomatic bony metastasis in shaft of femur. BS was positive in all patients with bone metastases. There were 11 cases of false positive findings on BS." ], "LABELS": [ "BACKGROUND", "MATERIALS AND METHODS", "RESULTS" ], "MESHES": [ "Adult", "Aged", "Aged, 80 and over", "Bone Neoplasms", "Breast Neoplasms", "Female", "Humans", "Middle Aged", "Neoplasm Staging", "Pelvis", "Prospective Studies", "Radiography, Abdominal", "Radiography, Thoracic", "Radionuclide Imaging", "Reproducibility of Results", "Sensitivity and Specificity", "Tomography, X-Ray Computed" ], "YEAR": "2008", "reasoning_required_pred": "yes", "reasoning_free_pred": "yes", "final_decision": "yes", "LONG_ANSWER": "Our findings suggest routine BS of patients presenting with MBC is not required if CT (thorax, abdomen, and pelvis) is performed." }, "22813804": { "QUESTION": "Does obesity impact the pattern and outcome of trauma in children?", "CONTEXTS": [ "Childhood obesity is pandemic condition. The effect of obesity on trauma outcomes in children has been relatively understudied. We conducted this study to ascertain the effects of obesity on the hospital outcome of injured children.", "A retrospective cohort study of patients aged 2 to 18 years admitted to the King Abdul Aziz Medical City between May 2001 and May 2009 was conducted. Patients were categorized as lean (body mass index<95th percentile) and obese (body mass index \u2265 95th percentile). Groups were compared regarding admission demographics, mechanism of injury, pattern of injury, length of stay, intensive care unit admission, ventilation duration, types of procedures performed, injury severity score, and mortality.", "Nine hundred thirty-three patients were included, of those 55 (5.89%) children were obese. The obese children were older than nonobese (P = .001) and had a higher injury severity score (P = .001) and a lower pediatric trauma score (P = .00), heart rate (P = .0081), and respiratory rate (P = .000). There were no differences between groups with regard to sex, mechanism of injury, and surgical procedures. Obese children were more likely to have rib fractures (P = .02) and pelvic injuries (P = .033). There was no significant association between mortality and obesity (P = .42)." ], "LABELS": [ "BACKGROUND", "METHODS", "RESULT" ], "MESHES": [ "Adolescent", "Child", "Child, Preschool", "Cohort Studies", "Female", "Hospitalization", "Humans", "Injury Severity Score", "Male", "Obesity", "Prognosis", "Proportional Hazards Models", "Registries", "Retrospective Studies", "Saudi Arabia", "Wounds and Injuries" ], "YEAR": "2012", "reasoning_required_pred": "yes", "reasoning_free_pred": "no", "final_decision": "no", "LONG_ANSWER": "Obesity does not seem to impact the severity of injury, mortality rate, types of injury, and procedure outcomes in children. Obese patients are more likely to have rib and pelvic injuries." }, "26923375": { "QUESTION": "Is non-invasive diagnosis of esophageal varices in patients with compensated hepatic cirrhosis possible by duplex Doppler ultrasonography?", "CONTEXTS": [ "Esophageal varices are present in 30% to 40% of patients in compensated cirrhosis (Child-Pugh class A) and in 60% to 85% of patients in decompensated cirrhosis (Child-Pugh classes B and C). It is important to identify patients with compensated cirrhosis at risk for esophageal varix development. We evaluated the accuracy of a duplex Doppler ultrasonographic index for predicting the presence or absence of esophageal varices in patients with compensated hepatic cirrhosis (Child-Pugh class A) by using endoscopy as the reference standard.", "Fifty-six enrolled patients underwent duplex Doppler ultrasonography followed by screening endoscopy. Mean portal vein velocity (PVV), splenic index (SI), splenoportal index (SPI), hepatic and splenic arterial resistive, and pulsatility indices (hepatic artery resistive index [HARI], hepatic artery pulsatility index [HAPI], splenic artery resistive index [SARI], splenic artery pulsatility index [SAPI]) were recorded. Univariate logistic regression analysis was followed by receiver operating characteristic (ROC) curve construction for the indices that were significant.", "The indices HARI, HAPI, SARI, SAPI were not helpful (p\u2009>\u20090.05). Mean PVV, SI, and SPI were all predictive of the presence of esophageal varices (p\u2009<\u20090.05) and SPI was found to be the most accurate parameter. Of the various cut-off levels of SPI evaluated, a cut-off value of SPI at 5.0, offered the highest diagnostic accuracy (88%). For the 28 patients with SPI<5.0, the absence of esophageal varices in 27 of them could be correctly diagnosed using only SPI without invasive screening endoscopy, with high negative predictive value (96%) and sensitivity (96%). Of the remaining 28 patients with SPI \u22655.0, presence of esophageal varices could be similarly correctly diagnosed in 22 of them by using SPI without screening endoscopy, with high positive predictive value (79%) and specificity (82%)." ], "LABELS": [ "BACKGROUND AND AIM", "METHODS", "RESULTS" ], "MESHES": [ "Adult", "Aged", "Aged, 80 and over", "Endosonography", "Esophageal and Gastric Varices", "Female", "Humans", "Liver Cirrhosis", "Male", "Middle Aged", "Prevalence", "Regression Analysis", "Sensitivity and Specificity", "Sex Distribution", "Ultrasonography, Doppler, Duplex" ], "YEAR": "2016", "reasoning_required_pred": "yes", "reasoning_free_pred": "yes", "final_decision": "yes", "LONG_ANSWER": "The SPI was accurate in predicting the presence or absence of esophageal varices in patients with compensated cirrhosis." }, "20304513": { "QUESTION": "Is (18)F-FDG a surrogate tracer to measure tumor hypoxia?", "CONTEXTS": [ "Fluorodeoxyglucose (FDG) has been reported as a surrogate tracer to measure tumor hypoxia with positron emission tomography (PET). The hypothesis is that there is an increased uptake of FDG under hypoxic conditions secondary to enhanced glycolysis, compensating the hypoxia-induced loss of cellular energy production. Several studies have already addressed this issue, some with conflicting results. This study aimed to compare the tracers (14)C-EF3 and (18)F-FDG to detect hypoxia in mouse tumor models.", "C3H, tumor-bearing mice (FSAII and SCCVII tumors) were injected iv with (14)C-EF3, and 1h later with (18)F-FDG. Using a specifically designed immobilization device with fiducial markers, PET (Mosaic\u00ae, Philips) images were acquired 1h after the FDG injection. After imaging, the device containing mouse was frozen, transversally sliced and imaged with autoradiography (AR) (FLA-5100, Fujifilm) to obtain high resolution images of the (18)F-FDG distribution within the tumor area. After a 48-h delay allowing for (18)F decay a second AR was performed to image (14)C-EF3 distribution. AR images were aligned to reconstruct the full 3D tumor volume, and were compared with the PET images. Image segmentation with threshold-based methods was applied on both AR and PET images to derive various tracer activity volumes. The matching index DSI (dice similarity index) was then computed. The comparison was performed under normoxic (ambient air", "n=4, SCCVII, n=5) and under hypoxic conditions (10% O(2) breathing", "n=4).", "On AR, under both ambient air and hypoxic conditions, there was a decreasing similarity between (14)C-EF3 and FDG with higher activity sub-volumes. Under normoxic conditions, when comparing the 10% of tumor voxels with the highest (18)F-FDG or (14)C-EF3 activity, a DSI of 0.24 and 0.20 was found for FSAII and SCCVII, respectively. Under hypoxic conditions, a DSI of 0.36 was observed for SCCVII tumors. When comparing the (14)C-EF3 distribution in AR with the corresponding (18)F-FDG-PET images, the DSI reached values of 0.26, 0.22 and 0.21 for FSAII and SCCVII under normoxia and SCCVII under hypoxia, respectively." ], "LABELS": [ "INTRODUCTION", "MATERIALS AND METHODS", ", FSAII", ", SCCVII", "RESULTS" ], "MESHES": [ "Animals", "Carbon Radioisotopes", "Disease Models, Animal", "Fiducial Markers", "Fluorodeoxyglucose F18", "Hypoxia", "Male", "Mice", "Neoplasms", "Nitroimidazoles", "Radionuclide Imaging" ], "YEAR": "2010", "reasoning_required_pred": "yes", "reasoning_free_pred": "no", "final_decision": "no", "LONG_ANSWER": "This study showed that FDG is not a good surrogate tracer for tumor hypoxia under either ambient or hypoxic conditions. Only specific hypoxia tracers should be used to measure tumor hypoxia." }, "21689015": { "QUESTION": "Can dogs prime autistic children for therapy?", "CONTEXTS": [ "Canine-assisted therapy has been receiving growing attention as a means of aiding children with autism spectrum disorder (ASD). Yet, only limited studies have been done and a great deal of literature related to this intervention is anecdotal. The present study aims at providing additional quantitative evidence on the potential of dogs to positively modulate the behavior of children with ASD.SETTINGS/", "A 12-year-old boy diagnosed with ASD was exposed, at his usual treatment location (the Portuguese Association for Developmental Disorders and Autism at Vila Nova de Gaia, Portugal), to the following treatment conditions: (1) one-to-one structured activities with a therapist assisted by a certified therapy dog, and (2) one-to-one structured activities with the same therapist alone (as a control). To accurately assess differences in the behavior of the participant between these treatment conditions, the therapist followed a strict research protocol. The behavior of the participant was continuously video-recorded during both treatment conditions for further analysis and comparison. Treatment outcomes: In the presence of the dog, the participant exhibited more frequent and longer durations of positive behaviors (such as smiling and positive physical contacting) as well as less frequent and shorter durations of negative behaviors (such as aggressive manifestations)." ], "LABELS": [ "BACKGROUND AND OBJECTIVES", "LOCATION, SUBJECTS, AND INTERVENTIONS" ], "MESHES": [ "Animal Assisted Therapy", "Animals", "Autistic Disorder", "Behavior Therapy", "Child", "Child Behavior", "Dogs", "Humans", "Male", "Portugal", "Social Behavior", "Treatment Outcome" ], "YEAR": "2011", "reasoning_required_pred": "yes", "reasoning_free_pred": "yes", "final_decision": "yes", "LONG_ANSWER": "These findings are in accordance with previous experimental work and provide additional support for the assertion that dogs can prime autistic children for therapy. Ultimately, this study may contribute toward a change for full acceptance of canine-assisted therapy programs within the medical milieu. Additional studies using a similar research protocol on more autistic children will certainly help professionals to work on the most effective methods to individually serve this population through canine-assisted interventions." }, "10811329": { "QUESTION": "Are 99mTc leukocyte scintigraphy and SBFT studies useful in children suspected of having inflammatory bowel disease?", "CONTEXTS": [ "The goal of this retrospective study was to assess whether 99mTc-white blood cell (WBC) scintigraphy and upper gastrointestinal small bowel follow-through (UGI-SBFT) could exclude inflammation in children suspected of having inflammatory bowel disease (IBD).", "Of a population of 313 children who had a 99mTc-WBC scan, 130 children were studied exclusively to rule out IBD. Sixty-nine colonoscopies with biopsies were done within a short time interval of the 99mTc-WBC scans. There were also 51 controls studied with 99mTc-WBC scintigraphy.", "Of the 130 children studied to exclude IBD, the final diagnosis was Crohn's disease in 27, ulcerative colitis in nine, miscellaneous colitis in 13, probably normal in 42, and normal in 39. The 99mTc-WBC scans were positive in all but three newly diagnosed Crohn's disease, ulcerative colitis, or miscellaneous colitis children. The false-negative 99mTc-WBC studies were seen in children with mild inflammation on biopsies and normal UGI-SBFT studies. In the 46 children with a true-positive 99mTc-WBC scan, 81% (17/21) of UGI-SBFT studies were normal. In five children with equivocal UGI-SBFT studies, the 99mTc-WBC scan correctly predicted if inflammation was present in the terminal ileum." ], "LABELS": [ "OBJECTIVE", "METHODS", "RESULTS" ], "MESHES": [ "Adolescent", "Child", "Colitis, Ulcerative", "Colonoscopy", "Crohn Disease", "Female", "Humans", "Inflammatory Bowel Diseases", "Intestines", "Leukocytes", "Male", "Predictive Value of Tests", "Radiography", "Retrospective Studies", "Sensitivity and Specificity", "Technetium", "Tomography, Emission-Computed, Single-Photon" ], "YEAR": "2000", "reasoning_required_pred": "yes", "reasoning_free_pred": "yes", "final_decision": "yes", "LONG_ANSWER": "Our results suggest that 99mTc-WBC is useful as an initial screening modality to exclude IBD, and is more sensitive than UGI-SBFT studies." }, "10834864": { "QUESTION": "Risk factors for avascular necrosis of bone in patients with systemic lupus erythematosus: is there a role for antiphospholipid antibodies?", "CONTEXTS": [ "Avascular necrosis of bone (AVN) is a well known complication in patients with systemic lupus erythematosus (SLE).", "To investigate the role of antiphospholipid antibody status (IgM and IgG anticardiolipin antibodies and lupus anticoagulant) with adjustment for corticosteroid use as risk factors for the development of AVN.", "A cohort of 265 patients receiving long term follow up in our SLE clinic from 1978 to 1998 was analysed. Patients with AVN complications were detected and then matched for age, sex, ethnicity, duration of disease, and organ disease with two other patients with SLE. A further 31 patients were chosen at random for the analysis.", "Eleven patients had AVN, giving a point prevalence of 4%. There were no significant differences demonstrable in the presence of individual antiphospholipid antibodies (aPL) or their combination between the group with AVN or the two control groups." ], "LABELS": [ "BACKGROUND", "OBJECTIVE", "METHODS", "RESULTS" ], "MESHES": [ "Adolescent", "Adult", "Antibodies, Anticardiolipin", "Antibodies, Antiphospholipid", "Biomarkers", "Case-Control Studies", "Female", "Follow-Up Studies", "Humans", "Immunoglobulin G", "Immunoglobulin M", "Lupus Coagulation Inhibitor", "Lupus Erythematosus, Systemic", "Middle Aged", "Osteonecrosis", "Risk Factors" ], "YEAR": "2000", "reasoning_required_pred": "no", "reasoning_free_pred": "no", "final_decision": "no", "LONG_ANSWER": "Incorporating an adjustment for corticosteroid use we were unable to show a link between the presence of aPL and the development of AVN in patients with SLE." }, "27909738": { "QUESTION": "Prostate bed target interfractional motion using RTOG consensus definitions and daily CT on rails : Does target motion differ between superior and inferior portions of the clinical target volume?", "CONTEXTS": [ "Using high-quality CT-on-rails imaging, the daily motion of the prostate bed clinical target volume (PB-CTV) based on consensus Radiation Therapy Oncology Group (RTOG) definitions (instead of surgical clips/fiducials) was studied. It was assessed whether PB motion in the superior portion of PB-CTV (SUP-CTV) differed from the inferior PB-CTV (INF-CTV).", "Eight pT2-3bN0-1M0 patients underwent postprostatectomy intensity-modulated radiotherapy, totaling 300\u00a0fractions. INF-CTV and SUP-CTV were defined as PB-CTV located inferior and superior to the superior border of the pubic symphysis, respectively. Daily pretreatment CT-on-rails images were compared to the planning CT in the left-right (LR), superoinferior (SI), and anteroposterior (AP) directions. Two parameters were defined: \"total PB-CTV motion\" represented total shifts from skin tattoos to RTOG-defined anatomic areas; \"PB-CTV target motion\" (performed for both SUP-CTV and INF-CTV) represented shifts from bone to RTOG-defined anatomic areas (i.\u2009e., subtracting shifts from skin tattoos to bone).", "Mean (\u00b1 standard deviation, SD) total PB-CTV motion was -1.5\u00a0(\u00b1\u202f6.0), 1.3\u00a0(\u00b1\u202f4.5), and 3.7\u00a0(\u00b1\u202f5.7)\u00a0mm in LR, SI, and AP directions, respectively. Mean (\u00b1\u202fSD) PB-CTV target motion was 0.2\u00a0(\u00b11.4), 0.3\u00a0(\u00b12.4), and 0\u00a0(\u00b13.1)\u00a0mm in the LR, SI, and AP directions, respectively. Mean (\u00b1\u202fSD) INF-CTV target motion was 0.1\u00a0(\u00b1\u202f2.8), 0.5\u00a0(\u00b1\u202f2.2), and 0.2 (\u00b1\u202f2.5)\u00a0mm, and SUP-CTV target motion was 0.3\u00a0(\u00b1\u202f1.8), 0.5\u00a0(\u00b1\u202f2.3), and 0\u00a0(\u00b1\u202f5.0)\u00a0mm in LR, SI, and AP directions, respectively. No statistically significant differences between INF-CTV and SUP-CTV motion were present in any direction." ], "LABELS": [ "PURPOSE", "PATIENTS AND METHODS", "RESULTS" ], "MESHES": [ "Aged", "Artifacts", "Guideline Adherence", "Humans", "Male", "Middle Aged", "Motion", "Patient Positioning", "Practice Guidelines as Topic", "Radiation Oncology", "Radiotherapy Dosage", "Radiotherapy Planning, Computer-Assisted", "Radiotherapy, Image-Guided", "Radiotherapy, Intensity-Modulated", "Reproducibility of Results", "Sensitivity and Specificity", "Tomography, X-Ray Computed", "Treatment Outcome", "Tumor Burden", "United States" ], "YEAR": "2017", "reasoning_required_pred": "no", "reasoning_free_pred": "no", "final_decision": "no", "LONG_ANSWER": "There are no statistically apparent motion differences between SUP-CTV and INF-CTV. Current uniform planning target volume (PTV) margins are adequate to cover both portions of the CTV." }, "26063028": { "QUESTION": "Esophagogastric devascularization without splenectomy in portal hypertension: safe and effective?", "CONTEXTS": [ "Esophagogastric variceal hemorrhage is a life-threatening complication of portal hypertension. In this study, we compared the therapeutic effect of a novel surgical procedure, esophagogastric devascularization without splenectomy (EDWS), with the widely used modified esophagogastric devascularization (MED) with splenectomy for the treatment of portal hypertension.", "Fifty-five patients with portal hypertension were included in this retrospective study. Among them, 27 patients underwent EDWS, and the other 28 patients underwent MED. Patients' characteristics, perioperative parameters and long-term follow-up were analyzed.", "The portal venous pressure was decreased by 20% postoperatively in both groups. The morbidity rate of portal venous system thrombosis in the EDWS group was significantly lower than that in the MED group (P=0.032). The 1- and 3-year recurrence rates of esophagogastric variceal hemorrhage were 0% and 4.5% in the EDWS group, and 0% and 8.7% in the MED group, respectively (P=0.631)." ], "LABELS": [ "BACKGROUND", "METHODS", "RESULTS" ], "MESHES": [ "Adult", "Disease-Free Survival", "Esophageal and Gastric Varices", "Female", "Gastrointestinal Hemorrhage", "Hemostatic Techniques", "Humans", "Hypertension, Portal", "Male", "Middle Aged", "Recurrence", "Retrospective Studies", "Splenectomy", "Time Factors", "Treatment Outcome", "Vascular Surgical Procedures", "Venous Pressure", "Venous Thrombosis" ], "YEAR": "2015", "reasoning_required_pred": "yes", "reasoning_free_pred": "yes", "final_decision": "yes", "LONG_ANSWER": "EDWS is a safe and effective treatment for esophagogastric varices secondary to portal hypertension in selected patients. Patients treated with EDWS had a lower complication rate of portal venous system thrombosis compared with those treated with conventional MED." }, "25034219": { "QUESTION": "Does maternal obesity have an influence on feeding behavior of obese children?", "CONTEXTS": [ "Obese children and adolescents referred to the pediatric endocrinology department were enrolled consecutively. Height and weight of all children and their mothers were measured. Maternal feeding practices were measured using an adapted version of the Child Feeding Questionnaire (CFQ). Answers were compared between obese (Body Mass Index [BMI] \u2265 30 kg/m2) and non-obese mothers.", "A total of 491 obese subjects (292 girls, mean age 12.0 \u00b1 2.8 years) and their mothers participated in this study. A direct correlation between children's BMI and their mothers' BMI was found (P<0.001) both in girls (r = 0.372) and boys (r = 0.337). While 64.4% of mothers were found obese in the study, only half of them consider themselves as obese. No difference were found in the scores of the subscales \"perceived responsibility\", \"restriction\", \"concern for child's weight\" and \"monitoring\" between obese and non-obese mothers. Child's BMI-SDS positively correlated with mothers' personal weight perception, concern for child's weight and restriction after adjustment for child's age (P<0.001, P = 0.012 and P = 0.002, respectively)." ], "LABELS": [ "METHODS", "RESULTS" ], "MESHES": [ "Adolescent", "Body Mass Index", "Child", "Cross-Sectional Studies", "Feeding Behavior", "Female", "Humans", "Male", "Mother-Child Relations", "Mothers", "Obesity", "Pediatric Obesity", "Prevalence", "Surveys and Questionnaires" ], "YEAR": "2015", "reasoning_required_pred": "yes", "reasoning_free_pred": "no", "final_decision": "no", "LONG_ANSWER": "Mothers' BMI highly correlate with children's BMI-z-scores. The degree of child's obesity increases mothers' concern and food restriction behavior. While mothers of obese children have a high prevalence of obesity, maternal obesity was found to have no significant influence on feeding behavior of obese school children." }, "25521278": { "QUESTION": "Is plate clearing a risk factor for obesity?", "CONTEXTS": [ "Identifying eating behaviors which contribute to excess weight gain will inform obesity prevention strategies. A tendency to clear one's plate when eating may be a risk factor for obesity in an environment where food is plentiful. Whether plate clearing is associated with increased body weight in a cohort of US participants was examined.", "Nine hundred and ninety-three US adults (60% male, 80% American European, mean age=31 years) completed self-report measures of habitual plate clearing together with behavioral and demographic characteristics known to be associated with obesity.", "Plate clearing tendencies were positively associated with BMI and remained so after accounting for a large number of other demographic and behavioral predictors of BMI in analyses (\u03b2=0.18, 95% CIs=0.07, 0.29, P<0.001); an increased tendency to plate clear was associated with a significantly higher body weight." ], "LABELS": [ "OBJECTIVES", "METHODS", "RESULTS" ], "MESHES": [ "Adolescent", "Adult", "Aged", "Body Mass Index", "Body Weight", "Cross-Sectional Studies", "Feeding Behavior", "Female", "Health Surveys", "Humans", "Male", "Middle Aged", "Obesity", "Risk Factors", "Self Report", "Surveys and Questionnaires", "United States", "Weight Gain", "Young Adult" ], "YEAR": "2015", "reasoning_required_pred": "yes", "reasoning_free_pred": "yes", "final_decision": "yes", "LONG_ANSWER": "The tendency to clear one's plate when eating is associated with increased body weight and may constitute a risk factor for weight gain." }, "27908583": { "QUESTION": "Should lower limb fractures be treated surgically in patients with chronic spinal injuries?", "CONTEXTS": [ "To report the outcomes of surgical treatment of lower limb fractures in patients with chronic spinal cord injuries.", "A total of 37 lower limb fractures were treated from 2003 to 2010, of which 25 fractures were treated surgically and 12 orthopaedically.", "Patients of the surgical group had better clinical results, range of motion, bone consolidation, and less pressure ulcers and radiological misalignment. No differences were detected between groups in terms of pain, hospital stay, and medical complications.", "There is no currently consensus regarding the management of lower limb fractures in patients with chronic spinal cord injuries, but the trend has been conservative treatment due to the high rate of complications in surgical treatment." ], "LABELS": [ "OBJECTIVE", "MATERIAL AND METHOD", "RESULTS", "DISCUSSION" ], "MESHES": [ "Adult", "Female", "Femoral Fractures", "Fracture Fixation", "Humans", "Male", "Middle Aged", "Retrospective Studies", "Spinal Cord Injuries", "Tibial Fractures", "Treatment Outcome" ], "YEAR": null, "reasoning_required_pred": "yes", "reasoning_free_pred": "yes", "final_decision": "yes", "LONG_ANSWER": "Chronic spinal cord injuries patients with lower limb fractures who are treated surgically achieved a more reliable consolidation, practically a free range of motion, low rate of cutaneous complications, and pain associated with the fracture. This allows a quick return to the previous standard of living, and should be considered as an alternative to orthopaedic treatment in these patients." }, "23422012": { "QUESTION": "Is vancomycin MIC creep a worldwide phenomenon?", "CONTEXTS": [ "Vancomycin is the primary treatment for infections caused by methicilin-resistant Staphylococcus aureus (MRSA). The association of vancomycin treatment failures with increased vancomycin minimum inhibitory concentration (MIC) is a well-recognized problem. A number of single-centre studies have identified progressive increases in glycopeptide MICs for S. aureus strains over recent years - a phenomenon known as vancomycin MIC creep. It is unknown if this is a worldwide phenomenon or if it is localized to specific centers.", "The aim of this study was to evaluate the trend of vancomycin MIC for isolates of MRSA over a 3-year period in a tertiary university hospital in Portugal. MRSA isolates from samples of patients admitted from January 2007 to December 2009 were assessed. Etest method was used to determine the respective vancomycin MIC. Only one isolate per patient was included in the final analysis.", "A total of 93 MRSA isolates were studied. The vancomycin MICs were 0.75, 1, 1.5 and 2 mg/L for 1 (1.1%), 19 (20.4%), 38 (40.9%), 35 (37.6%) isolates, respectively. During the 3 year period, we observed a significant fluctuation in the rate of MRSA with a vancomycin MIC\u2009>\u20091 mg/L (2007: 86.2%; 2008: 93.3%; 2009: 58.8%, p\u2009=\u20090.002). No MRSA isolate presented a MIC\u2009>\u20092 mg/L." ], "LABELS": [ "BACKGROUND", "METHODS", "RESULTS" ], "MESHES": [ "Anti-Bacterial Agents", "Hospitals, University", "Humans", "Microbial Sensitivity Tests", "Staphylococcus aureus", "Vancomycin" ], "YEAR": "2013", "reasoning_required_pred": "yes", "reasoning_free_pred": "no", "final_decision": "no", "LONG_ANSWER": "We were unable to find in our institution data compatible to the presence of vancomycin MIC creep during the study period. This phenomenon seems not to be generalized; as a result each institution should systematically monitor MRSA vancomycin MIC over time." }, "11500608": { "QUESTION": "Does short-term treatment with proton pump inhibitors cause rebound aggravation of symptoms?", "CONTEXTS": [ "Rebound acid hypersecretion might occur after treatment with proton pump inhibitors. This study looks for a rebound aggravation of symptoms after short-term treatment with lansoprazole.STUDY: Sixty-two patients (19 men and 43 women; mean age, 54 years; range, 32-77 years) with heartburn and regurgitation and normal upper endoscopy findings were studied in a randomized, double-blind, placebo-controlled trial with a crossover design. There were two 5-day treatment periods with lansoprazole 60 mg once daily or placebo in random order, separated by a 9-day washout period. Reflux, total, and antacid scores were calculated for each of the treatment periods. Higher scores during the placebo period in the group given lansoprazole first than in the group given placebo first indicated a rebound aggravation of symptoms.", "The mean symptom scores during the placebo period in the groups given lansoprazole first and placebo first were as follows: reflux score, 21.5 and 17.6, respectively (not significant); total score, 11.2 and 10.3, respectively (not significant); and antacid score, 8.2 and 7.2, respectively (not significant)." ], "LABELS": [ "BACKGROUND", "RESULTS" ], "MESHES": [ "2-Pyridinylmethylsulfinylbenzimidazoles", "Cross-Over Studies", "Double-Blind Method", "Enzyme Inhibitors", "Female", "Gastric Acid", "Gastroesophageal Reflux", "Heartburn", "Humans", "Lansoprazole", "Male", "Middle Aged", "Omeprazole", "Proton Pump Inhibitors", "Time Factors" ], "YEAR": "2001", "reasoning_required_pred": "no", "reasoning_free_pred": "no", "final_decision": "no", "LONG_ANSWER": "There is no indication of a rebound aggravation of symptoms 12 to 14 days after a 5-day treatment with lansoprazole 60 mg once daily in patients with reflux symptoms." }, "8111516": { "QUESTION": "Do family physicians make good sentinels for influenza?", "CONTEXTS": [ "To determine whether volunteer family physician reports of the frequency of influenza-like illness (ILI) usefully supplement information from other influenza surveillance systems conducted by the Centers for Disease Control and Prevention.", "Evaluation of physician reports from five influenza surveillance seasons (1987-88 through 1991-92).", "Family physician office practices in all regions of the United States.", "An average of 140 physicians during each of five influenza seasons.", "None.", "An office visit or hospitalization of a patient for ILI, defined as presence of fever (temperature>or = 37.8 degrees C) and cough, sore throat, or myalgia, along with the physician's clinical judgment of influenza. A subset of physicians collected specimens for confirmation of influenza virus by culture.", "Physicians attributed 81,408 (5%) of 1,672,542 office visits to ILI; 2754 (3%) patients with ILI were hospitalized. Persons 65 years of age and older accounted for 11% of visits for ILI and 43% of hospitalizations for ILI. In three of five seasons, physicians obtained influenza virus isolates from a greater proportion of specimens compared with those processed by World Health Organization laboratories (36% vs 12%). Influenza virus isolates from sentinel physicians peaked from 1 to 4 weeks earlier than those reported by World Health Organization laboratories. Physicians reported peak morbidity 1 to 4 weeks earlier than state and territorial health departments in four of five seasons and 2 to 5 weeks earlier than peak mortality reported by 121 cities during seasons with excess mortality associated with pneumonia and influenza." ], "LABELS": [ "OBJECTIVE", "DESIGN", "SETTING", "PARTICIPANTS", "INTERVENTIONS", "OUTCOME MEASURES", "RESULTS" ], "MESHES": [ "Adolescent", "Adult", "Aged", "Centers for Disease Control and Prevention (U.S.)", "Child", "Child, Preschool", "Family Practice", "Humans", "Infant", "Influenza, Human", "Middle Aged", "Population Surveillance", "United States" ], "YEAR": "1993", "reasoning_required_pred": "yes", "reasoning_free_pred": "yes", "final_decision": "yes", "LONG_ANSWER": "Family physicians provide sensitive, timely, and accurate community influenza morbidity data that complement data from other surveillance systems. This information enables monitoring of the type, timing, and intensity of influenza activity and can help health care workers implement prevention or control measures." }, "25604390": { "QUESTION": "Aberrant loss of dickkopf-3 in gastric cancer: can it predict lymph node metastasis preoperatively?", "CONTEXTS": [ "Dickkopf-3 (DKK3) may act as a tumor suppressor as it is down-regulated in various types of cancer. This study assessed the DKK3 protein expression in gastric cancer and its potential value as a prognostic marker.", "DKK3 expression was evaluated by immunohistochemistry in 158 gastric cancer samples from patients who underwent gastrectomy from 2002 to 2008. Clinicopathological parameters and survival data were analyzed.", "Loss of DKK3 expression was found in 64 of 158 (40.5%) samples, and it was associated with advanced T stage (p<0.001), lymph node metastasis (p<0.001), UICC TNM stage (p<0.001), tumor location (p = 0.029), lymphovascular invasion (p = 0.035), and perineural invasion (p = 0.032). Patients without DKK3 expression in tumor cells had a significantly worse disease-free and overall survival than those with DKK3 expression (p<0.001, and p = 0.001, respectively). TNM stage (p = 0.028 and p<0.001, respectively) and residual tumor (p<0.001 and p = 0.003, respectively) were independent predictors of disease-free and overall survival. Based on the preoperative clinical stage assessed by computed tomography (CT), loss of DKK3 expression was predominantly associated with worse prognosis in patients with clinically node-negative advanced gastric cancer (AGC). The combination of DKK3 expression status and CT increased the accuracy of CT staging for predicting lymph node involvement from 71.5 to 80.0% in AGC patients." ], "LABELS": [ "BACKGROUND", "METHODS", "RESULTS" ], "MESHES": [ "Adenocarcinoma", "Aged", "Disease-Free Survival", "Female", "Gastrectomy", "Humans", "Immunohistochemistry", "Intercellular Signaling Peptides and Proteins", "Lymphatic Metastasis", "Male", "Middle Aged", "Neoplasm Invasiveness", "Neoplasm Staging", "Neoplasm, Residual", "Preoperative Period", "Retrospective Studies", "Stomach Neoplasms", "Survival Rate", "Tomography, X-Ray Computed" ], "YEAR": "2015", "reasoning_required_pred": "yes", "reasoning_free_pred": "yes", "final_decision": "yes", "LONG_ANSWER": "Loss of DKK3 protein expression was significantly associated with poor survival in patients with gastric cancer and was strongly correlated with the TNM stage. DKK3 might be a potential biomarker of lymph node involvement that can improve the predictive power of CT." }, "25985014": { "QUESTION": "Is ganglionated plexi ablation during Maze IV procedure beneficial for postoperative long-term stable sinus rhythm?", "CONTEXTS": [ "We investigated the role of surgical ablation targeting the autonomous nervous system during a Cox-Maze IV procedure in the maintenance of sinus rhythm at long-term follow-up.", "The patient population consisted of 519 subjects with persistent or long-standing persistent atrial fibrillation (AF) undergoing radiofrequency Maze IV during open heart surgery between January 2006 and July 2013 at three institutions without (Group 1) or with (Group 2) ganglionated plexi (GP) ablation. Recurrence of atrial fibrillation off-antiarrhythmic drugs was the primary outcome. Predictors of AF recurrence were evaluated by means of competing risk regression. Median follow-up was 36.7 months.", "The percentage of patients in normal sinus rhythm (NSR) off-antiarrhythmic drugs did not differ between groups (Group 1-75.5%, Group 2-67.8%, p = 0.08). Duration of AF \u2265 38 months (p = 0.01), left atrial diameter \u2265 54 mm (0.001), left atrial area \u2265 33 cm(2) (p = 0.005), absence of connecting lesions (p= 0.04), and absence of right atrial ablation (p<0.001) were independently associated with high incidence of AF recurrence. In contrast the absence of GP ablation was not a significant factor (p = 0.12)." ], "LABELS": [ "BACKGROUND", "METHODS", "RESULTS" ], "MESHES": [ "Aged", "Atrial Fibrillation", "Autonomic Denervation", "Autonomic Nervous System", "Catheter Ablation", "Female", "Heart Rate", "Humans", "Male", "Middle Aged", "Pericardium", "Postoperative Period", "Retrospective Studies" ], "YEAR": "2015", "reasoning_required_pred": "no", "reasoning_free_pred": "no", "final_decision": "no", "LONG_ANSWER": "GP ablation did not prove to be beneficial for postoperative stable NSR. A complete left atrial lesion set and biatrial ablation are advisable for improving rhythm outcomes. Randomized controlled trials are necessary to confirm our findings." }, "14627582": { "QUESTION": "Double reading of barium enemas: is it necessary?", "CONTEXTS": [ "The purpose of our study was to determine the effectiveness, clinical impact, and feasibility of double reading barium enemas.", "Independent double readings of 1,003 consecutive barium enemas (822 double- and 181 single-contrast examinations) were prospectively performed. From this pool of 1,003 examinations, 994 were included in our study. Examinations showing at least one polyp or carcinoma 5 mm or larger were considered to have positive results. For combined readings, results were considered positive if either of the two interpreters reported finding a polyp or carcinoma. A McNemar test was used to compare the first reader's results with the combined results of the first and second readers. Results were retrospectively correlated with endoscopic or surgical results in 360 patients, and agreement between first and combined readings and endoscopic results was determined.", "Adding a second reader increased the number of positive results on examinations from 249 to 315 (p<0.0001) and resulted in potential alteration of clinical treatment in 98 patients (9.9%). Sensitivity of the first and combined readings for detection of all lesions was identical, 76.3% (95% CI, 65.4-87.1%). Specificity decreased from 91.0% (95% CI, 87.9-94.3%) for the first reading to 86.4% (95% CI, 82.2-90.0%) for the combined reading. The overall measurement of agreement decreased from a kappa value of 61.8 (95% CI, 51.2-72.4%) for the first reading to 52.9 (95% CI, 42.2-63.6%) for the combined reading. The second reading required an average of 3.3 min. Sensitivity for the detection of adenocarcinomas was 100%." ], "LABELS": [ "OBJECTIVE", "MATERIALS AND METHODS", "RESULTS" ], "MESHES": [ "Adult", "Barium Sulfate", "Colonic Polyps", "Colorectal Neoplasms", "Contrast Media", "Enema", "False Positive Reactions", "Female", "Humans", "Male", "Middle Aged", "Observer Variation", "Prospective Studies", "Radiography", "Reproducibility of Results", "Sensitivity and Specificity" ], "YEAR": "2003", "reasoning_required_pred": "no", "reasoning_free_pred": "no", "final_decision": "no", "LONG_ANSWER": "Although feasible, double reading of barium enemas does not improve sensitivity for detection of polyps and produces a higher false-positive rate." }, "18570208": { "QUESTION": "Is severe macrosomia manifested at 11-14 weeks of gestation?", "CONTEXTS": [ "To determine the association between fetal biometry in the first or early second trimester and severe macrosomia at delivery.", "This case-control study included 30 term severely macrosomic neonates; 90 appropriate-for-gestational age (AGA) neonates served as controls. All pregnancies underwent nuchal translucency (NT) screening at 11-14 weeks' gestation. Pregnancies were dated by accurate last menstrual period consistent with crown-rump length (CRL) measurements at the time of screening, early pregnancy CRL or date of fertilization. The association between birth weight and the difference between the measured and the expected CRL at the time of NT screening was analyzed.", "The difference between measured and expected CRL, expressed both in mm and in days of gestation, was statistically greater in the severely macrosomic neonates compared with controls (mean, 6.66 +/- 4.78 mm vs. 1.17 +/- 4.6 mm, P<0.0001 and 3 +/- 2.2 days vs. 0.5 +/- 2.3 days, P<0.0001, respectively). Furthermore, there were significant correlations between the extent of macrosomia and the discrepancy between expected and measured fetal size at the time of NT screening (r = 0.47, P<0.01 and r = 0.48, P<0.01, respectively)." ], "LABELS": [ "OBJECTIVE", "METHODS", "RESULTS" ], "MESHES": [ "Biometry", "Case-Control Studies", "Crown-Rump Length", "Female", "Fetal Macrosomia", "Humans", "Infant, Newborn", "Nuchal Translucency Measurement", "Pregnancy", "Pregnancy Trimester, First", "Pregnancy Trimester, Second", "Retrospective Studies", "Severity of Illness Index" ], "YEAR": "2008", "reasoning_required_pred": "yes", "reasoning_free_pred": "yes", "final_decision": "yes", "LONG_ANSWER": "Severe macrosomia apparently manifests as early as 11-14 weeks' gestation." }, "23448747": { "QUESTION": "Do older adults with cancer fall more often?", "CONTEXTS": [ "To examine whether a history of cancer increased the likelihood of a fall in community-dwelling older adults, and if cancer type, stage, or time since diagnosis increased falls.", "A longitudinal, retrospective, cohort study.", "A home- and community-based waiver program in Michigan.", "862 older adults aged 65 years or older with cancer compared to 8,617 older adults without cancer using data from the Minimum Data Set-Home Care and Michigan cancer registry.", "Reports of falls were examined for 90-180 days. Generalized estimating equations were used to compare differences between the groups.", "Cancer, falls, patient characteristics, comorbidities, medications, pain, weight loss, vision, memory recall, and activities, as well as cancer type, stage, and time since diagnosis.", "A fall occurred at a rate of 33% in older adults with cancer compared to 29% without cancer (p<0.00). Those with a history of cancer were more likely to fall than those without cancer (adjusted odds ratio 1.16; 95% confidence interval [1.02, 1.33]; p = 0.03). No differences in fall rates were determined by cancer type or stage, and the odds of a fall did not increase when adding time since cancer diagnosis." ], "LABELS": [ "OBJECTIVES", "DESIGN", "SETTING", "SAMPLE", "METHODS", "MAIN RESEARCH VARIABLES", "FINDINGS" ], "MESHES": [ "Accidental Falls", "Aged", "Aged, 80 and over", "Aging", "Comorbidity", "Female", "Humans", "Longitudinal Studies", "Male", "Michigan", "Neoplasms", "Oncology Nursing", "Prevalence", "Registries", "Residence Characteristics", "Retrospective Studies", "Risk Factors" ], "YEAR": "2013", "reasoning_required_pred": "yes", "reasoning_free_pred": "yes", "final_decision": "yes", "LONG_ANSWER": "The fall rate was higher in older adults with cancer than in older adults without cancer." }, "20156655": { "QUESTION": "Scrotal approach to both palpable and impalpable undescended testes: should it become our first choice?", "CONTEXTS": [ "To determine the advantages of scrotal incision in the treatment of undescended testis. Undescended testis is a common pediatric condition and is conventionally managed surgically by orchidopexy. A single scrotal incision orchidopexy has become accepted as a valid approach for patients with palpable undescended testicles. Because this approach also allows easy detection of atrophic testes or testicular remnants, it recently has also emerged as an alternative initial surgical approach to impalpable undescended testicles.", "All orchidopexies performed between 2004 and 2008 at our university hospital were prospectively included in this study. A total of 194 scrotal orchidopexies were performed in 154 patients (mean age, 71 months; range, 4-229 months). In all cases a scrotal approach was chosen irrespective of the initial position or presence of an open processus vaginalis. Testicular position was examined at follow-up after a mean period of 10 months (3-22 months).", "Overall, 36 of the 46 impalpable testicles (78%) could be diagnosed and treated accordingly, using only a scrotal incision. Conversion to laparoscopy was needed in 4 cases. A limited number of postoperative complications were seen. In all cases, the testes were palpable and remained in the scrotum on follow-up." ], "LABELS": [ "OBJECTIVES", "METHODS", "RESULTS" ], "MESHES": [ "Adolescent", "Child", "Child, Preschool", "Cryptorchidism", "Humans", "Infant", "Male", "Palpation", "Prospective Studies", "Scrotum", "Urologic Surgical Procedures, Male", "Young Adult" ], "YEAR": "2010", "reasoning_required_pred": "yes", "reasoning_free_pred": "yes", "final_decision": "yes", "LONG_ANSWER": "Initial single scrotal incision can be recommended for orchidopexy, even in the more difficult cases of impalpable undescended testes. Advantages seem to include shorter operative time, a cosmetically appealing single incision, and possibly less pain. The scrotal incision technique significantly reduces the need for laparoscopy in impalpable testes. Surprisingly, it even allows successful orchidopexy of abdominal testes, provided an open processus is present." }, "16147837": { "QUESTION": "Is grandmultiparity an independent risk factor for adverse perinatal outcomes?", "CONTEXTS": [ "To compare maternal and neonatal outcomes among grandmultiparous women to those of multiparous women 30 years or older.", "A database of the vast majority of maternal and newborn hospital discharge records linked to birth/death certificates was queried to obtain information on all multiparous women with a singleton delivery in the state of California from January 1, 1997 through December 31, 1998. Maternal and neonatal pregnancy outcomes of grandmultiparous women were compared to multiparous women who were 30 years or older at the time of their last birth.", "The study population included 25,512 grandmultiparous and 265,060 multiparous women 30 years or older as controls. Grandmultiparous women were predominantly Hispanic (56%). After controlling for potential confounding factors, grandmultiparous women were at significantly higher risk for abruptio placentae (odds ratio OR: 1.3; 95% confidence intervals CI: 1.2-1.5), preterm delivery (OR: 1.3; 95% CI: 1.2-1.4), fetal macrosomia (OR: 1.5; 95% CI: 1.4-1.6), neonatal death (OR: 1.5; 95% CI: 1.3-1.8), postpartum hemorrhage (OR: 1.2; 95% CI: 1.1-1.3) and blood transfusion (OR: 1.5; 95% CI: 1.3-1.8)." ], "LABELS": [ "OBJECTIVE", "METHODS", "RESULTS" ], "MESHES": [ "Female", "Humans", "Parity", "Pregnancy", "Pregnancy Complications", "Pregnancy Outcome", "Retrospective Studies", "Risk Factors" ], "YEAR": "2005", "reasoning_required_pred": "yes", "reasoning_free_pred": "yes", "final_decision": "yes", "LONG_ANSWER": "Grandmultiparous women had increased maternal and neonatal morbidity, and neonatal mortality even after controlling for confounders, suggesting a need for closer observation than regular multiparous patients during labor and delivery." }, "16249670": { "QUESTION": "Does the investing layer of the deep cervical fascia exist?", "CONTEXTS": [ "The placement of the superficial cervical plexus block has been the subject of controversy. Although the investing cervical fascia has been considered as an impenetrable barrier, clinically, the placement of the block deep or superficial to the fascia provides the same effective anesthesia. The underlying mechanism is unclear. The aim of this study was to investigate the three-dimensional organization of connective tissues in the anterior region of the neck.", "Using a combination of dissection, E12 sheet plastination, and confocal microscopy, fascial structures in the anterior cervical triangle were examined in 10 adult human cadavers.", "In the upper cervical region, the fascia of strap muscles in the middle and the fasciae of the submandibular glands on both sides formed a dumbbell-like fascia sheet that had free lateral margins and did not continue with the sternocleidomastoid fascia. In the lower cervical region, no single connective tissue sheet extended directly between the sternocleidomastoid muscles. The fascial structure deep to platysma in the anterior cervical triangle comprised the strap fascia." ], "LABELS": [ "BACKGROUND", "METHODS", "RESULTS" ], "MESHES": [ "Aged", "Aged, 80 and over", "Cervical Plexus", "Collagen", "Connective Tissue", "Epoxy Compounds", "Female", "Humans", "Male", "Microscopy, Confocal", "Neck", "Neck Muscles", "Plastic Embedding" ], "YEAR": "2005", "reasoning_required_pred": "yes", "reasoning_free_pred": "no", "final_decision": "no", "LONG_ANSWER": "This study provides anatomical evidence to indicate that the so-called investing cervical fascia does not exist in the anterior triangle of the neck. Taking the previous reports together, the authors' findings strongly suggest that deep potential spaces in the neck are directly continuous with the subcutaneous tissue." }, "24495711": { "QUESTION": "Is crime associated with over-the-counter pharmacy syringe sales?", "CONTEXTS": [ "More than 50,000 new HIV infections occur annually in the United States. Injection drug users represent twelve percent of incident HIV infections each year. Pharmacy sales of over-the-counter (OTC) syringes have helped prevent HIV transmission among injection drug users in many states throughout the United States. However, concerns exist among some law enforcement officials, policymakers, pharmacists, and community members about potential links between OTC syringe sales and crime.", "We used a geographic information system and novel spatial and longitudinal analyses to determine whether implementation of pharmacy-based OTC syringe sales were associated with reported crime between January 2006 and December 2008 in Los Angeles Police Department Reporting Districts. We assessed reported crime pre- and post-OTC syringe sales initiation as well as longitudinal associations between crime and OTC syringe-selling pharmacies.", "By December 2008, 9.3% (94/1010) of Los Angeles Police Department Reporting Districts had at least one OTC syringe-selling pharmacy. Overall reported crime counts and reported crime rates decreased between 2006 and 2008 in all 1010 Reporting Districts. Using generalized estimating equations and adjusting for potential confounders, reported crime rates were negatively associated with OTC syringe sales (adjusted rate ratio: 0.89; 95% confidence interval: 0.81, 0.99)." ], "LABELS": [ "BACKGROUND", "METHODS", "RESULTS" ], "MESHES": [ "Commerce", "Crime", "Drug Users", "Geographic Information Systems", "Los Angeles", "Models, Statistical", "Pharmacies", "Syringes" ], "YEAR": "2014", "reasoning_required_pred": "yes", "reasoning_free_pred": "no", "final_decision": "yes", "LONG_ANSWER": "Our findings demonstrate that OTC pharmacy syringe sales were not associated with increases in reported crime in local communities in Los Angeles during 2006-2008." }, "20101129": { "QUESTION": "Is prophylactic fixation a cost-effective method to prevent a future contralateral fragility hip fracture?", "CONTEXTS": [ ": A previous hip fracture more than doubles the risk of a contralateral hip fracture. Pharmacologic and environmental interventions to prevent hip fracture have documented poor compliance. The purpose of this study was to examine the cost-effectiveness of prophylactic fixation of the uninjured hip to prevent contralateral hip fracture.", ": A Markov state-transition model was used to evaluate the cost and quality-adjusted life-years (QALYs) for unilateral fixation of hip fracture alone (including internal fixation or arthroplasty) compared with unilateral fixation and contralateral prophylactic hip fixation performed at the time of hip fracture or unilateral fixation and bilateral hip pad protection. Prophylactic fixation involved placement of a cephalomedullary nail in the uninjured hip and was initially assumed to have a relative risk of a contralateral fracture of 1%. Health states included good health, surgery-related complications requiring a second operation (infection, osteonecrosis, nonunion, and malunion), fracture of the uninjured hip, and death. The primary outcome measure was the incremental cost-effectiveness ratio estimated as cost per QALY gained in 2006 US dollars with incremental cost-effectiveness ratios below $50,000 per QALY gained considered cost-effective. Sensitivity analyses evaluated the impact of patient age, annual mortality and complication rates, intervention effectiveness, utilities, and costs on the value of prophylactic fixation.", ": In the baseline analysis, in a 79-year-old woman, prophylactic fixation was not found to be cost-effective (incremental cost-effectiveness ratio = $142,795/QALY). However, prophylactic fixation was found to be a cost-effective method to prevent contralateral hip fracture in: 1) women 71 to 75 years old who had 30% greater relative risk for a contralateral fracture; and 2) women younger than age 70 years. Cost-effectiveness was greater when the additional costs of prophylaxis were less than $6000. However, for most analyses, the success of prophylactic fixation was highly sensitive to the effectiveness and the relative morbidity and mortality of the additional procedure." ], "LABELS": [ "OBJECTIVE", "METHODS", "RESULTS" ], "MESHES": [ "Age Factors", "Aged", "Aged, 80 and over", "Bone Nails", "Cost-Benefit Analysis", "Female", "Fracture Fixation, Internal", "Hip Fractures", "Humans", "Male", "Markov Chains", "Middle Aged", "Primary Prevention", "Protective Clothing", "Quality-Adjusted Life Years", "Sex Factors" ], "YEAR": "2010", "reasoning_required_pred": "maybe", "reasoning_free_pred": "maybe", "final_decision": "maybe", "LONG_ANSWER": ": Prophylactic fixation with a cephalomedullary nail was not found to be cost-effective for the average older woman who sustained a hip fracture. However, it may be appropriate for select patient populations. The study supports the need for basic science and clinical trials investigating the effectiveness of prophylactic fixation for patient populations at higher lifetime risk for contralateral hip fracture." }, "17593459": { "QUESTION": "Is double-balloon enteroscopy an accurate method to diagnose small-bowel disorders?", "CONTEXTS": [ "The aim of this study was to analyze the contribution of the double-balloon enteroscopy (DBE) for diagnosis of the small bowel disorders.", "Forty-four patients (20 women, 24 men; mean age 53.5 years-old, range 21-89 years) with chronic gastrointestinal bleeding, diarrhea, polyposis, weight-loss, Roux-en-Y surgery, and other indications underwent DBE.", "Twenty patients had occult or obscure gastrointestinal bleeding. The source of bleeding was identified in 15/20 (75%): multiple angiodysplasias in four, arterial-venous malformation beyond the ligament of Treitz in two that could be treated with injection successfully. Other diagnoses included: duodenal adenocarcinoma, jejunal tuberculosis, erosions and ulcer of the jejunum. Of 24 patients with other indications, the diagnosis could be achieved in 18 of them (75%), including: two lymphomas, plasmocytoma, Gardner's syndrome, Peutz-Jeghers' syndrome, familial adenomatous polyposis, Beh\u00e7et's disease, jejunal submucosal lesion, lymphangiectasia due to blastomycosis and unspecific chronic jejunitis. Of three cases with Roux-en-Y reconstruction, two underwent DBE in order to perform biopsies of the excluded duodenum. Additionally, two patients underwent DBE to exclude Crohn's disease and lymphoma of the small bowel. The mean length of small bowel examination was 240 +/- 50 cm during a single approach. The diagnostic yield was 75% (33/44 cases) and therapeutic yield was 63.6%. No major complications were observed, only minor complication such as sore throat in 4/44 (9.1%)." ], "LABELS": [ "BACKGROUND AND STUDY AIMS", "PATIENTS AND METHODS", "RESULTS" ], "MESHES": [ "Adult", "Aged", "Aged, 80 and over", "Catheterization", "Endoscopy, Gastrointestinal", "Female", "Humans", "Intestinal Diseases", "Intestine, Small", "Male", "Middle Aged", "Time Factors" ], "YEAR": "2007", "reasoning_required_pred": "yes", "reasoning_free_pred": "yes", "final_decision": "yes", "LONG_ANSWER": "1. DBE is a safe and and accurate method to diagnose small bowel disorders; 2. this method permits chromoscopy, biopsies and treatment of the lesions." }, "24577079": { "QUESTION": "Does strategy training reduce age-related deficits in working memory?", "CONTEXTS": [ "Older adults typically perform worse on measures of working memory (WM) than do young adults; however, age-related differences in WM performance might be reduced if older adults use effective encoding strategies.", "The purpose of the current experiment was to evaluate WM performance after training individuals to use effective encoding strategies.", "Participants in the training group (older adults: n = 39; young adults: n = 41) were taught about various verbal encoding strategies and their differential effectiveness and were trained to use interactive imagery and sentence generation on a list-learning task. Participants in the control group (older: n = 37; young: n = 38) completed an equally engaging filler task. All participants completed a pre- and post-training reading span task, which included self-reported strategy use, as well as two transfer tasks that differed in the affordance to use the trained strategies - a paired-associate recall task and the self-ordered pointing task.", "Both young and older adults were able to use the target strategies on the WM task and showed gains in WM performance after training. The age-related WM deficit was not greatly affected, however, and the training gains did not transfer to the other cognitive tasks. In fact, participants attempted to adapt the trained strategies for a paired-associate recall task, but the increased strategy use did not benefit their performance." ], "LABELS": [ "BACKGROUND", "OBJECTIVE", "METHODS", "RESULTS" ], "MESHES": [ "Adolescent", "Adult", "Aged", "Aging", "Cognition", "Female", "Humans", "Learning", "Male", "Memory, Short-Term", "Mental Recall", "Middle Aged", "Reading", "Task Performance and Analysis", "Transfer (Psychology)", "Young Adult" ], "YEAR": "2014", "reasoning_required_pred": "no", "reasoning_free_pred": "yes", "final_decision": "no", "LONG_ANSWER": "Strategy training can boost WM performance, and its benefits appear to arise from strategy-specific effects and not from domain-general gains in cognitive ability." }, "9582182": { "QUESTION": "Does the SCL 90-R obsessive-compulsive dimension identify cognitive impairments?", "CONTEXTS": [ "To investigate the relevance of the Symptom Checklist 90-R Obsessive-Compulsive subscale to cognition in individuals with brain tumor.", "A prospective study of patients assessed with a neuropsychological test battery.", "A university medical center.", "Nineteen adults with biopsy-confirmed diagnoses of malignant brain tumors were assessed prior to aggressive chemotherapy.", "Included in the assessment were the Mattis Dementia Rating Scale, California Verbal Learning Test, Trail Making Test B, Symptom Checklist 90-R, Mood Assessment Scale, Beck Anxiety Inventory, and Chronic Illness Problem Inventory.", "The SCL 90-R Obsessive-Compulsive subscale was not related to objective measures of attention, verbal memory, or age. It was related significantly to symptoms of depression (r = .81, P<.005), anxiety (r = .66, P<.005), and subjective complaints of memory problems (r = .75, P<.005). Multivariate analyses indicated that reported symptoms of depression contributed 66% of the variance in predicting SCL 90-R Obsessive-Compulsive Scores, whereas symptoms of anxiety contributed an additional 6% (P<.0001)." ], "LABELS": [ "OBJECTIVE", "DESIGN", "SETTING", "PATIENTS", "MAIN OUTCOME MEASURES", "RESULTS" ], "MESHES": [ "Adult", "Aged", "Brain Neoplasms", "Cognition Disorders", "Female", "Humans", "Male", "Middle Aged", "Multivariate Analysis", "Neuropsychological Tests", "Obsessive-Compulsive Disorder", "Personality Assessment", "Predictive Value of Tests", "Prospective Studies", "Sensitivity and Specificity" ], "YEAR": "1998", "reasoning_required_pred": "no", "reasoning_free_pred": "yes", "final_decision": "yes", "LONG_ANSWER": "Our data suggest that the SCL 90-R is best viewed as an indicator of unidimensional emotional distress and somatic effects of structural brain injury." }, "17715311": { "QUESTION": "Is fetal anatomic assessment on follow-up antepartum sonograms clinically useful?", "CONTEXTS": [ "The purpose of this study was to evaluate the clinical usefulness of a fetal anatomic survey on follow-up antepartum sonograms.", "A retrospective follow-up study was conducted at a low-risk maternity clinic from July 1, 2005, to June 30, 2006. Eligible women had at least 1 prior sonographic examination beyond 18 weeks' gestation with a complete and normal fetal anatomic assessment and at least 1 follow-up sonogram. Full fetal anatomic surveys were performed on all follow-up sonograms regardless of the indication. Neonatal charts were reviewed for those patients whose follow-up sonograms revealed unanticipated fetal anomalies. Neonatal intervention was defined as surgical or medical therapy or arranged subspecialty follow-up specifically for the suspected fetal anomaly.", "Of a total of 4269 sonographic examinations performed, 437 (10.2%) were follow-up studies. Of these, 101 (23.1%) were excluded because the initial sonogram revealed a suspected fetal anomaly, and 42 (9.8%) were excluded for other reasons. Of the remaining 294 women, 21 (7.1%) had an unanticipated fetal anomaly, most often renal pyelectasis. Compared with follow-up sonography for other reasons, repeated sonography for fetal growth evaluation yielded a higher incidence of unexpected fetal anomalies: 15 (12.3%) of 122 versus 6 (3.5%) of 172 (P = .01). When compared with the neonates in the nongrowth indications group, those neonates whose mothers had sonographic examinations for fetal growth had a higher rate of neonatal interventions: 6 (40.0%) of 15 versus 0 (0%) of 6 (P = .04)." ], "LABELS": [ "OBJECTIVE", "METHODS", "RESULTS" ], "MESHES": [ "Adult", "Chi-Square Distribution", "Congenital Abnormalities", "Female", "Fetal Diseases", "Follow-Up Studies", "Gestational Age", "Humans", "Postpartum Period", "Pregnancy", "Retrospective Studies", "Statistics, Nonparametric", "Ultrasonography, Prenatal" ], "YEAR": "2007", "reasoning_required_pred": "yes", "reasoning_free_pred": "yes", "final_decision": "yes", "LONG_ANSWER": "A fetal anatomic survey on follow-up sonograms may identify unanticipated fetal anomalies, especially when the indication is for fetal growth." }, "12094116": { "QUESTION": "Is muscle power related to running speed with changes of direction?", "CONTEXTS": [ "The purpose of this study was to identify the relationships between leg muscle power and sprinting speed with changes of direction.", "the study was designed to describe relationships between physical qualities and a component of sports performance.", "testing was conducted in an indoor sports hall and a biomechanics laboratory.", "15 male participants were required to be free of injury and have recent experience competing in sports involving sprints with changes of direction.", "subjects were timed in 8 m sprints in a straight line and with various changes of direction. They were also tested for bilateral and unilateral leg extensor muscle concentric power output by an isokinetic squat and reactive strength by a drop jump.", "The correlations between concentric power and straight sprinting speed were non-significant whereas the relationships between reactive strength and straight speed were statistically significant. Correlations between muscle power and speed while changing direction were generally low and non-significant for concentric leg power with some moderate and significant (p<0.05) coefficients found for reactive strength. The participants who turned faster to one side tended to have a reactive strength dominance in the leg responsible for the push-off action." ], "LABELS": [ "BACKGROUND", "EXPERIMENTAL DESIGN", "SETTING", "PARTICIPANTS", "MEASURES", "RESULTS" ], "MESHES": [ "Adolescent", "Adult", "Humans", "Kinesis", "Leg", "Male", "Muscle Contraction", "Muscle, Skeletal", "Reaction Time", "Running" ], "YEAR": "2002", "reasoning_required_pred": "yes", "reasoning_free_pred": "maybe", "final_decision": "yes", "LONG_ANSWER": "The relationships between leg muscle power and change-of-direction speed were not consistent. Reactive strength as measured by the drop jump appears to have some importance for lateral change-of-direction speed, possibly because of similar push-off actions. It was concluded that reactive strength of the leg extensor muscles has some importance in change-of-direction performance but the other technical and perceptual factors than influence agility performance should also be considered." }, "19430778": { "QUESTION": "Can magnetic resonance imaging accurately predict concordant pain provocation during provocative disc injection?", "CONTEXTS": [ "To correlate magnetic resonance (MR) image findings with pain response by provocation discography in patients with discogenic low back pain, with an emphasis on the combination analysis of a high intensity zone (HIZ) and disc contour abnormalities.", "Sixty-two patients (aged 17-68 years) with axial low back pain that was likely to be disc related underwent lumbar discography (178 discs tested). The MR images were evaluated for disc degeneration, disc contour abnormalities, HIZ, and endplate abnormalities. Based on the combination of an HIZ and disc contour abnormalities, four classes were determined: (1) normal or bulging disc without HIZ; (2) normal or bulging disc with HIZ; (3) disc protrusion without HIZ; (4) disc protrusion with HIZ. These MR image findings and a new combined MR classification were analyzed in the base of concordant pain determined by discography.", "Disc protrusion with HIZ [sensitivity 45.5%; specificity 97.8%; positive predictive value (PPV), 87.0%] correlated significantly with concordant pain provocation (P<0.01). A normal or bulging disc with HIZ was not associated with reproduction of pain. Disc degeneration (sensitivity 95.4%; specificity 38.8%; PPV 33.9%), disc protrusion (sensitivity 68.2%; specificity 80.6%; PPV 53.6%), and HIZ (sensitivity 56.8%; specificity 83.6%; PPV 53.2%) were not helpful in the identification of a disc with concordant pain." ], "LABELS": [ "OBJECTIVE", "MATERIALS AND METHODS", "RESULTS" ], "MESHES": [ "Adolescent", "Adult", "Aged", "Contrast Media", "Female", "Humans", "Injections", "Intervertebral Disc Displacement", "Low Back Pain", "Magnetic Resonance Imaging", "Male", "Middle Aged", "Reproducibility of Results", "Sensitivity and Specificity", "Young Adult" ], "YEAR": "2009", "reasoning_required_pred": "yes", "reasoning_free_pred": "yes", "final_decision": "yes", "LONG_ANSWER": "The proposed MR classification is useful to predict a disc with concordant pain. Disc protrusion with HIZ on MR imaging predicted positive discography in patients with discogenic low back pain." }, "12848629": { "QUESTION": "Is a 9-month treatment sufficient in tuberculous enterocolitis?", "CONTEXTS": [ "Tuberculosis has increased in parallel with the acquired immunodeficiency syndrome epidemic and the use of immunosuppressive therapy, and the growing incidence of extra-pulmonary tuberculosis, especially with intestinal involvement, reflects this trend. However, the duration of anti-tuberculous therapy has not been clarified in intestinal tuberculosis.AIM: To compare the efficacy of different treatment durations in tuberculous enterocolitis in terms of response and recurrence rates.", "Forty patients with tuberculous enterocolitis were randomized prospectively: 22 patients into a 9-month and 18 into a 15-month group. Diagnosis was made either by colonoscopic findings of discrete ulcers and histopathological findings of caseating granuloma and/or acid-fast bacilli, or by clinical improvement after therapeutic trial. Patients were followed up with colonoscopy every other month until complete response or treatment completion, and then every 6 months for 1 year and annually. Complete response was defined as a resolution of symptoms and active tuberculosis by colonoscopy.", "Complete response was obtained in all patients in both groups. Two patients in the 9-month group and one in the 15-month group underwent operation due to intestinal obstruction and perianal fistula, respectively. No recurrence of active intestinal tuberculosis occurred during the follow-up period in either group." ], "LABELS": [ "BACKGROUND", "METHODS", "RESULTS" ], "MESHES": [ "Adult", "Antitubercular Agents", "Colonoscopy", "Female", "Follow-Up Studies", "Humans", "Male", "Prospective Studies", "Recurrence", "Treatment Outcome", "Tuberculosis, Gastrointestinal" ], "YEAR": "2003", "reasoning_required_pred": "yes", "reasoning_free_pred": "yes", "final_decision": "yes", "LONG_ANSWER": "Tuberculous enterocolitis can be managed by 9-month chemotherapy without disease recurrence. Further investigations are needed in immunocompromised patients." }, "10732884": { "QUESTION": "Does coronary angiography before emergency aortic surgery affect in-hospital mortality?", "CONTEXTS": [ "To study the relationship between coronary angiography and in-hospital mortality in patients undergoing emergency surgery of the aorta without a history of coronary revascularization or coronary angiography before the onset of symptoms.", "In the setting of acute ascending aortic dissection warranting emergency aortic repair, coronary angiography has been considered to be desirable, if not essential. The benefits of defining coronary anatomy have to be weighed against the risks of additional delay before surgical intervention.", "Retrospective analysis of patient charts and the Cardiovascular Information Registry (CVIR) at the Cleveland Clinic Foundation.", "We studied 122 patients who underwent emergency surgery of the aorta between January 1982 and December 1997. Overall, in-hospital mortality was 18.0%, and there was no significant difference between those who had coronary angiography on the day of surgery compared with those who had not (No: 16%, n = 81 vs. Yes: 22%, n = 41, p = 0.46). Multivariate analysis revealed that a history of myocardial infarction (MI) was the only predictor of in-hospital mortality (relative risk: 4.98 95% confidence interval: 1.48-16.75, p = 0.009); however, coronary angiography had no impact on in-hospital mortality in patients with a history of MI. Furthermore, coronary angiography did not significantly affect the incidence of coronary artery bypass grafting (CABG) during aortic surgery (17% vs. 25%, Yes vs. No). Operative reports revealed that 74% of all CABG procedures were performed because of coronary dissection, and not coronary artery disease." ], "LABELS": [ "OBJECTIVES", "BACKGROUND", "METHODS", "RESULTS" ], "MESHES": [ "Acute Disease", "Adult", "Aged", "Aneurysm, Dissecting", "Aortic Aneurysm, Thoracic", "Combined Modality Therapy", "Coronary Aneurysm", "Coronary Angiography", "Coronary Artery Bypass", "Emergencies", "Female", "Hospital Mortality", "Humans", "Male", "Middle Aged", "Postoperative Complications", "Retrospective Studies", "Survival Analysis" ], "YEAR": "2000", "reasoning_required_pred": "no", "reasoning_free_pred": "no", "final_decision": "no", "LONG_ANSWER": "These data indicate that determination of coronary anatomy may not impact on survival in patients undergoing emergency surgery of the aorta and support the concept that once diagnosed, patients should proceed as quickly as possible to surgery." }, "23568387": { "QUESTION": "Is bicompartmental knee arthroplasty more favourable to knee muscle strength and physical performance compared to total knee arthroplasty?", "CONTEXTS": [ "Bicompartmental knee arthroplasty features bone and ligament sparing as unicompartmental knee arthroplasty and is presumably better in the recovery of muscle strength and function compared to total knee arthroplasty (TKA) though not previously reported in the literature. The aim of the study was to compare isokinetic knee muscle strength and physical performance in patients who underwent either bicompartmental knee arthroplasty or TKA.", "Each of 24 patients (31 knees) was prospectively examined preoperatively, at 6 and 12 months after each surgery. Isokinetic knee extensor and flexor strength as well as position sense were measured using the Biodex system. Timed up and go test, stair climbing test, and the 6-min walk test were used to assess physical performance. The results of each group were also compared with those from the corresponding healthy control, respectively.", "Demography showed significant difference in the mean age between bicompartment (54.8 \u00b1 5.6 years) and TKA groups (65.7 \u00b1 6.7 years). Comparing between the two groups, knee extensor and flexor torque, hamstring/Quadriceps ratio, position sense, and physical performance were not significantly different preoperatively, at 6 and 12 months after surgery. In intra-group analysis, muscle strength and position sense at each time point were not different in both groups. In physical performance, both groups resulted in improvement in the 6-min walk test, and only TKA group showed enhancement in stair climbing test." ], "LABELS": [ "PURPOSE", "METHODS", "RESULTS" ], "MESHES": [ "Arthroplasty, Replacement, Knee", "Female", "Humans", "Knee Joint", "Male", "Middle Aged", "Muscle Strength", "Muscle, Skeletal", "Osteoarthritis, Knee" ], "YEAR": "2013", "reasoning_required_pred": "no", "reasoning_free_pred": "no", "final_decision": "no", "LONG_ANSWER": "Although theoretically plausible, bicompartmental knee arthroplasty was not superior in knee muscle strength and physical performance at 1 year compared with total knee arthroplasty." }, "25675614": { "QUESTION": "Can gingival crevicular blood be relied upon for assessment of blood glucose level?", "CONTEXTS": [ "Diabetes mellitus (DM) is undiagnosed in approximately half of the patients actually suffering from the disease. In addition, the prevalence of DM is more than twice as high as in patients with periodontitis when compared to periodontally healthy subjects. Thus, a high number of patients with periodontitis may have undiagnosed DM. The purpose of the present study was to evaluate whether blood oozing from a gingival crevice during routine periodontal examination can be used for determining glucose levels.", "Observational cross-sectional studies were carried out in 75 patients (43 males and 32 females) with chronic periodontitis who were divided into two groups: Group I and Group II, respectively. Blood oozing from the gingival crevices of anterior teeth following periodontal probing was collected with the stick of glucose self-monitoring device, and the blood glucose levels were measured. At the same time, finger-prick blood was taken for glucometric analysis and subsequent readings were recorded.", "The patient's blood glucose values ranged from 74 to 256 mg/dl. The comparison between gingival crevicular blood and finger-prick blood showed a very strong correlation, with a t value of 3.97 (at P value = 0.001)." ], "LABELS": [ "UNLABELLED", "MATERIALS AND METHODS", "RESULTS" ], "MESHES": [ "Blood Glucose", "Blood Glucose Self-Monitoring", "Capillaries", "Chronic Periodontitis", "Cross-Sectional Studies", "Diabetes Mellitus", "Feasibility Studies", "Female", "Fingers", "Gingival Crevicular Fluid", "Gingival Hemorrhage", "Humans", "Male", "Reproducibility of Results" ], "YEAR": "2014", "reasoning_required_pred": "yes", "reasoning_free_pred": "yes", "final_decision": "yes", "LONG_ANSWER": "The data from this study has shown that GCB collected during diagnostic periodontal examination can be an excellent source of blood for glucometric analysis." }, "26864326": { "QUESTION": "Predicting admission at triage: are nurses better than a simple objective score?", "CONTEXTS": [ "In this single-centre prospective study, triage nurses estimated the probability of admission using a 100\u2005mm visual analogue scale (VAS), and GAPS was generated automatically from triage data. We compared calibration using rank sum tests, discrimination using area under receiver operating characteristic curves (AUC) and accuracy with McNemar's test.", "Of 1829 attendances, 745 (40.7%) were admitted, not significantly different from GAPS' prediction of 750 (41.0%, p=0.678). In contrast, the nurses' mean VAS predicted 865 admissions (47.3%), overestimating by 6.6% (p<0.0001). GAPS discriminated between admission and discharge as well as nurses, its AUC 0.876 compared with 0.875 for VAS (p=0.93). As a binary predictor, its accuracy was 80.6%, again comparable with VAS (79.0%), p=0.18. In the minority of attendances, when nurses felt at least 95% certain of the outcome, VAS' accuracy was excellent, at 92.4%. However, in the remaining majority, GAPS significantly outperformed VAS on calibration (+1.2% vs +9.2%, p<0.0001), discrimination (AUC 0.810 vs 0.759, p=0.001) and accuracy (75.1% vs 68.9%, p=0.0009). When we used GAPS, but 'over-ruled' it when clinical certainty was \u226595%, this significantly outperformed either method, with AUC 0.891 (0.877-0.907) and accuracy 82.5% (80.7%-84.2%)." ], "LABELS": [ "METHODS", "RESULTS" ], "MESHES": [ "Adult", "Female", "Humans", "Male", "Nursing Assessment", "Patient Admission", "Predictive Value of Tests", "Probability", "Prospective Studies", "Severity of Illness Index", "Triage" ], "YEAR": "2017", "reasoning_required_pred": "yes", "reasoning_free_pred": "maybe", "final_decision": "yes", "LONG_ANSWER": "GAPS, a simple clinical score, is a better predictor of admission than triage nurses, unless the nurse is sure about the outcome, in which case their clinical judgement should be respected." }, "26784147": { "QUESTION": "Target Serum Urate: Do Gout Patients Know Their Goal?", "CONTEXTS": [ "To examine gout patients' knowledge of their condition, including the central role of achieving and maintaining the serum urate (SU) goal with the use of urate-lowering therapy (ULT).", "This study of 612 gout patients was conducted at a Veterans Affairs medical center. Gout patients were included based on administrative diagnostic codes and receipt of at least 1 allopurinol prescription over a 1-year period. Questionnaires were mailed to patients and linked to medical records data. The questionnaire included gout-specific knowledge questions, the Patient Activation Measure, and self-reported health outcomes. Knowledge was assessed descriptively. Multivariable logistic regression was used to determine predictors of SU goal knowledge. Associations of knowledge with health outcomes were examined in exploratory analyses.", "The questionnaire had a 62% response rate. Only 14% of patients knew their SU goal, while the majority answered correctly for the other 5 gout-specific knowledge questions. In adjusted analyses, having a rheumatologist as initial prescriber (odds ratio [OR] 3.0 [95% confidence interval (95% CI) 1.4-6.2]) and knowing all of the other 5 gout-specific knowledge questions (OR 2.1 [95% CI 1.3-3.4]) were associated with greater odds of knowing the SU goal. SU goal knowledge was associated with self-reported global health status, but not with self-reported health-related quality of life or gout-specific health status." ], "LABELS": [ "OBJECTIVE", "METHODS", "RESULTS" ], "MESHES": [ "Aged", "Cross-Sectional Studies", "Female", "Gout", "Health Knowledge, Attitudes, Practice", "Humans", "Male", "Middle Aged", "Surveys and Questionnaires", "Uric Acid" ], "YEAR": "2016", "reasoning_required_pred": "no", "reasoning_free_pred": "no", "final_decision": "no", "LONG_ANSWER": "There is a knowledge deficit regarding the SU treatment goal among gout patients receiving ULT, despite generally high levels of other gout-specific knowledge. SU goal information may be an important and underutilized concept among providers treating gout patients." }, "20850631": { "QUESTION": "Diagnosis and follow-up in constipated children: should we use ultrasound?", "CONTEXTS": [ "We investigated the efficacy of ultrasound in determining megarectum and fecal load and the response to treatment in constipation and tried to specify objective criteria in this study.", "A total of 66 cases were queried and divided into 2 groups as constipated (n = 35; mean age, 6.8 \u00b1 2.9 years) and control (n = 31; mean age, 8.4 \u00b1 3.8 years) according to Rome III criteria. After the clinical evaluation, pelvic ultrasonography (US) was performed by 2 separate radiologists. The bladder capacity and the transverse rectal diameter were measured with a full bladder. Then the rectal diameter and rectal anterior wall thickness were measured, and the presence of fecal load in the rectum and sigmoid colon was recorded with an empty bladder. The examination and ultrasound were repeated after treatment for a month in these patients.", "Comparison of the US measurements of the 2 radiologists performing the US tests did not show any interobserver difference (r = 0.981; P<.001). We therefore believe our results are objective and reproducible. We found a positive correlation between the rectal diameters and the age, height, weight, and bladder capacity. The posturination mean rectal diameter was thicker in the constipated group (3.02 \u00b1 1.04 cm) than in the control group (1.98 \u00b1 0.64 cm) (P<.001). The cutoff point of rectal diameter for a diagnosis of constipation was determined as 2.44 cm (71% sensitive; 76% specific; area under curve, 0.825; P<.001). The rectal anterior wall thickness and fecal load were higher in the constipated patients (P<.001). There was a significant decrease in the constipation score and fecal load after treatment for a month (P<.001), but the rectal diameter had not reached normal limits yet despite the decrease (2.71 \u00b1 0.77 cm) (P>.05)." ], "LABELS": [ "PURPOSE", "METHODS", "RESULTS" ], "MESHES": [ "Child", "Child, Preschool", "Constipation", "Feces", "Female", "Follow-Up Studies", "Humans", "Male", "Prospective Studies", "Rectum", "Ultrasonography" ], "YEAR": "2010", "reasoning_required_pred": "yes", "reasoning_free_pred": "yes", "final_decision": "yes", "LONG_ANSWER": "The use of US helps in making a correct diagnosis and in the follow-up with objective criteria and also convinces the patient and the family that the treatment needs to be continued." }, "22849512": { "QUESTION": "Are neighbourhood social capital and availability of sports facilities related to sports participation among Dutch adolescents?", "CONTEXTS": [ "The aim of this study is to explore whether availability of sports facilities, parks, and neighbourhood social capital (NSC) and their interaction are associated with leisure time sports participation among Dutch adolescents.", "Cross-sectional analyses were conducted on complete data from the last wave of the YouRAction evaluation trial. Adolescents (n\u2009=\u2009852) completed a questionnaire asking for sports participation, perceived NSC and demographics. Ecometric methods were used to aggregate perceived NSC to zip code level. Availability of sports facilities and parks was assessed by means of geographic information systems within the zip-code area and within a 1600 meter buffer. Multilevel logistic regression analyses, with neighborhood and individual as levels, were conducted to examine associations between physical and social environmental factors and leisure time sports participation. Simple slopes analysis was conducted to decompose interaction effects.", "NSC was significantly associated with sports participation (OR: 3.51 (95%CI: 1.18;10.41)) after adjustment for potential confounders. Availability of sports facilities and availability of parks were not associated with sports participation. A significant interaction between NSC and density of parks within the neighbourhood area (OR: 1.22 (90%CI: 1.01;1.34)) was found. Decomposition of the interaction term showed that adolescents were most likely to engage in leisure time sports when both availability of parks and NSC were highest." ], "LABELS": [ "BACKGROUND", "METHODS", "RESULTS" ], "MESHES": [ "Adolescent", "Cross-Sectional Studies", "Female", "Fitness Centers", "Humans", "Leisure Activities", "Male", "Netherlands", "Odds Ratio", "Public Facilities", "Social Environment", "Sports" ], "YEAR": "2012", "reasoning_required_pred": "maybe", "reasoning_free_pred": "maybe", "final_decision": "maybe", "LONG_ANSWER": "The results of this study indicate that leisure time sports participation is associated with levels of NSC, but not with availability of parks or sports facilities. In addition, NSC and availability of parks in the zip code area interacted in such a way that leisure time sports participation is most likely among adolescents living in zip code areas with higher levels of NSC, and higher availability of parks. Hence, availability of parks appears only to be important for leisure time sports participation when NSC is high." }, "27642458": { "QUESTION": "Did the call for boycott by the Catholic bishops affect the polio vaccination coverage in Kenya in 2015?", "CONTEXTS": [ "Polio eradication is now feasible after removal of Nigeria from the list of endemic countries and global reduction of cases of wild polio virus in 2015 by more than 80%. However, all countries must remain focused to achieve eradication. In August 2015, the Catholic bishops in Kenya called for boycott of a polio vaccination campaign citing safety concerns with the polio vaccine. We conducted a survey to establish if the coverage was affected by the boycott.", "A cross sectional survey was conducted in all the 32 counties that participated in the campaign. A total of 90,157 children and 37,732 parents/guardians were sampled to determine the vaccination coverage and reasons for missed vaccination.", "The national vaccination coverage was 93% compared to 94% in the November 2014 campaign. The proportion of parents/guardians that belonged to Catholic Church was 31% compared to 7% of the children who were missed. Reasons for missed vaccination included house not being visited (44%), children not being at home at time of visit (38%), refusal by parents (12%), children being as leep (1%), and various other reasons (5%). Compared to the November 2014 campaign, the proportion of children who were not vaccinated due to parent's refusal significantly increased from 6% to 12% in August 2015." ], "LABELS": [ "INTRODUCTION", "METHODS", "RESULTS" ], "MESHES": [ "Catholicism", "Child", "Cross-Sectional Studies", "Humans", "Immunization Programs", "Kenya", "Poliomyelitis", "Poliovirus Vaccines", "Religion and Medicine", "Treatment Refusal" ], "YEAR": "2016", "reasoning_required_pred": "yes", "reasoning_free_pred": "no", "final_decision": "no", "LONG_ANSWER": "The call for boycott did not affect the campaign significantly. However, if the call for boycott is repeated in future it could have some significant negative implication to polio eradication. It is therefore important to ensure that any vaccine safety issues are addressed accordingly." }, "17403428": { "QUESTION": "Recurrent spontaneous abortion and skewed X-inactivation: is there an association?", "CONTEXTS": [ "The purpose of this study was to determine whether there is an association between skewed X-inactivation and recurrent spontaneous abortion in a large, well-defined sample of women with recurrent loss.", "X-chromosome inactivation patterns were compared in 5 groups of women. Group 1 (recurrent spontaneous abortion) consisted of 357 women with 2 or more spontaneous losses. In group 2 (infertility), there were 349 subjects from infertility practices recruited at the time of a positive serum beta-human chorionic gonadotropin. Group 3 (spontaneous abortion) women (n = 81) were recruited at the time of an ultrasound diagnosis of an embryonic demise or an anembryonic gestation. Groups 4 (primiparous) and 5 (multiparous) were healthy pregnant subjects previously enrolled in another study to determine the incidence and cause of pregnancy complications, such as preeclampsia and intrauterine growth restriction. The Primiparous group included 114 women in their first pregnancy, whereas the Multiparous group consisted of 79 women with 2 or more pregnancies but without pregnancy loss.", "The rate of extreme skewing (90% or greater) in the recurrent spontaneous abortion population was 8.6%, and not statistically different from any of the other groups, except the Primiparous group (1.0%, P<.01). The incidence of X-inactivation skewing of 90% or greater was no different whether there had been at least 1 live birth (9.9%), or no previous live births and at least 3 losses (5.6%, P>.05). When age and skewing of 90% or greater are compared, subjects with extreme skewing have a mean age of 2 years older than those without extreme skewing (P<.05)." ], "LABELS": [ "OBJECTIVE", "STUDY DESIGN", "RESULTS" ], "MESHES": [ "Abortion, Habitual", "Abortion, Spontaneous", "Adult", "Aneuploidy", "Case-Control Studies", "Cohort Studies", "Female", "Follow-Up Studies", "Genetic Predisposition to Disease", "Humans", "Incidence", "Parity", "Pregnancy", "Pregnancy Outcome", "Probability", "Prospective Studies", "Reference Values", "Risk Assessment", "Statistics, Nonparametric", "X Chromosome Inactivation" ], "YEAR": "2007", "reasoning_required_pred": "no", "reasoning_free_pred": "no", "final_decision": "no", "LONG_ANSWER": "Skewed X-inactivation is not associated with recurrent spontaneous abortion but is associated with increasing maternal age." }, "21252642": { "QUESTION": "Does increased patient awareness improve accrual into cancer-related clinical trials?", "CONTEXTS": [ "Oncology literature cites that only 2% to 4% of patients participate in research. Up to 85% of patients are unaware that clinical trials research is being conducted at their treatment facility or that they might be eligible to participate.", "It was hypothesized that patients' satisfaction with information regarding clinical trials would improve after targeted educational interventions, and accruals to clinical trials would increase in the year following those interventions.", "All new patients referred to the cancer center over a 4-month period were mailed a baseline survey to assess their knowledge of clinical research. Subsequently, educational interventions were provided, including an orientation session highlighting clinical trials, a pamphlet, and a reference to a clinical trials Web site. A postintervention survey was sent to the responders of the initial survey 3 months after the initial mailing.", "Patient satisfaction with information significantly increased after the interventions. There was no increase in subsequent enrollment in clinical trials. Patients who indicated an inclination to participate in clinical trials tended to have greater satisfaction with the information they received." ], "LABELS": [ "BACKGROUND", "OBJECTIVES", "METHODS", "RESULTS" ], "MESHES": [ "Clinical Trials as Topic", "Cross-Sectional Studies", "Female", "Humans", "Male", "Middle Aged", "Neoplasms", "Patient Education as Topic", "Patient Participation", "Patient Satisfaction", "Physician-Patient Relations", "Prospective Studies" ], "YEAR": null, "reasoning_required_pred": "no", "reasoning_free_pred": "yes", "final_decision": "no", "LONG_ANSWER": "A set of educational interventions designed for cancer patients significantly improved their satisfaction with information on clinical research, but did not improve clinical trial enrollment of these participants as of 1 year after the study." }, "25503376": { "QUESTION": "Does airway surgery lower serum lipid levels in obstructive sleep apnea patients?", "CONTEXTS": [ "Obstructive sleep apnea (OSA) is tightly linked to increased cardiovascular disease. Surgery is an important method to treat OSA, but its effect on serum lipid levels in OSA patients is unknown. We aimed to evaluate the effect of upper airway surgery on lipid profiles.", "We performed a retrospective review of 113 adult patients with OSA who underwent surgery (nasal or uvulopalatopharyngoplasty [UPPP]) at a major, urban, academic hospital in Beijing from 2012 to 2013 who had preoperative and postoperative serum lipid profiles.", "Serum TC (4.86\u00b10.74 to 4.69\u00b10.71) and LP(a) (median 18.50 to 10.90) all decreased significantly post-operatively (P<0.01, 0.01, respectively), with no changes in serum HDL, LDL, or TG (P>0.05, all). For UPPP patients (n=51), serum TC, HDL and LP(a) improved (P=0.01, 0.01,<0.01, respectively). For nasal patients (n=62), only the serum LP(a) decreased (P<0.01). In patients with normal serum lipids at baseline, only serum LP(a) decreased (P<0.01). In contrast, in patients with isolated hypertriglyceridemia, the serum HDL, TG and LP(a) showed significant improvements (P=0.02, 0.03,<0.01, respectively). In patients with isolated hypercholesterolemia, the serum LP(a) decreased significantly (P=0.01), with a similar trend for serum TC (P=0.06). In patients with mixed hyperlipidemia, the serum TC and LDL also decreased (P=0.02, 0.03, respectively)." ], "LABELS": [ "BACKGROUND", "MATERIAL AND METHODS", "RESULTS" ], "MESHES": [ "Adult", "Demography", "Female", "Humans", "Lipids", "Male", "Polysomnography", "Respiratory System", "Retrospective Studies", "Sleep Apnea, Obstructive" ], "YEAR": "2014", "reasoning_required_pred": "yes", "reasoning_free_pred": "no", "final_decision": "yes", "LONG_ANSWER": "Surgery may improve blood lipid levels in patients with OSA, especially in patients with preoperative dyslipidemia, potentially yielding a major benefit in metabolism and cardiovascular sequelae. Prospective studies should examine this potential metabolic effect of airway surgery for OSA." }, "12172698": { "QUESTION": "Is withdrawal-induced anxiety in alcoholism based on beta-endorphin deficiency?", "CONTEXTS": [ "Associations between several psychopathological alterations and lowered beta-endorphin(beta E) plasma levels have already been stated in former studies. However, whereas single measures during static conditions generally failed in linking beta E levels with psychopathology, dynamic changes of beta E in particular have been shown to be associated with spells of anxiety and depression. During alcohol withdrawal, a decreased secretion of beta E with a delayed normalization has been reported, but up to now only few data became available regarding the interaction of plasma beta E and psychopathological parameters.", "The aim of our study was to test the hypothesis whether beta E during acute alcohol withdrawal is associated with anxiety, depression, and craving.", "We observed self-rated anxiety, depression, and craving during alcohol withdrawal and assessed beta E levels (RIA) in a consecutive sample of 60 alcoholics on day 1 and day 14 after onset of withdrawal, and in 30 healthy volunteers. To control for mutual interactions of beta E and the pituitary-adrenocortical hormone secretion, plasma corticotropin (ACTH) and cortisol were also determined.", "In accordance with prior studies, beta E was significantly lowered on day 1 and day 14 of alcohol withdrawal relative to controls. Plasma levels of ACTH correlated significantly with beta E in alcoholics at both time points and in controls, without differing significantly between the groups. Self-rated anxiety, depression, and alcohol craving decreased significantly between day 1 and day 14. Levels of beta E were inversely correlated with anxiety day 1 (r=-0.58) and day 14 (r=-0.71). Partial correlation coefficients controlling for ACTH plasma levels revealed that this correlation was largely independent from ACTH. In addition, a significant inverse relationship was found between beta E and craving on day 14 (r=-0.28). No association appeared between beta E and depression." ], "LABELS": [ "RATIONALE", "OBJECTIVES", "METHODS", "RESULTS" ], "MESHES": [ "Adrenocorticotropic Hormone", "Adult", "Alcohol Drinking", "Alcoholism", "Anxiety", "Depression", "Disruptive, Impulse Control, and Conduct Disorders", "Female", "Humans", "Hydrocortisone", "Male", "Middle Aged", "Substance Withdrawal Syndrome", "Time Factors", "beta-Endorphin" ], "YEAR": "2002", "reasoning_required_pred": "yes", "reasoning_free_pred": "yes", "final_decision": "yes", "LONG_ANSWER": "Our results give first evidence that lowered beta E during alcohol withdrawal may contribute to anxiety as a common disturbance during this state." }, "19351635": { "QUESTION": "Do older patients receive adequate stroke care?", "CONTEXTS": [ "National guidelines and government directives have adopted policies for urgent assessment of patients with a transient ischaemic attack or minor stroke not admitted to hospital. The risk of recurrent stroke increases substantially with age, as does the potential benefit of secondary prevention. In order to develop effective strategies for older patients, it is important to identify how stroke care is currently provided for this patient group.", "Between 2004 and 2006, older patients (>75 years) referred to a neurovascular clinic were compared with younger patients (or =12 cm H2O). The predictor variables included apnea-hypopnea index (AHI), age, sex, body mass index (BMI), Epworth Sleepiness Scale (ESS), and the Multiple Sleep Latency Test (MSLT). High CPAP was required in 2 (6.9%), 6 (5.8%), and 63 (28.6%) patients with mild, moderate, and severe OSA, respectively. On univariate analysis, AHI, BMI, ESS score, and the proportion of males were significantly higher in those needing high CPAP. They also have a lower MSLT mean. On logistic regression, the use of high CPAP was 5.90 times more frequent (95% confidence interval 2.67-13.1) in severe OSA patients after adjustment for the other variables. The area under the receiver operator curve was 72.4%, showing that the model was adequate." ], "LABELS": [ "STUDY OBJECTIVES", "DESIGN", "PATIENTS", "MEASUREMENTS AND RESULTS" ], "MESHES": [ "Body Mass Index", "Continuous Positive Airway Pressure", "Female", "Humans", "Logistic Models", "Male", "Middle Aged", "Models, Theoretical", "Polysomnography", "Predictive Value of Tests", "Retrospective Studies", "Severity of Illness Index", "Sex Factors", "Sleep Apnea, Obstructive" ], "YEAR": "2006", "reasoning_required_pred": "yes", "reasoning_free_pred": "maybe", "final_decision": "maybe", "LONG_ANSWER": "Severe OSA patients are much more likely to need high CPAP levels. However, because of the low positive predictive value (only 28.6%), the clinical value of such information is limited. ESS and MSLT did not increase the predictive value for the need for high CPAP." }, "18670651": { "QUESTION": "Do the changes in the serum levels of IL-2, IL-4, TNFalpha, and IL-6 reflect the inflammatory activity in the patients with post-ERCP pancreatitis?", "CONTEXTS": [ "Acute pancreatitis is the major complication of endoscopic retrograde cholangiopancreatography (ERCP) procedure and there are some reports showing cytokine changes in ERCP-induced pancreatits.GOALS: To investigate the association between early changes (within 24 hours) in the serum interleukin (IL)-2, IL-4, tumor necrosis factor (TNF)alpha, and IL-6 levels and the development of post-ERCP pancreatitis.STUDY: Forty five consecutive patients who underwent therapeutic ERCP and 10 patients with acute pancreatitis without ERCP were enrolled to the study. Serum concentrations of IL-2, IL-4, TNFalpha, and IL-6 were determined immediately before, 12 hours and 24 hours after ERCP.", "Seven of the 45 patients (15.5%) developed post-ERCP pancreatitis. The levels of IL-4 at 24 hours after ERCP were significantly lower in the patients with post-ERCP pancreatitis than in those without pancreatitis, while TNFalpha levels at 12 hours after ERCP were higher in the complicated group than those of the uncomplicated group. The ratios of TNFalpha/IL-4 at 12 and 24 hours after ERCP were found significantly higher in the patients with post-ERCP pancreatitis than in those without pancreatitis. IL-6 in the complicated patients was found significantly increased at 24 hours after ERCP." ], "LABELS": [ "BACKGROUND", "RESULTS" ], "MESHES": [ "Cholangiopancreatography, Endoscopic Retrograde", "Cytokines", "Female", "Humans", "Inflammation", "Interleukin-2", "Interleukin-4", "Interleukin-6", "Male", "Middle Aged", "Pancreatitis", "Tumor Necrosis Factor-alpha" ], "YEAR": "2008", "reasoning_required_pred": "yes", "reasoning_free_pred": "maybe", "final_decision": "yes", "LONG_ANSWER": "The enhancement of serum TNFalpha and IL-6 levels in the patients with ERCP-induced pancreatitis reflects the inflammatory activity. Additionally, these cytokines together with IL-4 can be used in clinical laboratory monitoring of ERCP." }, "11034241": { "QUESTION": "Is routine intraoperative frozen-section examination of sentinel lymph nodes in breast cancer worthwhile?", "CONTEXTS": [ "Routine intraoperative frozen section (FS) of sentinel lymph nodes (SLN) can detect metastatic disease, allowing immediate axillary dissection and avoiding the need for reoperation. Routine FS is also costly, increases operative time, and is subject to false-negative results. We examined the benefit of routine intraoperative FS among the first 1000 patients at Memorial Sloan Kettering Cancer Center who had SLN biopsy for breast cancer.", "We performed SLN biopsy with intraoperative FS in 890 consecutive breast cancer patients, none of whom had a back-up axillary dissection planned in advance. Serial sections and immunohistochemical staining for cytokeratins were performed on all SLN that proved negative on FS. The sensitivity of FS was determined as a function of (1) tumor size and (2) volume of metastatic disease in the SLN, and the benefit of FS was defined as the avoidance of a reoperative axillary dissection.", "The sensitivity of FS ranged from 40% for patients with Tla to 76% for patients with T2 cancers. The volume of SLN metastasis was highly correlated with tumor size, and FS was far more effective in detecting macrometastatic disease (sensitivity 92%) than micrometastases (sensitivity 17%). The benefit of FS in avoiding reoperative axillary dissection ranged from 4% for Tla (6 of 143) to 38% for T2 (45 of 119) cancers." ], "LABELS": [ "BACKGROUND", "METHODS", "RESULTS" ], "MESHES": [ "Adult", "Aged", "Aged, 80 and over", "Breast Neoplasms", "Female", "Frozen Sections", "Humans", "Immunohistochemistry", "Intraoperative Period", "Lymphatic Metastasis", "Middle Aged", "Predictive Value of Tests", "Sensitivity and Specificity", "Sentinel Lymph Node Biopsy" ], "YEAR": "2000", "reasoning_required_pred": "yes", "reasoning_free_pred": "maybe", "final_decision": "maybe", "LONG_ANSWER": "In breast cancer patients having SLN biopsy, the failure of routine intraoperative FS is largely the failure to detect micrometastatic disease. The benefit of routine intraoperative FS increases with tumor size. Routine FS may not be indicated in patients with the smallest invasive cancers." }, "24013712": { "QUESTION": "Preoperative platelet count in esophageal squamous cell carcinoma: is it a prognostic factor?", "CONTEXTS": [ "Platelet count is inversely related to prognosis in many cancers; however, its role in esophageal cancer is still controversial. The purpose of this study was to determine the prognostic value of preoperative platelet count in esophageal squamous cell carcinoma (ESCC).", "From January 2006 to December 2008, a retrospective analysis of 425 consecutive patients with ESCC was conducted. A receiver operating characteristic (ROC) curve for survival prediction was plotted to verify the optimum cutoff point for preoperative platelet count. Univariate and multivariate analyses were performed to evaluate the prognostic parameters.", "A ROC curve for survival prediction was plotted to verify the optimum cutoff point for platelet count, which was 205 (\u00d7 10(9)/L). Patients with platelet count \u2264 205 had a significantly better 5-year survival than patients with a platelet count>205 (60.7 vs. 31.6 %, P<0.001). The 5-year survival of patients either with platelet count \u2264 205 or>205 were similar (68.6 vs. 58.8 %, P = 0.085) when the nodes were negative. However, the 5-year survival of patients with platelet count \u2264 205 was better than that of patients with a platelet count>205 when the nodes were involved (32.0 vs. 12.7 %, P = 0.004). Multivariate analysis showed that platelet count (P = 0.013), T grade (P = 0.017), and N staging (P<0.001) were independent prognostic factors." ], "LABELS": [ "PURPOSE", "METHODS", "RESULTS" ], "MESHES": [ "Carcinoma, Squamous Cell", "Esophageal Neoplasms", "Female", "Humans", "Male", "Middle Aged", "Neoplasm Staging", "Platelet Count", "Prognosis", "Retrospective Studies", "Sensitivity and Specificity", "Survival Rate" ], "YEAR": "2013", "reasoning_required_pred": "yes", "reasoning_free_pred": "yes", "final_decision": "yes", "LONG_ANSWER": "Preoperative platelet count is a predictive factor for long-term survival in ESCC, especially in nodal-positive patients. We conclude that 205 (\u00d710(9)/L) may be the optimum cutoff point for platelet count in predicting survival in ESCC patients." }, "23240452": { "QUESTION": "Can we predict urinary stone composition based on an analysis of microelement concentration in the hair and urine?", "CONTEXTS": [ "In recent years the role of trace elements in lithogenesis has received steadily increasing attention.", "This study was aimed to attempt to find the correlations between the chemical content of the stones and the concentration of chosen elements in the urine and hair of stone formers.", "The proposal for the study was approved by the local ethics committee. Specimens were taken from 219 consecutive stone-formers. The content of the stone was evaluated using atomic absorption spectrometry, spectrophotometry, and colorimetric methods. An analysis of 29 elements in hair and 21 elements in urine was performed using inductively coupled plasma-atomic emission spectrometry.", "Only a few correlations between the composition of stones and the distribution of elements in urine and in hair were found. All were considered incidental." ], "LABELS": [ "BACKGROUND", "OBJECTIVES", "MATERIAL AND METHODS", "RESULTS" ], "MESHES": [ "Adult", "Hair", "Humans", "Middle Aged", "Spectrophotometry", "Trace Elements", "Urinary Calculi", "Urine" ], "YEAR": null, "reasoning_required_pred": "no", "reasoning_free_pred": "no", "final_decision": "no", "LONG_ANSWER": "The data obtained did not allow for the creation of a proper and practical algorithm to predict stone chemical composition based on hair and urine analysis." }, "21420186": { "QUESTION": "Could ADMA levels in young adults born preterm predict an early endothelial dysfunction?", "CONTEXTS": [ "Sporadic data present in literature report how preterm birth and low birth weight are risk factors for the development of cardiovascular diseases in later life. High levels of asymmetric dimethylarginine (ADMA), a strong inhibitor of nitric oxide synthesis, are associated with the future development of adverse cardiovascular events and cardiac death.", "1) to verify the presence of a statistically significant difference between ADMA levels in young adults born preterm at extremely low birth weight (<1000 g; ex-ELBW) and those of a control group of healthy adults born at term (C) and 2) to seek correlations between ADMA levels in ex-ELBW and anthropometric and clinical parameters (gender, chronological age, gestational age, birth weight, and duration of stay in Neonatal Intensive Care Unit).", "Thirty-two ex-ELBW subjects (11 males [M] and 21 females [F], aged 17-29years, mean age 22.2 \u00b1 2.3 years) were compared with 25 C (7 M and 18F). ADMA levels were assessed by high-performance liquid chromatography with highly sensitive laser fluorescent detection.", "ADMA levels were reduced in ex-ELBW subjects compared to C (0.606+0.095 vs 0.562+0.101 \u03bcmol/L, p<0.05), and significantly correlated inversely with gestational age (r=-0.61, p<0.00001) and birth weight (r=-0.57, p<0.0002)." ], "LABELS": [ "BACKGROUND", "AIMS", "METHODS", "RESULTS" ], "MESHES": [ "Adolescent", "Adult", "Arginine", "Early Diagnosis", "Endothelium, Vascular", "Female", "Gestational Age", "Humans", "Infant, Extremely Low Birth Weight", "Infant, Low Birth Weight", "Infant, Newborn", "Male", "Predictive Value of Tests", "Premature Birth", "Vascular Diseases", "Young Adult" ], "YEAR": "2012", "reasoning_required_pred": "yes", "reasoning_free_pred": "yes", "final_decision": "yes", "LONG_ANSWER": "Our findings reveal a significant decrease in ADMA levels of ex-ELBW subjects compared to C, underlining a probable correlation with preterm birth and low birth weight. Taken together, these results may underlie the onset of early circulatory dysfunction predictive of increased cardiovascular risk." }, "22302761": { "QUESTION": "Can fractional lasers enhance transdermal absorption of topical lidocaine in an in vivo animal model?", "CONTEXTS": [ "It has been shown in vitro that pretreatment of skin with fractional lasers enhances transdermal delivery of drugs. The aim of this study is to demonstrate in vivo firstly that laser enhances transdermal drug absorption and secondly that this can be manipulated by altering laser settings.STUDY DESIGN/", "Four pigs were used in the IACUC approved animal study. On day 0, 5 g of 4% topical lidocaine was applied under occlusion for 60 minutes to a 400 cm(2) area on the abdomen. Blood was drawn at 0, 60, 90, 120, 180, and 240 minutes. On day 7, the Er:YAG laser was used at 500, 250, 50, and 25 \u00b5m ablative depth, respectively, over a 400 cm(2) area on the abdomen. Five grams of 4% topical lidocaine was applied immediately with occlusion for 60 minutes, and then removed. Blood was drawn at 0, 60, 90, 120, 180, and 240 minutes. The serum was extracted and analyzed for lidocaine and its metabolite monoethylglycinexylidide (MEGX).", "Serum levels of lidocaine and MEGX were undetectable in untreated skin. Following laser treatment both lidocaine and MEGX were detectable. Peak levels of lidocaine were significantly higher (P = 0.0002) at 250 \u00b5m (0.62 mg/L), compared to 500 \u00b5m (0.45 mg/L), 50 \u00b5m (0.48 mg/L), and 25 \u00b5m (0.3 mg/L). Peak levels of MEGX were significantly higher (P \u2264 0.0001) at 250 \u00b5m (0.048 mg/L), compared to 500 \u00b5m (0.018 mg/L), 50 \u00b5m (0.036 mg/L), and 25 \u00b5m (0.0144 mg/L)." ], "LABELS": [ "BACKGROUND AND OBJECTIVE", "MATERIALS AND METHODS", "RESULTS" ], "MESHES": [ "Administration, Cutaneous", "Anesthetics, Local", "Animals", "Drug Delivery Systems", "Lasers, Solid-State", "Lidocaine", "Skin Absorption", "Swine" ], "YEAR": "2012", "reasoning_required_pred": "yes", "reasoning_free_pred": "yes", "final_decision": "yes", "LONG_ANSWER": "This study demonstrates that laser pretreatment significantly increases absorption of topical lidocaine so that it is detectable in the blood and that manipulating laser settings can affect drug absorption. Future work will look at translating this effect into clinical benefit." }, "21979183": { "QUESTION": "Does intra-abdominal desmoid disease affect patients with an ileal pouch differently than those with an ileorectal anastomosis?", "CONTEXTS": [ "Nobody has analyzed the sequelae of desmoids according to the type of surgery that precipitated them.", "This study aims to determine whether the clinical effects of abdominal desmoids would be worse in patients with restorative proctocolectomy than in patients with ileorectal anastomosis.", "This is a retrospective, database study.", "Included were patients with familial adenomatous polyposis who had undergone proctocolectomy with IPAA or colectomy and ileorectal anastomosis, and subsequently developed an intra-abdominal desmoid tumor.", "The primary outcome measures were the clinical course of the desmoids; morbidity, and the requirement for stoma.", "There were 86 patients: 49 had restorative proctocolectomy and 37 had ileorectal anastomosis. Patient demographics were similar. Average follow-up was 9.8 years (range, 2.7-23.8) and 16.3 years (range, 2.3 - 42.9). Treatment of the desmoids included surgery (64.4% vs 65.6%), medical therapy (69.4% vs 59.5%), chemotherapy (36.2% vs 30.0%), and radiotherapy (4.5% vs 10.0%), and was the same for each group. The overall complication rate of desmoids was similar, approaching 70%. The risk of individual complications was also similar (bleeding (2.0% vs 0.0%), fistula (10.2% vs 13.5%), bowel obstruction (32.7% vs 48.6%), pain (34.7% vs 21.6%), and death related to desmoid tumors (2.0% vs 10.8%)); 38.8% of the restorative proctocolectomy group and 51.4% the ileorectal group had surgery for desmoid tumor complications (P = .21), and 22.4% and 22.2% of patients ultimately had permanent stomas.", "This study was limited by the relatively small numbers of patients." ], "LABELS": [ "BACKGROUND", "OBJECTIVE", "DESIGN", "PATIENTS", "MAIN OUTCOME MEASURES", "RESULTS", "LIMITATIONS" ], "MESHES": [ "Adenomatous Polyposis Coli", "Adolescent", "Adult", "Anastomosis, Surgical", "Child", "Child, Preschool", "Colonic Pouches", "Female", "Fibromatosis, Abdominal", "Fibromatosis, Aggressive", "Humans", "Ileum", "Laparoscopy", "Male", "Middle Aged", "Proctocolectomy, Restorative", "Rectum", "Retrospective Studies", "Young Adult" ], "YEAR": "2011", "reasoning_required_pred": "no", "reasoning_free_pred": "no", "final_decision": "no", "LONG_ANSWER": "The morbidity associated with desmoid tumors has not been shown to differ, whether they arise after restorative proctocolectomy or ileorectal anastomosis." }, "12607120": { "QUESTION": "Does anastomotic leakage affect functional outcome after rectal resection for cancer?", "CONTEXTS": [ "Anastomotic leakage is the most threatening early complication in sphincter-preserving rectal cancer surgery. While the oncological consequences have been well examined, only few data exist about the functional outcome.", "We investigated continence function in 150 patients after curative sphincter-preserving rectal cancer surgery. Functional results were compared in 22 patients with a clinically relevant anastomotic leakage, confirmed radiologically or endoscopically, and 128 patients with uneventful recovery. Evaluation of continence function was based on the Cleveland Clinic Continence Score and was examined in all patients with anastomotic leakage and in 111 patients without complications 107+/-46 weeks postoperatively. Additionally, 14 patients with anastomotic leakage and 58 patients with uneventful recovery underwent anorectal manometry 26+/-15 weeks postoperatively.", "The continence score in patients after anastomotic leakage did not differ significantly from that in patients without complications. Sphincter function was similar. Maximum tolerable volume and rectal compliance were slightly but not significantly worse after leakage." ], "LABELS": [ "BACKGROUND", "PATIENTS AND METHODS", "RESULTS" ], "MESHES": [ "Anastomosis, Surgical", "Fecal Incontinence", "Female", "Humans", "Male", "Manometry", "Middle Aged", "Postoperative Complications", "Rectal Neoplasms", "Rectum", "Statistics, Nonparametric", "Surgical Wound Dehiscence", "Surveys and Questionnaires", "Treatment Failure" ], "YEAR": "2003", "reasoning_required_pred": "no", "reasoning_free_pred": "no", "final_decision": "no", "LONG_ANSWER": "Continence function remained undisturbed after anastomotic leakage due to rectal resection" }, "21914194": { "QUESTION": "Health care for immigrants in Europe: is there still consensus among country experts about principles of good practice?", "CONTEXTS": [ "European Member States are facing a challenge to provide accessible and effective health care services for immigrants. It remains unclear how best to achieve this and what characterises good practice in increasingly multicultural societies across Europe. This study assessed the views and values of professionals working in different health care contexts and in different European countries as to what constitutes good practice in health care for immigrants.", "A total of 134 experts in 16 EU Member States participated in a three-round Delphi process. The experts represented four different fields: academia, Non-Governmental Organisations, policy-making and health care practice. For each country, the process aimed to produce a national consensus list of the most important factors characterising good practice in health care for migrants.", "The scoring procedures resulted in 10 to 16 factors being identified as the most important for each participating country. All 186 factors were aggregated into 9 themes: (1) easy and equal access to health care, (2) empowerment of migrants, (3) culturally sensitive health care services, (4) quality of care, (5) patient/health care provider communication, (6) respect towards migrants, (7) networking in and outside health services, (8) targeted outreach activities, and (9) availability of data about specificities in migrant health care and prevention. Although local political debate, level of immigration and the nature of local health care systems influenced the selection and rating of factors within each country, there was a broad European consensus on most factors. Yet, discordance remained both within countries, e.g. on the need for prioritising cultural differences, and between countries, e.g. on the need for more consistent governance of health care services for immigrants." ], "LABELS": [ "BACKGROUND", "METHODS", "RESULTS" ], "MESHES": [ "Attitude of Health Personnel", "Consensus", "Delivery of Health Care", "Delphi Technique", "Emigrants and Immigrants", "Europe", "Health Services Accessibility", "Humans", "Policy Making" ], "YEAR": "2011", "reasoning_required_pred": "yes", "reasoning_free_pred": "maybe", "final_decision": "maybe", "LONG_ANSWER": "Experts across Europe asserted the right to culturally sensitive health care for all immigrants. There is a broad consensus among experts about the major principles of good practice that need to be implemented across Europe. However, there also is some disagreement both within and between countries on specific issues that require further research and debate." }, "25725704": { "QUESTION": "Can clinical supervision sustain our workforce in the current healthcare landscape?", "CONTEXTS": [ "Clinical supervision is widely recognised as a mechanism for providing professional support, professional development and clinical governance for healthcare workers. There have been limited studies about the effectiveness of clinical supervision for allied health and minimal studies conducted within the Australian health context. The aim of the present study was to identify whether clinical supervision was perceived to be effective by allied health professionals and to identify components that contributed to effectiveness. Participants completed an anonymous online questionnaire, administered through the health service's intranet.", "A cross-sectional study was conducted with community allied health workers (n = 82) 8 months after implementation of structured clinical supervision. Demographic data (age, gender), work-related history (profession employment level, years of experience), and supervision practice (number and length of supervision sessions) were collected through an online survey. The outcome measure, clinical supervision effectiveness, was operationalised using the Manchester Clinical Supervision Scale-26 (MCSS-26). Data were analysed with Pearson correlation (r) and independent sample t-tests (t) with significance set at 0.05 (ie the probability of significant difference set at P<0.05).", "The length of the supervision sessions (r(s) \u2265 0.44), the number of sessions (r(s) \u2265 0.35) and the total period supervision had been received (r(s) \u2265 0.42) were all significantly positively correlated with the MCSS-26 domains of clinical supervision effectiveness. Three individual variables, namely 'receiving clinical supervision', 'having some choice in the allocation of clinical supervisor' and 'having a completed clinical supervision agreement', were also significantly associated with higher total MCSS-26 scores (P(s)<0.014)." ], "LABELS": [ "OBJECTIVE", "METHODS", "RESULTS" ], "MESHES": [ "Adult", "Allied Health Personnel", "Female", "Humans", "Male", "Personnel Administration, Hospital", "Queensland", "Surveys and Questionnaires" ], "YEAR": "2015", "reasoning_required_pred": "yes", "reasoning_free_pred": "maybe", "final_decision": "yes", "LONG_ANSWER": "The results of the study demonstrate that when clinical supervision uses best practice principles, it can provide professional support for allied health workers, even during times of rapid organisational change." }, "24516646": { "QUESTION": "Is the determination of specific IgE against components using ISAC 112 a reproducible technique?", "CONTEXTS": [ "The ImmunoCAP ISAC 112 is a fluoro-immunoassay that allows detection of specific IgE to 112 molecular components from 51 allergenic sources. We studied the reliability of this technique intra- and inter- assay, as well as inter-batch- and inter-laboratory-assay.", "Twenty samples were studied, nineteen sera from polysensitized allergic patients, and the technique calibrator provided by the manufacturer (CTR02). We measured the sIgE from CTR02 and three patients' sera ten times in the same and in different assays. Furthermore, all samples were tested in two laboratories and with two batches of ISAC kit. To evaluate the accuracy of ISAC 112, we contrasted the determinations of CTR02 calibrator with their expected values by T Student test. To analyse the precision, we calculated the coefficient of variation (CV) of the 15 allergens that generate the calibration curve, and to analyse the repeatability and the reproducibility, we calculated the intraclass coefficient correlation (ICC) to each allergen.", "The results obtained for CTR02 were similar to those expected in 7 of 15 allergens that generate the calibration curve, whereas in 8 allergens the results showed significant differences. The mean CV obtained in the CTR02 determinations was of 9.4%, and the variability of sera from patients was of 22.9%. The agreement in the intra- and inter-assay analysis was very good to 94 allergens and good to one. In the inter-batch analyse, we obtained a very good agreement to 82 allergens, good to 14, moderate to 5 allergens, poor to one, and bad to 1 allergen. In the inter-laboratory analyse, we obtained a very good agreement to 73 allergens, good to 22, moderate to 6 and poor to two allergens." ], "LABELS": [ "BACKGROUND", "METHODS", "RESULTS" ], "MESHES": [ "Calibration", "Fluoroimmunoassay", "Humans", "Immunoglobulin E", "Reproducibility of Results" ], "YEAR": "2014", "reasoning_required_pred": "yes", "reasoning_free_pred": "yes", "final_decision": "yes", "LONG_ANSWER": "The allergen microarray immunoassay, ISAC 112, is a repeatable and reproducible in vitro diagnostic tool for determination of sIgE beyond the own laboratory." }, "28247485": { "QUESTION": "Is the first urinary albumin/creatinine ratio (ACR) in women with suspected preeclampsia a prognostic factor for maternal and neonatal adverse outcome?", "CONTEXTS": [ "The aim of this study was to determine the prognostic value of the first urinary albumin/creatinine ratio (ACR) for adverse maternal and neonatal outcomes and how it relates to other prognostic factors.", "We performed a retrospective cohort study from December 2009 to February 2012 with analysis of demographic, clinical and biochemical data from two obstetric day assessment units in hospitals in Southeast Scotland. We included 717 pregnant women, with singleton pregnancies after 20 weeks' gestation, referred for evaluation of suspected preeclampsia and having their first ACR performed. The ability of ACR to predict future outcomes was assessed in both univariable and multivariable logistic regression models. The latter assessed its prognostic value independent of (adjusting for) existing prognostic factors. Primary outcome measures were maternal and neonatal composite adverse outcomes, and a secondary outcome was gestation at delivery.", "In all, 204 women (28.5%) experienced a composite adverse maternal outcome and 146 women (20.4%) experienced a composite adverse neonatal outcome. Multivariate analysis of log-transformed ACR demonstrated that a 1-unit increase in log ACR is associated with an increased odds of adverse maternal [odds ratio 1.60, 95% confidence interval (CI) 1.45-1.80] and adverse neonatal (odds ratio 1.15, 95% CI 1.02-1.29) composite outcomes, and with reduced gestational age at delivery (coefficient: -0.46, 95% CI -0.54 to -0.38)." ], "LABELS": [ "INTRODUCTION", "MATERIAL AND METHODS", "RESULTS" ], "MESHES": [ "Adult", "Albuminuria", "Cohort Studies", "Creatinine", "Female", "Humans", "Infant, Newborn", "Pre-Eclampsia", "Predictive Value of Tests", "Pregnancy", "Pregnancy Outcome", "Prenatal Diagnosis", "Prognosis", "Proteinuria", "Retrospective Studies", "Scotland" ], "YEAR": "2017", "reasoning_required_pred": "yes", "reasoning_free_pred": "yes", "final_decision": "yes", "LONG_ANSWER": "ACR is an independent prognostic factor for maternal and neonatal adverse outcomes in suspected preeclampsia. ACR may be useful to inform risk predictions within a prognostic model." }, "11334578": { "QUESTION": "Is there awareness of pharmaceutical expenditure in the reformed primary care system?", "CONTEXTS": [ "To evaluate the effectiveness of feeding information on pharmacy back to primary care doctors in order to create awareness (knowledge) of pharmaceutical expenditure (PE).", "Retrospective cross-sectional study, through personal interview.", "Reformed PC, Sabadell, Barcelona.", "The 80 PC doctors working with primary care teams.", "As the personal feed-back on PE, each doctor was asked for the PE generated during 1997 and the mean cost of prescriptions to active and pensioner patients. The statistical test used was the t test to compare means for paired data, with p<0.05 the required level of significance.", "Out of the total doctors interviewed (80), 71 replies were obtained for the annual PE and 76 for the mean cost of prescriptions, for both active and pensioner patients. Significant differences were found between the annual PE in reality and doctors' estimates: around twelve million pesetas. The differences between the real mean costs of prescription and the estimates were also significant." ], "LABELS": [ "OBJECTIVE", "DESIGN", "SETTING", "PARTICIPANTS", "INTERVENTIONS", "RESULTS" ], "MESHES": [ "Cross-Sectional Studies", "Drug Costs", "Health Care Reform", "Health Care Surveys", "Health Knowledge, Attitudes, Practice", "Humans", "Primary Health Care", "Retrospective Studies" ], "YEAR": "2001", "reasoning_required_pred": "no", "reasoning_free_pred": "no", "final_decision": "no", "LONG_ANSWER": "In view of the results, it is concluded that there is no awareness of PE among the doctors in the reformed PC in our area. This poses a question over the efficacy of feed-back of pharmacy information in order to create this awareness. This information needs to be more frequent and richer in content, and to be complemented by other measures." }, "21593045": { "QUESTION": "Do women with ovaries of polycystic morphology without any other features of PCOS benefit from short-term metformin co-treatment during IVF?", "CONTEXTS": [ "Women with ovaries of polycystic morphology (PCO), without any other features of polycystic ovary syndrome (PCOS), respond similarly to women with PCOS when stimulated with exogenous gonadotrophins, and both groups share various endocrinological disturbances underlying their pathology. In women with PCOS, metformin co-treatment during IVF has been shown to increase pregnancy rates and reduce the risk of ovarian hyperstimulation syndrome (OHSS). The aim of this study was to investigate whether metformin co-treatment before and during IVF can also increase the live birth rate (LBR) and lower severe OHSS rates for women with PCO, but no other manifestations of PCOS.", "This study was a double-blind, multi-centre, randomized, placebo-controlled trial. The study population included 134 women with ovulatory PCO (and no evidence of clinical or biochemical hyperandrogenism) undergoing IVF treatment at three tertiary referral IVF units. The primary outcome was LBR.", "In total, 134 women were randomized, 69 to metformin and 65 to placebo. There were no statistically significant differences between the two groups in baseline characteristics. With regard to IVF outcome, no significant improvements were found in the metformin group when compared with the placebo group. In particular, there was no difference between the groups in rates of live birth [metformin n = 27 (39.1%), placebo n = 30 (46.2), (95% confidence interval 0.38, 1.49, odds ratio = 0.75)], clinical pregnancy [metformin n = 29 (42.0%), placebo n = 33 (50.8%)]or severe OHSS [metformin n = 6 (8.7%), placebo n = 5 (7.7%)]." ], "LABELS": [ "BACKGROUND", "METHODS", "RESULTS" ], "MESHES": [ "Double-Blind Method", "Female", "Fertilization in Vitro", "Humans", "Live Birth", "Metformin", "Ovarian Hyperstimulation Syndrome", "Placebos", "Polycystic Ovary Syndrome", "Pregnancy" ], "YEAR": "2011", "reasoning_required_pred": "no", "reasoning_free_pred": "no", "final_decision": "no", "LONG_ANSWER": "There appears to be no benefit in metformin co-treatment before and during IVF in women with PCO without any other features of PCOS. Clinical Trials.gov: NCT01046032." }, "22902073": { "QUESTION": "Estimated fetal weight by ultrasound: a modifiable risk factor for cesarean delivery?", "CONTEXTS": [ "The purpose of this study was to investigate whether knowledge of ultrasound-obtained estimated fetal weight (US-EFW) is a risk factor for cesarean delivery (CD).", "Retrospective cohort from a single center in 2009-2010 of singleton, term live births. CD rates were compared for women with and without US-EFW within 1 month of delivery and adjusted for potential confounders.", "Of the 2329 women in our cohort, 50.2% had US-EFW within 1 month of delivery. CD was significantly more common for women with US-EFW (15.7% vs 10.2%; P<.001); after we controlled for confounders, US-EFW remained an independent risk factor for CD (odds ratio, 1.44; 95% confidence interval, 1.1-1.9). The risk increased when US-EFW was>3500 g (odds ratio, 1.8; 95% confidence interval, 1.3-2.7)." ], "LABELS": [ "OBJECTIVE", "STUDY DESIGN", "RESULTS" ], "MESHES": [ "Birth Weight", "Cesarean Section", "Female", "Fetal Weight", "Gestational Age", "Humans", "Predictive Value of Tests", "Pregnancy", "Retrospective Studies", "Risk Factors", "Ultrasonography, Prenatal" ], "YEAR": "2012", "reasoning_required_pred": "yes", "reasoning_free_pred": "yes", "final_decision": "yes", "LONG_ANSWER": "Knowledge of US-EFW, above and beyond the impact of fetal size itself, increases the risk of CD. Acquisition of US-EFW near term appears to be an independent and potentially modifiable risk factor for CD." }, "26471488": { "QUESTION": "Does Mammographic Density have an Impact on the Margin Re-excision Rate After Breast-Conserving Surgery?", "CONTEXTS": [ "Limited and conflicting data exist on an association between mammographic density (MD) and re-excision rates after breast-conserving surgery (BCS). Additionally, the correlation of MD with resection of unnecessary margins during initial BCS is unknown.", "All women with a diagnosis of breast cancer from 2003 to 2012 and enrolled in a larger study on MD were evaluated. Operative and pathology reports were reviewed to determine margin resection and involvement. Mammographic density was determined both by breast imaging-reporting and data system (BI-RADS) classification and by an automated software program (Volpara Solutions). Additional margins were deemed unnecessary if the lumpectomy specimen margin was free of invasive tumor [\u22652 mm for ductal carcinoma in situ (DCIS)] or if further re-excision was needed.", "Of 655 patients, 398 (60.8%) had BCS, whereas 226 (34.5%) underwent initial mastectomy. The women with denser breasts (BI-RADS 3 or 4) underwent initial mastectomy more frequently than the women with less dense breasts (40.0 vs. 30.5%, respectively; p = 0.0118). Of the patients with BCS, 166 (41.7%) required separate re-excision. Additional margins were taken during BCS in 192 (48.2%) patients, with 151 (78.6%) proving to be unnecessary. In the bivariable analysis, the patients with denser breasts according to BI-RADS classification and volumetric density showed a trend toward requiring more frequent re-excision, but this association was not seen in the multivariable analysis. The rate of unnecessary margins did not differ by breast density. In the multivariate analysis, the re-excision rates increased with DCIS (p<0.0003) and decreased with resection of additional margins (p = 0.0043)." ], "LABELS": [ "BACKGROUND", "METHODS", "RESULTS" ], "MESHES": [ "Aged", "Biomarkers, Tumor", "Breast Density", "Breast Neoplasms", "Carcinoma, Ductal, Breast", "Carcinoma, Intraductal, Noninfiltrating", "Female", "Follow-Up Studies", "Humans", "Immunoenzyme Techniques", "Mammary Glands, Human", "Mastectomy, Segmental", "Middle Aged", "Neoplasm Invasiveness", "Neoplasm Staging", "Prognosis", "Receptor, ErbB-2", "Receptors, Estrogen", "Receptors, Progesterone", "Retrospective Studies" ], "YEAR": "2016", "reasoning_required_pred": "no", "reasoning_free_pred": "no", "final_decision": "no", "LONG_ANSWER": "Mammographic density is not associated with an increased need for re-excision or resection of unnecessary margins at initial BCS." }, "16647887": { "QUESTION": "Are women who are treated for hypothyroidism at risk for pregnancy complications?", "CONTEXTS": [ "The purpose of this study was to investigate the outcomes that are associated with pregnancy and treated hypothyroidism.", "This was a retrospective cohort study of all women who received prenatal care and were delivered at the University of California, San Francisco, between 1989 and 2001. All patients with hypothyroidism diagnosed before pregnancy or early in pregnancy were identified. Maternal, fetal, and obstetric outcomes were then collected and analyzed for women with hypothyroidism and compared with women without hypothyroidism.", "Among 20,499 deliveries, there were 419 women (2.1%) who were treated for hypothyroidism during the study period. Hypothyroidism was more common among women>or =35 years old, white women, and women without Medicaid insurance. Treated hypothyroidism was not associated with any increase in maternal, fetal, or neonatal complications. In addition, hypothyroidism did not affect mode of delivery." ], "LABELS": [ "OBJECTIVE", "STUDY DESIGN", "RESULTS" ], "MESHES": [ "Adult", "Age Distribution", "Cohort Studies", "Female", "Humans", "Hypothyroidism", "Pregnancy", "Pregnancy Complications", "Prevalence", "Retrospective Studies", "Risk Assessment" ], "YEAR": "2006", "reasoning_required_pred": "no", "reasoning_free_pred": "no", "final_decision": "no", "LONG_ANSWER": "Compared with patients without hypothyroidism, patients with treated hypothyroidism are not at any increased risk for perinatal morbidity." }, "22668712": { "QUESTION": "Internal derangement of the temporomandibular joint: is there still a place for ultrasound?", "CONTEXTS": [ "The aim of this study was to assess the diagnostic value of articular sounds, standardized clinical examination, and standardized articular ultrasound in the detection of internal derangements of the temporomandibular joint.", "Forty patients and 20 asymptomatic volunteers underwent a standardized interview, physical examination, and static and dynamic articular ultrasound. Sensitivity, specificity, and predictive values were calculated using magnetic resonance as the reference test.", "A total of 120 temporomandibular joints were examined. Based on our findings, the presence of articular sounds and physical signs are often insufficient to detect disk displacement. Imaging by static and dynamic high-resolution ultrasound demonstrates considerably lower sensitivity when compared with magnetic resonance. Some of the technical difficulties resulted from a limited access because of the presence of surrounding bone structures." ], "LABELS": [ "OBJECTIVE", "STUDY DESIGN", "RESULTS" ], "MESHES": [ "Adult", "Case-Control Studies", "Female", "Humans", "Joint Dislocations", "Magnetic Resonance Imaging", "Male", "Middle Aged", "Prospective Studies", "Reference Standards", "Sensitivity and Specificity", "Statistics, Nonparametric", "Surveys and Questionnaires", "Temporomandibular Joint Disc", "Temporomandibular Joint Disorders", "Ultrasonography", "Young Adult" ], "YEAR": "2012", "reasoning_required_pred": "no", "reasoning_free_pred": "no", "final_decision": "no", "LONG_ANSWER": "The present study does not support the recommendation of ultrasound as a conclusive diagnostic tool for internal derangements of the temporomandibular joint." }, "18439500": { "QUESTION": "Laparoscopic myomectomy: do size, number, and location of the myomas form limiting factors for laparoscopic myomectomy?", "CONTEXTS": [ "To assess whether it is possible for an experienced laparoscopic surgeon to perform efficient laparoscopic myomectomy regardless of the size, number, and location of the myomas.", "Prospective observational study (Canadian Task Force classification II-1).", "Tertiary endoscopy center.", "A total of 505 healthy nonpregnant women with symptomatic myomas underwent laparoscopic myomectomy at our center. No exclusion criteria were based on the size, number, or location of myomas.", "Laparoscopic myomectomy and modifications of the technique: enucleation of the myoma by morcellation while it is still attached to the uterus with and without earlier devascularization.", "In all, 912 myomas were removed in these 505 patients laparoscopically. The mean number of myomas removed was 1.85 +/- 5.706 (95% CI 1.72-1.98). In all, 184 (36.4%) patients had multiple myomectomy. The mean size of the myomas removed was 5.86 +/- 3.300 cm in largest diameter (95% CI 5.56-6.16 cm). The mean weight of the myomas removed was 227.74 +/- 325.801 g (95% CI 198.03-257.45 g) and median was 100 g. The median operating time was 60 minutes (range 30-270 minutes). The median blood loss was 90 mL (range 40-2000 mL). Three comparisons were performed on the basis of size of the myomas (<10 cm and>or=10 cm in largest diameter), number of myomas removed (or=5 myomas), and the technique (enucleation of the myomas by morcellation while the myoma is still attached to the uterus and the conventional technique). In all these comparisons, although the mean blood loss, duration of surgery, and hospital stay were greater in the groups in which larger myomas or more myomas were removed or the modified technique was performed as compared with their corresponding study group, the weight and size of removed myomas were also proportionately larger in these groups. Two patients were given the diagnosis of leiomyosarcoma in their histopathology and 1 patient developed a diaphragmatic parasitic myoma followed by a leiomyoma of the sigmoid colon. Six patients underwent laparoscopic hysterectomy 4 to 6 years after the surgery for recurrent myomas. One conversion to laparotomy occurred and 1 patient underwent open subtotal hysterectomy for dilutional coagulopathy." ], "LABELS": [ "STUDY OBJECTIVE", "DESIGN", "SETTING", "PATIENTS", "INTERVENTIONS", "MEASUREMENTS AND MAIN RESULTS" ], "MESHES": [ "Adult", "Cohort Studies", "Female", "Gynecologic Surgical Procedures", "Humans", "Laparoscopy", "Leiomyomatosis", "Postoperative Complications", "Uterine Neoplasms" ], "YEAR": null, "reasoning_required_pred": "yes", "reasoning_free_pred": "no", "final_decision": "no", "LONG_ANSWER": "Laparoscopic myomectomy can be performed by experienced surgeons regardless of the size, number, or location of the myomas." }, "23347337": { "QUESTION": "Is intensive chemotherapy safe for rural cancer patients?", "CONTEXTS": [ "To provide equality of cancer care to rural patients, Townsville Cancer Centre administers intensive chemotherapy regimens to rural patients with node-positive breast and metastatic colorectal cancers at the same doses as urban patients. Side-effects were usually managed by rural general practitioners locally.AIM: The aim is to determine the safety of this practice by comparing the profile of serious adverse events and dose intensities between urban and rural patients at the Townsville Cancer Centre.", "A retrospective audit was conducted in patients with metastatic colorectal and node-positive breast cancers during a 24-month period. Fisher's exact test was used for analysis. Rurality was determined as per rural, remote and metropolitan classification.", "Of the 121 patients included, 70 and 51 patients had breast and colon cancers respectively. The urban versus rural patient split among all patients, breast and colorectal cancer subgroups was 68 versus 53, 43 versus 27 and 25 versus 26 respectively. A total of 421 cycles was given with dose intensity of>95% for breast cancer in both groups (P>0.05). Rate of febrile neutropenia was 9.3% versus 7.4% (P = 0.56). For XELOX, rate of diarrhoea was 20% versus 19% (P = 0.66) and rate of vomiting was 20% versus 11% (P = 0.11). Only two patients were transferred to Townsville for admission. No toxic death occurred in either group." ], "LABELS": [ "BACKGROUND", "METHOD", "RESULTS" ], "MESHES": [ "Adult", "Aged", "Antineoplastic Agents", "Breast Neoplasms", "Colonic Neoplasms", "Diarrhea", "Female", "Humans", "Male", "Middle Aged", "Retrospective Studies", "Rural Population", "Vomiting" ], "YEAR": "2013", "reasoning_required_pred": "yes", "reasoning_free_pred": "yes", "final_decision": "yes", "LONG_ANSWER": "It appears safe to administer intensive chemotherapy regimens at standard doses to rural patients without increased morbidity or mortality. Support for general practitioners through phone or videoconferencing may reduce the safety concerns." }, "22970993": { "QUESTION": "Does sex affect the outcome of laparoscopic cholecystectomy?", "CONTEXTS": [ "The aim of our study was to determine the effect of sex on the outcome of laparoscopic cholecystectomy in terms of operative time, conversion to open cholecystectomy, postoperative complications and mean hospital stay.", "In this retrospective observational study, we analyzed the medical records of 2061 patients who underwent laparoscopic cholecystectomy in the surgical department of Khyber Teaching Hospital (Peshawar, Pakistan) between March 2008 and January 2010. \u03c7(2) test and t-test were respectively used to analyze categorical and numerical variables. P\u2009\u2264\u20090.05 was considered significant.", "The study included 1772 female and 289 male patients. The mean age for male patients was 44.07\u2009\u00b1\u200911.91 years compared to 41.29\u2009\u00b1\u200912.18 years for female patients (P\u2009=\u20090.706). Laparoscopic cholecystectomy was successfully completed in 1996 patients. The conversion rate was higher in men (P\u2009<\u20090.001), and the mean operating time was longer in men (P\u2009<\u20090.001). Bile duct injuries occurred more frequently in men (P\u2009<\u20090.001). Gallbladder perforation and gallstone spillage also occurred more commonly in men (P\u2009=\u20090.001); similarly severe inflammation was reported more in male patients (P\u2009=\u20090001). There were no statistically significant differences in mean hospital stay, wound infection and port-site herniation between men and women. Multivariate regression analysis showed that the male sex is an independent risk factor for conversion to open cholecystectomy (odds ratio\u2009=\u20092.65, 95% confidence interval: 1.03-6.94, P\u2009=\u20090.041) and biliary injuries (odds ratio\u2009=\u20090.95, 95% confidence interval: 0.91-0.99, P-value\u2009=\u20090.036)." ], "LABELS": [ "INTRODUCTION", "METHODS", "RESULTS" ], "MESHES": [ "Adult", "Cholecystectomy, Laparoscopic", "Clinical Competence", "Female", "Gallbladder Diseases", "Humans", "Length of Stay", "Male", "Middle Aged", "Pakistan", "Patient Selection", "Retrospective Studies", "Risk Factors", "Sex Factors", "Time Factors", "Treatment Outcome" ], "YEAR": "2013", "reasoning_required_pred": "yes", "reasoning_free_pred": "yes", "final_decision": "yes", "LONG_ANSWER": "Laparoscopic cholecystectomy is often challenging in men on account of more adhesions and inflammation. This leads to higher conversion rates and more postoperative complications. Optimized planning and a more experienced operating surgeon may help overcome these problems." }, "16498158": { "QUESTION": "Is atropine needed with ketamine sedation?", "CONTEXTS": [ "To compare atropine with placebo as an adjunct to ketamine sedation in children undergoing minor painful procedures. Outcome measures included hypersalivation, side effect profile, parental/patient satisfaction, and procedural success rate.", "Children aged between 1 and 16 years of age requiring ketamine procedural sedation in a tertiary emergency department were randomised to receive 0.01 mg/kg of atropine or placebo. All received 4 mg/kg of intramuscular ketamine. Tolerance and sedation scores were recorded throughout the procedure. Side effects were recorded from the start of sedation until discharge. Parental and patient satisfaction scores were obtained at discharge and three to five days after the procedure, with the opportunity to report side effects encountered at home.", "A total of 83 patients aged 13 months to 14.5 years (median age 3.4 years) were enrolled over a 16 month period. Hypersalivation occurred in 11.4% of patients given atropine compared with 30.8% given placebo (odds ratio (OR) 0.29, 95% confidence interval (CI) 0.09 to 0.91). A transient rash was observed in 22.7% of the atropine group compared with 5.1% of the placebo group (OR 5.44, 95% CI 1.11 to 26.6). Vomiting during recovery occurred in 9.1% of atropine patients compared with 25.6% of placebo patients (OR 0.29, 95% CI 0.09 to 1.02). There was a trend towards better tolerance in the placebo group. No patient experienced serious side effects." ], "LABELS": [ "OBJECTIVE", "METHODS", "RESULTS" ], "MESHES": [ "Adjuvants, Anesthesia", "Adolescent", "Anesthetics, Dissociative", "Atropine", "Child", "Child, Preschool", "Double-Blind Method", "Drug Combinations", "Female", "Humans", "Infant", "Injections, Intramuscular", "Ketamine", "Male", "Minor Surgical Procedures", "Pain", "Patient Satisfaction", "Prospective Studies", "Sialorrhea" ], "YEAR": "2006", "reasoning_required_pred": "yes", "reasoning_free_pred": "yes", "final_decision": "yes", "LONG_ANSWER": "Ketamine sedation was successful and well tolerated in all cases. The use of atropine as an adjunct for intramuscular ketamine sedation in children significantly reduces hypersalivation and may lower the incidence of post-procedural vomiting. Atropine is associated with a higher incidence of a transient rash. No serious adverse events were noted." }, "24622801": { "QUESTION": "Does implant coating with antibacterial-loaded hydrogel reduce bacterial colonization and biofilm formation in vitro?", "CONTEXTS": [ "Implant-related infections represent one of the most severe complications in orthopaedics. A fast-resorbable, antibacterial-loaded hydrogel may reduce or prevent bacterial colonization and biofilm formation of implanted biomaterials.QUESTIONS/", "We asked: (1) Is a fast-resorbable hydrogel able to deliver antibacterial compounds in vitro? (2) Can a hydrogel (alone or antibacterial-loaded) coating on implants reduce bacterial colonization? And (3) is intraoperative coating feasible and resistant to press-fit implant insertion?", "We tested the ability of Disposable Antibacterial Coating (DAC) hydrogel (Novagenit Srl, Mezzolombardo, Italy) to deliver antibacterial agents using spectrophotometry and a microbiologic assay. Antibacterial and antibiofilm activity were determined by broth microdilution and a crystal violet assay, respectively. Coating resistance to press-fit insertion was tested in rabbit tibias and human femurs.", "Complete release of all tested antibacterial compounds was observed in less than 96 hours. Bactericidal and antibiofilm effect of DAC hydrogel in combination with various antibacterials was shown in vitro. Approximately 80% of the hydrogel coating was retrieved on the implant after press-fit insertion." ], "LABELS": [ "BACKGROUND", "PURPOSES", "METHODS", "RESULTS" ], "MESHES": [ "Absorbable Implants", "Animals", "Anti-Bacterial Agents", "Biofilms", "Coated Materials, Biocompatible", "Drug Carriers", "Drug Delivery Systems", "Feasibility Studies", "Femur", "Humans", "Hydrogel, Polyethylene Glycol Dimethacrylate", "In Vitro Techniques", "Microbial Sensitivity Tests", "Prosthesis-Related Infections", "Rabbits", "Staphylococcus aureus", "Staphylococcus epidermidis", "Tibia" ], "YEAR": "2014", "reasoning_required_pred": "yes", "reasoning_free_pred": "yes", "final_decision": "yes", "LONG_ANSWER": "Implant coating with an antibacterial-loaded hydrogel reduces bacterial colonization and biofilm formation in vitro." }, "15000338": { "QUESTION": "Do family physicians know the costs of medical care?", "CONTEXTS": [ "To determine the cost of 46 commonly used investigations and therapies and to assess British Columbia family doctors' awareness of these costs.", "Mailed survey asking about costs of 23 investigations and 23 therapies relevant to family practice. A random sample of 600 doctors was asked to report their awareness of costs and to estimate costs of the 46 items.", "British Columbia.", "Six hundred family physicians.", "Estimates within 25% of actual cost were considered correct. Associations between cost awareness and respondents'characteristics (eg, sex, practice location) were sought. Degree of error in estimates was also assessed.", "Overall, 283 (47.2%) surveys were returned and 259 analyzed. Few respondents estimated costs within 25% of true cost, and estimates were highly variable. Physicians underestimated costs of expensive drugs and laboratory investigations and overestimated costs of inexpensive drugs. Cost awareness did not correlate with sex, practice location, College certification, faculty appointment, or years in practice." ], "LABELS": [ "OBJECTIVE", "DESIGN", "SETTING", "PARTICIPANTS", "MAIN OUTCOME MEASURES", "RESULTS" ], "MESHES": [ "Awareness", "British Columbia", "Clinical Laboratory Techniques", "Diagnostic Imaging", "Fees, Medical", "Fees, Pharmaceutical", "Female", "Health Care Costs", "Health Care Surveys", "Humans", "Male", "Physicians, Family", "Practice Patterns, Physicians'" ], "YEAR": "2004", "reasoning_required_pred": "no", "reasoning_free_pred": "no", "final_decision": "no", "LONG_ANSWER": "Family doctors in British Columbia have little awareness of the costs of medical care." }, "22491528": { "QUESTION": "Combining process indicators to evaluate quality of care for surgical patients with colorectal cancer: are scores consistent with short-term outcome?", "CONTEXTS": [ "To determine if composite measures based on process indicators are consistent with short-term outcome indicators in surgical colorectal cancer care.", "Longitudinal analysis of consistency between composite measures based on process indicators and outcome indicators for 85 Dutch hospitals.", "The Dutch Surgical Colorectal Audit database, the Netherlands.", "4732 elective patients with colon carcinoma and 2239 with rectum carcinoma treated in 85 hospitals were included in the analyses.", "All available process indicators were aggregated into five different composite measures. The association of the different composite measures with risk-adjusted postoperative mortality and morbidity was analysed at the patient and hospital level.", "At the patient level, only one of the composite measures was negatively associated with morbidity for rectum carcinoma. At the hospital level, a strong negative association was found between composite measures and hospital mortality and morbidity rates for rectum carcinoma (p<0.05), and hospital morbidity rates for colon carcinoma." ], "LABELS": [ "OBJECTIVE", "DESIGN", "SETTING", "PARTICIPANTS", "MAIN OUTCOME MEASURES", "RESULTS" ], "MESHES": [ "Colorectal Neoplasms", "Databases, Factual", "Female", "Hospitals, Public", "Humans", "Longitudinal Studies", "Male", "Netherlands", "Outcome Assessment (Health Care)", "Quality Assurance, Health Care", "Quality Indicators, Health Care", "Surgical Procedures, Operative" ], "YEAR": "2012", "reasoning_required_pred": "maybe", "reasoning_free_pred": "maybe", "final_decision": "maybe", "LONG_ANSWER": "For individual patients, a high score on the composite measures based on process indicators is not associated with better short-term outcome. However, at the hospital level, a good score on the composite measures based on process indicators was consistent with more favourable risk-adjusted short-term outcome rates." }, "11247896": { "QUESTION": "Can APC mutation analysis contribute to therapeutic decisions in familial adenomatous polyposis?", "CONTEXTS": [ "In familial adenomatous polyposis (FAP), correlations between site of mutation in the adenomatous polyposis coli (APC) gene and severity of colonic polyposis or extracolonic manifestations are well known. While mutation analysis is important for predictive diagnosis in persons at risk, its relevance for clinical management of individual patients is open to question.", "We examined 680 unrelated FAP families for germline mutations in the APC gene. Clinical information was obtained from 1256 patients.", "APC mutations were detected in 48% (327/680) of families. Age at diagnosis of FAP based on bowel symptoms and age at diagnosis of colorectal cancer in untreated patients were used as indicators of the severity of the natural course of the disease. A germline mutation was detected in 230 of 404 patients who were diagnosed after onset of bowel symptoms (rectal bleeding, abdominal pain, diarrhoea). When these patients were grouped according to the different sites of mutations, mean values for age at onset of disease differed significantly: patients carrying APC mutations at codon 1309 showed a disease onset 10 years earlier (mean age 20 years) compared with patients with mutations between codons 168 and 1580 (except codon 1309) (mean age 30 years), whereas patients with mutations at the 5' end of codon 168 or the 3' end of codon 1580 were diagnosed at a mean age of 52 years. Within each group of patients however large phenotypic variation was observed, even among patients with identical germline mutations. A higher incidence of desmoids was found in patients with mutations between codons 1445 and 1580 compared with mutations at other sites, while no correlation between site of mutation and presence of duodenal adenomas was observed." ], "LABELS": [ "BACKGROUND AND AIMS", "METHODS", "RESULTS" ], "MESHES": [ "Adenomatous Polyposis Coli", "Adult", "Age of Onset", "Aged", "DNA Mutational Analysis", "Disease Progression", "Female", "Genes, APC", "Genotype", "Germ-Line Mutation", "Humans", "Male", "Middle Aged", "Patient Selection", "Pedigree", "Phenotype", "Polymerase Chain Reaction", "Polymorphism, Single-Stranded Conformational", "Severity of Illness Index" ], "YEAR": "2001", "reasoning_required_pred": "yes", "reasoning_free_pred": "maybe", "final_decision": "maybe", "LONG_ANSWER": "As age at manifestation and course of the disease may be rather variable, even in carriers of identical germline mutations, therapeutic decisions should be based on colonoscopic findings in individual patients rather than on the site of mutation. However, in patients with mutations within codons 1445-1580, it may be advisable to postpone elective colectomy because desmoids may arise through surgical intervention." }, "19145527": { "QUESTION": "Do emergency medical services professionals think they should participate in disease prevention?", "CONTEXTS": [ "The primary objective of the study was to determine emergency medical services (EMS) professionals' opinions regarding participation in disease and injury prevention programs. A secondary objective was to determine the proportion of EMS professionals who had participated in disease prevention programs.", "As part of the National Registry of Emergency Medical Technicians' biennial reregistration process, EMS professionals reregistering in 2006 were asked to complete an optional survey regarding their opinions on and participation in disease and injury prevention. Demographic characteristics were also collected. Data were analyzed using descriptive statistics and 99% confidence intervals (CIs). The chi-square test was used to compare differences by responder demographics (alpha = 0.01). A 10% difference between groups was determined to be clinically significant.", "The survey was completed by 27,233 EMS professionals. Of these responders, 82.7% (99% CI: 82.1-83.3) felt that EMS professionals should participate in disease prevention, with those working 20 to 29 hours per week being the least likely to think they should participate (67.4%, p<0.001). About a third, 33.8% (99% CI: 33.1-34.6), of the respondents reported having provided prevention services, with those having a graduate degree (43.5%, p<0.001), those working in EMS for more than 21 years (44%, p<0.001), those working for the military (57%, p<0.001), those working 60 to 69 hours per week (41%, p<0.001), and those responding to zero emergency calls in a typical week (43%, p<0.001) being the most likely to report having provided prevention services. About half, 51.1% (99% CI: 50.4-51.9), of the respondents agreed that prevention services should be provided during emergency calls, and 7.7% (99% CI: 7.3-8.1) of the respondents reported providing prevention services during emergency calls. No demographic differences existed. Those who had participated in prevention programs were more likely to respond that EMS professionals should participate in prevention (92% vs. 82%, p<0.001). Further, those who had provided prevention services during emergency calls were more likely to think EMS professionals should provide prevention services during emergency calls (81% vs. 51%, p<0.001)." ], "LABELS": [ "OBJECTIVE", "METHODS", "RESULTS" ], "MESHES": [ "Accidents, Home", "Accidents, Occupational", "Attitude of Health Personnel", "Data Collection", "Emergency Medical Technicians", "Health Promotion", "Humans", "Wounds and Injuries" ], "YEAR": null, "reasoning_required_pred": "maybe", "reasoning_free_pred": "yes", "final_decision": "maybe", "LONG_ANSWER": "The majority of EMS professionals thought that they should participate in disease and injury prevention programs. The respondents were mixed as to whether prevention services should be provided while on emergency calls, but those with experience providing these services were more likely to agree with providing them during emergency calls." }, "18568239": { "QUESTION": "Is the ability to perform transurethral resection of the prostate influenced by the surgeon's previous experience?", "CONTEXTS": [ "To evaluate the influence of the urologist's experience on the surgical results and complications of transurethral resection of the prostate (TURP).", "Sixty-seven patients undergoing transurethral resection of the prostate without the use of a video camera were randomly allocated into three groups according to the urologist's experience: a urologist having done 25 transurethral resections of the prostate (Group I - 24 patients); a urologist having done 50 transurethral resections of the prostate (Group II - 24 patients); a senior urologist with vast transurethral resection of the prostate experience (Group III - 19 patients). The following were recorded: the weight of resected tissue, the duration of the resection procedure, the volume of irrigation used, the amount of irrigation absorbed and the hemoglobin and sodium levels in the serum during the procedure.", "There were no differences between the groups in the amount of irrigation fluid used per operation, the amount of irrigation fluid absorbed or hematocrit and hemoglobin variation during the procedure. The weight of resected tissue per minute was approximately four times higher in group III than in groups I and II. The mean absorbed irrigation fluid was similar between the groups, with no statistical difference between them (p=0.24). Four patients (6%) presented with TUR syndrome, without a significant difference between the groups." ], "LABELS": [ "PURPOSE", "PATIENTS AND METHODS", "RESULTS" ], "MESHES": [ "Aged", "Anti-Infective Agents, Local", "Clinical Competence", "Ethanol", "Humans", "Hyponatremia", "Indicators and Reagents", "Male", "Middle Aged", "Organ Size", "Prospective Studies", "Prostate", "Quality of Health Care", "Sorbitol", "Statistics, Nonparametric", "Syndrome", "Time Factors", "Transurethral Resection of Prostate", "Urology" ], "YEAR": "2008", "reasoning_required_pred": "no", "reasoning_free_pred": "yes", "final_decision": "yes", "LONG_ANSWER": "The senior urologist was capable of resecting four times more tissue per time unit than the more inexperienced surgeons. Therefore, a surgeon's experience may be important to reduce the risk of secondary TURP due to recurring adenomas or adenomas that were incompletely resected. However, the incidence of complications was the same between the three groups." }, "22075911": { "QUESTION": "Is there a differential in the dental health of new recruits to the British Armed Forces?", "CONTEXTS": [ "Figures from the British Defence Dental Services reveal that serving personnel in the British Army have a persistently lower level of dental fitness than those in the Royal Navy or the Royal Air Force. No research had been undertaken to ascertain if this reflects the oral health of recruits joining each Service. This study aimed to pilot a process for collecting dental and sociodemographic data from new recruits to each Service and examine the null hypothesis that no differences in dental health existed.", "Diagnostic criteria were developed, a sample size calculated and data collected at the initial training establishments of each Service.", "Data for 432 participants were entered into the analysis. Recruits in the Army sample had a significantly greater prevalence of dental decay and greater treatment resource need than either of the other two Services. Army recruits had a mean number of 2.59 (2.08, 3.09) decayed teeth per recruit, compared to 1.93 (1.49, 2.39 p<0.01) in Royal Navy recruits and 1.26 (0.98, 1.53 p<0.001) in Royal Air Force recruits. Among Army recruits 62.7% were from the two most deprived quintiles of the Index of Multiple Deprivation compared to 42.5% of Royal Naval recruits and 36.6% of Royal Air Force recruits." ], "LABELS": [ "BACKGROUND AND AIM", "METHOD", "RESULTS" ], "MESHES": [ "Adolescent", "Cross-Sectional Studies", "DMF Index", "Dental Caries", "Female", "Health Status Disparities", "Humans", "Male", "Military Dentistry", "Military Personnel", "Oral Health", "Pilot Projects", "Prevalence", "Smoking", "Statistics, Nonparametric", "United Kingdom", "Young Adult" ], "YEAR": "2011", "reasoning_required_pred": "yes", "reasoning_free_pred": "yes", "final_decision": "yes", "LONG_ANSWER": "A significant difference in dental health between recruits to each Service does exist and is a likely to be a reflection of the sociodemographic background from which they are drawn." }, "10401824": { "QUESTION": "Is laparoscopic reoperation for failed antireflux surgery feasible?", "CONTEXTS": [ "Laparoscopic techniques can be used to treat patients whose antireflux surgery has failed.", "Case series.", "Two academic medical centers.", "Forty-six consecutive patients, of whom 21 were male and 25 were female (mean age, 55.6 years; range, 15-80 years). Previous antireflux procedures were laparoscopic (21 patients), laparotomy (21 patients), thoracotomy (3 patients), and thoracoscopy (1 patient).", "The cause of failure, operative and postoperative morbidity, and the level of follow-up satisfaction were determined for all patients.", "The causes of failure were hiatal herniation (31 patients [67%]), fundoplication breakdown (20 patients [43%]), fundoplication slippage (9 patients [20%]), tight fundoplication (5 patients [11%]), misdiagnosed achalasia (2 patients [4%]), and displaced Angelchik prosthesis (2 patients [4%]). Twenty-two patients (48%) had more than 1 cause. Laparoscopic reoperative procedures were Nissen fundoplication (n = 22), Toupet fundoplication (n = 13), paraesophageal hernia repair (n = 4), Dor procedure (n = 2), Angelchik prosthesis removal (n = 2), Heller myotomy (n = 2), and the takedown of a wrap (n = 1). In addition, 18 patients required crural repair and 13 required paraesophageal hernia repair. The mean +/- SEM duration of surgery was 3.5+/-1.1 hours. Operative complications were fundus tear (n = 8), significant bleeding (n = 4), bougie perforation (n = 1), small bowel enterotomy (n = 1), and tension pneumothorax (n = 1). The conversion rate (from laparoscopic to an open procedure) was 20% overall (9 patients) but 0% in the last 10 patients. Mortality was 0%. The mean +/- SEM hospital stay was 2.3+/-0.9 days for operations completed laparoscopically. Follow-up was possible in 35 patients (76%) at 17.2+/-11.8 months. The well-being score (1 best; 10, worst) was 8.6+/-2.1 before and 2.9+/-2.4 after surgery (P<.001). Thirty-one (89%) of 35 patients were satisfied with their decision to have reoperation." ], "LABELS": [ "HYPOTHESIS", "DESIGN", "SETTING", "PATIENTS", "MAIN OUTCOME MEASURES", "RESULTS" ], "MESHES": [ "Adolescent", "Adult", "Aged", "Aged, 80 and over", "Feasibility Studies", "Female", "Follow-Up Studies", "Gastroesophageal Reflux", "Humans", "Laparoscopy", "Male", "Middle Aged", "Postoperative Complications", "Reoperation", "Treatment Failure" ], "YEAR": "1999", "reasoning_required_pred": "yes", "reasoning_free_pred": "yes", "final_decision": "yes", "LONG_ANSWER": "Antireflux surgery failures are most commonly associated with hiatal herniation, followed by the breakdown of the fundoplication. The laparoscopic approach may be used successfully to treat patients with failed antireflux operations. Good results were achieved despite the technical difficulty of the procedures." }, "21880023": { "QUESTION": "Does exercise during pregnancy prevent postnatal depression?", "CONTEXTS": [ "To study whether exercise during pregnancy reduces the risk of postnatal depression.", "Randomized controlled trial.", "Trondheim and Stavanger University Hospitals, Norway.", "Eight hundred and fifty-five pregnant women were randomized to intervention or control groups.", "The intervention was a 12 week exercise program, including aerobic and strengthening exercises, conducted between week 20 and 36 of pregnancy. One weekly group session was led by physiotherapists, and home exercises were encouraged twice a week. Control women received regular antenatal care.", "Edinburgh Postnatal Depression Scale (EPDS) completed three months after birth. Scores of 10 or more and 13 or more suggested probable minor and major depression, respectively.", "Fourteen of 379 (3.7%) women in the intervention group and 17 of 340 (5.0%) in the control group had an EPDS score of \u226510 (p=0.46), and four of 379 (1.2%) women in the intervention group and eight of 340 (2.4%) in the control group had an EPDS score of \u226513 (p=0.25). Among women who did not exercise prior to pregnancy, two of 100 (2.0%) women in the intervention group and nine of 95 (9.5%) in the control group had an EPDS score of \u226510 (p=0.03)." ], "LABELS": [ "OBJECTIVE", "DESIGN", "SETTING", "POPULATION AND SAMPLE", "METHODS", "MAIN OUTCOME MEASURES", "RESULTS" ], "MESHES": [ "Adult", "Depression, Postpartum", "Exercise", "Exercise Therapy", "Female", "Humans", "Pregnancy", "Prenatal Care", "Self Report", "Treatment Outcome" ], "YEAR": "2012", "reasoning_required_pred": "yes", "reasoning_free_pred": "no", "final_decision": "no", "LONG_ANSWER": "We did not find a lower prevalence of high EPDS scores among women randomized to regular exercise during pregnancy compared with the control group. However, a subgroup of women in the intervention group who did not exercise regularly prior to pregnancy had a reduced risk of postnatal depression." }, "21726930": { "QUESTION": "Is endometrial polyp formation associated with increased expression of vascular endothelial growth factor and transforming growth factor-beta1?", "CONTEXTS": [ "Endometrial polyp is a common cause of abnormal uterine bleeding, but the etiology and pathogenesis remain unclear. Vascular endothelial growth factor (VEGF) is angiogenic, related to thick walled vessels and transforming growth factor-beta1 (TGF-\u03b21) is related to fibrotic tissue, which are characteristics of endometrial polyps. The primary objective of this study was to find out if endometrial polyp formation is associated with increased expression of VEGF or TGF-\u03b21, or both. A secondary objective is to determine if the changes are related to steroid receptor expression.", "This prospective study compared VEGF and TGF-\u03b21 expression of endometrial polyps and adjacent endometrial tissue in 70 premenopausal women. The comparison of results was separately made for endometrium specimens obtained in the proliferative and secretory phases. The results were correlated with the steroid receptors (estrogen receptor and progesterone receptor) expression.", "The score of VEGF in glandular cells of endometrial polyps was significantly higher than the score in adjacent endometrium, both in the proliferative phase (P<0.001) and the secretory phase (P=0.03); the score of VEGF in stromal cells of endometrial polyps was significantly higher than the score in adjacent endometrium only in proliferative phase (P=0.006). The score of TGF-\u03b21 in glandular cells of endometrial polyps was significantly higher than the score in adjacent endometrium in proliferative phase (P=0.02); whereas the score of TGF-\u03b21 in stromal cells of endometrial polyps was significantly higher than the score in adjacent endometrium, both in the proliferative phase (P=0.006) and the secretory phase (P=0.008). There was a significant correlation between the expression of steroid receptors and VEGF and TGF-\u03b21 (Spearman's correlation P<0.001 and P<0.05, respectively)." ], "LABELS": [ "OBJECTIVE", "STUDY DESIGN", "RESULTS" ], "MESHES": [ "Adult", "Biopsy", "Endometrium", "Female", "Follicular Phase", "Humans", "Hysteroscopy", "Immunohistochemistry", "Luteal Phase", "Middle Aged", "Polyps", "Prospective Studies", "Receptors, Estrogen", "Receptors, Progesterone", "Stromal Cells", "Transforming Growth Factor beta1", "Up-Regulation", "Uterine Diseases", "Vascular Endothelial Growth Factor A", "Young Adult" ], "YEAR": "2011", "reasoning_required_pred": "yes", "reasoning_free_pred": "yes", "final_decision": "yes", "LONG_ANSWER": "There was increased expression of TGF-\u03b21 and VEGF in polyps compared to adjacent normal endometrial tissue. It suggested that these cytokines might play a role in endometrial polyp formation. In addition, there was a significant correlation between steroid receptor expression and VEGF and TGF-\u03b21 expression." }, "22768311": { "QUESTION": "Is human cytomegalovirus infection associated with hypertension?", "CONTEXTS": [ "Recent studies have implicated the human cytomegalovirus (HCMV) as a possible pathogen for causing hypertension. We aimed to study the association between HCMV infection and hypertension in the United States National Health and Nutrition Examination Survey (NHANES).", "We analyzed data on 2979 men and 3324 women in the NHANES 1999-2002. We included participants aged 16-49 years who had valid data on HCMV infection and hypertension.", "Of the participants, 54.7% had serologic evidence of HCMV infection and 17.5% had hypertension. There were ethnic differences in the prevalence of HCMV infection (P<0.001) and hypertension (P<0.001). The prevalence of both increased with age (P<0.001). Before adjustment, HCMV seropositivity was significantly associated with hypertension in women (OR=1.63, 95% CI=1.25-2.13, P=0.001) but not in men. After adjustment for race/ethnicity, the association between HCMV seropositivity and hypertension in women remained significant (OR=1.55, 95% CI=1.20-2.02, P=0.002). Further adjustment for body mass index, diabetes status and hypercholesterolemia attenuated the association (OR=1.44, 95% CI=1.10-1.90, P=0.010). However, after adjusting for age, the association was no longer significant (OR=1.24, 95% CI=0.91-1.67, P=0.162)." ], "LABELS": [ "PURPOSE", "METHODS", "RESULTS" ], "MESHES": [ "Adolescent", "Adult", "Age Factors", "Cytomegalovirus", "Cytomegalovirus Infections", "Data Collection", "Female", "Humans", "Hypertension", "Male", "Middle Aged", "Prevalence", "Risk Factors", "Sex Characteristics", "United States" ], "YEAR": "2012", "reasoning_required_pred": "no", "reasoning_free_pred": "yes", "final_decision": "no", "LONG_ANSWER": "In this nationally representative population-based survey, HCMV seropositivity is associated with hypertension in women in the NHANES population. This association is largely explained by the association of hypertension with age and the increase in past exposure to HCMV with age." }, "25636371": { "QUESTION": "Is it possible to stop treatment with nucleos(t)ide analogs in patients with e-antigen negative chronic hepatitis B?", "CONTEXTS": [ "Treatment of HBeAg-negative chronic hepatitis B (CHB) with nucleos(t)ide analogues (NA) is usually indefinite, since the loss of HBsAg, as a criterion for its discontinuation, is a rare event. Recent evidence suggests that discontinuing NA therapy may be feasible in selected patients.", "To analyze the rate of virological relapse in patients with HBeAg-negative CHB who discontinued treatment with NAs.", "We performed a single-center observational study that included 140 patients with HBsAg-negative CHB. Twenty-two patients, who received only NAs, discontinued treatment for different reasons and were subsequently monitored. All had normal ALT and AST, undetectable DNA and absence of cirrhosis or significant comorbidities before stopping treatment.", "Twelve patients showed virologic relapse (54.54%). The mean interval between discontinuation and relapse was 6.38 months (\u00b1 1.9) (75% relapsed during the first 12 months after discontinuation). Five received adefovir, 1 lamivudine and adefovir, 1 tenofovir and 5 lamivudine alone. The mean treatment duration in this group was 38.5 months (\u00b1 4.5). The sustained response group had a higher mean age and longer treatment duration than patients with virologic relapse but these differences were not statistically significant." ], "LABELS": [ "BACKGROUND", "OBJECTIVES", "METHODS", "RESULTS" ], "MESHES": [ "Adult", "Aged", "Alanine Transaminase", "Antiviral Agents", "Aspartate Aminotransferases", "DNA, Viral", "Drug Therapy, Combination", "Female", "Hepatitis B e Antigens", "Hepatitis B virus", "Hepatitis B, Chronic", "Humans", "Liver Cirrhosis", "Male", "Middle Aged", "Nucleotides", "Recurrence", "Treatment Outcome" ], "YEAR": "2015", "reasoning_required_pred": "yes", "reasoning_free_pred": "maybe", "final_decision": "maybe", "LONG_ANSWER": "The results suggest that NA treatment can be stopped in selected patients with CHB as long as they are not cirrhotic, have completed a minimum period of treatment, have normal ALT and sustained undetectable DNA. These patients should be closely monitored during the first year and then indefinitely." }, "15050326": { "QUESTION": "Does radiotherapy around the time of pregnancy for Hodgkin's disease modify the risk of breast cancer?", "CONTEXTS": [ "To determine whether the risk of secondary breast cancer after radiotherapy (RT) for Hodgkin's disease is greater among women who underwent RT around time of pregnancy.", "The records of 382 women treated with RT for Hodgkin's disease were reviewed and divided into those who received RT around the time of pregnancy and those who were not pregnant. Comparisons of the overall incidence, actuarial rates, and latency to breast cancer between the two groups were made. Multivariate Cox regression modeling was performed to determine possible contributing factors.", "Of the 382 women, 14 developed breast cancer (3.7%). The increase in the overall incidence (16.0% vs. 2.3%, p = 0.0001) and the actuarial rate of breast cancer among the women in the pregnant group (p = 0.011) was statistically significant. The women treated around the time of pregnancy had a 10- and 15-year actuarial rate of breast cancer of 6.7% and 32.6%, respectively. The 10-year and 15-year actuarial rate for the nonpregnant women was 0.4% and 1.7%, respectively. The median latency from RT to the diagnosis of breast cancer was 13.1 and 18.9 years for women in the pregnant and nonpregnant groups, respectively. In the multivariate analysis, pregnancy around the time of RT was the only variable associated with an increased risk of breast cancer. The risk was dependent on the length of time from pregnancy to RT, with women receiving RT during pregnancy and within 1 month of pregnancy having an increased risk of breast cancer compared with nonpregnant women and women irradiated later than 1 month after pregnancy (hazard ratio, 22.49; 95% confidence interval, 5.56-90.88; p<0.001)." ], "LABELS": [ "PURPOSE", "METHODS AND MATERIALS", "RESULTS" ], "MESHES": [ "Adolescent", "Adult", "Breast Neoplasms", "Epidemiologic Methods", "Female", "Hodgkin Disease", "Humans", "Neoplasms, Radiation-Induced", "Neoplasms, Second Primary", "Pregnancy", "Time Factors" ], "YEAR": "2004", "reasoning_required_pred": "yes", "reasoning_free_pred": "yes", "final_decision": "yes", "LONG_ANSWER": "The results of this study indicate that the risk of breast cancer after RT is greater with irradiation around the time of pregnancy. This suggests that pregnancy is a time of increased sensitivity of breast tissue to the carcinogenic effects of radiation. Because of the small sample size and limited follow-up, additional studies are recommended to confirm these findings." }, "23412195": { "QUESTION": "Should displaced midshaft clavicular fractures be treated surgically?", "CONTEXTS": [ "This study was designed to compare clinical effectiveness of operative with nonoperative treatment for displaced midshaft clavicular fractures (DMCF).", "We systematically searched electronic databases (MEDILINE, EMBASE, CLINICAL, OVID, BIOSIS and Cochrane registry of controlled clinical trials) to identify randomized controlled trials (RCTs) in which operative treatment was compared with nonoperative treatment for DMCF from 1980 to 2012. The methodologic quality of trials was assessed. Data from chosen studies were pooled with using of fixed-effects and random-effects models with mean differences and risk ratios for continuous and dichotomous variables, respectively.", "Four RCTs with a total of 321 patients were screened for the present study. Results showed that the operative treatment was superior to the nonoperative treatment regarding the rate of nonunion [95\u00a0% confidence interval (CI) (0.05, 0.43), P\u00a0=\u00a00.0004], malunion [95\u00a0% CI (0.06, 0.34), P\u00a0<\u00a00.00001] and overall complication [95\u00a0% CI (0.43-0.76), P\u00a0=\u00a00.0001]. Subgroup analyses of complications revealed that significant differences were existed in the incidence of neurologic symptoms [95\u00a0% CI (0.20, 0.74), P\u00a0=\u00a00.004] and dissatisfaction with appearance [95\u00a0% CI (0.19, 0.65), P\u00a0=\u00a00.001]. Lack of consistent and standardized assessment data, insufficiency analysis that carried out showed improved functional outcomes (P\u00a0<\u00a00.05) in operative treatment." ], "LABELS": [ "PURPOSE", "METHODS", "RESULTS" ], "MESHES": [ "Clavicle", "Databases, Factual", "Fracture Fixation, Internal", "Fractures, Bone", "Humans", "Postoperative Complications", "Randomized Controlled Trials as Topic", "Recovery of Function", "Treatment Outcome" ], "YEAR": "2013", "reasoning_required_pred": "yes", "reasoning_free_pred": "yes", "final_decision": "yes", "LONG_ANSWER": "The available evidence suggests that the operative treatment for DMCF is associated with a lower rate of nonunion, malunion and complication than nonoperative treatment. This study supports traditional primary operative treatment for DMCF in active adults." }, "16968183": { "QUESTION": "Is laparoscopic cholecystectomy safe and acceptable as a day case procedure?", "CONTEXTS": [ "This study reviewed the results of performing day case laparoscopic cholecystectomy to assess the feasibility and safety of the procedure as a day case.", "This is a prospective study of 150 day case laparoscopic cholecystectomies performed between September 1999 and December 2004 under the care of the senior author. The results of a follow-up questionnaire to assess post-discharge clinical course and patient satisfaction were analyzed. All patients had commenced eating and drinking and were fully mobile before discharge home. The length of hospital stay was 4-8 hours.", "The mean age of the patients was 43 years; 134 patients had an American Society of Anesthesiologists grade I, the remaining 16 patients were grade II. The mean operative time was 41 minutes. There were no conversions to open procedures. There was no bleeding, no visceral injury, and no mortality. There was one admission directly from the day surgical unit (admission rate of 0.6%), but no readmission following discharge. No patients were admitted due to postoperative nausea or pain. Ninety-nine (66%) of 150 patients responded to our questionnaire: 97% were satisfied about the information they had received. Patients rated their satisfaction with the procedure as follows: 75% excellent, 21% good, 3% satisfied, and 1 patient un-satisfied. Ninety-four percent of the patients would recommend the procedure as a day case." ], "LABELS": [ "BACKGROUND", "MATERIALS AND METHODS", "RESULTS" ], "MESHES": [ "Adolescent", "Adult", "Aged", "Ambulatory Surgical Procedures", "Cholecystectomy, Laparoscopic", "Cost-Benefit Analysis", "Drinking", "Eating", "Feasibility Studies", "Female", "Follow-Up Studies", "Gallbladder Diseases", "Humans", "Length of Stay", "Male", "Middle Aged", "Patient Admission", "Patient Discharge", "Patient Satisfaction", "Postoperative Nausea and Vomiting", "Prospective Studies", "Surveys and Questionnaires", "Treatment Outcome", "United Kingdom" ], "YEAR": "2006", "reasoning_required_pred": "yes", "reasoning_free_pred": "yes", "final_decision": "yes", "LONG_ANSWER": "Day case laparoscopic cholecystectomy is safe, feasible, and cost-effective when patients are carefully selected. It provides good patient satisfaction." }, "23359100": { "QUESTION": "Is etoricoxib effective in preventing heterotopic ossification after primary total hip arthroplasty?", "CONTEXTS": [ "Heterotopic ossification is a common complication after total hip arthroplasty. Non-steroidal anti-inflammatory drugs (NSAIDs) are known to prevent heterotopic ossifications effectively, however gastrointestinal complaints are reported frequently. In this study, we investigated whether etoricoxib, a selective cyclo-oxygenase-2 (COX-2) inhibitor that produces fewer gastrointestinal side effects, is an effective alternative for the prevention of heterotopic ossification.", "We investigated the effectiveness of oral etoricoxib 90 mg for seven days in a prospective two-stage study design for phase-2 clinical trials in a small sample of patients (n\u2009=\u200942). A cemented primary total hip arthroplasty was implanted for osteoarthritis. Six months after surgery, heterotopic ossification was determined on anteroposterior pelvic radiographs using the Brooker classification.", "No heterotopic ossification was found in 62 % of the patients that took etoricoxib; 31 % of the patients had Brooker grade 1 and 7 % Brooker grade 2 ossification." ], "LABELS": [ "PURPOSE", "METHODS", "RESULTS" ], "MESHES": [ "Administration, Oral", "Adult", "Aged", "Aged, 80 and over", "Arthroplasty, Replacement, Hip", "Cyclooxygenase 2 Inhibitors", "Dose-Response Relationship, Drug", "Female", "Hip Joint", "Hip Prosthesis", "Humans", "Male", "Middle Aged", "Ossification, Heterotopic", "Osteoarthritis, Hip", "Prospective Studies", "Pyridines", "Radiography", "Severity of Illness Index", "Sulfones", "Treatment Outcome" ], "YEAR": "2013", "reasoning_required_pred": "yes", "reasoning_free_pred": "yes", "final_decision": "yes", "LONG_ANSWER": "Etoricoxib seems effective in preventing heterotopic ossification after total hip arthroplasty. This finding further supports the use of COX-2 inhibitors for the prevention of heterotopic ossification following total hip arthroplasty." }, "15943725": { "QUESTION": "Should serum pancreatic lipase replace serum amylase as a biomarker of acute pancreatitis?", "CONTEXTS": [ "Serum pancreatic lipase may improve the diagnosis of pancreatitis compared to serum amylase. Both enzymes have been measured simultaneously at our hospital allowing for a comparison of their diagnostic accuracy.", "Seventeen thousand five hundred and thirty-one measurements of either serum amylase and or serum pancreatic lipase were made on 10 931 patients treated at a metropolitan teaching hospital between January 2001 and May 2003. Of these, 8937 were initially treated in the Emergency Department. These results were collected in a database, which was linked by the patients' medical record number to the radiology and medical records. Patients with either an elevated lipase value or a discharge diagnosis of acute pancreatitis had their radiological diagnosis reviewed along with their biochemistry and histology record. The diagnosis of acute pancreatitis was made if there was radiological evidence of peripancreatic inflammation.", "One thousand eight hundred and twenty-five patients had either elevated serum amylase and or serum pancreatic lipase. The medical records coded for pancreatitis in a further 55 whose enzymes were not elevated. Three hundred and twenty of these had radiological evidence of acute pancreatitis. Receiver operator characteristic analysis of the initial sample from patients received in the Emergency Department showed improved diagnostic accuracy for serum pancreatic lipase (area under the curve (AUC) 0.948) compared with serum amylase (AUC, 0.906, P<0.05). A clinically useful cut-off point would be at the diagnostic threshold; 208 U/L (normal<190 U/L) for serum pancreatic lipase and 114 U/L (normal 27-100 U/L) for serum amylase where the sensitivity was 90.3 cf., 76.8% and the specificity was 93 cf., 92.6%. 18.8% of the acute pancreatitis patients did not have elevated serum amylase while only 2.9% did not have elevated serum pancreatic lipase on the first emergency department measurement." ], "LABELS": [ "BACKGROUND", "METHODS", "RESULTS" ], "MESHES": [ "Acute Disease", "Amylases", "Biomarkers", "Humans", "Lipase", "Pancreas", "Pancreatitis", "Radiography", "Sensitivity and Specificity", "Time Factors" ], "YEAR": "2005", "reasoning_required_pred": "yes", "reasoning_free_pred": "yes", "final_decision": "yes", "LONG_ANSWER": "It is concluded that serum pancreatic lipase is a more accurate biomarker of acute pancreatitis than serum amylase." }, "25735444": { "QUESTION": "Rectal cancer threatening or affecting the prostatic plane: is partial prostatectomy oncologically adequate?", "CONTEXTS": [ "A multicentre, retrospective study was conducted of patients with rectal cancer threatening or affecting the prostatic plane, but not the bladder, judged by magnetic resonance imaging (MRI). The use of preoperative chemoradiotherapy and the type of urologic resection were correlated with the status of the pathological circumferential resection margin (CRM) and local recurrence.", "A consecutive series of 126 men with rectal cancer threatening (44) or affecting (82) the prostatic plane on preoperative staging and operated with local curative intent between 1998 and 2010 was analysed. In patients who did not have chemoradiotherapy but had a preoperative threatened anterior margin the CRM-positive rate was 25.0%. In patients who did not have preoperative chemoradiotherapy but did have an affected margin, the CRM-positive rate was 41.7%. When preoperative radiotherapy was given, the respective CRM infiltration rates were 7.1 and 20.7%. In patients having preoperative chemoradiotherapy followed by prostatic resection the rate of CRM positivity was 2.4%. Partial prostatectomy after preoperative chemoradiotherapy resulted in a free anterior CRM in all cases, but intra-operative urethral damage occurred in 36.4% of patients who underwent partial prostatectomy, resulting in a postoperative urinary fistula in 18.2% of patients." ], "LABELS": [ "METHOD", "RESULTS" ], "MESHES": [ "Aged", "Chemoradiotherapy, Adjuvant", "Humans", "Magnetic Resonance Imaging", "Male", "Middle Aged", "Neoadjuvant Therapy", "Neoplasm Invasiveness", "Neoplasm Recurrence, Local", "Neoplasm, Residual", "Prostate", "Prostatectomy", "Radiotherapy, Adjuvant", "Rectal Neoplasms", "Retrospective Studies", "Urethra", "Urinary Fistula" ], "YEAR": "2015", "reasoning_required_pred": "no", "reasoning_free_pred": "maybe", "final_decision": "maybe", "LONG_ANSWER": "Preoperative chemoradiation is mandatory in male patients with a threatened or affected anterior circumferential margin on preoperative MRI. In patients with preoperative prostatic infiltration, prostatic resection is necessary. In this group of patients partial prostatectomy seems to be oncologically safe." }, "9722752": { "QUESTION": "Does bone anchor fixation improve the outcome of percutaneous bladder neck suspension in female stress urinary incontinence?", "CONTEXTS": [ "To evaluate the outcome of a new modification of percutaneous needle suspension, using a bone anchor system for fixing the suture at the public bone, and to compare the results with those published previously.", "From March 1996, 37 patients with stress urinary incontinence (>2 years) were treated using a bone anchor system. On each side the suture was attached to the pubocervical fascia and the vaginal wall via a broad 'Z'-stitch. A urodynamic investigation performed preoperatively in all patients confirmed stress incontinence and excluded detrusor instability. The outcome was assessed by either by a clinical follow-up investigation or using a standardized questionnaire, over a mean follow-up of 11 months (range 6-18).", "In the 37 patients, the procedure was successful in 25 (68%), with 16 (43%) of the patients completely dry and nine (24%) significantly improved. Removal of the bone anchor and suture was necessary in two patients, because of unilateral bacterial infection in one and a bilateral soft tissue granuloma in the other. One bone anchor became dislocated in a third patient. In two cases where the treatment failed, new detrusor instability was documented urodynamically. Minor complications were prolonged wound pain in 10 (26%) and transient urinary retention or residual urine in 12 patients (32%)." ], "LABELS": [ "OBJECTIVE", "PATIENTS AND METHODS", "RESULTS" ], "MESHES": [ "Adult", "Aged", "Aged, 80 and over", "Bone Nails", "Female", "Follow-Up Studies", "Humans", "Middle Aged", "Suture Techniques", "Treatment Outcome", "Urinary Bladder Diseases", "Urinary Incontinence, Stress", "Urodynamics", "Urologic Surgical Procedures" ], "YEAR": "1998", "reasoning_required_pred": "yes", "reasoning_free_pred": "no", "final_decision": "yes", "LONG_ANSWER": "The poor success rate in the study corresponds with the long-term results of conventional or modified needle suspension procedures and does not reinforce the optimistic results of bone anchoring published recently. Because of the poorer long-term results from percutaneous needle suspension than from other techniques of open retropubic bladder neck suspension, it remains questionable whether percutaneous needle suspension should be considered a first-line procedure for the treatment of female stress urinary incontinence." }, "23870157": { "QUESTION": "Are intraoperative precursor events associated with postoperative major adverse events?", "CONTEXTS": [ "Precursor events are undesirable events that can lead to a subsequent adverse event and have been associated with postoperative mortality. The purpose of the present study was to determine whether precursor events are associated with a composite endpoint of major adverse cardiac events (MACE) (death, acute renal failure, stroke, infection) in a low- to medium-risk coronary artery bypass grafting, valve, and valve plus coronary artery bypass grafting population. These events might be targets for strategies aimed at quality improvement.", "The present study was a retrospective cohort design performed at the Queen Elizabeth Health Science Centre. Low- to medium-risk patients who had experienced postoperative MACE were matched 1:1 with patients who had not experienced postoperative MACE. The operative notes, for both groups, were scored by 5 surgeons to determine the frequency of 4 precursor events: bleeding, difficulty weaning from cardiopulmonary bypass, repair or regrafting, and incomplete revascularization or repair. A univariate comparison of \u22651 precursor events in the matched groups was performed.", "A total of 311 MACE patients (98.4%) were matched. The primary outcome occurred more frequently in the MACE group than in the non-MACE group (33% vs 24%; P\u00a0=\u00a0.015). The incidence of the individual events of bleeding and difficulty weaning from cardiopulmonary bypass was significantly higher in the MACE group. Those patients with a precursor event in the absence of MACE also appeared to have a greater prevalence of other important postoperative outcomes." ], "LABELS": [ "OBJECTIVES", "METHODS", "RESULTS" ], "MESHES": [ "Aged", "Blood Loss, Surgical", "Cardiopulmonary Bypass", "Coronary Artery Bypass", "Female", "Heart Valve Prosthesis Implantation", "Humans", "Incidence", "Male", "Middle Aged", "Nova Scotia", "Postoperative Complications", "Prevalence", "Quality Improvement", "Quality Indicators, Health Care", "Registries", "Reoperation", "Retrospective Studies", "Risk Factors", "Time Factors", "Treatment Outcome" ], "YEAR": "2014", "reasoning_required_pred": "yes", "reasoning_free_pred": "yes", "final_decision": "yes", "LONG_ANSWER": "Patients undergoing cardiac surgery who are exposed to intraoperative precursor events were more likely to experience a postoperative MACE. Quality improvement techniques aimed at mitigating the consequences of precursor events might improve the surgical outcomes for cardiac surgical patients." }, "22303473": { "QUESTION": "Does a family meetings intervention prevent depression and anxiety in family caregivers of dementia patients?", "CONTEXTS": [ "Family caregivers of dementia patients are at increased risk of developing depression or anxiety. A multi-component program designed to mobilize support of family networks demonstrated effectiveness in decreasing depressive symptoms in caregivers. However, the impact of an intervention consisting solely of family meetings on depression and anxiety has not yet been evaluated. This study examines the preventive effects of family meetings for primary caregivers of community-dwelling dementia patients.", "A randomized multicenter trial was conducted among 192 primary caregivers of community dwelling dementia patients. Caregivers did not meet the diagnostic criteria for depressive or anxiety disorder at baseline. Participants were randomized to the family meetings intervention (n\u200a=\u200a96) or usual care (n\u200a=\u200a96) condition. The intervention consisted of two individual sessions and four family meetings which occurred once every 2 to 3 months for a year. Outcome measures after 12 months were the incidence of a clinical depressive or anxiety disorder and change in depressive and anxiety symptoms (primary outcomes), caregiver burden and quality of life (secondary outcomes). Intention-to-treat as well as per protocol analyses were performed.", "A substantial number of caregivers (72/192) developed a depressive or anxiety disorder within 12 months. The intervention was not superior to usual care either in reducing the risk of disorder onset (adjusted IRR 0.98; 95% CI 0.69 to 1.38) or in reducing depressive (randomization-by-time interaction coefficient\u200a=\u200a-1.40; 95% CI -3.91 to 1.10) or anxiety symptoms (randomization-by-time interaction coefficient\u200a=\u200a-0.55; 95% CI -1.59 to 0.49). The intervention did not reduce caregiver burden or their health related quality of life." ], "LABELS": [ "BACKGROUND", "METHODS", "RESULTS" ], "MESHES": [ "Aged", "Anxiety", "Caregivers", "Delivery of Health Care", "Dementia", "Demography", "Depression", "Female", "Follow-Up Studies", "Humans", "Longitudinal Studies", "Male", "Social Support", "Treatment Outcome" ], "YEAR": "2012", "reasoning_required_pred": "no", "reasoning_free_pred": "no", "final_decision": "no", "LONG_ANSWER": "This study did not demonstrate preventive effects of family meetings on the mental health of family caregivers. Further research should determine whether this intervention might be more beneficial if provided in a more concentrated dose, when applied for therapeutic purposes or targeted towards subgroups of caregivers." }, "24995509": { "QUESTION": "HIF1A as a major vascular endothelial growth factor regulator: do its polymorphisms have an association with age-related macular degeneration?", "CONTEXTS": [ "To investigate the association between age-related macular degeneration (AMD) and the polymorphisms of HIF1A, a major vascular epithelial growth factor regulator under hypoxic conditions. The associations of AMD and polymorphisms of genes CFH, SKIV2L and MYRIP were also studied.", "Prospective study.", "Eighty-seven AMD patients and 80 healthy subjects admitted to the Department of Ophthalmology at Pamukkale University Hospital, Denizli, Turkey, were included: 45 (52%) had wet type AMD, and 42 (48%) had dry type AMD.", "Polymorphisms rs1061170 (CFH), rs429608 (SKIV2L), rs2679798 (MYRIP) and both rs11549465 and rs11549467 (HIF1A) were investigated in DNA isolated from peripheral blood samples of the cases and controls by dye-termination DNA sequencing.", "Genotype distribution of rs1061170 (CFH), rs429608 (SKIV2L), rs2679798 (MYRIP) and both rs11549465 and rs11549467 (HIF1A) in AMD cases and healthy controls; association between genotypes and AMD subtypes.", "Given the significant difference between the mean age of case and control groups (72.13\u2009\u00b1\u20095.77 vs. 62.80\u2009\u00b1\u20095.22, respectively) (P\u2009=\u2009.000), subsequent analyses were adjusted for age. We found that having at least one C allele for polymorphism rs1061170 increases AMD risk independent of age (OR\u2009=\u20092.42, 95% confidence interval [CI], 1.22-4.81). The ancestral T allele for polymorphism rs1061170 has a protective effect for AMD (OR\u2009=\u20090.53, 95% CI, 0.34-0.83). No statistically significant difference for distributions of other single nucleotide polymorphisms (SNPs) emerged between patients and healthy subjects." ], "LABELS": [ "BACKGROUND", "DESIGN", "PARTICIPANTS", "METHODS", "MAIN OUTCOME MEASURES", "RESULTS" ], "MESHES": [ "Aged", "Complement Factor H", "DNA Helicases", "Female", "Gene Frequency", "Genotype", "Geographic Atrophy", "Humans", "Hypoxia-Inducible Factor 1, alpha Subunit", "Male", "Middle Aged", "Polymerase Chain Reaction", "Polymorphism, Single Nucleotide", "Prospective Studies", "Sequence Analysis, DNA", "Vascular Endothelial Growth Factor A", "Vesicular Transport Proteins", "Wet Macular Degeneration" ], "YEAR": null, "reasoning_required_pred": "yes", "reasoning_free_pred": "maybe", "final_decision": "maybe", "LONG_ANSWER": "No associations appeared between HIF1A SNPs and AMD, which were studied here for the first time; however, polymorphism rs1061170 of the CFH gene is associated with AMD in our population." }, "24270957": { "QUESTION": "Is combined therapy more effective than growth hormone or hyperbaric oxygen alone in the healing of left ischemic and non-ischemic colonic anastomoses?", "CONTEXTS": [ "Our aim was to investigate the effects of growth hormone (GH), hyperbaric oxygen and combined therapy on normal and ischemic colonic anastomoses in rats.", "Eighty male Wistar rats were divided into eight groups (n\u200a=\u200a10). In the first four groups, non-ischemic colonic anastomosis was performed, whereas in the remaining four groups, ischemic colonic anastomosis was performed. In groups 5, 6, 7, and 8, colonic ischemia was established by ligating 2 cm of the mesocolon on either side of the anastomosis. The control groups (1 and 5) received no treatment. Hyperbaric oxygen therapy was initiated immediately after surgery and continued for 4 days in groups 3 and 4. Groups 2 and 6 received recombinant human growth hormone, whereas groups 4 and 8 received GH and hyperbaric oxygen treatment. Relaparotomy was performed on postoperative day 4, and a perianastomotic colon segment 2 cm in length was excised for the detection of biochemical and mechanical parameters of anastomotic healing and histopathological evaluation.", "Combined treatment with hyperbaric oxygen and GH increased the mean bursting pressure values in all of the groups, and a statistically significant increase was noted in the ischemic groups compared to the controls (p<0.05). This improvement was more evident in the ischemic and normal groups treated with combined therapy. In addition, a histopathological evaluation of anastomotic neovascularization and collagen deposition showed significant differences among the groups." ], "LABELS": [ "OBJECTIVE", "METHODS", "RESULTS" ], "MESHES": [ "Anastomosis, Surgical", "Animals", "Collagen", "Colon", "Combined Modality Therapy", "Disease Models, Animal", "Human Growth Hormone", "Hyperbaric Oxygenation", "Ischemia", "Male", "Necrosis", "Neovascularization, Physiologic", "Pressure", "Rats", "Rats, Wistar", "Reproducibility of Results", "Treatment Outcome", "Wound Healing" ], "YEAR": "2013", "reasoning_required_pred": "yes", "reasoning_free_pred": "yes", "final_decision": "yes", "LONG_ANSWER": "Combined treatment with recombinant human growth hormone and hyperbaric oxygen resulted in a favorable therapeutic effect on the healing of ischemic colonic anastomoses." }, "23848044": { "QUESTION": "Does oxybutynin hydrochloride cause arrhythmia in children with bladder dysfunction?", "CONTEXTS": [ "This study represents a subset of a complete data set, considering only those children aged admitted to the Pediatric Surgery and Pediatric Nephrology Clinics during the period January 2011 to July 2012.", "In this study, we have determined that the QT interval changes significantly depending on the use of oxybutynin. The QT changes increased cardiac arrhythmia in children." ], "LABELS": [ "METHOD", "RESULT" ], "MESHES": [ "Adolescent", "Arrhythmias, Cardiac", "Child", "Child, Preschool", "Electrocardiography", "Female", "Humans", "Male", "Mandelic Acids", "Muscarinic Antagonists", "Retrospective Studies", "Urinary Bladder, Overactive" ], "YEAR": "2013", "reasoning_required_pred": "yes", "reasoning_free_pred": "yes", "final_decision": "yes", "LONG_ANSWER": "For this reason, children using such drugs should be closely monitored for cardiac arrhythmia." }, "9920954": { "QUESTION": "Do \"America's Best Hospitals\" perform better for acute myocardial infarction?", "CONTEXTS": [ "\"America's Best Hospitals,\" an influential list published annually by U.S. News and World Report, assesses the quality of hospitals. It is not known whether patients admitted to hospitals ranked at the top in cardiology have lower short-term mortality from acute myocardial infarction than those admitted to other hospitals or whether differences in mortality are explained by differential use of recommended therapies.", "Using data from the Cooperative Cardiovascular Project on 149,177 elderly Medicare beneficiaries with acute myocardial infarction in 1994 or 1995, we examined the care and outcomes of patients admitted to three types of hospitals: those ranked high in cardiology (top-ranked hospitals); hospitals not in the top rank that had on-site facilities for cardiac catheterization, coronary angioplasty, and bypass surgery (similarly equipped hospitals); and the remaining hospitals (non-similarly equipped hospitals). We compared 30-day mortality; the rates of use of aspirin, beta-blockers, and reperfusion; and the relation of differences in rates of therapy to short-term mortality.", "Admission to a top-ranked hospital was associated with lower adjusted 30-day mortality (odds ratio, 0.87; 95 percent confidence interval, 0.76 to 1.00; P=0.05 for top-ranked hospitals vs. the others). Among patients without contraindications to therapy, top-ranked hospitals had significantly higher rates of use of aspirin (96.2 percent, as compared with 88.6 percent for similarly equipped hospitals and 83.4 percent for non-similarly equipped hospitals; P<0.01) and beta-blockers (75.0 percent vs. 61.8 percent and 58.7 percent, P<0.01), but lower rates of reperfusion therapy (61.0 percent vs. 70.7 percent and 65.6 percent, P=0.03). The survival advantage associated with admission to top-ranked hospitals was less strong after we adjusted for factors including the use of aspirin and beta-blockers (odds ratio, 0.94; 95 percent confidence interval, 0.82 to 1.08; P=0.38)." ], "LABELS": [ "BACKGROUND", "METHODS", "RESULTS" ], "MESHES": [ "Adrenergic beta-Antagonists", "Aged", "Angioplasty, Balloon, Coronary", "Aspirin", "Female", "Health Care Surveys", "Hospitals", "Humans", "Logistic Models", "Male", "Medicare", "Multivariate Analysis", "Myocardial Infarction", "Outcome and Process Assessment (Health Care)", "Quality of Health Care", "Severity of Illness Index", "Thrombolytic Therapy", "United States" ], "YEAR": "1999", "reasoning_required_pred": "no", "reasoning_free_pred": "yes", "final_decision": "yes", "LONG_ANSWER": "Admission to a hospital ranked high on the list of \"America's Best Hospitals\" was associated with lower 30-day mortality among elderly patients with acute myocardial infarction. A substantial portion of the survival advantage may be associated with these hospitals' higher rates of use of aspirin and beta-blocker therapy." }, "21276532": { "QUESTION": "Does preoperative anemia adversely affect colon and rectal surgery outcomes?", "CONTEXTS": [ "Complications associated with blood transfusions have resulted in widespread acceptance of low hematocrit levels in surgical patients. However, preoperative anemia seems to be a risk factor for adverse postoperative outcomes in certain surgical patients. This study investigated the National Surgical Quality Improvement Program (NSQIP) database to determine if preoperative anemia in patients undergoing open and laparoscopic colectomies is an independent predictor for an adverse composite outcome (CO) consisting of myocardial infarction, stroke, progressive renal insufficiency or death within 30 days of operation, or for an increased hospital length of stay (LOS).", "Hematocrit levels were categorized into 4 classes: severe, moderate, mild, and no anemia. From 2005 to 2008, the NSQIP database recorded 23,348 elective open and laparoscopic colectomies that met inclusion criteria. Analyses using multivariable models, controlling for potential confounders and stratifying on propensity score, were performed.", "Compared with nonanemic patients, those with severe, moderate, and mild anemia were more likely to have the adverse CO with odds ratios of 1.83 (95% CI 1.05 to 3.19), 2.19 (95 % CI 1.63 to 2.94), and 1.49 (95% CI 1.20 to 1.86), respectively. Patients with a normal hematocrit had a reduced hospital LOS, compared with those with severe, moderate, and mild anemia (p<0.01). A history of cardiovascular disease did not significantly influence these findings." ], "LABELS": [ "BACKGROUND", "STUDY DESIGN", "RESULTS" ], "MESHES": [ "Adult", "Aged", "Aged, 80 and over", "Anemia, Hypochromic", "Colectomy", "Comorbidity", "Female", "Hematocrit", "Humans", "Laparoscopy", "Length of Stay", "Logistic Models", "Male", "Middle Aged", "Multivariate Analysis", "Myocardial Infarction", "Odds Ratio", "Perioperative Period", "Postoperative Complications", "Renal Insufficiency", "Risk Factors", "Severity of Illness Index", "Stroke", "Transfusion Reaction", "Treatment Outcome" ], "YEAR": "2011", "reasoning_required_pred": "yes", "reasoning_free_pred": "yes", "final_decision": "yes", "LONG_ANSWER": "This large multicenter database analysis suggests that the presence of severe and moderate and even mild preoperative anemia is an independent risk factor for complications and a longer hospital stay after colon surgery." }, "23379759": { "QUESTION": "Can early second-look tympanoplasty reduce the rate of conversion to modified radical mastoidectomy?", "CONTEXTS": [ "The aims of the study were to report the rates of recurrent and residual cholesteatoma following primary CAT surgery and to report the rate of conversion to a modified radical mastoidectomy.", "This was a retrospective review of a single surgeon series between 2006 and 2012.", "In total 132 second-look operations were undertaken, with a mean interval between primary surgery and second-look procedures of 6 months. The rate of cholesteatoma at second-look surgery was 19.7%, which was split into residual disease (10.6%) and recurrent disease (9.09%). New tympanic membrane defects with cholesteatoma were considered as recurrent disease. Residual disease was defined as cholesteatoma present behind an intact tympanic membrane. The majority of recurrent and residual disease was easily removed at second look (73.1%). Only four cases were converted to a modified radical mastoidectomy (3%) and three cases required a third-look procedure." ], "LABELS": [ "OBJECTIVES", "METHODS", "RESULTS" ], "MESHES": [ "Adolescent", "Adult", "Child", "Child, Preschool", "Cholesteatoma, Middle Ear", "Humans", "Middle Aged", "Recurrence", "Retrospective Studies", "Second-Look Surgery", "Treatment Outcome", "Tympanoplasty", "Young Adult" ], "YEAR": "2013", "reasoning_required_pred": "yes", "reasoning_free_pred": "yes", "final_decision": "yes", "LONG_ANSWER": "Combined approach tympanoplasty (CAT) allows for successful treatment of cholesteatoma with rates of recurrent and residual disease comparable to open mastoid surgery. Early timing of second-look procedures allows easier removal of any recurrent or residual disease, which reduces the conversion rate to open mastoidectomy." }, "25891436": { "QUESTION": "Is serum total bilirubin useful to differentiate cardioembolic stroke from other stroke subtypes?", "CONTEXTS": [ "Previous studies have reported that the total bilirubin (TB) level is associated with coronary artery disease, heart failure and atrial fibrillation. These heart diseases can produce cardiogenic cerebral embolism and cause cardioembolic stroke. However, whether the serum TB could be a biomarker to differentiate cardioembolic stroke from other stroke subtypes is unclear.", "Our study consisted of 628 consecutive patients with ischaemic stroke. Various clinical and laboratory variables of the patients were analysed according to serum TB quartiles and stroke subtypes.", "The higher TB quartile group was associated with atrial fibrillation, larger left atrium diameter, lower left ventricular fractional shortening and cardioembolic stroke (P<0.001, P = 0.001, P = 0.033, P<0.001, respectively). Furthermore, serum TB was a statistically significant independent predictor of cardioembolic stroke in a multivariable setting (Continuous, per unit increase OR = 1.091, 95%CI: 1.023-1.164, P = 0.008)." ], "LABELS": [ "BACKGROUND", "METHODS", "RESULTS" ], "MESHES": [ "Adult", "Aged", "Aged, 80 and over", "Atrial Fibrillation", "Bilirubin", "Biomarkers", "Brain Ischemia", "Diagnosis, Differential", "Feasibility Studies", "Female", "Humans", "Intracranial Embolism", "Male", "Middle Aged", "Stroke" ], "YEAR": "2015", "reasoning_required_pred": "yes", "reasoning_free_pred": "yes", "final_decision": "yes", "LONG_ANSWER": "Serum TB level was independently associated with cardioembolic stroke. The combination of clinical data and serum TB may be a feasible strategy to diagnose cardioembolic stroke in the acute phase." }, "24450673": { "QUESTION": "Delayed imaging in routine CT examinations of the abdomen and pelvis: is it worth the additional cost of radiation and time?", "CONTEXTS": [ "The purpose of this study was to retrospectively assess the potential benefits of delayed phase imaging series in routine CT scans of the abdomen and pelvis.", "Routine contrast-enhanced abdominopelvic CT scans of 1000 consecutively examined patients (912 men, 88 women; average age, 60 years; range, 22-94 years) were retrospectively evaluated, and the added benefits of the delayed phase series through the abdomen were recorded for each examination. Examinations performed for indications requiring multiphasic imaging were excluded. Images were reviewed by two fellowship-trained abdominal radiologists, who were blinded to official CT reports. All examinations were performed between July 2008 and February 2010 at a single institution. Radiation doses for both the portal venous and delayed phases, when available, were analyzed to assess the effect of the delayed phase on overall radiation exposure.", "Forty-two patients (4.2%) had findings that were further characterized or were observed only in the delayed phase. Most were incidental findings that could have been confirmed at noninvasive follow-up imaging, such as sonography or unenhanced CT or MRI. The most common findings were liver hemangioma (n = 12), adrenal adenoma (n = 12), and parapelvic renal cysts (n = 6). The most important finding was detection of a renal mass in one patient (0.1%). The mass was seen only on the delayed phase images but was difficult to appreciate in the portal venous phase. In the other 958 patients (95.8%), delayed imaging was of no benefit. In addition, use of the delayed phase resulted in a mean 59.5% increase in effective radiation dose." ], "LABELS": [ "OBJECTIVE", "MATERIALS AND METHODS", "RESULTS" ], "MESHES": [ "Adult", "Aged", "Aged, 80 and over", "Contrast Media", "Female", "Humans", "Incidental Findings", "Iohexol", "Male", "Middle Aged", "Pelvis", "Radiation Dosage", "Radiography, Abdominal", "Retrospective Studies", "Time Factors", "Tomography, X-Ray Computed", "Triiodobenzoic Acids" ], "YEAR": "2014", "reasoning_required_pred": "no", "reasoning_free_pred": "no", "final_decision": "no", "LONG_ANSWER": "An additional delayed phase through the abdomen in routine contrast-enhanced CT examinations of the abdomen and pelvis is of low yield, particularly if reliable follow-up imaging to further elucidate uncertain findings is available." }, "17914515": { "QUESTION": "Suturing of the nasal septum after septoplasty, is it an effective alternative to nasal packing?", "CONTEXTS": [ "To discuss and compare the results of suturing the nasal septum after septoplasty with the results of nasal packing.", "A prospective study, which was performed at Prince Hashem Military Hospital in Zarqa, Jordan and Prince Rashed Military Hospital in Irbid, Jordan between September 2005 and August 2006 included 169 consecutive patients that underwent septoplasty. The patients were randomly divided into 2 groups. After completion of surgery, the nasal septum was sutured in the first group while nasal packing was performed in the second group.", "Thirteen patients (15.3%) in the first group and 11 patients (13%) in the second group had minor oozing in the first 24 hours, 4 patients (4.8%) had bleeding after removal of the pack in the second group. Four patients (4.8%) developed septal hematoma in the second group. Two patients (2.4%) had septal perforation in the second group. One patient (1.1%) in the first group, and 5 patients (5.9%) in the second group had postoperative adhesions. Five patients (5.9%) were found to have remnant deviated nasal septum in each group. The operating time was 4 minutes longer in the first group." ], "LABELS": [ "OBJECTIVE", "METHODS", "RESULTS" ], "MESHES": [ "Adolescent", "Adult", "Bandages", "Female", "Humans", "Male", "Middle Aged", "Nasal Septum", "Postoperative Care", "Prospective Studies", "Suture Techniques", "Treatment Outcome" ], "YEAR": "2007", "reasoning_required_pred": "yes", "reasoning_free_pred": "yes", "final_decision": "yes", "LONG_ANSWER": "Septal suturing after septoplasty offers the following advantages: elimination of discomfort for the patients, minimal complications, the outcome is almost the same as with nasal packing, and finally the hospital stay is less than with nasal packing. Therefore, suturing of the nasal septum after septoplasty should be a preferred alternative to nasal packing." }, "24739448": { "QUESTION": "Have antiepileptic drug prescription claims changed following the FDA suicidality warning?", "CONTEXTS": [ "In January 2008, the Food and Drug Administration (FDA) communicated concerns and, in May 2009, issued a warning about an increased risk of suicidality for all antiepileptic drugs (AEDs). This research evaluated the association between the FDA suicidality communications and the AED prescription claims among members with epilepsy and/or psychiatric disorder.", "A longitudinal interrupted time-series design was utilized to evaluate Oklahoma Medicaid claims data from January 2006 through December 2009. The study included 9289 continuously eligible members with prevalent diagnoses of epilepsy and/or psychiatric disorder and at least one AED prescription claim. Trends, expressed as monthly changes in the log odds of AED prescription claims, were compared across three time periods: before (January 2006 to January 2008), during (February 2008 to May 2009), and after (June 2009 to December 2009) the FDA warning.", "Before the FDA warning period, a significant upward trend of AED prescription claims of 0.01% per month (99% CI: 0.008% to 0.013%, p<0.0001) was estimated. In comparison to the prewarning period, no significant change in trend was detected during (-20.0%, 99% CI: -70.0% to 30.0%, p=0.34) or after (80.0%, 99% CI: -20.0% to 200.0%, p=0.03) the FDA warning period. After stratification, no diagnostic group (i.e., epilepsy alone, epilepsy and comorbid psychiatric disorder, and psychiatric disorder alone) experienced a significant change in trend during the entire study period (p>0.01)." ], "LABELS": [ "OBJECTIVE", "METHODS", "RESULTS" ], "MESHES": [ "Adolescent", "Adult", "Anticonvulsants", "Child", "Child, Preschool", "Drug Prescriptions", "Epilepsy", "Humans", "Infant", "Interrupted Time Series Analysis", "Medicaid", "Middle Aged", "Suicide", "United States", "United States Food and Drug Administration", "Young Adult" ], "YEAR": "2014", "reasoning_required_pred": "no", "reasoning_free_pred": "no", "final_decision": "no", "LONG_ANSWER": "During the time period considered, the FDA AED-related suicidality warning does not appear to have significantly affected prescription claims of AED medications for the study population." }, "19444061": { "QUESTION": "Does ossification of the posterior longitudinal ligament affect the neurological outcome after traumatic cervical cord injury?", "CONTEXTS": [ "Retrospective outcome measurement study.", "The purpose of this study is to assess whether ossification of the posterior longitudinal ligament (OPLL) affects neurologic outcomes in patients with acute cervical spinal cord injury (SCI).", "There have so far been few reports examining the relationship between OPLL and SCI and there is controversy regarding the deteriorating effects of OPLL-induced canal stenosis on neurologic outcomes.", "To obtain a relatively uniform background, patients nonsurgically treated for an acute C3-C4 level SCI without any fractures or dislocations of the spinal column were selected, resulting in 129 patients. There were 110 men and 19 women (mean age was 61.1 years), having various neurologic conditions on admission (American Spinal Injury Association [ASIA] impairment scale A, 43; B, 16; C, 58; D, 12). The follow-up period was the duration of their hospital stay and ranged from 50 to 603 days (mean, 233 days). The presence of OPLL, the cause of injury, the degree of canal stenosis (both static and dynamic), and the neurologic outcomes in motor function, including improvement rate, were assessed.", "Of the 129 patients investigated in this study, OPLL was identified at the site of the injury in 13 patients (10.1%). In this OPLL+ group, the static and dynamic canal diameters at C3 and C4 were significantly smaller than those of the remaining 116 patients (OPLL- group). However, no significant difference was observed between the 2 groups in terms of ASIA motor score both at the time of administration and discharge, and the mean improvement rate in ASIA motor score was 55.5 +/- 9.0% in OPLL+ group, while it was 43.1 +/- 2.8% in the OPLL-group. Furthermore, no significant correlation was observed between the static/dynamic canal diameters and neurologic outcome in all 129 patients." ], "LABELS": [ "STUDY DESIGN", "OBJECTIVES", "SUMMARY OF BACKGROUND DATA", "METHODS", "RESULTS" ], "MESHES": [ "Adult", "Aged", "Aged, 80 and over", "Cervical Vertebrae", "Female", "Humans", "Magnetic Resonance Imaging", "Male", "Middle Aged", "Ossification of Posterior Longitudinal Ligament", "Range of Motion, Articular", "Retrospective Studies", "Spinal Cord Injuries", "Tomography, X-Ray Computed" ], "YEAR": "2009", "reasoning_required_pred": "no", "reasoning_free_pred": "no", "final_decision": "no", "LONG_ANSWER": "No evidence was found for OPLL to have any effect on the initial neurologic status or recovery in motor function after traumatic cervical cord injury, suggesting that the neurologic outcome is not significantly dependent on canal space." }, "25669733": { "QUESTION": "Can distal ureteral diameter predict reflux resolution after endoscopic injection?", "CONTEXTS": [ "To test the predictive value of distal ureteral diameter (UD) on reflux resolution after endoscopic injection in children with primary vesicoureteral reflux (VUR).", "This was a retrospective review of patients diagnosed with primary VUR between 2009 and 2012 who were managed by endoscopic injection. Seventy preoperative and postoperative voiding cystourethrograms were reviewed. The largest UD within the false pelvis was measured. The UD was divided by the L1-L3 vertebral body distance to get the UD ratio (UDR). One radiologist interpreted the findings of voiding cystourethrography in all patients. Clinical outcome was defined as reflux resolution.", "Seventy patients were enrolled in this series (17 boys and 53 girls). Mean age was 5.9 years (1.2-13 years). Grade III presented in 37 patients (53%), and 33 patients (47%) were of grade IV. Mean distal UD was 5.5\u00a0mm (2.5-13\u00a0mm). Mean UDR was 37.8% (18%-70%). Macroplastique injection was performed in all. Subureteric injection was performed in 60 patients (86%), whereas intraureteric injection was performed in 10 patients. No postoperative complications were detected. The effect of grade, UD, and UDR on success after endoscopic injection was tested. UD and UDR were significant predictors of reflux resolution on logistic regression analysis (P\u00a0<.007 and .001, respectively)." ], "LABELS": [ "OBJECTIVE", "MATERIALS AND METHODS", "RESULTS" ], "MESHES": [ "Adolescent", "Child", "Child, Preschool", "Dimethylpolysiloxanes", "Female", "Humans", "Infant", "Lumbar Vertebrae", "Male", "Predictive Value of Tests", "Radiography", "Retrospective Studies", "Ureter", "Ureteroscopy", "Urological Agents", "Vesico-Ureteral Reflux" ], "YEAR": "2015", "reasoning_required_pred": "yes", "reasoning_free_pred": "yes", "final_decision": "yes", "LONG_ANSWER": "UDR provides an objective measurement of VUR and appears as a predictive tool of success after endoscopic injection." }, "20577124": { "QUESTION": "Is leptin involved in phagocytic NADPH oxidase overactivity in obesity?", "CONTEXTS": [ "Hyperleptinemia and oxidative stress play a major role in the development of cardiovascular diseases in obesity. This study aimed to investigate whether there is a relationship between plasma levels of leptin and phagocytic nicotinamide adenine dinucleotide phosphate (NADPH) oxidase activity, and its potential relevance in the vascular remodeling in obese patients.", "The study was performed in 164 obese and 94 normal-weight individuals (controls). NADPH oxidase activity was evaluated by luminescence in phagocytic cells. Levels of leptin were quantified by ELISA in plasma samples. Carotid intima-media thickness (cIMT) was measured by ultrasonography. In addition, we performed in-vitro experiments in human peripheral blood mononuclear cells and murine macrophages.", "Phagocytic NADPH oxidase activity and leptin levels were enhanced (P<0.05) in obese patients compared with controls. NADPH oxidase activity positively correlated with leptin in obese patients. This association remained significant in a multivariate analysis. cIMT was higher (P<0.05) in obese patients compared with controls. In addition, cIMT also correlated positively with leptin and NADPH oxidase activity in obese patients. In-vitro studies showed that leptin induced NADPH oxidase activation. Inhibition of the leptin-induced NADPH oxidase activity by wortmannin and bisindolyl maleimide suggested a direct involvement of the phosphatidylinositol 3-kinase and protein kinase C pathways, respectively. Finally, leptin-induced NADPH oxidase activation promoted macrophage proliferation." ], "LABELS": [ "OBJECTIVES", "METHODS", "RESULTS" ], "MESHES": [ "Animals", "Atherosclerosis", "Carotid Arteries", "Case-Control Studies", "Cell Line", "Cell Proliferation", "Female", "Humans", "In Vitro Techniques", "Leptin", "Macrophages", "Male", "Mice", "Middle Aged", "NADPH Oxidases", "Obesity", "Oxidative Stress", "Phagocytes", "Superoxides", "Tunica Intima" ], "YEAR": "2010", "reasoning_required_pred": "yes", "reasoning_free_pred": "yes", "final_decision": "yes", "LONG_ANSWER": "These findings show that phagocytic NADPH oxidase activity is increased in obesity and is related to preclinical atherosclerosis in this condition. We also suggest that hyperleptinemia may contribute to phagocytic NADPH oxidase overactivity in obesity." }, "21592383": { "QUESTION": "Canada's Compassionate Care Benefit: is it an adequate public health response to addressing the issue of caregiver burden in end-of-life care?", "CONTEXTS": [ "An increasingly significant public health issue in Canada, and elsewhere throughout the developed world, pertains to the provision of adequate palliative/end-of-life (P/EOL) care. Informal caregivers who take on the responsibility of providing P/EOL care often experience negative physical, mental, emotional, social and economic consequences. In this article, we specifically examine how Canada's Compassionate Care Benefit (CCB)--a contributory benefits social program aimed at informal P/EOL caregivers--operates as a public health response in sustaining informal caregivers providing P/EOL care, and whether or not it adequately addresses known aspects of caregiver burden that are addressed within the population health promotion (PHP) model.", "As part of a national evaluation of Canada's Compassionate Care Benefit, 57 telephone interviews were conducted with Canadian informal P/EOL caregivers in 5 different provinces, pertaining to the strengths and weaknesses of the CCB and the general caregiving experience. Interview data was coded with Nvivo software and emerging themes were identified by the research team, with such findings published elsewhere. The purpose of the present analysis was identified after comparing the findings to the literature specific to caregiver burden and public health, after which data was analyzed using the PHP model as a guiding framework.", "Informal caregivers spoke to several of the determinants of health outlined in the PHP model that are implicated in their burden experience: gender, income and social status, working conditions, health and social services, social support network, and personal health practises and coping strategies. They recognized the need for improving the CCB to better address these determinants." ], "LABELS": [ "BACKGROUND", "METHODS", "RESULTS" ], "MESHES": [ "Aged", "Canada", "Caregivers", "Cost of Illness", "Female", "Humans", "Interviews as Topic", "Male", "Middle Aged", "Program Evaluation", "Public Health", "Terminal Care" ], "YEAR": "2011", "reasoning_required_pred": "no", "reasoning_free_pred": "no", "final_decision": "no", "LONG_ANSWER": "This study, from the perspective of family caregivers, demonstrates that the CCB is not living up to its full potential in sustaining informal P/EOL caregivers. Effort is required to transform the CCB so that it may fulfill the potential it holds for serving as one public health response to caregiver burden that forms part of a healthy public policy that addresses the determinants of this burden." }, "11053064": { "QUESTION": "Fatigue in primary Sj\u00f6gren's syndrome: is there a link with the fibromyalgia syndrome?", "CONTEXTS": [ "To determine whether fibromyalgia (FM) is more common in patients with primary Sj\u00f6gren's syndrome (pSS) who complain of fatigue. The association and prevalence of fatigue and FM was recorded in a group of patients with pSS and a control group of lupus patients, a subset of whom had secondary Sj\u00f6gren's syndrome (sSS).", "74 patients with pSS and 216 patients with lupus were assessed with a questionnaire to identify the presence of fatigue and generalised pain. From the lupus group, in a subset of 117 lupus patients (from the Bloomsbury unit) those with sSS were identified. All patients were studied for the presence of FM.", "50 of 74 patients with pSS (68%) reported fatigue-a prevalence significantly higher than in the lupus group (108/216 (50%); p<0.0087). Fatigue was present in 7/13 (54%) patients with SLE/sSS. FM was present in 9/74 patients with pSS (12%), compared with 11/216 lupus patients (5%), and in none of the patients with SLE/sSS. None of these values corresponds with previously reported figures of the incidence of FM in pSS." ], "LABELS": [ "OBJECTIVE", "METHODS", "RESULTS" ], "MESHES": [ "Adult", "Aged", "Aged, 80 and over", "Case-Control Studies", "Fatigue", "Female", "Fibromyalgia", "Humans", "Lupus Erythematosus, Systemic", "Male", "Middle Aged", "Prevalence", "Sjogren's Syndrome", "United Kingdom" ], "YEAR": "2000", "reasoning_required_pred": "yes", "reasoning_free_pred": "no", "final_decision": "no", "LONG_ANSWER": "The results show that fatigue in patients with pSS and sSS is not due to the coexistence of FM in most cases. A lower incidence in the United Kingdom of FM in patients with pSS was found than has been previously reported." }, "9745063": { "QUESTION": "Is laparoscopic sonography a reliable and sensitive procedure for staging colorectal cancer?", "CONTEXTS": [ "Laparoscopic colectomy has developed rapidly with the explosion of technology. In most cases, laparoscopic resection is performed for colorectal cancer. Intraoperative staging during laparoscopic procedure is limited. Laparoscopic ultrasonography (LUS) represents the only real alternative to manual palpation during laparoscopic surgery.", "We evaluated the diagnostic accuracy of LUS in comparison with preoperative staging and laparoscopy in 33 patients with colorectal cancer. Preoperative staging included abdominal US, CT, and endoscopic US (for rectal cancer). Laparoscopy and LUS were performed in all cases. Pre- and intraoperative staging were related to definitive histology. Staging was done according to the TNM classification.", "LUS obtained good results in the evaluation of hepatic metastases, with a sensitivity of 100% versus 62.5% and 75% by preoperative diagnostic means and laparoscopy, respectively. Nodal metastases were diagnosed with a sensitivity of 94% versus 18% with preoperative staging and 6% with laparoscopy, but the method had a low specificity (53%). The therapeutic program was changed thanks to laparoscopy and LUS in 11 cases (33%). In four cases (12%), the planned therapeutic approach was changed after LUS alone." ], "LABELS": [ "BACKGROUND", "METHODS", "RESULTS" ], "MESHES": [ "Biopsy", "Colorectal Neoplasms", "Endosonography", "False Negative Reactions", "False Positive Reactions", "Female", "Humans", "Laparoscopy", "Liver Neoplasms", "Male", "Monitoring, Intraoperative", "Neoplasm Staging", "Preoperative Care", "Sensitivity and Specificity" ], "YEAR": "1998", "reasoning_required_pred": "maybe", "reasoning_free_pred": "yes", "final_decision": "yes", "LONG_ANSWER": "The results obtained in this study demonstrate that LUS is an accurate and highly sensitive procedure in staging colorectal cancer, providing a useful and reliable diagnostic tool complementary to laparoscopy." }, "11138995": { "QUESTION": "Is alexithymia a risk factor for unexplained physical symptoms in general medical outpatients?", "CONTEXTS": [ "Alexithymia is presumed to play an important predisposing role in the pathogenesis of medically unexplained physical symptoms. However, no research on alexithymia has been done among general medical outpatients who present with medically unexplained physical symptoms as their main problem and in which anxiety and depression have been considered as possible confounding factors. This study investigated whether patients with medically unexplained physical symptoms are more alexithymic than those with explained symptoms and whether, in patients with unexplained symptoms, alexithymia is associated with subjective health experience and use of medical services.", "We conducted a cross-sectional study among patients attending an internal medicine outpatient clinic. All patients were given a standardized interview and completed a number of questionnaires.", "After complete physical examinations, 169 of 321 patients had unexplained physical symptoms according to two independent raters. Patients with medically unexplained symptoms more often had a mental disorder, but overall they were not more alexithymic. In patients with unexplained physical symptoms, alexithymia was not associated with subjective health experience or use of medical services. However, patients with both unexplained symptoms and a mental disorder who also denied any possible connection between emotional problems and their physical symptoms did have more alexithymic traits." ], "LABELS": [ "OBJECTIVE", "METHODS", "RESULTS" ], "MESHES": [ "Adult", "Affective Symptoms", "Ambulatory Care", "Anxiety", "Depression", "Dissociative Disorders", "Female", "Humans", "Internal Medicine", "Male", "Middle Aged", "Netherlands", "Patient Acceptance of Health Care", "Risk Factors", "Sick Role", "Somatoform Disorders" ], "YEAR": null, "reasoning_required_pred": "yes", "reasoning_free_pred": "no", "final_decision": "no", "LONG_ANSWER": "In the majority of patients with medically unexplained physical symptoms, alexithymia does not play a role of clinical significance. Patients with unexplained physical symptoms are heterogeneous with respect to psychiatric syndrome pathology and probably also with respect to personality pathology." }, "15475728": { "QUESTION": "Alcohol consumption and acute myocardial infarction: a benefit of alcohol consumed with meals?", "CONTEXTS": [ "The apparent favorable effect of alcohol on the risk of acute myocardial infarction (MI) may be related to its hypoinsulinemic effect when consumed with meals. We studied how the timing of alcohol consumption in relation to meals might affect the risk of MI in a population with relatively high regular alcohol consumption.", "We conducted a case-control study between 1995 and 1999 in Milan, Italy. Cases were 507 subjects with a first episode of nonfatal acute MI, and controls were 478 patients admitted to hospitals for other acute diseases. Odds ratios (ORs) and 95% confidence intervals (CIs) were calculated by multiple logistic regression models.", "Compared with nondrinkers, an inverse trend in risk was observed when alcohol was consumed during meals only (for>or =3 drinks per day: OR = 0.50; 95% CI = 0.30-0.82). In contrast, no consistent trend in risk was found for subjects drinking outside of meals (for>or =3 drinks per day: 0.98; 0.49-1.96). The pattern of risk was similar when we considered people who drank only wine." ], "LABELS": [ "BACKGROUND", "METHODS", "RESULTS" ], "MESHES": [ "Age Factors", "Alcohol Drinking", "Feeding Behavior", "Female", "Humans", "Italy", "Male", "Multivariate Analysis", "Myocardial Infarction", "Odds Ratio", "Risk Factors", "Time Factors" ], "YEAR": "2004", "reasoning_required_pred": "yes", "reasoning_free_pred": "yes", "final_decision": "yes", "LONG_ANSWER": "Alcohol drinking during meals was inversely related with risk of acute MI, whereas alcohol drinking outside meals only was unrelated to risk." }, "12121321": { "QUESTION": "Do mossy fibers release GABA?", "CONTEXTS": [ "Mossy fibers are the sole excitatory projection from dentate gyrus granule cells to the hippocampus, forming part of the trisynaptic hippocampal circuit. They undergo significant plasticity during epileptogenesis and have been implicated in seizure generation. Mossy fibers are a highly unusual projection in the mammalian brain; in addition to glutamate, they release adenosine, dynorphin, zinc, and possibly other peptides. Mossy fiber terminals also show intense immunoreactivity for the inhibitory neurotransmitter gamma-aminobutyric acid (GABA), and immunoreactivity for GAD67. The purpose of this review is to present physiologic evidence of GABA release by mossy fibers and its modulation by epileptic activity.", "We used hippocampal slices from 3- to 5-week-old guinea pigs and made whole-cell voltage clamp recordings from CA3 pyramidal cells. We placed stimulating electrodes in stratum granulosum and adjusted their position in order to recruit mossy fiber to CA3 projections.", "We have shown that electrical stimuli that recruit dentate granule cells elicit monosynaptic GABAA receptor-mediated synaptic signals in CA3 pyramidal neurons. These inhibitory signals satisfy the criteria that distinguish mossy fiber-CA3 synapses: high sensitivity to metabotropic glutamate-receptor agonists, facilitation during repetitive stimulation, and N-methyl-D-aspartate (NMDA) receptor-independent long-term potentiation." ], "LABELS": [ "PURPOSE", "METHODS", "RESULTS" ], "MESHES": [ "Animals", "Excitatory Postsynaptic Potentials", "Glutamic Acid", "Guinea Pigs", "In Vitro Techniques", "Mossy Fibers, Hippocampal", "Neural Inhibition", "Patch-Clamp Techniques", "Pyramidal Cells", "Receptors, GABA-A", "Signal Transduction", "Synapses", "Synaptic Transmission", "gamma-Aminobutyric Acid" ], "YEAR": "2002", "reasoning_required_pred": "yes", "reasoning_free_pred": "yes", "final_decision": "yes", "LONG_ANSWER": "We have thus provided compelling evidence that there is a mossy fiber GABAergic signal. The physiologic role of this mossy fiber GABAergic signal is uncertain, but may be of developmental importance. Other evidence suggests that this GABAergic signal is transiently upregulated after seizures. This could have an inhibitory or disinhibitory effect, and further work is needed to elucidate its actual role." }, "17342562": { "QUESTION": "The clinical significance of bile duct sludge: is it different from bile duct stones?", "CONTEXTS": [ "Some patients with suspected common bile duct (CBD) stones are found to have sludge and no stones. Although sludge in the gallbladder is a precursor of gallbladder stones, the significance of bile duct sludge (BDS) is poorly defined. This study aimed to compare BDS with bile duct stones in terms of frequency, associated risk factors, and clinical outcome after endoscopic therapy.", "The study enrolled 228 patients who underwent therapeutic endoscopic retrograde cholangiopancreatography (ERCP) for suspected choledocholithiasis. The patients were divided into two groups: patients with BDS but no stones on ERCP and patients with CBD stones. The presence of risk factors for bile duct stones (age, periampullary diverticulum, ductal dilation or angulation, previous open cholecystectomy) were assessed at ERCP. Follow-up data (36 +/- 19 months) were obtained from medical records and by patient questioning.", "Bile duct sludge occurred in 14% (31/228) of patients and was more common in females. After endoscopic clearance, CBD stones recurred in 17% (33/197) of the patients with CBD stones, and in 16% (5/31) of the patients with BDS (p = 0.99). Common bile duct dilation was less common in the sludge group. The other known risk factors for recurrent CBD stones (age, previous open cholecystectomy, bile duct angulation, and the presence of a peripampullary diverticulum) were not statistically different between the two groups." ], "LABELS": [ "BACKGROUND", "METHODS", "RESULTS" ], "MESHES": [ "Adolescent", "Adult", "Aged", "Aged, 80 and over", "Child", "Cholangiopancreatography, Endoscopic Retrograde", "Choledocholithiasis", "Cohort Studies", "Female", "Humans", "Incidence", "Male", "Middle Aged", "Recurrence", "Retrospective Studies", "Risk Factors", "Sex Distribution", "Sphincterotomy, Endoscopic" ], "YEAR": "2007", "reasoning_required_pred": "yes", "reasoning_free_pred": "no", "final_decision": "no", "LONG_ANSWER": "The findings indicate that the clinical significance of symptomatic BDS is similar to that of CBD stones. Bile duct sludge seems to be an early stage of choledocholithiasis." }, "28011794": { "QUESTION": "Can Ambu self-inflating bag and Neopuff infant resuscitator provide adequate and safe manual inflations for infants up to 10\u2005kg weight?", "CONTEXTS": [ "Manual resuscitation devices for infants and newborns must be able to provide adequate ventilation in a safe and consistent manner across a wide range of patient sizes (0.5-10\u2005kg) and differing clinical states. There are little comparative data assessing biomechanical performance of common infant manual resuscitation devices across the manufacturers' recommended operating weight ranges. We aimed to compare performance of the Ambu self-inflating bag (SIB) with the Neopuff T-piece resuscitator in three resuscitation models.", "Five experienced clinicians delivered targeted ventilation to three lung models differing in compliance, delivery pressures and inflation rates; Preterm (0.5\u2005mL/cmH2O, 25/5 cmH2O, 60 per minute), Term (3\u2005mL/cmH2O, 30/5 cmH2O, 40 per minute) and Infant (9\u2005mL/cmH2O, 35/5 cmH2O, 30 per minute). The Neopuff was examined with three gas inflow rates (5 litres per minute (LPM), 10 LPM and 15 LPM) and the Ambu with no gas inflow.", "3309 inflations were collected and analysed with analysis of variance for repeated measures. The Neopuff was unable to reach set peak inflation pressures and exhibited seriously elevated positive end expiratory pressure (PEEP) with all inflow gas rates (p<0.001) in this infant model. The Ambu SIB accurately delivered targeted pressures in all three models." ], "LABELS": [ "BACKGROUND", "METHODS", "RESULTS" ], "MESHES": [ "Equipment Design", "Female", "Humans", "Infant Care", "Infant, Newborn", "Infant, Postmature", "Male", "Positive-Pressure Respiration", "Respiration, Artificial", "Resuscitation" ], "YEAR": "2017", "reasoning_required_pred": "maybe", "reasoning_free_pred": "maybe", "final_decision": "maybe", "LONG_ANSWER": "The Ambu SIB was able to accurately deliver targeted pressures across all three models from preterm to infant. The Neopuff infant resuscitator was unable to deliver the targeted pressures in the infant model developing clinically significant levels of inadvertent PEEP which may pose risk during infant resuscitation." }, "24098953": { "QUESTION": "Are IgM-enriched immunoglobulins an effective adjuvant in septic VLBW infants?", "CONTEXTS": [ "All VLBW infants from January 2008 to December 2012 with positive blood culture beyond 72 hours of life were enrolled in a retrospective cohort study. Newborns born after June 2010 were treated with IgM-eIVIG, 250 mg/kg/day iv for three days in addition to standard antibiotic regimen and compared to an historical cohort born before June 2010, receiving antimicrobial regimen alone. Short-term mortality (i.e. death within 7 and 21 days from treatment) was the primary outcome. Secondary outcomes were: total mortality, intraventricular hemorrhage, necrotizing enterocolitis, periventricular leukomalacia, bronchopulmonary dysplasia at discharge.", "79 neonates (40 cases) were enrolled. No difference in birth weight, gestational age or SNAP II score (disease severity score) were found. Significantly reduced short-term mortality was found in treated infants (22% vs 46%; p = 0.005) considering all microbial aetiologies and the subgroup affected by Candida spp. Secondary outcomes were not different between groups." ], "LABELS": [ "METHODS", "RESULTS" ], "MESHES": [ "Adjuvants, Immunologic", "Analysis of Variance", "Cohort Studies", "Confidence Intervals", "Dose-Response Relationship, Drug", "Drug Administration Schedule", "Drug Combinations", "Female", "Hospital Mortality", "Humans", "Immunoglobulin A", "Immunoglobulin M", "Immunoglobulins, Intravenous", "Infant, Newborn", "Infant, Very Low Birth Weight", "Infusions, Intravenous", "Intensive Care Units, Neonatal", "Italy", "Length of Stay", "Male", "Odds Ratio", "Retrospective Studies", "Risk Assessment", "Sepsis", "Severity of Illness Index", "Survival Rate", "Treatment Outcome" ], "YEAR": "2013", "reasoning_required_pred": "yes", "reasoning_free_pred": "yes", "final_decision": "yes", "LONG_ANSWER": "This hypothesis-generator study shows that IgM-eIVIG is an effective adjuvant therapy in VLBW infants with proven sepsis. Randomized controlled trials are warranted to confirm this pilot observation." }, "24827404": { "QUESTION": "Is resected stomach volume related to weight loss after laparoscopic sleeve gastrectomy?", "CONTEXTS": [ "Laparoscopic sleeve gastrectomy (LSG) was initially performed as the first stage of biliopancreatic diversion with duodenal switch for the treatment of super-obese or high-risk obese patients but is now most commonly performed as a standalone operation. The aim of this prospective study was to investigate outcomes after LSG according to resected stomach volume.", "Between May 2011 and April 2013, LSG was performed in 102 consecutive patients undergoing bariatric surgery. Two patients were excluded, and data from the remaining 100 patients were analyzed in this study. Patients were divided into three groups according to the following resected stomach volume: 700-1,200 mL (group A, n\u2009=\u200921), 1,200-1,700 mL (group B, n\u2009=\u200962), and>1,700 mL (group C, n\u2009=\u200917). Mean values were compared among the groups by analysis of variance.", "The mean percentage excess body weight loss (%EBWL) at 3, 6, 12, and 24 months after surgery was 37.68\u2009\u00b1\u200910.97, 50.97\u2009\u00b1\u200913.59, 62.35\u2009\u00b1\u200911.31, and 67.59\u2009\u00b1\u20099.02 %, respectively. There were no significant differences in mean %EBWL among the three groups. Resected stomach volume was greater in patients with higher preoperative body mass index and was positively associated with resected stomach weight." ], "LABELS": [ "BACKGROUND", "METHODS", "RESULTS" ], "MESHES": [ "Adult", "Bariatric Surgery", "Body Mass Index", "Comorbidity", "Female", "Gastrectomy", "Humans", "Laparoscopy", "Male", "Middle Aged", "Obesity, Morbid", "Prospective Studies", "Stomach", "Treatment Outcome", "Weight Loss" ], "YEAR": "2014", "reasoning_required_pred": "no", "reasoning_free_pred": "no", "final_decision": "no", "LONG_ANSWER": "Mean %EBWL after LSG was not significantly different among three groups of patients divided according to resected stomach volume. Resected stomach volume was significantly greater in patients with higher preoperative body mass index." }, "28407529": { "QUESTION": "Resection of colorectal liver metastases after second-line chemotherapy: is it worthwhile?", "CONTEXTS": [ "Patient outcome after resection of colorectal liver metastases (CLM) following second-line preoperative chemotherapy (PCT) performed for insufficient response or toxicity of the first-line, is little known and has here been compared to the outcome following first-line.", "From January 2005 to June 2013, 5624 and 791 consecutive patients of a prospective international cohort received 1 and 2 PCT lines before CLM resection (group 1 and 2, respectively). Survival and prognostic factors were analysed.", "After a mean follow-up of 30.1 months, there was no difference in survival from CLM diagnosis (median, 3-, and 5-year overall survival [OS]: 58.6 months, 76% and 49% in group 2 versus 58.9 months, 71% and 49% in group 1, respectively, P\u00a0=\u00a00.32). After hepatectomy, disease-free survival (DFS) was however shorter in group 2: 17.2 months, 27% and 15% versus 19.4 months, 32% and 23%, respectively (P\u00a0=\u00a00.001). Among the initially unresectable patients of group 1 and 2, no statistical difference in OS or DFS was observed. Independent predictors of worse OS in group 2 were positive primary lymph nodes, extrahepatic disease, tumour progression on second line, R2 resection\u00a0and number of hepatectomies/year<50. Positive primary nodes, synchronous and bilateral metastases were predictors of shorter DFS. Initial unresectability did not impact OS or DFS in group 2." ], "LABELS": [ "PURPOSE", "PATIENTS AND METHODS", "RESULTS" ], "MESHES": [ "Antibodies, Monoclonal", "Antineoplastic Combined Chemotherapy Protocols", "Camptothecin", "Catheter Ablation", "Cetuximab", "Colonic Neoplasms", "Disease Progression", "Disease-Free Survival", "Female", "Hepatectomy", "Humans", "Liver Neoplasms", "Lymphatic Metastasis", "Male", "Middle Aged", "Postoperative Care", "Preoperative Care", "Prospective Studies", "Rectal Neoplasms", "Registries" ], "YEAR": "2017", "reasoning_required_pred": "no", "reasoning_free_pred": "maybe", "final_decision": "no", "LONG_ANSWER": "CLM resection following second-line PCT, after oncosurgically favourable selection, could bring similar OS compared to what observed after first-line. For initially unresectable patients, OS or DFS is comparable between first- and second-line PCT. Surgery should not be denied after the failure of first-line chemotherapy." }, "27928673": { "QUESTION": "Do ART patients face higher C-section rates during their stage of delivery?", "CONTEXTS": [ "The mode of delivery depends on multiple parameters. After assisted reproductive technology (ART), previous studies have shown elevated C-section rates but few studies differentiated between elective and emergency operations and different protocols of cryopreservation. Because these studies did not use multiparity as exclusion criteria which reduces confounding with previous pregnancies, aim of this study is to compare mode of delivery of different techniques of ART using data of primiparae only [1, 2].", "Retrospective analysis of patient data treated at the university hospital of Luebeck in a period of 12 years. Patients were divided in different groups according to their way of conception: spontaneous conception and conception after\u00a0ART. The group of ART was further divided into: (a) a group of fresh transferred embryos (IVF/ICSI), (b) vitrification and (c) slow freezing. Exclusion criteria were defined as: multiparity, delivery<24.\u00a0+\u00a00\u00a0p.m., incomplete data and treatment outside university of Luebeck. Main parameter of this study was mode of delivery which was divided into spontaneous delivery or C-section. C-sections were further differentiated into elective or emergency C-sections.", "The group of fresh transferred embryos and slow freezing showed higher risks for elective and emergency C-sections (elective C-sections odds ratio 2.0, CI 95% 1.6-2.6, emergency C-sections odds ratio 1.4, CI 95% 1.1-1.9). Moreover, all groups of ART show enhanced risk of significant perinatal bleeding." ], "LABELS": [ "PURPOSE", "METHODS", "RESULTS" ], "MESHES": [ "Adult", "Cesarean Section", "Female", "Humans", "Pregnancy", "Reproductive Techniques, Assisted", "Retrospective Studies" ], "YEAR": "2017", "reasoning_required_pred": "yes", "reasoning_free_pred": "yes", "final_decision": "yes", "LONG_ANSWER": "Patients after ART treatment suffer from higher C-section rates during their stage of delivery." }, "22011946": { "QUESTION": "Does a preoperative medically supervised weight loss program improve bariatric surgery outcomes?", "CONTEXTS": [ "Many insurance payors mandate that bariatric surgery candidates undergo a medically supervised weight management (MSWM) program as a prerequisite for surgery. However, there is little evidence to support this requirement. We evaluated in a randomized controlled trial the hypothesis that participation in a MSWM program does not predict outcomes after laparoscopic adjustable gastric banding (LAGB) in a publicly insured population.", "This pilot randomized trial was conducted in a large academic urban public hospital. Patients who met NIH consensus criteria for bariatric surgery and whose insurance did not require a mandatory 6-month MSWM program were randomized to a MSWM program with monthly visits over 6 months (individual or group) or usual care for 6 months and then followed for bariatric surgery outcomes postoperatively. Demographics, weight, and patient behavior scores, including patient adherence, eating behavior, patient activation, and physical activity, were collected at baseline and at 6 months (immediately preoperatively and postoperatively).", "A total of 55 patients were enrolled in the study with complete follow-up on 23 patients. Participants randomized to a MSWM program attended an average of 2 sessions preoperatively. The majority of participants were female and non-Caucasian, mean age was 46 years, average income was less than $20,000/year, and most had Medicaid as their primary insurer, consistent with the demographics of the hospital's bariatric surgery program. Data analysis included both intention-to-treat and completers' analyses. No significant differences in weight loss and most patient behaviors were found between the two groups postoperatively, suggesting that participation in a MSWM program did not improve weight loss outcomes for LAGB. Participation in a MSWM program did appear to have a positive effect on physical activity postoperatively." ], "LABELS": [ "BACKGROUND", "METHODS", "RESULTS" ], "MESHES": [ "Adult", "Body Mass Index", "Exercise", "Feeding Behavior", "Female", "Gastroplasty", "Health Behavior", "Humans", "Laparoscopy", "Male", "Medication Adherence", "Middle Aged", "Obesity, Morbid", "Pilot Projects", "Preoperative Care", "Treatment Outcome", "Weight Reduction Programs" ], "YEAR": "2012", "reasoning_required_pred": "yes", "reasoning_free_pred": "no", "final_decision": "no", "LONG_ANSWER": "MSWM does not appear to confer additional benefit as compared to the standard preoperative bariatric surgery protocol in terms of weight loss and most behavioral outcomes after LAGB in our patient population." }, "23517744": { "QUESTION": "Is solitary kidney really more resistant to ischemia?", "CONTEXTS": [ "To our knowledge there are no evidence-based medicine data to date to critically judge the vulnerability of a solitary kidney to warm ischemia compared to paired kidneys.", "Ten dogs were exposed to open right nephrectomy to create a solitary kidney model (group 1). Ten dogs with both kidneys were considered group 2. All dogs underwent warm ischemia by open occlusion of the left renal artery for 90 minutes. Dogs were sacrificed at different intervals (3 days to 4 weeks). All dogs were reevaluated by renogram before sacrifice and histopathology of the investigated kidney. The proinflammatory markers CD95 and tumor necrosis factor-\u03b1 were assessed using real-time polymerase chain reaction.", "In group 1 clearance decreased by 20% at 1 week but basal function was regained starting at week 2. In group 2 clearance decreased more than 90% up to week 2. Recovery started at week 3 and by 4 weeks there was a 23% clearance reduction. Histopathological examination in group 1 revealed significant tubular necrosis (60%) at 3 days with regeneration starting at 1 week. In group 2 there was more pronounced tubular necrosis (90%) with regeneration starting at 2 weeks. The expression of proinflammatory markers was up-regulated in each group with higher, more sustained expression in group 2." ], "LABELS": [ "PURPOSE", "MATERIALS AND METHODS", "RESULTS" ], "MESHES": [ "Animals", "Biopsy, Needle", "Disease Models, Animal", "Dogs", "Glomerular Filtration Rate", "Immunohistochemistry", "Ischemia", "Kidney", "Nephrectomy", "Random Allocation", "Reference Values", "Warm Ischemia" ], "YEAR": "2013", "reasoning_required_pred": "yes", "reasoning_free_pred": "yes", "final_decision": "yes", "LONG_ANSWER": "Solitary kidney in a canine model is more resistant to ischemia than paired kidneys based on radiological, pathological and genetic evidence." }, "20842006": { "QUESTION": "Neck pain treatment with acupuncture: does the number of needles matter?", "CONTEXTS": [ "Acupuncture has been successfully used in myofascial pain syndromes. However, the number of needles used, that is, the dose of acupuncture stimulation, to obtain the best antinociceptive efficacy is still a matter of debate. The question was addressed comparing the clinical efficacy of two different therapeutic schemes, characterized by a different number of needles used on 36 patients between 29-60 years of age with by a painful cervical myofascial syndrome.", "Patients were divided into two groups; the first group of 18 patients were treated with 5 needles and the second group of 18 patients were treated with 11 needles, the time of needle stimulation was the same in both groups: 100 seconds. Each group underwent six cycles of somatic acupuncture. Pain intensity was evaluated before, immediately after and 1 and 3 months after the treatment by means of both the Mc Gill Pain Questionnaire and the Visual Analogue Scale (VAS). In both groups, the needles were fixed superficially excluding the two most painful trigger points where they were deeply inserted.", "Both groups, independently from the number of needles used, obtained a good therapeutic effect without clinically relevant differences." ], "LABELS": [ "OBJECTIVES", "METHODS", "RESULTS" ], "MESHES": [ "Acupuncture Therapy", "Adult", "Humans", "Middle Aged", "Myofascial Pain Syndromes", "Neck Pain", "Needles", "Pain Measurement", "Severity of Illness Index", "Surveys and Questionnaires", "Treatment Outcome" ], "YEAR": null, "reasoning_required_pred": "no", "reasoning_free_pred": "no", "final_decision": "no", "LONG_ANSWER": "For this pathology, the number of needles, 5 or 11, seems not to be an important variable in determining the therapeutic effect when the time of stimulation is the same in the two groups." }, "9140335": { "QUESTION": "Does fluoridation reduce the use of dental services among adults?", "CONTEXTS": [ "The authors determine whether prevention influences the use of health services. Fluoridation's effect on restorative dental demand among 972 Washington state employees and spouses, aged 20 to 34 years, in two fluoridated communities and a nonfluoridated community was examined.", "At baseline, adults were interviewed by telephone, and oral assessments were conducted to measure personal characteristics, lifetime exposure to fluoridated water, oral disease, and the quality of restorations. Adults were followed for 2 years to measure dental demand from dental claims. Each adult's baseline and claims data were linked with provider and practice variables collected from the dentist who provided treatment.", "Relative to adults with no lifetime exposure to fluoridated water, adults drinking fluoridated water for half or more of their lives had less disease at baseline and a lower but nonsignificant probability of receiving a restoration in the follow-up period. In the 2-year follow-up period, however, more than half of the restorations were performed to replace fillings of satisfactory or ideal quality at baseline. When only teeth with decay and unsatisfactory fillings at baseline were considered, adults with high fluoridation exposure had a lower probability of receiving a restoration than adults with no exposure. Market effects also were detected in demand equations; relative to adults in the nonfluoridated community, adults residing in the fluoridated community with a large dentist supply received a greater number of restorations, suggesting potential supplier-induced demand from less disease and fewer patients." ], "LABELS": [ "OBJECTIVES", "METHODS", "RESULTS" ], "MESHES": [ "Adult", "Dental Health Surveys", "Dental Restoration, Permanent", "Female", "Fluoridation", "Health Services Needs and Demand", "Humans", "Insurance Claim Reporting", "Insurance, Dental", "Male", "Marketing of Health Services", "Multivariate Analysis", "Washington" ], "YEAR": "1997", "reasoning_required_pred": "yes", "reasoning_free_pred": "maybe", "final_decision": "maybe", "LONG_ANSWER": "Among adults aged 20 to 34 years with private dental insurance, fluoridation reduces oral disease but may or may not reduce use of restorative services, depending on dentists' clinical decisions." }, "12142826": { "QUESTION": "Pertrochanteric fractures: is there an advantage to an intramedullary nail?", "CONTEXTS": [ "To compare the results between a sliding compression hip screw and an intramedullary nail in the treatment of pertrochanteric fractures.", "Prospective computer-generated randomization of 206 patients into two study groups: those treated by sliding compression hip screw (Group 1; n = 106) and those treated by intramedullary nailing (Group 2; n = 100).", "University Level I trauma center.", "All patients over the age of fifty-five years presenting with fractures of the trochanteric region caused by a low-energy injury, classified as AO/OTA Type 31-A1 and A2.", "Treatment with a sliding compression hip screw (Dynamic Hip Screw; Synthes-Stratec, Oberdorf, Switzerland) or an intramedullary nail (Proximal Femoral Nail; Synthes-Stratec, Oberdorf, Switzerland).", "Intraoperative: operative and fluoroscopy times, the difficulty of the operation, intraoperative complications, and blood loss. Radiologic: fracture healing and failure of fixation. Clinical: pain, social functioning score, and mobility score.", "The minimum follow-up was one year. We did not find any statistically significant difference, intraoperatively, radiologically, or clinically, between the two groups of patients." ], "LABELS": [ "OBJECTIVES", "DESIGN", "SETTING", "PATIENTS", "INTERVENTION", "MAIN OUTCOME MEASUREMENTS", "RESULTS" ], "MESHES": [ "Aged", "Aged, 80 and over", "Bone Nails", "Bone Screws", "Equipment Design", "Female", "Fracture Fixation, Intramedullary", "Hip Fractures", "Humans", "Male", "Prospective Studies" ], "YEAR": "2002", "reasoning_required_pred": "no", "reasoning_free_pred": "no", "final_decision": "no", "LONG_ANSWER": "There is no advantage to an intramedullary nail versus a sliding compression hip screw for low-energy pertrochanteric fractures AO/OTA 31-A1 and A2, specifically with its increased cost and lack of evidence to show decreased complications or improved patient outcome." }, "22382608": { "QUESTION": "SPECT study with I-123-Ioflupane (DaTSCAN) in patients with essential tremor. Is there any correlation with Parkinson's disease?", "CONTEXTS": [ "The differential diagnosis between essential tremor (ET) and Parkinson's disease (PD) may be, in some cases, very difficult on clinical grounds alone. In addition, it is accepted that a small percentage of ET patients presenting symptoms and signs of possible PD may progress finally to a typical pattern of parkinsonism. Ioflupane, N-u-fluoropropyl-2a-carbomethoxy-3a-(4-iodophenyl) nortropane, also called FP-CIT, labelled with (123)I (commercially known as DaTSCAN) has been proven to be useful in the differential diagnosis between PD and ET and to confirm dopaminergic degeneration in patients with parkinsonism. The aim of this study is to identify dopaminergic degeneration in patients with PD and distinguish them from others with ET using semi-quantitative SPECT (123)I-Ioflupane (DaTSCAN) data in comparison with normal volunteers (NV), in addition with the respective ones of patients referred as suffering from ET, as well as, of patients with a PD diagnosis at an initial stage with a unilateral presentation of motor signs.", "Twenty-eight patients suffering from ET (10 males plus 18 females) and 28 NV (12 males and 16 females) were enroled in this study. In addition, 33 patients (11 males and 22 females) with an established diagnosis of PD with unilateral limb involvement (12 left hemi-body and 21 right hemi-body) were included for comparison with ET. We used DaTSCAN to obtain SPECT images and measure the radiopharmaceutical uptake in the striatum (S), as well as the caudate nucleus (CN) and putamen (P) in all individuals.", "Qualitative (Visual) interpretation of the SPECT data did not find any difference in the uptake of the radiopharmaceutical at the level of the S, CN and P between NV and ET patients. Reduced accumulation of the radiopharmaceutical uptake was found in the P of all PD patients. Semiquantitative analysis revealed significant differences between NV and ET patients in the striatum, reduced in the latter. There was also a significant reduction in the tracer accumulation in the left putamen of patients with right hemi-parkinsonism compared to ET and NV. Patients with left hemi-parkinsonism, demonstrated reduced radioligand uptake in the right putamen in comparison with ET and NV. Clinical follow-up of 20 patients with ET at (so many months afterwards) revealed no significant change in clinical presentation, particularly no signs of PD. Follow-up DaTSCAN performed in 10 of them (so many months afterwards) was negative in all but one. This one had an equivocal baseline study which deteriorated 12\u00a0months later." ], "LABELS": [ "OBJECTIVES", "METHODS", "RESULTS" ], "MESHES": [ "Case-Control Studies", "Diagnosis, Differential", "Dopamine", "Essential Tremor", "Female", "Humans", "Male", "Motor Activity", "Nortropanes", "Parkinson Disease", "Tomography, Emission-Computed, Single-Photon" ], "YEAR": "2012", "reasoning_required_pred": "yes", "reasoning_free_pred": "no", "final_decision": "no", "LONG_ANSWER": "Our results do not support the hypothesis of a link between essential tremor and Parkinson's disease. However, it appears that ET patients have a small degree of striatal dopaminergic degeneration. If this is due to alterations in the nigrostriatl pathway or of other origin it is not clear. Follow-up studies of essential tremor patients are warranted to assess progression of disease and to understand better the possible cause for striatal dopaminergic degeneration." }, "17329379": { "QUESTION": "Are the GFRUP's recommendations for withholding or withdrawing treatments in critically ill children applicable?", "CONTEXTS": [ "To evaluate feasibility of the guidelines of the Groupe Francophone de R\u00e9animation et Urgence P\u00e9diatriques (French-speaking group of paediatric intensive and emergency care; GFRUP) for limitation of treatments in the paediatric intensive care unit (PICU).", "A 2-year prospective survey.", "A 12-bed PICU at the H\u00f4pital Jeanne de Flandre, Lille, France.", "Were included when limitation of treatments was expected.", "Of 967 children admitted, 55 were included with a 2-day median delay. They were younger than others (24 v 60 months), had a higher paediatric risk of mortality (PRISM) score (14 v 4), and a higher paediatric overall performance category (POPC) score at admission (2 v 1); all p<0.002. 34 (50% of total deaths) children died. A limitation decision was made without meeting for 7 children who died: 6 received do-not-resuscitate orders (DNROs) and 1 received withholding decision. Decision-making meetings were organised for 31 children, and the following decisions were made: 12 DNROs (6 deaths and 6 survivals), 4 withholding (1 death and 3 survivals), with 14 withdrawing (14 deaths) and 1 continuing treatment (survival). After limitation, 21 (31% of total deaths) children died and 10 survived (POPC score 4). 13 procedures were interrupted because of death and 11 because of clinical improvement (POPC score 4). Parents' opinions were obtained after 4 family conferences (for a total of 110 min), 3 days after inclusion. The first meeting was planned for 6 days after inclusion and held on the 7th day after inclusion; 80% of parents were immediately informed of the decision, which was implemented after half a day." ], "LABELS": [ "OBJECTIVE", "DESIGN", "SETTING", "PATIENTS", "RESULTS" ], "MESHES": [ "Child, Preschool", "Critical Illness", "Decision Making", "France", "Humans", "Infant", "Infant Mortality", "Infant, Newborn", "Intensive Care, Neonatal", "Parents", "Patient Discharge", "Practice Guidelines as Topic", "Prognosis", "Prospective Studies", "Time Factors", "Withholding Treatment" ], "YEAR": "2007", "reasoning_required_pred": "yes", "reasoning_free_pred": "yes", "final_decision": "yes", "LONG_ANSWER": "GFRUPs procedure was applicable in most cases. The main difficulties were anticipating the correct date for the meeting and involving nurses in the procedure. Children for whom the procedure was interrupted because of clinical improvement and who survived in poor condition without a formal decision pointed out the need for medical criteria for questioning, which should systematically lead to a formal decision-making process." }, "27288618": { "QUESTION": "Is inhaled prophylactic heparin useful for prevention and Management of Pneumonia in ventilated ICU patients?", "CONTEXTS": [ "To determine whether prophylactic inhaled heparin is effective for the prevention and treatment of pneumonia patients receiving mechanical ventilation (MV) in the intensive care unit.", "A phase 2, double blind randomized controlled trial stratified for study center and patient type (non-operative, post-operative) was conducted in three university-affiliated intensive care units. Patients aged \u226518years and requiring invasive MV for more than 48hours were randomized to usual care, nebulization of unfractionated sodium heparin (5000 units in 2mL) or placebo nebulization with 0.9% sodium chloride (2mL) four times daily with the main outcome measures of the development of ventilator associated pneumonia (VAP), ventilator associated complication (VAC) and sequential organ failure assessment scores in patients with pneumonia on admission or who developed VAP.", "Australian and New Zealand Clinical Trials Registry ACTRN12612000038897.", "Two hundred and fourteen patients were enrolled (72 usual care, 71 inhaled sodium heparin, 71 inhaled sodium chloride). There were no differences between treatment groups in terms of the development of VAP, using either Klompas criteria (6-7%, P=1.00) or clinical diagnosis (24-26%, P=0.85). There was no difference in the clinical consistency (P=0.70), number (P=0.28) or the total volume of secretions per day (P=.54). The presence of blood in secretions was significantly less in the usual care group (P=0.005)." ], "LABELS": [ "PURPOSE", "METHODS", "TRIAL REGISTRATION", "RESULTS" ], "MESHES": [ "Administration, Inhalation", "Adolescent", "Adult", "Aged", "Aged, 80 and over", "Australia", "Double-Blind Method", "Female", "Fibrinolytic Agents", "Heparin", "Humans", "Intensive Care Units", "Male", "Middle Aged", "Nebulizers and Vaporizers", "New Zealand", "Pneumonia, Ventilator-Associated", "Respiration, Artificial", "Young Adult" ], "YEAR": "2016", "reasoning_required_pred": "no", "reasoning_free_pred": "no", "final_decision": "no", "LONG_ANSWER": "Nebulized heparin cannot be recommended for prophylaxis against VAP or to hasten recovery from pneumonia in patients receiving MV." }, "21873082": { "QUESTION": "Is the h-index predictive of greater NIH funding success among academic radiologists?", "CONTEXTS": [ "Despite rapid adoption of the Hirsch index (h-index) as a measure of academic success, the correlations between the h-index and other metrics of productivity remain poorly understood. The aims of this study were to determine whether h-indices were associated with greater National Institutes of Health (NIH) funding success among academic radiologists.", "Using the Scopus database, h-indices were calculated for a random sample of academic radiologists with the rank of professor. Using the NIH tool Research Portfolio Online Reporting Tools Expenditures and Reports, we determined the number, classification, and total years of NIH grant funding as principal investigator for each radiologist. Differences in h-index, sorted by funding status, were determined using Wilcoxon's tests. Associations between h-index and funding status were determined using logistic regression. Significant correlations between h-index and grant metrics were determined using Spearman's \u03c1.", "Among 210 professors of radiology, 48 (23%) secured at least one NIH grant. The mean h-index was significantly higher among individuals who secured at least one NIH grant (19.1) compared to those who did not (10.4) (P<.0001). Professors with h-indices<10 compared to those with h-indices>10 were significantly less likely to receive NIH funding (odds ratio, 0.07; P = .0321). However, h-indices>10 were not significantly predictive of greater funding. No significant relationships were observed between h-index and the number of grant awards, years of prior funding, the amounts of grant awards, or grant classification." ], "LABELS": [ "RATIONALE AND OBJECTIVES", "MATERIALS AND METHODS", "RESULTS" ], "MESHES": [ "Academic Medical Centers", "Bibliometrics", "Biomedical Research", "Humans", "Journal Impact Factor", "National Institutes of Health (U.S.)", "Periodicals as Topic", "Publishing", "Radiology", "Research Support as Topic", "United States" ], "YEAR": "2011", "reasoning_required_pred": "yes", "reasoning_free_pred": "maybe", "final_decision": "yes", "LONG_ANSWER": "Having obtained at least one NIH grant was associated with a higher h-index, yet multiple or large grants, such as those for program projects, were not predictive of higher h-indices." }, "25810292": { "QUESTION": "Is minimally invasive mitral valve repair with artificial chords reproducible and applicable in routine surgery?", "CONTEXTS": [ "Traditional resectional techniques and chordal transfer are difficult to apply in video-assisted mitral valve repair. Using artificial chords appears easier in this setting. The purpose of this study was to review the effectiveness and reproducibility of neochordal repair as a routine approach to minimally invasive mitral repair, and to assess the stability of neochord implantation using the figure-of-eight suture without pledgets in this setting.", "This is a retrospective review of all patients who underwent minimally invasive video-assisted mitral valve repair from 2008 to 2013. The primary endpoints were recurrent mitral regurgitation and reoperation.", "A total of 426 consecutive patients were included during the study period, with a mean age of 55 \u00b1 18 years. Neochords were used in all patients, and in association with leaflet resection in 47 patients. One patient was not repairable and underwent valve replacement (repair rate, 99.8%). Fifteen patients had Grade I (3.5%) regurgitation, whereas the remainder had none. Patients were fast-tracked, with 25% extubated in the operation theatre and the remainder within 6 h. There were 5 deaths within 30 days (1.2%). Follow-up ranged 3-60 months, during which all of the patients remained with no or trace mitral regurgitation. No de-insertion or rupture of any neochords was found, and no patients required a reoperation." ], "LABELS": [ "OBJECTIVES", "METHODS", "RESULTS" ], "MESHES": [ "Adult", "Aged", "Chordae Tendineae", "Female", "Heart Valve Prosthesis", "Heart Valve Prosthesis Implantation", "Humans", "Kaplan-Meier Estimate", "Male", "Middle Aged", "Minimally Invasive Surgical Procedures", "Mitral Valve", "Mitral Valve Insufficiency", "Prosthesis Design", "Recurrence", "Reoperation", "Retrospective Studies", "Risk Factors", "Suture Techniques", "Time Factors", "Treatment Outcome", "Video-Assisted Surgery" ], "YEAR": "2015", "reasoning_required_pred": "yes", "reasoning_free_pred": "yes", "final_decision": "yes", "LONG_ANSWER": "Minimally invasive mitral valve repair using neochords provided a high rate of repair, reproducible results in a routine cardiac surgery setting and stable repair during follow-up. This has become our preferred technique for mitral valve surgery." }, "21739621": { "QUESTION": "Does obesity predict knee pain over fourteen years in women, independently of radiographic changes?", "CONTEXTS": [ "To examine longitudinal patterns in body mass index (BMI) over 14 years and its association with knee pain in the Chingford Study.", "We studied a total of 594 women with BMI data from clinic visits at years (Y) 1, 5, 10, and 15. Knee pain at Y15 was assessed by questionnaire. Associations between BMI over 14 years and knee pain at Y15 were examined using logistic regression.", "BMI significantly increased from Y1 to Y15 (P<0.0005) with medians (interquartile ranges) of 24.5 kg/m(2) (22.5-27.2 kg/m(2) ) and 26.5 kg/m(2) (23.9-30.1 kg/m(2) ), respectively. At Y15, 45.1% of subjects had knee pain. A greater BMI at Y1 (odds ratio [OR] 1.34, 95% confidence interval [95% CI]1.05-1.69), at Y15 (OR 1.34, 95% CI 1.10-1.61), and change in BMI over 15 years (OR 1.40, 95% CI 1.00-1.93) were significant predictors of knee pain at Y15 (P<0.05). BMI change was associated with bilateral (OR 1.61, 95% CI 1.05-1.76, P = 0.024) but not unilateral knee pain (OR 1.22, 95% CI 0.73-1.76, P = 0.298). The association between BMI change and knee pain was independent of radiographic knee osteoarthritis (OA). The strength of association between BMI and knee pain at Y15 was similar during followup measurements." ], "LABELS": [ "OBJECTIVE", "METHODS", "RESULTS" ], "MESHES": [ "Aged", "Arthralgia", "Body Mass Index", "England", "Female", "Humans", "Knee Joint", "Logistic Models", "Longitudinal Studies", "Middle Aged", "Obesity", "Odds Ratio", "Osteoarthritis, Knee", "Pain Measurement", "Prospective Studies", "Radiography", "Risk Assessment", "Risk Factors", "Surveys and Questionnaires", "Time Factors" ], "YEAR": "2011", "reasoning_required_pred": "yes", "reasoning_free_pred": "yes", "final_decision": "yes", "LONG_ANSWER": "Over 14 years, a higher BMI predicts knee pain at Y15 in women, independently of radiographic knee OA. When adjusted, the association was significant in bilateral, not unilateral, knee pain, suggesting alternative pathologic mechanisms may exist. The longitudinal effect of BMI on knee pain at Y15 is equally important at any time point, which may assist reducing the population burden of knee pain." }, "20073599": { "QUESTION": "Do liquid-based preparations of urinary cytology perform differently than classically prepared cases?", "CONTEXTS": [ "The cytomorphology of liquid-based preparations in urine cytology is different than classic slide preparations.", "To compare the performance of liquid-based preparation specimens to classically prepared urine specimens with a malignant diagnosis in the College of American Pathologists Interlaboratory Comparison Program in Nongynecologic Cytology.", "Participant responses between 2000 and 2007 for urine specimens with a reference diagnosis of high-grade urothelial carcinoma/carcinoma in situ/dysplasia (HGUCA), squamous cell carcinoma, or adenocarcinoma were evaluated. ThinPrep and SurePath challenges were compared with classic preparations (smears, cytospins) for discordant responses.", "There were 18 288 pathologist, 11 957 cytotechnologist, and 8086 \"laboratory\" responses available. Classic preparations comprised 90% (n = 34 551) of urine challenges; 9% (n = 3295) were ThinPrep and 1% (n = 485) were SurePath. Concordance to the general category of \"positive-malignant\" was seen in 92% of classic preparations, 96.5% of ThinPrep, and 94.6% of SurePath challenges (P<.001). These results were statistically different for the exact reference interpretation of HGUCA (P<.001) but not for adenocarcinoma (P = .22). Cytotechnologists demonstrate statistically better performance for the general category of \"positive-malignant\" compared with pathologists for all urinary slide types and for the exact reference interpretation of HGUCA (94% versus 91.1%; P<.001) but not adenocarcinoma (96.3% versus 95.8%; P = .77) or squamous cell carcinoma (93.6% versus 87.7%; P = .07)." ], "LABELS": [ "CONTEXT", "OBJECTIVES", "DESIGN", "RESULTS" ], "MESHES": [ "Adenocarcinoma", "Carcinoma in Situ", "Carcinoma, Squamous Cell", "Cytological Techniques", "Diagnosis, Differential", "Humans", "Pathology, Clinical", "Societies, Medical", "United States", "Urinary Bladder Neoplasms", "Urine" ], "YEAR": "2010", "reasoning_required_pred": "yes", "reasoning_free_pred": "yes", "final_decision": "yes", "LONG_ANSWER": "Liquid-based preparations performed significantly better in urinary cytology challenges when evaluating malignant categories in the College of American Pathologists interlaboratory comparison program. The liquid-based preparation challenges also performed better for the exact reference interpretation of HGUCA, but no difference was observed for adenocarcinoma challenges. Cytotechnologists perform better than pathologists for all slide types, as well as those demonstrating HGUCA. These results suggest that liquid-based preparations facilitate a more accurate diagnosis than conventional preparations." }, "21845457": { "QUESTION": "Outcomes of severely injured adult trauma patients in an Australian health service: does trauma center level make a difference?", "CONTEXTS": [ "Trauma centers are designated to provide systematized multidisciplinary care to injured patients. Effective trauma systems reduce patient mortality by facilitating the treatment of injured patients at appropriately resourced hospitals. Several U.S. studies report reduced mortality among patients admitted directly to a level I trauma center compared with those admitted to hospitals with less resources. It has yet to be shown whether there is an outcome benefit associated with the \"level of hospital\" initially treating severely injured trauma patients in Australia. This study was designed to determine whether the level of trauma center providing treatment impacts mortality and/or hospital length of stay.", "Outcomes were evaluated for severely injured trauma patients with an Injury Severity Score (ISS)>15 using NSW Institute of Trauma and Injury Management data from 2002-2007 for our regional health service. To assess the association between trauma centers and binary outcomes, a logistic regression model was used. To assess the association between trauma centers and continuous outcomes, a multivariable linear regression model was used. Sex, age, and ISS were included as covariates in all models.", "There were 1,986 trauma presentations during the 6-year period. Patients presenting to a level III trauma center had a significantly higher risk of death than those presenting to the level I center, regardless of age, sex, ISS, or prehospital time. Peer review of deaths at the level III center identified problems in care delivery in 15 cases associated with technical errors, delay in decision making, or errors of judgement." ], "LABELS": [ "BACKGROUND", "METHODS", "RESULTS" ], "MESHES": [ "Adolescent", "Adult", "Aged", "Australia", "Female", "Humans", "Injury Severity Score", "Male", "Middle Aged", "Trauma Centers", "Treatment Outcome", "Wounds and Injuries", "Young Adult" ], "YEAR": "2011", "reasoning_required_pred": "yes", "reasoning_free_pred": "yes", "final_decision": "yes", "LONG_ANSWER": "Severely injured patients treated at a level III center had a higher mortality rate than those treated at a level I center. Most problems identified occurred in the emergency department and were related to delays in care provision. This research highlights the importance of efficient prehospital, in-hospital, and regional trauma systems, performance monitoring, peer review, and adherence to protocols and guidelines." }, "16216859": { "QUESTION": "Does a well developed collateral circulation predispose to restenosis after percutaneous coronary intervention?", "CONTEXTS": [ "To evaluate whether a well developed collateral circulation predisposes to restenosis after percutaneous coronary intervention (PCI).", "Prospective observational study.", "58 patients undergoing elective single vessel PCI in a tertiary referral interventional cardiac unit in the UK.", "Collateral flow index (CFI) was calculated as (Pw-Pv)/(Pa-Pv), where Pa, Pw, and Pv are aortic, coronary wedge, and right atrial pressures during maximum hyperaemia. Collateral supply was considered poor (CFI<0.25) or good (CFI>or = 0.25).", "In-stent restenosis six months after PCI, classified as neointimal volume>or = 25% stent volume on intravascular ultrasound (IVUS), or minimum lumen area10% residual stenosis, and smoking history were predictive of restenosis." ], "LABELS": [ "OBJECTIVE", "DESIGN", "PATIENTS AND SETTING", "METHODS", "MAIN OUTCOME MEASURES", "RESULTS" ], "MESHES": [ "Angioplasty, Balloon, Coronary", "Case-Control Studies", "Collateral Circulation", "Coronary Angiography", "Coronary Restenosis", "Coronary Stenosis", "Female", "Humans", "Male", "Middle Aged", "Predictive Value of Tests", "Prospective Studies", "Risk Factors", "Stents", "Ultrasonography" ], "YEAR": "2006", "reasoning_required_pred": "yes", "reasoning_free_pred": "no", "final_decision": "no", "LONG_ANSWER": "A well developed collateral circulation does not predict an increased risk of restenosis after PCI." }, "19103915": { "QUESTION": "Are home sampling kits for sexually transmitted infections acceptable among men who have sex with men?", "CONTEXTS": [ "There is an urgent need to increase opportunistic screening for sexually transmitted infections (STIs) in community settings, particularly for those who are at increased risk including men who have sex with men (MSM). The aim of this qualitative study was to explore whether home sampling kits (HSK) for multiple bacterial STIs are potentially acceptable among MSM and to identify any concerns regarding their use. This study was developed as part of a formative evaluation of HSKs.", "Focus groups and one-to-one semi-structured interviews with MSM were conducted. Focus group participants (n = 20) were shown a variety of self-sampling materials and asked to discuss them. Individual interviewees (n = 24) had experience of the self-sampling techniques as part of a pilot clinical study. All data were digitally recorded and transcribed verbatim. Data were analysed using a framework analysis approach.", "The concept of a HSK was generally viewed as positive, with many benefits identified relating to increased access to testing, enhanced personal comfort and empowerment. Concerns about the accuracy of the test, delays in receiving the results, the possible lack of support and potential negative impact on 'others' were raised." ], "LABELS": [ "OBJECTIVE", "METHODS", "RESULTS" ], "MESHES": [ "Adult", "Bisexuality", "Feasibility Studies", "Focus Groups", "Health Knowledge, Attitudes, Practice", "Homosexuality, Male", "Humans", "Interviews as Topic", "Male", "Mass Screening", "Middle Aged", "Patient Acceptance of Health Care", "Qualitative Research", "Reagent Kits, Diagnostic", "Risk-Taking", "Sexually Transmitted Diseases", "United Kingdom", "Young Adult" ], "YEAR": "2009", "reasoning_required_pred": "maybe", "reasoning_free_pred": "yes", "final_decision": "maybe", "LONG_ANSWER": "The widespread acceptability of using HSKs for the diagnosis of STIs could have important public health impacts in terms of earlier diagnosis of asymptomatic infections and thus a decrease in the rate of onward transmission. In addition, HSKs could potentially optimize the use of genitourinary medicine services and facilitate patient choice." }, "22521460": { "QUESTION": "Does route of delivery affect maternal and perinatal outcome in women with eclampsia?", "CONTEXTS": [ "The route of delivery in eclampsia is controversial. We hypothesized that adverse maternal and perinatal outcomes may not be improved by early cesarean delivery.", "This was a randomized controlled exploratory trial carried out in a rural teaching institution. In all, 200 eclampsia cases, carrying \u226534 weeks, were allocated to either cesarean or vaginal delivery. Composite maternal and perinatal event rates (death and severe morbidity) were compared by intention-to-treat principle.", "Groups were comparable at baseline with respect to age and key clinical parameters. Maternal event rate was similar: 10.89% in the cesarean arm vs 7.07% for vaginal delivery (relative risk, 1.54; 95% confidence interval, 0.62-3.81). Although the neonatal event rate was less in cesarean delivery-9.90% vs 19.19% (relative risk, 0.52; 95% confidence interval, 0.25-1.05)-the difference was not significant statistically." ], "LABELS": [ "OBJECTIVE", "STUDY DESIGN", "RESULTS" ], "MESHES": [ "Adult", "Anticonvulsants", "Antihypertensive Agents", "Apgar Score", "Cesarean Section", "Critical Care", "Eclampsia", "Female", "Humans", "Infant, Newborn", "Intention to Treat Analysis", "Labetalol", "Magnesium Sulfate", "Outcome Assessment (Health Care)", "Perinatal Mortality", "Pilot Projects", "Pregnancy", "Pregnancy Outcome", "Time Factors" ], "YEAR": "2012", "reasoning_required_pred": "no", "reasoning_free_pred": "no", "final_decision": "no", "LONG_ANSWER": "A policy of early cesarean delivery in eclampsia, carrying \u226534 weeks, is not associated with better outcomes." }, "17062234": { "QUESTION": "Surgical management of the atherosclerotic ascending aorta: is endoaortic balloon occlusion safe?", "CONTEXTS": [ "Occlusion of the atherosclerotic ascending aorta by an endoaortic inflatable balloon has been proposed as an alternative to conventional cross-clamping to prevent injury to the vessel and distal embolization of debris. The safety and the effectiveness of endoaortic occlusion have not been documented in this setting.", "Endoaortic occlusion was employed in 52 of 2,172 consecutive patients. Surgeon's choice was based on preoperative identification of aortic calcifications or intraoperative epiaortic ultrasonographic scanning. Deaths and strokes were analyzed casewise and in aggregate.", "In 10 patients (19.2%), the endoaortic balloon had to be replaced by the ordinary cross-clamp because of incomplete occlusion (n = 5), hindered exposure (n = 2), or balloon rupture (n = 3). In-hospital death occurred in 13 patients (25%), and stroke on awakening from anesthesia in 2 (3.8%). The death rate of patients treated by endoaortic occlusion was significantly higher compared with all other patients (4.2%, p<0.0001) and with the expected estimate by European System for Cardiac Operative Risk Evaluation (10.5%, p = 0.05). By multivariable analysis, use of endoaortic occlusion was independently associated with in-hospital death (odds ratio = 5.609, 95% confidence interval: 2.684 to 11.719). Although the stroke rate was higher in the endoaortic occlusion group compared with all other patients, the difference was only possibly significant (3.8% versus 0.8%, p = 0.067)." ], "LABELS": [ "BACKGROUND", "METHODS", "RESULTS" ], "MESHES": [ "Aged", "Aged, 80 and over", "Aorta", "Aortic Diseases", "Atherosclerosis", "Balloon Occlusion", "Constriction", "Coronary Artery Bypass", "Female", "Hospital Mortality", "Humans", "Male", "Middle Aged", "Retrospective Studies", "Stroke", "Treatment Outcome" ], "YEAR": "2006", "reasoning_required_pred": "no", "reasoning_free_pred": "no", "final_decision": "no", "LONG_ANSWER": "In this series, the endoaortic occlusion was frequently ineffective, and was associated with a significantly higher risk of in-hospital death and a numerically higher risk of stroke." }, "26126304": { "QUESTION": "Estradiol and Antagonist Pretreatment Prior to Microdose Leuprolide in in Vitro Fertilization. Does It Improve IVF Outcomes in Poor Responders as Compared to Oral Contraceptive Pill?", "CONTEXTS": [ "To compare in vitro fertilization (IVF) outcomes in low responders stimulated with microdose leuprolide protocol (ML) following pretreatment with either oral contraceptive pill (OCP) or luteal estradiol (E2) + GnRH antagonist (E2 + antag) for follicular synchronization prior to controlled ovarian hyperstimulation (COH).", "This was a retrospective study of 130 women, who were poor responders, undergoing IVF with either OCP/ML or E2+ antag/ML protocols. The main outcome measures were ongoing pregnancy rates, number of oocytes retrieved, and cancellation rate.", "Both groups were similar in baseline characteristics. There were no significant differences in gonadotropin requirement, cancellation rate, and number of embryos transferred. Ongoing pregnancy rates (40% vs. 15%) were significantly higher in the OCP/ML group. Trends toward greater number of oocytes retrieved (7.7 \u00b1 3.4 vs. 5.9 \u00b1 4.2) and improved implantation rates (20% vs. 12%) were also noted, but these did not reach statistical significance." ], "LABELS": [ "OBJECTIVE", "STUDY DESIGN", "RESULTS" ], "MESHES": [ "Adult", "Contraceptives, Oral, Hormonal", "Estradiol", "Estrogens", "Female", "Fertility Agents, Female", "Fertilization in Vitro", "Gonadotropin-Releasing Hormone", "Hormone Antagonists", "Humans", "Leuprolide", "Oocyte Retrieval", "Pregnancy", "Pregnancy Rate", "Premedication", "Retrospective Studies" ], "YEAR": null, "reasoning_required_pred": "no", "reasoning_free_pred": "no", "final_decision": "no", "LONG_ANSWER": "E2+antag pretreatment does not appear to improve IVF outcomes in ML protocol when compared to the standard OCP in poor responders. Randomized trials with adequate power to study the optimal method of steroid pretreatments appear justified." }, "19578820": { "QUESTION": "Are opioid dependence and methadone maintenance treatment (MMT) documented in the medical record?", "CONTEXTS": [ "Opioid-dependent patients often have co-occurring chronic illnesses requiring medications that interact with methadone. Methadone maintenance treatment (MMT) is typically provided separately from medical care. Hence, coordination of medical care and substance use treatment is important to preserve patient safety.", "To identify potential safety risks among MMT patients engaged in medical care by evaluating the frequency that opioid dependence and MMT documentation are missing in medical records and characterizing potential medication-methadone interactions.", "Among patients from a methadone clinic who received primary care from an affiliated, but separate, medical center, we reviewed electronic medical records for documentation of methadone, opioid dependence, and potential drug-methadone interactions. The proportions of medical records without opioid dependence and methadone documentation were estimated and potential medication-methadone interactions were identified.", "Among the study subjects (n = 84), opioid dependence documentation was missing from the medical record in 30% (95% CI, 20%-41%) and MMT documentation was missing from either the last primary care note or the last hospital discharge summary in 11% (95% CI, 5%-19%). Sixty-nine percent of the study subjects had at least 1 medication that potentially interacted with methadone; 19% had 3 or more potentially interacting medications." ], "LABELS": [ "BACKGROUND", "OBJECTIVE", "METHODS", "RESULTS" ], "MESHES": [ "Adult", "Documentation", "Drug Interactions", "Female", "Humans", "Male", "Medical Records", "Medical Records Systems, Computerized", "Methadone", "Middle Aged", "Opioid-Related Disorders", "Retrospective Studies", "Risk Factors" ], "YEAR": "2009", "reasoning_required_pred": "maybe", "reasoning_free_pred": "maybe", "final_decision": "maybe", "LONG_ANSWER": "Among patients receiving MMT and medical care at different sites, documentation of opioid dependence and MMT in the medical record occurs for the majority, but is missing in a substantial number of patients. Most of these patients are prescribed medications that potentially interact with methadone. This study highlights opportunities for improved coordination between medical care and MMT." }, "21228436": { "QUESTION": "Can teaching medical students to investigate medication errors change their attitudes towards patient safety?", "CONTEXTS": [ "The purpose of this study was to evaluate the impact of a patient-safety curriculum administered during a paediatric clerkship on medical students' attitudes towards patient safety.", "Medical students viewed an online video introducing them to systems-based analyses of medical errors. Faculty presented an example of a medication administration error and demonstrated use of the Learning From Defects tool to investigate the defect. Student groups identified and then analysed medication errors during their clinical rotation using the Learning From Defects framework to organise and present their findings. Outcomes included patient safety attitudinal changes, as measured by questions derived from the Safety Attitudes Questionnaire.", "108 students completed the curriculum between July 2008 and July 2009. All student groups (25 total) identified, analysed and presented patient safety concerns. Curriculum effectiveness was demonstrated by significant changes on questionnaire items related to patient safety attitudes. The majority of students felt that the curriculum was relevant to their clinical rotation and should remain part of the clerkship." ], "LABELS": [ "BACKGROUND", "METHODS", "RESULTS" ], "MESHES": [ "Attitude of Health Personnel", "Clinical Clerkship", "Curriculum", "Humans", "Medication Errors", "Pediatrics", "Program Evaluation", "Safety Management", "Students, Medical", "Surveys and Questionnaires" ], "YEAR": "2011", "reasoning_required_pred": "yes", "reasoning_free_pred": "yes", "final_decision": "yes", "LONG_ANSWER": "An active learning curriculum integrated into a clinical clerkship can change learners' attitudes towards patient safety. Students found the curriculum relevant and recommended its continuation." }, "23972333": { "QUESTION": "Has the prevalence of health care services use increased over the last decade (2001-2009) in elderly people?", "CONTEXTS": [ "(1) To describe the prevalence of general practitioner visits and hospitalization according to sex and age groups; (2) to identify which factors are independently associated with a higher use of health care services among elderly Spanish; and (3) to study the time trends in the prevalence of use of health care services 2001-2009.", "Observational study. We analyzed data from the Spanish National Health Surveys conducted in 2001 (n=21,058), 2003 (n=21,650), 2006 (n=29,478) and 2009 (n=22,188). We included responses from adults aged 65 years and older.", "The main variables were the number of general practitioner visits in the last 4 weeks and hospitalization in the past year. We stratified the adjusted models by the main variables. We analyzed socio-demographic characteristics, health related variables, using multivariate logistic regression models.", "The total number of subjects was 24,349 (15,041 woman, 9309 men). Women were significantly older than men (P<0.001). Women had higher prevalence of general practitioner visits than men in all surveys. Men had significantly higher prevalence of hospitalizations than women in the years 2001, 2006 and 2009. When we adjusted the hospitalization by possible confounders using logistic regressions, men had a higher probability of being hospitalized than women (OR 1.53, 1.39-1.69). The variables that were significantly associated with a higher use of health care services were lower educational level, worse self-rated health, chronic conditions, polypharmacy, and the level of disability. The number of general practitioner visits among women and men significantly increased from 2001 to 2009 (women: OR 1.43, 1.27-1.61; men: OR 1.71, 1.49-1.97)." ], "LABELS": [ "OBJECTIVES", "STUDY DESIGN", "OUTCOME MEASURES", "RESULTS" ], "MESHES": [ "Aged", "Aged, 80 and over", "Female", "Health Care Surveys", "Health Services", "Health Status", "Hospitalization", "Humans", "Logistic Models", "Male", "Odds Ratio", "Office Visits", "Patient Acceptance of Health Care", "Polypharmacy", "Prevalence", "Self Report", "Socioeconomic Factors", "Spain" ], "YEAR": "2013", "reasoning_required_pred": "yes", "reasoning_free_pred": "yes", "final_decision": "yes", "LONG_ANSWER": "The current study revealed an increase in health care services utilization from 2001 to 2009 in the older Spanish population." }, "23719685": { "QUESTION": "Does high-dose radiotherapy benefit palliative lung cancer patients?", "CONTEXTS": [ "The present analysis compares two palliative treatment concepts for lung cancer in terms of overall survival.", "Survival data from 207\u00a0patients were used in a retrospective analysis. All patients received palliative treatment comprising either 25\u00a0Gy applied in 5\u00a0fractions or 50\u00a0Gy in 20\u00a0fractions. A subgroup analysis was performed to compare patients with a good-fair vs. poor overall condition.", "Median survival times were 21\u00a0weeks (range\u00a06-26\u00a0weeks) for patients treated with 25\u00a0Gy in 5\u00a0fractions and 23\u00a0weeks (range\u00a014.5-31.5\u00a0weeks) for patients treated with 50\u00a0Gy in 20\u00a0fractions (95\u2009% confidence interval, CI; p\u2009=\u20090.334). For patients with a good-fair overall condition, median survival times were 30\u00a0weeks (21.8-39.2\u00a0weeks) for 25\u00a0Gy in 5\u00a0fractions and 28\u00a0weeks (14.2-41.8\u00a0weeks) for 50\u00a0Gy in 20\u00a0fractions (CI 95\u2009%, p\u2009=\u20090.694). In patients with a poor overall condition, these values were 18\u00a0weeks (14.5-21.5\u00a0weeks) and 21\u00a0weeks (13.0-29.0\u00a0weeks), respectively (CI 95\u2009%, p\u2009=\u20090.248)." ], "LABELS": [ "BACKGROUND AND PURPOSE", "PATIENTS AND METHODS", "RESULTS" ], "MESHES": [ "Adult", "Aged", "Aged, 80 and over", "Bronchial Neoplasms", "Dose Fractionation", "Dose-Response Relationship, Radiation", "Female", "Germany", "Humans", "Incidence", "Male", "Middle Aged", "Palliative Care", "Risk Factors", "Survival Analysis", "Survival Rate", "Treatment Outcome" ], "YEAR": "2013", "reasoning_required_pred": "yes", "reasoning_free_pred": "no", "final_decision": "no", "LONG_ANSWER": "The palliative treatment concept of 25\u00a0Gy applied in 5\u00a0fractions is sufficient for radiation of lung cancer, given that there was no obvious survival improvement in patients treated with the higher total dose regimen." }, "11601252": { "QUESTION": "Is endothelin-1 an aggravating factor in the development of acute pancreatitis?", "CONTEXTS": [ "We have reported previously that cerulein-induced edematous pancreatitis would transform into hemorrhagic pancreatitis by administration of endothelin-1 in rats. In the present study, we tried to protect rat model from developing into hemorrhagic pancreatitis with BQ123 (an ETA receptor antagonist).", "The rat model was made by 5-hour restraint water-immersion stress and two intraperitoneal injections of cerulein (40 micrograms/kg) at hourly interval. BQ123 (3 or 6 mg/kg) was administered intravenously 30 minutes before and 2 hours after the first cerulein injection.", "Acute hemorrhagic pancreatitis was induced in all rats treated with cerulin + stress. The score for pancreatic hemorrhage was 2.4 +/- 0.2 in this group. In the rats pretreated with BQ123, the score was reduced to 1.0 +/- 0.0, pancreas wet weight and serum amylase activity were significantly reduced, and histologic alterations in the pancreas lightened, also the local pancreatic blood flow improved without affecting the systemic blood pressure." ], "LABELS": [ "OBJECTIVE", "METHODS", "RESULTS" ], "MESHES": [ "Acute Disease", "Animals", "Ceruletide", "Endothelin Receptor Antagonists", "Endothelin-1", "Male", "Pancreatitis", "Peptides, Cyclic", "Rats", "Rats, Sprague-Dawley" ], "YEAR": "1999", "reasoning_required_pred": "yes", "reasoning_free_pred": "yes", "final_decision": "yes", "LONG_ANSWER": "These results suggest that endothelin-1 should play a role in aggravating the development of acute hemorrhagic pancreatitis, through its action on the pancreatic microcirculation." }, "23506394": { "QUESTION": "Malnutrition, a new inducer for arterial calcification in hemodialysis patients?", "CONTEXTS": [ "Arterial calcification is a significant cardiovascular risk factor in hemodialysis patients. A series of factors are involved in the process of arterial calcification; however, the relationship between malnutrition and arterial calcification is still unclear.", "68 hemodialysis patients were enrolled in this study. Nutrition status was evaluated using modified quantitative subjective global assessment (MQSGA). Related serum biochemical parameters were measured. And the radial artery samples were collected during the arteriovenous fistula surgeries. Hematoxylin/eosin stain was used to observe the arterial structures while Alizarin red stain to observe calcified depositions and classify calcified degree. The expressions of bone morphogenetic protein 2 (BMP2) and matrix Gla protein (MGP) were detected by immunohistochemistry and western blot methods.", "66.18% hemodialysis patients were malnutrition. In hemodialysis patients, the calcified depositions were mainly located in the medial layer of the radial arteries and the expressions of BMP2 and MGP were both increased in the calcified areas. The levels of serum albumin were negatively associated with calcification score and the expressions of BMP2 and MGP. While MQSGA score, serum phosphorus and calcium\u2009\u00d7\u2009phosphorus product showed positive relationships with calcification score and the expressions of BMP2 and MGP." ], "LABELS": [ "BACKGROUND", "METHODS", "RESULTS" ], "MESHES": [ "Arteries", "Blotting, Western", "Bone Morphogenetic Protein 2", "Calcinosis", "Calcium", "Calcium-Binding Proteins", "Extracellular Matrix Proteins", "Humans", "Immunohistochemistry", "Malnutrition", "Phosphorus", "Renal Dialysis", "Serum Albumin" ], "YEAR": "2013", "reasoning_required_pred": "yes", "reasoning_free_pred": "yes", "final_decision": "yes", "LONG_ANSWER": "Malnutrition is prevalent in hemodialysis patients and is associated with arterial calcification and the expressions of BMP2 and MGP in calcified radial arteries. Malnutrition may be a new inducer candidate for arterial calcification in hemodialysis patients." }, "19575307": { "QUESTION": "Does glomerular hyperfiltration in pregnancy damage the kidney in women with more parities?", "CONTEXTS": [ "We aimed to investigate the glomerular hyperfiltration due to pregnancy in women with more parities.", "Five hundred women aged 52.57 +/- 8.08 years, without a history of hypertension, diabetes mellitus or complicated pregnancy were involved in the study. They were divided into three groups. Group 1: women with no or one parity (n = 76); group 2: women with two or three parities (n = 333); group 3: women with four or more parities (n = 91). Laboratory parameters and demographical data were compared between the three groups.", "Mean age, serum urea and serum creatinine were similar between three groups. Patients in group 3 had significantly higher GFR values compared to groups 1 and 2 (109.44 +/- 30.99, 110.76 +/- 30.22 and 121.92 +/- 34.73 mL/min/1.73 m(2) for groups 1, 2 and 3, respectively; P = 0.008 for group 1 vs group 3; P = 0.002 for group 2 vs group 3)." ], "LABELS": [ "OBJECTIVES", "METHODS", "RESULTS" ], "MESHES": [ "Adult", "Aged", "Analysis of Variance", "Body Mass Index", "Cohort Studies", "Confidence Intervals", "Creatinine", "Female", "Glomerular Filtration Rate", "Humans", "Kidney Diseases", "Kidney Function Tests", "Middle Aged", "Parity", "Pregnancy", "Pregnancy Complications", "Risk Factors", "Sensitivity and Specificity", "Urea", "Uric Acid", "Urinalysis" ], "YEAR": "2009", "reasoning_required_pred": "no", "reasoning_free_pred": "no", "final_decision": "no", "LONG_ANSWER": "In our study, we suggest that glomerular hyperfiltration due to pregnancy does not have adverse effects on kidney in women with more parities. Pregnancy may have possible protective mechanisms for kidney against adverse effects of glomerular hyperfiltration." }, "11977907": { "QUESTION": "Subclavian steal syndrome: can the blood pressure difference between arms predict the severity of steal?", "CONTEXTS": [ "A side-to-side difference in systolic brachial arterial blood pressure is a common finding in subclavian artery stenosis and is frequently used as a screening tool for subclavian steal syndrome (SSS). It was the goal of this retrospective study to investigate the relationship between different vertebral artery waveform types and the side-to-side difference in systolic blood pressure in patients with sonographically proven SSS.", "The records of 1860 patients from the Neuroultrasound Laboratory between January 2000 and December 2000 were screened for the diagnosis of SSS in the final ultrasound report. In all patients, bilateral brachial arterial blood pressure was measured in a sitting position prior to the ultrasound examination. Vertebral artery waveforms were classified as (1) systolic deceleration, (2) alternating flow, and (3) complete reversal at rest. Blood pressure difference as calculated by normal-side blood pressure minus lesion-side blood pressure was compared with the 3 Doppler waveform types.", "SSS was found in 51 of 1860 (2.7%) ultrasonography studies of 49 patients (17 men, 32 women; mean age 65.3 +/- 10.5 years). Two patients (4%) had bilateral SSS. In 3 patients (6%), SSS was related to an innominate artery stenosis. Waveform analysis showed a completely reversed flow in 16 (31%), an alternating flow in 24 (47%), and a systolic deceleration in 11 (22%) cases. Systolic blood pressure difference was significantly higher in the complete reversal and alternating groups than in the systolic deceleration group (P<.001)." ], "LABELS": [ "BACKGROUND AND PURPOSE", "METHODS", "RESULTS" ], "MESHES": [ "Aged", "Analysis of Variance", "Arm", "Blood Flow Velocity", "Blood Pressure", "Female", "Hemodynamics", "Humans", "Male", "Retrospective Studies", "Risk Factors", "Subclavian Steal Syndrome", "Ultrasonography, Doppler", "Vertebral Artery" ], "YEAR": "2002", "reasoning_required_pred": "yes", "reasoning_free_pred": "yes", "final_decision": "yes", "LONG_ANSWER": "Brachial systolic blood pressure difference is related to the severity of SSS and can be used as a screening tool for SSS. However, it performed better in severe steal than milder steal phenomena." }, "12626177": { "QUESTION": "Can the Internet be used to improve sexual health awareness in web-wise young people?", "CONTEXTS": [ "To assess Internet use amongst young people to determine whether it would be a practical way to provide sex education and information.", "Year 10 students (aged 14-15 years) from North Nottinghamshire schools were asked to participate in focus groups to discuss the Internet. A series of predefined questions were directed to the whole group to generate debate. Areas explored included: Internet access and site; frequency and purpose of Internet use; websites visited; ideas for a genitourinary medicine (GUM) website. Responses were recorded by a hand count or as individual verbal responses.", "Thirteen focus groups were held involving 287 students of approximately equal sex distribution. All had access to Internet facilities at school and 224 (78.0%) had access elsewhere. Access was at least once a week by 178 (62.0%) mostly for e-mail, games, chatlines and homework. No one accessed for health information. One hundred and seventy-nine (62.4%) participants said they would use a GUM website. A 'question line' where they could e-mail questions to a health care professional was of interest to 202 (70.4%) participants." ], "LABELS": [ "OBJECTIVES", "METHODS", "RESULTS" ], "MESHES": [ "Adolescent", "England", "Female", "Focus Groups", "Health Education", "Humans", "Information Dissemination", "Internet", "Male", "Public Health", "Reproductive Medicine", "Safe Sex", "School Health Services", "Sex Education" ], "YEAR": "2003", "reasoning_required_pred": "maybe", "reasoning_free_pred": "yes", "final_decision": "maybe", "LONG_ANSWER": "The Internet would be a practical and accessible way of delivering sexual health education to young people, particularly if it is incorporated into activities and websites they enjoy." }, "15483019": { "QUESTION": "Is eligibility for a chemotherapy protocol a good prognostic factor for invasive bladder cancer after radical cystectomy?", "CONTEXTS": [ "To assess whether eligibility to an adjuvant chemotherapy protocol in itself represents a good prognostic factor after radical cystectomy for bladder cancer.", "Between April 1984 and May 1989, our institution entered 35 patients with invasive bladder cancer into the Swiss Group for Clinical and Epidemiological Cancer Research (SAKK) study 09/84. They were randomly assigned to either observation or three postoperative courses of cisplatin monotherapy after cystectomy. This study had a negative result. The outcome of these 35 patients (protocol group) was compared with an age- and tumor-stage-matched cohort (matched group; n = 35) who also underwent cystectomy during the same period, but were not entered into the SAKK study, as well as the remaining 57 patients treated during the study period for the same indication (remaining group).", "Median overall survival decreased from 76.3 months in the protocol group to 52.1 months in the matched group and to 20.3 months in the remaining group. The respective times of median recurrence-free survival were 67.2, 16.0, and 9.4 months. Tumor progression occurred in 46% of the protocol group compared with 69% in the matched group and 65% in the remaining group (P<.05). Cancer-related death was noted in 40% of the protocol group, 57% in the matched group, and 56% in the remaining group." ], "LABELS": [ "PURPOSE", "PATIENTS AND METHODS", "RESULTS" ], "MESHES": [ "Aged", "Antineoplastic Agents", "Case-Control Studies", "Chemotherapy, Adjuvant", "Cisplatin", "Cohort Studies", "Combined Modality Therapy", "Cystectomy", "Disease-Free Survival", "Female", "Humans", "Male", "Prognosis", "Survival Rate", "Urinary Bladder Neoplasms" ], "YEAR": "2004", "reasoning_required_pred": "yes", "reasoning_free_pred": "yes", "final_decision": "yes", "LONG_ANSWER": "These data suggest that being willing and fit enough for a chemotherapy protocol is a good prognostic factor for invasive bladder cancer. This eligibility bias emphasizes the need for prospective, randomized trials, and indicates that single-group studies using historical or matched controls have to be interpreted with caution." }, "18693227": { "QUESTION": "Does a geriatric oncology consultation modify the cancer treatment plan for elderly patients?", "CONTEXTS": [ "This study was performed to describe the treatment plan modifications after a geriatric oncology clinic. Assessment of health and functional status and cancer assessment was performed in older cancer patients referred to a cancer center.", "Between June 2004 and May 2005, 105 patients 70 years old or older referred to a geriatric oncology consultation at the Institut Curie cancer center were included. Functional status, nutritional status, mood, mobility, comorbidity, medication, social support, and place of residence were assessed. Oncology data and treatment decisions were recorded before and after this consultation. Data were analyzed for a possible correlation between one domain of the assessment and modification of the treatment plan.", "Patient characteristics included a median age of 79 years and a predominance of women with breast cancer. About one half of patients had an independent functional status. Nearly 15% presented severe undernourishment. Depression was suspected in 53.1% of cases. One third of these patients had>2 chronic diseases, and 74% of patients took>or =3 medications. Of the 93 patients with an initial treatment decision, the treatment plan was modified for 38.7% of cases after this assessment. Only body mass index and the absence of depressive symptoms were associated with a modification of the treatment plan." ], "LABELS": [ "BACKGROUND", "PATIENTS AND METHODS", "RESULTS" ], "MESHES": [ "Activities of Daily Living", "Affect", "Aged", "Aged, 80 and over", "Cancer Care Facilities", "Female", "Geriatric Assessment", "Humans", "Male", "Medical Oncology", "Neoplasms", "Referral and Consultation" ], "YEAR": "2008", "reasoning_required_pred": "yes", "reasoning_free_pred": "yes", "final_decision": "yes", "LONG_ANSWER": "The geriatric oncology consultation led to a modification of the cancer treatment plan in more than one third of cases. Further studies are needed to determine whether these modifications improve the outcome of these older patients." }, "27131771": { "QUESTION": "Does left atrial appendage (LAA) occlusion device alter the echocardiography and electrocardiogram parameters in patients with atrial fibrillation?", "CONTEXTS": [ "The alterations of echocardiography and electrocardiogram (ECG) in patients received left atrial appendage LAA occlusion therapy are still unclear. The present study was to evaluate the influence of LAA occlusion device on echocardiography and ECG changes in patients with atrial fibrillation (AF).", "Seventy-three patients who had undergone Watchman, LAmbre and Lefort were enrolled in this study. Echocardiography and ECG results at pre- and post-operation were collected. Besides, echocardiography was also performed during follow-up visits at 1, 6 and 12months after discharge.", "After LAA occlusion, a slight and measureable movement of QRS electric axis was observed in most patients. The significant differences were also observed in heart rate (HR) and the mean-mean QT interval between pre- and post-operation for all patients. There existed no significant difference in echocardiographic parameters between before and after device implantation. However, a larger left atrial (LA) diameter was detected by echocardiography during follow-up visit at 6months when compared with pre-operation parameters. Similarly, aortic root diameter (ARD) was also larger during follow-up at 12months than the baseline dimension in pre-operation." ], "LABELS": [ "BACKGROUND", "METHODS", "RESULTS" ], "MESHES": [ "Aged", "Aged, 80 and over", "Atrial Appendage", "Atrial Fibrillation", "Cardiac Catheterization", "Cardiac Surgical Procedures", "Echocardiography", "Electrocardiography", "Female", "Humans", "Male", "Middle Aged", "Prostheses and Implants", "Treatment Outcome" ], "YEAR": "2016", "reasoning_required_pred": "yes", "reasoning_free_pred": "yes", "final_decision": "yes", "LONG_ANSWER": "LAA occlusion device resulted in a slightly movement in QRS axis, reduced HR and increased the mean-mean QT interval duration. In addition, LA diameter and ARD seemed to be larger after device implantation." }, "24783217": { "QUESTION": "Can ki-67 play a role in prediction of breast cancer patients' response to neoadjuvant chemotherapy?", "CONTEXTS": [ "Currently the choice of breast cancer therapy is based on prognostic factors. The proliferation marker Ki-67 is used increasingly to determine the method of therapy. The current study analyses the predictive value of Ki-67 in foreseeing breast cancer patients' responses to neoadjuvant chemotherapy.", "This study includes patients with invasive breast cancer treated between 2008 and 2013. The clinical response was assessed by correlating Ki-67 to histological examination, mammography, and ultrasonography findings.", "The average Ki-67 value in our patients collectively (n = 77) is 34.9 \u00b1 24.6%. The average Ki-67 value is the highest with 37.4 \u00b1 24.0% in patients with a pCR. The Ki-67 values do not differ significantly among the 3 groups: pCR versus partial pathological response versus stable disease/progress (P = 0.896). However, Ki-67 values of patients with luminal, Her2 enriched, and basal-like cancers differed significantly from each other. Furthermore, within the group of luminal tumors Ki-67 values of patients with versus without pCR also differed significantly." ], "LABELS": [ "BACKGROUND", "METHODS", "RESULTS" ], "MESHES": [ "Adult", "Breast Neoplasms", "Female", "Humans", "Ki-67 Antigen", "Middle Aged", "Neoadjuvant Therapy", "Receptor, ErbB-2", "Retrospective Studies" ], "YEAR": "2014", "reasoning_required_pred": "yes", "reasoning_free_pred": "yes", "final_decision": "yes", "LONG_ANSWER": "Our data shows that the Ki-67 value predicts the response to neoadjuvant chemotherapy as a function of the molecular subtype, reflecting the daily routine concerning Ki-67 and its impressing potential and limitation as a predictive marker for neoadjuvant chemotherapy response." }, "17916877": { "QUESTION": "Intravenous administration of metoclopramide by 2 min bolus vs 15 min infusion: does it affect the improvement of headache while reducing the side effects?", "CONTEXTS": [ "To determine the therapeutic effect (alleviation of vascular type headache) and side effects of a slow intravenous metoclopramide infusion over 15 min compared with those effects of a bolus intravenous metoclopramide infusion over 2 min in the treatment of patients with recent onset vascular type headache.", "All adults treated with metoclopramide for vascular type headache were eligible for entry into this clinical randomised double blinded trial. This study compared the effects of two different rates of intravenous infusion of metoclopramide over a period of 13 months at a university hospital emergency department. During the trial, side effects and headache scores were recorded at baseline (0 min), and then at 5, 15, 30 and 60 min. Repeated measures analysis of variance was used to compare the medication's efficacy and side effects.", "A total of 120 patients presenting to the emergency department met the inclusion criteria. Of these, 62 patients (51.7%) were given 10 mg metoclopramide as a slow intravenous infusion over 15 min (SIG group) and 58 patients (48.3%) were given 10 mg metoclopramide intravenous bolus infusion over 2 min (BIG group). 17 of the 58 patients in the BIG group (29.3%) and 4 of the 62 patients (6.5%) in the SIG group had akathisia (p = 0.001). There were no significant differences between the BIG and SIG groups in terms of mean headache scores (p = 0.34) and no adverse reactions in the study period. Metoclopramide successfully relieved the headache symptom(s) of patients in both the BIG and SIG groups." ], "LABELS": [ "OBJECTIVE", "MATERIAL AND METHODS", "RESULTS" ], "MESHES": [ "Adult", "Akathisia, Drug-Induced", "Dopamine Antagonists", "Double-Blind Method", "Drug Administration Schedule", "Emergency Service, Hospital", "Female", "Humans", "Infusions, Intravenous", "Male", "Metoclopramide", "Vascular Headaches" ], "YEAR": "2007", "reasoning_required_pred": "yes", "reasoning_free_pred": "yes", "final_decision": "yes", "LONG_ANSWER": "Slowing the infusion rate of metoclopramide is an effective strategy for the improvement of headache and reducing the incidence of akathisia in patients with vascular type headache." }, "8375607": { "QUESTION": "Is the breast best for children with a family history of atopy?", "CONTEXTS": [ "Previous studies reported that breast-feeding protects children against a variety of diseases, but these studies were generally conducted on \"high-risk\" or hospitalized children. This paper describes the results of our study on the effects of breast-feeding on rate of illness in normal children with a family history of atopy.", "A historic cohort approach of 794 children with a family history of atopy was used to assess the effects of breast-feeding on illness rates. Family history of atopy was based on allergic diseases in family members as registered by the family physician. Illness data from birth onwards were available from the Continuous Morbidity Registration of the Department of Family Medicine. Information on breast-feeding was collected by postal questionnaire. We then compared rates of illness between children with a family history of atopy who were and who were not breast-fed.", "Breast-feeding was related to lower levels of childhood illness both in the first and the first three years of life. In the first year of life they had fewer episodes of gastroenteritis, lower respiratory tract infections, and digestive tract disorders. Over the next three years of life they had fewer respiratory tract infections and skin infections." ], "LABELS": [ "BACKGROUND", "METHODS", "RESULTS" ], "MESHES": [ "Breast Feeding", "Cohort Studies", "Humans", "Hypersensitivity, Immediate", "Infant, Newborn", "Morbidity", "Risk Factors" ], "YEAR": null, "reasoning_required_pred": "yes", "reasoning_free_pred": "yes", "final_decision": "yes", "LONG_ANSWER": "Our results suggest a protective effect of breast-feeding among children with a family history of atopy that is not confined to the period of breast-feeding but continues during the first three years of life. Breast-feeding should be promoted in children with a family history of atopy." }, "11955750": { "QUESTION": "Does escalation of the apical dose change treatment outcome in beta-radiation of posterior choroidal melanomas with 106Ru plaques?", "CONTEXTS": [ "To show the results of treating posterior uveal melanomas with 106Ru plaque beta-ray radiotherapy and to review and discuss the literature concerning the optimal apical dose prescription (100 vs. 160 Gy).", "Forty-eight patients with uveal melanomas (median height 3.85 mm + 1 mm sclera) were treated with ruthenium plaques. The median apical dose was 120 Gy, the median scleral dose 546 Gy.", "After 5.8 years of follow-up, the overall 5-year survival rate was 90%, the disease specific 5-year survival rate was 92% (3 patients alive with metastasis). Six percent received a second ruthenium application, 10% of the eyes had to be enucleated. Local control was achieved in 90% of the patients with conservative therapy alone. Central or paracentral tumors showed 50% of the pretherapeutic vision after 4 years, and 80% of the vision was preserved in those with peripheral tumors. The main side effects were mostly an uncomplicated retinopathy (30%); macular degeneration or scarring led to poor central vision in 30% of cases." ], "LABELS": [ "PURPOSE", "METHODS AND MATERIALS", "RESULTS" ], "MESHES": [ "Adult", "Aged", "Aged, 80 and over", "Choroid Neoplasms", "Female", "Humans", "Male", "Melanoma", "Middle Aged", "Radiotherapy Dosage", "Ruthenium Radioisotopes", "Survival Rate", "Visual Acuity" ], "YEAR": "2002", "reasoning_required_pred": "yes", "reasoning_free_pred": "no", "final_decision": "no", "LONG_ANSWER": "Brachytherapy with ruthenium applicators is an effective therapy for small- and medium-size posterior uveal melanomas. Our results are comparable to other series. The treatment outcome does not seem to be capable of improvement by increasing the apical dose. An internationally accepted model for defining the dosage in brachytherapy is needed." }, "27040842": { "QUESTION": "Does septoplasty change the dimensions of compensatory hypertrophy of the middle turbinate?", "CONTEXTS": [ "To measure the dimensions of compensatory hypertrophy of the middle turbinate in patients with nasal septal deviation, before and after septoplasty.", "The mucosal and bony structures of the middle turbinate and the angle of the septum were measured using radiological analysis before septoplasty and at least one year after septoplasty. All pre- and post-operative measurements of the middle turbinate were compared using the paired sample t-test and Wilcoxon rank sum test.", "The dimensions of bony and mucosal components of the middle turbinate on concave and convex sides of the septum were not significantly changed by septoplasty. There was a significant negative correlation after septoplasty between the angle of the septum and the middle turbinate total area on the deviated side (p = 0.033)." ], "LABELS": [ "OBJECTIVE", "METHODS", "RESULTS" ], "MESHES": [ "Adolescent", "Adult", "Aged", "Female", "Humans", "Hypertrophy", "Male", "Middle Aged", "Nasal Mucosa", "Nasal Obstruction", "Nasal Septum", "Nose Deformities, Acquired", "Retrospective Studies", "Rhinoplasty", "Tomography, X-Ray Computed", "Turbinates", "Young Adult" ], "YEAR": "2016", "reasoning_required_pred": "maybe", "reasoning_free_pred": "no", "final_decision": "no", "LONG_ANSWER": "The present study findings suggest that compensatory hypertrophy of the middle turbinate is not affected by septoplasty, even after one year." }, "17244100": { "QUESTION": "Can patient coaching reduce racial/ethnic disparities in cancer pain control?", "CONTEXTS": [ "Minority patients with cancer experience worse control of their pain than do their white counterparts. This disparity may, in part, reflect more miscommunication between minority patients and their physicians. Therefore, we examined whether patient coaching could reduce disparities in pain control in a secondary analysis of a randomized controlled trial.", "Sixty-seven English-speaking adult cancer outpatients, including 15 minorities, with moderate pain over the prior 2 weeks were randomly assigned to the experimental (N = 34) or control group (N = 33). Experimental patients received a 20-minute individualized education and coaching session to increase knowledge of pain self-management, to redress personal misconceptions about pain treatment, and to rehearse an individually scripted patient-physician dialog about pain control. The control group received standardized information on controlling pain. Data on average pain (0-10 scale) were collected at enrollment and 2-week follow-up.", "At enrollment, minority patients had significantly more pain than their white counterparts (6.0 vs 5.0, P = 0.05). At follow-up, minorities in the control group continued to have more pain (6.4 vs 4.7, P = 0.01), whereas in the experimental group, disparities were eliminated (4.0 vs 4.3, P = 0.71). The effect of the intervention on reducing disparities was significant (P = 0.04)." ], "LABELS": [ "PURPOSE", "METHODS", "RESULTS" ], "MESHES": [ "Communication", "Communication Barriers", "Female", "Humans", "Male", "Middle Aged", "Minority Groups", "Neoplasms", "Pain", "Pain Management", "Patient Education as Topic", "Physician-Patient Relations", "Randomized Controlled Trials as Topic" ], "YEAR": null, "reasoning_required_pred": "yes", "reasoning_free_pred": "yes", "final_decision": "yes", "LONG_ANSWER": "Patient coaching offers promise as a means of reducing racial/ethnic disparities in pain control. Larger studies are needed to validate these findings and to explore possible mechanisms." }, "15708048": { "QUESTION": "Does prior benign prostate biopsy predict outcome for patients treated with radical perineal prostatectomy?", "CONTEXTS": [ "To determine the effect of prior benign prostate biopsies on the surgical and clinical outcomes of patients treated with radical perineal prostatectomy for prostate cancer.", "A total of 1369 patients with clinically localized prostate cancer underwent radical prostatectomy by a single surgeon between 1991 and 2001. A subset of 203 patients (14.9%), who had undergone at least one prior benign prostate biopsy for a rising prostate-specific antigen and/or abnormal digital rectal examination, constituted our study population. A total of 1115 patients with no prior biopsy represented our control group. After prostatectomy, patients were evaluated at 6-month intervals for biochemical evidence of recurrence, defined as a prostate-specific antigen level of 0.5 ng/mL or greater.", "Patients with a prior benign biopsy had more favorable pathologic features with more organ-confined (74% versus 64%; P<0.001) and less margin-positive (9.8% versus 18%) disease. Only 24 patients (12%) in the study group (versus 20% in control group; P = 0.01) had eventual evidence of biochemical failure. Kaplan-Meier analyses suggested that patients with prior benign biopsies have improved biochemical disease-free survival, especially for those with more aggressive disease (Gleason sum 7 or greater; P<0.01). Overall, patients in the study group had lower probability (odds ratio 0.57, P<0.001) of biochemical failure compared with those in the control group." ], "LABELS": [ "OBJECTIVES", "METHODS", "RESULTS" ], "MESHES": [ "Adenocarcinoma", "Aged", "Biomarkers, Tumor", "Biopsy", "Cohort Studies", "Disease-Free Survival", "Follow-Up Studies", "Humans", "Life Tables", "Male", "Middle Aged", "Neoplasm Proteins", "Proportional Hazards Models", "Prostate", "Prostate-Specific Antigen", "Prostatectomy", "Prostatic Neoplasms", "Treatment Outcome" ], "YEAR": "2005", "reasoning_required_pred": "yes", "reasoning_free_pred": "yes", "final_decision": "yes", "LONG_ANSWER": "A prior benign prostate biopsy may be independently associated with more favorable surgical and biochemical outcomes after prostatectomy. Additional studies are needed to confirm these findings." }, "11438275": { "QUESTION": "Does patient position during liver surgery influence the risk of venous air embolism?", "CONTEXTS": [ "It is generally believed that positioning of the patient in a head-down tilt (Trendelenberg position) decreases the likelihood of a venous air embolism during liver resection.", "The physiological effect of variation in horizontal attitude on central and hepatic venous pressure was measured in 10 patients during liver surgery. Hemodynamic indices were recorded with the operating table in the horizontal, 20 degrees head-up and 20 degrees head-down positions.", "There was no demonstrable pressure gradient between the hepatic and central venous levels in any of the positions. The absolute pressures did, however, vary in a predictable way, being highest in the head-down and lowest during head-up tilt. However, on no occasion was a negative intraluminal pressure recorded." ], "LABELS": [ "BACKGROUND", "METHODS", "RESULTS" ], "MESHES": [ "Adult", "Aged", "Central Venous Pressure", "Embolism, Air", "Female", "Head-Down Tilt", "Hepatectomy", "Hepatic Veins", "Humans", "Male", "Middle Aged", "Posture", "Risk Factors", "Vena Cava, Inferior", "Venous Pressure" ], "YEAR": "2001", "reasoning_required_pred": "no", "reasoning_free_pred": "no", "final_decision": "no", "LONG_ANSWER": "The effect on venous pressures caused by the change in patient positioning alone during liver surgery does not affect the risk of venous air embolism." }, "22513023": { "QUESTION": "Do Indigenous Australians age prematurely?", "CONTEXTS": [ "To assess whether Indigenous Australians age prematurely compared with other Australians, as implied by Australian Government aged care policy, which uses age 50 years and over for population-based planning for Indigenous people compared with 70 years for non-indigenous people.", "Cross-sectional analysis of aged care assessment, hospital and health survey data comparing Indigenous and non-indigenous age-specific prevalence of health conditions. Analysis of life tables for Indigenous and non-indigenous populations comparing life expectancy at different ages.", "At age 63 for women and age 65 for men, Indigenous people had the same life expectancy as non-indigenous people at age 70. There is no consistent pattern of a 20-year lead in age-specific prevalence of age-associated conditions for Indigenous compared with other Australians. There is high prevalence from middle-age onwards of some conditions, particularly diabetes (type unspecified), but there is little or no lead for others." ], "LABELS": [ "OBJECTIVE", "METHODS", "RESULTS" ], "MESHES": [ "Adult", "Aged", "Aged, 80 and over", "Australia", "Cross-Sectional Studies", "Geriatric Assessment", "Geriatric Nursing", "Health Policy", "Health Status Indicators", "Humans", "Life Expectancy", "Life Tables", "Middle Aged", "Oceanic Ancestry Group" ], "YEAR": "2012", "reasoning_required_pred": "no", "reasoning_free_pred": "no", "final_decision": "no", "LONG_ANSWER": "The idea that Indigenous people age prematurely is not well supported by this study of a series of discrete conditions. The current focus and type of services provided by the aged care sector may not be the best way to respond to the excessive burden of chronic disease and disability of middle-aged Indigenous people." }, "27146470": { "QUESTION": "Are tuberculosis patients adherent to prescribed treatments in China?", "CONTEXTS": [ "Tuberculosis (TB) patients face numerous difficulties adhering to the long-term, rigorous TB treatment regimen. Findings on TB patients' treatment adherence vary across existing literature and official reports. The present study attempted to determine the actual treatment adherence of new TB patients and to identify factors leading to non-adherence.", "A prospective cohort of 481 newly confirmed TB patients from three counties in western China were enrolled during June to December 2012 and was followed until June 2013. Patients who missed at least one dose of drugs or one follow-up re-examination during the treatment course were deemed as non-adherent. Influencing factors were identified using a logistic regression model.", "A total of 173 (36.0 %) patients experienced non-adherence and the loss to follow-up cases reached 136 (28.2 %). Only 13.9 % of patients took drugs under direct observation, and 60.5 % of patients were supervised by phone calls. Factor analyses suggested that patients who were observed by family members (OR:5.54, 95 % CI:2.87-10.69) and paying monthly service expenses above 450 RMB (OR:2.08, 95 % CI:1.35-3.19) were more likely to be non-adherent, while supervision by home visit (OR:0.06, 95 % CI:0.01-0.28) and phone calls (OR:0.27, 95 % CI:0.17-0.44) were protective factors." ], "LABELS": [ "BACKGROUND", "METHODS", "RESULTS" ], "MESHES": [ "Adolescent", "Adult", "Aged", "Aged, 80 and over", "Antitubercular Agents", "China", "Female", "Humans", "Lost to Follow-Up", "Male", "Medication Adherence", "Middle Aged", "Patients", "Prospective Studies", "Tuberculosis", "Young Adult" ], "YEAR": "2016", "reasoning_required_pred": "maybe", "reasoning_free_pred": "no", "final_decision": "maybe", "LONG_ANSWER": "Despite recent efforts, a large proportion of newly confirmed TB patients could not adhere to standard TB treatment, and patients' lost to follow-up was still a serious problem. Poor treatment supervision and heavy financial burden might be the main causes for non-adherence. More needs to be done to enhance treatment supervision policies and financial supports to both health providers and TB patients." }, "8165771": { "QUESTION": "Ultrasound in squamous cell carcinoma of the penis; a useful addition to clinical staging?", "CONTEXTS": [ "As part of the staging procedure in squamous cell carcinoma of the penis, we assessed the role of ultrasound examination, in particular its role in assessing the extent and the invasion into the corpora.", "From 1988 until 1992, all patients referred for primary treatment underwent ultrasound assessment with a 7.5 MHz linear array small parts transducer as part of the clinical workup. All ultrasound images were reviewed by one radiologist, without knowledge of the clinical outcome and were compared with the results obtained at histopathologic examination.", "In 16 patients the primary tumor and in 1 patient a recurrent cancer after primary therapy were examined. All tumors were identified as hypoechoic lesions. Ultrasound examination in the region of the glans was not able to differentiate between invasion of the subepithelial tissue and invasion into the corpus spongiosum, but absence or presence of invasion into the tunica albuginea of the corpus cavernosum was clearly demonstrated. Accurate measurement by ultrasound of maximum tumor thickness was seen in seven of sixteen examinations." ], "LABELS": [ "OBJECTIVE", "METHODS", "RESULTS" ], "MESHES": [ "Carcinoma, Squamous Cell", "Humans", "Male", "Middle Aged", "Neoplasm Invasiveness", "Neoplasm Staging", "Penile Neoplasms", "Penis", "Ultrasonography" ], "YEAR": "1994", "reasoning_required_pred": "yes", "reasoning_free_pred": "maybe", "final_decision": "yes", "LONG_ANSWER": "While ultrasound examination is inexpensive and easily done, it is not accurate enough for staging small penile cancers located at the glans penis. However, for larger tumors ultrasound can be a useful addition to physical examination by delineating reliably the anatomic relations of the tumor to structures such as the tunica albuginea, corpus cavernosum, and urethra." }, "23677366": { "QUESTION": "Do oblique views add value in the diagnosis of spondylolysis in adolescents?", "CONTEXTS": [ "Anteroposterior, lateral, and right and left oblique lumbar spine radiographs are often a standard part of the evaluation of children who are clinically suspected of having spondylolysis. Recent concerns regarding radiation exposure and costs have brought the value of oblique radiographs into question. The purpose of the present study was to determine the diagnostic value of oblique views in the diagnosis of spondylolysis.", "Radiographs of fifty adolescents with L5 spondylolysis without spondylolisthesis and fifty controls were retrospectively reviewed. All controls were confirmed not to have spondylolysis on the basis of computed tomographic scanning, magnetic resonance imaging, or bone scanning. Anteroposterior, lateral, and right and left oblique radiographs of the lumbar spine were arranged into two sets of slides: one showing four views (anteroposterior, lateral, right oblique, and left oblique) and one showing two views (anteroposterior and lateral only). The slides were randomly presented to four pediatric spine surgeons for diagnosis, with four-view slides being presented first, followed by two-view slides. The slides for twenty random patients were later reanalyzed in order to calculate of intra-rater agreement. A power analysis demonstrated that this study was adequately powered. Inter-rater and intra-rater agreement were assessed on the basis of the percentage of overall agreement and intraclass correlation coefficients (ICCs). PCXMC software was used to generate effective radiation doses. Study charges were determined from radiology billing data.", "There was no significant difference in sensitivity and specificity between four-view and two-view radiographs in the diagnosis of spondylolysis. The sensitivity was 0.59 for two-view studies and 0.53 for four-view studies (p = 0.33). The specificity was 0.96 for two-view studies and 0.94 for four-view studies (p = 0.60). Inter-rater agreement, intra-rater agreement, and agreement with gold-standard ICC values were in the moderate range and also demonstrated no significant differences. Percent overall agreement was 78% for four-view studies and 82% for two-view studies. The radiation effective dose was 1.26 mSv for four-view studies and 0.72 mSv for two-view studies (difference, 0.54 mSv). The charge for four-view studies was $145 more than that for two-view studies." ], "LABELS": [ "BACKGROUND", "METHODS", "RESULTS" ], "MESHES": [ "Adolescent", "Case-Control Studies", "Cohort Studies", "Hospital Costs", "Humans", "Lumbar Vertebrae", "Observer Variation", "Philadelphia", "Radiation Dosage", "Radiography", "Retrospective Studies", "Sensitivity and Specificity", "Spondylolysis" ], "YEAR": "2013", "reasoning_required_pred": "no", "reasoning_free_pred": "no", "final_decision": "no", "LONG_ANSWER": "There is no difference in sensitivity and specificity between four-view and two-view studies. Although oblique views have long been considered standard practice by some, our data could not identify a diagnostic benefit that might outweigh the additional cost and radiation exposure." }, "19237087": { "QUESTION": "Are many colorectal cancers due to missed adenomas?", "CONTEXTS": [ "An unknown number of colorectal cancers could be due to missed adenomas during previous endoscopy. Data in the literature are sparse. A large cross-sectional study was done in a prospective database of all patients diagnosed with colorectal cancer.", "All consecutive endoscopies over a period of 15 years, in which colorectal cancer was diagnosed were included. All patients who underwent more than one endoscopy and in whom ultimately cancer was diagnosed were studied separately.", "Colorectal cancer was diagnosed in 835 patients. Twenty-five patients underwent a previous endoscopy without a cancer diagnosis. These 25 patients were divided into three groups according to the time between the endoscopy in which the cancer was detected and the previous endoscopy. Five out of these 25 patients underwent regular surveillance. Only 11 patients had no argument for regular follow-up. Assuming that these cancers developed from an adenoma than only 11 out of 835 (1.3%) cancers were missed in the adenoma phase. There was no difference in the size of the tumour between the three groups of patients." ], "LABELS": [ "BACKGROUND", "METHODS", "RESULTS" ], "MESHES": [ "Adenoma", "Adenomatous Polyps", "Aged", "Aged, 80 and over", "Cecal Neoplasms", "Colonic Diseases", "Colonoscopy", "Colorectal Neoplasms", "Cross-Sectional Studies", "Databases, Factual", "False Negative Reactions", "Female", "Humans", "Male", "Middle Aged" ], "YEAR": "2009", "reasoning_required_pred": "no", "reasoning_free_pred": "no", "final_decision": "no", "LONG_ANSWER": "In normal daily practice, only a small number of clinically important adenomas are missed. The problem of missed adenomas probably is being exaggerated." }, "10456814": { "QUESTION": "Does desflurane alter left ventricular function when used to control surgical stimulation during aortic surgery?", "CONTEXTS": [ "Although desflurane is commonly used to control surgically induced hypertension, its effects on left ventricular (LV) function have not been investigated in this clinical situation. The purpose of the present study was to evaluate the LV function response to desflurane, when used to control intraoperative hypertension.", "In 50 patients, scheduled for vascular surgery, anesthesia was induced with sufentanil 0.5 microg/kg, midazolam 0.3 mg/kg and atracurium 0.5 mg/kg. After tracheal intubation, anesthesia was maintained with increments of drugs with controlled ventilation (N2O/O2=60/40%) until the start of surgery. A 5 Mhz transesophageal echocardiography (TEE) probe was inserted after intubation. Pulmonary artery catheter and TEE measurements were obtained after induction (to)(control value), at surgical incision (t1) if it was associated with an increase in systolic arterial pressure (SAP) greater than 140 mmHg (hypertension) and after control of hemodynamic parameters by administration of desflurane (return of systolic arterial pressure to within 20% of the control value) (t2) in a fresh gas flow of 31/ min.", "Sixteen patients developed hypertension at surgical incision. SAP was controlled by desflurane in all 16 patients. Afterload assessed by systemic vascular resistance index (SVRI), end-systolic wall-stress (ESWS) and left-ventricular stroke work index (LVSWI) increased with incision until the hypertension returned to post-induction values with mean end-tidal concentration of 5.1+/-0.7% desflurane. No change in heart rate, cardiac index, mean pulmonary arterial pressure, stroke volume, end-diastolic and end-systolic cross-sectional areas, fractional area change and left ventricular circumferential fiber shortening was noted when desflurane was added to restore blood pressure." ], "LABELS": [ "BACKGROUND", "METHODS", "RESULTS" ], "MESHES": [ "Anesthetics, Inhalation", "Anesthetics, Intravenous", "Aorta", "Atracurium", "Blood Pressure", "Cardiac Output", "Catheterization, Swan-Ganz", "Diastole", "Echocardiography, Transesophageal", "Female", "Heart Rate", "Heart Ventricles", "Humans", "Hypertension", "Intraoperative Complications", "Intubation, Intratracheal", "Isoflurane", "Male", "Midazolam", "Middle Aged", "Neuromuscular Nondepolarizing Agents", "Nitrous Oxide", "Oxygen", "Stroke Volume", "Sufentanil", "Systole", "Vascular Resistance", "Ventricular Function, Left" ], "YEAR": "1999", "reasoning_required_pred": "no", "reasoning_free_pred": "yes", "final_decision": "no", "LONG_ANSWER": "This study demonstrates that in patients at risk for cardiac morbidity undergoing vascular surgery, desflurane is effective to control intraoperative hypertension without fear of major cardiac depressant effect." }, "8921484": { "QUESTION": "Does gestational age misclassification explain the difference in birthweights for Australian aborigines and whites?", "CONTEXTS": [ "After 34 weeks gestation, summary measures of location for birthweight (e.g means and centiles) increase more slowly for Australian Aborigines than for whites. A similar pattern has been observed for blacks in the US. This study tests whether the reported pattern is due to differential misclassification of gestational age.", "Simulation was used to measure the potential effect of differential misclassification of gestational age. Reported gestational age data were obtained from Queensland Perinatal Data Collection (QPDC). Estimates of the true distributions of gestational age were obtained by assuming various (plausible) types of misclassification and applying these to the reported distributions. Previous studies and data from the QPDC were used to help specify the birthweight distributions used in the simulations.", "At full term, the parameters of the birthweight distributions were robust to gestational age misclassification. At preterm, the 10th centiles were robust to misclassification. In contrast, the 90th centiles were sensitive to even minor misclassification. Extreme types of misclassification were required to remove the divergence in median birthweights for Aborigines and whites." ], "LABELS": [ "BACKGROUND", "METHODS", "RESULTS" ], "MESHES": [ "Adult", "Australia", "Birth Weight", "Classification", "Computer Simulation", "European Continental Ancestry Group", "Female", "Gestational Age", "Humans", "Male", "Oceanic Ancestry Group", "Pregnancy" ], "YEAR": "1996", "reasoning_required_pred": "yes", "reasoning_free_pred": "no", "final_decision": "no", "LONG_ANSWER": "Gestational age misclassification is an unlikely explanation for the reported divergence in average birth-weights for Aborigines and whites. The results might help with the interpretation of other between-population comparisons." }, "16564683": { "QUESTION": "Is there any interest to perform ultrasonography in boys with undescended testis?", "CONTEXTS": [ "To evaluate the accuracy of ultrasonographic examination in boys with an undescended testis.", "All patients who were referred to the paediatric surgeon after detection of an undescended testis were evaluated prospectively between November 2001 and November 2004. Among these 377 patients, 87 were referred with an ultrasonogram previously prescribed by the referring primary physician. The results of the ultrasonogram were compared to the results of the clinical examination of the paediatric surgeon and, in cases of no palpable testis, to the surgical findings.", "Ultrasonography did not detect the retractile testes. Ultrasonography detected 67% of the palpable undescended testes. In cases of no palpable testis, the ultrasonographic examination missed the abdominal testes and sometimes other structures were falsely interpreted as a testis." ], "LABELS": [ "OBJECTIVE", "MATERIAL AND METHODS", "RESULTS" ], "MESHES": [ "Child", "Child, Preschool", "Cryptorchidism", "Humans", "Infant", "Male", "Prospective Studies", "Reproducibility of Results", "Ultrasonography" ], "YEAR": "2006", "reasoning_required_pred": "no", "reasoning_free_pred": "no", "final_decision": "no", "LONG_ANSWER": "Sonography has no place in the diagnosis of undescended testis." }, "23147106": { "QUESTION": "Is peak concentration needed in therapeutic drug monitoring of vancomycin?", "CONTEXTS": [ "We analyzed the pharmacokinetic-pharmacodynamic relationship of vancomycin to determine the drug exposure parameters that correlate with the efficacy and nephrotoxicity of vancomycin in patients with methicillin-resistant Staphylococcus aureus pneumonia and evaluated the need to use peak concentration in therapeutic drug monitoring (TDM).", "Serum drug concentrations of 31 hospitalized patients treated with vancomycin for methicillin-resistant S. aureus pneumonia were collected.", "Significant differences in trough concentration (Cmin)/minimum inhibitory concentration (MIC) and area under the serum concentration-time curve (AUC0-24)/MIC were observed between the response and non-response groups. Significant differences in Cmin and AUC0-24 were observed between the nephrotoxicity and non-nephrotoxicity groups. Receiver operating characteristic curves revealed high predictive values of Cmin/MIC and AUC0-24/MIC for efficacy and of Cmin and AUC0-24 for safety of vancomycin." ], "LABELS": [ "BACKGROUND", "METHODS", "RESULTS" ], "MESHES": [ "Aged", "Aged, 80 and over", "Anti-Bacterial Agents", "Area Under Curve", "Female", "Half-Life", "Humans", "Male", "Methicillin-Resistant Staphylococcus aureus", "Microbial Sensitivity Tests", "Middle Aged", "Pneumonia", "ROC Curve", "Retrospective Studies", "Vancomycin" ], "YEAR": "2012", "reasoning_required_pred": "yes", "reasoning_free_pred": "no", "final_decision": "no", "LONG_ANSWER": "These results suggest little need to use peak concentration in vancomycin TDM because Cmin/MIC and Cmin are sufficient to predict the efficacy and safety of vancomycin." }, "21550158": { "QUESTION": "Can autologous platelet-rich plasma gel enhance healing after surgical extraction of mandibular third molars?", "CONTEXTS": [ "This investigation assesses the effect of platelet-rich plasma (PRP) gel on postoperative pain, swelling, and trismus as well as healing and bone regeneration potential on mandibular third molar extraction sockets.", "A prospective randomized comparative clinical study was undertaken over a 2-year period. Patients requiring surgical extraction of a single impacted third molar and who fell within the inclusion criteria and indicated willingness to return for recall visits were recruited. The predictor variable was application of PRP gel to the socket of the third molar in the test group, whereas the control group had no PRP. The outcome variables were pain, swelling, and maximum mouth opening, which were measured using a 10-point visual analog scale, tape, and millimeter caliper, respectively. Socket healing was assessed radiographically by allocating scores for lamina dura, overall density, and trabecular pattern. Quantitative data were presented as mean. Mann-Whitney test was used to compare means between groups for continuous variables, whereas Fischer exact test was used for categorical variables. Statistical significance was inferred at P<.05.", "Sixty patients aged 19 to 35 years (mean: 24.7 \u00b1 3.6 years) were divided into both test and control groups of 30 patients each. The mean postoperative pain score (visual analog scale) was lower for the PRP group at all time points and this was statistically significant (P<.05). Although the figures for swelling and interincisal mouth opening were lower in the test group, this difference was not statistically significant. Similarly, the scores for lamina dura, trabecular pattern, and bone density were better among patients in the PRP group. This difference was also not statistically significant." ], "LABELS": [ "PURPOSE", "PATIENTS AND METHODS", "RESULTS" ], "MESHES": [ "Adult", "Bone Regeneration", "Chi-Square Distribution", "Female", "Gels", "Humans", "Male", "Mandible", "Molar, Third", "Pain Measurement", "Pain, Postoperative", "Platelet-Rich Plasma", "Prospective Studies", "Radiography", "Range of Motion, Articular", "Single-Blind Method", "Statistics, Nonparametric", "Tooth Extraction", "Tooth Socket", "Tooth, Impacted", "Wound Healing", "Young Adult" ], "YEAR": "2011", "reasoning_required_pred": "no", "reasoning_free_pred": "yes", "final_decision": "yes", "LONG_ANSWER": "The PRP group recorded reduced pain, swelling, and trismus as well as enhanced and faster bone healing compared with those in the control. Hence the study showed that topical application of PRP gel has a beneficial effect in enhancing socket healing after third molar surgery." }, "17559449": { "QUESTION": "Are sugars-free medicines more erosive than sugars-containing medicines?", "CONTEXTS": [ "The reduced use of sugars-containing (SC) liquid medicines has increased the use of other dose forms, potentially resulting in more widespread dental effects, including tooth wear. The aim of this study was to assess the erosive potential of 97 paediatric medicines in vitro.", "The study took the form of in vitro measurement of endogenous pH and titratable acidity (mmol). Endogenous pH was measured using a pH meter, followed by titration to pH 7.0 with 0.1-M NaOH.", "Overall, 55 (57%) formulations had an endogenous pH of<5.5. The mean (+/- SD) endogenous pH and titratable acidity for 41 SC formulations were 5.26 +/- 1.30 and 0.139 +/- 0.133 mmol, respectively; for 56 sugars-free (SF) formulations, these figures were 5.73 +/- 1.53 and 0.413 +/- 1.50 mmol (P>0.05). Compared with their SC bioequivalents, eight SF medicines showed no significant differences for pH or titratable acidity, while 15 higher-strength medicines showed lower pH (P = 0.035) and greater titratable acidity (P = 0.016) than their lower-strength equivalents. Chewable and dispersible tablets (P<0.001), gastrointestinal medicines (P = 0.002) and antibiotics (P = 0.007) were significant predictors of higher pH. In contrast, effervescent tablets (P<0.001), and nutrition and blood preparations (P = 0.021) were significant predictors of higher titratable acidity." ], "LABELS": [ "OBJECTIVE", "METHODS", "RESULTS" ], "MESHES": [ "Child", "Child, Preschool", "Drug-Related Side Effects and Adverse Reactions", "Humans", "Hydrogen-Ion Concentration", "Metabolic Clearance Rate", "Mouth", "Pharmaceutical Preparations", "Pharmaceutical Vehicles", "Sucrose", "Sweetening Agents", "Titrimetry", "Tooth Erosion" ], "YEAR": "2007", "reasoning_required_pred": "no", "reasoning_free_pred": "no", "final_decision": "no", "LONG_ANSWER": "Paediatric SF medicines were not more erosive than SC medicines in vitro; a more significant predictor of their erosive potential was dose form." } }